Вы находитесь на странице: 1из 385

Anatomic Science

1. At which of the following ages does fetal movement first occur?


A.
B.
C.
D.
E.

1 month
2 months
4 months
6 months
7 months

The correct answer is B. Neuromuscular development is sufficient to allow fetal movement in the
eighth week of life. Other features of Week 8 include the first appearance of a thin skin, a head as
large as the rest of the body, forward-looking eyes, appearance of digits on the hands and feet,
appearance of testes and ovaries (but not distinguishable external genitalia), and a crown-rump length
of approximately 30 mm. By the end of the eighth week, nearly all adult structures have at least begun
to develop, and the fetus "looks like a baby."
2. Most of the oocytes in the ovary of a prepubescent girl are in which meiotic stage?
A.
B.
C.
D.
E.

Anaphase of the second meiotic division


Metaphase of the first meiotic division
Metaphase of the second meiotic division
Prophase of the first meiotic division
Telophase of the first meiotic division

The correct answer is D. The first meiotic division is the "reduction" meiotic division, in which the
diploid complement of DNA is reduced to a haploid complement. The bulk of oocytes in
premenopausal women, girls, and babies are arrested at prophase of the first meiotic division.
Postmenopausal women have very few viable oocytes. It is important to note that ovulation occurs
before the oocyte is completely mature. The secondary oocyte leaving the follicle is in metaphase of
the second meiotic division (choice C). The cell's metabolic operations have been discontinued and
the oocyte drifts in a state of "suspended animation," awaiting the necessary stimulus for further
development. If fertilization does not occur, the oocyte disintegrates without completing meiosis.
3. An abrasion results in the total loss of epidermis over a large area of an arm, but one month later,
the abrasion has healed, with regrowth of the epidermis. Which of the following mechanisms accounts
for the restoration of the epidermis over the abraded area?
A.
B.
C.
D.
E.

Growth of epidermis from hair follicles and sweat glands in the dermis
Migration of endothelial cells from newly grown capillaries
Transformation of dermal fibroblasts into epidermal cells
Transformation of macrophages into epidermal cells
Transformation of melanocytes into epidermal cells

The correct answer is A. The dermis contains skin appendages (e.g., hair follicles), which contain
epithelial stem cells. In the process of healing a large area where the epidermis has been lost but the
dermis is intact, re-epithelialization occurs by growth of epidermal cells from the underlying skin
appendages, as well as from the intact epidermis along the wound edges. Physiologically, the dermis
lies beneath the epidermis. It has two major components, a superficial papillary layer and a deeper
reticular layer. The papillary layer contains the capillaries and the sensory neurons, which supply the
surface of the skin. The reticular layer consists of an interwoven meshwork of dense irregular
connective tissue.
None of the other cell types are known to directly contribute to the regeneration of epidermis over
abraded skin.

4. Hirschsprung's disease indicates a developmental abnormality in which of the following embryonic


tissues?
A.
B.
C.
D.
E.

Ectoderm
Endoderm
Neural crest
Neural ectoderm
Splanchnic mesoderm

The correct answer is C. The baby has Hirschsprung's disease, which is due to an absence of
ganglion cells in the wall of the colon. Neural crest cells contribute to the formation of many adult
structures. Among these are all of the postganglionic neurons of the autonomic nervous system and
the sensory neurons of the peripheral nervous system.
Ectoderm (choice A) forms the epidermis of the skin and the parenchymal cells of glands associated
with the skin such as the sweat glands, sebaceous glands, and mammary glands.
Endoderm (choice B) forms the epithelial lining of the gut tube and the parenchymal cells of glands
associated with the gut tube, such as the liver and pancreas.
Neural ectoderm (choice D) forms the central nervous system, the somatic motor neurons of the
peripheral nervous system, and the preganglionic neurons of the autonomic nervous system.
Splanchnic mesoderm (choice E) forms the visceral peritoneum, visceral pleura, visceral pericardium,
and the stroma and muscle of the wall of the gut, among other structures.
5. Injury at the lower border of a rib will most likely damage which structure?
A.
B.
C.
D.

intercostal artery
intercostal nerve
intercostal vein
internal intercostal muscle

The correct answer is B. The three structures in the intercostal space are, from superior to inferior,
the intercostal vein, artery, and nerve. The proper site for insertion of an intercostal drain is superior to
a rib, not directly at the level of the superior border but slightly higher to avoid the collateral branches
of the nerve, artery, and vein. The nerve is the most inferior structure and thus most likely to be
damaged by the drain (producing an anesthetic dermatome). In addition, the nerve is the least
protected by the costal groove.
The intercostal artery (choice A) is the middle structure and thus is not the most likely to be
damaged.
The intercostal vein (choice C) is the most superior structure and is thus the least likely to be
damaged. In addition, the vein is the structure best protected by the costal groove (the further superior
the structure, the more protected by the costal groove).
The internal intercostal fibers (choice D) will tend to be separated by the drain. In addition, the
internal intercostal muscle is membranous (internal intercostal membrane) posteriorly, from the neck
of the rib to its angle; therefore, many drains, when inserted, will not even pass through a layer of
internal intercostal muscle.
6. Following the healing of a tibial fracture, a patient forms a small, new piece of viable bone near the
joint (heterotopic ossification), which is interfering with joint mobility. Which of the following is the
source of the osteoblasts that formed this bone?
A. Cancellous bone near the fracture

B.
C.
D.
E.

Circulating stem cells


Compact bone near the fracture
Marrow
Periosteum

The correct answer is E. The source of bone-forming cells following a fracture is the damaged
periosteum. Occasionally, some of these cells become "lost" and can form small pieces of
inappropriately located bone that impair joint movement. Except in joint cavities, where they are
covered by a layer of hyaline cartilage, bone surfaces are covered by the periosteum. The periosteum
is composed of an outer fibrous and inner cellular layer. The periosteum assists in the attachment of a
bone to surrounding tissues and associated tendons and ligaments.
Surprisingly, neither the cancellous (choice A) nor the compact (choice C) bone near the fracture is
the source of the bone-forming cells.
Circulating stem cells (choice B) can repopulate damaged bone marrow (choice D), but neither the
bone marrow nor the circulating stem cells is the source of the bone-forming cells.
7. A patient has a severe headache, and is unable to move his right leg. There is no higher cortical
function loss, but deep deep tendon reflexes and Babinski's sign are absent. The medial aspect of the
cerebral hemispheres is most affected. Which artery is most likely involved?
A.
B.
C.
D.
E.

left anterior cerebral artery


left middle cerebral artery
left posterior cerebral artery
right anterior cerebral artery
right posterior cerebral artery

The correct answer is A. The medial aspect of the cerebral hemispheres is supplied by the anterior
cerebral arteries. The small portion of primary motor cortex located here sends fibers to spinal-cord
segments innervating the lower limbs. Because the corticospinal tracts cross over to the opposite side
in the medulla oblongata, the left cerebral hemisphere controls the right side of the body. Hence,
paralysis of the right leg results from loss of blood flow to the portion of the left hemisphere supplied
by the left anterior cerebral artery.
The middle cerebral artery (choice B) supplies the lateral convexity of the brain. Primary motor
cortices on the lateral aspects of the hemispheres send fibers to the brainstem (innervating the face)
and to spinal-cord segments innervating the upper limbs, the trunk, and the proximal part of the lower
extremities. As with the anterior cerebral artery territories, the right hemisphere innervates the left
body, and the left hemisphere innervates the right body.
The posterior cerebral artery (choices C and E) supplies the occipital cortex; interruption of blood
flow through this artery would probably produce visual deficits rather than paralysis.
The right anterior cerebral artery (choice D) supplies blood to the medial aspect of the right cerebral
hemisphere, which controls the left leg. This patient has paralysis of the right leg.
8. An infant is born with an abnormally developed falciform ligament. The hepatogastric and
hepatoduodenal ligaments are also malformed. These developmental anomalies are most likely due
to abnormal development of the
A.
B.
C.
D.
E.

dorsal mesoduodenum
dorsal mesogastrium
pericardioperitoneal canal
pleuropericardial membranes
ventral mesentery

The correct answer is E. The ventral mesentery forms the falciform ligament, ligamentum teres, and
lesser omentum, which can be divided into the hepatogastric and hepatoduodenal ligament.
The dorsal mesoduodenum (choice A) is the mesentery of the developing duodenum, which later
disappears so that the duodenum and pancreas lie retroperitoneally.
Both omental bursa and the greater omentum are derived from the dorsal mesogastrium (choice B),
which is the mesentery of the stomach region.The pericardioperitoneal canal (choice C)
embryologically connects the thoracic and peritoneal canals.The pleuropericardial membranes
(choice D) become the pericardium and contribute to the diaphragm.
9. Which of the following structures is lined with epithelium derived from mesoderm of the ureteric
bud?
A.
B.
C.
D.
E.

Bowman's capsule
Distal convoluted tubule
Loop of Henle
Proximal convoluted tubule
Ureter

The correct answer is E. The transitional epithelium that lines the ureter, the renal pelvis, and the
major and minor calyces is derived from mesoderm of the ureteric bud, as is the cuboidal epithelium
of the collecting tubules.
The simple squamous epithelium lining Bowman's capsule (choice A) is derived from mesoderm of
the metanephric vesicle.
The simple cuboidal epithelium lining the distal convoluted tubule
(choice B) is derived from mesoderm of the metanephric vesicle.
The simple squamous
epithelium lining the loop of Henle (choice C) is derived from mesoderm of the metanephric vesicle.
The proximal convoluted tubule (choice D) is lined with simple columnar epithelium derived from
mesoderm of the metanephric vesicle.
10. A patient's left hypoglossal nerve (CN XII) is injured during a carotid endarterectomy. Which of
the following would most likely result from this injury?
A. Decreased gag reflex on the left
B. Decreased salivation from the left submandibular and sublingual salivary glands
C. Deviation of the tongue to the left on protrusion
D. Inability to elevate the pharynx on the left during swallowing
E. Inability to perceive sweet and salt taste sensation on the anterior part of the left side of the
tongue
The correct answer is C. The hypoglossal nerve is a pure motor nerve (general somatic efferent) to
the intrinsic and most extrinsic muscles of the tongue. If the nerve is damaged, denervation atrophy of
the affected side will permit the intact musculature of the opposite side to operate unopposed, thereby
protruding the tongue to the side of the injury.
The gag reflex (choice A) is mediated by the glossopharyngeal nerve (CN IX; afferent limb) and the
vagus nerve (CN X; efferent limb).
Choice B is incorrect because the preganglionic parasympathetic fibers that regulate these two
salivary glands are carried by the chorda tympani (which joins with the lingual nerve) to the
submandibular ganglion. Postganglionic fibers are then distributed to these glands.
The muscles responsible for elevation of the pharynx (choice D) are innervated primarily by the
vagus nerve (CN X).

Choice E is incorrect because the taste fibers for the anterior two thirds of the tongue are carried via
the chorda tympani to the facial nerve (CN VII) and hence to the brainstem.
11. A patient with paresthesia of the medial side of the hand and forearm undergoes surgery to
correct it. If the anterior scalene muscle is cut during this surgery, what nearby nerve must be
avoided?
A.
B.
C.
D.
E.

Phrenic nerve
Recurrent laryngeal nerve
Superior laryngeal nerve
Suprascapular nerve
Vagus nerve

The correct answer is A. The patient is suffering from scalene triangle syndrome (thoracic outlet
syndrome). The lower trunk of the brachial plexus and the subclavian artery are being compressed
between the anterior scalene muscle and the middle scalene muscle. Incision of the anterior scalene
muscle will relieve this compression. The phrenic nerve lies on the anterior surface of the anterior
scalene muscle deep to the prevertebral fascia. If this nerve is cut, the diaphragm on that side of the
body will be paralyzed. Anatomically, the phrenic nerve is one of the nerves of the cervical plexus.
This nerve distributes to the diaphragm other nerves of the cervical plexusm including the ansa
cervicalis, lesser occiptital, transverse cervical, supraclavical, and greater auricular nerves, as well as
the cervical nerves.
The recurrent laryngeal nerve (choice B) is a branch of the vagus nerve. On the left side it recurs
around the aortic arch, and on the right side it recurs around the right subclavian artery. It ascends to
the larynx in the tracheoesophageal groove. The nerve innervates several laryngeal muscles and the
laryngeal mucosa inferior to the vocal folds. It is not in contact with the anterior scalene muscle.
The superior laryngeal nerve (choice C) is a branch of the vagus nerve that arises just after the vagus
nerve passes through the jugular foramen. It innervates the cricothyroid muscle of the larynx and the
laryngeal mucosa above the vocal folds. The superior laryngeal nerve is not in contact with the
anterior scalene muscle.
The suprascapular nerve (choice D) is a branch of the upper trunk of the brachial plexus. It arises
from the upper trunk after the upper trunk has passed between the anterior and middle scalene
muscles. The suprascapular nerve innervates the supraspinatus and infraspinatus muscles.
The vagus nerve (choice E) lies within the carotid sheath within the neck. The carotid sheath is
anterior to the prevertebral fascia. The vagus nerve is not in contact with the anterior scalene muscle.
12. Compression of a cranial nerve by a large aneurysm of the right superior cerebellar artery
immediately distal to its origin from the basilar artery would cause which of the following clinical
findings?
A.
B.
C.
D.
E.

Loss of abduction of the right eye


Loss of adduction of the right eye
Loss of depression of the right eye from the adducted position
Loss of sensation on the right side of the face
Loss of visual field of the right eye

The correct answer is B. The oculomotor nerve emerges from the interpeduncular fossa of the
midbrain and then passes between the superior cerebellar artery and the posterior cerebral artery
immediately lateral to the basilar artery. Aneurysm of any of these three arteries may compress the
nerve. The oculomotor nerve innervates a number of extraocular muscles in the orbit, including the
medial rectus muscle, which is responsible for adduction of the eye.

Abduction of the eye (choice A) is accomplished by the lateral rectus muscle. This muscle is
innervated by the abducens nerve, which arises more caudally from the brainstem and is not in
contact with this artery.
Depression of the eye from the adducted position (choice C) is accomplished by the superior oblique
muscle. This muscle is innervated by the trochlear nerve, which emerges from the dorsal surface of
the midbrain and passes lateral to the cerebral peduncle before entering the cavernous sinus. A
superior cerebellar artery aneurysm would not compress this nerve.
Sensation on the face (choice D) is mediated by the trigeminal nerve. This nerve arises from the
anterolateral surface of the pons and is not in contact with this artery.
Loss of visual field (choice E) results from lesions of the optic nerve or other elements of the visual
pathway. These are not in contact with this artery.

13.The two most important hormones controlling Calcium metabolism are secreted from the:
A.
B.
C.
D.
E.

Parathyroid and thyroid glands


Thyroid and thymus glands
Adenohypophysis and parathyroid glands
Adrenal cortex and thyroid glands
Parathyroid and adrenal glands

The correct answer is A.PTH, or parathyroid hormone, is considered to be the most important
hormone controlling calcium metabolism. It increases serum calcium by increasing bone mineral
resorption by osteoclasts, increasing digestive absorption of calcium, and decreasing calcium
excretion by the kidney. PTH is secreted by principal cells of the parathyroid. The antagonist
hormone, calcitonin, is secreted by parafollicular cells of the thyroid (follicular cells secrete thyroxin). It
reduces serum calcium by promoting calcium depositon in bones, decreasing digestive absorption of
calcium, and promoting calcium excretion. Note that the term adenohypophysis in answer choice C
refers to the anterior pituitary. This term and neurohypophysis (posterior pituitary) are often used on
NBDE.
14. During a cranial nerve test, the patient cannot elevate her right eye from the abducted position.
Which of the following muscles is paralyzed?
A.
B.
C.
D.
E.

Right inferior oblique


Right inferior rectus
Right lateral rectus
Right superior oblique
Right superior rectus

The correct answer is E. The superior rectus muscle can elevate and adduct the eye from the
neutral position. From the abducted position, it is the only muscle that can elevate the eye.
The inferior oblique muscle (choice A) can elevate and abduct the eye from the neutral position.
From the adducted position, it is the only muscle that can elevate the eye.
The inferior rectus muscle (choice B) can depress and adduct the eye from the neutral position. From
the abducted position, it is the only muscle that can depress the eye.
The lateral rectus muscle (choice C) can abduct the eye.
The superior oblique muscle (choice D) can depress and abduct the eye from the neutral position.
From the adducted position, it is the only muscle that can depress the eye.

15. Several arterial branches penetrate into the distal end of the lesser curvature of the stomach.
Which of the following arteries usually supplies these branches?
A.
B.
C.
D.
E.

Left gastric
Left gastroepiploic
Right gastric
Right gastroepiploic
Short gastric

The correct answer is C. The arterial supply of the stomach is complex; it therefore has a good
potential to appear on the NBDE. The right gastric artery supplies the distal lesser curvature.
The left gastric artery (choice A) supplies the proximal lesser curvature.
The left gastroepiploic
artery (choice B) supplies the proximal greater curvature below the splenic artery.
The right
gastroepiploic artery (choice D) supplies the distal greater curvature.
The short gastric artery
(choice E) supplies the proximal greater curvature above the splenic artery.
16. Contraction of which of the following muscles contributes most to the backward movement of the
lower jaw during the process of mastication?
A.
B.
C.
D.
E.

Digastric
Lateral pterygoid
Medial pterygoid
Mylohyoid
Temporalis

The correct answer is E. Mastication is a complex process involving alternating elevation,


depression, forward movement, and backward movement of the lower jaw. The backward-movement
step is accomplished by the posterior fibers of the temporalis muscle. The temporalis originates along
the temporal lines of the temporal bone and inserts into the coronoid process of the mandible. It is
innervated by the manibular branch of the trigeminal nerve.
The digastric (choice A) helps to
depress (open) the lower jaw during chewing.
The lateral pterygoid (choice B) helps to move
the lower jaw forward, laterally, or downward during chewing.
The medial pterygoid (choice C)
helps to elevate (close) the lower jaw during chewing.
The mylohyoid (choice D) helps to
depress (open) the lower jaw during chewing.
17. Most fascia of the body that attach to bones attach by which of the following mechanisms?
A.
B.
C.
D.
E.

Blending with the covering periosteum


Inserting deeply into the cancellous bone
Inserting deeply into the cartilage
Inserting deeply into the diaphysis
Inserting deeply into the marrow

The correct answer is A. Fascial straps (retinacula) and fascial coverings of muscles or muscle
groups characteristically attach to nearby bones by blending with the covering periosteum. No deep
attachments are usually made by fascia.
Cancellous bone (choice B) is spongy bone, which is usually found in marrow and is not the site for
fascial attachment.
Fascia do not usually attach to cartilage (choice C).
Fascia attaches to bony shafts, or diaphyses (choice D), superficially via the periosteum.
Fascia do not penetrate the bone to reach the marrow (choice E).

18. A newborn baby is noted to have a left unilateral cleft lip. There are no abnormalities of the baby's
palate. Which of the following developmental defects accounts for this occurrence?
A.
B.
C.
D.
E.

Failure of the left lateral palatine process to fuse with the median palatine process
Failure of the left maxillary prominence to unite with the left medial nasal prominence
Failure of the primary palate to fuse with the secondary palate
Failure of the right and left medial nasal prominences to merge
Failure of the right palatine process to fuse with the left palatine process

The correct answer is B. In the formation of the upper lip, the right and left medial nasal
prominences merge to form the philtrum of the upper lip. The lateral maxillary prominence then
merges with the merged medial nasal prominences. Failure of this merger to occur results in a
unilateral cleft lip.
Failure of a lateral palatine process to fuse with the median palatine process (choice A), which is
synonymous with a failure of the primary palate to fuse with the secondary palate (choice C), results
in a primary cleft palate. Primary clefts of the palate are found anterior to the incisive foramen of the
palate. Primary cleft palates may occur along with cleft lips but are the result of a separate
developmental defect.
Failure of the right and left medial nasal prominences to merge (choice D) results in a median cleft
lip. This is a rare anomaly. Normally, the right and left medial nasal prominences merge into a single
prominence that forms the philtrum of the lip.
Failure of the right and left palatine processes to fuse (choice E) results in a secondary cleft palate.
Secondary cleft palates are found posterior to the incisive foramen. Normally, the right and left
palatine processes fuse together and fuse to the primary palate.
19. A patient experiences visual difficulties. When a light is shined in her right eye, there is no
pupillary response in either eye. However, upon shining a light in her left eye, both ipsilateral and
contralateral pupillary responses are apparent. Her extraocular movements are intact. What is the
most likely location of her lesion?
A.
B.
C.
D.
E.

Oculomotor nerve, left side


Oculomotor nerve, right side
Optic nerve, left side
Optic nerve, right side
Trochlear nerve, left side

The correct answer is D. Know your cranial nerves! This woman has a "Marcus-Gunn pupil" with a
defect in the afferent pathway of the optic nerve (in this case, on the right side). Recall that the
afferent limb of the pupillary light reflex is the optic nerve (CN II); the efferent limb is the oculomotor
nerve (CN III; parasympathetic fibers). When light is shined into her right eye, because her right optic
nerve is not functioning properly, the light signal is not transmitted to the central nervous system
(CNS), resulting in no pupillary response. As light is shined into her left eye, the left optic nerve
transmits the signal to the CNS, which then sends an outbound signal through both the right and left
oculomotor nerves to cause pupillary constriction in both eyes. Anatomically, the optic nerve is a
special sensory nerve that originates in the retina of the eye, passes through the optic foramen of the
sphenoid bone, and has a destination in the diencephalon via the optic chiasm.
The oculomotor nerve (choices A and B) innervates all extraocular muscles except the lateral rectus
(innervated by the abducens nerve) and the superior oblique (innervated by the trochlear nerve
choice E). The oculomotor nerve also mediates pupillary constriction (parasympathetic fibers), eyelid
opening (levator palpebrae), and innervates the ciliary muscle (allowing accommodation).
A patient with a lesion of the left optic nerve (choice C) would have no pupillary responses in either
eye when shining a light in the left eye; pupillary responses would be present in both eyes when
shining a light in the right eye.

20. If the tongue deviates to the right side when protruded, the most likely cause is paralysis of which
of the following muscles?
A.
B.
C.
D.
E.

Left genioglossus
Left hyoglossus
Left palatoglossus
Right genioglossus
Right hyoglossus

The correct answer is D. The genioglossus muscle is innervated by the hypoglossal nerve. The
function of the genioglossus muscle is to pull the tongue forward (protrude) and toward the opposite
side. When the right genioglossus muscle is paralyzed, the left genioglossus muscle pulls the tongue
forward and to the right.
If the left genioglossus muscle were paralyzed (choice A), the tongue would deviate toward the left
on protrusion because of the unopposed action of the right genioglossus muscle. The left
genioglossus muscle is innervated by the left hypoglossal nerve.
The hyoglossus muscles (choices B and E) are innervated by the hypoglossal nerves. The function
of these muscles is to retract the tongue. These muscles are not active during protrusion of the
tongue.
The palatoglossus muscle (choice C) are innervated by the vagus nerves rather than the hypoglossal
nerves. Their function is to pull the tongue back (retract) and upward toward the palate.
21. Which of the following is a derivative of the second pharyngeal arch?
A.
B.
C.
D.
E.

Eustachian tube
External auditory meatus
Palatine tonsil
Stylohyoid muscle
Tensor tympani

The correct answer is D. The stylohyoid is derived from the second pharyngeal arch, which also
gives rise to the muscles of facial expression, the stapedius, the posterior belly of the digastric
muscle, Reichert's cartilage, and the facial nerve.
The Eustachian tubes (choice A) are derived from the first pharyngeal pouch, which also gives rise to
the middle ear cavity and the inner epithelial lining of the tympanic membrane.
The external auditory meatus (choice B is derived from the first pharyngeal cleft, which also gives
rise to the outer epithelial lining of the tympanic membrane.
The palatine tonsil (choice C) is derived from the epithelial lining of the second pharyngeal pouch.
The tensor tympani (choice E) is derived from the first pharyngeal arch, which also gives rise to the
muscles of mastication, the anterior belly of the digastric muscle, the mylohyoideus, the tensor veli
palantini, the maxillary and mandibular bones, and the maxillary and mandibular divisions of the
trigeminal nerve.
Note that knowing the embryology of these structures helps you remember the innervation of the
muscles of the face.
First pharyngeal arch muscles of mastication trigeminal nerve
Second pharyngeal arch muscles of facial expression facial nerve

22. Damage to the parasagittal region and falx cerebri will most likely result in which of the following
neurologic deficits?
A.
B.
C.
D.
E.

Altered taste
Leg paralysis
Loss of facial sensation
Ptosis
Unilateral deafness

The correct answer is B. A meningioma of the parasagittal region and the falx cerebri would be
located at the top of the brain, near the midline. In this position, it could compress the sensory or
motor cortex supplying the lower extremities. The falx cerebri is a fold of dura mater that projects
between the cerebral hemispheres in the longitudinal tissues. Its interior portions attach anteriorly to
the crista galli and posteriorly to the internal occipital crest.
Taste (choice A) is supplied by cranial nerves VII, IX, and X. These nerves arise from the brainstem.
Facial sensation (choice C) is supplied by cranial nerve V, the nuclei of which are in the brainstem.
Furthermore, the area of the sensory cortex that subserves the face is on the lateral aspect of the
cortex and would not be affected by a tumor in the parasagittal region.
Ptosis (choice D) can be caused by a deficit in cranial nerve III, which arises from the brainstem.
Unilateral deafness (choice E) suggests damage to cranial nerve VIII, which arises from the
brainstem.
23. In which of the following structures are the opacifications located with cataract formation?
A.
B.
C.
D.
E.

Aqueous humor
Cornea
Lens
Optic nerve
Retina

The correct answer is C. Cataracts are lens opacifications. It is not known whether senile cataracts
represent disease or normal opacification with age. Cataracts may occur as a consequence of
diabetes mellitus, long-term steroids, or congenital infections. They are successfully treated at present
with lens extraction and implantation of prosthetic lenses. The diagnostic characteristics for cataracts
are as follows: 1) blurred vision that is progressive over months to years, 2) no pain or redness is
seen, and 3) lens opacities may be invisible or grossly visible.
Aqueous humor (choice A) is continually replaced due to active secretion by the ciliary body. As
such, it does not undergo opacification; it is in constant flux.
Corneal opacification (choice B) is generally a consequence of squamous metaplasia, in which the
transparent, nonkeratinized, squamous cells are replaced by opaque, keratinized, squamous cells.
Squamous metaplasia is a reparative process, usually due to friction injury to the cornea or a vitamin
A deficiency.
The optic nerve (choice D) is not transparent, and it does not undergo opacification injury. It may,
however, atrophy due to ischemic, traumatic, infective, or metabolic insults.
The retina (choice E) consists of multiple layers of neural cells. The retina is transparent but is not the
site of cataract formation.

10

24. A sharp instrument passing through the superior orbital fissure would most likely sever which of
the following structures?
A.
B.
C.
D.
E.

Abducens nerve
Facial nerve
Mandibular nerve
Maxillary nerve
Middle meningeal artery

The correct answer is A. A good way to remember what passes through the superior oribital fissure
is that everything that innervates the eye, other than the optic nerve, passes through this fissure. This
incudes the oculomotor nerve(CN III), the trochlear nerve (CN IV), the ophthalmic nerve (V1), and the
abducens nerve (CN VI).
The facial nerve (CN VII; choice B) passes through the internal auditory meatus.
The mandibular nerve (V3; choice C) passes through the foramen ovale.
The maxillary nerve (V2; choice D) passes through the foramen rotundum.
The middle meningeal artery (choice E) passes through the foramen spinosum.

25. A knife wound to the neck damages the posterior cord of the brachial plexus. Which of the
following muscles would be most likely being paralyzed?
A.
B.
C.
D.
E.

Deltoid
Flexor carpi ulnaris
Flexor digitorum superficialis
Flexor pollicis brevis
Palmaris longus

The correct answer is A. The posterior cord supplies the axillary and radial nerves. Of the muscles
listed, only the deltoid is supplied by one of these two nerves, specifically the axillary nerve. The
deltoid originates from the clavicle and scapula. It inserts into the deltoid tuberosity of the humerus.
This muscle is responsible for abduction of the arm.
The flexor carpi ulnaris (choice B) is supplied by the ulnar nerve.
The flexor digitorum superficialis (choice C), the flexor pollicis brevis (choice D), and the palmaris
longus (choice E) are supplied by the median nerve.

26. During the process of meiosis, a single homologous chromosome pair fails to separate during the
first meiotic division. This failure would be most likely to produce which of the following conditions if
fertilization occurs and an embryo later develops?
A. Balanced translocation
B. Triploidy

11

C. Trisomy
D. Unbalanced translocation
E. Uniploidy
The correct answer is C. Meiosis is cell division that produces gametes with half of the normal
somatic chromosome complement. The process described is nondisjunction, which will cause one
daughter cell to have 24 chromosomes, while the other will have 22 chromosomes. When a gamete
with the normal 23 chromosomes combines at fertilization with a gamete with 22 or 24 chromosomes,
the embryo will have 47 chromosomes (trisomy) or 45 chromosomes (monosomy). Nondisjunction
can occur in either the first or second meiotic division.
Balanced translocation (choice A) occurs when non-homologous chromosomes exchange genetic
material in such a way that no critical genetic material is lost.
Triploidy (choice B) is the term used when a cell has 69 chromosomes (3N or 3 sets), and can occur
in tumors or when an egg is fertilized by two sperm.
An unbalanced translocation (choice D) occurs when non-homologous chromosomes exchange
genetic material with a net loss or gain of critical genetic material.
Uniploidy (choice E) is the state of having 23 chromosomes, seen normally in sperm and eggs.

27. The superior ophthalmic vein directly communicates with which of the following dural venous
sinuses?
A.
B.
C.
D.
E.

Cavernous sinus
Occipital sinus
Sigmoid sinus
Superior petrosal sinus
Straight sinus

The correct answer is A. The anterior continuation of the cavernous sinus, the superior ophthalmic
vein, passes through the superior orbital fissure to enter the orbit. Veins of the face communicate with
the superior ophthalmic vein. Because of the absence of valves in emissary veins, venous flow may
occur in either direction. Cutaneous infections may be carried into the cavernous sinus and result in a
cavernous sinus infection, which may lead to an infected cavernous sinus thrombosis. The cavernous
sinus is lateral to the pituitary gland and contains portions of cranial nerves III, IV, V1, V2, and VI, and
the internal carotid artery.
The occipital sinus (choice B) is at the base of the falx cerebelli in the posterior cranial fossa. It drains
into the confluence of sinuses.
The sigmoid sinus (choice C) is the anterior continuation of the transverse sinus in the middle cranial
fossa. The sigmoid sinus passes through the jugular foramen and drains into the internal jugular vein.
The superior petrosal sinus (choice D) is at the apex of the petrous portion of the temporal bone and
is a posterior continuation of the cavernous sinus. The superior petrosal sinus connects the cavernous
sinus with the sigmoid sinus.
The straight sinus (choice E) is at the intersection of the falx cerebri and the falx cerebelli in the
posterior cranial fossa. The straight sinus connects the inferior sagittal sinus with the confluence of
sinuses.

28. In preparation for a procedure to remove the fingernail on an index finger, the physician would
most likely anesthetize a branch of the

12

A.
B.
C.
D.
E.

anterior interosseus nerve


median nerve
musculocutaneous nerve
radial nerve
ulnar nerve

The correct answer is B. The median nerve supplies the surface of the lateral palm, the palmar
surface of the first three digits, and the distal dorsal surface of the index and middle fingers (including
the nail beds). Therefore, prior to performing surgery in this area, it is essential to anesthetize a
branch of this nerve (possibly a proper digital branch) to eliminate pain sensation around the nail bed
of the index finger. The median nerve of the branchial plexus distributes to the flexor muscles on the
forearm (flexor carpi radialis and palmaris longus), the pronators (p. quadratus and p. teres), digital
flexors, and skin over the lateral surface of the hand.
Neither the anterior interosseus (choice A) nor the musculocutaneous (choice C) nerves supplies the
hand. The anterior interosseous nerve supplies the flexor pollicis longus, the lateral half of flexor
digitorum profundus, and pronator quadratus. The musculocutaneous nerve supplies the
coracobrachialis, biceps, and most of the brachialis muscle, then becomes the lateral cutaneous
nerve of the forearm.
The radial nerve (choice D) supplies skin on the radial side of the dorsal surface of the hand, but not
the fingertips.
The ulnar nerve (choice E) supplies the palmar and dorsal surfaces of the medial hand, including the
palmar and dorsal surfaces of the fourth and fifth digits.

29. Which glandular area secretes hormones which are products of tyrosine metabolism?
A.
B.
C.
D.
E.

Alpha cells of pancreas


Beta cells of pancreas
Adrenal cortex
Adrenal medulla
Testes

The correct answer is D.The products of the adrenal medulla are epinephrine (adrenalin) and
norepinephrine (noradrenalin). The pathway of production of these compunds is a s follows: tyrosine
to DOPA to dopamine to norepinephrine to epinephrine. The hormone secreted by alpha cells of the
pancreas is glucagon, while beta cells secrete insulin. Both are peptide hormones. The adrenal cortex
secretes a variety of hormones including cortisol and aldosterone, both steroid hormones. The testes
secrete testosterone, also a steroid hormone.

30. A newborn infant has some of its abdominal viscera protruding through a defect in the abdominal
wall. Which of the following is the likely cause of this defect?
A.
B.
C.
D.
E.

Failure of the intestinal loop to retract from the umbilical cord


Failure of the yolk stalk to degenerate
Failure of peritoneal fusion
Incomplete fusion of the lateral body folds
Umbilical herniation

The correct answer is D. During the fourth week of development, the lateral body folds move
ventrally and fuse in the midline to form the anterior body wall. Incomplete fusion results in a defect
that allows abdominal viscera to protrude from the abdominal cavity, a condition known as
gastroschisis.

13

During development, the midgut normally herniates into the umbilical cord and then subsequently
retracts into the abdominal cavity. Failure of the intestinal loop to retract from the umbilical cord
(choice A) results in omphalocele.
Failure of the yolk stalk to degenerate (choice B) results in an ileal (Meckel's) diverticulum or a
vitelline fistula or cyst. In the early embryo, the gut tube is connected to the yolk sac by a narrow
connection known as the yolk stalk. Normally, this connection degenerates.
During development, certain peritoneal organs fuse with the posterior abdominal wall to become
secondarily retroperitoneal. Failure of this peritoneal fusion (choice C) will result in certain organs that
are normally immobile being mobile (e.g., mobile cecum).
Umbilical herniation (choice E) results from abdominal viscera protruding through a weakness in the
abdominal wall after development. Such protrusions are covered by subcutaneous fascia and skin,
distinguishing them from gastroschisis.
31. If a patient has a drooping right eyelid and a dilated right pupil, which of the following neural
structures is most likely affected?
A.
B.
C.
D.
E.

Cervical sympathetic chain


Facial nerve
Oculomotor nerve
Superior cervical ganglion
Trigeminal nerve

The correct answer is C. The oculomotor nerve innervates the levator palpebrae superioris, which
elevates the eyelid. This nerve also innervates the inferior oblique muscles, as well as the superior,
inferior, and medial rectus muscles. The oculomotor nerve also contains preganglionic
parasympathetic fibers that synapse, in the ciliary ganglion, on postganglionic parasympathetic nerve
fibers that innervate the sphincter pupillae muscle, which constricts the pupil. A lesion of the
oculomotor nerve may therefore result in both drooping of the eyelid (ptosis) and dilation of the pupil
(mydriasis).
The cervical sympathetic chain (choice A) contains preganglionic sympathetic nerve fibers, arising
from the upper thoracic spinal cord, which ascend to the cervical sympathetic ganglia. A lesion of
these nerves may result in Horner's syndrome, which includes a ptosis and miosis (pupillary
constriction) and, often, anhidrosis (lack of sweating).
The facial nerve (choice B) innervates the muscles of facial expression, including the orbicularis oculi
muscle. A lesion of this nerve may therefore result in the inability to close the eye.
The superior cervical ganglion (choice D) contains the cell bodies of postganglionic sympathetic
nerves that innervate structures in the head. A lesion of this structure will cause Horner's syndrome.
The trigeminal nerve (choice E) provides sensory innervation to much of the head. A lesion of this
nerve may interfere with the corneal blink reflex.

32. An injection to anesthetize pain from a fracture of the seventh rib should be gien in what area?

A.
B.
C.
D.

Seventh intercostal space immediately below the seventh rib in the midclavicular line
Seventh intercostal space immediately below the seventh rib just lateral to the angle of the rib
Seventh intercostal space immediately below the seventh rib just medial to the angle of the rib
Sixth intercostal space immediately above the seventh rib in the midclavicular line

14

E. Sixth intercostal space immediately above the seventh rib just lateral to the angle of the rib
The correct answer is B. The seventh intercostal nerve (the anterior ramus of the seventh thoracic
spinal nerve) innervates the seventh rib. After passing through the intervertebral foramen between the
seventh and eighth thoracic vertebrae, the nerve lies in the seventh intercostal space. After passing
the angle of the rib, it occupies a position along the lower border of the rib, in the costal groove. Use
of a local anesthetic at this point will anesthetize the rib.
By the time the intercostal nerve has reached the midclavicular line (choice A), it has already
innervated most of the rib. Use of an anesthetic at this point would not be effective.
While the intercostal nerve is in the intercostal space medial to the angle of the rib (choice C), it is not
along the lower border of the rib.
The intercostal nerve does not lie along the upper border of the rib (choices D and E). Thus, injection
at these sites would not be effective.

33. The nucleus that lies immediately medial (and deep) to the uncus is the
A.
B.
C.
D.
E.

amygdala
caudate nucleus
claustrum
hippocampus
putamen

The correct answer is A. The uncus, which is the medial protrusion of the parahippocampal gyrus, is
an external structure seen on the ventral surface of the temporal lobe. The amygdala is a collection of
nuclei that lies directly beneath the uncus.
The caudate nucleus (choice B) is a deep nuclear structure that lies lateral to the lateral ventricles.
The claustrum (choice C) is a thin and elongated nucleus that lies just medial to the insular cortex.
The hippocampus (choice D) is a nuclear structure that lies in the interior of the parahippocampal
gyrus.
The putamen (choice E) is a nuclear structure that resides lateral to the caudate and medial to the
claustrum.

34. A nursing home patient who aspirates while lying on his back would be most likely to develop
pneumonia involving which of the following sites?
A.
B.
C.
D.
E.

Anterior segment of the right upper lobe


Apical segment of the right lower lobe
Inferior lingular segment of the left upper lobe
Lateral segment of the right middle lobe
Superior lingular segment of the left upper lobe

The correct answer is B. Aspiration pneumonia is a common complication observed in nursing home
patients. The most probable site of the pneumonia can be anticipated by knowing the anatomy of the
bronchial tree because the aspirated fluid usually flows downhill. In a supine or nearly supine patient,

15

the fluid flows into the trachea and then into either of the (typically the right) main bronchi. The first
posteriorly located branch is the one leading to the apical aspect of (either) lower lobe. The lateral and
posterior segments of the lower lobes are also supplied by posteriorly branching segmental bronchi.
In contrast, the posterior aspects of the upper lobes are somewhat protected by an initial anteriorly
directed bifurcation before their segmental bronchi arise. All other segments of the bronchial tree and
their corresponding portions of lung are more anterior.

35. The hormone most responsible for regulating sodium balance is secreted from:
A.
B.
C.
D.
E.

Zona glomerulosa of the adrenal cortex


Zona reticularis of the adrenal medulla
Zona fasiculata of the adrenal medulla
Zona fasiculata of the adrenal cortex
Zona glomerulosa of the adrenal medulla

The correct answer is choice A.Firstly, the hormone involved is aldosterone, which acts to increase
sodium resorption in the kidney. Note that aldosterone (a mineralcorticoid) and the glucocorticoids
(cortisol, cortisone) are produced by the adrenal cortex, not medulla. The medulla, with a different
developmental origin and cell type, produces catecholamines such as epinephrine and
norepinephrine. The cortex, which is outside the medulla, is in three regions. On the outside is the
Zona Glomerulosa, source of the mineralcorticoids. Inside of that is the Zona Fasiculata, which
together with the innermost layer of the cortex, the Zona Reticularis, produce glucocortoids.
Remember that interior to the Zona Reticularis, you will find the adrenal medulla. Also note that as a
memory aid, the cortex layers from the outside in are G-F-R, like the GFR of the kidney.

36. During embryological development, hematopoiesis occurs in different organs at different times.
Which of the following are the correct organs, in the correct sequence, at which hematopoiesis occurs
embryologically?

A.
B.
C.
D.
E.

Amnion, yolk sac, placenta, bone marrow


Placenta, liver and spleen, yolk sac, bone marrow
Placenta, spleen and lymphatic organs, bone marrow
Yolk sac, bone marrow, liver and spleen
Yolk sac, liver, spleen and lymphatic organs, bone marrow

The correct answer is E. By the third week of development, hematopoiesis begins in the blood
islands of the yolk sac. Beginning at 1 month of age and continuing until 7 months of age, blood
elements are also formed in the liver. Hematopoiesis occurs in the spleen and lymphatic organs
between 2 and 4 months, and in the bone marrow after 4 months.

16

37. A woman suffers a fracture of the left tenth and eleventh ribs. Which of the following organs is
most likely to have been injured by these fractured ribs?
A.
B.
C.
D.
E.

Descending colon
Jejunum
Left adrenal gland
Left kidney
Spleen

The correct answer is E. The spleen is a soft, friable organ with a thin capsule and is subject to
injury upon trauma to the left side of the abdomen. It is located in the upper left quadrant of the
abdomen, deep to the left ninth, tenth, and eleventh ribs. It is the most commonly injured organ in the
abdomen. The adult spleen contains the largest number of lymphoid tissues in the human body.
The descending colon (choice A) lies in a retroperitoneal position on the left side of the posterior
abdominal wall. The descending colon begins at the splenic flexure immediately inferior to the spleen.
The jejunum (choice B) is a peritoneal structure suspended by a long mesentery. It is located
primarily in the upper left quadrant of the abdomen. Its long mesentery allows the jejunum to be highly
mobile and thus is not likely to be injured by trauma to the body wall.
The left adrenal gland (choice C) is a retroperitoneal structure that lies near the upper pole of the left
kidney. It is embedded within fat and is thus well protected from injury.
The left kidney (choice D) is a retroperitoneal structure that is well protected by fat.
38. The tongue will move in which direction when protuded with surgical damage to the right
hypoglossal nerve?
A.
B.
C.
D.
E.

Downward
Upward
Directly forward
To the right
To the left

The correct answer is D. There are two ways to answer this question. The formula method is that a
tongue with muscle or nerve injury will protrude toward the side of injury, in this case the right side.
This is similar to the case of the mandible protruding toward the side of injury when a lateral pterygoid
is injured.The logical method is to imagine intrinsic muscles and extrinsic protruders of the tongue on
the right side not receiving stimulation from innervation. In this case, only the left side protruders will
operate. The tongue will protrude only on the left side, with the immobile right side acting as a
stationary pivot while the tongue moves from left to right (toward the injured side).

39. You are asked to hold your upper arm against your lateral chest wall, with the palm upward. You
then rotate the hand so that the palm faces downward, without bending the wrist. This motion is
known as:
A.
B.
C.
D.
E.

abduction of the forearm


adduction of the forearm
flexion of the forearm
pronation of the forearm
supination of the forearm

17

The correct answer is D. When the forearm is rotated from anatomic position (palms facing forward,
thumbs out) so that the palm faces posteriorly, the forearm is said to be pronated.
Abduction (choice A) raises the arm to a horizontal position away from the body; adduction (choice
B) is the reverse.
Flexion (choice C) brings the arm or forearm forward, in front of the plane of the body.
Rotation of the forearm so that the palm faces forward (i.e., into anatomic position) is referred to as
supination (choice E).
40. An otherwise healthy student taking no medications is concerned because he has noticed several
painless uniform "large bumps" at the back of his tongue. These are most likely
A.
B.
C.
D.
E.

aphthous ulcers
candidal colonies
circumvallate papillae
filiform papillae
fungiform papillae

The correct answer is C. The large bumps at the back of his tongue are circumvallate papillae.
These are large circular structures surrounded by moat-like depressions. The lateral surfaces of these
papillae contain taste buds. There are also small serous-only salivary glands in these papillae.
Aphthous ulcers (choice A) are small, white, or red mouth lesions.
Candidal colonies (choice B) appear in thrush, which occurs more commonly in the
immunocompromised host or in those taking antibacterial drugs. You are told that the patient is
healthy and not taking medications, making this condition unlikely.
Filiform papillae (choice D) are the most numerous papillae of the tongue. They are small, elongated
cones that create the tongue's rough texture. They do not contain taste buds.
Fungiform papillae (choice E) are mushroom-shaped structures scattered among the filiform papillae.
They frequently contain taste buds. They are intermediate in size between filiform and circumvallate
papillae.

41. Attempts to straighten out a flexed thigh cause great pain in a patient with appendicitis. This is
due to the position of the appendix near which muscle?

A.
B.
C.
D.
E.

Adductor magnus
Biceps femoris
Gluteus maximus
Gracilis
Psoas major

The correct answer is E. The path of the psoas major lies in the retroperitoneum and comes close to
the appendix. Acute appendicitis can cause either infection or a sympathetic inflammation of the
psoas. This produces clinically a "positive psoas sign," in which attempts to straighten the patient's

18

flexed (to relieve pain) hip produce sometimes marked exacerbation of the pain. None of the other
muscles listed pass near the appendix.

42. Which of the following structures does the fetal allantoic duct become in the adult?

A.
B.
C.
D.
E.

Cloaca
Medial umbilical ligament
Urachus
Ureter
Urethra

The correct answer is C. The urachus is a fibrous remnant that extends from the umbilicus to the
urinary bladder. It is also known as the median umbilical ligament of the anterior abdominal wall.
The cloaca (choice A) is the primitive, endoderm-lined region that receives the terminal portion of the
hindgut. It is later subdivided into urogenital and anal areas.
The medial umbilical ligament (choice B) is a paired structure located deep to the peritoneum of the
anterior abdominal wall. It is formed by the obliterated umbilical artery.
The ureter (choice D) is the muscular tube that conveys urine from the kidney to the urinary bladder.
The urethra (choice E) is the passageway that carries urine from the bladder to the perineum.

43. The smooth part of the right atrium derives from which of the following embryonic structures?
A. Bulbus cordis
B. Primitive atrium
C. Primitive ventricle
D. Sinus venosus
E. Truncus arteriosus
The correct answer is D. The smooth part of the right atrium (the sinus venarum) is derived from the
sinus venosus. The coronary sinus and the oblique vein of the left atrium also derive from the sinus
venosus.
The bulbus cordis (choice A) gives rise to the smooth part of the right ventricle (conus arteriosus) and
the smooth part of the left ventricle (aortic vestibule).
The primitive atrium (choice B) gives rise to the trabeculated part of the right and left atria.
The primitive ventricle (choice C) gives rise to the trabeculated part of the right and left ventricles.
The truncus arteriosus (choice E) gives rise to the proximal part of the aorta and the proximal part of
the pulmonary artery.
44. Microscopic examination of a PAS-stained histological section of a Graafian follicle demonstrates
a bright reddish-pink, acellular ring around the ovum. Which of the following terms most accurately
describes this ring?

19

A.
B.
C.
D.
E.

Corona radiata
Cumulus oophorus
Theca externa
Theca interna
Zona pellucida

The correct answer is E. The ring described is the zona pellucida, which surrounds the ovum. The
zona pellucida is rich in polysaccharides and glycoproteins and consequently stains brightly pink or
red with PAS stain. Binding of the sperm cell membrane to the zona pellucida triggers the acrosome
reaction, during which acrosomal enzymes are released that digest the zona pellucida, allowing the
spermatozoon to contact and fuse with the ovum cell membrane. Anatomically, as layers of granulosa
cells develop around the primary oocyte, microvilli from the surrounding granulosa cells intermingle
with cells of the primary oocyte. The microvilli are surrounded by a layer of glycoproteins, and the
entire region is called the zona pellucida.
The follicular cells immediately outside the zona pellucida form the corona radiata (choice A). The
larger cumulus oophorus (choice B) is the hill of follicular cells that surrounds the ovum.
The theca interna (choice D) and externa (choice C) are formed from the connective tissue
surrounding the follicle.

45. Following a surgical procedure on the right side of the neck, a patient can no longer raise his right
arm above the horizontal position. The patient also cannot shrug his right shoulder. Which of the
following nerves was injured?
A.
B.
C.
D.
E.

Axillary nerve
Great auricular nerve
Greater occipital nerve
Spinal accessory nerve
Transverse cervical nerve

The correct answer is D. The spinal accessory nerve crosses the posterior triangle of the neck
immediately deep to the investing fascia of the neck. This nerve innervates the trapezius muscle,
which is responsible for upward rotation and elevation of the scapula. A lesion of this nerve in the
posterior triangle leads to paralysis of the trapezius. Without the ability to upwardly rotate the scapula,
abduction and flexion of the arm above the horizontal plane is not possible. Also, shrugging of the
shoulder is impaired with paralysis of the trapezius muscle.
The axillary nerve (choice A) does not pass through the neck. It is a branch of the brachial plexus,
and it leaves the axilla to innervate the deltoid and teres minor muscles.
The great auricular nerve (choice B) and transverse cervical nerve (choice E) are branches of the
cervical plexus, which provide cutaneous innervation to the skin of the neck. No muscles are
innervated by these nerves.
The greater occipital nerve (choice C) is the dorsal ramus of the second cervical spinal nerve. It
provides cutaneous innervation to the skin of the back of the head. No muscles are innervated by this
nerve.
46. Zygomycosis, a destructive fungal infection of the sinuses, is likely to reach the brain by which of
the following routes?
A. Cavernous sinus
B. External carotid artery

20

C. Internal carotid artery


D. Superior sagittal sinus
E. Superior vena cava
The correct answer is A. This question requires knowledge of pathophysiology with a basic
understanding of anatomy. The cavernous sinuses are located on either side of the body of the
sphenoid bone and become a potential route of infection because they receive blood both from the
face (via the ophthalmic veins and sphenoparietal sinus) and from some of the cerebral veins. The
spread of infection into the cavernous sinus can produce either central nervous system (CNS)
infection or cavernous sinus thrombosis, both of which are potentially fatal.
The route from the face to the brain is not arterial (choices B and C).
The superior sagittal sinus (choice D) is located in the falx cerebri and drains venous blood from the
brain to other dural sinuses, from which it eventually drains into the jugular vein. Zygomycosis does
not reach the brain by way of the superior sagittal sinus.
The superior vena cava (choice E) drains blood from the upper part of the body into the heart.
47. A surgeon inadvertently sections the recurrent laryngeal nerve during a procedure. Which of the
following muscles would retain its innervation subsequent to this injury?
A.
B.
C.
D.
E.

Cricothyroid
Lateral cricoarytenoid
Posterior cricoarytenoid
Thyroarytenoid
Vocalis

The correct answer is A. The recurrent laryngeal nerve is a branch of the vagus nerve, which
innervates all of the intrinsic laryngeal muscles, except for the cricothyroid muscle. The cricothyroid is
attached to the cricoid cartilage and the thyroid cartilage; contraction of this muscle tends to stretch
and adduct the vocal ligament. The cricothyroid is innervated by the external laryngeal nerve.
The lateral cricoarytenoid muscle (choice B) is innervated by the recurrent laryngeal nerve and is
attached to the cricoid cartilage and the arytenoid cartilage. Its contraction causes adduction of the
vocal ligament.
The posterior cricoarytenoid muscle (choice C) is innervated by the recurrent laryngeal nerve and is
attached to the cricoid cartilage and the arytenoid cartilage. Its contraction causes abduction of the
vocal ligament.
The thyroarytenoid muscle (choice D) is innervated by the recurrent laryngeal nerve and is attached
to the thyroid cartilage and the arytenoid cartilage. Its contraction causes slackening of the vocal
ligament.
The vocalis muscle (choice E) is the most medial part of the thyroarytenoid muscle. It attaches to
either the thyroid cartilage and the vocal ligament, or the arytenoid cartilage and the vocal ligament. It
is innervated by the recurrent laryngeal nerve. Its contraction causes tension on segments of the
vocal ligament.
48. As a result of a viral infection, a patient has swelling of the left facial nerve within the facial canal.
The patient's face appears asymmetrical, and he complains that saliva drips from his mouth while he
is chewing. Paralysis of which of the following muscles accounts for these symptoms?
A.
B.
C.
D.

Buccinator
Masseter
Palatoglossus
Palatopharyngeus

21

E. Temporalis

The correct answer is A. Compression of the facial nerve within the facial canal may result in facial
palsy (Bell's palsy). Because the muscles on one side of the face are paralyzed, the face appears
asymmetrical. The buccinator muscle, which is located within the cheek and is innervated by the facial
nerve, functions to hold food against the teeth while it is being chewed. Paralysis of this muscle can
result in food and saliva accumulating between the teeth and the cheek. The buccinator originates
from the alveolar processes of the maxilla and mandible. It inserts into the fibers of the obicularis oris.
The masseter and temporalis muscles (choices B and E) are innervated by the mandibular division
of the trigeminal nerve. These muscles of mastication function to elevate the mandible.
The
palatoglossus (choice C) and palatopharyngeus (choice D) muscles are innervated by the vagus
nerve. The palatoglossus, with its mucosal covering, forms the palatoglossal fold (anterior pillar of the
fauces), immediately anterior to the palatine tonsil. This muscle functions to draw the tongue and soft
palate closer together, as occurs during swallowing. The palatopharyngeus, with its mucosal covering,
forms the palatopharyngeal fold (posterior pillar of the fauces), which is immediately posterior to the
palatine tonsil. This muscle causes elevation of the pharynx, as occurs during swallowing.
49. Biopsy demonstrates epithelial metaplasia. Which of the following cell types was most likely
observed in the involved areas?
A. Ciliated columnar epithelium
B. Cuboidal epithelium
C. Keratinizing squamous epithelium
D. Nonciliated columnar epithelium
E. Nonkeratinizing squamous epithelium
The correct answer is D. The medical condition is Barrett's esophagus, in which the normally
nonkeratinizing squamous epithelium (choice E) of the esophagus undergoes metaplasia to gastric or
intestinal-like epithelium composed of nonciliated columnar epithelial cells. Barrett's esophagus
typically develops in the setting of chronic gastroesophageal reflux and significantly increases the risk
of later development of adenocarcinoma of the distal esophagus.
Ciliated columnar epithelium
(choice A) is found in the respiratory tract.
Cuboidal epithelium (choice B) is found in the kidney,
peritoneal lining, and pleural lining.
Keratinizing squamous epithelium (choice C) is found in
skin.
Nonkeratinizing squamous epithelium (choice E), in addition to being the normal
epithelium of the esophagus, is found in mouth, nose, and vagina.
50. Despite blockage of the celiac trunk, the organs receiving their blood supply from the trunk
continue to operate normally. This is due to anastomoses between which vessels?
A. Left gastroepiploic artery and right gastroepiploic artery
B. Left gastroepiploic artery and right gastroepiploic artery
C. Proper hepatic artery and gastroduodenal artery
D. Right colic artery and middle colic artery
E. Superior pancreaticoduodenal artery and inferior pancreaticoduodenal artery
The correct answer is E. The superior pancreaticoduodenal artery is a branch of the gastroduodenal
artery, which is a branch of the common hepatic artery, itself a branch of the celiac trunk. The inferior
pancreaticoduodenal artery is a branch of the superior mesenteric artery. Occlusion of the celiac trunk
would allow blood from the superior mesenteric artery to reach the branches of the celiac trunk via the
connections between the superior and inferior pancreaticoduodenal arteries.
The left gastric and
right gastric arteries (choice A) both receive their blood from the celiac trunk. The left gastric artery is
a direct branch of the celiac trunk. The right gastric artery is usually a branch of the proper hepatic
artery, which is a branch of the common hepatic artery (a branch of the celiac trunk).
The left and
right gastroepiploic arteries (choice B) both receive their blood supply from the celiac trunk. The left
gastroepiploic artery is a branch of the splenic artery, which is a branch of the celiac trunk. The right
gastroepiploic artery is a branch of the gastroduodenal artery, which is a branch of the common
hepatic artery (a branch of the celiac trunk).
The proper hepatic and gastroduodenal arteries
(choice C) are branches of the common hepatic artery, which is a branch of the celiac trunk.
The
right colic and middle colic arteries (choice D) are both branches of the superior mesenteric artery.

22

51. Damage to which of the following structures might produce hair cell loss?

A.
B.
C.
D.
E.

Basilar membrane
Organ of Corti
Reissner's membrane
Scala tympani
Scala vestibuli

1H

The correct answer is B. Hearing is the detection of sound, which consists of pressure waves
conducted through air or water. The receptors of the cochlear duct provide us with a sense of hearing
that allows us to detect the quietest whisper and yet remain functional in a crowded, noisy
environment. The organ of Corti contains hair cells from the cochlear branch of the vestibulocochlear
nerve (CN VIII). These cells rest on the basilar membrane (choice A), which separates the scala
tympani (choice D) from the scala media. The hair cells are embedded in the tectorial membrane,
and movement of the basilar membrane below the cells causes the hairs to bend, which generates
action potentials. The tectorial membrane that lies on the hair cells does not form a boundary between
the different scala; the membrane separating the scala media from the scala vestibuli (choice E) is
Reissner's (vestibular) membrane (choice C).

52. Which of the following locations in the embryo later forms the dorsal horn of the spinal cord?

A.
B.
C.
D.
E.

Alar plate
Basal plate
Neural crest
Rostral end of neural tube
Sulcus limitans

The correct answer is A. The spinal cord arises from the caudal end of the neural tube. During
development, an alar and a basal plate is formed, separated by a longitudinal groove called the sulcus
limitans (choice E). The alar plate forms the dorsal (posterior) part of the spinal cord and becomes
the sensory or afferent portion of the cord. The basal plate (choiceB) is the ventral (anterior) part of
the cord and becomes the motor, or efferent, portion of the spinal cord and therefore would contain
anterior horn cells.
The neural crest (choice C) develops into multipolar ganglion cells of autonomic ganglia,
pseudounipolar cells of spinal and cranial nerve ganglia, leptomeningeal cells, Schwann cells,
melanocytes, chromaffin cells of the adrenal medulla, and odontoblasts.
The brain forms from the rostral end of the neural tube (choice D).

53. From which of the following fetal vessels do the umbilical arteries arise?
A.
B.
C.
D.
E.

Aorta
Carotid arteries
Ductus arteriosus
Iliac arteries
Pulmonary arteries

23

The correct answer is D. The paired umbilical arteries arise from the iliac arteries. They supply
unoxygenated fetal blood to the placenta. The single umbilical vein takes the newly oxygenated fetal
blood from the placenta to the liver and then to the inferior vena cava via the ductus venosus. Near
the level of vertebra L4, the terminal segment of the abdominal aorta divides to form the right and left
common iliac arteries. These arteries carry blood to the pelvis and lower limbs. As these arteries
travel along the inner surface, they descend behind the cecum and sigmoid colon, where each divides
to form the internal iliac artery and external iliac artery.
54. A mass in the anterior midline of the neck, slightly above the larynx is mobile and elevates upon
protrusion of the tongue. This mass is most likely a cyst that developed from which of the following
embryonic structures?
A.
B.
C.
D.
E.

First pharyngeal cleft


First pharyngeal pouch
Second pharyngeal cleft
Second pharyngeal pouch
Thyroglossal duct

The correct answer is E. The thyroglossal duct develops as an evagination of the floor of the
pharynx in the region where the tongue develops. The adult foramen cecum of the tongue marks the
site of this evagination. The distal end of this duct normally forms the thyroid gland; the proximal part
of the duct normally degenerates. Failure of a part of the duct to degenerate may lead to a
thyroglossal duct cyst or a median cervical cyst, as seen in this patient.
The first pharyngeal cleft (choice A) forms the external ear canal. This cleft normally remains patent.
The first pharyngeal pouch (choice B) forms the middle ear cavity and the auditory tube. This pouch
normally remains patent.
The second pharyngeal cleft (choice C) normally does not remain patent. It is typically covered over
by the overgrowth of the second pharyngeal arch. If part of this pouch does remain patent, it may form
a lateral cervical cyst, which is seen on the lateral side of the neck along the anterior border of the
sternocleidomastoid muscle.
The second pharyngeal pouch (choice D) forms the tonsillar fossa of the pharynx. The pharyngeal
mucosa in this area arises from the endoderm of the pouch. Ingrowth of mesoderm cells results in the
formation of the palatine tonsil.

55. Which of the following sites contains striated muscle that is not under voluntary control?
A. Bladder
B. Colon
C. Esophagus
D. Gallbladder
E. Stomach
The correct answer is C. Striated (skeletal) muscle not under voluntary control is an unusual feature
of the upper and middle thirds of the esophagus. The middle third of the esophagus contains roughly
half striated and half smooth muscle; the lower third contains only smooth muscle. All the other
structures listed in the answer choices contain smooth muscle.
As a side note, cardiac muscle cells do not rely on nerve activity to start a contraction. Instead,
specialized pacemaker cells establish a regular rate of contraction. Because these pacemaker cells
regulate the heart and the central nervous system does not, cardiac muscle is also considered
striated, involuntary muscle.

24

56. An ulcer damages an artery supplying the area of the greater curvature of the stomach. Which
artery is involved?

A.
B.
C.
D.
E.

Left gastric
Left gastroepiploic
Right gastric
Right gastroepiploic
Short gastric

The correct answer is D. The right gastroepiploic artery, off the gastroduodenal artery, supplies the
right half of the greater curvature of the stomach and could be directly affected by ulceration of the
greater curvature of the stomach at a site this close (4 cm) to the pyloric sphincter.
The left gastric artery (choice A), off the celiac trunk, supplies the left half of the lesser curvature of
the stomach.
The left gastroepiploic artery (choice B), off the splenic artery, supplies the left half of the greater
curvature of the stomach. Although it anastomoses with the right gastroepiploic artery, it is unlikely
that this artery would be directly damaged by ulceration of the stomach near the pyloric sphincter.
The right gastric artery (choice C), off the proper hepatic artery, supplies the right half of the lesser
curvature of the stomach.
The short gastric artery (choice E), actually one of several (4 to 5) short gastric arteries, off the
splenic artery (occasionally the left gastroepiploic), supplies the fundus of the stomach, which is the
most distant from the pylorus.

57. An elderly patient has had multiple small strokes lending to an absence of the gag reflex. These
findings suggest involvement of the nucleus of which of the following cranial nerves?
A.
B.
C.
D.
E.

Facial (VII)
Glossopharyngeal (IX)
Hypoglossal (XII)
Spinal accessory (XI)
Vestibulocochlear (VIII)

The correct answer is B. Cranial nerve IX is the glossopharyngeal nerve, which has a nucleus in the
medulla and is necessary for the gag reflex. The gag reflex is elicited by touching either side of the
posterior pharynx with a tongue blade, producing bilateral elevation of the palate and bilateral
contraction of the pharyngeal muscles. The afferent of this reflex arc consists of the ipsilateral
glossopharyngeal nerve, while the vagus nerve, bilaterally, supplies the efferent limb. Although the
glossopharyngeal nerve may seem to be one of the less important cranial nerves, you should
remember to test for its function, as a loss of gag reflex can lead to the patient's death secondary to
an aspiration pneumonia. The glossopharyngeal nerve is a mixed sensory and motor nerve to the
head and neck. It originates from the posterior 1/3 of the tongue, pharynx, palate, and carotid arteries
of the neck. Its destination includes sensory nuclei of the medulla oblongata, as well as the
pharyngeal muscles involved in swallowing.
Cranial nerve VII (choice A) is the facial nerve, which supplies motor function to the face, but it does
not supply the oropharynx.
Cranial nerve XII (choice C) is the hypoglossal nerve, which supplies
the intrinsic and most extrinsic muscles of the tongue. It is not involved in the gag reflex.

25

Cranial nerve XI (choice D) is the spinal accessory nerve, which supplies the trapezius and
sternocleidomastoid.
Cranial nerve VIII (choice E) is the vestibulocochlear nerve, responsible
for hearing and equilibrium.
58. Which of the following respiratory system components is derived from neural crest?
A.
B.
C.
D.
E.

Endothelial cells
Epithelium of primary bronchi
Laryngeal cartilage
Tracheal glands
Type I pneumocytes

The correct answer is C. Laryngeal cartilages are derived from neural crest. The larynx is composed
of three cartilages that form the "body of the larynx": the thyroid cartilage, the cricoid cartilage, and the
epiglottis. The larynx also contains three pairs of smaller hyaline cartilages: the arytenoid, corniculate,
and cuneiform cartilages.
The endothelial cells (choice A), in the simple squamous epithelium that lines the pulmonary
capillaries, are derived from visceral mesoderm.
The epithelial lining of primary bronchi (choice B) is derived from endoderm.
Tracheal glands (choice D) and epithelium both derive from endoderm.
(choice E) are derived from endoderm.

Type I pneumocytes

59. The extraocular muscles are derived from which of the following structures?

A.
B.
C.
D.
E.

Branchial arches
Optic cup ectoderm
Somites
Somitomeres
Splanchnic mesoderm

The correct answer is D. The somitomeres are specialized masses of mesoderm found in the head
region that give rise to the muscles of the head. The extraocular muscles are derived from
somitomeres 1, 2, 3, and 5.
The branchial arches (choice A) give rise to muscles of mastication (arch 1), muscles of facial
expression (arch 2), and muscles of the pharynx and larynx (arches 3-6), as well as additional small
muscles.
The optic cup ectoderm (choice B) gives rise to the muscles of the iris (sphincter and dilator pupillae).
These are the only muscles not formed from mesoderm.
Somites (choice C) give rise to the inferior muscles of the neck.
Splanchnic mesoderm (choice E) gives rise to smooth muscle of the viscera and the heart muscle.
60. A CT scan reveals a small tumor at the cerebellopontine angle of the brain. Which of the following
nerves is most likely to be affected by this tumor?
A.
B.
C.
D.

Facial nerve
Glossopharyngeal nerve
Optic nerve
Trigeminal nerve

26

E. Vagus nerve
The correct answer is A. The facial nerve and the vestibulocochlear nerves emerge from the brain
stem at the cerebellopontine angle. These are the two nerves that will be initially affected by a tumor
in this region. The entire anatomy of the facial nerve is as follows: the facial nerve originates from the
taste receptors on the anterior 2/3 of the tongue (sensory) and from nuclei of the pons (motor). It
passes through the internal acoustic canal of the temporal bone to reach the stylomastoid foramen.
The glossopharyngeal and vagus nerves (choices B and E) emerge from the brain stem at the
postolivary sulcus. This is caudal to the cerebellopontine angle.
The optic nerve (choice C) exits from the optic chiasm on the ventral surface of the diencephalon.
This is rostral to the cerebellopontine angle.
The trigeminal nerve (choice D) emerges from the brain stem at the anterolateral surface of the pons.
This is rostral and ventral to the cerebellopontine angle.
61. A patient is unable to close her right eye. Physical examination reveals weakness of the right
orbicularis oculi. Which of the following additional symptoms would likely also be present?
A.
B.
C.
D.
E.

Blurred vision
Hyperacusis
Inability to chew
Inability to feel the face
Inability to shrug the shoulder

The correct answer B. The facial nerve innervates the muscles of facial expression. The visceral
motor portion innervates the lacrimal gland and nasal mucous glands via the sphenopalatine ganglion.
The submandibular and sublingual salivary glands are innervated via the submandiublar ganglion.
This patient has a lesion of the facial nerve (VII), which leads to an inability to close the ipsilateral eye
because of damaged motor fibers to the orbicularis oculi. This patient would also lose her corneal
reflex on that side because of an inability to blink and would have ipsilateral paralysis of the muscles
of facial expression distal to the lesion. If the lesion affected the facial nerve more proximally,
additional findings would be hyperacusis (increased sensitivity to sound because of stapedius muscle
paralysis), lack of taste sensation in the anterior two-thirds of the tongue, and disturbed lacrimation
and salivation.
Blurred vision (choice A) could occur with lesions of the oculomotor (CN III), abducens (CN VI), or
trochlear (CN IV) nerves, which innervate the extraocular muscles. CN III innervates the medial
rectus, inferior rectus, superior rectus, and inferior oblique muscles. CN VI innervates the lateral
rectus and CN IV innervates the superior oblique.
An inability to chew (choice C) would probably be the result of a lesion of the trigeminal nerve (CN V).
Motor fibers of CN V innervate the muscles of mastication (temporalis, masseter, and medial and
lateral pterygoid muscles), and a lesion of these fibers may cause the jaw to deviate to the side of the
weak muscles.
An inability to feel the face (choice D) would also be the result of a CN V lesion. This lesion could
result in the ipsilateral loss of general sensation of the face and also of the mucous membranes of the
oral and nasal cavities.
A lesion of the accessory nerve (CN XI) would cause paralysis of the trapezius muscle, which results
in a sagging of the shoulder and a weakness in attempting to shrug the shoulder (choice E).
62. A patient received a severe blow to the lateral side of the head, resulting in an epidural
hematoma. Which of the following blood vessels was most likely torn?
A. Anterior cerebral artery

27

B.
C.
D.
E.

Middle cerebral artery


Middle meningeal artery
Superficial temporal artery
Superior cerebral vein

The correct answer is C. The middle meningeal artery is in the interior of the lateral portion of the
cranial cavity, embedded in the periosteal (outer) layer of the dura. A tear of this artery results in blood
entering the potential space between the outer dural layer and the skull (epidural space), causing an
epidural hematoma.
The anterior cerebral artery and middle cerebral artery (choices A and B) lie on the surface of the
brain. The anterior cerebral arteries supply the medial surface of the cerebral hemispheres, and the
middle cerebral arteries supply the lateral surface of the cerebral hemispheres. A tear of either of
these arteries would result in blood entering the subarachnoid space (subarachnoid hemorrhage).
The superficial temporal artery (choice D) is a branch of the external carotid artery and is external to
the skull. This artery supplies the skin and other tissue of the temple region.
The superior cerebral veins (choice E) drain the cerebral hemisphere and enter the superior sagittal
sinus. A tear of these veins results in blood entering the potential space between the dura and
arachnoid (subdural space), causing a subdural hematoma.
63. The x-ray of a child's arm after a fall appears to show a fracture near, but not at, the distal end of
the ulna. Before diagnosing a fracture, you should also consider the possibility that this is actually
which of the following?
A.
B.
C.
D.
E.

Articular cartilage
Epiphyseal plate
Perichondrium
Primary ossification center
Secondary ossification center

The correct answer is B. The epiphyseal plate of the bone contains cartilage that is radiolucent. The
plate in a bone that is not yet fully ossified can produce a "line" crossing the bone near the end. This
may be easily mistaken for a fracture by the inexperienced. Anatomically, the epiphyseal plate
separates the epiphysis from the diaphysis.
Articular cartilage (choice A) is radiolucent, but occurs at the very tip of the long bones.
Perichondrium (choice C) is usually difficult to see on x-ray.
Primary (choice D) and secondary (choice E) ossification centers are radiopaque.
64. Which of the following embryonic structures gives rise to the adrenal cortex?
A. Ectoderm
B. Endoderm
C. Mesoderm
D. Mesonephros
E. Neural crest cells
The correct answer is C. The mesoderm gives rise to the adrenal cortex. In addition, it also gives
rise to connective tissue, cartilage, bone, muscle, blood and lymph vessels, kidneys, gonads, serous
membranes lining body cavities, and the spleen.
The
ectoderm (choice A) gives rise to the central nervous system, peripheral nervous system, epidermis
and its appendages, mammary glands, pituitary gland, tooth enamel, and the neural crest. The
endoderm (choice B) gives rise to the parenchyma of the tonsils, thyroid and parathyroid glands,
thymus, liver, pancreas, the epithelial lining of the gastrointestinal and respiratory tracts, urinary
bladder, urethra, and auditory tube. The mesonephros (choice D) functions as an interim kidney in
the embryo. The neural crest cells (choice E) give rise to cells of the spinal and cranial nerves,
autonomic ganglia, melanocytes, leptomeninges, connective tissue and bone of branchial arch origin,
and the adrenal medulla.

28

65. The major structural component of the sperm flagellum is the:


A. microtubule
B. microfilament
C. actin filament
D. mysosin filament
E. ciliary body
The correct answer is choice A. This is a basic histological question of a type found commonly on
NBDE, which asks about basic cellular organelles and structures. The sperm cell flagellum is notable
for a 9+2 arrangement of fibers formed from microtubules. The 9 single fibers form a circular ring in
cross section around the 2 doubled fibers in the center. One of the significant facts about the fibers is
that they are composed of microtubules, made up of tubulin. The other significant fact is that all
eucaryotic flagella and cilia have this same structure (for example, cilia form the tracheal lining). Note
that procaryotic (bacterial) flagellea do NOT share this structure.
66. In cases of dysphagia where the esophagus is compressed, which structure would be most likely
to cause the compression?
A.
B.
C.
D.
E.

Left atrium
Left ventricle
Pulmonary trunk
Right atrium
Right ventricle

The correct answer is A. The left atrium forms most of the posterior wall of the heart. The
esophagus passes immediately posterior to the heart. Enlargement of the left atrium may compress
the esophagus and cause dysphagia. Anatomically, the esophagus begins posterior to the cricoid
cartilage, at the level of vertebrum C6. From this point, it descends toward the thoracic cavity posterior
to the trachea, passes inferiorly along the dorsal wall of the mediastinum, and enters the
abdominopelvic cavity through an opening in the diaphragm, the diaphragmatic hiatus.
The left ventricle (choice B) forms most of the left border of the heart and most of the diaphragmatic
surface of the heart. The left ventricle is not related to the esophagus.
The pulmonary trunk (choice C) emerges from the right ventricle on the anterior surface of the heart.
The pulmonary trunk is not related to the esophagus.
The right atrium (choice D) forms the right border of the heart. It is not related to the esophagus.
The right ventricle (choice E) forms most of the anterior wall of the heart and a small portion of the
diaphragmatic surface of the heart. It is not related to the esophagus.
67. Which of the following tissues normally has the highest percentage of mucus-secreting cells?
A.
B.
C.
D.
E.

Esophageal mucosa
Oral mucosa
Parotid gland
Sublingual gland
Submandibular gland

The correct answer is D. The sublingual salivary glands are located beneath the mucous membrane
of the floor of the mouth. Numerous sublingual ducts open along either side of the lingual frenulum.
Salivary glands can contain predominantly serous cells, predominantly mucous cells, or both in their
acini. As you proceed from the midline laterally, acini in the sublingual gland are almost pure mucous
cells, whereas acini in the submandibular gland (choice E) contain a mixture of serous and mucous
cells. Acini in the parotid gland (choice C) are mostly pure serous cells.

29

The esophageal mucosa (choice A) and the oral mucosa (choice B) are squamous epithelia that do
not form acini.

68. The primary arterial supply to the nasal mucosa is a direct branch of which of the following
arteries?
A.
B.
C.
D.
E.

Facial artery
Maxillary artery
Superficial temporal artery
Superior labial artery
Transverse facial artery

The correct answer is B. The major source of blood supply to the nasal mucosa is the
sphenopalatine artery, which is the terminal branch of the maxillary artery. The sphenopalatine artery
enters the nasal cavity from the pterygopalatine fossa by passing through the sphenopalatine foramen
in the lateral wall of the nasal cavity.
The facial artery (choice A) is a direct branch of the external carotid artery. It provides most of the
blood supply to the superficial face.
The superficial temporal artery (choice C) is a terminal branch of the external carotid artery. The
other terminal branch is the maxillary artery. The superficial temporal artery provides blood supply to
the temporal region and the lateral portion of the scalp.
The superior labial artery (choice D) is a branch of the facial artery. The superior labial artery
provides blood supply to the upper lip. It has a septal branch that provides some of the blood supply
to the anterior portion of the septal mucosa of the nasal cavity. However, this is not the major blood
supply to the nasal cavity.
The transverse facial artery (choice E) is a branch of the superficial temporal artery. It provides blood
supply to the parotid gland, the parotid duct, and the skin of the lateral face.

69. Which of the following cranial nerves carries the pain sensation from the tip of tongue?
A.
B.
C.
D.
E.

V2
V3
VII
IX
X

The correct answer is B. This question is essentially asking "which of the following nerves
innervates the tip of the tongue?" The innervation of the tongue is complex. The mandibular division
of the trigeminal nerve (V3) carries general somatic sensation from the anterior two-thirds of the
tongue. The mandibular branch is also a sensory nerve for the lower gingiva, teeth, and lips, as well
as the palate.
The maxillary division (V2, choice A) carries somatic sensation from the palate, upper gingiva, and
upper lip, as well as the lower eyelid, cheek, nose, and a portion of the pharynx.
The facial nerve (VII, choice C) carries taste from the anterior two-thirds of the tongue, as well as
innervating the lacrimal gland and the submandibular and sublingual salivary glands.

30

The glossopharyngeal nerve (IX, choice D) carries sensation and taste from the posterior one-third of
the tongue.
The vagus nerve (X, choice E) carries sensation from the lower pharynx.

70. A mass is noted at the back of a young man's tongue. A biopsy's pathology report comes back
with a diagnosis of normal thyroid tissue. This finding is related to the embryonic origin of the thyroid
near which of the following structures?
A.
B.
C.
D.
E.

First pharyngeal pouch


Foramen cecum
Nasolacrimal duct
Second pharyngeal arch
Third pharyngeal pouch

The correct answer is B. The thyroid gland originates as a mass of endodermal tissue near the
foramen cecum, which is near the tuberculum impar (which becomes the central part of the tongue).
During development, the thyroid descends in front of the pharynx, maintaining a connection to the
tongue via the thyroglossal duct. Usually, the thyroglossal duct disappears. Uncommonly, residual
ectopic thyroid tissue can be left anywhere along the path, including at the back of the tongue. (In rare
cases, all of the thyroid tissue remains at this site, forming a mass that should not be excised, for
obvious reasons!) Anatomically, the thyroid gland curves across the anterior surface of the trachea
just below the thyroid cartilage that forms the majority of the anterior surface of the larynx. The two
lobes of the thyroid gland are united by a slender connection, the isthmus.
The first pharyngeal pouch (choice A) develops into the middle ear and eustachian tube.The
nasolacrimal ducts (choice C) connect the eyes to the oropharynx. The second pharyngeal arch
(choice D) develops into many muscles of the face and the styloid process of the temporal bone. The
third pharyngeal pouch (choice E) develops into the thymus and inferior parathyroid glands.
71. If the nerve that accompanies the superior laryngeal artery is damaged, which of the following
functional losses will ensue?
A.
B.
C.
D.
E.

Loss of sensation in the laryngeal mucosa above the vocal folds


Loss of sensation in the laryngeal mucosa below the vocal folds
Loss of sensation in the pharyngeal mucosa
Paralysis of the cricothyroid muscle
Paralysis of the lateral cricoarytenoid muscle

The correct answer is A. The superior laryngeal artery is a branch of the superior thyroid artery. It
enters the larynx by passing through the cricothyroid membrane. In this region, it is accompanied by
the internal branch of the superior laryngeal nerve. This nerve provides sensory innervation to the
laryngeal mucosa above the vocal folds.
The laryngeal mucosa below the vocal folds (choice B) receives its sensory innervation from the
recurrent laryngeal nerve.
The pharyngeal mucosa (choice C) receives its sensory innervation
from the glossopharyngeal nerve.
The cricothyroid muscle (choice D) receives its motor
innervation from the external branch of the superior laryngeal nerve.
The lateral cricoarytenoid
muscle (choice E) receives its motor innervation from the recurrent laryngeal nerve.

72. An atrial septal defect results from failure of the


A.
B.
C.
D.

ostium primum to form within the septum primum


ostium secundum to form within the septum primum
septum primum to fuse with the endocardial cushions
septum primum to fuse with the septum secundum

31

E. septum secundum to fuse with the endocardial cushions


The correct answer is C. The septum primum (first interatrial septum) develops by growing from the
cranial end of the embryonic atrium toward the endocardial cushions. The gap that exists between the
two atria during this period is the ostium primum. As the septum primum continues its growth, the
ostium primum gets smaller until it is closed when the septum primum completes its growth and
completely fuses with the endocardial cushions. Failure of the septum primum to fuse completely with
the endocardial cushions leaves a persistent ostium primum, known as a primum-type atrial septal
defect.
The ostium primum does not form within the septum primum (choice A). The ostium primum is the
communication between the two atria that exists during the formation of the septum primum. That is,
the ostium primum is the space within the developing atrium not yet occupied by the septum primum.
The ostium secundum normally forms within the septum primum (choice B) before the ostium primum
closes by fusion of the septum primum with the endocardial cushions. Failure of the ostium secundum
to form would result in embryonic death because there would be no pathway for blood to pass from
the right atrium to the left atrium when the ostium primum closes, thus depriving the embryo of
oxygenated blood. Most of the septum primum normally disappears. The part that remains forms the
valve of the foramen ovale. This part of the septum primum normally does not fuse with the septum
secundum (choice D) during prenatal life. After birth, the valve of the foramen ovale is pushed against
the septum secundum as a result of the increased pressure in the left atrium. This achieves functional
closure of the foramen ovale. Fusion does not normally occur at this time; it usually occurs later in life
in most people. In some people, however, complete fusion never occurs (probe patency). The septum
secundum normally does not fuse with the endocardial cushions (choice E).
73. A CT scan of the head demonstrates a mass in the olfactory groove area. The axons likely
compressed by this mass project to which of the following structures?
A.
B.
C.
D.
E.

Insula
Nucleus ambiguus
Postcentral gyrus
Precentral gyrus
Pyriform cortex

The correct answer is E. The olfactory nerves can be damaged by head injury, severe infection, and
tumors (such as meningiomas) that may compress the olfactory bulbs. Unilateral damage is often not
noticed by the patient, but bilateral damage (which surprisingly can also be missed if it develops
slowly) may lead to occasional bizarre errors in odor identification. The olfactory nerves are
technically the neurons on the mucosal side of the cribriform plate that send processes through the
plate to synapse in the olfactory bulb, which in turn sends axons to the pyriform cortex (the primary
olfactory cortex).
The insula (choice A) is thought to contain the primary gustatory (taste) cortex.
The nucleus ambiguus (choice B) provides the special visceral efferent fibers carried by the
glossopharyngeal and vagus nerves.
The postcentral gyrus (choice C) contains the primary sensory cortex.
The precentral gyrus (choice D) contains the primary motor cortex.
74. A biopsy specimen demonstrates a ciliated columnar epithelium. From which of the following
locations in the female genital tract was the biopsy obtained?
A.
B.
C.
D.

Cervix
Endometrium
Fallopian tube
Ovary

32

E. Vagina
The correct answer is C.The fallopian tube is the only structure in the female genital tract with a
ciliated columnar epithelium; the beating of the cilia helps move the egg into the uterus. This fact is
also sometimes clinically helpful because dilated and deformed fallopian tubes can be microscopically
distinguished from cystic ovarian tumors by the presence of the cilia. Each uterine tube is a hollow,
muscular tube measuring approximately 13 cm in length. Each uterine tube is divided into three
regions: the infundibulum, the ampulla, and the isthmus.
The cervix (choice A) and vagina (choice E) are lined by squamous epithelium.
The endometrium (choice B) is lined by columnar epithelium (although a few ciliated cells may be
present).
The covering of the ovary (choice D) is cuboidal epithelium, and cysts within the ovary can be lined
by cuboidal or nonciliated columnar epithelium.
75. A person sits with his legs together and then opens his legs against lateral resistance. Which
nerve controls this type of movement?
A.
B.
C.
D.
E.

deep peroneal nerve


femoral nerve
obturator nerve
sciatic nerve
superior gluteal nerve

The correct answer is E. The motion described is hip abduction. The superior gluteal nerve from
roots L4-S1 is responsible for hip abduction.
The deep peroneal nerve (choice A) controls dorsiflexion of the foot. If you see a patient on the
NBDE with "foot drop," think of damage to the deep peroneal nerve.
The femoral nerve (choice B) is responsible for knee extension.
The obturator nerve (choice C) is responsible for hip adduction. The hip adduction machine would
provide resistance to the medial aspects of the thighs, and the individual would push his/her legs
together against the machine's resistance.
The sciatic nerve (choice D) controls knee flexion.

76. During anatomy lab, a dental student notes a fibrous band that runs on the visceral surface of the
liver. It is attached on one end to the inferior vena cava and on the other end to the left branch of the
portal vein. In the embryo, this structure corresponds to the
A.
B.
C.
D.
E.

ductus venosus
ligamentum teres
ligamentum venosum
umbilical arteries
umbilical vein

The correct answer is A. This question could have tricked you if you didn't catch the key words, "in
the embryo." If you read the question too quickly and thought you were going to be asked to identify
the structure described, you probably chose choice C (ligamentum venosum) because that is indeed
the structure in question. In the embryo, however, this fibrous band is actually the ductus venosus.
The ductus venosus is an embryonic vessel that allows blood to bypass the fetal liver; this prevents
the depletion of oxygen and nutrient-rich blood in the hepatic sinusoids.

33

The embryonic umbilical arteries (choice D) become the medial umbilical ligaments.
The embryonic umbilical vein (choice E) actually becomes the fibrous ligamentum teres (choice B).
The ligamentum teres is located in the free margin of the falciform ligament.

77. Damage to the upper fibers of the trapezius muscle would most likely impair which of the
following movements?
A.
B.
C.
D.
E.

Depression of the medial end of the scapula


Elevation of the acromion
Flexion of the neck
Raising of the medial border of the scapula
Turning of the face sideways

The correct answer is B. The trapezius is a large muscle arising in the posterior midline from the
superior nuchal line of the skull, ligamentum nuchae, and spinous processes of all of the thoracic
vertebrae. The upper fibers, which are potentially damaged by wounds to the posterior and lateral
neck, insert on the crest of the spine of the scapula, the medial border of the acromion, and the lateral
third of the clavicle. The upper fibers serve to elevate the acromion.
The lower fibers of the trapezius depress the medial end of the scapula (choice A).
The sternocleidomastoids are important in extension of the head at the atlanto-occipital joint and
flexion of the cervical vertebral column (choice C).
The rhomboids and levator scapulae are important in raising the medial border of the scapula (choice
D).
Contraction of a single sternocleidomastoid muscle is important in turning of the face sideways
(choice E) in the contralateral direction.

78. A man's chest is compressed during a car accident, causing a posterior displacement of the
clavicle at the sternoclavicular joint. Which of the following structures would be most at risk?
A.
B.
C.
D.
E.

Aorta
Esophagus
Heart
Superior vena cava
Trachea

The correct answer is E. The sternoclavicular joint is quite strong and dislocates only with difficulty.
Dislocations, particularly posterior dislocations, are occasionally seen and may be dangerous
because of impingement onto the trachea, causing respiratory difficulties. The trachea is a tough
flexible tube with a diameter of approximately 2.5 cm and a length of approximately 11 cm. The
trachea begins anterior to the vertebra C6 in a ligamentous attachment to the cricoid cartilage and
ends in the mediastinum at the level of vertebra T5. At this point, it branches to form the right and left
primary bronchi.
Both the aorta (choice A) and the superior vena cava (choice D) are more posterior than the trachea
and are therefore less vulnerable.

34

The esophagus (choice B) is located behind the trachea and is less vulnerable.
The heart (choice C) lies below the clavicle and would not be at particular risk.
79. Which of the following cell types is derived from neuroepithelial cells?
A.
B.
C.
D.
E.

Astrocytes
Enterochromaffin cells
Melanocytes
Odontoblasts
Schwann cells

The correct answer is A. Astrocytes and oligodendrocytes are both derived from glioblasts, which, in
turn, are derived from neuroepithelial cells. Other neuroepithelial cell derivatives include neuroblasts
and ependymal cells. The astrocytes are the largest and most numerous glial cells. These cells are
responsible for maintaining the blood-brain barrier, creating a three-dimensional framework for the
central nervous system, performing repairs in damaged neural tissues, and controlling the interstitial
environment.
All the other choices are derived from neural crest cells. Other neural crest derivatives include the
neurons of the parasympathetic and sympathetic ganglia (including the adrenal medulla), the dorsal
root ganglia of the peripheral nervous system, the sensory ganglia of cranial nerves V, VII, IX, and X,
and the leptomeninges (pia and arachnoid).

80. Following a fracture of the humerus, which of the following is responsible for producing the
majority of the new bone that will reunite the two fragments?
A.
B.
C.
D.
E.

Cancellous bone
Cartilage
Compact bone
Marrow
Periosteum

The correct answer is E. When the periosteum is torn during a fracture, it supplies cells that develop
into osteoblasts and are the major producers of the new bone that reunites the two ends. Heterotopic
ossification (bone formed outside the regular bone) can occur as a complication of fracture if some of
the osteoblastic cells are misdirected into adjacent tissues. The periosteum assists in the attachment
of the osteoblasts to surrounding tissues and to associated tendons and ligaments. This cellular layer
functions in bone growth and participates in repair after an injury.
Pre-existing cancellous bone (choice A) and compact bone (choice C) are not the major source of
osteoblasts that form the new bone.
Cartilage (choice B) and marrow (choice D) do not contribute to new bone formation after fracture.

81. Which membrane is in the way is you try to reach the lesser peritoneal sac and head of the
pancreas after penetrating the greater peritoneal sac?
A. Falciform ligament

35

B.
C.
D.
E.

Gastrohepatic ligament
Gastrosplenic ligament
Hepatoduodenal ligament
Splenorenal ligament

The correct answer is B. The gastrohepatic ligament is the part of the lesser omentum that
separates the greater peritoneal sac from the right portion of the lesser peritoneal sac. This portion of
the lesser omentum has no significant blood vessels within it and may be incised for surgical access.
The falciform ligament (choice A) is a mesenteric membrane between the liver and the anterior
abdominal wall. This ligament is within the greater peritoneal sac and does not separate it from the
lesser peritoneal sac.
The gastrosplenic ligament (choice C) passes from the greater curvature of the stomach to the
spleen. It separates the greater peritoneal sac from the left portion of the lesser peritoneal sac.
Incision of this structure would be the appropriate surgical approach to gain access to the left side of
the lesser peritoneal sac.
The hepatoduodenal ligament (choice D) is part of the lesser omentum and separates the greater
peritoneal sac from the right portion of the lesser peritoneal sac. It forms the anterior border of the
epiploic foramen. However, the hepatoduodenal ligament contains the common bile duct, the proper
hepatic artery, and the portal vein, and therefore may not be incised for surgical access.
The splenorenal ligament (choice E) passes from the spleen to the parietal peritoneum on the
anterior surface of the left kidney. It separates the greater peritoneal sac from the left portion of the
lesser peritoneal sac. This ligament contains the splenic artery, splenic vein, and the tail of the
pancreas, and therefore may not be incised to gain access to the lesser peritoneal sac.

82. Which of the following fibers provide the only output from the cerebellar cortex?
A.
B.
C.
D.
E.

Climbing
Golgi cell
Granule cell
Mossy
Purkinje

The correct answer is E. Two basic things about cerebellar circuitry that are well-worth knowing are
that Purkinje cells of the cerebellar cortex project to the deep cerebellar nuclei and that these nuclei
project out of the cerebellum. Purkinje cells are located in the second (Purkinje) cell layer of the
cerebellar cortex and form inhibitory synapses on the deep cerebellar nuclei.
Climbing fibers (choice A) are afferents to the cerebellum. Specifically, they originate from the
medullary olivary nuclear complex, enter the cerebellum through the inferior cerebellar peduncle, and
project to the deep cerebellar nuclei and the lateral cerebellar hemispheres.
Golgi cell bodies (choice B) reside in the granule cell layer of the cerebellar cortex and project
predominantly to the granule cells, where they form inhibitory synapses. Therefore, they reside in and
project to the cerebellar cortex.
Granule cells (choice C) reside in the granule cell layer of the cerebellar cortex and send their axons
to the moleculary layer, where they bifurcate into parallel fibers. Granule cells, therefore, both reside
in and project to the cerebellar cortex.
Mossy fibers (choice D) consist of all of the afferents (except the climbing fibers) to the cerebellum.
These fibers project to deep cerebellar nuclei and glomeruli in the granular layer of the cerebellar
cortex.

36

83. A child who has had abnormal development of the membranous bones has a broad skull with
associated facial and dental anomalies. Which other bones are most likely to also be affected?
A.
B.
C.
D.
E.

Clavicles
Femurs
Metatarsals
Phalanges
Tibias

The correct answer is A. In a syndrome called cleidocranial dysostosis, absence of part of the
clavicles accompanies a broad skull and facial and dental anomalies. This syndrome affects bones
formed by intramembranous ossification.
The femurs (choice B), metatarsals (choice C), phalanges (choice D), and tibias (choice E) are
cartilaginous (formed by endochondral ossification) rather than membranous bones.

84. If cirrhosis causes obstruction of the portal circulation within the liver, portal blood could still be
conveyed to the caval system via which of the following?
A. Azygos and hemiazygos veins
B. Gonadal veins
C. Internal iliac veins
D. Splenic vein
E. Vesical venous plexus
The correct answer is A. The esophageal venous plexus, which drains into the azygos and
hemiazygos veins within the thorax, has anastomoses with branches of the left gastric vein. Thus,
following blockage of the portal vein, portal blood may enter the superior vena cava via the azygos
system. Other important portacaval connections include the superior rectal vein with the middle and
inferior rectal veins; paraumbilical veins and epigastric veins (engorgement of these vessels results in
caput medusae); and the colic and splenic veins with renal veins and veins of the posterior body wall.
The gonadal veins (choice B) exclusively drain the gonads (although in the female, the ovarian vein
communicates with the uterovaginal plexus). These vessels have no anastomoses with portal veins.
The internal iliac veins (choice C), which drain most of the pelvis and much of the inferior extremities,
have no demonstrated portal anastomoses. The splenic vein (choice D) is incorrect because it's in
itself a component of the portal venous system. The vesical venous plexus (choice E), which is
situated well within the pelvis and drains the bladder and the prostate (or uterus and vagina) has no
association with portal vessels.

85. Secretory basket cells are located


A.
B.
C.
D.

inside secretory acini


surrounding the intercalated duct
between secretory cells and the basal lamina
surrounding the striated duct

The correct answer is C. Secretory basket cells are found in salivary gland acini and are also called
myoepithelial cells. The name implies that they are epithelial cells with muscle-like contractile
properties. The are found between secretory cells and the basal lamina and exert pressure on the
secretory cells to release their secretions. Secretory acini themselves are composed of mucous or
serous secreting cells. Intercalated duct cells are nonspecialized cuboidal epithelial cells. Striated duct
cells are columnar epithelial cells with striations (stripes) of rows of mitochondria used for ATP
generation for active transport.

86. Which muscle is the chief mover of the mandible TOWARD the left?

37

A. Left medial pterygoid


B. Left lateral pterygoid
C. Right medial pterygoid
D. Right lateral pterygoid
The correct answer is D. There are two methods for answering this question. The formula method
states that the right lateral pterygoid moves the mandible left, whereas the left lateral pterygoid moves
the mandible right. The understanding method begins with the idea that the medial pterygoid is a
closer (elevator). The lateral pterygoid connects to the condyle and is a protruder. If the right lateral
pterygoid pulls the right condyle out while the left lateral pterygoid and condyle remain stable, the left
side will act as a stationary pivot point while the right side protrudes outward and also toward the left
(medially).

87.Which of the following cells is the germ cell closest to the basal lamina in the seminiferous tubule?
A.
B.
C.
D.
E.

Primary spermatocyte
Secondary spermatocyte
Spermatid
Spermatogonia
Spermatozoa

The correct answer is D. Maturation of germ cells (spermatogenesis) within the seminiferous tubules
occurs in a concentric pattern, with the less mature spermatogonia near the basal lamina and the
mature forms near the tubule center. Along this route the developing sperm are nurtured by sertoli
cells. Spermatogonia are 2N cells and mature into larger primary spermatocytes (4N) (choice A).
These mature into secondary spermatocytes (2N) (choice B), and finally into spermatids (1N)
(choice C). Spermatids undergo spermiogenesis to become mature spermatozoa (choice E).
Acrosomes form from the Golgi apparatus, and a flagellum forms from microtubules. Unneeded
organelles are shed. The seminiferous tubules of a reproductive-age male should exhibit all stages of
maturation, with mature flagellated sperm in their centers, no longer associated with sertoli cells.

88. If a person has normal musculature, but has difficulty swallowing, which nerves should be tested
for function?
A.
B.
C.
D.
E.

Hypoglossal and phrenic


Hypoglossal and splanchnic
Glossopharyngeal and vagus
Phrenic and vagus
Splanchnic and vagus

The correct answer is E. The upper 2/3 of the esophagus contains striated muscle. It is derived from
the pharyngeal arches and innervated by the vagus nerve (CN X). The lower 1/3 contains smooth
muscle from splanchnic mesoderm and is innervated by the splanchnic plexus. The vagus nerve has
mixed sensory and motor functions. The sensory fibers innervate the sensory nuclei and autonomic
centers of the medulla oblongata. The motor fibers innervate muscles of the palate, pharynx, and
respiratory and cardiovascular systems.
The hypoglossal nerve (choices A and B), or CN XII, moves the tongue.
The phrenic nerve (choices A and D), derived from C3, C4, and C5, innervates the muscle of the
diaphragm.
The glossopharyngeal nerve (choice C), or CN IX, functions in taste, swallowing, and salivation, as
well as monitoring the activity of the carotid body.

38

89. The left adrenal vein drains directly into which of the following veins?
A.
B.
C.
D.
E.

Hemiazygos vein
Inferior vena cava
Left renal vein
Splenic vein
Superior mesenteric vein

The correct answer is C. The left adrenal vein and the left gonadal vein (either testicular or ovarian)
drain into the left renal vein. The left renal vein then drains into the inferior vena cava. In contrast, the
right adrenal vein and right gonadal vein drain directly into the inferior vena cava.
The hemiazygos vein (choice A) receives the venous drainage from the body wall on the left side of
the thorax and abdomen. No visceral organs drain directly to the azygos or hemiazygos veins.
The inferior vena cava (choice B) receives the direct venous drainage from the right adrenal vein, but
not the left adrenal vein. Remember, the inferior vena cava is on the right side of the abdomen.
The splenic vein (choice D) receives the venous drainage from the spleen and part of the pancreas
and stomach. The splenic vein is part of the portal venous system.
The superior mesenteric vein (choice E) receives venous drainage from much of the intestinal tract. It
is part of the portal venous system and joins with the splenic vein to form the portal vein.
90. Which of the following is present in males but not in females?
A.
B.
C.
D.
E.

Bulbospongiosus muscle
Bulbourethral gland
Corpus cavernosum
Membranous urethra
Perineal body

The correct answer is B. The bulbourethral glands are paired structures located within the deep
perineal pouch, embedded within the sphincter urethrae. Their ducts pass to the spongy urethra. The
homologous female structures are the greater vestibular (Bartholin's) glands, which are located in the
superficial perineal pouch.
The bulbospongiosus muscles (choice A) lie superficial to the bulb of the penis in males and to the
bulbs of the vestibule in females.
The corpora cavernosa (choice C) are paired structures, consisting of cavernous erectile tissue that
form a large portion of the penile shaft in males and of the body of the clitoris in females.
The membranous urethra (choice D) is the portion of the urethra that passes through the urogenital
diaphragm in both males and females.
The perineal body (choice E) is the centrally located tendinous structure that provides attachment for
perineal musculature in both males and females. It separates the urogenital area from the anal area
and is an important obstetric landmark.

91. Luminal narrowing of which of the following vessels would compromise blood flow through the
renal arteries?
A. Abdominal aorta
B. Celiac trunk

39

C. Common iliac artery


D. Inferior mesenteric artery
E. Superior mesenteric artery
The correct answer is A. The renal arteries emerge from the abdominal aorta at about the level of
the L1/L2 intervertebral disk and travel at nearly right angles to it (on the right, passing posterior to the
inferior vena cava) to enter the hilum of the kidney.
The celiac trunk (choice B) gives off the common hepatic, splenic, and left gastric arteries.
The common iliac artery (choice C) gives off the internal and external iliac arteries. In addition, an
unascended pelvic kidney may be supplied by the common iliac artery.
The inferior mesenteric artery (choice D) gives off the superior rectal, sigmoid, and left colic arteries.
The superior mesenteric artery (choice E) gives off the inferior pancreaticoduodenal, intestinal (ileal
and jejunal), right colic, middle colic, and ileocolic arteries.

92. Which muscle has fibers that enter the articular disc and capsule?
A. Medial pterygoid
B. Buccinator
C. Temporalis
D. Lateral pterygoid
E. Masseter
The correct answer is D. Only the lateral pterygoid enters the temperomandibular joint (TMJ). Its
fibers join the articular disc and capsule. These fibers are from the superior head of the muscle.
Fibers from the inferior head enter the bony head of the mandibular condyle. The lateral pterygoid
protrudes the mandible, depresses (opens) it, and moves it left and right. None of the other muscles
listed enter the TMJ. Medial pterygoid and masseter primarily elevate (close) the mandible, whereas
temporalis elevates (anterior fibers) and retrudes (posterior fibers). Buccinator forms the substance of
the cheek and is also a muscle of facial expression. It is not a muscle of mastication.

93. While lying supine in bed eating, a child aspirates a peanut. Which of the following
bronchopulmonary segments would this foreign object most likely enter?
A.
B.
C.
D.
E.

Apical segment of the left upper lobe


Apical segment of the right upper lobe
Medial segment of the right middle lobe
Posterior basal segment of the left lower lobe
Superior segment of the right lower lobe

The correct answer is E. Because the right main bronchus is wider and more vertical than the left,
foreign objects are more likely to be aspirated into the right main bronchus. The superior segmental
bronchus of the lower lobar bronchus is the only segmental bronchus that exits from the posterior wall
of the lobar bronchi. Therefore, if a patient is supine at the time of aspiration, the object is most likely
to enter the superior segmental bronchus of the lower lobe.
None of the segmental bronchi of the left lung (choices A and D) are likely to receive the object
because the object is less likely to enter the left main bronchus.
The apical segment of the right upper lobe (choice B) is not likely to receive the foreign object
because of the sharp angle that the upper lobar bronchus makes with the right main bronchus and the
sharp angle that the apical segmental bronchus makes with the lobar bronchus.

40

The medial segmental bronchus of the right middle lobe (choice C) arises from the anterior wall of the
right middle lobar bronchus. Therefore, when the patient is supine, the effect of gravity will tend to
prevent the object from entering this segmental bronchus.

94. A physician notes weakness of a patient's thumb in extension, although rotation, flexion,
abduction, adduction, and opposition are normal. Which of the following nerves is most likely
involved?
A.
B.
C.
D.
E.

Median and radial


Median and ulnar
Median only
Radial only
Ulnar only

The correct answer is D. All three of the nerves listed innervate muscles that supply the thumb.
Extension is provided by the extensors pollicis longus and brevis, which are innervated by the radial
nerve.
The median nerve (choices A, B, and C) supplies the thenar group, which allows the thumb to
oppose, flex, abduct, and rotate.
The ulnar nerve (choices E and B) supplies the adductor pollicis, which adducts the thumb.

95. From which of the following arteries does the sphenopalatine artery arise?
A.
B.
C.
D.
E.

External carotid
Facial
Maxillary
Ophthalmic
Transverse facial

The correct answer is C. The sphenopalatine artery is the terminal branch of the maxillary artery.
The maxillary artery arises from the external carotid artery and then passes through the infratemporal
fossa, giving off branches to structures in this region. The artery then passes through the
pterygomaxillary fissure to enter the pterygopalatine fossa. The terminal branch then passes through
the sphenopalatine foramen to enter the nasal cavity and supply much of the nasal mucosa,
particularly in the posterior region of the nasal cavity.
The external carotid artery (choice A) arises from the common carotid artery at the carotid bifurcation.
The external carotid artery has eight branches that supply the head and neck region: the superior
thyroid, ascending pharyngeal, occipital, lingual, facial, posterior auricular, maxillary, and superficial
temporal arteries.
The facial artery (choice B) arises from the external carotid artery. It provides blood supply to much of
the facial region. The nasal cavity (particularly the anterior portion) receives some of its blood supply
from branches of the facial artery.

41

The ophthalmic artery (choice D) arises from the internal carotid artery immediately after the internal
carotid artery emerges from the cavernous sinus. The ophthalmic artery passes through the optic
canal to supply structures in the orbit.
The transverse facial artery (choice E) is a branch of the superficial temporal artery. It supplies blood
to structures in the upper portion of the lateral face.
96. Which of the following helps to anchor an epithelial cell to the basement membrane?
A.
B.
C.
D.
E.

Adherent junction
Connexon
Gap junction
Hemidesmosome
Tight junction

The correct answer is D. Desmosomes are specializations of the lateral surface of the cell formed
from the juxtaposition of two disc-shaped structures in adjacent cells, acting much like spot welds
between the two cells. Hemidesmosomes are basically desmosomes between a single cell and an
extracellular matrix structure, such as a basement membrane.
Adherent junctions (choice A), or zonula adherens are bandlike junctions that help attach adjacent
epithelial cells to each other.
Gap junctions consist of a hexagonal lattice of tubular proteins called connexons (choice B), which
form channels allowing communication between cells.
Tight junctions (choice E), or zonula occludens, are formed by the fusion of the outer leaflets of
apposed cell membranes on the lateral cell surfaces, just beneath the apical poles. They form a
barrier to permeability, or a seal around the cell.

97. A patient is involved in an accident which tears the left lateral pterygoid muscle completely. On
attempting to open, the patients mandible will move:
A. Left
B. Right
C. In an elevating direction
D. In a direct protruding direction
The correct answer is A.There are two ways to answer this question. The formulaic method says
that damage to a lateral pterygoid or the innervation to it will make the mandible deviate toward the
side of the damage. A deeper understanding of the question would show that the lateral pterygoid is
the primary protruder of the mandible. Contraction of both lateral pterygoids produces straight
protrusion. If the left lateral pterygoid is torn, the right muscle will begin to protrude the right side, with
the left side remaining stable, and acting as a stationary pivot point. As a result, the mandible will pivot
out and toward the left (the side of injury). Note that damage to muscles or innervation to one side of a
tongue will also produce deviation toward the injured side.

42

98. Which of the tongue papillae are NOT vascular?


A. Fungiform
B. Filiform
C. Vallate
D. Foliate

2H

The correct answer is B. The four major lingual papillae types are listed in the answer. Fungiform
papillae are fairly numerous, vascular, have taste buds, and are found primarily on the anterior
tongue. Filiform are avasacular, without taste buds, are the most numerous, and are found in rows.
Vallate are largest, usually 7-9 in number, contain taste buds, and serous salivary glands of Von
Ebner. Foliate are on the lateral surface of the tongue in ridges but are usually considered
rudimentary and nonfunctional in man.

99. Mucopurulent exudate from maxillary sinusitis would be most likely to drain through an ostium in
the
A. bulla ethmoidalis
B. hiatus semilunaris
C. inferior nasal meatus
D. sphenoethmoidal recess
E. superior nasal meatus
The correct answer is B. This patient has two risk factors for sinusitis: chronic rhinitis and allergy.
She probably also has aspirin allergy, which is associated with the triad of nasal polyps, asthma, and
sinusitis. In maxillary sinusitis, exudate may drain into the middle meatus through an ostium in the
hiatus semilunaris, which contains openings to the frontal and maxillary sinuses and anterior
ethmoidal cells.The bulla ethmoidalis (choice A), also part of the middle meatus, contains an opening
to the middle ethmoidal air cells.
The inferior nasal meatus (choice C) receives fluid from the
nasolacrimal duct, which drains tears from the medial aspect of the orbit to the nasal cavity.
The
sphenoethmoidal recess (choice D) is located above the superior concha and contains an opening for
the sphenoid sinus. The superior nasal meatus (choice E) is located above the superior concha and
contains an opening for the sphenoid sinus.

100. A wound to the posterior left axillary line, between the ninth and tenth rib, and extending
approximately 5 cm deep, will most likely damage which organ?
A. Ascending colon
B. Duodenum
C. Left kidney
D. Left lobe of the liver
E. Spleen
The correct answer is E. The spleen follows the long axes of ribs 9 to 11 and lies mostly posterior to
the stomach, above the colon, and partly anterior to the kidney. It is attached to the stomach by a
broad mesenterial band, the gastrosplemic ligament. Therefore, it is the most likely organ of the group
to be pierced by a sharp object penetrating just above rib 10 at the posterior axillary line. Note that the
pleural cavity, and possibly the lower part of the inferior lobe of the lung, would be pierced before the
spleen. The ascending colon (choice A) is on the wrong side (the right) to be penetrated by a sharp
instrument piercing the left side. Most of the duodenum (choice B) is positioned too far to the right to
be affected by this injury. Even the third part of the duodenum, which runs from right to left, would still
be out of harm's way. In addition, the duodenum lies at about levels L1 to L3, placing it too low to be

43

injured in this case. The superior pole of the left kidney (choice C) is bordered by the lower part of the
spleen. However, it is crossed by rib 12 and usually does not extend above rib 11. It would probably
be too low and medial to be injured in this case because this penetration is at the posterior axillary
line. The left lobe of the liver (choice D) is positioned just beneath the diaphragm, just over and
anterior to the stomach. The anterior positioning of this structure makes it an unlikely candidate for
injury in this case. Even with deep penetration at the correct angle, it would not be penetrated before
the spleen.

44

101. Which salivary gland cells have folded cell membranes at their base that are filled with large
numbers of mitochondria?
A.
B.
C.
D.
E.

Intercalated duct cells


Striated duct cells
Serous secretory cells
Mucus secretory cells
Myoepithelial (basket) cells

The correct answer is B. Striated duct cells line sections of salivary ducts, which modify the
composition of saliva. In particular, they may add potassium, remove sodium, or add bicarbonate.
This ionic movement, mostly against a concentration gradient, requires energy in the form of ATP.
This ATP is made in mitochondria. Rows of mitochondria in these cells, located near the base, appear
in light microscopy to be stripes or striations. Mucous and serous secretory cells, because of their
protein secreting function, may have highly developed ribosomes, rough endoplasmic reticulum, and
golgi complexes. Intercalated duct cells have no unique specializations of note. Myoepithelial cells
have contractile ability, somewhat like muscle cells.

102. Poor blood supply to which of the following tissues is most often problematic during the healing
process after surgery?

A. Adipose tissue
B. Skin
C. Loose connective tissue
D. Muscle

The correct answer is A. Adipose tissue is loose connective tissue dominated by adipocytes.
Surgeons worry about their obese patients more than their skinny ones because a thick layer of
relatively poorly vascularized, subcutaneous fatty tissue is both mechanically unstable (it holds
stitches poorly) and heals very slowly. These patients have a frequent rate of dehiscence (tearing
open of the incision site) with subsequent, difficult-to-control infection (access by antibiotics,
leukocytes, and serum antibodies are all hampered by the poor blood supply).
Loose connective tissue (choice C) is well vascularized and surgeons do not usually worry much
about it during the healing process.
Muscle (choice D) usually heals well after surgery.
Skin (choice E) usually heals well, unless it becomes infected.

103. Radiographic studies confirm that an embryo has split at the blastocyst stage. Splitting of the
embryo at the blastocyst stage results in which of the following?

A. Conjoined twins
B. Dizygotic twins
C. Fraternal twins

51

D. Monozygotic twins
E. Single gestation
The correct answer is D. Monozygotic twins, or identical twins, develop from a single fertilized egg
that subsequently splits during either the blastomere or blastocyst stage. It is more common in the
blastocyst stage at the end of the first week. This results in two inner cell masses in the same
blastocyst cavity. They usually develop a common placenta and chorionic cavities but separate
amniotic cavities. Splitting in the second week usually results in shared amniotic cavities as well.
Conjoined twins (choice A), or Siamese twins, result from incomplete splitting of the embryo.
Dizygotic twins (choice B) and fraternal twins (choice C) are the same, and are the most common
type of twins. They share the same genetic relatedness as do siblings of separate pregnancies. This
type of twinning occurs because of simultaneous double ovulation followed by fertilization by two
sperm. They each develop their own placenta and membranes.
A single gestation (choice E), and a single birth, is the result of a single fertilization without any
splitting of the embryo.

104. A nerve running around the neck of the humerus is damaged. The nerve damage causes muscle
denervation, and the inability to raise the arm over the head. The denervated muscle arises from
which of the following?

A. Acromion alone
B. Clavicle and acromion
C. Clavicle and coracoid
D. Coracoid alone
E. Coracoid and acromion
The correct answer is B. The nerve is the axillary branch of the posterior cord (also called the
axillary nerve), which can be damaged by either fracture or dislocations of the humeral neck. The
muscle supplied by this nerve is the deltoid muscle. The deltoid arises from the lateral portions of the
clavicle and the acromion and inserts into the deltoid tuberosity of the humerus.
The deltoid arises from both the acromion (choice A) and the clavicle.
The deltoid does not arise from the coracoid process (choices C, D, and E).

105. If the tendon of the biceps brachii is severed at the elbow, and the cut causing this injury
extends 2 cm medially, which of the following structures will be injured?
A. Brachial artery
B. Musculocutaneous nerve
C. Profunda brachii artery

52

D. Radial nerve
E. Ulnar nerve
The correct answer is A. The brachial artery is immediately medial to the tendon of the biceps
brachii at the elbow. As the artery enters the forearm, it is covered by the bicipital aponeurosis, a
broadening of the biceps tendon.
The musculocutaneous nerve (choice B) does not cross the elbow. The musculocutaneous nerve
gives off all of its muscular branches to muscles in the arm. The remainder of the nerve is then
renamed the lateral cutaneous nerve of the forearm, which passes the elbow lateral to the tendon of
the biceps.
The
profunda brachii artery (choice C) arises from the brachial artery in the proximal part of the arm. It
accompanies the radial artery in the musculospiral groove and then divides into the radial collateral
artery and middle collateral artery, which cross the elbow lateral to the tendon of the biceps.
The
radial nerve (choice D) lies within the musculospiral groove along the back of the humerus, then
passes between the brachioradialis muscle and the brachialis muscle at the elbow, lateral to the
tendon of the biceps.
The ulnar nerve (choice E) crosses the elbow posterior to the medial
epicondyle of the humerus. It then passes between the two heads of the flexor carpi ulnaris and
courses through the forearm deep to this muscle.
106. A patient with damage to the humerus and weakness in lateral rotation and abduction of the
arm has probably damaged which nerve?
A. axillary
B. dorsal scapular
C. radial
D. suprascapular
E. thoracodorsal
The correct answer is A. Because of the proximity of the axillary nerve to the glenohumeral joint, a
fracture of the surgical neck of the humerus or an inferior dislocation of the humerous could damage
the nerve. The axillary nerve innervates the deltoid muscle. The deltoid abducts, adducts, flexes,
extends, and rotates the arm medially. The axillary nerve also innervates the teres minor, which
rotates the arm laterally.
The dorsal scapular nerve (choice B) innervates both the major and minor rhomboid muscles. These
muscles raise the medial border of the scapula upward and laterally.
The radial nerve
(choice C) innervates muscles involved in the movement of the forearm and hand.
The suprascapular nerve (choice D) innervates the supraspinatus and infraspinatus. The
supraspinatus abducts the arm, whereas the infraspinatus rotates the arm laterally. This nerve
travels along the posterior aspect of the scapula and would not easily be subjected to injury in a
dislocation of the shoulder joint.
The thoracodorsal nerve (choice E) innervates the latissimus
dorsi, which adducts, extends, and rotates the arm medially.
107. Weakness when a patient attempts internal rotation of the right arm at the shoulder could be
caused by weakness in which of the following muscles?
A. Infraspinatus
B. Pectoralis minor

53

C. Subscapularis
D. Supraspinatus
E. Teres minor
The correct answer is C. The subscapularis muscle arises from the anterior surface of the scapula
and inserts onto the lesser tubercle of the humerus. It is one of the rotator cuff muscles. Its tendon
passes on the anterior side of the shoulder joint capsule, where it reinforces the capsule. Contraction
of this muscle causes internal rotation of the arm at the shoulder.
The infraspinatus muscle (choice A) arises from the posterior surface of the scapula in the
infraspinous fossa and inserts on the greater tubercle of the humerus. It is one of the rotator cuff
muscles. Its tendon passes along the posterior surface of the shoulder joint capsule, where it
reinforces the capsule. Contraction of the infraspinatus causes external rotation of the arm at the
shoulder.
The pectoralis minor muscle (choice B) arises from the chest wall and inserts onto the coracoid
process of the scapula. The pectoralis minor does not attach to the humerus and therefore does not
cause movement of the humerus at the shoulder.
The supraspinatus muscle (choice D) arises from the posterior surface of the scapula in the
supraspinous fossa and inserts onto the greater tubercle. It is one of the rotator cuff muscles. Its
tendon passes along the superior surface of the shoulder joint capsule, where it reinforces the
capsule. Contraction of the supraspinatus causes abduction of the arm at the shoulder.
The teres minor muscle (choice E) arises from the axillary border of the scapula and inserts onto the
greater tubercle of the humerus. It is one of the rotator cuff muscles. Its tendon passes along the
posterior surface of the shoulder joint capsule, where it reinforces the capsule. Contraction of the
teres minor causes external rotation of the arm at the shoulder.
108. A patient has suffered a penetrating wound in the left fourth intercostal space immediately
lateral to the sternal border. Which of the following thoracic structures is most likely to have been
injured?
A. Left atrium
B. Left ventricle
C. Right atrium
D. Right ventricle
E. Upper lobe of the left lung
The correct answer is D. The right ventricle forms most of the anterior wall of the heart and extends
from approximately the right border of the sternum to approximately 2 inches to the left of the
sternum at the level of the fourth intercostal space. The left atrium (choice A) forms the posterior
wall of the heart. The only portion of the left atrium seen on the anterior surface of the heart is the left
auricular appendage, which is at the level of the second intercostal space on the left.
The left ventricle (choice B) forms most of the left border of the heart and the diaphragmatic surface
of the heart. It forms the anterior wall of the heart in a region from approximately 2 to 3 inches from
the left border of the sternum from the third to the fifth intercostal space.
The right atrium (choice C) forms the right border of the heart. Its anterior surface is on the right side

54

of the sternum from approximately the third to the sixth rib.


The left lung (choice E) is displaced away from the sternum on the left side by the presence of the
heart.
109. The heart of an embryo first begins beating at which of the following ages?
A. 2 weeks
B. 3 weeks
C. 4 weeks
D. 6 weeks
E. 8 weeks
The correct answer is C. While the third-week embryo is a primitive trilaminar plate, in the fourth
week, the heart begins to form and begins beating almost immediately. Hematopoiesis occurs in the
yolk sac, and a primitive circulatory system connects the capillary plexuses of the yolk sac and
chorion to the embryo. Partitioning of the atrium also begins in the fourth week. During the fifth week,
cardiac septa form and the atrioventricular (AV) cushions fuse. By the sixth week, the heart is close
to fully formed. This early sequence for the heart explains why it is so difficult to try to prevent
congenital malformations of the cardiovascular system from occurring because the mother of a 6week-old fetus is only about 8 weeks from her last menstrual period and may have assumed that she
just "missed a period" (a very common phenomenon) for reasons other than pregnancy.
110. Which of the following renal structures is most medially located?
A. Major calyx
B. Minor calyx
C. Renal cortex
D. Renal pelvis
E. Renal pyramid
The correct answer is D. This is a relatively simple question that requires you to visualize the
relationship among the key parts of the kidney and to identify the one that lies most medially.
Because the kidneys ultimately drain into the ureter at their medial poles, you are looking for the
structure that is closest to the ureter. The correct answer is the renal pelvis. The renal pelvis is the
dilated upper portion of the ureter that receives the major calyces.
In terms of the other answer choices, the order from most lateral to most medial is as follows: renal
cortex (choice C), renal pyramid (choice E), minor calyx (choice B), major calyx (choice A), and
then renal pelvis (choice D).
111. Brunner's glands secrete an alkaline product that helps achieve optimal pH for the activity of
pancreatic enzymes. Where are these glands located?
A. At the base of villi throughout the small intestine

55

B. In the epithelium of the ampulla of Vater


C. In the mucosa and submucosa of the jejunum
D. In the submucosa of the duodenum
E. In the submucosa of the ileum
The correct answer is D. Brunner's glands are located in the submucosa of the duodenum. These
glands are connected to the intestinal lumen by ducts that open into certain crypts. They secrete an
alkaline product that protects the duodenal mucosa from the acidic chyme and helps achieve optimal
pH for pancreatic enzymes.
Note that if you did not recall the location of Brunner's glands, the
question's description of their function allowed you to deduce it on the basis of your knowledge of the
anatomy of the small intestine. You should have immediately ruled out choices C and E because
they are too far from the pancreas. Choices B and D would therefore remain as the best possible
answers because of their proximity to the pancreas. If you remembered the structure and function of
the ampulla of Vater, you were left with the correct answer. The small intestinal villi (choice A) are
outgrowths of the mucosa into the lumen. Their epithelium contains columnar absorptive cells and
goblet cells (which produce acid glycoproteins that protect and lubricate the lining of the intestine).
Near the base of each villus are tubular glands called crypts, whose lining is continuous with the
simple columnar epithelium of the villus. The crypts include Paneth cells, which produce acidophilic
cytoplasmic granules containing bactericidal enzymes.
The ampulla of Vater (choice B) receives
bile from the common bile duct and the main pancreatic duct, delivering it to the duodenum through
the major duodenal papilla.
The mucosa and submucosa of the jejunem (choice C) are both
included in the permanent folds called the plica circulares. The submucosa of the ileum (choice E)
is the home of Peyer's patches, which are large aggregates of lymphoid nodules.
112. Intramuscular injections should be given in the upper, outer quadrant of the buttocks to prevent
damage to which of the following nerves?
A. Common peroneal
B. Lateral femoral cutaneous
C. Obturator
D. Sciatic
E. Superior gluteal
The correct answer is D. Injections are given in the upper, outer quadrant of the buttocks to prevent
damage to the sciatic nerve, which is present in the lower quadrant. The other nerves listed are not
particularly vulnerable to injections into the buttocks. The common peroneal nerve (choice A) is a
branch of the sciatic nerve that diverges from it in the popliteal fossa. It then divides into the
superficial and deep peroneal nerves. The lateral femoral cutaneous nerve (choice B derives from
the lumbar plexus, emerges slightly below the anterior superior iliac spine, and supplies the skin of the
anterior thigh down to the knee.
The obturator nerve (choice C) derives from the lumbar plexus, diverges from the femoral nerve in
the psoas muscle, and passes medially along the lateral pelvic wall to run in the obturator canal,
where it divides into anterior and posterior divisions. The anterior division generally supplies the
gracilis, adductor brevis, and adductor longus; the posterior division generally supplies the obturator
externus and the adductor part of the adductor magnus. The superior gluteal nerve (choice E) is a
branch of the sacral plexus. It supplies the gluteus minimus and medius and the tensor fascia lata.
Small branches of this nerve are likely to be encountered in the upper outer quadrant of the buttock,
but injection here is relatively safe.

56

113. If a patient is unable to touch the tip of his tongue to the roof of his mouth, dysfunction of which
of the following muscles is most likely the cause?
A. Buccinator
B. Geniohyoid
C. Palatoglossus
D. Palatopharyngeus
E. Tensor palati
The correct answer is C. Elevation of the tongue is carried out by the styloglossus (innervated by the
hypoglossal nerve, CN XII) and the palatoglossus (innervated by the pharyngeal plexus). The
palatoglossus originates on the anterior surface of the soft palate and inserts in the side of the tongue.
The buccinator (choice A) functions in storing, filling, and emptying the vestibule.
The geniohyoid (choice B) moves the hyoid anteriorly to open the pharynx.
The palatopharyngeus (choice D) produces a "stripping wave" on the posterior pharyngeal wall.
The tensor palati (choice E) tenses the soft palate.
114. An inflammatory process in the temporal bone has resulted in a swelling of the facial nerve
within the facial canal. Which muscle may be paralyzed as a result of this compression?
A. Anterior belly of the digastric
B. Geniohyoid
C. Stapedius
D. Stylopharyngeus
E. Masseter
The correct answer is C. The stapedius muscle is innervated by the facial nerve (cN VII). This
muscle is located in the middle ear and attaches to the neck of the stapes. Contraction of the
stapedius reduces the amplitude of oscillation of the stapes and thus reduces the perceived loudness
of a sound. Paralysis of this muscle may result in hyperacusis. Note that the seventh nerve also
innervates all muscles of facial expression. The anterior belly of the digastric muscle (choice A) is a
muscle in the floor of the mouth that is innervated by the mandibular division of the trigeminal nerve.
The geniohyoid muscle (choice B) is one of the suprahyoid muscles in the neck. This muscle is
innervated by C1 spinal nerve fibers that travel for a short distance with the hypoglossal nerve. The
stylopharyngeus muscle (choice D) is one of the longitudinal muscles of the pharynx that acts to
elevate the pharynx.
The masseter muscle (choice E) is one of the muscles of mastication. All of
the muscles of mastication are innervated by the mandibular division of the trigeminal nerve.
115. Tissue served by branches of which of the following arteries would be most likely affected by
pressure on the splenic artery?

57

A. Left gastric
B. Left gastroepiploic
C. Right gastric
D. Right gastroepiploic
E. Short gastric
The correct answer is E. The splenic artery passes behind the stomach and gives off the short
gastric artery and the left gastroepiploic artery immediately after passing the greater curvature. The
left gastroepiploic artery has a strong anastomotic connection to another arterial supply, while the
short gastric does not, so the area supplied by branches of the short gastric arteries is more
vulnerable to ischemia in this setting. If the block had occurred proximal to, instead of at, the branch
point, the short gastric vessels could be supplied by backflow from the left gastroepiploic artery.
The left gastric artery (choice A) is not supplied by the splenic artery.
The left gastroepiploic artery (choice B) can be alternatively supplied by its anastomotic connection
to the right gastroepiploic artery.
The right gastric artery (choice C) is not supplied by the splenic artery.
The right gastroepiploic artery (choice D) is normally supplied by the gastroduodenal artery.
116. An angiographic study of the distal part of the circle of Willis requires access to the vertebral
artery. Access to this artery can be obtained via the
A. anterior triangle of the neck
B. muscular triangle of the neck
C. posterior triangle of the neck
D. submental triangle
E. suboccipital triangle
The correct answer is E. The suboccipital triangle lies in the neck at the base of the skull. It is
bounded by the inferior oblique, rectus major, and superior oblique muscles. The vertebral artery can
be found within the suboccipital triangle, lying on the posterior arch of the atlas, lateral to the midline,
typically at about the level of the lower portion of the ear lobe. Care should be taken in accessing the
vessel because the first cervical nerve lies between the vertebral artery and the posterior arch of the
atlas. The circle of Willis contains the following arteries: anterior communicating, anterior cerebral,
posterior communicating, and posterior cerebral.
The anterior triangle of the neck (choice A) is in the anterior neck and is bounded by the neck
midline, the mandible, and the sternocleidomastoid muscle. The anterior triangle of the neck is
subdivided into the muscular, carotid, submandibular, and submental triangles.
The small muscular triangle (choice B) lies between the superior belly of the omohyoid and the

58

sternohyoid muscle.
The posterior triangle of the neck (choice C) is on the side of the neck and is bounded by the
sternocleidomastoid muscle, the trapezius, and the middle third of the clavicle.
The submental triangle (choice D), also called the suprahyoid triangle, is found below the lower jaw
and is bounded by the hyoid bone and anterior bellies of the digastric muscles.
117. The large cephalic vein can best be found between which of the following muscles?
A. Biceps and triceps
B. Brachialis and medial head of the triceps
C. Coracobrachialis and brachialis
D. Deltoid and pectoralis major
E. Sternocleidomastoid and deltoid
The correct answer is D. The cephalic vein arises from the elbow along the anterior lateral surface
of the arm. Specifically, the superficial arch empties into the cephalic vein, which ascends along the
radial side of the forearm. It can be reliably found in the deltopectoral groove between the deltoid and
pectoralis major. This groove lies diagonally (arising medially) below the clavicle. The cephalic vein is
good for cannulation because its location is particularly constant.
The cephalic vein is unrelated to the triceps, although it does pass in the arm over the biceps
(choice A).
The cephalic vein is not related to either the triceps or the brachialis (choice B).
The cephalic vein is not related to either the coracobrachialis or the brachialis (choice C).
The cephalic vein is not related to the sternocleidomastoid (choice E).
118. A patient who has been exhibiting various endocrine abnormalities has an MRI scan of the
head. This scan reveals a small tumor of the pituitary gland. If this tumor expands laterally, which of
the following nerves will most likely be affected first?
A. Abducens nerve
B. Oculomotor nerve
C. Optic nerve
D. Trigeminal nerve
E. Trochlear nerve
The correct answer is A. The pituitary gland is located in the pituitary fossa within the skull. The
floor of this fossa is formed by the sella turcica. The lateral walls of the fossa are formed by the
cavernous sinuses. The abducens nerve passes through the cavernous sinus along with the internal
carotid artery. As the tumor expands laterally, the first nerve that will be encountered is the abducens
nerve, producing a lateral rectus palsy. The abducens nerve or cranial nerve VI originates from the
pons and enters the lateral rectus muscle. It is responsible for aiding in eye movements.
The oculomotor nerve (choice B) lies in the lateral wall of the cavernous sinus. It is farther from the
pituitary gland than is the abducens nerve. The optic nerve (choice C) is anterosuperior to the
pituitary gland. Upward expansion of the tumor may compress the optic chiasm. The trigeminal nerve

59

(choice D) is found posterior to the cavernous sinus. Two of its three divisions (ophthalmic and
maxillary divisions) pass through the lateral wall of the cavernous sinus and are farther from the
pituitary gland than is the abducens nerve. The trochlear nerve (choice E) is also in the lateral wall
of the cavernous sinus and would be affected later if the tumor continued to expand.
119. An infant is noted to have an adducted and internally rotated shoulder with an extended,
pronated elbow. Loss of flexion and supination by which of the following muscles accounts for the
elbow position?
A. Biceps
B. Brachialis
C. Coracobrachialis
D. Deltoid
E. Pronator teres
The correct answer is A. Loss of abductors and external rotators can cause an adducted and
internally rotated shoulder. The biceps accomplish both of these motions. Loss of flexion and
supination by the biceps muscle causes the extended, pronated elbow ("porter's tip" or "waiter's tip"
position). The brachialis muscle (choice B) is a pure flexor of the elbow. The coracobrachialis
muscle (choice C) functions at the elbow by helping to hold an object under the arm. The deltoid
muscle (choice D) is a powerful abductor of the shoulder. The pronator teres muscle (choice E) is a
powerful pronator of the elbow.
120. Most afferents to the cerebral cortex derive from the
A. basal ganglia
B. cerebral cortex
C. hippocampus
D. spinal cord
E. thalamus
The correct answer is B. Most afferents to the cerebral cortex derive from the cerebral cortex. This
type of extensive interconnection of cortical areas facilitates communication and integration between
cortical areas on the same side or on different sides of the brain.
The basal ganglia (choice A) receive their most prominent input from the cortex but output to the
globus pallidus and the substantia nigra pars reticulata, rather than directly to the cerebral cortex.
The hippocampus (choice C) sends most of its projections to the mammillary bodies and the
thalamus via the fornix, rather than to the cerebral cortex. The ascending tracts of the spinal cord
(choice D) carry sensory information to brainstem nuclei, thalamus, and cerebellum, rather than
directly to the cerebral cortex. The thalamus (choice E) projects to the cerebral cortex, but the cortex
receives quantitatively more fibers from other areas of cortex than from the thalamus.

121. A nerve is located close to the wrist portion of the tendons of the flexor carpi radialis and

60

palmaris longus. This nerve arises from which part(s) of the brachial plexus?
A. Lateral and medial cords
B. Lateral cord
C. Middle and lower trunks
D. Posterior cord
E. Upper and middle trunks
The correct answer is A. The nerve in question is the median nerve, which lies between the
palmaris longus and flexor carpi radialis tendons on the anterior aspect of the forearm. The median
nerve is formed from both the lateral and medial cords of the brachial plexus. The brachial plexus
innervates the shoulder girdle and upper limb with contributions from the ventral rami of spinal
nerves C5 through T1. The nerves that form this plexus originate from trunks and cords named
accordingly.

122. Which of the following best describes the direction of blood flow through a patent ductus
arteriosus in an infant?
A. From aorta to left pulmonary artery
B. From aorta to left pulmonary vein
C. From aorta to right pulmonary artery
D. From left pulmonary artery to aorta
E. From right pulmonary artery to aorta
The correct answer is A. The ductus arteriosus connects the left pulmonary artery to the aortic
arch. It is derived from the left sixth aortic arch. During prenatal life, the pressure gradient causes
blood to flow from the left pulmonary artery to the aorta. This allows fetal blood to bypass the
pulmonary system. However, after birth, the pressure gradient reverses, and if the ductus arteriosus
remains patent, the flow is from the aorta to the left pulmonary artery.
The ductus arteriosus does not connect to the pulmonary veins or the right pulmonary artery
(choices B, C, and E).
The flow through the ductus arteriosus is from the left pulmonary artery to the aorta (choice D) prior
to birth, it but reverses after birth.

61

123. Which of the following structures lie in the free edge of the lesser omentum?
A. Common bile duct, cystic duct, and hepatic artery
B. Cystic duct, hepatic artery, and hepatic vein
C. Hepatic vein and cystic duct
D. Portal vein, common bile duct, and hepatic artery
E. Portal vein, hepatic artery, and hepatic vein
The correct answer is D. The free edge of the lesser omentum contains three important structures:
the common bile duct, the hepatic artery, and the portal vein.
Neither the cystic duct (choices A, B, and C) nor the hepatic vein (choices B, C, and E) lies in the
free edge of the lesser omentum.

124. The layers of the abdominal wall are shown below.


1. Internal oblique 2. External oblique 3. Peritoneum 4. Transversus abdominis
Which of the following corresponds to the order of the layers of the abdominal wall from superficial to
deep?
A. 1-3-4-2
B. 2-1-3-4
C. 2-1-4-3
D. 4-1-2-3
E. 4-2-1-3
The correct answer is C. Questions like these are particularly amenable to the use of test-taking
strategies because even if you don't know 100% of the answer, you will be able to eliminate some
answer choices on the basis of knowledge you do have. For example, you probably are aware that
the peritoneum does not lie above the transversus abdominis muscle, enabling you to eliminate
choice A and increase your chances of answering correctly by 20%. Likewise, the external oblique
must be external to the internal oblique, which eliminates choices A and D.

62

125. The notochord forms on approximately what day after conception?


A. Day 2
B. Day 7
C. Day 17
D. Day 28
E. Day 60
The correct answer is C. Formation of the notochord is a third week event, occurring as a relatively
early part of development of the trilaminar disc. More specifically, notochord formation begins about
day 16, and the neuroenteric canal forms on about day 18. Two days after conception (choice A),
implantation has not yet occurred. By seven days after conception (choice B), the blastocyst has
usually implanted in the endometrium. By day 28 (choice D), gastrulation has occurred, and
organogenesis is under way. By day 60 (choice E), organogenesis is complete.
126. Which of the following cell types is derived from neural crest cells?
A. Astrocytes
B. Ependymal cells
C. Microglia
D. Oligodendroglia
E. Pseudounipolar cells
The correct answer is E. Pseudounipolar cells of spinal and cranial nerve ganglia derive from the
neural crest. Other cell types that derive from the neural crest include Schwann cells, multipolar
ganglion cells of autonomic ganglia, chromaffin cells of the adrenal medulla, odontoblasts,
melanocytes, and cells that make up the leptomeninges (pia and arachnoid). All of the other cell
types listed derive from the neural tube. Physiologically, in a unipolar or pseudounipolar neuron, the
dendritic and axonal processes are continuous. The cell body lies off to one side. Sensory neurons of
the peripheral nervous system are usually unipolar, and their axons may be myelinated.

127. Injury to the lower division of the facial nerve during parotid surgery will result in
A. inability to furrow the brow (to frown) on the same side
B. numbness over the angle and mental region of the jaw on the same side
C. ptosis of the eye on the same side
D. weakness in closing the eye on the same side
E. weakness of the lower lip on the same side

63

The correct answer is E. The motor component (special visceral efferent) of the facial nerve (CN
VII) exits the skull via the stylomastoid foramen, passes lateral to the styloid process, and then enters
the parotid gland. Within the gland, two divisions can usually be identified (upper and lower), which in
turn give off five named branches that innervate the muscles of the face. The upper division gives
rise to the temporal and zygomatic branches, which collectively innervate the frontalis, corrugator,
and orbicularis oculi muscles. The lower division gives off the buccal, mandibular, and cervical
branches. The largest, the buccal, innervates the muscles attaching to the upper lip, including the
orbicularis oris and the levators, as well as the buccinator and the muscles of the nose. The
mandibular branches innervate the muscles of the lower lip and of the chin, whereas the cervical
branch innervates the platysma muscle. There are usually communicating branches between the
named terminal nerves so that overlapping innervation of the muscles occurs. If the lower division is
injured, there will be weakness (not frank paralysis because of the innervation overlap) of the
muscles that attach to the lower lip.
An inability to furrow the brow (choice A) would be caused by denervation of the corrugator supercilii
and frontalis muscles, which are innervated by the upper division of the facial nerve.
Choice B is wrong because once the facial nerve emerges from the stylomastoid foramen, it is a
pure motor nerve (special visceral efferent, or branchiomotor nerve). It carries no sensory nerve
fibers.
Ptosis (a drooping of the upper eyelid; choice C) is the result of a paralysis of the levator palpebrae
muscle, which is innervated by the oculomotor (CN III) nerve.
Choice D is not correct because the orbicularis oculi muscle is innervated by branches from the
upper division of the facial nerve.
128. A man cannot abduct and adduct the digits of his hand, or oppose his thumb. This occurred
after trying to break a fall. Which neural structure has been injured?
A. Lower trunk of the brachial plexus
B. Median nerve
C. Musculocutaneous nerve
D. Upper trunk of the brachial plexus
E. Ulnar nerve
The correct answer is A. The lower trunk of the brachial plexus contains nerve fibers from the
eighth cervical and first thoracic spinal nerves. These nerve fibers innervate the intrinsic muscles of
the hand, including the interosseous muscles, responsible for abduction and adduction of the digits,
and the opponens muscle, responsible for opposition of the thumb. The lower trunk ascends from the
lower neck and upper thorax to reach the axilla. Upward traction on the upper limb may stretch the
lower trunk and injure these nerve fibers. The median nerve (choice B) innervates many muscles of
the anterior compartment of the forearm and some muscles in the hand, including the opponens
muscle. The median nerve, however, does not innervate the interosseous muscles, responsible for
abduction and adduction of the digits. The musculocutaneous nerve (choice C) innervates the
muscle of the anterior compartment of the arm. It does not innervate any muscles in the hand. The
upper trunk of the brachial plexus (choice D) contains nerve fibers from the fifth and sixth cervical
spinal nerves. The nerve fibers innervate muscles in the proximal part of the upper limb, including
muscles around the shoulder and axilla. No muscles in the hand are innervated by these nerve
fibers. The ulnar nerve (choice E) innervates many muscles in the hand, including the interosseous
muscles, which are responsible for abduction and adduction of the digits. However, it does not
innervate the opponens muscle, which is responsible for opposition of the thumb.

64

129. Incomplete fusion of the embryonic endocardial cushions can produce which of the following
congenital defects?
A. Atrioventricular septal defect
B. Coarctation of the aorta
C. Pulmonary stenosis
D. Tetralogy of Fallot
E. Transposition of the great vessels
The correct answer is A. The endocardial cushions form the fibrous tissue "skeleton" of the heart,
which lies between the chambers of the heart. A failure of fusion of the cushions can cause an
atrioventricular defect, which, in extreme cases, can make the heart behave as if it were a single
chamber. Surgical correction of a large defect may be difficult and may not yield an effective heart
because other malformations, such as small ventricular size, may coexist.
The causes of coarctation of the aorta (choice B) are unclear but are not thought to be related to
incomplete fusion of the endocardial cushions.
Pulmonary stenosis (choice C) can be caused by fusion of the cusps of the semilunar valve or by
unequal division of the truncus arteriosus.
Tetralogy of Fallot (choice D) is caused by failure of the pulmonary trunk and aorta to line up with the
ventricular openings.
Transposition of the great vessels (choice E) results from abnormal migration of neural crest cells,
leading to failure of spiral development of the aorticopulmonary septum.
130. Which cranial nerve carries pain sensation from the posterior third of the tongue?
A. Trigeminal
B. Facial
C. Hypoglossal
D. Glossopharyngeal
E. Vagus
The correct answer is D. The sensory innervation of the tongue is as follows: the anterior two thirds
of the tongue is innervated by CN V (trigeminal) for touch sensation, and CN VII (facial) through the
chorda tympani for taste sensation. CN IX (glossopharyngeal) innervates the posterior one third for
touch and taste. Some texts state that the most posterior areas of the tongue may have innervation
from CN X (vagus) also. The dividing line for the two thirds and one third is the row of circumvallate
papillae. These papillae themselves are innervated by CN IX (glossopharyngeal).

65

131. Which of the following hormones is secreted by anterior pituitary cells that stain with acidic dyes?
A. ACTH
B. FSH
C. LH
D. Prolactin
E. TSH
The correct answer is D. The cells of the anterior pituitary can be classified as chromophils (stain
with dyes) or chromophobes (do not stain with dyes). The chromophils can be further divided in
acidophils (stain with acidic dyes) and basophils (stain with basic dyes). The acidophils include the
somatotrophs, which secrete growth hormone, and the mammotrophs, which secrete prolactin.
The basophils include the corticotrophs, which secrete ACTH (choice A), the gonadotrophs, which
secrete FSH and LH (choices B and C), and the thyrotrophs, which secrete TSH (choice E).
132. Atherosclerotic occlusion of which of the following arteries would result in insufficient perfusion
of the urinary bladder?
A. External iliac
B. Inferior epigastric
C. Internal iliac
D. Internal pudendal
E. Lateral sacral
The correct answer is C. The bladder is supplied by the vesicular branches of the internal iliac
arteries. The internal iliacs arise from the common iliac artery.
The external iliac (choice A) also arises from the common iliac artery. It makes no contribution to the
blood supply of the bladder.
The inferior epigastric (choice B) is a branch of the external iliac artery. It serves as a landmark in
the inguinal region.
The internal pudendal (choice D) is a branch of the anterior division of the internal iliac artery. It
supplies the rectal area and the organs.
The lateral sacral (choice E) is a branch of the posterior division of the internal iliac artery.It supplies
sacral structures.

66

133. In which regions of the thorax is the thoracic duct found?


A. Anterior and middle mediastinum
B. Anterior and superior mediastinum
C. Middle and posterior mediastinum
D. Middle and superior mediastinum
E. Posterior and superior mediastinum
The correct answer is E. The mediastinum is divided into four regions. The region above the
manubriosternal junction (level of fourth thoracic vertebra) is the superior mediastinum. The region
below the manubriosternal junction is divided into the anterior mediastinum (anterior to the
pericardium), the middle mediastinum (within the pericardium), and the posterior mediastinum
(posterior to the pericardium). The thoracic duct enters the thorax through the aortic hiatus of the
diaphragm. At this point it lies in the posterior mediastinum, the region posterior to the pericardium.
As it ascends through the thorax and passes the level of the fourth thoracic vertebra, it enters the
superior mediastinum.
The anterior mediastinum (choices A and B) contains the thymus and fatty connective tissue.
The middle mediastinum (choices A, C, and D) contains the heart and the roots of the great vessels.
In addition to the thoracic duct, the superior mediastinum (choices B, D, and E) contains the
ascending aorta, aortic arch, branches of the aortic arch, descending aorta, superior vena cava,
brachiocephalic veins, thymus, trachea, and esophagus. In addition to the thoracic duct, the
posterior mediastinum (choices C and E) contains the descending aorta, azygos vein, hemiazygos
vein, and the esophagus.
134. Which structure passes near the uterine artery, and might be confused with it during surgery?
A. Internal iliac artery
B. Internal iliac vein
C. Ovarian artery
D. Ureter
E. Uterine vein
The correct answer is D. The ureter passes directly inferior to the uterine artery, lateral to the body
of the uterus near its junction with the cervix ("water flows under the bridge"). During a hysterectomy,
therefore, the ureter (instead of the uterine artery) may be inadvertently ligated.
The internal iliac artery (choice A) gives rise to the uterine artery, the primary blood supply of the
uterus.
The internal iliac vein (choice B) receives blood from the uterine vein (choice E), the primary venous
drainage of the uterus.
The ovarian artery (choice C) arises from the abdominal aorta and is the primary blood supply of the
ovaries.

67

135. A muscle arises from the medial and lateral processes of the tuberosity of the calcaneus and
inserts into the lateral side of the base of the proximal phalanx of the fifth toe. How does this muscle
act on the fifth toe?
A. Abduction
B. Adduction
C. Extension
D. Flexion of the middle phalanx on the proximal phalanx
E. Flexion of the proximal phalanx on the metatarsal
The correct answer is A. Abduction is movement away from the longitudinal axis of the body. The
muscle is the abductor digiti minimi, the function of which is to abduct (separate) the toe away from
the fourth toe. The abductor digiti minimi is supplied by the lateral plantar nerve. This muscle also
helps act as a "spring" on the sole of the foot during walking. The key to this question is the insertion
on the lateral side of the toe. Contraction on this side pulls the toe away from midline.
Adduction (opposition of the toes, choice B) is supplied by the plantar interossei. Extension (choice
C) is supplied by the extensor digitorum longus. Flexion is movement in the anterioral posterior plane
that reduces the angle between the articulating joints. Extension occurs in the same plane but
increases the angle between articulating elements. The flexor digitorum brevis flexes both the middle
phalanx on the proximal phalanx (choice D) and the proximal phalanx on the metatarsal (choice E).
136. Which of the following nuclei is the most important source of noradrenergic innervation to the
cerebral cortex?

A. Basal nucleus of Meynert


B. Caudate nucleus
C. Locus coeruleus
D. Raphe nucleus
E. Substantia nigra
The correct answer is C. The locus coeruleus is a dense collection of neuromelanin-containing cells
in the rostral pons, near the lateral edge of the floor of the fourth ventricle. The fact that it appears
blue-black in unstained brain tissue gave rise to its name, which means "blue spot" in Latin. These
cells, which contain norepinephrine, provide the majority of noradrenergic innervation to the
forebrain, including the cerebral cortex.The basal nucleus of Meynert (choice A), a part of the
substantia innominata, is a major collection of forebrain cholinergic neurons. These neurons
(together with neurons in septal nuclei) innervate the neocortex, hippocampal formation, and the
amygdala. The basal nucleus is one of the structures that degenerates in Alzheimer's disease.The
caudate nucleus (choice B) is part of the basal ganglia, located immediately lateral to the lateral
ventricles. There are at least two important cell types in the caudate. GABAergic projection neurons
(the majority) innervate the globus pallidus and substantia nigra pars reticulata. The GABAergic
neurons degenerate in Huntington's disease, leading to enlarged lateral ventricles that are clearly
visible on MRI. The caudate also contains cholinergic interneurons, which provide most of the
acetylcholine to the striatum (caudate and putamen). The balance of striatal acetylcholine and
dopamine is important for the treatment of patients with extrapyramidal symptoms, sucha as
Parkinson's disease or parkinsonism accompanying therapy with antipsychotic medications.

68

The raphe nuclei (choice D) are located in the midline at most levels of the brainstem. They contain
seratonergic cell bodies that innervate virtually every part of the central nervous system. The
substantia nigra (choice E) is located in the midbrain and consists of the substantia nigra pars
compacta and the substantia nigra pars reticulata. The substantia nigra pars compacta contains the
nigrostriatal neurons that are the source of striatal dopamine. This cell degenerates in Parkinson's
disease or in response to neurotoxic agents such as MPTP. The substantia nigra pars reticulata
consists predominately of GABAergic neurons that innervate the thalamus.

137. Which of the following muscles allow a patient to continue to flex the elbow after rupture of the
biceps tendon?
A. Brachialis and brachioradialis
B. Flexor carpi ulnaris and flexor carpi radialis
C. Flexor digitorum superficialis and flexor digitorum profundus
D. Pronator teres and supinator
E. Triceps and coracobrachialis
The correct answer is A. The long head of the biceps tendon is vulnerable to abrasion as it moves
within the bicipital grove between the greater and lesser tuberosities of the humerus. The tendon
occasionally snaps, particularly in the elderly. Fortunately, the brachialis and brachioradialis are
sufficiently strong flexors of the elbow in which function is retained even if no specific repair of the
tendon is undertaken.
The flexor carpi ulnaris and the flexor carpi radialis (choice B) are flexors at the wrist, not elbow.
The flexor digitorum superficialis and the flexor digitorum profundus (choice C) are flexors of the wrist
and fingers.
The pronator teres and the supinator (choice D) pronate and supinate the forearm, respectively.
The triceps and the coracobrachialis (choice E) extend the elbow and flex the shoulder, respectively.
138. A middle-aged patient is diagnosed with a condition that causes excruciating pain near her nose
and mouth when touched. The involved nerve is derived from which of the following branchial arches?
A. First
B. Second
C. Third
D. Fourth
The correct answer is A. The clinical history suggests trigeminal neuralgia, which is characterized by
extreme pain along the distributions of the maxillary and mandibular subdivisions of the fifth cranial
nerve, associated with a "trigger zone." The trigeminal nerve is derived from the first branchial arch.
The second branchial arch (choice B) gives rise to the muscles of facial expression and is innervated
by the facial nerve, cranial nerve VII. The third branchial arch (choice C) is innervated by the ninth
cranial nerve, the glossopharyngeal, which innervates the stylopharyngeus muscle.
The fourth branchial arch (choice D) gives rise to most pharyngeal constrictor muscles and is
innervated by the tenth cranial nerve, the vagus nerve.

69

139. The primitive streak normally gives rise to which of the following structures?
A. Dorsal root ganglia
B. Lining of the gastrointestinal tract
C. Notochord
D. Spinal cord
E. Thyroid gland
The correct answer is C. The primitive streak is the region of the epiblast through which the cells
that give rise to the notochord and the mesoderm of the embryo pass. The only adult derivative of the
notochord is the nucleus pulposus of the intervertebral disk. The mesoderm gives rise to many
different tissue types, including muscle, connective tissue, and blood.
The dorsal root ganglia (choice A) are derived from neural crest cells. The neural crest develops at
the time of neurulation, which is a process of infolding of the neural ectoderm that is induced by the
notochord.
The lining of the gastrointestinal tract (choice B) is derived from endoderm. The endoderm also gives
rise to evaginations of the gastrointestinal tract, such as the liver and pancreas. The smooth muscle
and connective tissue of the gastrointestinal tract are derived from mesoderm.
The spinal cord (choice D) is derived from neural ectoderm, which invaginates during neurulation to
form the neural tube. The caudal part of the neural tube forms the spinal cord, and the rostral part of
the neural tube forms the brain.
The thyroid gland (choice E) is derived from endoderm. The endoderm of the floor of the pharynx
evaginates to form the thyroglossal duct, which descends to form the thyroid gland. The adult site of
the evagination of the thyroglossal duct is marked by the foramen cecum on the tongue.
140. A deep venous thrombosis embolizes, and the embolus lodges at the bifurcation of the
pulmonary trunk, almost completely occluding the vessel. Which part of the heart would be most
significantly and immediately affected by this event?
A. Left main coronary artery
B. Left ventricle
C. Right atrium
D. Right main coronary artery
E. Right ventricle
The correct answer is E. A massive pulmonary embolus can interrupt pulmonary blood flow,
producing acute cor pulmonale with abruptly developing right ventricular dilatation because the right
ventricle pumps blood directly into the area of blockage, the pulmonary artery. Other parts of the heart
are secondarily affected somewhat later. Acute cor pulmonale is a surgical emergency, requiring
immediate correction of the underlying problem, which is usually a pulmonary embolus lodged early in
the pulmonary circulation. Acute cor pulmonale is less common than chronic cor pulmonale, which is
seen as a complication of many chronic lung diseases.
The left main coronary artery (choice A) and right main coronary artery (choice D) would be affected
secondarily to the reduced blood flow to the left heart and aorta, from which the coronary arteries
arise. The left ventricle (choice B) would be affected secondarily by reduced blood flow from the
pulmonary veins to the left atrium. The right atrium (choice C) would be affected after the right
ventricle because it is farther from the circulatory block.

70

141. Another term used to describe compact bone is:


A. Spongy bone
B. Intramembranous bone
C. Bundle bone
D. Woven bone
E. Lamellar bone
The correct answer is E. There are many overlapping and sometimes confusing terms for bone
types. Compact bone is dense and contains Haversian systems. It is also known as lamellar (layered)
bone. Spongy bone is also known as trabecular bone and does not contain Haversian systems.
Compact bone is normally found on the outside surface of bones, whereas spongy bone is found on
the inside. Bundle bone refers to compact bone containing collagen attachment fibers, such as those
found in the alveolar bone of the periodontal ligament. Intramembranous bone refers to one type of
bone formation, bone formed from a primitive connective tissue model (not collagen). Woven bone is
early intramembranous bone. The other type of bone is formed from ossification of a cartilaginous
model.

142. An aneurysm of the axillary artery within the axilla is most likely to compress which of the
following neural structures?
A. Axillary nerve
B. Long thoracic nerve
C. Lower trunk of the brachial plexus
D. Medial cord of the brachial plexus
E. Musculocutaneous nerve
The correct answer is D. Within the axilla, the axillary artery is within the axillary sheath and is
surrounded by the three cords of the brachial plexus, which are also within the axillary sheath. An
aneurysm of the axillary artery may compress any of the three cords. The brachial plexus innervates
the shoulder girdle and upper limb with contributions from the ventral rami of spinal nerves C5-T1.
The nerves that form this plexus originate from trunks and cords named according to their anatomical
location.
The axillary nerve (choice A) is a branch of the posterior cord that leaves the axillary sheath, then
exits the axilla through the quadrangular space to innervate the deltoid muscle.
The long thoracic nerve (choice B) is not within the axillary sheath. It arises from the anterior rami of
the fifth, sixth, and seventh cervical nerves in the neck and courses along the chest wall to innervate
the serratus anterior muscle.
The lower trunk of the brachial plexus (choice C) is not in the axilla. It is formed in the neck from the
anterior rami of the eighth cervical and first thoracic spinal nerves.
The musculocutaneous nerve (choice E) is not within the axillary sheath. It arises as a branch of the
lateral cord of the brachial plexus and enters the arm to innervate the muscles of the anterior
compartment of the arm.

71

143. Which of the following structures constitutes part of the ventricles of the heart?
A. Auricle
B. Crista terminalis
C. Fossa ovalis
D. Sinus venarum
E. Trabeculae carneae
The correct answer is E. The trabeculae carneae are ridges of myocardium in the ventricular wall.
Remember that the Latin root "carne-" means "flesh" and that the ventricles are "fleshier" than the
atria.
The auricle (choice A) is derived from the fetal atrium; it has rough myocardium known as musculi
pectinati, or pectinate muscle.
The crista terminalis (choice B) is the vertical ridge that separates the smooth portion of the right
atrium from the rough portion; it extends longitudinally from the superior vena cava to the inferior vena
cava.
The fossa ovalis (choice C) is the remnant of the fetal foramen ovale, an opening in the interatrial
septum that allows blood entering the right atrium from the inferior vena cava to pass directly to the
left side of the heart.
The sinus venarum (choice D) is the smooth-walled portion of the atrium that receives blood from the
superior and inferior vena-cava. It is derived from the fetal sinus venosus.
44. An esophageal biopsy shows normal smooth muscle and striated muscle in the same section.
Which portion of the esophagus was the source of this biopsy?
A. Lower esophageal sphincter
B. Lower third of the esophagus
C. Middle third of the esophagus
D. Upper esophageal sphincter
E. Upper third of the esophagus
The correct answer is C. The middle third of the esophagus contains both striated and smooth
muscle.
The lower third (choice B) of the esophagus, including the lower esophageal sphincter (choice A)
contains only smooth muscle.
Some authors identify the cricopharyngeus muscle as an upper esophageal sphincter (choice D),
despite the fact that unlike a true sphincter, this muscle does not completely encircle the esophagus.
The cricopharyngeus is composed exclusively of skeletal muscle; therefore, the biopsy could not have
been from this area of the esophagus.
The muscularis of the upper third of the esophagus (choice E) is composed entirely of striated
muscle.

72

45. If a clavicle fractures, which muscle might prevent the fractured bone from damaging the
subclavian vessels or large nerves of the arm?
A. Deltoid
B. Pectoralis major
C. Sternocleidomastoid
D. Subclavius
E. Trapezius
The correct answer is D. The clavicle is the most frequently broken long bone of the body. However,
despite what one might expect, the underlying subclavian vessels and parts of the brachial plexus are
only rarely injured. This is because the subclavius muscle, which occupies a small groove on the
undersurface of the clavicle, apparently protects the underlying structures. Other functions of this
small muscle are not well understood.
The deltoid (choice A) originates from the lateral aspect of the clavicle and is a powerful abductor of
the arm.
The pectoralis major (choice B) originates from the medial aspect of the clavicle and is not as
important as the subclavius with regard to protecting underlying structures.
The sternocleidomastoid (choice C), an important muscle of the anterior neck, attaches to the medial
aspect of the posterosuperior border of the clavicle.
The trapezius (choice E), an important muscle of the posterior neck, attaches to the lateral aspect of
the posterosuperior border of the clavicle.

46. A person skiing for the first time goes too fast and grabs hold of a tree with one arm to stop
himself. In doing so, he is most likely to injure which of the following?
A. Lower part of the brachial plexus
B. Median nerve
C. Radial nerve
D. Ulnar nerve
E. Upper part of the brachial plexus

The correct answer is A. Grabbing at something to stop oneself, as in the above example, or when a
skidding motorcyclist tries to catch a lamp pole, or delivering the arms in an extended position during
a breech delivery, are the more common mechanisms by which the lower part of the brachial plexus is
damaged. The result of this type of injury is the loss of motor function of the intrinsic hand muscles
and numbness along the inner aspect of the hand (ulnar side). The brachial plexus innervates the
shoulder girdle and upper limb and divides into the median, radial, and ulner nerves and their
branches. There are contributions from the ventral rami and spinal nerves C5-7 into these structures.
Damage to the median nerve (choice B) can happen in carpal tunnel syndrome or when a person
attempts to cut their wrists.
Damage to the radial nerve (choice C) can occur with humeral fractures or when one (often under the
influence of alcohol) falls asleep with his or her arm over the back of a chair. Damage to the ulnar
nerve (choice D) can occur when the elbow is struck ("funny bone"). Damage to the upper part of the
brachial plexus (choice E) can occur when someone falls on the point of the shoulder.

73

47. Stimulation and pressure on the ear canal can sometimees result in fainting. This is because the
posterior half of the external ear canal receives innervation from the
A. auricular branch of the vagus nerve
B. auriculotemporal nerve
C. greater auricular nerve
D. lesser occipital nerve
E. vestibulocochlear nerve
The correct answer is A. The vagus nerve, through its auricular branch, supplies the posterior half of
the external auditory meatus. Because the vagus also supplies many other organs, reflex symptoms
may occur, including fainting (typically in the elderly), coughing, and gagging. The vagus nerve is
sensory from the pharynx, pinna and external auditory canal, diaphragm, and visceral organs in the
thoracic and abdominopelvic cavities. It is a motor nerve to the palatal and pharyngeal muscles and
visceral organs in thoracic and abdominal cavities.
The auriculotemporal nerve (choice B) supplies the anterior half of the external auditory meatus and
the facial surface of the upper part of the auricle.
The greater auricular nerve (choice C) supplies both surfaces of the lower part of the auricle.
The lesser occipital nerve (choice D) supplies the cranial surface of the upper part of the auricle.
The vestibulocochlear nerve (choice E) supplies hearing and motion sense.

48. Which of the following placental components is derived from the mother rather than the fetus?
A. Chorion
B. Cytotrophoblast
C. Lacunar network
D. Primary villus
E. Syncytiotrophoblast
The correct answer is C. The placenta is derived from both maternal and fetal tissues. The mother
produces the capillary bed that forms the lacunar network of the placenta.
The fetus produces both the cytotrophoblast (choice B) and the syncytiotrophoblast (choice E),
which together form the chorion (choice A). The earliest projections of this are called the primary villi
(choice D).

74

49. While testing the gag reflex during a cranial nerve examination, it is noted that when the right side
of the pharyngeal mucosa is touched, the patient's uvula deviates to the right. When the left side of
the pharyngeal mucosa is touched, the patient does not gag. Which of the following is the most likely
location of his lesion?
A. Left glossopharyngeal nerve and left vagus nerve
B. Left glossopharyngeal nerve only
C. Left vagus nerve only
D. Right glossopharyngeal nerve and right vagus nerve
E. Right glossopharyngeal nerve only

The correct answer is A. The gag reflex requires the glossopharyngeal nerve for the sensory limb of
the reflex (unilateral) and the vagus nerve for the motor limb of the reflex (bilateral). A lesion of the left
glossopharyngeal nerve will denervate the sensory receptors on the left side of the pharynx, so when
the left side is touched, the patient does not feel it and does not gag. If the left vagus nerve is
lesioned, the left side of the soft palate will not elevate during a gag, and the uvula will deviate to the
right. In this case, the patient only feels the touch on the right side (left glossopharyngeal lesion) and
only elevates the right side of the palate (left vagus lesion).
If the patient had a lesion of the left glossopharyngeal nerve only (choice B), there would have been
no gag reflex when the left side was touched, but there would be a normal gag, without deviation of
the uvula, when the right side was touched.
If the patient had a lesion of the left vagus nerve only (choice C), the patient would have deviation of
the uvula to the right when a gag was elicited, but touching either side of the pharynx would elicit a
gag.
If the patient had a lesion of the right glossopharyngeal nerve and the right vagus nerve (choice D),
touching the right side of the pharynx would not elicit a gag, and touching the left side of the pharynx
would elicit a gag with the uvula deviating to the left. If the patient had a lesion of the right
glossopharyngeal nerve only (choice E), there would be no gag when the right side was touched, but
there would be a normal gag, without deviation of the uvula, when the left side was touched.
50. When removing an impacted mandibular third molar, the oral surgeon must warn the patient of
possible lasting numbness of the tip of the tongue. This loss of general sensation is due to damage to
the
A. auriculotemporal nerve
B. chorda tympani
C. lingual nerve
D. mental nerve
E. mylohyoid nerve
The correct answer is C. The lingual nerve is a branch of the mandibular division of the trigeminal
nerve that conveys general sensation from the anterior two thirds of the tongue. It enters the oral
cavity by passing just under the mandibular third molar between the medial pterygoid muscle and the
mandibular ramus. The auriculotemporal nerve (choice A) is a branch of V3 that passes from the
infratemporal fossa to the parotid region. It contains sensory fibers from the region in front of the ear
and the temporomandibular joint and also conveys postganglionic parasympathetic fibers to the
parotid salivary gland. The chorda tympani (choice B) is a branch of CN VII that travels with the
lingual nerve in the floor of the mouth. It carries taste fibers from the anterior two thirds of the tongue
and preganglionic parasympathetic fibers that synapse in the submandibular ganglion. The mental
nerve (choice D) is a sensory branch of the inferior alveolar nerve that supplies the skin of the chin
and lower lip. The mylohyoid nerve (choice E) is a motor branch of the inferior alveolar nerve that
supplies the mylohyoid and anterior belly of the digastric muscles.

75

51. A pathologist looks at a histologic section of an aorta stained to demonstrate the many wavy
fibers that provide the aorta with the flexibility to stretch and return to its original shape. These fibers
are also found in substantial numbers in the
A. biceps muscle
B. liver
C. mesentery
D. spleen
E. vocal cords
The correct answer is E. The fibers are elastic fibers. You should remember three specific sites
where these are found: large arteries (particularly the aorta), vocal cords (or cords), and ligamenta
flava (which connect the vertebrae). Small amounts of elastic tissue are also found in skin and to a
much lesser degree in other tissues. The vocal folds (or cords) protect the entrance to the glottis.
They are located inferior to the ventricular folds. The vocal folds are highly elastic because they
contain bands of elastic tissue called the vocal ligaments. The vocal folds are involved in the
production of sounds.
Elastic tissue is not found in muscle (choice A) or mesentery (choice C).
Elastic tissue is not found in substantial amounts in the liver (choice B) or spleen (choice D).
52. A thyroid mass usually moves with swallowing because the thyroid gland is enclosed by which of
the following fascia?
A. Carotid sheath
B. Investing layer of the deep cervical fascia
C. Pretracheal fascia
D. Prevertebral fascia
E. Superficial fascia
The correct answer is C. The pretracheal layer of the cervical fascia runs from the investing layers in
both sides of the lateral neck and splits to enclose the thyroid gland. Superiorly, it attaches to the
laryngeal cartilages; inferiorly, it fuses with the pericardium. As a result of these connections, the
thyroid gland moves with laryngeal movements. Anatomically, the thyroid gland curves across the
anterior surface of the trachea just below the thyroid cartilage that forms most of the anterior surface
of the larynx. The two lobes of the thyroid gland are united by a slender connection, the isthmus.
The carotid sheath (choice A) contains the vagus nerve, internal jugular vein, carotid artery, and
lymph nodes.
The investing layer of the deep cervical fascia (choice B) splits to enclose the trapezius and
sternocleidomastoid muscles.
The prevertebral fascia (choice D) covers muscles arising from the vertebrae.
The superficial fascia (choice E) lies immediately deep to the platysma muscle.

76

53. Within which of the following anatomic spaces is the meningeal artery normally found?
A. Anterior cranial fossa
B. Inferior orbital fissure
C. Middle cranial fossa
D. Posterior cranial fossa
E. Superior orbital fissure
The correct answer is C. The middle meningeal artery is a branch of the maxillary artery. It enters
the intracranial cavity through the foramen spinosum, which is located in the floor of the middle cranial
fossa. A tear of the middle meningeal artery results in accumulation of blood between the inner table
of the skull and the dura mater. This is called an epidural hematoma.
The anterior cranial fossa (choice A) receives the central processes of the bipolar neurons that form
the olfactory nerve. This enters the intracranial cavity via the cribriform plate of the ethmoid bone.
The inferior orbital fissure (choice B) contains veins that communicate with venous plexuses in the
pterygopalatine and infratemporal fossae.
The posterior cranial fossa (choice D) contains the brainstem and cerebellum. The vertebral arteries
enter the posterior cranial fossa through the foramen magnum, where they unite to form the basilar
artery.
The superior orbital fissure (choice E) contains the ophthalmic vein and all of the cranial nerves that
innervate the eye (CN III, IV, VI), except for the optic nerve (CN II).
54. A 5-year-old child who has not had routine pediatric care develops a febrile disease with cough
and a blotchy rash, and is brought to the emergency department. On physical examination, there is
cervical and axillary lymphadenopathy. Also noted is an erythematous, macropapular rash behind the
ears and along the hairline, involving the neck and, to a lesser extent, the trunk. Examination of this
patient's oropharynx would likely reveal which of the following lesions?
A.
B.
C.
D.
E.

Adherent thin, whitish patch on gingiva


Cold sores on the lips
Curdy white material overlying an erythematous base on the oral mucosa
Large shallow ulcers on the oral mucosa
Multiple small white spots on the buccal mucosa

The correct answer is E. The disease described is measles (rubeola), which has the typical
presentation described in the question stem. Measles is caused by a Morbillivirus, an RNA virus
belonging to the Paramyxovirus family. Koplik spots, which are pathognomonic for measles, are small,
bluish-white spots on the buccal mucosa in the early stages of measles. These lesions appear just
before the onset of the characteristic rash (which can also involve extremities) and fade as the rash
develops.
Leukoplakia is a premalignant condition characterized by adherent whitish patches on the gingiva
(choice A) and other sites in the oral cavity.
Cold sores on the lips (choice B) are caused by infection with Herpes virus.
Candida infection (thrush) produces curdy white material loosely attached to an erythematous base
(choice C). Aphthous ulcers are large shallow ulcers of the oral mucosa (choice D), commonly
known as canker sores.

77

55. Ligation of which of the following arteries would be most likely to produce the most significant
ischemic damage to the cerebrum?
A. Anterior cerebral artery at its origin from the internal carotid artery
B. Anterior communicating artery
C. Middle cerebral artery at its origin from the internal carotid artery
D. Posterior cerebral artery at its origin from the basilar artery
E. Posterior communicating artery
The correct answer is C. The circle of Willis is a series of arterial anastomoses that circle around the
pituitary stalk at the ventral surface of the brain. Specifically, the circle of Willis encircles the
infundibulum of the pituitary gland. Hence, the brain can receive blood from either the carotids or the
vertebrals. If any of the vessels of the circle of Willis become blocked, blood can be routed through
the remaining vessels. The object of this question is to determine which vessel does not belong to the
circle of Willis. Of the options listed, only the middle cerebral artery does not belong to the circle and
instead courses laterally to supply the entire lateral convexity of the brain. Therefore, ligation of this
vessel would produce the most ischemic damage.
56. The endothelial lining of the aorta is composed of which of the following tissue types?
A. Pseudostratified epithelium
B. Simple columnar epithelium
C. Simple cuboidal epithelium
D. Simple squamous epithelium
E. Stratified columnar epithelium
The correct answer is D. Endothelium lines the cardiovascular and lymphatic vessels and is
composed of simple squamous epithelium. The mesothelium that lines the pleural, pericardial, and
peritoneal cavities is also composed of a single-layer of simple squamous epithelium.
Pseudostratified epithelium (choice A) is found in the epithelial lining of the respiratory tract, as well
as in the transitional epithelium of the urinary system.
In simple columnar epithelium (choice B), the cells are taller than they are wide. This epithelium can
be found in the intestinal absorptive surface.
Simple cuboidal epithelium (choice C) can be found in the ducts of many glands.
Stratified columnar epithelium (choice E) can be found in portions of the male urethra.

57. The parotid duct pierces the buccinator muscle nearest to the maxillary
A. First molar
B. Second molar
C. Third molar
D. First premolar
E. Second premolar

78

The correct answer is B. The parotid gland lies lateral to the buccinator muscle. The facial nerve and
its branches pass through it. It produces a serous-only salivary secretion, which is second in volume
to that produced by the submandibular. The parotid is the largest salivary gland in size. Its duct,
known as Stensen's duct, pierces the buccinator in the vicinity of the maxillary second molar.
58. The greater omentum is derived from which of the following embryonic structures?
A. Dorsal mesoduodenum
B. Dorsal mesogastrium
C. Pericardioperitoneal canal
D. Pleuropericardial membranes
E. Ventral mesentery
The correct answer is B. Both the omental bursa and the greater omentum are derived from the
dorsal mesogastrium, which is the mesentery of the stomach region. The dorsal mesentery of the
stomach becomes greatly enlarged and forms a large pouch that extends inferiorly between the body
wall and the anterior surface of the small intestine. This is the greater omentum.
The dorsal mesoduodenum (choice A) is the mesentery of the developing duodenum, which later
disappears so that the duodenum and pancreas come to lie retroperitoneally.
The pericardioperitoneal canal (choice C) embryologically connects the thoracic and peritoneal
canals.
The pleuropericardial membranes (choice D) become the pericardium and contribute to the
diaphragm.
The ventral mesentery (choice E) forms the falciform ligament, ligamentum teres, and lesser
omentum.
59. Damage to the cervical sympathetic trunk in a patient is most likely to be seen in which of the
following findings?
A. Constriction of the right pupil
B. Dilation of the right pupil
C. Inability to abduct the right eye
D. Inability to close the right eye
E. Paralysis of the platysma muscle on the right side

The correct answer is A. The sympathetic trunk lies posterior to the internal jugular vein.
Preganglionic sympathetic nerve fibers can be damaged in a cervical trunk injury. These nerve fibers
synapse in the superior cervical ganglion on postganglionic sympathetic neurons that innervate
structures in the head. The dilator pupillae muscle (smooth muscle of the iris that dilates the pupil) is
sympathetically innervated; paralysis of this muscle due to interruption of its innervation results in
constriction of the pupil (miosis).
Dilation of the pupil (choice B) would result from paralysis of the sphincter pupillae muscle (smooth
muscle of the iris that constricts the pupil). This muscle is parasympathetically innervated and would
not be affected by this injury.

79

An inability to abduct the eye (choice C) would result from paralysis of the lateral rectus muscle, an
extraocular muscle that is innervated by the abducens nerve. It would not be affected by this injury.
An inability to close the eye (choice D) would result from paralysis of the orbicularis oculi muscle, a
skeletal muscle of the face. This muscle is innervated by the facial nerve and would not be affected by
this injury.
The platysma muscle (choice E) is a skeletal muscle in the superficial fascia of the neck. It is
innervated by the facial nerve and would not be affected by this injury.

60. Which pair of muscles is innervated by Cranial Nerve V?


A. Masseter, buccinator
B. Mylohyoid, geniohyoid
C. Lateral pterygoid, medial pterygoid
D. Anterior digastric, posterior digastric
E. Levator palatini, tensor palatini

The correct answer is choice CIn brief, Cranial Nerve V, the trigeminal, innervates the muscles of
mastication, the two tensors, the anterior digastric and the mylohyoid. The muscles of mastication are
the medial and lateral pterygoid, the temporalis, and the masseter. The two tensors are the tensor
tympani and the tensor (veli) palatini. Note that among the incorrect answers, buccinator, as a muscle
of facial expression, is innervated by CN VII, the facial nerve, geniohyoid is innervated by CN XII, the
hypoglossal, the posterior digastric is innervated by CN VII, the facial, and levator palatini is
innervated by the cervical plexus.

80

161. A person lifts one foot prior to taking a step. Which of the following nerves innervates
the muscle group that allows the person to maintain balance by holding the weight of his body
over the foot remaining on the ground?
A. Femoral nerve
B. First and second sacral nerves
C. Obturator nerve
D. Superior gluteal nerve
E. Tibial nerve
The correct answer is D. The muscles in question are the gluteus medius and gluteus
minimus, which are the innervated by the superior gluteal nerve. Textbooks often describe
these muscles as abductors of the hip, but in real life, they usually function as described in
the question.
The femoral nerve (choice A) innervates muscles of the anterior thigh, allowing extension of
the leg at the knee.
The first and second sacral nerves (choice B) innervate the piriformis muscle, which is a
lateral rotator of the thigh at the hip.
The obturator nerve (choice C) innervates muscles of the medial thigh.
The tibial nerve (choice E) innervates muscles of the posterior lower leg.
162. Before removing a spleen, the splenic artery and splenic vein are ligated. Within which
of the following peritoneal structures are the splenic artery and vein found?
A. Gastrocolic ligament
B. Gastrosplenic ligament
C. Lesser omentum
D. Splenorenal ligament
The correct answer is D. The splenorenal ligament is the portion of the dorsal mesentery
between the posterior abdominal wall and the spleen. This mesentery transmits the splenic
artery and vein from their retroperitoneal position in the proximal portion of their course to the
peritoneal spleen.
The gastrocolic ligament (choice A) is the portion of the greater omentum between the
greater curvature of the stomach and the transverse colon. This portion of mesentery is not
related to the spleen.
The gastrosplenic ligament (choice B) is the portion of the dorsal mesogastrium between the
greater curvature of the stomach and the spleen. There are no splenic vessels in this
mesentery.
The lesser omentum (choice C) is derived from the ventral mesentery. It is the mesentery
between the lesser curvature of the stomach and the liver and between the first portion of the
duodenum and the liver. It is not related to the spleen.

81

163 Observation of a hematoxylin and eosin-stained microscope slide reveals that the nuclei
are blue. What is the basis for this observation?
A. Eosin binds to carbohydrates
B. Eosin binds to lipids
C. Eosin binds to nucleic acids
D. Hematoxylin binds to lipids
E. Hematoxylin binds to nucleic acids
The correct answer is E. Blue hematoxylin binds to polyanions such as RNA and DNA.
Nuclei contain large amounts of DNA and RNA, and they are consequently almost always
blue. The nuclei of dysplastic and cancerous cells are often enlarged and hyperchromatic
(e.g., darker blue) compared with normal cells of similar cell lines because these altered cells
often have extra DNA (are aneuploid) and/or RNA (are metabolically active).
Pink eosin binds relatively nonselectively to cellular components, particularly proteins.
Cytoplasm of different cell lines can be pink, purple, or blue, depending principally on the
number of ribosomes in the cytoplasm. Consequently, blue-tinged cytoplasm tends to suggest
high synthetic activity (i.e., abundant ribosomes).
164. Which of the following pharyngeal pouches develops into the palatine tonsil?
A. First
B. Second
C. Third
D. Fourth
E. Fifth
The correct answer is B. The epithelial lining of the second pharyngeal pouch buds into the
mesenchyme to form the palatine tonsil. Part of the pouch remains in the adult as the tonsillar
fossa.
It is important to review the other choices because pharyngeal pouch derivatives are typically
tested on the NBDE part 1.
The first pharyngeal pouch (choice A) develops into the middle ear cavity and eustachian
tube.
The third pharyngeal pouch (choice C) develops into the thymus and the inferior parathyroid
glands.
The fourth pharyngeal pouch (choice D) gives rise to the superior parathyroid glands. Recall
that abnormal development of the third and fourth pouches leads to DiGeorge syndrome and
results in hypocalcemia, as well as abnormal cellular immunity and consequent susceptibility
to viral and fungal illnesses.
The fifth pharyngeal pouch (choice E) gives rise to the C cells of the thyroid gland. These
cells secrete calcitonin, a hormone that lowers serum calcium.

82

165. When a large bolus of food is swallowed, the hyoid bone moves anteriorly to open the
pharynx. The muscle responsible for this action is innervated by which of the following
nerves?
A. C1 (ansa cervicalis)
B. CN IX
C. Pharyngeal plexus
D. CN V3
E. CN VII
The correct answer is A. The muscle that moves the hyoid anteriorly to open the pharynx is
the geniohyoid. This muscle is innervated by the ansa cervicalis branch of C1. The ansa
cervicalis also innervates the thyrohyoid and the intra hyoids.
Cranial nerve IX (choice B) innervates the stylopharyngeus.
The pharyngeal plexus (choice C) innervates the palatoglossus, palatopharyngeus, and
cricopharyngeus.
Cranial nerve V-3 (choice D) innervates most of the muscles of mastication.
Cranial nerve VII (choice E) innervates the buccinator and other muscles of facial expression.
166. In a posterolateral herniation of the nucleus pulposus of the intervertebral disk between
vertebrae C4 and C5, what neural structure is most likely to be injured?
A. Anterior ramus C5
B. Posterior ramus C4
C. Spinal cord
D. Spinal nerve C4
E. Spinal nerve C5
The correct answer is E. A posterolateral herniation of the nucleus pulposus is the most
common type of herniation. This herniation results in the nucleus pulposus occupying space
within the intervertebral foramen. The spinal nerve in the intervertebral foramen between
vertebrae C4 and C5 is the C5 spinal nerve.
The anterior and posterior rami (choices A and B) are found lateral to the intervertebral
foramina and are not compressed by a herniated nucleus pulposus.
The spinal cord (choice C) is within the vertebral canal. A posterior herniation of a nucleus
pulposus (less common) may compress the spinal cord.
Spinal nerve C4 (choice D) is found in the intervertebral foramen between vertebrae C3 and
C4 and would not be compressed by this herniation.

83

167. A football player experiences an anterior dislocation of the shoulder. Cutaneous


sensation over the lower half of the deltoid muscle is impaired. These findings suggest
damage to which of the following nerves?
A. Axillary
B. Median
C. Musculocutaneous
D. Radial
E. Ulnar
The correct answer is A. The axillary nerve can be damaged during anterior dislocation of
the shoulder, causing loss of sensation in the skin overlying the lower half of the deltoid
muscle.
The median nerve (choice B) supplies sensation to the anterior arm, palm, and distal aspects
of the lateral three-and-a-half fingers.
The musculocutaneous nerve (choice C) supplies sensation to the lateral surface of the arm
and forearm.
The radial nerve (choice D) supplies sensation to the back of the arm, forearm, and hand.
The ulnar nerve (choice E) supplies sensation to the medial side of the arm, forearm, and
hand.

168. Karyotypic analysis of a spontaneously aborted fetus demonstrates trisomy of one of


the chromosomes. Which chromosome is most likely to be affected?
A. 8
B. 13
C. 16
D. 18
E. 21
The correct answer is C. Approximately half of all spontaneous abortions are of fetuses with
major chromosomal defects, most commonly trisomy 16, triploidy (due to fertilization of an
egg by two sperm), and 45 X,0 (Turner's syndrome). Trisomy 16 and triploidy do not produce
viable offspring, unlike 45 X,0.
Trisomy 8 (choice A) is one of the very rare causes of live birth trisomies.
Trisomy 13 (choice B) is one of the more common live birth trisomy syndromes (Patau
syndrome).
Trisomy 18 (choice D) is one of the more common live birth trisomy syndromes (Edwards'
syndrome)
Trisomy 21 (choice E) causes Down's syndrome.

84

169. A lesion in which of the following structures could be responsible for unilateral hearing
loss?
A. Inferior colliculus
B. Lateral lemniscus
C. Medial geniculate body
D. Medial lemniscus
E. Organ of Corti
The correct answer is E. The sequence of the auditory pathway is as follows: Organ of Corti
spiral ganglion in the cochlea vestibulocochlear nerve (CN VIII) cochlear nuclei (dorsal
and ventral) superior olivary nuclei lateral lemniscus inferior colliculus medial
geniculate nucleus of the thalamus (MGN) primary auditory cortex (Heschls gyrus).
Each ear projects to both sides of the brainstem and cortex by way of multiple commissures,
including the trapezoid body (which contains fibers crossing contralateral to the superior
olivary nucleus), the commissure of the inferior colliculus (connecting the right and left inferior
colliculi), and another commissure that connects the right and the left nuclei of the lateral
lemniscus. A lesion of any structure up until the superior olivary nuclei therefore will produce
an ipsilateral deafness. The only structure listed that is proximal to the superior olivary nuclei
is the organ of Corti (choice E).
The inferior colliculus (choice A), the lateral lemniscus (choice B) and the medial geniculate
body (choice C) all receive information from both ears, and unilateral hearing loss could not
result from a lesion of any of these structures. The medial lemniscus (choice D) is not a part
of the auditory system. It is part of the somatosensory system, which conveys proprioception,
discriminative touch, and vibration information. More specifically, neurons of the gracile and
cuneate nuclei send projections that decussate as the internal arcuate fibers and ascend as
the medial lemniscus to synapse in the ventroposterolateral nucleus (VPL) of the thalamus.
170. A mailman gets a severe bite wound from a pit bull guarding a junkyard. The wound is
cleansed and he receives a booster injection of tetanus toxoid and an injection of penicillin G.
Several days later, the wound is inflamed and purulent. The exudate is cultured on blood agar
and yields gram-negative rods. Antibiotic sensitivity tests are pending. The most likely agent
to be isolated is
A. Bartonella henselae
B. Brucella canis
C. Clostridium tetani
D. Pasteurella multocida
The correct answer is D. Pasteurella multocida is a gram-negative rod that is normal flora of
the oral cavity of dogs and cats. It often causes a local abscess following introduction under
the skin by an animal bite. Most cases occur in children who are injured while playing with a
pet.
Bartonella henselae (choice A) is a very small, gram-negative bacterium that is closely
related to the rickettsia, although it is able to grow on lifeless media. It is the cause of catscratch disease (a local, chronic lymphadenitis most commonly seen in children) and bacillary
angiomatosis (seen particularly in patients with AIDS). In this latter patient population, the
organism causes proliferation of blood and lymphatic vessels, causing a characteristic
"mulberry" lesion in the skin and subcutaneous tissues of the afflicted individual.
Brucella canis(choice B) is a gram-negative rod that is a zoonotic agent. Its normal host is
the dog, but when it gains access to humans, it causes an undulating febrile disease with

85

malaise, lymphadenopathy, and hepatosplenomegaly. The normal route of exposure is by


way of ingestion of the organism.
Clostridium tetani(choice C) is a gram-positive spore-forming anaerobic rod. It causes
tetanus (a spastic paralysis caused by tetanospasmin). There may be no lesion at the site of
inoculation, and exudation would be extremely rare.
171. On microscopic examination of bone, small thread-like canals are seen radiating out
from the area of each osteocyte. These structures are the
A. Volkman canals
B. Canaliculi
C. Osteons
D. Haversian canals
E. Lacunae
The correct answer is B. In compact bone, the bone is in concentric layers, and small
spaces between layers house the osteocytes, or mature bone cells. These small spaces are
called lacunae. Radiating out from the lacunae are small conducting vessels known as
canaliculi. They help bring nutrients to and waste from the osteocytes. Remember that bone
matrix does not allow diffusion readily. The original source of nutrients to compact bone is
Haversian canals, which contain arteries, veins, and nerves. Smaller perpendicular branches
of these canals are sometimes called Volkman canals. Osteons are also called Haversian
systems, and consist of the Haversian canal, the concentric bone layers around them, the
osteocytes in the concentric rings, the lacunae and canaliculi. Osteons are found in compact
(lamelllar) bone.

172. If vertical dimension of occlusion is VDO, and vertical dimension of rest is VDR, and
freeway space is FS, then:
A. VDO + VDR = FS
B. VDO + FS = VDR
C. VDR + FS = VDO
D. None of the above
The correct answer is B. The vertical dimension of occlusion (VDO) is a vertical
measurement on the front of the face when the teeth are in full occlusion (centric occlusion).
When the face is at rest, the teeth are slightly apart, and the vertical dimension of the front of
the face is slightly longer. This is the vertical dimension of rest (VDR). The distance between
the teeth at this point is the freeway space (FS). So when we take the smaller VDO and add
the few millimeters of the FS, we get the slightly longer VDR. VDR is generally 2 to 5 mm
more than VDO.
173 Food and oxygen can reach the osteocytes of compact bone through:a) osseous matrixb)
canaliculi c) capillaries d) cartilaginous matrix e) Volkman canals
A. a, b, and c
B. a, c, and e
C. b, c, and d
D. a, c, and d
E. b, c, and e

86

The correct answer is E. Bone matrix is generally impermeable to diffusion of nutrients for
any great distance. Osteocytes in need of food and oxygen, and also needing to excrete
waste, therefore need to be near some transport system. In compact bone, Haversian canals
carry arteries, veins, capillaries, and nerves. Horizontal branches of these canals are
sometimes termed Volkman canals. Small canaliculi (small canals) branch out of these
canals, and also radiate from the lacunae that house the osteocytes. All of these structures
therefore can be involved in nutrient transport. Cartilaginous matrix, unlike osseous (bone)
matrix, is permeable, slowly, to nutrient travel. There is no cartilaginous matrix, however, in
fully formed bone tissue.
174. Parathormone will have its greatest effect on which cell type?
A. Osteoclasts
B. Osteoblasts
C. Osteocytes
D. Osteosarcomas
The correct answer is A. Parathormone (PTH) is secreted by principal cells of the
parathyroid glands, small gland pairs embedded in thyroid tissue. Parathyroid hormone will
increase the level of serum calcium by increasing bone calcium resorption, increasing
digestive absorption of calcium, and decreasing kidney excretion of calcium. The bone
resorbing cells are osteoclasts, which are large, multinucleated cells. Osteoblasts are boneformers and secrete bone matrix, primarily of collagen. Osteocytes are mature bone cells
living in lacunae in compact bone. Osteosarcomas are malignant tumors of bone.
175. In the adult, neurons are an example of which of the following?
A.
B.
C.
D.
E.

Continuously dividing cells


Labile cells
Permanent cells
Quiescent cells
Stable cells

The correct answer is C. Permanent (non-dividing) cells have left the cell cycle and can no
longer undergo mitotic division. Examples that are usually cited include neurons and cardiac
myocytes. One of the major disadvantages of these cells is that once they are damaged or
destroyed, they do not regenerate.
Examples of labile (continuously dividing) cells (choices A and B) include the epithelia of the
body surface, gastrointestinal tract, and urinary tract.
Examples of stable (quiescent) cells (choices D and E) include the cells of the liver and the
glandular organs of the body.
176. The interlobular arteries of the kidney result from subdivision of the
A. efferent arterioles
B. arcuate arteries
C. interlobar arteries
D. renal artery
The correct answer is B. The renal artery (choice D) enters the kidney at the hilum and
immediately divides into the interlobar arteries (choice C). These arteries supply the renal
pelvis and capsule before passing between the medullary pyramids to the corticomedullary

87

junction. They then bend almost 90 degrees to form the short, arching arcuate arteries
(choice B), which run along the corticomedullary junction.
The arcuate arteries subdivide into the fine interlobular arteries. These interlobular arteries
give off branches that become the afferent arterioles (choice A) of the glomerulus.

177. Following thyroidectomy, hoarseness of the voice may occur. This condition is caused
by damage to the
A.
B.
C.
D.
E.

internal laryngeal nerve


recurrent laryngeal nerve
thyroarytenoid muscle
vestibular folds
vocal folds

The correct answer is B. The recurrent laryngeal nerves are branches of the vagus (CN X)
and supply all intrinsic muscles of the larynx, except the cricothyroid. The right recurrent
laryngeal nerve recurs around the right subclavian artery. The left recurrent laryngeal nerve
recurs in the thorax around the arch of the aorta and ligamentum arteriosum. Both nerves
ascend to the larynx by passing between the trachea and esophagus, close to the thyroid
gland.
The internal laryngeal (choice A) nerve is a purely sensory branch of the superior laryngeal
nerve.
The thyroarytenoid (choice C) is an intrinsic muscle of the larynx; its inner fibers are
specialized as the vocalis muscle, which is related to the vocal ligament. It is not usually at
risk during thyroid surgery.
The vestibular folds (choice D), or false vocal folds, are located superior to the true vocal
folds inside the larynx. They are not concerned with phonation.
The vocal folds (choice E) form the boundaries of the rima glottidis inside the larynx and are
not vulnerable during thyroidectomy.
178. Compression of the obturator nerve will result in weakness of which of the following
muscles?
A. Adductor magnus
B. Biceps femoris
C. Rectus femoris
D. Sartorius
E. Vastus medialis
The correct answer is A. The obturator nerve innervates the muscles of the medial
compartment of the thigh. The adductors are located inferior to the acetabular surface. These
include the adductor longus, adductor brevis, adductor magnus, and gracilis muscles. The
adductor magnus is also innervated by the tibial nerve.
The biceps femoris (choice B) is in the posterior compartment of the thigh. The long head of
the biceps femoris is innervated by the tibial portion of the sciatic nerve, and the short head of
the biceps femoris is innervated by the common peroneal portion of the sciatic nerve.

88

The rectus femoris (choice C) and vastus medialis (choice E) are two of the four heads of
the quadriceps femoris muscle. All four heads of the quadriceps femoris muscle are in the
anterior compartment of the thigh and are innervated by the femoral nerve.
The sartorius muscle (choice D) is in the anterior compartment of the thigh and is innervated
by the femoral nerve.
179. The first bony structure that can be palpated below the inferior margin of the medial
portion of the clavicle is the
A.
B.
C.
D.
E.

acromion
atlas
first rib
manubrium
second rib

The correct answer is E. The palpable space immediately inferior to the clavicle is the first
intercostal space, and the bone below it is the second rib.
The acromion (choice A) is the lateral extension of the scapular spine.
The atlas (choice B) is the first cervical vertebra, articulating with the occipital bone above
and the axis below.
The first rib (choice C) is hidden under the clavicle.
The manubrium (choice D) is the most superior portion of the sternum.
180. A patient develops a form of lung cancer that spreads to occlude the thoracic duct.
Edema involving which of the following sites might be expected as a potential complication?
A.
B.
C.
D.
E.

Entire left side and right leg


Entire right side and left leg
Left arm only
Right arm and right half of head only
Right arm only

The correct answer is A.. The right lymphatic duct drains the right arm, the right side of the
chest, and the right side of the head. The thoracic duct drains the rest of the body. Both the
right lymphatic duct and the thoracic duct dump into the large venous channels at the base of
the neck at the junction of the subclavian vein and internal jugular vein. Occlusion of this
drainage can produce intractable edema in sites feeding these ducts.
The left side and right leg would be affected, rather than the right side and left leg (choice B).
The entire left side (rather than just the left arm, choice C) and right leg drain into the thoracic
duct.
The right arm and the right half of the head (choices D and E) drain to the right lymphatic
duct.

89

181. During development, the formation of the kidney is induced by which of the following
structures?
A. Allantois
B. Mesonephric duct
C. Mesonephros
D. Metanephric duct
E. Urogenital ridge
The correct answer is D. The metanephric duct (also known as the ureteric bud) is a
diverticulum of the mesonephric duct. It grows to the metanephric mass of the urogenital
ridge. It induces the development of the metanephros, which will give rise to the excretory
units of the definitive kidney. The metanephros, in turn, induces the metanephric duct to
divide into the calyces and the collecting tubules.
The allantois (choice A) is an endodermal diverticulum of the yolk sac. It becomes
incorporated into the urogenital sinus and contributes to the formation of the urinary bladder.
It is not involved with the formation of the kidney.
The mesonephric duct (choice B) is a mesodermal duct into which the mesonephric tubules
drain. The mesonephric duct is the same as the pronephric duct, but at a later stage of
development. The mesonephric duct develops into the ductus deferens in the male. The
metanephric duct is a diverticulum of the mesonephric duct.
The mesonephros (choice C) is the embryonic kidney, which functions in the embryo before
the metanephros (or definitive kidney) develops. The mesonephros develops from the
urogenital ridge.
The urogenital ridge (choice E) is a longitudinal elevation of the intermediate mesoderm. The
embryonic and adult kidneys, as well as the gonads, develop from this mesoderm.
182. A fall, with hyperextension of the hand, causes anterior dislocation of a carpal bone.
Which nerve is compressed by this injury?
A. Anterior interosseous nerve
B. Median nerve
C. Posterior interosseous nerve
D. Radial nerve
E. Ulnar nerve
The correct answer is B. The carpal bone that has been dislocated is the lunate, one of the
four proximal carpal bones. This bone has been dislocated into the carpal tunnel, which is
anterior to the carpal bones. The median nerve is the only nerve that traverses the carpal
tunnel.
The anterior interosseous nerve (choice A) is a branch of the median nerve that arises in the
proximal forearm and innervates the deep muscles of the anterior forearm. This nerve does
not traverse the carpal tunnel.

90

The posterior interosseous nerve (choice C) is a branch of the radial nerve that arises in the
proximal forearm and innervates several muscles of the posterior compartment of the
forearm. This nerve does not traverse the carpal tunnel.
The radial nerve (choice D) innervates posterior muscles of the arm, some posterior muscles
of the forearm, and areas of skin on the posterior surface of the upper limb. It does not pass
through the carpal tunnel.
The ulnar nerve (choice E) innervates some muscles in the anterior compartment of the
forearm and many muscles in the hand. The nerve crosses the wrist superficial to the flexor
retinaculum and is therefore not within the carpal tunnel. It is spared compression by the
dislocation of the lunate.
183. Overgrowth of the frontal bone, hands and feet has occurred, as well as tingling in the
first, second, and third digits. There is also loss of strength of the thumb. Which nerve is most
likely to be involved?
A.
B.
C.
D.
E.

Anterior interosseous nerve


Median nerve
Musculocutaneous nerve
Radial nerve
Ulnar nerve

The correct answer is B. This question is essentially asking, "Which of the following nerves
innervates the anatomical structures listed?" This patient has acromegaly, which is
characterized by overgrowth of the face, jaws, hands, and feet, enlargement of internal
organs, hyperglycemia, hypertension, and osteoporosis. It is caused by hypersecretion of
growth hormone, often attributed to an adenohypophyseal tumor. Complications include
degenerative joint disease, muscular weakness, neuropathies, and diabetes mellitus. In this
question, although the patient's sensory symptoms may be caused by a neuropathy, it is very
likely that overgrowth in the wrist area has compressed the carpal tunnel, thereby impinging
on the median nerve. Note that the median nerve (root C5-T1) provides motor innervation to
the forearm flexors, thenar muscles, and radial lumbricals. It provides sensory innervation to
the radial 2/3 of the palm, volar surfaces of the thumb, second and third digits, and radial 1/2
of the fourth digit, as well as the skin over the lateral surface of the hand.
Damage to the anterior interosseus nerve (choice A), also known as the deep branch of the
median nerve, results in the inability to form a round "O" with the thumb and forefinger. This is
due to impaired function of the flexor pollicis longus. Damage to the anterior interosseus
nerve could explain the patient's thumb dysfunction, but it would not account for the patient's
paresthesias in the first three digits of the hand.
The musculocutaneous nerve (choice C) innervates the arm flexors and provides sensory
information to the anterolateral forearm. It is composed of contributions from C5-C7.
The radial nerve (choice D) innervates the extensors of the arm and forearm and skin of the
posterior arm, forearm, and radial half of the dorsum of the hand (not including the fingertips).
It is composed of contributions from C6-C8.
The ulnar nerve (choice E) provides motor innervation to the ulnar flexors, adductor pollicis,
hypothenar muscles, interosseus muscles, and lumbricals 4 and 5. It provides sensory
innervation to the ulnar half of the wrist, palm, and fourth and fifth digits. It is composed of
contributions from C8-T1.

91

184. Through which of the following structures does the greater saphenous vein pass to
eventually join the femoral vein?
A.
B.
C.
D.
E.

Anatomic snuff box


Antecubital fossa
Fossa ovalis
Inguinal canal
Popliteal fossa

The correct answer is C. The greater (long) saphenous vein starts on the dorsal surface of
the foot, wraps proximally to follow the medial aspect of the leg below and past the knee, and
then dives into the deep fascia through the fossa ovalis (also called the saphenous opening)
to reach the femoral vein, which becomes the external iliac vein shortly after it is joined by the
greater saphenous vein.
The anatomic snuff box (choice A) is on the wrist. Some authors also refer to an "anatomic
snuff box of the foot," through which the saphenous vein does pass, but this is early in its
course, and not just before it joins the femoral vein.
The antecubital fossa (choice B) is at the elbow.
The inguinal canal (choice D) contains structures going to and from the testes.
The saphenous vein passes near, but not through, the popliteal fossa (choice E) to cross the
knee.
185. In which of the following organs are fenestrated endothelial cells common?
A. Heart
B. Liver
C. Lungs
D. Pancreas
E. Stomach
The correct answer is B. Fenestrated ("windowed") endothelial cells, which permit free flow
of serum across the endothelium, are an unusual feature of the hepatic sinusoids. They are
not seen in the other organs listed.
Along the same line, fenestrated capillaries are capillaries containing "windows" or pores that
span the endothelial lining. The pores permit the rapid exchange of water and solutes as large
as small peptides between the plasma and interstitial fluid.
186. The suprahyoids, including mylohyoid, digastric, and geniohyoid, act as accessory:
A. elevators
B. depressors
C. retruders
D. protruders
E. none of the above

92

The correct answer is B. The major depressor (opener) is the lateral pterygoid, which also
protrudes and causes lateral motion. Mylohyoid, digastric, and geniohyoid can aid this motion.
Mylohyoid can produce opening movement by contracting between the hyoid bone and the
mylohyoid line on both sides of the mandible. The anterior digastric produces an opening
motion as it contracts between the digastric facial sling on the hyoid bone and the interior of
the mandible (digastric fossa). The geniohyoid can contract between the hyoid bone and the
genial tubercle on the mandibular symphysis.
187. A fracture of the pterygoid hamulus affects the function of which muscle?
A.
B.
C.
D.

Tensor veli palatini


Levator veli palatini
Medial pterygoid
Lateral pterygoid

The correct answer is A. This may seem like an obscure fact, but it is a perennial NBDE
favorite. The tensor veli palatini wraps around a small hook-like extension of the medial
pterygoid plate of the sphenoid bone, known as the pterygoid hamulus. The tensor (veli)
palatini originates from the greater wing of the sphenoid and from cartilage of the auditory
tube. The fibers converge into a tendon, which sharply hooks around the pterygoid hamulus.
The fibers then join with their partners from the other side to form the palatal aponeurosis to
which most of the muscles of the soft palate are attached. The muscle tenses the soft palate,
especially during swallowing.
188. Pain experienced in the anterior half of the external ear canal is transmitted by which of
the following nerves?
A. Auricular branch of the vagus
B. Auriculotemporal nerve
C. Greater auricular nerve
D. Lesser occipital nerve
E. Vestibulocochlear nerve
The correct answer is B. The ear has a complex sensory nerve supply, which includes all of
the nerves listed. A consequence of this complexity is that pain actually originating in other
sites (teeth and sinuses are notorious) may be misinterpreted as ear pain; or, less commonly,
pain originating in the ear may be misinterpreted as arising from other sites. The anterior half
of the external ear canal is supplied by the auriculotemporal nerve, which also supplies the
facial surface of the upper part of the auricle.
The auricular branch of the vagus (choice A) supplies the posterior half of the external ear
canal.
The greater auricular nerve (choice C) supplies both surfaces of the lower part of the auricle.
The lesser occipital nerve (choice D) supplies the cranial surface of the upper part of the
auricle.
The vestibulocochlear nerve (choice E) supplies hearing and motion sense.

93

189. The intraembryonic mesoderm formed during the third week of embryonic life is
principally derived from
A. definitive ectoderm
B. definitive endoderm
C. epiblast
D. primitive node
E. yolk sac
The correct answer is C. The bilaminar disc forms in the second week of embryonic life by
differentiation of embryoblast cells into epiblast (primary ectoderm adjacent to the amniotic
cavity) and hypoblast (primary endoderm adjacent to the yolk sac; (choice E) layers.
During the third week of life, a linear thickening of some of the epiblast cells produces a
primitive streak, bounded rostrally by the primitive node (choice D).
Epiblast cells migrate from the primitive streak downward, replacing the original hypoblast
with definitive endoderm (choice B).
The migrating epiblast also fills the potential space between the two layers with a third germ
layer, the intraembryonic mesoderm (choice C). The remaining epiblast cells (which don't
migrate) form the definitve ectoderm (choice A).
190. Microscopic examination of a section of a normal young adult ovary reveals large
numbers of unusually large cells surrounded by a single layer of flat epithelial cells. In which
phase of the cell cycle are these cells arrested?
A. Diplotene stage of the first meiotic division
B. Diplotene stage of the second meiotic division
C. Metaphase stage of mitosis
D. Prophase stage of mitosis
E. Prophase stage of the second meiotic division
The correct answer is A. The cells described are the primordial eggs, which remain stopped
in the diplotene stage of the first meiotic division from before birth until fertilization, a period
which may be 40 or more years.
Choices B and E are incorrect because the oocytes are stopped in the first, not the second,
meiotic division.
Choices C and D are incorrect because the cells described are oocytes and are not in
mitosis.

94

191. The primordium of the respiratory system that forms the internal epithelial lining of the
treachea, bronchi, and lungs is derived enterily from which of the following embryologic
entities?
A.
B.
C.
D.
E.

Foregut endoderm
Midgut endoderm
Splanchnic mesoderm
Stomodeum
Both endoderm and mesoderm

The correct answer is A. The internal epithelial lining of the trachea, bronchi, and lungs form
from a diverticulum of the ventral foregut wall (endoderm) called the respiratory diverticulum.
The midgut endoderm (choice B) gives rise to the internal epithelial lining of much of the
gastrointestinal tract, duodenum, small intestine, ascending colon, and right two thirds of
transverse colon.
The splanchnic mesoderm (choices C and E) contributes only to the smooth muscle coat
and cartilaginous skeleton of the trachea and bronchi.
The stomodeum (choice D) is of ectodermal origin and gives rise to the mouth. The
stomodeum eventually becomes continuous with the endodermal foregut of the larynx (and
trachea) and the esophagus.
192. A neonate has a prominent defect at the base of his spine through which his meninges
and spinal cord protrude. A failure of which of the following processes is the most common
cause of this type of defect?
A. Development of the body
B. Development of the pedicle
C. Development of primary vertebral ossification centers
D. Development of the superior articular process
E. Fusion of the vertebral arches
The correct answer is E. The condition described is spina bifida with myelomeningocele. A
failure of the posteriorly located vertebral arches to fuse posteriorly causes spina bifida, which
can vary in severity from a completely asymptomatic, minor abnormality to protrusion of the
spinal cord and roots through an open skin defect, with a very real risk of minor trauma or
infection causing paralysis.
The bodies of the vertebrae (choice A) are the stacking ovoid structures on the anterior
aspect of the spinal canal.
The pedicles (choice B) attach the body ring that surrounds the spinal canal to the body of
the vertebrae.
Failure of development of one of the paired primary ossification centers (choice C) of the
body can produce very severe scoliosis.
The superior articular process (choice D) articulates with the inferior articular facet of the
vertebra above it.

95

193. Following fracture of the humerus, an adult patient has a biopsy of the healing area.
Which of the following types of bone will the biopsy most likely show?
A. Cancellous
B. Compact
C. Spongy
D. Trabecular
E. Woven
The correct answer is E. Bone is formed by type I collagen fibers, ground substance, and
hydroxyapatite crystals. The collagen is oriented in a layered or lamellar fashion. It can be
parallel (trabecular bone and periosteum) or concentric (Haversian system). When bone is
formed quickly, as in a healing fracture site, metabolic bone disease, or tumor, the collagen is
randomly oriented and is called woven bone.
Compact bone (choice B) is the dense, calcified, external part of the bone. It is lamellar bone.
Cancellous (choice A), spongy (choice C), and trabecular (choice D) bone are all
synonymous terms for the thinner network of bone within the cortex.
194. After an automobile accident, a patient has difficulty opening her mouth but has no
difficulty closing it. Which of the following muscles was most likely injured?
A.
B.
C.
D.
E.

Lateral pterygoid
Masseter
Medial pterygoid
Orbicularis oris
Temporalis

The correct answer is A. The lateral pterygoid muscles consist of two heads, an upper head
arising from the infratemporal surface of the greater wing of the sphenoid bone, and a lower
head arising from the lateral pterygoid plate. The lateral pterygoids act to pull the mandible
forward in the process of opening the jaw. Other actions include protracting the mandible and
moving it from side to side (as in chewing). The lateral pterygoid is innervated by the
mandibular branch of the trigerminal nerve.
The masseter (choice B) and medial pterygoid (choice C) muscles help to close the jaw,
while the temporalis (choice E) closes and retrudes the mandible. All of these muscles are
innervated by the mandibular branch of the trigeminal nerve.
The orbicularis oris (choice D) muscle purses the lips. It is innervated by the facial nerve.
195. A patient has a herniated intervertebral disc impinging on the right C5 nerve roots. Which
of the following movements would most likely be affected?
A. Extension of the fingers
B. Extension of the shoulder
C. Flexion of the elbow
D. Flexion of the wrist
E. Pronation of the elbow

96

The correct answer is C. C5 helps mediate flexion, abduction, and lateral rotation of the
shoulder, and flexion of the elbow. Both C5 and C6 mediate extension of the elbow.
Extension of the fingers (choice A) is mediated by C7 and C8.
Extension of the shoulder (choice B) is mediated by C7 and C8.
Flexion of the wrist (choice D) is mediated by C6 and C7.
Pronation of the elbow (choice E) is mediated by C7 and C8.
196. A hypophysectomy is scheduled for removal of a pituitary adenoma. The tumor will be
approached transnasally, through the
A. cavernous sinus
B. ethmoid sinus
C. frontal sinus
D. mastoid sinus
E. sphenoid sinus
The correct answer is E. The hypophysis, or pituitary gland, is approached transnasally
through the sphenoid sinus. This small gland lies within the sella turcica, a depression in the
sphenoid bone. The pituitary gland hangs beneath the hypothalamus, connected by a slender
stalk, the infundibulum. The base of the infundibulum lies between the optic chiasm and
mamillary bodies.
Inadvertent transgression into the cavernous sinus (choice A) would lead to torrential venous
or arterial hemorrhage from the cavernous sinus itself or the internal carotid artery contained
within.
The ethmoid sinus (choice B) is related to the medial wall of the orbit.
The frontal lobe lies behind the frontal sinus (choice C).
The mastoid sinus (choice D) lies within the temporal bone and communicates with the
middle ear.
197. Mandibulofacial dysostosis is due to abnormal development of which of the following
structures?
A.
B.
C.
D.
E.

First pharyngeal arch


First pharyngeal cleft
Second pharyngeal arch
Second pharyngeal cleft
Third pharyngeal cleft

The correct answer is A. The pharyngeal arches are outpouchings of tissue visible on the
external neck of the embryo. They are separated by the pharyngeal clefts (each one caudal to
its arch). The pharyngeal pouches are the outpouchings of the pharynx visible inside the
embryo that correspond to the arches. The defect described is due to abnormal development
of the derivatives of the first pharyngeal arch. It is thought that the initial defect may be
insufficient migration of neural crest cells.

97

198. If a lumbar puncture needle passes through the interlaminar space between the fourth
and fifth lumbar vertebrae, which space does it enter?
A. Epidural
B. Intramuscular
C. Subarachnoid
D. Subcutaneous
E. Subdural
The correct answer is A. The interlaminar space is the space between the laminae of the
lumbar vertebrae. The ligamentum flavum crosses the interlaminar space. Because there is a
gap between the right and left ligamentum flavum in the midline, this ligament is not
penetrated if the needle is in the midline. Immediately within the bony vertebra is the epidural
space, which lies between the bone and the dura. The epidural space contains a considerable
amount of fat and a venous plexus.
Because the muscles of the back are located on either side of the midline, no muscle is
penetrated when doing a lumbar puncture (choice B).
The subarachnoid space (choice C), which is filled with cerebrospinal fluid, is reached after
the needle passes through the epidural space and pierces the dura and arachnoid. The cauda
equina, rather than the spinal cord, is located in the subarachnoid space of the low lumbar
region where a lumbar puncture is performed.
The subcutaneous space (choice D) contains superficial fascia and fat and is found
immediately beneath the skin. After passing through the subcutaneous fascia, the needle
passes through the supraspinous ligament and the interspinous ligament.
The subdural space (choice E) is a potential space between the dura and arachnoid.
Normally, this space does not exist because the arachnoid is closely applied to the dura. If
there were bleeding into this potential space, the two layers would separate and a subdural
hematoma would be produced.
199. A patient is able to say "Kuh, kuh, kuh," and "Mi-mi-mi." The patient is unable to say "Lala-la." A lesion of which of the following cranial nerves should be suspected?
A.
B.
C.
D.
E.

VII
VIII
X
XI
XII

The correct answer is E. Saying "la" requires the tongue to be placed against the roof of the
mouth, which cannot be accomplished if the hypoglossal nerve (CN XII) is nonfunctional. CN
XII innervates the intrinsic muscles of the tongue. The hypoglossal nerve innervates the
genioglossus, hypoglossus, palatoglossus, and styloglossus muscles of the tongue.
The facial nerve, CN VII (choice A), is needed to move the lips while saying "mi." CN VII
innervates the muscles of facial expression.
The vestibulocochlear nerve, CN VIII (choice B), mediates hearing and maintains equilibrium
and balance. The vagus nerve, CN X (choice C), is needed to raise the palate while saying
"kuh." The spinal accessory nerve, CN XI (choice D), mediates head and shoulder
movements.

98

200. Which of the following is secreted by the neurohypophysis?


A. FSH
B. TSH
C. ADH
D. LH
E. STH
The correct answer is C.The first part of answering the question is to recognize the
terminology. In particular, the NBDE often uses the term neurohypophysis for posterior
pituitary, and adenohypophysis for anterior pituitary. The only two hormones of the posterior
pituitary (neurohypophysis) are ADH (vasopressin), which increases water reabsorption by
the kidney, and oxytocin, which stimulates uterine contractions during labor and delivery. All
other pituitary hormones are anterior, form the adenohypophysis. They include FSH (follicle
stimulating hormone), which acts on the ovary, TSH (Thyroid stimulating hormone), which
acts on the thyroid, LH (luteinizing hormone), which acts on the ovary, and STH (GH, growth
hormone) which stimulates long bone growth, and especially acts on epiphyseal plates.
201. Several arterial branches penetrate into the distal end of the lesser curvature of the
stomach. Which of the following arteries usually supplies these branches?
A. Left gastric
B. Left gastroepiploic
C. Right gastric
D. Right gastroepiploic
E. Short gastric
The correct answer is C. The arterial supply of the stomach is complex; it therefore has a
good potential to appear on the NBDE. The right gastric artery supplies the distal lesser
curvature.
The left gastric artery (choice A) supplies the proximal lesser curvature.
The left gastroepiploic artery (choice B) supplies the proximal greater curvature below the
splenic artery.
The right gastroepiploic artery (choice D) supplies the distal greater curvature. The short
gastric artery (choice E) supplies the proximal greater curvature above the splenic artery.
202. A woman who recently gave birth has elevated prolactin levels. The gland responsible
for secretion of this hormone is derived from which of the following structures?
A. Cerebral vesicle
B. Infundibulum
C. Neurohypophysis
D. Proctodeum
E. Rathke's pouch

99

The correct answer is E. The anterior pituitary produces prolactin. The structure originates
from Rathke's pouch, which is itself a diverticulum of the roof of the stomodeum.
The cerebral vesicle (choice A) lies close to Rathke's pouch.
The infundibulum (choice B) comes in contact with Rathke's pouch at the fifth week of
development.
The neurohypophysis (choice C) gives rise to the posterior pituitary.
The proctodeum (choice D) is also known as the anal pit.
203. During a mastectomy, the surgeon notes that the breast tumor has spread to involve the
muscle layer immediately deep to the breast. Which muscle is involved?
A. External oblique
B. Pectoralis major
C. Platysma
D. Rectus abdominis
E. Sternocleidomastoid
The correct answer is B. The muscle deep to the breast is the pectoralis major. Breast
cancer can invade this muscle but usually does not because the relatively thick, deep fascial
layer serves as a barrier. Formerly, the pectoralis major was removed during radical
mastectomy, leaving the patient with a major loss of function of the adjacent arm. The now
more frequently used modified radical mastectomy spares the pectoralis major.
The external obliques (choice A) and rectus abdominis (choice D) muscles are in the
abdomen.
The platysma (choice C) and sternocleidomastoid (choice E) muscles are in the neck.
204. Which of the following cell types is derived from neuroepithelial cells?
A. Astrocytes
B. Enterochromaffin cells
C. Melanocytes
D. Odontoblasts
E. Schwann cells
The correct answer is A. Astrocytes and oligodendrocytes are both derived from glioblasts,
which, in turn, are derived from neuroepithelial cells. Other neuroepithelial cell derivatives
include neuroblasts and ependymal cells. The astrocytes are the largest and most numerous
glial cells. These cells are responsible for maintaining the blood-brain barrier, creating a
three-dimensional framework for the central nervous system, performing repairs in damaged
neural tissues, and controlling the interstitial environment.
All the other choices are derived from neural crest cells. Other neural crest derivatives include
the neurons of the parasympathetic and sympathetic ganglia (including the adrenal medulla),
the dorsal root ganglia of the peripheral nervous system, the sensory ganglia of cranial nerves
V, VII, IX, and X, and the leptomeninges (pia and arachnoid).

100

205. Sensory innervation of the tongue, in order, from anterior to posterior, shows the effect of
development of which branchial arches?
A. First, second, third
B. Third, second, first
C. Second, third, fourth
D. Fourth, third, second
The correct answer is A. Although many students memorize the innervation of the tongue, it
can also be explained in a developmental way. In particular, the tongue is derived from the
first three pharyngeal (branchial) arches, and the cranial nerves associated with each arch
appear from anterior to posterior in the tongue innervation. Note that some texts also state
that the most posterior sections of the tongue are developed form the fourth arch as well. The
cranial nerves of each arch are as follows: Arch 1 (CN V), Arch 2 (CN VII), Arch 3 (CN IX),
and Arch 4 (CN X). Note that general sensation from the anterior two thirds of the tongue is
supplied by CN V, taste by CN VII, taste and general sensation from the posterior one third
are supplied by CN IX, and some innervation of the most posterior tongue by CN X.
206. A mutation affecting the development of the diencephalon could interfere with the
secretion of which of the following hormones?
A. Adrenocorticotrophic hormone (ACTH)
B. Epinephrine
C. Oxytocin
D. Prolactin
E. Thyroid-stimulating hormone (TSH)
The correct answer is C. The neurohypophysis (posterior pituitary) is derived from an
evagination of the diencephalic neurectoderm. This structure is responsible for releasing
oxytocin and vasopressin to the general circulation. Both hormones are synthesized in cell
bodies contained within the hypothalamus. Oxytocin is a hormone produced by the
hypothalamic cells and secreted into capillaries at the posterior pituitary. It stimulates smooth
muscle contractions of the uterus or mammary glands in the female, but has no known
function in the male.
ACTH (choice A), prolactin (choice D), and TSH (choice E) are all synthesized and released
by the anterior pituitary, or adenohypophysis, which is derived from an evagination of the
ectoderm of Rathke's pouch, a diverticulum of the primitive mouth. Remnants of this pouch
may give rise to a craniopharyngioma in later life.
Epinephrine (choice B) is synthesized and released into the circulation by the adrenal
medulla, a neural crest derivative.
207. Which of the following neurologic deficits would a large meningioma involving the brain's
parasagittal region and the falx cerebri be expected to produce?
A. Altered taste
B. Leg paralysis
C. Loss of facial sensation
D. Ptosis
E. Unilateral deafness
The correct answer is B. A meningioma (a tumor of the meninges) of the parasagittal region
and the falx cerebri would be located superiorly, between the two hemispheres. In this

101

position, it could compress the sensory (postcentral gyrus) or motor cortex (precentral gyrus)
supplying the lower extremities.
Taste (choice A) is supplied by cranial nerves VII, IX, and X. These nerves arise in the
brainstem.
Facial sensation (choice C) is supplied by cranial nerve V, the nuclei of which are in the
brainstem.
Ptosis (choice D) can be caused by a deficit in cranial nerve III, which arises from the
brainstem.
Unilateral deafness (choice E) suggests damage to cranial nerve VIII, which arises from the
brainstem.
208. Nissl bodies correspond to which of the following cytoplasmic organelles?
A. Golgi apparatus
B. Mitochondria
C. Nucleoli
D. Rough endoplasmic reticulum
E. Smooth endoplasmic reticulum
The correct answer is D. Rough endoplasmic reticulum present in neurons is called Nissl
substance, or Nissl bodies. Nissl bodies stain intensely with basic dyes and are found in the
cell body and proximal dendrites, but not in the axon hillock or axon.
209. Which of the following anatomic landmarks is the site of the anterior fontanelle in an
infant?
A. Bregma
B. Coronal suture
C. Lambda
D. Pterion
E. Sagittal suture
The correct answer is A. Bregma represents the point where the coronal and sagittal
sutures intersect; it is the site of the anterior fontanelle.
The coronal suture (choice B) lies between the frontal and parietal bones.
Lambda (choice C) represents the point where the sagittal and lambdoid sutures intersect; it
is the site of the posterior fontanelle in infants.
The pterion (choice D) is the point on the lateral aspect of the skull where the greater wing of
the sphenoid, parietal, frontal, and temporal bones converge. Recall that the pterion is the
landmark for the middle meningeal artery and that a blow to the temple (e.g., as could occur
in boxing) can lead to a middle meningeal arterial bleed and an epidural hemorhage.
The sagittal suture (choice E) is located between the two parietal bones.

102

210. A patient suffers severe head trauma, including a fracture in the region of the foramen
ovale. Which of the following functional losses would most likely be related to this injury?
A. Loss of abduction of the eye
B. Loss of sensation over the forehead
C. Loss of sensation over the zygoma
D. Loss of taste sensation on the anterior 2/3 of the tongue
E. Paralysis of muscles of mastication
The correct answer is E. The mandibular nerve (V3) passes through the foramen ovale and
may be injured by this fracture. The mandibular nerve is responsible for the innervation of all
of the muscles of mastication: the masseter, the temporalis, the medial pterygoid, and the
lateral pterygoid muscles, as well as the tensor palatini, tensor tympani, and mylohyoid.
Abduction of the eye (choice A) is produced by the lateral rectus muscle, which is innervated
by the abducens nerve. The abducens nerve leaves the cranial cavity and enters the orbit by
passing through the superior orbital fissure.
Sensation in the skin over the forehead (choice B) is provided by the ophthalmic division of
the trigeminal nerve. The ophthalmic division leaves the cranial cavity and enters the orbit by
passing through the superior orbital fissure.
Sensation in the skin over the zygoma (choice C) is provided by the maxillary division of the
trigeminal nerve. The maxillary division leaves the cranial cavity by passing through the
foramen rotundum and enters the pterygopalatine fossa.
Taste sensation on the anterior 2/3 of the tongue (choice D) is provided by the chordae
tympani, a branch of the facial nerve. The facial nerve leaves the cranial cavity by passing
into the internal auditory meatus and enters the temporal bone. The chordae tympani leaves
the temporal bone by passing through the petrotympanic fissure and enters the infratemporal
fossae.
211. A neonate is observed to have a cleft lip. Which of the following is the most likely
etiology of this malformation?
A. Abnormal development of the third and fourth pharyngeal pouches
B. Bony defects of the malar bone and mandible
C. Failure of the maxillary processes and medial nasal swellings to fuse
D. Incomplete joining of the palatine shelves
E. Insufficient migration of neural crest cells
The correct answer is C. Cleft lip is caused by the failure of the maxillary processes to fuse
with the medial nasal swellings.
Abnormal development of the third and fourth pharyngeal pouches (choice A) can give rise to
DiGeorge syndrome, which is characterized by the underdevelopment or absence of several
structures, including the thymus and parathyroids. Bony defects of the malar bone and
mandible (choice B) are associated with mandibulofacial dysostosis, which is mainly due to
abnormal development of derivatives of the first arch. This condition is characterized by
downward sloping palpebral fissures; hypoplasia of the malar and mandibular bones;
macrostomia; high or cleft palate; abnormally shaped, low-set ears; and unusual hair growth
patterns. Insufficient migration of neural crest cells (choice E) is an important factor as well.
When the palatine shelves fail to join together (choice D), cleft palate results. Cleft lip and
cleft palate commonly co-occur.

103

212. Which of the following is the embryologic precursor of the fibrous remnant that runs in a
fissure on the visceral surface of the liver?
A. Ductus venosus
B. Hepatic portal vein
C. Lateral splanchnic artery
D. Ligamentum venosum
E. Vitelline vein
The correct answer is A. The structure described is the ligamentum venosum (choice D),
which is derived from the ductus venosus.
The hepatic portal vein (choice B) is derived from the vitelline vein (choice E).
The lateral splanchnic arteries (choice C) arise from each side of the dorsal aorta. They
supply intermediate mesoderm and derivatives and give rise to renal, suprarenal, phrenic,
and testicular or ovarian arteries.
213. Which of the following veins empties into the left renal vein?
A. Hepatic
B. Left suprarenal
C. Right gonadal
D. Right renal
The correct answer is B. The left suprarenal vein empties into the left renal vein, which
crosses the vertebral column to reach the inferior vena cava. The left renal vein also receives
the left gonadal vein.
Hepatic veins (choice A) convey blood from the liver to the inferior vena cava as it
approaches the caval orifice of the diaphragm.
The right gonadal vein (testicular in male, ovarian in female; choice C) drains directly to the
inferior vena cava.
The left and right (choice D) renal veins enter the inferior vena cava at vertebral level L2.
214. The postganglionic signals carrying the impulses to constrict arterioles are transmitted
along which of the following fiber types?
A. A- fibers
B. B fibers
C. C fibers
D. Ia fibers
E. Ib fibers
The correct answer is C. There are two systems currently used for classifying nerve fibers.
The first system groups both sensory and motor fibers together, describing A-, A-, A-, A, B, and C fibers. Another system relates only to sensory fibers, describing Ia, Ib, II, III, and
IV categories. Both classification schemes begin with large, myelinated fibers, progressing to
finer, unmyelinated fibers.

104

The C fiber (or IV fibers) is the only type of fiber that is unmyelinated. Remember that
preganglionic neurons are myelinated, but postganglionic neurons are unmyelinated. Neurons
that carry slow pain and temperature information are also classified as C fibers. See the table
below for more information.
A- or Ia (choice D) Alpha motor neurons, primary afferents of muscle spindles
A- or Ib (choice E) Golgi tendon organ afferents, touch and pressure
A- or II Secondary afferents of muscle spindles, touch and pressure
A- Gamma motor neurons
A- (choice A) Touch, pressure, pain and temperature (fast)
B (choice B) Preganglionic autonomic, visceral afferents
C (choice C) IV Postganglionic autonomic, pain and temperature (slow)
215. Injury to which nerve would result in wrist drop, and inability to make a tight fist, even
though all finders can be flexed?
A. Axillary nerve
B. Long thoracic nerve
C. Median nerve
D. Musculocutaneous nerve
E. Radial nerve
The correct answer is E. Physiologically, the distribution of the radial nerve is as follows:
extensor muscles on the arm and forearm (triceps bracii, brachioradialis, extensor carpi
radials, and extersor carpi ulnaris), ditigal extensors and abductor pollicis, and skin over the
posterolateral surface of the arm. The radial nerve lies in the musculospiral groove of the
humerus and is subject to injury in association with a fracture of the midshaft of the humerus.
The radial nerve innervates the extensor muscles of the forearm, including the muscles that
extend the wrist; paralysis of these muscles results in a wrist drop. Although the muscles
responsible for flexing the digits are not innervated by the radial nerve, the making of a tight
fist requires that the wrist be stabilized with the wrist extensors.
The axillary nerve (choice A) leaves the axilla through its posterior wall and is not located in
the region of the midshaft of the humerus. The axillary nerve innervates the deltoid and the
teres minor muscles, neither of which have any function in the hand.
The long thoracic nerve (choice B) lies against the chest wall and is not subject to injury as a
result of a humeral fracture. The long thoracic nerve innervates the serratus anterior muscle.
The median nerve (choice C) passes though the arm, but is not in close contact with the
humerus in the midshaft region. The nerve is separated from the bone by the brachialis
muscle. The median nerve innervates many muscles of the anterior compartment of the
forearm responsible for flexion of the wrist and digits.
The musculocutaneous nerve (choice D) is not in contact with the humerus. It innervates the
muscles in the anterior compartment of the arm. These muscles have no function in the hand.

105

216. After falling on his laterally outstretched arm, a patient suffered a dislocation of the
glenohumeral joint. Which of the following nerves is most likely to have been injured from this
dislocation?
A. Axillary nerve
B. Dorsal scapular nerve
C. Lateral pectoral nerve
D. Medial pectoral nerve
E. Suprascapular nerve
The correct answer is A. When the head of the humerus dislocates from the glenohumeral
joint, it exits inferiorly, where the joint capsule is the weakest. Immediately inferior to the
glenohumeral joint, the axillary nerve exits from the axilla by passing through the
quadrangular space. At this location, the downward movement of the head of the humerus
can stretch the axillary nerve. The axillary nerve innervates the deltoid muscle after leaving
the axilla.
The dorsal scapular nerve (choice B) passes along the medial border of the scapula to
innervate the rhomboid muscles. The nerve does not pass in the region of the glenohumeral
joint.
The lateral and medial pectoral nerves (choices C and D) branch from the lateral and medial
cords of the brachial plexus, respectively, and exit through the anterior wall of the axilla to
innervate the pectoralis major and minor. These nerves do not pass in the region of the
glenohumeral joint.
The suprascapular nerve (choice E) is a branch of the upper trunk of the brachial plexus and
passes over the superior border of the scapula to innervate the supraspinatus and
infraspinatus muscles. This nerve does not pass in the region of the glenohumeral joint.
217. Which of the following areas of the heart have most likely suffered ischemic necrosis
following a myocardial infarct due to blockage of the left circumflex artery in a patient that has
left dominant coronary circulation?
A. Apex of left ventricle and anterior portion of septum
B. Lateral left ventricular wall and posterior portion of the septum
C. Lateral wall of the left ventricle only
D. Posterior portion of the septum only
E. Right ventricular wall
The correct answer is B. A right dominant coronary circulation is present when the posterior
descending branch originates from the right coronary artery (80% of individuals). On the
contrary, the posterior descending artery originates from the left circumflex artery in a left
dominant circulation (20% of individuals). The posterior descending branch gives blood to the
posterior half of the interventricular septum. Occlusion of the left circumflex artery in a left
dominant circulation will therefore lead to ischemic necrosis in the left ventricular wall and the
posterior interventricular septum.
The apex of the left ventricle (choice A) is dependent on the anterior descending branch;
thus, occlusion of the left circumflex does not affect this portion of the left ventricle.
Infarction of the lateral (free) wall alone (choice C) will result from occlusion of the circumflex
in a right dominant circulation.

106

An isolated infarct of the posterior interventricular septum (choice D) arises from occlusion of
the posterior descending branch.
Isolated infarcts of the right ventricular wall (choice E) are very rare and would be caused by
occlusion of branches of the right coronary artery.
218. A man is injured when a bony fragment penetrates the lateral portion of the dorsal
columns in his neck. Which of the following functions would most likely be affected by a lesion
at this site?
A. Fine motor control of the ipsilateral fingers
B. Motor control of the contralateral foot
C. Sweating of the ipsilateral face
D. Proprioception from the ipsilateral leg
E. Vibratory sense from the ipsilateral arm
The correct answer is E. At this level, the lateral portion of the dorsal columns (funiculus) is
composed of the fasciculus cuneatus. Axons carrying tactile, proprioceptive, and vibratory
information from the ipsilateral arm enter the spinal cord via the dorsal root, ascend the cord
in the fasciculus cuneatus, and synapse in the nucleus cuneatus of the caudal medulla.
Secondary neurons from this nucleus give rise to internal arcuate fibers, which decussate and
ascend to the thalamus (ventral posterolateral nucleus, VPL) as the medial lemniscus.
Tertiary neurons from the VPL project to the ipsilateral somatosensory cortex. Because the
fibers that carry this information do not cross until they reach the medulla, damage to the
fasciculus cuneatus would result in a deficit in tactile, proprioceptive, and vibratory sense in
the ipsilateral arm.
Fine motor control of the fingers (choice A) would be carried principally by the ipsilateral
lateral corticospinal tract in the lateral funiculus of the cord.
Motor control of the contralateral foot (choice B) is carried by the ipsilateral corticospinal tract
in the lateral funiculus of the cord.
Hemianhidrosis (lack of sweating) over half of the face (choice C) could be produced by
interruption of sympathetic innervation to the face, which projects from the hypothalamus to
the intermediolateral cell column at levels T1 and T2. It descends in the lateral funiculus of the
cord. Interruption of this tract results in Horner's syndrome (miosis, ptosis, and
hemianhidrosis).
Proprioception from the ipsilateral leg (choice D) is carried by the fasciculus gracilis in the
medial part of the dorsal columns.
219. Damage from extension of a tumor in the body of the right lateral ventricle would most
likely affect which other brain structures?

A. Caudate nucleus
B. Cerebellum
C. Hippocampus
D. Hypothalamus
E. Pons

107

The correct answer is A. Tumors of the ventricular system of the brain can affect the brain
tissue either directly, via pressure on or invasion into a physically close structure, or indirectly,
by obstructing cerebral spinal fluid (CSF) flow and causing hydrocephalus. The caudate
nucleus is a C-shaped structure that composes part of the wall of the lateral ventricle
throughout its extent. The only structure listed that is adjacent to the body of the lateral
ventricle and would therefore be directly affected by the large tumor described in the question
is the caudate nucleus. Anatomically, the caudate nucleus has a massive head and a slender
curving tail that follows the curve of the lateral ventricle. The head of the caudate nucleus lies
superior to the lentiform nucleus.
The cerebellum (choice B) overlies the fourth ventricle.
The hippocampus (choice C) is adjacent to the inferior (temporal) horn of the lateral ventricle.
The hypothalamus (choice D) abuts the third ventricle.
The pons (choice E) forms part of the floor of the fourth ventricle.
220. Examination of a karyotype taken from a metaphase preparation reveals the presence of
an extra chromosome. Which of the following is the most common mechanism of producing
this phenomenon?
A. Balanced translocation
B. Chromosomal breakage
C. Fertilization by two sperm
D. Nondisjunction
E. Unbalanced translocation
The correct answer is D. Nondisjunction can occur in both meiosis and mitosis and refers to
a failure of paired chromosomes to separate and go to different daughter cells. When this
happens, one daughter cell gets an extra chromosome, and the other daughter cell is "short"
one chromosome.
A balanced translocation (choice A) is an exchange of genetic material between
nonhomologous chromosomes that preserves all critical genetic material.
Chromosomal breakage (choice B) produces fragmented chromosomes and can contribute
to tumorigenesis.
Fertilization by two sperm (choice C) produces triploidy and is seen in many spontaneously
aborted fetuses.
An unbalanced translocation (choice E) occurs when nonhomologous chromosomes
exchange genetic material with a net loss or gain of critical genetic material.
221. To anesthetize an area under the nailbed of the index finger, which nerve should be
injected?
A. Axillary
B. Median
C. Musculocutaneous
D. Radial
E. Ulnar

108

The correct answer is B. The tumor in question is probably a benign glomus tumor, which is
notorious for producing pain far out of proportion to its small size. The question is a little tricky
because the most distal aspect of the dorsal skin of the fingers, including the nail beds, is
innervated by the palmar digital nerves rather than the dorsal digital nerves. Specifically, the
median nerve, through its palmar digital nerves, supplies the nail beds of the thumb, index
finger, middle finger, and half the ring finger.
The axillary nerve (choice A), musculocutaneous nerve (choice C), and radial nerve (choice
D) do not supply the nail beds. The radial nerve does supply the more proximal skin of the
back of the index finger.
The ulnar nerve (choice E) supplies the nail beds of the small finger and half of the ring
finger.
222. An individual has an eye that is persistently directed toward his nose. A lesion of which
of the following nerves could produce this finding?
A. CN III
B. CN IV
C. CN V
D. CN VI
E. CN VII
The correct answer is D. Cranial nerve VI is the abducens nerve, which supplies the
abductor of the eye, the lateral rectus. A paralysis of the lateral rectus leads to unopposed
adduction, causing the eye to point toward the nose.
Cranial nerve III (choice A) is the oculomotor nerve, which supplies all of the muscles of the
eye, except the superior oblique and lateral rectus. Paralysis of III would impair adduction, not
abduction, of the eye. The eye would tend to rotate downward and outward.
Cranial nerve IV (choice B) is the trochlear nerve, which supplies the superior oblique
muscle. This muscle serves to depress and abduct (down and out) the eyeball. Paralysis of IV
tends to produce double vision but does not cause an obvious deficit in conjugate gaze
without careful testing.
Cranial nerve V (choice C) is the trigeminal nerve, which is a mixed sensory and motor nerve
that supplies the face. It provides sensory innervation to the face and innervates the muscles
of mastication. It does not innervate the eye muscles.
Cranial nerve VII (choice E) is the facial nerve, which innervates the muscles of facial
expression but not the muscles of the orbit. CN VII is additionally involved in salivation,
lacrimation, taste, and general sensation from the external ear.
223. The patient closes both of his eyelids in response to the left eye being touched with a
thin wisp of cotton as he looks to the right. Which of the following cranial nerves is responsible
for the motor limb of this reflex?
A. Abducens
B. Facial
C. Optic
D. Trigeminal
E. Trochlear

109

The correct answer is B. The corneal reflex is tested by touching the cornea of one eye with
a cotton wisp; this causes both eyes to close. The afferent, or sensory, component of the
corneal reflex is mediated by the ophthalmic division of the ipsilateral trigeminal nerve (V-1).
The efferent, or motor, component is mediated by the facial nerve (CN VII), bilaterally.
The abducens nerve (CN VI; choice A) innervates the lateral rectus muscles, which abduct
the eyes.
The optic nerve (CN II; choice C) is responsible for vision, providing the afferent limb of the
pupillary light reflex. The Edinger Westphal nucleus mediates part of this reflex.
The trigeminal nerve (CN V; choice D) is responsible for the afferent limb of the corneal
reflex. It also innervates the muscles of mastication and provides sensory innervation to the
face.
The trochlear nerve (CN IV; choice E) innervates the superior oblique muscles, which
depress, intort, and abduct the eyes.
224. Which of the following veins may be anastomosed to accomplish a porto-caval shunt?
A. Left renal vein and left testicular vein
B. Right renal vein and right suprarenal vein
C. Splenic vein and left renal vein
D. Superior mesenteric vein and inferior mesenteric vein
E. Superior mesenteric vein and splenic vein
The correct answer is C. The splenic vein drains directly into the portal vein. The left renal
vein drains directly into the inferior vena cava. Anastomosis of these veins would allow blood
from the portal vein to drain retrograde though the splenic vein into the renal vein and then
into the inferior vena cava.
The left renal vein (choice A) drains directly into the inferior vena cava. The left testicular vein
drains directly into the left renal vein. Thus, these veins are already in communication, and
neither vein is part of the portal venous system.
The right renal vein (choice B) drains directly into the inferior vena cava. The right suprarenal
vein also drains directly into the inferior vena cava. Thus, neither vein is part of the portal
venous system.
The superior mesenteric vein (choice D) drains directly into the portal vein. The inferior
mesenteric vein drains into the splenic vein, which then drains into the portal vein. Thus,
neither vein is part of the caval venous system. The superior mesenteric vein (choice E)
drains directly into the portal vein. The splenic vein also drains directly into the portal vein.
Thus, neither vein is part of the caval venous system.
225. In a histological section of a normal ovary, an oocyte is surrounded by several layers of
follicular cells. A small antrum is present. Which of the following is the correct term for the
entire structure, composed of the oocyte, follicular cells, and antrum?
A. Corpus luteum
B. Graafian follicle
C. Primary follicle
D. Primordial follicle
E. Secondary follicle

110

The correct answer is E. Follicles in different stages of maturation have different


appearances. The most primitive follicles, primordial follicles (choice D), are inactive reserve
follicles that contain primary oocytes (arrested in prophase of the first meiotic division)
surrounded by a single layer of flattened follicular cells. Primary follicles (choice C), the next
stage, are slightly larger and contain a central oocyte surrounded by one or several cuboidal
follicular cells. When several small spaces in the follicular mass fuse to form the antrum
(follicular cavity), the follicle is termed a secondary follicle (choice E). The secondary follicles
continue to enlarge and develop a more complex structure that includes cumulus oophorus,
corona radiata, theca interna, theca externa, and zona pellucida. The Graafian follicle (choice
B) is the mature form of the follicle, which extends through the entire cortex and bulges out at
the ovarian surface. After it ruptures and releases the ovum, the corpus luteum (choice
A)develops and secretes progesterone as the cells of the follicle and the theca interna cells
enlarge, become epithelioid, and secrete estrogen. If pregnancy occurs, it is maintained
throughout the pregnancy; if pregnancy does not occur, the corpus luteum eventually
degenerates.
226. Within which of the following peritoneal structures is the cystic artery located?
A. Falciform ligament
B. Gastrocolic ligament
C. Gastrohepatic ligament
D. Hepatoduodenal ligament
E. Splenorenal ligament
The correct answer is D. The hepatoduodenal ligament is the portion of the lesser omentum
that connects the liver to the first part of the duodenum. Within the hepatoduodenal ligament
are found the proper hepatic artery and its branches, the common bile duct and its branches,
and the portal vein. The cystic artery is usually a branch of the right hepatic artery, which is a
branch of the proper hepatic artery.
The falciform ligament (choice A) is a mesentery that connects the liver to the anterior
abdominal wall. Within the free edge of the falciform ligament is found the round ligament of
the liver (ligamentum teres hepatis), the adult remnant of the umbilical vein.
The gastrocolic ligament (choice B) is the portion of the greater omentum between the
greater curvature of the stomach and the transverse colon. The gastroepiploic arteries lie
within the gastrocolic ligament along the greater curvature of the stomach.
The gastrohepatic ligament (choice C) is the portion of the lesser omentum between the liver
and the lesser curvature of the stomach. The right and left gastric arteries are within the
gastrohepatic ligament along the lesser curvature of the stomach. The splenorenal ligament
(choice E) is the mesentery that connects the spleen to the posterior abdominal wall. The
splenic artery and splenic vein are within the splenorenal ligament.
227. While performing a tonsillectomy, the surgeon accidentally damages the
glossopharyngeal nerve. Which of the following functional losses is likely to result from this
injury?
A. Loss of sensation on the posterior 1/3 of the tongue
B. Loss of taste on the anterior 2/3 of the tongue
C. Paralysis of the constrictor muscles of the palate
D. Paralysis of the muscles of the soft palate
E. Paralysis of the muscles of the tongue

111

The correct answer is A.The glossopharyngeal nerve, or cranial nerve IX, lies in the tonsillar
fossa. This nerve provides general sensory innervation to the mucosa of the pharynx, and
general sensory and taste sensation to the mucosa of the posterior 1/3 of the tongue. The
nerve also innervates the stylopharyngeus muscle, but this innervation occurs proximal to the
point at which the nerve crosses the tonsillar fossa. The full anatomical picture of the
glossopharyngeal nerve is as follows: It originates from the posterior 1/3 of the tongue, part of
the pharynx and palate, as well as the carotid arteries of the neck (sensory). It passes through
the jugular foramen of the temporal bone to foramen lacerum between the occipital and
temporal bones. The destination is to the sensory nuclei of the medulla oblongata, pharyngeal
muscles involved in swallowing, as well as the parotid.
Taste sensation to the anterior 2/3 of the tongue (choice B) is provided by the facial nerve.
The chordae tympani branch of the facial nerve travels with the lingual nerve to reach the
tongue.
The constrictor muscles of the pharynx (choice C) are innervated by the vagus nerve. The
vagus nerve fibers contribute to the pharyngeal plexus.
The muscles of the soft palate (choice D) are mostly innervated by the vagus nerve. The
tensor palati is innervated by the trigeminal nerve (V3).
The muscles of the tongue (choice E) are innervated by the hypoglossal nerve.
228. If the lateral wall of the left ventricle receives insufficient blood supply, which artery is
most likely involved?
A. Left anterior descending
B. Left circumflex
C. Left main coronary
D. Right coronary
The correct answer is B. In some patients with coronary artery disease, thallium stress tests
may be performed instead of cardiac catheterization to determine the vessels involved and
the extent of occlusion. The left circumflex (LCx) branch supplies the lateral wall of the left
ventricle; in 10% of the population, it also supplies the posterior wall and AV node.
The left anterior descending (LAD) branch of the left coronary artery (choice A) supplies the
anterior wall of the left ventricle and the anterior portion of the interventricular septum.
The left main coronary artery (choice C) gives rise to both LCx and the LAD.
The right coronary artery (RCA; choice D) supplies the right ventricle; in 90% of the
population, it supplies the AV node and posterior and inferior walls of the left ventricle.
229. Which of the following products would mast cells be most likely to secrete?
A. Bradykinin
B. Complement factor 3a
C. Histamine
D. Interleukin-2
E. Nitric oxide

112

The correct answer is C. The cells in question are mast cells, which play an important role in
IgE-mediated allergic responses. It is also important to note that histamine is found in
basophils. They are a normal (minor) constituent of dermal skin and are most definitely
identified with stains, such as the Giemsa stain, that highlight the granularity of their
cytoplasm (mast cells are closely related to blood basophils). Mast cells secrete histamine,
serotonin, many leukotrienes, and platelet-aggregating factor (PAF). Their histamine secretion
after IgE stimulation helps to trigger the acute inflammatory part of the allergic response.
Bradykinin (choice A) is a nonapeptide found in the plasma. It is a potent vasodilator and
stimulates pain receptors.
Complement factor 3a (choice B) is a plasma protein that induces vascular leakage as part of
the acute inflammatory response.
Interleukin-2 (choice D) is secreted by lymphocytes and augments the immune response.
Nitric oxide (choice E) is released by macrophages and endothelium. It causes vasodilation
and cytotoxicity.
230. Which of the following gastrointestinal hormones is structurally related to secretin?
A. Cholecystokinin
B. Gastrin
C. Glucagon
D. Somatostatin
E. Substance P
The correct answer is C. Glucagon, secretin, and vasoactive intestinal polypeptide (VIP) are
all structurally related.
Cholecystokinin (choice A) and gastrin (choice B) form another family of related hormones.
Neither somatostatin (choice D) nor substance P (choice E) are structurally related to
secretin. In addition to their role in the gastrointestinal system, both hormones are also
present in the brain.
231. In which of the following regions of the liver is the oxidase P450 system located?
A. Bile ducts
B. Intermediate zone
C. "Ito" cells
D. Pericentral vein zone
E. Periportal zone
The correct answer is D. The liver can be divided into three zones. Zone 1 is periportal,
zone 2 is intermediate (between 1 and 3), and zone 3 is associated with the central vein.
Zone 3 contains the P450 oxidase enzyme system and is most sensitive to injury. The CP450
system is very important to be familiar with because it is the major site of drug interaction. For
example, the macrolides, which are commonly used in dentistry in penicillin-allergic patients
are potent hepatic enzyme inhibitors, as are antifungals like ketoconazole and fluconazole.
The P450 system is not located in the bile ducts (choice A).

113

The intermediate zone (choice B), or zone 2, is the second area most sensitive to ischemic
injury.
"Ito" cells (choice C) are the fat-containing mesenchymal cells located in the space of Disse.
This is the site of vitamin A storage.
The periportal zone (choice E), or zone 1, is the area most sensitive to toxic injury. This is the
area that shows infiltration with hepatitis.
232. Which part of the hypothalamus controls satiety, and would lead to obesity if destroyed?
A. Lateral nucleus
B. Septal nucleus
C. Suprachiasmatic nucleus
D. Supraoptic nucleus
E. Ventromedial nucleus
The correct answer is E. The ventromedial nucleus is thought to be the satiety center of the
brain. Bilateral destruction leads to hyperphagia, obesity, and savage behavior. Stimulation
inhibits the urge to eat.
Destruction of the lateral nucleus (choice A) results in starvation, whereas stimulation of this
nucleus induces eating.
Destruction of the septal nucleus (choice B) produces aggressive behavior.
The suprachiasmatic nucleus (choice C) receives direct input from the retina and plays a role
in controlling circadian rhythms.
The supraoptic nucleus (choice D), along with the periventricular nucleus, regulates water
balance and produces antidiuretic hormone (ADH) and oxytocin.
233. A premature infant develops a progressive difficulty with breathing over the first few days
of life. Deficient surfactant synthesis by which of the following cell types may have contributed
to the infant's respiratory problems?
A. Alveolar capillary endothelial cells
B. Bronchial mucous cells
C. Bronchial respiratory epithelium
D. Type I pneumocytes
E. Type II pneumocytes
The correct answer is E. This child has neonatal respiratory distress syndrome (hyaline
membrane disease), which is caused by the inability of the immature lungs to synthesize
adequate amounts of surfactant. Surfactant, which reduces surface tension, helps keep
alveoli dry, and aids in expansion of the lungs, is synthesized by type II pneumocytes.
Alveolar capillary endothelial cells (choice A) are important in maintaining the capillary
structure and permitting flow of gases into and out of the blood stream.
Bronchial mucous cells (choice B) produce the usually thin (in healthy individuals) coat of
mucus that lines the bronchi.

114

The ciliated bronchial respiratory epithelium (choice C) is responsible for moving the dustcoated mucus layer out of the bronchi.
Type I pneumocytes (choice D) are the squamous cells that line alveoli and permit easy gas
exchange. These cells tend to be immature (and thick) in premature infants but do not
produce surfactant.
234. Inability to extend the hand at the wrist is often accompanied by loss of sensation to
which area?
A. Lower part of the back of the little finger
B. Lower part of the back of the thumb
C. Palmar aspect of the forefinger
D. Palmar aspect of the little finger
E. Thumbnail bed
The correct answer is B. Associate "wrist drop" with a radial nerve lesion. The radial nerve
also provides sensation to the dorsal aspect of the radial side of the hand, including the lower
part of the dorsal aspect of the thumb and all the fingers, except for the little finger and half of
the ring finger.
The lower part of the back of the little finger (choice A) is supplied by the ulnar nerve.
The palmar aspects of all of the fingers (choices C and D) are supplied by the median nerve,
as are the nailbeds, including the thumbnail bed (choice E).
235. At which of the following locations might a penetrating wound to the heart damage the
AV node?
A. Apex of the heart
B. Interatrial septum
C. Interventricular septum
D. Wall of the right atrium
E. Wall of the left atrium
The correct answer is B. This question is essentially asking "where is the AV node located
on the heart?" The atrioventricular (AV) node is in the subendocardium of the interatrial
septum. The AV node can also be described as being within the floor of the right atrium near
the opening of the coronary sinus. From the AV node, the Purkinje fibers of the
atrioventricular bundle enter the interventricular septum to carry impulses to the ventricle. The
function of the AV node is to retard the conduction of the cardiac impulses so that ventricular
systole occurs after atrial systole.
The apex of the heart (choice A) is composed of ventricular wall musculature of the left
ventricle. There is no nodal tissue in this region.
The interventricular septum (choice C) contains the common bundle (AV bundle) and the
right and left bundle branches of the cardiac conduction system. These bundles are
composed of Purkinje fibers, which are specialized cardiac muscle cells.
The sinoatrial (SA) node is located in the right atrial wall (choice D), near the entrance of the
superior vena cava. The SA node contains specialized cardiac muscle cells that depolarize
more rapidly than do typical cardiac muscle cells, thereby serving as the pacemaker of the
heart. The wall of the left atrium (choice E) does not contain any nodal cells.

115

236. Following thyroid surgery, hoarseness and difficulty speaking can be attributed to
damage to a branch of which cranial nerve?
A. Facial
B. Glossopharyngeal
C. Hypoglossal
D. Trigeminal
E. Vagus
The correct answer is E. The recurrent laryngeal nerves are branches of the vagus (CN X)
and supply all intrinsic muscles of the larynx, except the cricothyroid. The right recurrent
laryngeal nerve recurs around the right subclavian artery. The left recurrent laryngeal nerve
recurs in the thorax around the arch of the aorta and ligamentum arteriosum. Both nerves
ascend to the larynx by passing between the trachea and esophagus, in close proximity to the
thyroid gland. The recurrent laryngeal nerves are therefore particularly vulnerable during
thyroid surgery, and damage may cause extreme hoarseness.
The facial nerve (choice A) innervates the muscles of facial expression, the stapedius
muscle, and the lacrimal, submandibular, and sublingual glands. It also mediates taste
sensation from the anterior two-thirds of the tongue.
The glossopharyngeal nerve (choice B) innervates the stylopharyngeus muscle and the
parotid gland. Visceral afferents supply the carotid sinus baroreceptors and carotid body
chemoreceptors and mediate taste from the posterior one-third of the tongue. Somatosensory
fibers supply pain, temperature, and touch information from the posterior one-third of the
tongue, upper pharynx, middle ear, and eustachian tube.
The hypoglossal nerve (choice C) innervates the intrinsic muscles of the tongue, the
genioglossus, hypoglossus, and styloglossus muscles.
The trigeminal nerve (choice D) receives sensory information from the face and also
innervates the muscles of mastication.
237. A knife wound to the face selectively damages the facial nerve. Salivation from which of
the following would be impaired?
A. Parotid and sublingual glands
B. Parotid and submandibular glands
C. Parotid gland only
D. Sublingual gland only
E. Submandibular and sublingual glands
The correct answer is E. The facial nerve (CN VII) is intimately related anatomically to the
parotid gland but controls salivation from both the submandibular and sublingual glands. The
sublingual salivary glands are located beneath the mucous membrane on the floor of the
mouth. The submandibular salivary glands are found in the floor of the mouth along the inner
surface of the mandible within the manibular groove.
The parotid gland (choices A, B, and C) is innervated by the glossopharyngeal nerve (CN
IX). The parotid glands are located inferiorly to the zygomatic arch beneath the skin covering
the lateral and posterior surfaces of the mandible. Although the facial nerve most often runs
through the parotid gland, it does not supply any innervation there.

116

238. Which area of the stomach has numerous, normal cuboidal-to-columnar cells with apical,
membrane-bound secretion granules in the gastric glands?
A. Cardiac region
B. Columns of Morgagni
C. Fundic region
D. Greater omentum
E. Pyloric region
The correct answer is C. The pathologist saw normal chief cells, which are abundant in the
body and fundus of the stomach. Chief cells secrete pepsinogen, which is stored in apical
membrane-bound granules. The body and fundus of the stomach contain high concentrations
of four other types of cells in the epithelium. The parietal (oxyntic) cells are large, pyramidal,
and acidophilic with central nuclei (like a "fried egg"). They make and secrete hydrochloric
acid. The mucous neck cells secrete mucus and appear clear. The enteroendocrine cells
have affinity for silver stains and exhibit a positive chromaffin reaction; these cells synthesize
amines, polypeptides, or proteins.
The cardiac region (choice A) is a narrow, circular band at the transition between the
esophagus and stomach, consisting of shallow gastric pits and mucous glands. It does not
normally contain an abundance of chief cells.
The columns of Morgagni (choice B) are found in the rectum, not in the stomach. These are
mucous membrane infoldings in the submucosa of the proximal anal canal. They would not
contain chief cells.
The greater omentum (choice D) is a four-layered fold of peritoneum that hangs from the
greater curvature of the stomach and attaches to the transverse colon. It would not contain
chief cells.
The pyloric region (choice E) has deep gastric pits into which tubular glands open. The
predominant secretion is mucus. It does not normally contain an abundance of chief cells.
Note that in this question you could have automatically eliminated choices B and D because
they are not gastric structures. If nothing else, you have improved your guessing odds to
33%.
239. A man suffers a direct blow to the right eye, producing eye pain and severe bleeding
from the nose. X-rays reveal a blowout fracture of the floor of the orbit. Involvement of which
of the following structures is suggested by the patient's symptoms?
A. Ethmoid air cells
B. Maxillary sinus
C. Oral cavity
D. Pterygopalatine fossa
E. Sphenoid sinus
The correct answer is B. The floor of the orbit is also the roof of the maxillary sinus. The
bone separating the orbit from the maxillary sinus is frequently quite thin and may fracture
from increased intraorbital pressure caused by a direct blow to the eye. The subsequent
bleeding into the maxillary sinus will result in blood draining from the sinus into the nasal
cavity. The maxillary sinuses are the largest sinuses in the skull. These sinuses lighten the
portion of the maxillae above the teeth and produce mucous secretions that help to "flush" the
interior surfaces of the nasal cavities.

117

The ethmoid air cells (choice A) are medial to the orbit. This bone is also frequently quite
thin.
The oral cavity (choice C) is separated from the orbit by the maxillary sinus.
The pterygopalatine fossa (choice D) is posteromedial to the orbit. The medial portion of the
inferior orbital fissure communicates between the orbit and the pterygopalatine fossa.
The sphenoid sinus (choice E) is posteromedial to the orbit. There is no communication
between the sphenoid sinus and the orbit.
240. A parent is startled to find that squeezing her infant's shoulders with her hand can bring
them nearly together in front of the body. Congenital absence of which of the following bones
should be suspected?
A. Clavicles
B. First ribs
C. Humeri
D. Scapulae
E. Sternum
The correct answer is A. Congenital absence of the clavicles is a fairly common and not
particularly troublesome anomaly, although affected children have some trouble with sports. It
can be confirmed radiologically. In rare (particularly, obese) patients, the defect is not
diagnosed until a routine chest x-ray performed for other reasons demonstrates the anomaly.
Congenital defects in ribs (choice B) can occur but would not allow the shoulders to be
brought together.
Congenital absence of the humeri (choice C) would produce absent or markedly shortened
upper arms.
Congenital anomalies of the scapulae (choice D) are rare and would affect posterior shoulder
movement.
Congenital absence of part of the sternum (choice E) produces a hole in the lower part of the
body of the sternum; complete absence is rare.

118

Physiology
1. Serum chemistry studies of a patient reveal that her aspartate aminotransferase (AST) is
markedly elevated, whereas her alanine aminotransferase (ALT), gamma-glutamyl
transpeptidase (GGT), and alkaline phosphatase are all within normal limits. Disease of which
of the following organs would be most likely to cause this serum enzyme pattern?
A. Colon
B. Duodenum
C. Heart
D. Pancreas
E. Stomach
The correct answer is C. Myocardial infarction (MI) can cause AST elevation without accompanying
elevation of ALT or other liver enzymes. This is an important fact to remember because it may be the
first clue for heart disease in a patient who has an atypical presentation of MI (as is common in
women with MI). MI can be confirmed with measurement of the MB fraction of creatine phosphokinase
(CPK-MB). In addition, Troponin T and Troponin I can be diagnostic.
Unfortunately, diseases of the tubular organs of the gastrointestinal tract, including colon (choice A),
duodenum (choice B), and stomach (choice E), do not produce distinctive serum enzyme patterns.
Damage to the pancreas (choice D) is associated with elevated amylase levels.
2. A newborn infant has multiple hemorrhages. Clotting studies demonstrate an elevated
prothrombin time and elevated INR. An abnormality of which of the following biochemical
processes is likely present in this patient?
A. Conversion of homocysteine to methionine
B. Conversion of methylmalonyl CoA to succinyl CoA
C. Degradation of cystathionine
D. Formation of gamma-carboxyglutamate residues
E. Hydroxylation of proline
The correct answer is D. Deficiency of vitamin K produces a clotting disorder characterized by an
elevated prothrombin time and easy bleeding, particularly in neonates (hemorrhagic disease of the
newborn). The biochemical basis for this hemorrhagic tendency is that glutamate residues on Factors
II (Thrombin), VII, IX, and X must be converted to gamma-carboxyglutamate residues (in a vitamin Krequiring reaction) for optimal activity.
The conversion of homocysteine to methionine (choice A) requires vitamin B12. Vitamin B12
deficiency can result in the development of pernicious anemia. Conversion of methylmalonyl CoA to
succinyl CoA (choice B) requires vitamin B12.
Degradation of cystathionine (choice C) requires vitamin B6 and can result in the development of
mouth soreness, glossitis, cheilosis, and weakness.
Hydroxylation of proline (choice E) requires vitamin C. Vitamin C deficiency can cause easy bruising,
but will not prolong the prothrombin time. Vitamin C deficiency is commonly known as scurvy.
3. Where are the body's temperature regulation centers located?
A. Midbrain
B. Pons
C. Medulla
D. Hypothalamus
The correct answer is D. Temperature regulation centers and thirst and food intake regulatory
centers are located in the hypothalamus. Choice A - Midbrain contains the micturation center. Choice

B - Pneumotaxic centers are located in the pons. Choice C - The medulla contains the vasomotor
center, respiratory center, vomiting, swallowing, and coughing centers.
4. An animal is made diabetic by injection of a drug that destroys pancreatic cells. Removal
of which of the following organs would most likely produce a decrease in blood glucose
concentration in this animal?
A. Anterior pituitary
B. Colon
C. Gonads
D. Kidney
E. Pancreas
The correct answer is A. The anterior pituitary produces the following hormones: thyroid-stimulating
hormone (TSH), adrenocorticotropic hormone (ACTH), follicle stimulating hormone (FSH), leutinizing
hormone (LH), prolactin, and growth hormone (GH). Two of these secretions (GH, ACTH) affect the
sensitivity of peripheral tissues to the action of insulin. Growth hormone has a direct effect on liver and
muscle to decrease insulin sensitivity. This may be partly through a growth hormone-induced decline
in insulin receptors or to unknown postreceptor defects. In excess, growth hormone is "diabetogenic,"
and approximately 25% of patients with acromegaly have diabetes. ACTH indirectly has antiinsulin
effects by virtue of the cortisol secretion it evokes. Like growth hormone, cortisol also decreases
insulin sensitivity in peripheral tissues. A third anterior pituitary hormone, TSH, also tends to increase
blood glucose levels. In this case, the effect is probably mediated mostly through increased glucose
absorption by the gut. Patients with hyperthyroidism can sometimes exhibit a postprandial glucosuria
because of excessive intestinal glucose absorption. In diabetic animals, the removal of the anterior
pituitary may lower blood glucose by increasing tissue sensitivity to whatever insulin remains.
Removal of the colon (choice B) should have little effect on blood glucose because dietary glucose is
absorbed in the small intestine.
Sex steroids secreted by the gonads (choice C) have little effect on blood glucose concentration.
The kidney (choice D) plays an important role in reabsorbing filtered glucose. In diabetes, the tubular
reabsorption maximum is exceeded and glucose spills over into the urine. The loss of glucose in the
urine helps to reduce the severity of the plasma hyperglycemia. Removal of the kidneys would, if
anything, make the hyperglycemia worse. Pancreatectomy (choice E) would make the hyperglycemia
worse by removing the source of any remaining insulin.
5. Which of the following is most likely to decrease in the skeletal muscles of a healthy 22year-old woman during exercise?
A. Arteriolar resistance
B. Carbon dioxide concentration
C. Lactic acid concentration
D. Sympathetic nervous activity
E. Vascular conductance
The correct answer is A. The increase in muscle blood flow that occurs during exercise is caused
by dilation of the arterioles (i.e., decreased arteriolar resistance) attributed to the dilatory actions of
metabolic factors (e.g., adenosine, lactic acid, carbon dioxide) produced by the exercising muscles. In
normal skeletal muscles, the blood flow can increase as much as 20-fold during strenuous exercise.
Exercise causes the concentration of carbon dioxide (choice B) and lactic acid (choice C) to increase
in the muscles.
Mass discharge of the sympathetic nervous system (choice D) occurs throughout the body during
exercise, causing arterioles to constrict in most tissues. The arterioles in the exercising muscles,
however, are strongly dilated by vasodilator substances released from the muscles.

A decrease in vascular conductance (choice E) occurs when the vasculature is constricted.


Resistance and conductance are inversely related, so that a decrease in arteriolar resistance is
associated with an increase in arteriolar conductance.
6. Which of the following parameters is expected to increase in response to a 50% reduction in
sodium intake for a 2-month period?
A. Arterial pressure
B. Atrial natriuretic peptide release
C. Extracellular fluid volume
D. Renin release
E. Sodium excretion
The correct answer is D. Renin is an enzyme released by the juxtaglomerular cells when renal blood
pressure or pO2 declines. Renin converts angiotensinogen to angiotensin I. A reduction in sodium
intake leads to a decrease in extracellular fluid volume (choice C) and therefore a decrease in arterial
pressure (choice A). The decrease in arterial pressure stimulates renin release that in turn leads to
an increase in the formation of angiotensin II. The angiotensin II increases the renal retention of salt
and water (i.e., decreases sodium excretion, choice E), which returns the extracellular fluid volume
nearly back to normal.
Atrial natriuretic peptide (choice B) is released from the two atria of the heart as a result of an
increase in the extracellular fluid volume. A decrease in sodium intake therefore would tend to
decrease the release of atrial natriuretic peptide.
7. Which of the following hormones is most important in the initiation of gallbladder
contraction following a fatty meal?
A. CCK
B. Gastrin
C. GIP
D. Secretin
E. VIP
The correct answer is A. Cholecystokinin, or CCK, is synthesized in the duodenal and jejunal
mucosa and stimulates gallbladder contraction and pancreatic enzyme secretion. Other functions
include slowing of gastric emptying, an atrophic effect on the pancreas, and secretion of antral
somatostatin, which in turn decreases gastric acid secretion.
Gastrin (choice B) prepares the stomach and small intestine for food processing, including
stimulating secretion of HCl, histamine, and pepsinogen, increasing gastric blood flow, lower
esophageal sphincter tone, and gastric contractions. Gastric inhibitory peptide, or GIP (choice C),
stimulates pancreatic insulin secretion at physiologic doses and inhibits gastric acid secretion and
gastric motility at pharmacologic doses. Secretin (choice D) stimulates secretion of bicarbonatecontaining fluid from the pancreas and biliary ducts. Vasoactive intestinal polypeptide, or VIP (choice
E), relaxes intestinal smooth muscle and stimulates gut secretion of water and electrolytes.
8. During spermatogenesis, crossing over occurs during the meiotic division of which of the
following cells?
A. Primary spermatocytes
B. Secondary spermatocytes
C. Spermatids
D. Spermatogonia
E. Spermatozoa
The correct answer is A. Crossing over, a transposition of genetic information, occurs during the
first meiotic (reduction) division, when the primary spermatocyte divides to form two secondary

spermatocytes. This division does not consist of separation of sister chromatids after DNA replication,
but rather involves the separation of previously paired, homologous chromosomes. Crossing over
occurs during prophase of meiosis I.
The secondary spermatocyte (choice B) undergoes the second meiotic division, which results in four
spermatids (choice C), each with the haploid number of chromosomes. The spermatids are located
adjacent to the lumen of the seminiferous tubules and are distinguished by their small size. These
cells undergo no further division, but become transformed into mature spermatozoa (choice E)
through the prcess of spermiogenesis.
The spermatogonia (choice D) are the primitive germ cells. These cells give rise to the primary
spermatocytes through repeated mitotic divisions.
Remember the sequence of spermatogenesis:
Spermatogonia (2n) ? REPEATED MITOSES ? Primary spermatocytes (2n) ? FIRST MEIOTIC
DIVISION ? Secondary spermatocytes (n) ? SECOND MEIOTIC DIVISION ? Spermatids (n) ?
SPERMIOGENESIS ? Spermatozoa (n)
9. A decrease in which of the following is the most likely cause of peripheral edema in a
patient with long-term alcoholism and liver disease?
A. Capillary hydrostatic pressure
B. Interstitial colloid osmotic pressure
C. Interstitial hydrostatic pressure
D. Plasma colloid osmotic pressure
E. Precapillary arteriolar resistance
The correct answer is D. Osmotic pressure of a solution is an indicator of the force of water
movement resulting from its solute concentration. The higher the solute concentration of a solution,
the greater its osmotic pressure. The plasma colloid osmotic pressure is often low in alcoholics with
chronic liver disease (cirrhosis). The diseased liver cannot produce adequate amounts of albumin,
which leads to a decrease in the concentration of albumin in the plasma (i.e., hypoalbuminemia).
Because approximately 75% of the plasma colloid osmotic pressure can be attributed to the presence
of albumin in the plasma, the decrease in plasma albumin concentration that occurs in the latter
stages of cirrhosis often leads to peripheral edema. Cirrhosis also causes excess fluid to accumulate
in the peritoneal cavity as ascites. In the case of ascites, the edema results not only from
hypoalbuminemia, but also from portal vein obstruction (which increases capillary hydrostatic
pressure) and the obstruction of lymphatic drainage of the liver. In fact, ascites is observed more often
than peripheral edema in liver disease.
A decrease in capillary hydrostatic pressure (choice A) would tend to decrease fluid loss from the
capillaries, and thereby oppose the development of edema. A decrease in the colloid osmotic
pressure of the interstitial fluid (choice B) would decrease fluid loss from the capillaries, thereby
opposing the development of edema. A decrease in interstitial hydrostatic pressure (choice C) would
tend to increase fluid loss from the capillaries, but this cannot be considered a primary cause of
edema because the interstitial hydrostatic pressure actually increases when a tissue becomes
edematous. A decrease in precapillary arteriolar resistance (choice E), which means arteriolar
dilation, would increase capillary hydrostatic pressure and tend to cause edema. Decreased
precapillary arteriolar resistance in the peripheral vasculature, however, is not associated with
cirrhosis.
10. The medical record of a patient indicates a systolic murmur due to increase in afterload,
producing a pressure gradient between the ventricle and aorta during ejection. Which of the
following best describes the condition?
A. Aortic insufficiency
B. Aortic stenosis
C. Mitral insufficiency
D. Mitral stenosis

The correct answer is B.Aortic stenosis increases afterload and produces a pressure gradient
between ventricle and aorta during ejection. Aortic insufficiency (choice A) increases preload and
produces retrograde flow from the aorta to the ventricle, leading to a diastolic murmur. Mitral
insufficiency (choice C) increases volume and pressure in the atrium and ventricle, producing a
systolic murmur. Mitral stenosis (choice D) increases left-atrial volume and pressure, producing a
diastolic murmur.
11. At 25 weeks of pregnancy, an unidentified infection greatly compromises the viability of a
developing fetus. The level of which of the following hormones in the mother's blood is most
likely to be affected?
A.
B.
C.
D.
E.

Estriol
Free thyroxine
Human chorionic gonadotropin
Human chorionic somatomammotropin
Progesterone

The correct answer is A. Plasma levels of maternal estrogens during pregnancy depend on a
functioning fetus. The fetal adrenal cortex and liver produce the weak androgens, DHEA-S and 16-OH
DHEA-S, which are carried to the placenta by the fetal circulation. The placenta then desulfates the
androgens and aromatizes them to estrogens (16-OH DHEA-S, estriol) before delivery to the maternal
circulation. Estradiol and estrone increase approximately 50-fold during pregnancy, but estriol
increases approximately 1,000 fold. When estriol is assayed daily, a significant drop may be a
sensitive early indicator of fetal jeopardy. Total serum thyroxine concentration may be increased in
pregnancy because of an increase in circulating TBG resulting from increased estrogen. Free
thyroxine, however, (choice B) remains within the normal range because of feedback regulation. The
decline in estrogen with fetal compromise may gradually decrease serum thyroxine, but the free
thyroxine will remain unchanged. Human chorionic gonadotropin (choice C) and human chorionic
somatomammotropin (choice D) are both secreted by syncytiotrophoblasts of the placenta. As long
as placental function is intact, blood levels of these two hormones should not change with fetal
compromise. Placental secretion of progesterone (choice E) during pregnancy is also independent of
any fetal contribution. The placenta relies on maternal cholesterol for progesterone production. Fetal
death has no immediate influence on progesterone production by the placenta.
12. A healthy 20-year-old man deprived of water for several days has a plasma concentration
of antidiuretic hormone (ADH) 5 times greater than normal. Which of the following is the most
likely explanation for the increase in ADH concentration?
A. Decreased plasma aldosterone
B. Decreased plasma renin activity
C. Increased extracellular fluid volume
D. Increased left atrial pressure
E. Increased plasma osmolality
The correct answer is E. An obligatory loss of water from the body continues to occur even when a
person is deprived of water. This loss of water from the body tends to concentrate the extracellular
fluid, causing it to become hypertonic. Both the decrease in extracellular fluid (compare with choice
C) and the increase in osmolarity act as stimuli for increased thirst and increased secretion of ADH.
The decrease in extracellular fluid volume also tends to decrease arterial pressure that in turn
increases plasma renin activity (compare with choice B) and aldosterone levels in the plasma
(compare with choice A). Water deprivation tends to decrease left atrial pressure (compare with
choice D).
13. Which of the following indices would be expected to be decreased in a kidney donor after
full recovery from the removal of the kidney?
A. Creatinine clearance
B. Creatinine production
C. Daily excretion of sodium
D. Plasma creatinine concentration
E. Renal excretion of creatinine

The correct answer is A.Because creatinine is freely filtered by the glomerulus but not secreted or
reabsorbed to a significant extent, the renal clearance of creatinine is approximately equal to the
glomerular filtration rate. In fact, creatinine clearance is commonly used to assess renal function in the
clinical setting. When a kidney is removed, the total glomerular filtration rate decreases because 50%
of the nephrons have been removed, which causes the creatinine clearance to decrease. In turn, the
plasma creatinine concentration (choice D) increases until the rate of creatinine excretion by the
kidneys (choice E) is equal to the rate of creatinine production by the body. Recall that creatinine
excretion = GFR x plasma creatinine concentration. Creatinine excretion is therefore normal when
GFR is decreased following removal of a kidney because the plasma concentration of creatinine is
elevated. Creatinine is a waste product of metabolism. Creatinine production (choice B) is directly
related to the muscle mass of an individual, but is independent of renal function. The daily excretion of
sodium (choice C) is unaffected by the removal of a kidney. The amount of sodium excreted each
day by the remaining kidney exactly matches the amount of sodium entering the body in the diet.
14. An elevated level of which of the following substances would identify a specimen as
plasma rather than serum?
A. Albumin
B. Erythrocytes
C. Fibrinogen
D. Granulocytes
E. Serotonin
The correct answer is C. This is simply a definition What is the difference between serum and
plasma? Essentially, serum is derived from plasma by the extraction of fibrinogen and coagulation
factors II, V, and VIII. This can be achieved by allowing whole blood to clot, then removing the clot.
Albumin (choice A) is present in serum and plasma. Neither erythrocytes (choice B) nor
granulocytes (choice D) are present in either serum or plasma. Serotonin (choice E) levels may be
increased in serum because of the platelet breakdown that occurs during the extraction process.
Serotonin is normally found in the highest concentration in platelets, and in the enterochromaffin cells
and myenteric plexus of the gastrointestinal tract. The brain and the retina contain smaller amounts.
15. A dietary deficiency of which of the following is most strongly associated with a neural
tube defect at the base of the fetus's spine?
A. Calcium
B. Folate
C. Iron
D. Vitamin C
E. Vitamin K
The correct answer is B. Neural tube defects most commonly occur in the lumbosacral region,
typically resulting in motor and sensory deficits in the lower extremities, and bowel and bladder
dysfunction. This condition is now known to be associated with low maternal folate during the first 3-4
weeks of pregnancy, a time when many women may be unaware of their pregnancy. It is now
recommended that all women of childbearing age consume at least 400 grams of folic acid daily.
Calcium deficiency (choice A) can cause osteoporosis and osteopenia. Iron deficiency (choice C)
can cause iron deficiency anemia. Vitamin C deficiency (choice D) can cause scurvy. Vitamin K
deficiency (choice E) can cause a bleeding diathesis.
16. Which of the following substances can be converted to glucose to supply the needs of the
brain during starvation?
A. Acetoacetate
B. Acetone
C. Amino acids
D. Beta-hydroxybutyrate
E. Fatty acids

The correct answer is C. During starvation, the diet is inadequate to provide sufficient glucose to
maintain the brain, yet the brain requires glucose as an energy source. Glucose used in the brain
during starvation is synthesized from amino acids, primarily derived from muscle protein. This use of
amino acids in starvation leads to profound muscle wasting.
The ketone bodies (acetoacetate, choice A; acetone, choice B; and beta-hydroxybutyrate, choice D)
produced during starvation and diabetic ketoacidosis are derived from adipose triacylglycerols.
Although these compounds can be used in biochemical pathways in the brain, they cannot completely
replace glucose in that organ. Furthermore, glucose cannot be synthesized from these precursors.
Fatty acid (choice E) degradation cannot be used to produce glucose. It can be used, however, to
produce ketone bodies that can be used by the brain as a source of intermediates for some synthetic
pathways.
17. Absence of which of the following cell types is responsible for loss of vitamin B12
absorption?
A. Chief cells
B. G cells
C. Goblet cells
D. Mucous neck cells
E. Parietal cells
The correct answer is E. The parietal cells of the stomach produce intrinsic factor, a glycoprotein
that binds vitamin B12 in the lumen of the stomach and facilitates its absorption in the terminal ileum.
Patients without a stomach and those with pernicious anemia (autoimmune destruction of parietal
cells) require B12 replacement therapy. Recall that B12 deficiency will lead to megaloblastic anemia
and a blood smear with hypersegmented neutrophils. Note that parietal cells also synthesize and
secrete HCl. Chief cells (choice A) are responsible for secreting pepsinogen, the precursor to pepsin.
G cells (choice B) secrete gastrin, which stimulates secretion of acid by the parietal cells found in the
body and fundus of the stomach. Zollinger-Ellison syndrome is caused by a pancreatic or duodenal
tumor that secretes gastrin (a gastrinoma). It is characterized by the development of severe peptic
ulcer disease. Goblet cells (choice C) are part of the mucosa of the small intestine, not the stomach.
They produce glycoproteins (mucins) that protect and lubricate the lining of the intestine. Mucous
neck cells (choice D) secrete mucus and are located in the necks of the gastric glands.
18. Which of the following is most likely to relieve the symptoms caused by hyperventilation?
A. Breathing a 10% oxygen/90% nitrogen mixture
B. Breathing 100% nitrogen
C. Breathing in and out of a plastic bag
D. Intravenous administration of bicarbonate
E. Lying down
The correct answer is C. Hyperventilation os often associated with states of anxiety and can lead to
feelings of faintness, suffocation, tightness in the chest, and blurred vision. Individuals undergoing
such an attack may not be aware of overbreathing. The anxious, hyperventilating patient is "blowing
off" carbon dioxide, which lowers the arterial PCO2. Many of the symptoms associated with anxiety
attacks are probably caused by a decrease in cerebral blood flow secondary to low arterial PCO2.
Recall that carbon dioxide is a major regulator of cerebral blood flow, i.e., carbon dioxide dilates the
brain vasculature, and conversely, the vasculature constricts when carbon dioxide levels are low. The
decrease in cerebral blood flow leads to cerebral hypoxia, which is probably responsible for the
fainting and blurred vision. An attack may be terminated by breathing in and out of a plastic bag
because this can increase carbon dioxide levels in the blood. Inhaling a 5% carbon dioxide mixture
would also be effective. Breathing a mixture of 10% oxygen/90% nitrogen (choice A) or 100%
nitrogen (choice B) can decrease oxygen delivery to the brain and thereby worsen the symptoms
caused by hyperventilation. Hyperventilation results in hypocapnia (low PCO2), which causes
alkalosis (high blood pH). Bicarbonate (choice D) should not be administered to an alkalotic patient.

The feelings of faintness and blurred vision resulting from hyperventilation are not relieved by lying
down (choice E).
19. A student volunteers to have his lung volumes and capacities measured. He is instructed
to breathe several times until the helium has equilibrated between the spirometer and his
lungs, and then exhale as much air as he possibly can. Calculations are made to determine the
amount of air remaining in his lungs, which is called the
A. expiratory reserve volume
B. functional residual capacity
C. inspiratory capacity
D. inspiratory reserve volume
E. residual volume
The correct answer is E. There are two ways to arrive at the correct answer to this question. The
first is to simply remember the definition of residual volume (RV): the amount of air remaining in the
lungs after maximal exhalation. The second way is to recall that the helium dilution technique
described above is used to measure functional residual capacity (FRC) and RV, which narrows the
reasonable option choices to B and E. All of the other volumes and capacities can be directly
measured with spirometry because they are blown into the spirometer. Only FRC and RV represent
amounts of air that remain in the lungs. Expiratory reserve volume (choice A) is the volume expelled
by an active expiratory effort after passive expiration. Functional residual capacity (choice B) is
defined as the amount of air remaining in the lungs after passive expiration. Inspiratory capacity
(choice C) is the maximal amount of air inspired after a passive expiration. Inspiratory reserve volume
(choice D) is the amount of air inspired with a maximal inspiratory effort over and above the tidal
volume.
20. Which of the following characteristics is similar for spermatogenesis and oogenesis?
A. Age at which meiosis begins
B. Amount of cytoplasm retained
C. DNA replication during meiosis
D. Length of prophase I
E. Transmission to fetus of mitochondrial DNA
The correct answer is C. The actual process of manipulation of DNA and chromosomes during
meiosis is very similar in spermatogenesis and oogenesis. The processes also differ in many other
respects, however:
In oogenesis, the process of meiosis begins before birth and arrests between birth and puberty in
prophase I. In contrast, spermatogenesis does not begin until puberty (choice A).
The egg retains a large volume of cytoplasm (choice B), where nearly all the cytoplasm is stripped
during formation of a sperm. As noted, in oogenesis, meiosis is arrested in prophase I, which is
consequently very prolonged in the female. In spermatogenesis, meiosis is completed in a much
shorter time (choice D). Both the egg and the sperm have mitochondria, but those of the sperm are
left outside when the sperm nucleus enters the egg and consequently do not contribute to the
mitochondrial genome of the fetus. Instead, the mitochondria are transmitted from the egg to the fetus
(choice E). Traits coded for by mitochondrial DNA are therefore inherited in a matrilineal fashion.
21. Increases in which of the following normal physiological parameters may have contributed
to the formation of acute gastric stress ulcers?
A. Bicarbonate transport
B. Epithelial regenerative capacity
C. Mucosal blood flow
D. Mucus secretion
E. Pepsin production

The correct answer is E. Pepsin is produced in the chief cells in the stomach. This proteolytic
enzyme functions most effectively at a strong acid pH of 1.5 - 2.0. Pepsin production is a normal
physiologic activity of the stomach that, in conditions of stress, may overwhelm the stomach's
weakened defenses and result in gastric ulceration. Gastric acid production is another condition that
may increase and cause acute ulceration. Furthermore, these two factors may remain unchanged and
still result in gastric ulcers if the gastric defenses are weakened by stress. All of the other choices
represent normal defensive forces in the stomach.
Increased bicarbonate transport (choice A) would protect the gastric epithelium from the potentially
harmful acidity of the gastric contents. The adherent mucus is relatively alkaline, providing local
protection to the superficial mucosa.
Gastric epithelial cells can normally replicate rapidly, allowing mucosal defects to be rapidly repaired.
Increasing the regenerative capacity of the epithelium (choice B) would have a protective effect
against ulceration.
The gastric mucosa is richly supplied with blood, providing the epithelial cells with an ample supply of
nutrients, oxygen, and bicarbonate to contend with the harsh gastric microenvironment. Stress ulcers
are associated with compromised gastric blood flow, not increased flow (choice C).
Mucus protects the gastric epithelium by virtue of being water insoluble, impermeable to pepsin, and
slowly permeated by acid (H+). Increasing mucus production (choice D) has a protective effect for the
gastric mucosa.
22. Inhibition of which of the following could be responsible for impotence as a side effect of a
medication?
A. Conversion of DHT to testosterone
B. Forward motility factor
C. Nitric oxide synthase (NOS)
D. Oxytocin
E. Prostaglandins
The correct answer is C. Penile erection is mediated by the parasympathetic nervous system. The
neurons involved are termed nonadrenergic, noncholinergic (NANC) autonomic neurons, and they
release nitric oxide (NO). NO binds to the iron in the heme molecule of guanylate cyclase, activating it
to form cGMP. This results in a decrease in intracellular calcium and subsequent smooth muscle
relaxation and vasodilation in the corpus cavernosa, producing erection. Nitric oxide synthase (NOS)
is the enzyme required for the formation of NO from circulating arginine, and androgens are
necessary to maintain normal amounts of this enzyme. Inhibition of this enzyme could result in
impotence. Inhibition of the conversion of testosterone to DHT, not DHT to testosterone (choice A),
would be accomplished by a 5-alpha-reductase inhibitor. This could cause impotence by its
antiandrogen effect. In fact, finasteride, a clinically used 5-alpha-reductase inhibitor, produces drastic
decreases in libido in some men. Inhibition of forward motility factor (choice B) would hamper sperm
motility and result in infertility, not impotence. Inhibition of oxytocin (choice D) or prostaglandins
(choice E) would result in failure to ejaculate (ejaculatory incompetence), not impotence.
23. A research physiologist decides to use a marker to measure the volume of total body water
in a volunteer. Which of the following substances would he most likely use?
A. Antipyrine
B. Cresyl violet
C. Evans blue
D. I131-albumin
E. Inulin
The correct answer is A. Antipyrine and tritium are both markers for total body water. Cresyl violet
(choice B) is a histologic dye used to stain Missl substance in neurons. It stains cell bodies. Evans
blue (choice C) is used to measure the plasma compartment.

I131-albumin (choice D) is used to measure the plasma compartment. Inulin (choice E) is used to
measure the extracellular fluid compartment.
24. Which of the following vascular structures contains the largest proportion of the total
blood volume in a normal individual?
A. Aorta and large arteries
B. Arterioles
C. Capillaries
D. Chambers of the heart
E. Venules and veins
The correct answer is E. The total blood volume of the body is approximately 5000 mL. The
systemic veins contain approximately 64% of this volume or approximately 3200 mL. No other
segment of the circulation comes close to the amount of blood contained by the systemic veins: the
chambers of the heart (choice D) contain approximately 350 mL; the aorta and large arteries (choice
A) together contain approximately 650 mL; and the arterioles and capillaries (choices B and C)
together contain approximately 350 mL. Although the capillaries contain less than 7% of the total
blood volume, they have a very large surface area that facilitates diffusion exchange of nutrients and
metabolites between the blood and tissue spaces.
25. Which of the following substances is likely to be present in significant amounts in urine
that has a stable, frothy, white foam on top?
A. Bilirubin
B. Blood
C. Glucose
D. Ketones
E. Protein
The correct answer is E. In this case, you need to know that a stable froth on urine is usually caused
by proteinuria (more than several grams per 24 hours). High levels of bilirubin (choice A) in urine can
cause an unusual yellow foam. Blood in the urine (choice B) might be present in some forms of renal
disease, but would not explain the stable foam. High levels of glucose (choice C) in urine can cause it
to develop a sweet smell and taste; smelling and tasting urine was an ancient method of diagnosing
diabetes mellitus, but is no longer recommended. Ketones (choice D) may give urine an acetone-like
odor.
26. Tissue taken from a fracture site of a patient with end stage renal failure is noted to have
increased osteoclastic activity, with notable tunnel-like dissection by osteoclasts into the bony
trabeculae. Increased levels of which of the following hormones is most likely to be
responsible for this lesion?
A. Calcitonin
B. Cortisol
C. Erythropoietin
D. Glucagon
E. Parathyroid hormone
The correct answer is E. The histological picture suggests hyperparathyroidism, a well-recognized
sequela of chronic renal failure. This condition is caused by high levels of serum phosphate and low
serum calcium, which stimulate the release of parathyroid hormone (PTH) in an effort to normalize the
calcium/phosphate ratio. PTH stimulates osteoblasts to become osteoclasts, which dissolve the bone
reservoir of calcium and release it into the blood. This condition, known as renal osteodystrophy, may
lead to osteomalacia and osteitis fibrosa cystica, which is classically associated with dissecting
osteitis, as described in this question. With respect to parathyroid hormone physiology, the following
apply. The PTH has 5 effects: (1) stimulates osteoclasts; (2) inhibits osteoblasts; (3) promotes

10

intestinal absorption of calcium and phosphate ions; (4) reduces urinary excretion of calcium; (5)
stimulates formation and secretion of calcitrol at the kidneys.
Calcitonin (choice A), which serves to lower serum calcium levels, is produced in small quantities in
chronic renal failure, as serum calcium is already pathologically low.
Excess cortisol (choice B; Cushing's syndrome) may produce osteoporosis and pathologic fractures,
but there is no relationship between chronic renal failure and cortisol excess. Cushing's syndrome is
generally secondary to adrenal or pituitary adenomas or primary adrenal hyperfunction.
Erythropoietin (choice C) production is often deficient in chronic renal failure. Excess erythropoietin
activates erythrocyte precursors, but does not activate osteoclasts.
Glucagon (choice D) excess is a very rare entity, occurring in a minority of islet cell tumors, and is not
associated with renal failure. High glucagon produces a transitory skin rash, anemia, and a form of
diabetes mellitus.
27. Nonfunctioning of complement component C36 would be expected to interfere with which
of the following biologic functions?
A. Decreased appetite
B. Fever
C. Increased collagen synthesis by fibroblasts
D. Increased leukocyte adherence to endothelium
E. Opsonization to facilitate phagocytosis
The correct answer is E. C3b acts as an opsonin, together with IgG, to facilitate phagocytosis. None
of the other functions listed in the answer choices is attributable to C3b. Instead, all of the other
answer choices are functions of the cytokines interleukin-1 (IL-1) and tumor necrosis factor (TNF).
Tumor necrosis factor is produced by macrophages and activated T-cells, whereas many different cell
types (including macrophages) produce IL-1.
Decreased appetite (choice A) is typical during an acute illness (such responses are called acute
phase reactions). Other acute phase reactions include fever (choice B), increased sleep, shock,
leukocytosis, and increased serum acute phase proteins.
Fibroblasts proliferate and increase their synthesis of collagen (choice C), collagenase, protease, and
prostaglandin E in response to IL-1 and TNF.
Effects of IL-1 and TNF on endothelium include increased synthesis of mediators (prostaglandins, IL1, IL- 8, platelet-derived growth factor), increased leukocyte adherence (choice D), and increased
procoagulant activity with decreased anticoagulant activity.
28. If a creatinine clearance of 100 mL/min changes to 50 mL/min over 1 year, assuming that
there has been no change in diet, which of the following physiologic changes can be
expected?
A. A 2-fold decrease in blood urea nitrogen concentration
B. A 2-fold decrease in creatinine excretion rate
C. A 2-fold increase in creatinine excretion rate
D. A 2-fold increase in creatinine reabsorption
E. A 2-fold increase in plasma creatinine concentration
The correct answer is E. Creatinine clearance is used clinically to estimate glomerular filtration rate
(GFR). The 50% decrease in creatinine clearance in this patient therefore suggests that GFR has
decreased by 50% over the past year. Because creatinine is freely filtered but not reabsorbed (choice
D), the filtration rate and excretion rate of creatinine are equal during steady state conditions. When
GFR decreases, the rate of creatinine excretion will also decrease, causing the rate of creatinine
excretion to decrease to below the rate of creatinine production. The result is an increase in plasma

11

creatinine concentration. When plasma creatinine levels have increased by 2-fold, normal amounts of
creatinine can then again be excreted (compare with choices B and C) because the excretion rate of
creatinine is equal to the product of GFR and plasma creatinine concentration. In summary, when
GFR decreases, the plasma creatinine concentration continues to increase until the rate of creatinine
filtration (and excretion) becomes equal to the rate of creatinine production by the body.
The blood urea nitrogen concentration (choice A) increases when GFR is reduced.
29. A cell biologist wants to activate protein kinase C in cultured cells. Stimulation of which of
the following receptor types would most likely produce the greatest activation?
A. Alpha 1 adrenergic
B. Beta 1 adrenergic
C. Dopamine-2 (D2)
D. Gamma-aminobutyric acid, type A (GABAA)
E. Nicotinic cholinergic
The correct answer is A. It is a good idea to know the mechanism of action of various receptors.
Alpha-1 adrenergic receptors are coupled to the G protein, Gq. This G protein causes breakdown of
the membrane phospholipid phosphatidylinositol bisphosphate (PIP2), forming the products
diacylglycerol, which stimulates protein kinase C, and inositol triphosphate (IP3), which releases
calcium from the endoplasmic reticulum. It is important to note that cyclic AMP activates protein
kinases by dissociating the complex of regularity and catalytic subunits, which results in activation of
the cellular response.
Beta-1 adrenergic receptors (choice B), like all beta receptors, are coupled to the Gs G protein,
leading to the activation of adenylate cyclase. This increases intracellular concentrations of cAMP by
converting ATP to cAMP. cAMP in turn stimulates protein kinase A, also known as cAMP-dependent
protein kinase.
Dopamine-2 receptors (choice C) are coupled to Gi. Stimulation of these receptors inhibits adenylate
cyclase, thus lowering cAMP levels and protein kinase A activity.
Gamma-aminobutyric acid, type A (GABAA) receptors (choice D) are not G-protein-coupled
receptors, but are instead ligand-gated ion channel receptors. Stimulation of these inhibitory receptors
causes a chloride flux, thus stabilizing the membrane potential of the cell.
Nicotinic cholinergic receptors (choice E) are not G-protein-coupled receptors, but are instead ligandgated ion channel receptors. Stimulation of these receptors causes sodium influx, leading to excitation
of the cells.
30. which type of blood vessel is the mean linear velocity of a red blood cell the lowest?
A. Aorta and large arteries
B. Arterioles
C. Capillaries
D. Small arteries
E. Vena cavae and large veins
The correct answer is C. The same volume of blood flows through each of the different types of
blood vessels each minute. Because the capillaries have the largest cross-sectional area (averaging
2500-5000 cm2), and the velocity of blood flow is inversely related to cross-sectional area, the mean
linear velocity of a red blood cell is lowest in the capillaries. Under resting conditions, the mean linear
velocity of a red blood cell in the capillaries is 0.3-0.6 mm/sec, whereas the velocity in the aorta
(choice A) is approximately 200 mm/sec. This low velocity of red blood cells in the capillary network
allows plenty of time for oxygen to diffuse to the tissues.
The velocity of blood flow is ranked from highest to lowest as follows: aorta (choice A) > vena cavae
(choice E) > large veins (choice E) > small arteries (choice D) > arterioles (choice B) > capillaries.

12

When the vena cavae are partially collapsed, however, (which occurs often) they have a lower crosssectional area and a higher velocity of blood flow compared with the aorta.
31 cells that have a low affinity for MHC class I molecules differentiate in the thymus to
become which type of cell?
A. CD8+ cytotoxic lymphocyte
B. Gamma-delta T cell
C. Natural killer cell
D. T helper 1 cell
E. T helper 2 cell
The correct answer is A. CD8 cells are generated after mature T cells interact with certain foreign
antigens. They are responsible for defense against intracellular pathogens, tumor immunity, and
organ graft rejection. CD8+ cytotoxic T lymphocytes are positively selected in the thymus because
they have low affinity for MHC class I molecules. If they had possessed high affinity for the MHC class
I molecules, they would have been eliminated (negative selection) because of the danger of
autoimmune disease. Also, cells with no affinity for MHC class I molecules would be eliminated.
The gamma-delta designation in a T cell (choice B) refers to type of receptor on the cell. Most T-cell
receptors are alpha-beta receptors, but some are of a different isotype termed gamma-delta.
Natural killer cells (choice C) are large granular lymphocytes that are part of the innate immune
response. Natural killer cell function does not depend on MHC class I or class II molecules; it simply
kills tumor cells or virally infected cells.
T helper 1 (choice D) and T helper 2 cells (choice E) would have a low affinity for class II MHC
molecules to survive in the thymus. If they had no affinity or strong affinity for these molecules, they
would have been eliminated.
32. Maintenance of the corpus luteum during the first trimester of pregnancy is accomplished
principally by the secretion of
A. antidiuretic hormone (ADH)
B. follicle stimulating hormone (FSH)
C. human chorionic gonadotropin (hCG)
D. luteinizing hormone (LH)
E. progesterone
The correct answer is C. The corpus luteum secretes estrogens, progesterone, and relaxin. hCG,
secreted by the syncytiotrophoblast lining the placental villi, maintains the corpus luteum during the
first trimester of pregnancy. hCG appears in the maternal blood stream soon after implantation
occurs. This hormone, in addition to maintaining the corpus luteum, also promotes the continued
secretion of progesterone. As a result, the endometrial lining remains functional and menses does not
occur. Antidiuretic hormone (vasopressin; choice A) does not play a significant role during
pregnancy. FSH (choice B) acts on granulosa cells to promote the conversion of androstenedione to
estradiol. LH (choice D) acts on theca cells to promote androstenedione secretion. Progesterone
(choice E) is important for maintaining the pregnancy, however, it does not act to maintain the corpus
luteum.
33. Metabolism of 100 grams of fat will yield approximately how many kilocalories?
A. 200
B. 400
C. 600
D. 900
E. 1200
The correct answer is D. Carbohydrates contain approximately 3.4 kcal of energy and proteins
contain 4 kcal of energy per gram. Fats are the more energy-efficient storage form, containing 9 kcal
of energy per gram. A quantity of 100 grams of fat would therefore yield 900 kcal of energy.

13

34. Which of the following hormones of pregnancy is most likely to be significantly decreased
compared with normal in the blood sample of a woman whose fetus has died during the
second trimester?
A. Estriol
B. Human chorionic gonadotropin
C. Human chorionic somatomammotropin
D. Progesterone
E. Prolactin
The correct answer is A. Maternal blood levels of estriol, a weak estrogen, depend on a viable fetus.
The fetal adrenal cortex and liver produce the weak androgen, 16-OH dehydroepiandrosterone sulfate
(16-OH DHEA-S) that is carried by the fetal circulation to the placenta, where it is converted to estriol
and secreted into the maternal circulation. Although maternal blood levels of estradiol and estrone
increase by 50-fold during pregnancy, maternal blood levels of estriol increase 1,000-fold. Increasing
maternal blood levels of estriol is the best indicator of fetal well-being. A significant drop in maternal
estriol may indicate fetal jeopardy.
Human chorionic gonadotropin (hCG, choice B) is secreted by syncytiotrophoblast cells and does not
depend on a viable fetus. Human chorionic gonadotropin (hCG) is in the same hormone family as
TSH, FSH, and LH. The maternal blood or urinary level of hCG is used to confirm the presence of
pregnancy. Its function is to "rescue" the corpus luteum and maintain the pregnancy until the placenta
can produce sufficient estrogen and progesterone.
Human chorionic somatomammotropin (hCS; choice C) is secreted by syncytiotrophoblast cells and
does not depend on the presence of a viable fetus. It is also known as human placental lactogen, and
is in the same hormone family as growth hormone and prolactin. Its function in pregnancy is not
completely understood, but it may serve to reduce maternal glucose use and allow for "shunting" of
glucose to the fetus.
Maternal progesterone (choice D) depends on a viable placenta, but does not depend on a fetal
contribution. The precursor for placental secretion of progesterone is maternal (not fetal) cholesterol.
Prolactin (choice E) levels steadily increase during pregnancy. This contributes to the final
development of a mature mammary gland. It is secreted by the anterior pituitary and does not require
a fetal contribution.
35. With time, blood stored in a blood bank tends to become relatively depleted of 2,3diphosphoglycerate (2,3-DPG). What effect does this have on the hemoglobin-oxygen
dissociation curve?
A. Shifts the curve to the left, so that the hemoglobin has a decreased oxygen affinity
B. Shifts the curve to the left so that the hemoglobin has an increased oxygen affinity
C. Shifts the curve to the right, so that the hemoglobin has a decreased oxygen affinity
D. Shifts the curve to the right, so that the hemoglobin has an increased oxygen affinity
E. Does not change the dissociation curve
The correct answer is B. 2,3-diphosphoglycerate (2,3-DPG) is produced in red cells (RBCs) by a
variation on the glycolytic pathway, and levels diminish when glycolysis by the RBCs slows. The
depletion of 2,3-DPG in stored blood causes the hemoglobin dissociation curve to shift to the left,
leading to an increase in oxygen affinity. This increase is helpful in the picking up of oxygen by
hemoglobin from the lungs, but it can be very problematic in the release of oxygen from the blood to
the tissues. This is not just a theoretical point: considerable effort has been expended in developing
improved solutions for storing packed RBCs and methods for "restoring" older stored cells so that the
2,3-DPG levels are adequate. In practice, in otherwise reasonably healthy patients, older transfused
blood will quickly regenerate 2,3-DPG when placed in the glucose-containing environment of the
serum; however, even transiently decreased 2,3-DPG levels in a severely compromised patient can
be dangerous.

14

36. Under normal conditions, the main drive for respiration is the
A. arterial PCO2 acting through central chemoreceptors
B. arterial PCO2 acting through peripheral chemoreceptors
C. arterial pH acting through central chemoreceptors
D. arterial pH acting through peripheral chemoreceptors
E. arterial PO2 acting through central chemoreceptors
The correct answer is A. The most important factor in the control of minute-to-minute ventilation is
arterial PCO2, which influences chemoreceptors located near the ventral surface of the medulla. As
arterial PCO2 rises, CO2 diffuses from cerebral blood vessels into the CSF. Carbonic acid is formed
and dissociates into bicarbonate and protons. Protons directly stimulate these central
chemoreceptors, resulting in hyperventilation. Hyperventilating reduces the PCO2 in the arterial blood
and subsequently in the CSF.
Peripheral chemoreceptors located in the carotid and aortic bodies respond to increases in
PCO2(choice B), but are less important than the central chemoreceptors. It is estimated that when a
normal subject hyperventilates in response to inhalation of CO2, less than 20% of the response can
be attributed to the peripheral receptors. However, they respond more quickly than their central
counterparts and are thought to play a role in regulating ventilation after abrupt changes in PCO2.
There are no known central chemoreceptors that respond to either arterial pH (choice C) or arterial
PO2(choice E).
Carotid chemoreceptors (choice D) cause hyperventilation in response to decreases in arterial pH;
however, CO2 acting through central chemoreceptors is the most important regulator of ventilation
under normal conditions.
37. Which of the following types of nerve fibers carry pain that is aching, burning, sometimes
throbbing, and poorly localized?
A. Group II fibers
B. Group III fibers
C. Type C fibers
D. Substance P
The correct answer is C. Slow pain as described in this question is carried by type C fibers. ChoiceA - The dorsal column system consists of type II fibers that carry touch and pressure sensation.
Choice B - Fast pain is carried by group III fibers. It has a rapid onset and offset and is localized. The
anterolateral system, which processes sensations of temperature, pain, and light touch, consists
primarily of group III and group IV fibers. Choice D - Pain is associated with detection and perception
of noxious stimuli (nociception). The neurotransmitters for nociceptors include substance P. Inhibition
of the release of substance P is the basis of pain relief by opioids.
38. If a drug is taken that stimulates sympathetic cholinergic neurons, which of the following
responses is expected?
A. Bradycardia
B. Bronchoconstriction
C. Diaphoresis
D. Increased gastrointestinal motility
E. Increased peripheral vascular resistance
The correct answer is C. Sympathetic cholinergic neurons are sympathetic postganglionic neurons
that happen to release acetylcholine instead of norepinephrine. The vast majority of sweat glands in
the body are innervated by sympathetic cholinergic neurons (choice C). Bradycardia (choice A),
bronchoconstriction (choice B), and increased gastrointestinal motility (choice D) would all result
from stimulating parasympathetic cholinergic neurons.

15

Increased peripheral vascular resistance (choice E) would result from stimulating sympathetic
adrenergic neurons.
39. A 3-year-old child has inhaled a peanut that has lodged in the right mainstem bronchus,
largely occluding it. The child is cyanotic, with a PO2 of 60 mm Hg. Which of the following
mechanisms best accounts for the child's hypoxemia?
A. Decreased capacity of pulmonary diffusion
B. Decreased PO2 in inspired air
C. Hypoventilation of central origin
D. Hypoventilation of peripheral origin
E. Inequalities of ventilation and perfusion
The correct answer is E. Inequalities of ventilation and perfusion contribute to hypoxia in many
settings. In this case, blood goes to both lungs (perfusion), but air is prevented from entering one of
the lungs (ventilation). Because the right lung is being perfused, but not ventilated, hypoxemia ensues
when the deoxygenated blood from the right lung mixes with oxygenated blood from the left lung. If
the inadequate ventilation of the lung persists long enough, the lung tissue itself can be damaged,
causing a secondary local dilation of arterioles, making the problem even worse. Peanuts are
notorious for producing this type of problem in young children because of their size and shape, which
allow them to lodge in the trachea or main bronchus after aspiration.
Decreased diffusion capacity (choice A) can occur when the blood-gas barrier is thickened (e.g.,
diffuse interstitial fibrosis, sarcoidosis, asbestosis, respiratory distress syndrome), when the surface
area of the blood-gas barrier is reduced (e.g., pneumonectomy, emphysema), or when less
hemoglobin is available to carry oxygen (e.g., anemia, pulmonary embolism).
Decreased PO2 in inspired air (choice B) is seen at high altitudes and when the settings are wrong
during artificial ventilation.
Hypoventilation of central origin (choice C) is seen in morphine and barbiturate overdose.
Hypoventilation of peripheral origin (choice D) is seen in poliomyelitis and chest trauma.
40. One hour before a dental procedure a patient receives four 500-mg amoxicillin capsules.
On arriving at the dental office the dentist notices that the patient has maculopapular rash and
is wheezing. The effector cell in this type of hypersensitivity is a(n)
A. eosinophil
B. mast cell
C. megakaryocyte
D. neutrophil
E. TH1 CD4+ lymphocyte
The correct answer is B. This patient is experiencing the early phase of type I hypersensitivity,
characterized by bronchospasm and development of a maculopaplar rash. Other potential symptoms
of this allergic drug reaction include pruritus and watery discharge from the nose, mucus secretion in
the airways, and a wheal-and-flare response with pruritus in the skin. The mechanism of
hypersensitivity involves prior sensitization of a population of TH2 cells that produced cytokines,
including interleukin-4. The interleukin-4 causes B cells to switch their heavy chain class from IgM to
IgE. The IgE molecules then attach to mast cells or basophils. With subsequent antigen (allergen)
challenge, the mast cells degranulate and release mediators, including histamine, which produces the
anaphylactic response.
Eosinophils (choice A) are involved in late-phase type I hypersensitivity reactions and type II
antibody-dependent cell cytotoxicity reactions directed against parasites. Eosinophils enter the area of
the reaction because of the release of eosinophil chemotactic factor (eotaxin) and the beta-chemokine
RANTES released from TH2 CD4+ cells and mast cells. Eosinophils are recruited into the tissues as

16

part of the late-phase reaction of type I hypersensitivity. Their survival in tissue is dependent on IL-3,
IL-5, and granulocyte-monocyte colony stimulating factor (GM-CSF) released from TH2 cells.
Megakaryocytes (choice C) are bone marrow cells that produce platelets. They are not involved in
type I hypersensitivity.
Neutrophils (choice D) are not a cell type that is key to the development of type I hypersensitivity.
TH1 CD 4+ lymphocytes (choice E) are associated with delayed type hypersensitivity reactions such
as contact dermatitis involving exposure to poison ivy or poison oak.
41. An increase in which of the following is the most likely explanation for the swelling in the
legs seen with congestive heart failure?
A. Interstitial colloid osmotic pressure
B. Lymph flow
C. Plasma colloid osmotic pressure
D. Right atrial pressure
E. Stroke volume
The correct answer is D. Congestive heart failure typically develops when the heart becomes
damaged (usually as a result of myocardial infarction) and the cardiac output cannot be maintained at
a normal level. A low cardiac output has profound effects on the kidneys, causing salt and water
retention. If the heart is not damaged too badly, the increase in blood volume caused by the fluid
retention can increase venous return sufficiently to totally compensate for the diminished pumping
capacity of the heart. The increase in venous return causes the right atrial pressure to increase, which
elevates venous pressure throughout the body. This increase in venous pressure can cause
excessive fluid loss from the microcirculation and the development of peripheral edema. The labored
breathing experienced by the patient is indicative of mild pulmonary edema caused by increased
pressure in the pulmonary microvasculature.
The increase in fluid loss from the microcirculation would be expected to literally wash protein
molecules out of the interstitial compartment and thereby decrease the interstitial colloid osmotic
pressure (choice A), and increase the flow of lymph (choice B) from the tissue as a consequence
rather than a cause of the edema.
Increased plasma colloid osmotic pressure (choice C) would tend to decrease the development of
edema. Also, fluid retention by the kidneys tends to dilute the plasma and thus decrease the plasma
colloid osmotic pressure.
Stroke volume (choice E) has no direct role in the formation of peripheral edema.
42. An increase in which of the following best explains the mechanism by which the cardiac
output increases in severe anemia?
A. Arteriolar diameter
B. Blood viscosity
C. Peripheral vascular resistance
D. Splanchnic blood flow
E. Tissue oxygen tension
The correct answer is A. Anemia is present in adults if the hematocrit is less than 41% in men and
37% in women. Poor diet and pathologic bleeding can lead to the development of folic acid deficiency
and iron deficiency anemia, respectively. In severe anemia, diminished transport of oxygen in the
blood leads to hypoxia in the tissues. The hypoxia (compare with choice E) causes small arteries and
arterioles to dilate, which allows greater than normal amounts of blood to return to the heart. In severe
anemia, the viscosity of blood (choice B) may decrease by 50% or more because blood viscosity
depends largely on the concentration of red blood cells. This decrease in viscosity lowers the

17

resistance to blood flow in the peripheral tissues (i.e., decreases peripheral vascular resistance,
choice C) allowing even greater amounts of blood to return to the heart.
Blood is often shunted away from the splanchnic vascular bed (choice D) in anemia, which can cause
gastrointestinal problems.
43. Which of the following is found in the respiratory zone of the lung?
A. Goblet cells
B. Main bronchi
C. Mucous cells
D. Terminal bronchioles
E. Type I epithelial cells
The correct answer is E. There are two zones in the lung: the conducting zone (where there is no
gas exchange) and the respiratory zone (where gas exchange takes place). Of all of the structures
listed, only type I epithelial cells are located in the respiratory zone. Type I epithelial cells are the
primary structural cells of the alveolar wall. Type II epithelial cells, also located in the alveoli, produce
surfactant. Goblet cells (choice A), which are mucus-secreting cells, are present in the conducting
airways. The main bronchi (choice B) are part of the conducting airways. Mucous cells (choice C),
which are mucus-secreting cells, are also present in the conducting airways. Terminal bronchioles
(choice D) are the most distal part of the conducting airways. Respiratory bronchioles, which are just
distal to the terminal bronchioles, are part of the respiratory zone. These two types of bronchioles can
be differentiated from each other by whether they have alveoli budding from their walls. Respiratory
bronchioles have alveoli, terminal bronchioles do not.
44. Laboratory studies of an automobile accident victim show a significant elevation of
creatine kinase (CK). Which of the following tissues is most likely to be the source of the
enzyme?
A. Bone
B. Brain
C. Heart
D. Kidney
E. Skeletal muscle
The correct answer is E. Creatine kinase occurs in several different forms, or isoenzymes, in the
body. Most tissues contain a mix of creatine kinase isoenzymes, but one species often predominates.
The MB isoenzyme of creatine kinase is associated with heart damage; the MM isoenzyme is
associated with muscle damage; and the BB isoenzyme is associated with brain damage. Surgery
and general trauma are commonly implicated as causes of sufficient skeletal muscle trauma to
markedly elevate creatine kinase by way of the CK-MM (skeletal muscle isoenzyme) isoenzyme.
Bone (choice A) is not a significant source of CK. Brain (choice B) can occasionally be a source of
creatine kinase (BB isoenzyme), but would not usually be the major source following an automobile
accident. Heart damage (choice C), as in myocardial infarction, can elevate creatine kinase, but
would not usually be the major source in an automobile accident. If the dilemma arises as to whether
an automobile accident was possibly secondary to a myocardial infarction, the ratio of CK-MB to total
CK may be helpful in demonstrating a cardiac contribution (CK-MB is characteristic of heart muscle).
Kidney (choice D) is not a major source of creatine kinase.
45. Which of the following binds to von Willebrand factor at the platelet membrane?
A. Adenosine diphosphate (ADP)
B. Calcium
C. Glycoprotein GPIb
D. Platelet factor 3 (PF3)
E. Prostacyclin

18

The correct answer is C. Glycoprotein GPIb on the platelet membrane binds von Willebrand factor, a
plasma protein that circulates in a complex with factor VIII. ADP (choice A) is a powerful inducer of
platelet aggregation and strengthens the platelet plug by the addition of more activated platelets.
Calcium (choice B) is essential for increasing the degree of platelet aggregation and strengthening
the platelet plug. It is also a necessary cofactor in the coagulation cascade (required for the
conversion of factors IX to IXa, X to Xa, V to Va, and prothrombin [factor II] to thrombin [factor IIa]).
Platelet factor 3 (PF3; choice D) is involved in platelet plug formation. Prostacyclin (choice E) is
synthesized by blood vessel endothelial cells and inhibits platelet aggregation.
46. Which of the following statements about Troponin C is true? Troponin C
A. inhibits the interaction of actin and myosin
B. attaches the troponin complex to tropomyosin
C. permits the interaction of actin and myosin
D. is located at the junction of A band and I band
The correct answer is C. Troponin C is the calcium binding protein that when bound to calcium
permits the interaction of actin and myosin. Cross bridges continue as long as calcium is bound to
troponin C. Choice A - Troponin I inhibits the interaction of actin and myosin. Choice B - Troponin T
attaches the troponin complex to tropomyosin. Choice D - T tubules are the extensive tubular network
that carry the depolarization from sarcolemmal membrane to the cell interior. These are located at the
junction of A band and I band.
47 .Which of the following can be determined by calculation of the clearance of
paraaminohippuric acid (PAH)?
A. Extracellular fluid (ECF) volume
B. Effective renal plasma flow (ERPF)
C. Glomerular filtration rate (GFR)
D. Plasma volume
E. Total body water (TBW)
The correct answer is B. At less than saturating concentrations, PAH is completely secreted into the
proximal tubule and excreted into the urine. The volume of plasma cleared of PAH is therefore
approximately equal to the volume of plasma flowing through the peritubular capillaries, also called
the effective renal plasma flow, or ERPF. The ECF volume (choice A) can be calculated by
measuring the volume of distribution of solutes that move freely across capillary walls but cannot
permeate cell membranes (e.g., inulin and mannitol). GFR (choice C) is best calculated using a
substance that is freely filtered at the glomerulus, not reabsorbed, and only minimally secreted into
the urine. Creatinine fits the bill and is used clinically to measure the GFR (inulin also works and is
used experimentally). Although the creatinine excretion exceeds filtration by 10-20% (because of the
secretion), creatinine clearance is still a good approximation for GFR because the error caused by
secretion is balanced by an overestimation of plasma creatinine inherent in the measurement
technique. The plasma volume (choice D) can be measured by measuring the volume of distribution
of radioactively labeled serum albumin or of Evans blue dye (binds to albumin). Total body water
(TBW; choice E) can be measured by measuring the volume of distribution of tritium, deuterium, or
antipyrine.
48. The clearance of several substances was measured at a constant glomerular filtration rate
and constant urine flow rate, but at increasing plasma concentrations of the substance. Under
these conditions, clearance will increase at high plasma concentrations for which of the
following substances?
A. Creatinine
B. Mannitol
C. Penicillin
D. Phosphate
E. Urea
The correct answer is D. Clearance of a substance will change with increasing plasma
concentration if that substance is secreted or reabsorbed by a facilitated mechanism. As the

19

concentration of the substance increases, the transporter becomes saturated, and its contribution to
excretion changes, changing the clearance. If the substance is reabsorbed by a facilitated
mechanism, clearance will eventually increase with increasing plasma concentrations. Approximately
80% of filtered phosphate is reabsorbed in the proximal tubule by a sodium-phosphate cotransporter,
which is a facilitated mechanism. The phosphate buffer system consists of an anion, H2PO4, that is a
weak acid. The basic operation of the phosphate buffer system resembles that of the carbonic acidbicarbonate buffer system.
49. Gastrin stimulates the secretion of HCl. Which of the following cell types located at the
fundus of the stomach produces HCl?
A. Parietal cells
B. Chief cells
C. G cells
D. Mucous cells
The correct answer is A. Parietal cells located in the body of the stomach secrete HCl and intrinsic
factor. Stimuli for secretion are gastrin, histamine, and Ach. Parietal cells secrete HCl into the lumen
of the stomach and concurrently absorb bicarbonate into the blood stream. Choice B - Chief cells,
located in fundus, secrete pepsinogen under vagal stimulation. Choice C - Vagal stimulation secretes
gastrin from the G cells of the antrum of the stomach. Choice D - Mucous cells, located in the antrum
of the stomach, secrete mucus and pepsinogen under vagal stimulation.
50. Adrenocorticotropic hormone (ACTH) promotes the production of cortisol by stimulating
which of the following reactions?
A. 11-Deoxycortisone to cortisol
B. 17-Hydroxyprogesterone to 11-deoxycortisol
C. Cholesterol to pregnenolone
D. Pregnenolone to progesterone
E. Progesterone to 17-hydroxyprogesterone
The correct answer is C. All of the choices listed are reactions that occur in the synthetic pathway
from cholesterol to cortisol. ACTH stimulates the first reaction in the pathway: cholesterol to
pregnenolone. This reaction is catalyzed by the enzyme cholesterol desmolase.
The next step in the pathway is pregnenolone to progesterone (choice D); progesterone is then
converted to 17-hydroxyprogesterone (choice E); 17-hydroxyprogesterone is converted to 11deoxycortisol (choice B) by the enzyme 21 beta-hydroxylase; and the 11-deoxycortisol is then
converted to cortisol (choice A).
51. Laboratory studies for an alcoholic patient indicate that serum magnesium is depressed.
Which of the following findings would be most consistent with this information?
A. Decreased serum calcium
B. Decreased serum phosphate
C. Increased bone density
D. Increased plasma parathyroid hormone concentration
The correct answer is A. Malnutrition associated with chronic alcoholism can lead to a severe
magnesium deficiency. The effect of low serum magnesium on parathyroid hormone secretion (PTH)
depends on severity and duration. An acute decrease in serum magnesium will increase PTH
secretion, whereas a prolonged severe deficiency results in decreased PTH secretion. There is also
evidence that the action of PTH is decreased with chronic magnesium deficiency. Hence, this patient
is suffering from "functional" hypoparathyroidism, causing low serum calcium (choice A) that can
produce weakness, tremors, muscle fasciculations, and seizures. In patients with magnesium
deficiency, magnesium administration will produce a prompt increase in plasma PTH with subsequent
restoration of serum calcium concentration to normal.

20

With functional hypoparathyroidism bone density would be decreased (not increased, choice C). The
combination of decreased PTH secretion (not increased, choice D) and decreased effectiveness of
PTH produce hypocalcemia and hyperphosphatemia (not hypophosphatemia, choice B).
Phosphorous is needed for development of high-energy compounds, nucleic acids, and bone matrix.
Respiratory rate 15; blood pressure 120/80 mm Hg; cardiac output 5 L/min; pulse 50
52. Basal measurements are shown above. What is the stroke volume during resting
conditions (in mL/min)?
A. 50
B. 75
C. 100
D. 125
E. 150
The correct answer is C. The cardiac output (CO) is equal to the volume of blood ejected from the
heart during each systole (i.e., the stroke volume; SV) multiplied by the number of times the heart
beats each minute (heart rate; HR). In other words, CO = SV x HR. Therefore, SV = CO/HR, and
because CO = 5,000 mL/min and HR = 50 beats/min, SV = 5,000/50 = 100 mL/beat.
In the examination of a single cardiac cycle, stroke volume is the most important factor to consider. In
the consideration of cardiac function over time, clinicians are concerned with cardiac output (the
volume of blood ejected in 1 minute). Physiologically, cardiac output is precisely regulated so that
peripheral tissues receive an adequate circulatory supply under a variety of conditions; for example,
stroke volume can almost triple when neccessary.
53. Which of the following would most likely be an associated finding during fasting for a
patient with an insulin-secreting tumor that is localized to the tail of the pancreas?
A. Glycosylated hemoglobin level is increased
B. Plasma concentration of C peptide is decreased
C. Plasma concentration of glucagon is decreased
D. Plasma concentration of glucose is increased
E. Plasma concentration of proinsulin is increased
The correct answer is E. Hypersecretion of insulin by a pancreatic cell tumor is a major cause of
fasting hypoglycemia (plasma glucose is not increased, choice D). Symptoms are related to
neuroglycopenia and could include recurrent central nervous system dysfunction during fasting or
exercise. Although proinsulin only makes up approximately 20% of plasma immunoreactive insulin in
normal individuals, in patients with an insulinoma it contributes 30-90% of the immunoreactive insulin.
The increased secretion of insulin by the tumor will also lead to an increase in C peptide secretion
(not decreased, choice B) since cells secrete insulin and C peptide on a one-to-one molar ratio.
Given the prolonged hypoglycemia, the amount of glycosylated hemoglobin may also be decreased,
although this is not a universal finding. Certainly an increase in glycosylated hemoglobin would not be
expected (choice A). Glucagon secretion is increased by hypoglycemia and its plasma level in a
patient with an insulinoma would be expected to be increased compared with normal (not decreased,
choice C).
54. Physical examination of a woman with advanced, metastatic lung cancer demonstrates
hyperpigmentation of skin, even in areas protected from the sun. ACTH levels are found to be
elevated. Tumor inducement of which endocrine organ is most likely?
A. Adrenal gland
B. Endocrine pancreas
C. Ovaries
D. Pituitary gland
E. Thyroid gland
The correct answer is A. This is Addison disease, in which severe adrenal disease produces
adrenocortical insufficiency. Causes include autoimmune destruction, congenital adrenal hyperplasia,
hemorrhagic necrosis, and replacement of the glands by either tumor (usually metastatic) or
granulomatous disease (usually tuberculosis). The symptoms can be subtle and nonspecific (such as

21

those listed). Skin hyperpigmentation is a specific clue that may be present on physical examination,
suggesting excess pituitary ACTH secretion. Most patients have symptoms (fatigue, gastrointestinal
distress) related principally to glucocorticoid deficiency. In some cases, however, mineralocorticoid
replacement may also be needed for symptoms of salt wasting with lower circulating volume.
Except in the case of primary pancreatic cancer, complete tumor replacement of the endocrine
pancreas (choice B) would be uncommon. In any event, pancreatic involvement would be associated
with diabetes mellitus.
Involvement of the ovaries (choice C) by metastatic tumor (classically gastric adenocarcinoma) would
produce failure of menstruation.
Involvement of the pituitary gland (choice D) could produce Addisonian symptoms, but the pigmented
skin suggests a primary adrenal problem rather than pituitary involvement.
Replacement of the thyroid gland (choice E) by tumor would cause hypothyroidism with lesser
degrees of fatigue and cold intolerance; this is an unusual cause of hypothyroidism.
55. The resultant decrease in blood flow causes by partial obstruction of the renal artery by an
atherosclerotic plaque increases release of an enzyme from which of the following structures?
A. Afferent arterioles
B. Arcuate arteries
C. Juxtaglomerular cells
D. Kupffer cells
E. Proximal convoluted tubule
The correct answer is C. The juxtaglomerular cells are in the wall of the afferent arteriole, close to
the glomerulus. Together the macula densa and juxtaglomerular cells form the juxtaglomerular
apparatus. In response to decreased blood pressure, they secrete renin, an enzyme that converts
angiotensinogen to angiotensin I. Angiotensin converting enzyme, found in the lungs, converts
angiotensin I to angiotensin II. Angiotensin II increases peripheral vascular resistance directly and
stimulates aldosterone secretion, resulting in increased reabsorption of sodium and water in the distal
convoluted tubules.
The afferent arteriole (choice A) carries blood from the interlobular arteries to the glomerulus.
Filtration of blood occurs in the glomerulus, with the filtrate entering Bowman's capsule.
The arcuate arteries (choice B) are branches of the interlobar arteries of the kidney. The arcuate
arteries lie in the corticomedullary junction of the kidney and give rise to interlobular arteries, which
enter the cortex of the kidney and supply the glomeruli.
Kupffer cells (choice D) are found in the liver, along the sinusoids. They are phagocytic cells that are
part of the reticuloendothelial system.
The proximal convoluted tubule (choice E) is directly continuous with Bowman's capsule. Most of the
resorption of the glomerular filtrate occurs in this part of the nephron.
56. In a person weighing 75 kg, the volume of total body water is 45 L. What is the interstitial
volume and plasma volume?
A. 4 L, 12 L
B. 12 L, 4 L
C. 8 L, 8 L
D. 5 L, 10 L
The correct answer is B. Interstitial volume = 12 L and plasma volume = 4 L. The calculation is
based on the following estimates: of total body weight, approximately 60% is water. One third of this
water is extracellular, and two thirds is intracellular. Of the one third that is extracellular, approximately
one quarter is plasma volume, and the other three quarters is interstitial volume. Using the figures in
the problem:

22

57. When a 10-day-old infant jumps and jerks his arms to his chest in response to a noice, the
infant is exhibiting the
A. Babinski reflex
B. deep tendon reflex
C. Moro reflex
D. palmar reflex
E. tonic neck reflex
The correct answer is C. The identification of reflexes is an important part of human physiology. The
Moro reflex can be elicited in the infant by any startling event; it consists of extension and abduction of
the arms, followed by flexion and adduction of the arms. This is a normal reflex that appears between
the ages of 25 and 36 weeks of gestation, and will normally disappear between 3-6 months of age.
The Babinski reflex (choice A) is elicited when the lateral surface of the sole of the foot is stroked
resulting in the great toe going up and the other toes fanning. It normally disappears at 1 year of age.
Deep tendon reflexes (choice B) can be elicited by tapping a tendon with a reflex hammer, which
stretches the tendon, producing contraction in the corresponding muscle. These are present
throughout life.
The palmar grasp reflex (choice D) is characterized by the infant's hand closing over an object that is
placed in the palm of the hand. This reflex normally disappears at 2 months of age.
The tonic neck reflex (choice E) consists of extension of the ipsilateral leg and flexion of the
contralateral arm and leg when the head is turned. This reflex normally disappears between 7-8
months of life.
58. Which of the following is the primary opsonin in the complement system?
A. C1q
B. C3b
C. C5
D. C5a
E. Factor B
The correct answer is B. Complement consists of 11 plasma proteins that interact in a chain reaction
following exposure to activated antibodies or surfaces of certain pathogens. Complement promotes
cell lysis, phagocytosis, and often defense mechanisms. C3b is the most critical molecule in both the
classical and alternative complement pathways. C3, the most abundant protein of all the complement
proteins, is cleaved into C3a and C3b. C3b attaches to bacterial surfaces for opsonization by
phagocytes. C3a binds to mast cells and basophils, activating them and producing histamine release.
C1q (choice A) is a complement component in the classical pathway. It binds to the constant heavy
domain of an IgG molecule that has reacted with the bacterial surface epitope. C1q is not involved in
opsonization as it simply functions as an enzyme in the early complement cascade.
59. Constriction of which of the following types of blood vessels is most important for
minimizing the decrease in mean systemic filling pressure caused by loss of 1 liter of blood?
A. Aorta and large arteries
B. Arterioles
C. Capillaries
D. Small arteries
E. Venules and veins
The correct answer is E. The venous system serves as an important blood reservoir for the
circulation. When blood is lost from the body and blood pressure begins to decrease, pressure

23

reflexes are elicited that send sympathetic nerve signals to venules and veins causing them to
constrict. By "tightening" the circulation and taking up much of the "slack" caused by the blood loss,
nearly normal function can usually be restored with up to a 20% loss of blood volume.
Constriction of the arterial system has relatively little effect on mean systemic filling pressure because
the arterial system (choices A, B, and D) contains a relatively small volume of blood.
Capillaries do not constrict because they do not contain smooth muscle cells in their walls (choice C).
60. Which of the following regulates the frequency of gastrointestinal slow waves?
A. Circadian rhythms
B. Dietery intake of fiber
C. Intrinsic pacemaker regions within the smooth muscle
D. Local levels of cholecystokinin secretion
E. Systemic levels of catecholamines and corticosteroids
The correct answer is C. Slow waves are oscillating membrane potentials that are generated locally
within the gastrointestinal smooth muscle. In each region of the gastrointestinal tract, intrinsic
pacemaker cells within the smooth muscle independently generate slow waves. Slow-wave
frequencies are relatively constant and are characteristic of each region of the gut.
There is no evidence that either circadian rhythms (choice A) or dietary fiber intake levels (choice B)
have any effect on gastrointestinal slow wave frequencies.
Cholecystokinin (choice D), corticosteroids, and sympathetic stimulation (choice E) may modify the
frequency of action potentials and contractions by affecting the amplitude, rather than the frequency,
of the slow waves.
61. A 72-year-old woman with mild heart failure is treated overzealously with a thiazide
diuretic. A few days later, the woman complains of muscle weakness, and laboratory tests
demonstrate hypokalemia. Which of the following is most likely increased in this woman?
A. Arterial H+ concentration
B. Plasma aldosterone
C. Plasma sodium
D. Potassium retention
E. Sodium retention
The correct answer is B. Aldosterone is a minerlocorticoid produced by the zona glomerulosa or the
adrenal cortex. It stimulates sodium and water conservation at the kidneys. This hormone is secreted
in response to the presence of angiotensin II. Diuretics lead to aldosterone excess and hypokalemia
by a variety of mechanisms. (1) Diuretic induces volume depletion stimulated the formation of
angiotensin II, which, in turn, causes a secondary increase in plasma aldosterone concentration. This
increase in plasma aldosterone stimulates potassium excretion, contributing to the hypokalemia. (2)
The saline diuresis increases sodium delivery to the collecting tubule. The increased availability of
sodium along with the elevated plasma aldosterone augments sodium reabsorption in the collecting
tubule, thereby raising luminal negativity. This high luminal negativity in the collecting tubule promotes
secretion of cations, especially hydrogen ions, which raises bicarbonate reabsorption. (3) The saline
diuresis causes rapid fluid flow in the distal tubule, which, in turn, keeps luminal potassium
concentration low by carrying it away and thus preventing the assimilation of any potassium that
enters the lumen. This low luminal concentration of potassium creates a steep concentration gradient
for additional potassium losses in the urine. The treatment of edema with thiazide or loop diuretics is
the most common cause of metabolic alkalosis. Arterial pH is increased and arterial H+ concentration
(choice A) is decreased with metabolic alkalosis.
Sodium depletion tends to decrease plasma sodium (choice C) levels, although the effect is usually
small. Overuse of the thiazide diuretic has caused depletion of sodium and potassium by the kidneys,
not retention of sodium and potassium (choices D and Choice E).

24

62. Increasing urine flow rate from 1 mL/min to 10 mL/min will significantly increase the
clearance of
A. creatinine
B. inulin
C. penicillin
D. phosphate
E. urea
The correct answer is E. Urine flow rate is controlled primarily by antidiuretic hormone (ADH), which
regulates the amount of pure water retained in the urine. ADH also controls the reabsorption of urea in
the papillary collecting duct. High urine flow rates indicate low ADH, which increases urea clearance.
In contrast, low urine flow rates indicate high ADH, which results in a greater reabsorption of urea and
a lower urea clearance.
The concentration (and osmolarity) of all other solutes (choices A, B, C, and D) varies inversely with
urine flow rate, resulting in no change in clearance. Recall that clearance = (urine concentration x
urine flow rate)/plasma concentration.
63. A 3-year-old boy with chronic granulomatous disease (CGD) has a culture of a purulent
discharge from an abscess that shows Staphylococcus aureus. Immunoglobulin and
complement levels are normal. The boy received all of the standard immunizations without any
adverse effects. His immune deficiency most likely involves which of the following?
A. B cells
B. Chemotaxis
C. IgG subclass 2
D. Phagocytic cells
E. T cells
The correct answer is D. This patient has chronic granulomatous disease (CGD), which is
associated with defective phagocytes. CGD is inherited as an X-linked or autosomal recessive trait
and consists of a group of heterogenous disorders of oxidative metabolism affecting the normal
respiratory burst, consisting of increased oxygen consumption, leading to the generation of
intracellular hydrogen peroxide and superoxide radicals by phagocytic cells. The hydrogen peroxide
and superoxide radicals are required for the killing of ingested intracellular organisms. The reaction is
catalyzed by an NADPH oxidase that seems to be defective in the phagocytes of patients with CGD.
In CGD patients, the engulfment process by the phagocytic cells is normal; however, the pathogenic
organism is not killed, but persists within the cell. The patients suffer from infections with organisms
that are normally considered of low virulence (e.g., Staphylococcus aureus, Aspergillus, Candida,
Escherichia coli, and Serratia marcescens). The nitroblue tetrazolium (NBT) test is used to screen for
CGD. B cells (choice A) are responsible for antibody-mediated immunity, and the immunoglobulin
levels in this patient were normal. Chemotaxis (choice B) is important in the migration of the
phagocytic cell toward the site of infection, not in intracellular killing. IgG subclass 2 (choice C) is the
most important immunoglobulin in the protection against encapsulated organisms. T cells (choice E)
are important in the host response to viruses, fungi, and intracellular bacterial organisms. The patient
was immunized normally (including the live, attenuated MMR vaccine), making T-cell dysfunction
unlikely.
64. Which of the following indices would be expected to be higher in an individual living at sea
level versus an individual living in the mountains?
A. Diameter of pulmonary vessels
B. Erythropoietin production
C. Mitochondrial density in a muscle biopsy
D. Renal bicarbonate (HCO3-) excretion
E. Respiratory rate

25

The correct answer is A. Several physiologic changes occur in a person living at high altitude. The
diminished barometric pressure at high altitude causes alveolar hypoxia and arterial hypoxia.
Pulmonary vasoconstriction occurs in response to alveolar hypoxia; therefore, the diameter of the
pulmonary vessels would be greater in the brother living at sea level. All the other choices describe
physiologic processes that would be enhanced by living at high altitude.
Increased erythropoietin production (choice B) caused by arterial hypoxia leads to increases in
hematocrit and increased oxygen carrying capacity in people living at high altitude.
Mitochondrial density increases (choice C) in people chronically exposed to the hypoxemia caused
by living at high altitude.
At high altitudes, the ventilation rate increases, causing a respiratory alkalosis. The kidney then
compensates by increasing the excretion of HCO3- (choice D).
Increasing the rate of respiration (choice E) is a very useful adaptation to the hypoxic conditions of
high altitude. The primary stimulus is the hypoxic stimulation of peripheral chemoreceptors.
65. Which of the following substances will be elevated in the plasma of a patient with
nephrotic syndrome caused by membranous glomerulonephritis?
A. Albumin
B. Ammonia
C. Cholesterol
D. Glucose
E. Potassium
The correct answer is C. Nephrotic syndrome is characterized by urinary protein excretion,
hypoalbuminemia and peripheral edema. The nephrotic syndrome describes a group of laboratory
findings associated with glomerular diseases that share the common characteristic of "leaky
glomeruli." Large biochemicals, normally unable to cross out of glomerular capillaries into Bowman
space, are lost into the urine. Serum proteins are lost in large quantities and can be detected as both
hypoproteinemia and massive proteinuria. Albumin (choice A), a relatively small plasma protein (MW
66,000) will be lost very readily, leading to hypoalbuminemia. Serum concentrations of small
compounds such as potassium (choice E) and glucose (choice D), which are highly permeable in the
normal glomerulus, are unaffected by glomerulonephritis producing the nephrotic syndrome. Similarly,
blood urea nitrogen is unaffected, and serum ammonia levels (choice B) are unchanged. The final
component of the nephrotic syndrome (besides hypoproteinemia, hypoalbuminemia and massive
proteinuria) is hyperlipidemia. It is apparently a function of both increased hepatic fat synthesis and
decreased fat catabolism. Increased cholesterol (choice C), triglycerides, and lipoproteins are found
in serum in membranous glomerulonephritis, and these lipids also leak into the urine, producing
lipiduria.
66. An endometrial biopsy demonstrates a thick endometrium with long, coiled glands lined
by a columnar epithelium with prominent cytoplasmic vacuoles adjacent to the gland lumen.
Earlier in the menstrual cycle, the glands were much smaller and were lined with cells that did
not have vacuoles. Which of the following hormones is primarily responsible for inducing this
change in appearance?
A. Aldosterone
B. Cortisol
C. Estrogen
D. Progesterone
E. Thyroxine
The correct answer is D. The endometrial phase with small glands is the proliferative phase; the
one with large glands with secretory cells is the secretory phase. Estrogen (choice C) is necessary
for both phases, but it is the addition of progesterone (choice D), secreted by the corpus luteum after
the Graafian follicle ruptures, that triggers the switch from proliferative to secretory endometrium.

26

Progesterone prepares the uterus for implantation and prepares mammary glands for secreting
functions. Progesterone is stimulated by leutinizing hormone from the anterior pituitary.
Glucocorticoids (choice B) and the mineralocorticoid aldosterone (choice A) are secreted by the
adrenal glands. They do not produce the endometrial changes described. Glucocorticoids cause a
release of amino acids from skeletal muscles, lipids from adipose tissue and promote liver glycogen
and glucose formation. They also have anti-inflammatory effects. Mineralocorticoids increase renal
absorption of sodium ions and water and accelerate urinary loss of potassium ions. Thyroxine (choice
E) is secreted by the thyroid gland, and is unrelated to the observed morphologic changes in the
endometrium. It is responsible for metabolic regulation.
67. In which segment of the systemic circulation does the greatest decrease in blood pressure
occur?
A. Aorta and large arteries
B. Arterioles
C. Capillaries
D. Small arteries
E. Venules and small veins
The correct answer is B. As blood flows through the systemic circulation the mean pressure of the
blood decreases from approximately 100 mm Hg in the aorta to approximately 0 mm Hg in the right
atrium. The mean blood pressure is approximately the same in all portions of the aorta (choice A)
and it only decreases by a few mm Hg in the large arteries (choice A). The blood pressure decreases
by 10-20 mm Hg in the small arteries (choice D) so that blood entering the arterioles has a pressure
averaging 80-90 mm Hg. By the time the blood has reached the ends of the arterioles (choice B) the
pressure has decreased to approximately 35 mm Hg. The pressure decreases another 25 mm Hg as
it flows through the capillary network (choice C), so that blood entering the venules has a pressure of
approximately 10 mm Hg. The blood pressure decreases by approximately 10 mm Hg as it flows
along the venous system (choice E) to the right atrium. This decrease in blood pressure (expressed
as percent of total peripheral resistance)(from 100 to 0 mm Hg) through the circulatory tree is
summarized in the following table. Type of blood vessel: decrease in blood pressure (mm Hg)
(percent of total peripheral resistance) through the circulatory tree. Aorta and large arteries: less than
1; Small arteries: 10-20 ; Arterioles: 50 ; Capillaries: 25 ; Venules and small veins: 9 ; Vena cava: less
than 1
68. Which of the following adjustments have probably occurred in the cardiac cycle if the
pulse is greater than 200/min and the blood pressure is 75/40 mm Hg?
A. Diastolic time has decreased and systolic time had increased
B. Diastolic time has decreased but systolic time had decreased more
C. Systolic time has decreased and diastolic time has increased
D. Systolic time has decreased but diastolic time has decreased more
E. Systolic time has decreased but diastolic time has not changed
The correct answer is D. Under normal conditions one third of the cardiac cycle is spent in systole
and two thirds is spent in diastole. As pulse increases dramatically, the time spent in diastole
decreases precipitously but the time spent in systole decreases only slightly. A large increase in pulse
must produce a decrease in diastole and systole (compare with choice A). The major change with
increased pulse is in diastole, not systole (compare with choice B). Pulse cannot increase if diastolic
time increases (choice C). An increase in pulse must be accompanied by a decrease in diastolic time
(compare with choice E).
69. Following a serious accident, a patient's rib cage and abdominal muscles become
completely paralyzed. He is still able to breathe, however, because his diaphragm continues to
contract. At which level might his spinal cord injury have occurred?
A. C2
B. C7
C. L3
D. T5
E. T12

27

The correct answer is B. Trauma to the lower cervical cord (at C7, for example) can cause the
pattern described in the question, because the lesion is below the origin of the phrenic nerve (C3-C5
nerve roots), but above the origin of the nerves innervating the muscles of the rib cage and abdomen.
Trauma high in the neck at C2 (choice A), above the origin of the phrenic nerve, would cause
diaphragmatic paralysis also. A lesion at L3 (choice C)or T12(choice E) would spare all of the
accessory respiratory muscles and the diaphragm. A lesion at T5 (choice D) would spare part of the
accessory muscles of the rib cage and the diaphragm.
70. Which of the following events occurs first in the differentiation sequence of human B cells
in the bone marrow?
A. Cytoplasmic mu chains present in the B cell
B. Immunoglobulin heavy chain rearrangement
C. Immunoglobulin light chain rearrangement
D. Surface IgD and IgM present on the B cell
E. Surface IgM present on the B cell
The correct answer is B. The first event that occurs in the pre-B cell (progenitor) is gene
rearrangement of the heavy chain. The D gene and J gene recombination event occurs first, followed
by V recombination with the D-J region.
The cytoplasmic mu chains (choice A) are the result of immunoglobulin heavy chain rearrangement,
the production of the messenger RNA from this rearrangement, and the ribosomal synthesis of the
protein mu chain. This is the second event that occurs in the B-cell maturational sequence.
Light chain rearrangement (choice C) occurs when recombination events occur with the V gene and J
gene from the light chain germ line. After this recombination has occurred, and the messenger RNA
for this germ line has produced the light chain protein, the light chains and heavy chains form.
The complete IgM molecule and IgD molecules (choice D) are present on the surface of only the
mature B cells. This event is the last event to occur during the differentiation and development of B
cells in the bone marrow.
The complete IgM molecule (choice E) is present on the surface of the immature B cell. This event is
one of the last events to occur in the differentiation and development of B cells.
71. A patient experiences episodic abdominal pain, especially after the ingestion of a fatty
meal. The action of which of the following hormones is most likely to be responsible for the
postprandial intensification of the symptoms?
A. Cholecystokinin (CCK)
B. Gastrin
C. Pepsin
D. Secretin
E. Somatostatin
The correct answer is A. This symptomatology is consistent with gallstones (cholelithiasis). As would
be expected, contraction of the gallbladder following a fatty meal often exacerbates the pain caused
by gallstones. Cholecystokinin (CCK) is the hormone responsible for stimulation of gallbladder
contraction; the release of CCK is stimulated by dietary fat. It is produced in I cells of the duodenum
and jejunum. In addition to gallbladder contraction, CCK also stimulates pancreatic enzyme secretion
and decreases the rate of gastric emptying.
Gastrin (choice B) is produced by the G cells of the antrum and duodenum. Gastrin stimulates the
secretion of HCl from the parietal cells and pepsinogen from the chief cells of the stomach. Gastrin
secretion is stimulated by gastric distention, digestive products (e.g., amino acids), and vagal
discharge. Pepsin (choice C) is a protease produced by the chief cells of the stomach (as
pepsinogen). It is involved in the digestion of proteins. Pepsinogen release is stimulated by vagal

28

stimulation, gastrin, local acid production, secretin, CCK, and histamine. Secretin (choice D) is
produced by the S cells of the duodenum. It is secreted primarily in response to acidification of the
duodenal mucosa. Secretin stimulates the secretion of bicarbonate-containing fluid from the pancreas
and biliary ducts. This neutralization allows pancreatic enzymes to function. Secretin also inhibits
gastric acid production and gastric emptying.
Somatostatin (choice E) is produced by the D cells of the pancreatic islets and in the gastric and
intestinal mucosa. Somatostatin is an inhibitory hormone; it inhibits most gastrointestinal hormones,
gallbladder contraction, gastric acid and pepsinogen secretion, pancreatic and small intestinal fluid
secretion, and both glucagon and insulin release.
72. Release of which of the following peptides leads to an increase in the secretion of
pancreatic enzymes into the small intestine?
A. Cholecystokinin
B. Gastrin
C. Motilin
D. Secretin
E. Somatostatin
The correct answer is A. Release of cholecystokinin is stimulated by the presence of peptides,
amino acids, or fatty acids in the small intestine. Cholecystokinin acts on the pancreas to stimulate
secretion of pancreatic enzymes that aid in the digestion of these compounds.
Gastrin (choice B) secretion, which is stimulated by the presence of peptides or amino acids in the
lumen of the stomach, produces an increase in gastric H+ secretion.
Motilin (choice C) is a hormone that regulates the migrating myoelectric complex, a series of
contractions that occurs during fasting, clearing the stomach and small intestine of any residual food.
Secretin (choice D) secretion, which is stimulated by the presence of H+ and fatty acids in the
duodenum, causes an increase in pancreatic and biliary HCO3-release and decrease in gastric H+
release.
Somatostatin (choice E) secretion, which is stimulated by the presence of H+ in the lumen, results in
decreased release of all gastrointestinal hormones and decreased H+ secretion in the stomach.
73. A 60-year-old man presents with signs of hypoxemia due to hypoventilation. Which of the
following could be the cause of hypoxemia in this patient?
A. Anemia
B. Fibrosis
C. Chronic obstructive pulmonary disease (COPD)
D. Right-to-left cardiac shunt
The correct answer is C.Hypoventilation is the underlying mechanism for developing hypoxemia in
the COPD patient. Decreased oxygen-carrying capacity is the underlying cause for hypoxemia in
anemia (choice A). Hypoxemia in fibrosis (choice B) is due to a V/Q mismatch. The mechanism for
hypoxemia in right-to-left cardiac shunt (choice D) is a venous admixture.
74. The blood from an 8-year-old boy was analyzed by flow cytometry. The exact number of B
cells was counted. Which of the following cell surface markers was likely used to identify the B
cells in this blood sample?
A. CD3
B. CD4
C. CD8
D. CD19
E. CD56

29

The correct answer is D. B cells are lymphocytes capable of differentiating into the plasma cells that
produce antibodies. The best markers for identification of B cells are CD19, CD20, and CD21. The
CD21 marker is a receptor for EBV (Epstein-Barr virus).
The CD3 marker (choice A) is present on all T cells with either a CD4 or CD8 marker. This is the
marker that is used to identify total T cell count in a blood sample. The CD3 marker is used for signal
transduction in the different T cells.
The CD4 marker (choice B) is used to identify T helper cells. They are also important sites for HIV
viral infection gaining entry into these cells. These are the cells that recognize exogenous peptides
presented on MHC class II molecules by macrophages. CD4+ T helper cells are also involved in cellmediated delayed hypersensitivity, production of cytokines for stimulation of antibody production by B
cells, and stimulation of macrophages. The CD8 marker (choice C) is used to identify cytotoxic T
cells. These are the cells that recognize viral epitopes attached to the MHC class I molecules of a
virally infected cell. The CD56 marker (choice E) is used to identify NK (natural killer) cells. These
cells are important in innate host defense, specializing in killing virally infected cells and tumor cells by
secreting granzymes and perforins.
75. A deficiency of the complement protein C4 would inhibit which of the following
complement activities?
A. Completion of the classic pathway to the splitting of C3
B. Formation of C3b for opsonization
C. Formation of C5 convertase by way of the alternative pathway
D. Formation of C5a for chemotactic attractant for neutrophils
E. Formation of the membrane attack complex
The correct answer is A. By definition, complement is 11 plasma proteins that interact in a chain
reaction following exposure to activated antibody surfaces at certain pathogens. This will promote cell
lysis, phagocytosis, and other defense mechanisms. The classic complement pathway involves C1,
C4, and C2 to the point where C3 splits. C3 is then split to yield C3b and C3a. A deficiency of C4
would have no effect on any of the other answer choices listed.
The complement protein C3 can be split into C3a and C3b (choice B) using the alternative pathway.
The additional proteins required in this pathway would be factors B and D and properdin. The C5
convertase enzyme can be formed in the alternative pathway (choice C) without using C4. The C5
molecule could be split into C5a (choice D) and C5b using the alternative pathway, without the
involvement of C4. The membrane attack complex (choice E; C5b, C6, C7, C8, and polymers of C9)
lyses the pathogenic cell. This process does not require the alternative pathway and would not require
C4.
76. What substance allows platelet adhesion to exposed collagen fibers following surgery?
A. Factor VIII
B. Factor IX
C. Fibronectin
D. Tissue factor
E. Von Willebrand factor
The correct answer is E. Von Willebrand factor (vWF) is a self-polymerizing clotting protein present
in the serum and the subendothelial basal lamina, which has binding sites for collagen, platelets, and
fibrin. At a site of injury, vWF forms the bridge between the exposed collagen fibers and platelets in
circulation, stimulating platelet degranulation and initiating the cellular component of the clotting
cascade. An equally important role for vWF is binding platelets to the newly formed fibrin strands in a
blood clot. Factor VIII (choice A) and Factor IX (choice B) are clotting proteins of the intrinsic
pathway. Factor VIII acts in concert with activated Factor IX (IXa) to cleave Factor X to Xa. Xa is the
prothrombin activator central to both the intrinsic and extrinsic pathways. Fibronectin (choice C) is a
serum protein that acts as an opsonin for phagocytic cells in clots. Fibronectin binds non-specifically
to bacteria and other materials in the newly forming clots, and draws the cell membrane of
phagocytes into contact with these substances. Tissue factor (choice D) is a protein released from
injured tissues that works in concert with Factor VII to initiate the extrinsic pathway of coagulation.
Like Factors VIII and IX, tissue factor and Factor VII cleave Factor X to Xa.

30

77. The clearance of several substances was measured at a constant glomerular filtration rate
and constant urine flow rate, but at increasing plasma concentrations of the substance. Under
these conditions, clearance will increase at high plasma concentrations for which of the
following substances?
A. Creatinine
B. Mannitol
C. Penicillin
D. Phosphate
E. Urea
The correct answer is D. Clearance of a substance will change with increasing plasma concentration
if that substance is secreted or reabsorbed by a facilitated mechanism. Most phosphate is reabsorbed
by a facilitated sodium-phosphate cotransporter in the proximal tubule. As the concentration of
phosphate increases, the transporter saturates, leaving more phosphate available for excretion.
78. During a fast, a brief phase of intense sequential contractions begins in the stomach and
gradually migrates to the ileum. Release of which of the following intestinal hormones is most
likely responsible for this observed effect?
A. Cholecystokinin
B. Gastrin
C. Gastrin-releasing peptide
D. Motilin
E. Secretin
The correct answer is D. Motilin is a hormone released by the small intestine during the fasting
state. Its waxing and waning blood levels correlate with the initiation and ending of migrating motor
complexes (MMC). Furthermore, injection of motilin has been shown to evoke MMC activity. The
MMC typically begins in the stomach, and over a 90-120 minute period, migrates to the ileum, where it
dies out. As one complex dies out in the ileum, another complex begins in the stomach provided the
fasting state continues. Eating a meal interrupts the MMC activity.
Cholecystokinin (choice A) is released during the intestinal phase of the digestive period (not during a
fast). Its secretion is evoked by the presence of fat and protein digestion products in the duodenum. It
induces contraction of the gall bladder and relaxation of the sphincter of Oddi.
Gastrin (choice B) is released from G cells in the antrum, mostly during the gastric phase of the
digestive period (not during a fast). It tends to increase stomach motility, although the rate of emptying
is decreased because gastrin also causes the pyloric sphincter to contract. It also may contribute to
the increase in ileal and colonic motility as part of the gastroileal and gastrocolic reflexes, respectively.
Gastrin-releasing peptide (choice C) mediates the neural release of gastrin. Antral enteric neurons
that are activated by vagal efferents or by local reflexes release gastrin-releasing peptide, which
stimulates the G cells to secrete gastrin. Secretin (choice E) is a duodenal hormone that is released
during the intestinal phase of the digestive period (not during a fast). Its secretion is evoked by a
duodenal pH less than 4.5. Secretin tends to decrease the rate of stomach emptying.
79. Which of the following conditions is associated with a lifelong requirement for
administration of parenteral vitamin B12?
I. Pernicious anemia II. Removal of colon III. Removal of gallbladder IV. Removal of ileum V.
Removal of jejunum VI. Removal of stomach
A. I only
B. I and V
C. I, IV, and V
D. I, IV, and VI
E. III only

31

The correct answer is D. This question tests your knowledge of how vitamin B12 is normally
absorbed. In summary, parietal cells in the gastric lining secrete a glycoprotein called intrinsic factor
into the gastric lumen. This protein binds to vitamin B12, protecting it from degradation and allowing
for its eventual absorption. At the level of the ileum, B12 bound to intrinsic factor is actively
reabsorbed. Therefore, a loss of intrinsic factor or of its reabsorption site, the ileum, would lead to the
need for lifelong injection of vitamin B12. Removal of the stomach would obviously lead to loss of
intrinsic factor. Pernicious anemia is actually an autoimmune disease that targets the gastric
epithelium. In many cases, patients have autoantibodies against intrinsic factor, preventing the
absorption of ingested vitamin B12.
Removal of the colon, jejunum, or gallbladder would not affect reabsorption of intrinsic factor or B12.
A total absence of bile (which occurs with bile duct blockage, but not with cholecystectomy) may lead
to malabsorption of fat-soluble vitamins such as A, D, E, and K, but not of B12.
80. Which of the following procoagulant proteins binds to tissue factor exposed beneath the
endothelium of traumatized blood vessels and initiates the clotting cascade?
A. Factor V
B. Factor VII
C. Factor X
D. Fibrinogen
E. Prothrombin
The correct answer is B. The extrinsic pathway of clotting begins with tissue factor binding to Factor
VII or Factor VIIa. All other clotting proteins require proteolytic cleavage to become active; however,
Factor VII has a low level of activity in its inactive form and can act with tissue factor and
phospholipids to initiate the clotting cascade. In the extrinsic pathway, Factor VII cleaves Factor X to
Xa (choice C), which acts in concert with Factor V (choice A) to cleave prothrombin to thrombin
(choice E). The final step in the coagulation pathway is the cleavage of fibrinogen to fibrin by
thrombin (choice D). Fibrin polymerizes and cross-links, thereby forming a hemostatic net of insoluble
protein.
81. Which sequence of the numbered statements below correctly describes the pathway of
lymphatic flow on the left side of the body?
1) Junction of the left internal jugular and left subclavian 2) Lymph capillaries 3) Thin lymph
vessels 4) Thoracic duct
A. 2-1-3-4
B. 2-3-1-4
C. 2-3-4-1
D. 2-4-1-3
E. 2-4-3-1
The correct answer is C. On the left side of the body, the lymphatic fluid flows from the lymphatic
capillaries to the thin lymphatic vessels and then to the thoracic duct, which empties into the junction
of the left internal jugular and left subclavian veins. On the right side, the lymphatic fluid flows from the
lymphatic capillaries to the thin lymphatic vessels to the right thoracic duct, which empties into the
junction of the right internal jugular and the right subclavian veins.
82. The oxygen-hemoglobin dissociation curve shifts to the left under which of the following
circumstances?
A. Carbon monoxide poisoning
B. Decreased pH
C. Increased 2,3-diphosphoglycerate (2,3-DPG)
D. Increased PCO2
E. Increased temperature

32

The correct answer is A. The loading of O2 is facilitated when the oxygen dissociation curve shifts
to the left, and the unloading of O2 is facilitated when the oxygen dissociation curve shifts to the right.
Carbon monoxide (CO) poisoning is extremely dangerous for several reasons. CO left-shifts the
oxygen dissociation curve, which interferes with the unloading of O2. Also, CO, which has
approximately 240 times the affinity for hemoglobin than O2 does, preferentially binds to available
sites on hemoglobin.
The remaining answer choices all shift the oxygen-hemoglobin dissociation curve to the right. A good
way to remember the conditions that promote dissociation of O2 is to think of exercising muscle,
which has decreased pH (choice B) from lactic acid buildup and increased PCO2 (choice D),
increased 2,3-DPG (2,3-diphosphoglycerate; choice C) because of increased glycolysis, and
increased temperature (choice E).
82. A patient has recurrent attacks of asthma that occur most frequently after aspirin
administration. Which of the following is the pathogenetic mechanism in this form of asthma?
A. Direct release of bronchoconstrictor mediators
B. Enhanced sensitivity to vagal stimulation
C. Inhibition of cyclooxygenase pathway
D. Type I hypersensitivity reaction
E. Type IV hypersensitivity reaction
The correct answer is C. Aspirin-induced asthma is an infrequent form of asthma. It is related to the
direct pharmacologic action of aspirin on the metabolism of arachidonic acid. Aspirin inhibits the
cyclooxygenase pathway without affecting the lipoxygenase pathway, leading to a decreased ratio of
prostaglandins (bronchodilators) to leukotrienes (bronchoconstrictors). The disrupted balance
between these two groups of arachidonic acid metabolites leads to bronchoconstriction in
predisposed patients.
Direct release of bronchoconstrictor substances (choice A) is one of several pathogenetic
mechanisms that may mediate occupational asthma, triggered by inhalation of several chemicals,
including epoxy resins, plastics, cotton fibers, toluene, formaldehyde, and penicillin products.
Enhanced vagal stimulation (choice B) plays an essential role in nonatopic asthma. This variety of
asthma is initiated by viral infections of the upper respiratory tract (e.g., common cold and flu), which
appear to lower the threshold of respiratory mucosa to parasympathetic (vagal) stimulation and its
bronchoconstrictor influence on the lungs.
Type I hypersensitivity reactions (choice D) are crucial in the pathogenesis of atopic asthma following
prior exposure to several allergens. T-cell activation instructs B cells to produce IgE directed against a
given allergen. On reexposure, IgE on the surface of mast cells binds the allergen and induces mast
cell degranulation. The mediators released from mast cells cause bronchospasm and recruit more
inflammatory cells, including eosinophils, lymphocytes, and basophils. Type IV hypersensitivity
reactions (choice E) do not occur in asthma.
84.The hormone that is essential for nervous tissue maturation, ossification of bone,
conversion of carotene to vitamin A, and increasing absorption of glucose from the small
intestine is:
A. a peptide hormone released from the parathyroid
B. formed from iodine, peroxide, and thyroglobin
C. secreted by alpha cells of pancreatic islets
D. secreted from the adrenal medulla
The correct answer is choice B. Thyroid hormone is a major anabolic hormone essential for normal
growth and maturation. It is synthesized from iodine, peroxidase, and thyroglobulin through oxidation,
iodination, and coupling. Choice A Parathyroid hormone increases calcium reabsorption and
decreases phosphate reabsorption, thus increasing free calcium level. Choice C Glucagon is
secreted by alpha cells of pancreatic islets. It increases serum glucose through liver glycogenolysis
and gluconeogenesis. Choice D Most of the adrenal medulla secretion is epinephrine and some
norepinephrine. It increases glucose output by liver and muscle glycogenolysis and increased lipolysis
in adipose tissue.

33

85. In which portion of the kidney tubule is most water normally reabsorbed?
A. Cortical collecting duct
B. Distal tubules
C. Loops of Henle
D. Medullary collecting duct
E. Proximal tubule
The correct answer is E.choice C), 10% in the distal tubules (choice B), and less than 10% in the
collecting ducts (choices A and D); approximately 1 liter of water is normally excreted as urine each
day. The amount of water reabsorbed in the proximal tubule and loop of Henle is not affected by ADH,
because ADH does not affect tubular permeability in these segments of the nephron. ADH increases
the permeability of the distal tubules and collecting duct, however, which increases reabsorption of
water. When ADH levels are high, the urine output can decrease to less than 0.5 L/day; when ADH
levels are low, the output of urine can increase to more than 30 L/day. Even at these extremes,
however, most of the water in the glomerular filtrate is still reabsorbed in the proximal tubule.
86. Which of the following series of test results will confirm that a woman is postmenopausal?
A. Decreased LH, decreased FSH, increased estrogen
B. Decreased LH, increased FSH, decreased estrogen
C. Increased LH, decreased FSH, decreased estrogen
D. Increased LH, increased FSH, decreased estrogen
E. Increased LH, increased FSH, increased estrogen
The correct answer is D. During menopause there is a loss of functioning follicles in the ovaries such
that GnRH-stimulated LH and FSH secretion do not result in normal estrogen secretion. The low
estrogen levels cannot inhibit gonadotropin secretion in a negative-feedback fashion, resulting in very
high levels of LH and FSH. Choices A, B, C, and E do not accurately describe normal levels in
menopause.
87. Cancer of which of the following organs is the most likely cause of an elevation of serum
erythropoietin levels in a patient with a hematocrit of 52%?
A. Breast
B. Colon
C. Kidney
D. Stomach
E. Thyroid
The correct answer is C. This pathophysiology question is essentially asking, "Which of the following
organs produces erythropoietin?" The kidney normally produces erythropoietin, which stimulates the
erythroid cell line in the bone marrow. Renal cell carcinoma can lead to overproduction of
erythropoietin and thereby cause secondary polycythemia. Other causes of secondary polycythemia
are diseases that impair oxygenation, including pulmonary diseases (including smoking) and
congestive heart failure. Breast cancer (choice A) can present as an axillary mass or as Paget
disease of the nipple. Colon cancer (choice B) and stomach cancer (choice D) can present with
anemia secondary to blood loss. Thyroid cancer (choice E) can present with hypothyroidism.
88. In the clotting process, as the hemostatic plug develops, fibrin polymerizes into
monomeric threads that are held together by noncovalent bonds. Which clotting protein
increases the strength of the clot by cross-linking the newly formed fibrin threads?
A. Factor XIII
B. High molecular weight kininogen (HMWK)
C. Plasminogen
D. Thrombin
E. von Willebrand factor (vWF)

34

The correct answer is A. Fibrinogen is cleaved by thrombin twice as it is activated to form fibrin. The
initial cleavage causes it to polymerize, and the second causes it to branch. Thrombin also activates
Factor XIII and XIIIa, which cross-link the fibrin strands and strengthen the clot. High molecular weight
kininogen (HMWK; choice B) is a cofactor in the intrinsic pathway that converts Factor XI to XIa.
Plasminogen (choice C) is a central proenzyme in clot lysis. When plasminogen is converted to
plasmin, it digests fibrin threads and several protein factors, including Factors V, VIII, XII, and
prothrombin. Thrombin (choice D) is an enzyme derived from prothrombin. It converts fibrinogen to
fibrin and activates Factor XIII. von Willebrand Factor (vWF; choice E) is a tissue-bound protein that
is exposed with vascular trauma and helps in the process of platelet adhesion.
89. Which of the following agents affecting neuromuscular transmission acts by blocking
release of acetylcholine (ACH) from presynaptic terminals?
A. Curare
B. Neostigmine
C. Hemicholinium
D. Botulinus toxin
The correct answer is D. Botulinus toxin causes total blockade by blocking the release of ACH from
the presynaptic terminal. Curare (choice A) decreases the size of EPP by competing with ACH for
receptors on the motor endplate. Neostigmine (choice B) prolongs action of ACh, acting as an
anticholinesterase inhibitor. Hemicholinium (choice C) depletes ACH stores from the presynaptic
terminal.
90. During an exercise stress test, which of the following is most likely to occur in the skeletal
muscles?
A. Decreased blood flow
B. Decreased metabolite concentrations
C. Increased arteriolar diameter
D. Increased oxygen concentration
E. Increased vascular resistance
The correct answer is C. Blood flow can increase as much as 20-fold in exercising skeletal muscle,
which is a greater increase than in any other tissue in the body. This tremendous increase in blood
flow results almost entirely from the actions of local vasodilator substances on the muscle arterioles.
During exercise, the muscles use oxygen more rapidly than it can be delivered by the blood, which
decreases the oxygen concentration (choice D) in the tissues. The oxygen deficiency causes
vasodilator metabolites (choice B) such as adenosine, carbon dioxide, lactic acid, and others to
accumulate in the tissues. The vasodilator metabolites acting on the arterioles lead to a reduction in
vascular resistance (choice E) and an increase in blood flow (choice A).
91. In the transition from a Graafian follicle to a functional corpus luteum, which of the
following cellular events occurs?
A. Granulosa cells begin to express aromatase
B. Granulosa cells begin to express FSH receptors
C. Granulosa cells begin to express LH receptors
D. Theca cells begin to express LH receptors
E. Theca cells begin to express side-chain cleavage enzyme
The correct answer is C. The secretion of estrogen by the developing follicle can best be explained
using the "two cell" hypothesis. Theca cells are stimulated by LH (theca cells express LH receptors
before formation of the corpus luteum, choice D) to secrete the androgens androstenedione and
testosterone. The androgens then diffuse into the granulosa cells, where they are aromatized to
estrogens. Hence, theca cells express side-chain cleavage enzyme (first step in steroidogenesis)
before the formation of the corpus luteum (choice E). FSH stimulates aromatase activity in the
granulosa cells (receptors for FSH and aromatase enzyme are present before the formation of the

35

corpus luteum, choices A and B). The granulosa cells apparently have the ability to produce steroids
(progesterone), but lack 17-hydroxylase activity and cannot synthesize estrogen themselves. Only as
the follicle approaches ovulation do LH receptors begin to be expressed by the granulosa cells.
Estrogen and FSH probably are responsible for the change. After ovulation, the scar of the follicle
undergoes luteinization. The theca cells decrease 17-hydroxylase activity and secrete more
progesterone. The granulosa cells decrease aromatase activity and also secrete more progesterone.
92. What role do the lungs play in the pathogenesis of systemic hypertension?
A. The alveolar capillaries contain aldosterone
B. The alveolar capillaries contain angiotensin-converting enzyme (ACE)
C. The alveolar capillaries contain antidiuretic hormone (ADH)
D. The type II pneumocytes contain aldosterone
E. The type II pneumocytes contain angiotensin-converting enzyme (ACE)
The correct answer is B. Renin is a renal protease that is produced when there is decreased renal
perfusion. Renin then cleaves angiotensinogen (secreted into the plasma by the liver) to form
angiotensin I. Angiotensin I is cleaved by angiotensin-converting enzyme (found in very high
concentrations in the endothelial cells of the pulmonary capillary bed) to form angiotensin II.
Angiotensin II triggers release of aldosterone from the adrenal cortex. Aldosterone increases Na+
resorption from the cortical collecting duct of the kidney, thereby tending to increase blood volume
that in turn increases blood pressure. Under normal circumstances, angiotensin-converting enzyme is
present in large amounts and is not rate-limiting in this process; however, it does provide an effective
point at which the pathway can be interrupted by ACE inhibitors. Physiologically, the ACE inhibitors
primarily inhibit the renin-angiotensin-aldosterone system. These agents also inhibit bradykinin
degradation and stimulate vasodilating prostaglandin synthesis.
Aldosterone (choices A and D) is produced in the adrenal cortex. This hormone stimulates sodium
and water conservation in the kidneys.
ADH (choices C and E) is secreted by the posterior pituitary. This hormone decreases water
secretion in the kidneys.
Angiotensin-converting enzyme is present in the endothelial cells of the alveolar capillary bed, not in
the type II pneumocytes (choice E).
93. Which of the following would be expected in a woman with isolated ACTH-deficiency?
A. Decreased pubic and axillary hair
B. Decreased serum sodium concentration
C. Hyperpigmentation
D. Increased serum cortisol
E. Increased serum potassium
The correct answer is A. Isolated ACTH-deficiency is a pituitary disorder characterized by
decreased secretion of ACTH, but not of the other hormones of the anterior pituitary. Because of the
decreased effect on the inner two zones of the adrenal cortex, the secretion of adrenal androgens is
decreased. Because pubic and axillary hair in females depends on adrenal androgens, ACTH
deficiency would lead to a decrease in both. Like primary adrenal insufficiency, isolated ACTHdeficiency would lead to low serum levels of cortisol (not increased, choice D). With lower levels of
ACTH and cortisol, patients may also present with signs and symtoms of Addison disease, such as
low blood pressure, dehydration, and nausea. Unlike primary adrenal insufficiency, however, serum
levels of aldosterone are usually unchanged with isolated ACTH deficiency. This is because ACTH is
not a long-term regulator of aldosterone secretion (angiotensin II and potassium are the main longterm regulators). Because aldosterone is unchanged, serum sodium (choice B) and serum potassium
(choice E) are normal. Hyperpigmentation (choice C) is associated with increased, not decreased,
serum levels of ACTH.

36

94. Which of the following is most likely to be decreased during resting conditions for a
patient that has been severely anemic (Hb = 6.0 g/dL) for several months?
A. 2,3-diphosphoglycerate
B. Arterial PO2
C. Cardiac output
D. Mixed venous PO2
E. Red blood cell H+ concentration
The correct answer is D. Oxygen delivery to the tissues is approximately equal to cardiac output x
hemoglobin concentration x the amount of oxygen extracted from the blood (oxygen extraction =
arterial oxygen content - venous oxygen content). Some of the compensatory mechanisms that come
into play in an anemic person with a low hemoglobin content include the following: increased levels of
2,3 diphosphoglycerate (2,3-DPG; choice A), increased cardiac output (choice C), and increased red
blood cell H+ concentration (choice E). The increase in 2,3-DPG and red blood cell H+ concentration
cause the oxygen-hemoglobin dissociation curve to shift to the right, which facilitates unloading of
oxygen in the tissues.
The arterial PO2(choice B) and arterial oxygen saturation of hemoglobin are independent of
hemoglobin concentration. A decrease in the hemoglobin concentration of the blood, however, causes
a proportionate decrease in the oxygen-carrying capacity of the blood. (Think of a glass beaker
containing a solution of hemoglobin. If a gas having a PO2 of 100 mm Hg is bubbled through the
solution, the PO2 of the solution will be 100 mm Hg, regardless of the hemoglobin concentration. If
the concentration of hemoglobin in the beaker is doubled, however, the beaker will now contain twice
as much oxygen still at 100 mmHg.) Because each gram of hemoglobin can normally carry a total of
1.34 grams of oxygen, the arterial oxygen content of this patient can be calculated as follows: 1.34 mL
O2/g Hb x 6.0 g Hb/dL blood = 8.04 mL O2/dL blood (normal = 20 mL O2/dL blood). When the oxygen
content of the arterial blood is decreased to only 8 mL O2 /dL, the loss of oxygen to the tissues will
cause the venous oxygen tension to decrease to lower than normal levels during rest and to very low
levels whenever exercise is attempted.
95. Which of the following regions of the nephron is capable of the greatest level of glucose
reabsorption?
A. Proximal convoluted tubule
B. Thick ascending limb of loop of henle
C. Distal convoluted tubule
D. Collecting duct
The correct answer is A.At plasma concentrations less than 250 mg/dL, all of the filtered glucose is
reabsorbed in the proximal convoluted tubule and excretion is zero. The point at which the carriers are
saturated is Tm, that is, plasma concentration greater than 350 mg/dL. As the plasma concentration
increases, the additional filtered glucose cannot be reabsorbed and is excreted in the urine.
Glucosuria is an important clinical clue to diabetes mellitus.
96. Which of the following cytokines is associated with the development of cell-mediated
immunity?
A. IL-3
B. IL-4
C. IL-5
D. IL-10
E. IL-I2
The correct answer is E. Interleukins are peptides released by activated monocytes and
lymphocytes that assist in the coordination of cellular and humoral responses. Gamma interferon from
T helper 1 CD4+ lymphocytes and IL-12 from macrophages are major stimulators of the development
of a cell-mediated immune response to intracellular organisms such as tuberculosis, leishmaniasis,
histoplasmosis, and many other organisms.

37

IL-3 (choice A) is a cell growth factor produced by thymic epithelial cells and T cells to regulate
different T cell differentiation processes. IL-4 (choice B) is produced by T helper 2 CD4+ lymphocytes
and is involved in class switching to IgE. It also downregulates T helper 1 CD4+ lymphocytes. IL-5
(choice C) is produced by T helper 2 CD4+ lymphocytes and is involved in IgA production, eosinophil
differentiation, and B cell differentiation. IL-10 (choice D) is produced by T helper 2 CD4+
lymphocytes. It downregulates the cell-mediated immune system.
97. Which of the following pairs of hormones are most important to replace immediately after
removal of a tumor near the sella turcica, before life-threatening symptoms develop due to
anterior pituitary insufficiency?
A. Estrogens and mineralocorticoids
B. Glucocorticoids and thyroid hormones
C. Growth hormone and luteinizing hormone
D. Prolactin and glucocorticoids
E. Testosterone and follicle stimulating hormone
The correct answer is B. The anterior pituitary produces the following hormones: thyroid stimulating
hormone (TSH), adrenocorticotropic hormone (ACTH), follicle stimulating hormone (FSH), leutinizing
hormone (LH), prolactin, and growth hormone. Although deficiencies of any of these hormones are
serious, the most dangerous hormonal deficiencies involve glucocorticoids and thyroid hormone. It
has consequently become customary to "cover" patients for these potential deficiencies immediately
after any event (including surgery and possible pituitary apoplexy) in which function of the anterior
pituitary may be lost. If it turns out that the function was not lost, these hormones can be discontinued
later.
Deficiency of vasopressin produces decreased mineralocorticoid release (choice A), and clinically
recognizable diabetes insipidus (high urinary output with inability to concentrate urine), but
replacement is usually not begun unless symptoms become apparent.
Gonadal steroids (not LH and FSH) are given eventually, but they are not urgently required (choices
A, C, and E).
98. Which of the following is most likely to be decreased in the skeletal muscles of the legs of
a patient with essential hypertension following a severe laceration on the right hand?
A. Adenosine levels
B. Arterial blood pressure
C. Arteriolar resistance
D. Blood flow
E. Venous oxygen concentration
The correct answer is A. The skeletal muscles of the body have a normal blood flow even when the
blood pressure is chronically elevated. Organs and tissues in which the vasculature has primarily a
nutritive function (e.g., brain, heart, and skeletal muscle) regulate their blood flow in accordance with
the metabolic needs of the tissues. These tissues exhibit short-term autoregulation of blood flow such
that the increase in flow caused by an elevated arterial pressure is minimized by constriction of the
arterioles. The constriction is caused in part by decreased levels of adenosine (an endogenous
vasodilator) in the tissues. The rate of adenosine production in a tissue is a function of its metabolic
rate, which is not affected significantly by an increase in systemic pressure. When blood flow to the
muscle increases, the adenosine is literally washed from the muscle, lowering the tissue levels of
adenosine. The decrease in adenosine concentration causes small arteries and arterioles in the
muscle to constrict, and this increase in resistance (choice C) maintains blood flow (choice D) at a
normal rate in the face of increased arterial pressure (choice B). The overall process is called
autoregulation of blood flow.
Venous oxygen concentration (choice E) does not decrease in the skeletal muscles of patients with
hypertension because blood flow is maintained at an adequate level to meet the nutritional demands
of the muscles.

38

99. Which of the following would be present in a patient with a small-cell carcinoma of the
lung that autonomously secretes vasopressin?
A. Blood volume contraction
B. Decreased plasma atrial natriuretic peptide
C. Hypernatremia
D. Inappropriately concentrated urine
E. Increased thirst
The correct answer is D. A tumor that ectopically secretes vasopressin can produce a disorder
called the syndrome of inappropriate antidiuretic hormone (SIADH). Hypovolemia physiologically
stimulates ADH (vasopressin) secretion, so the diagnosis of SIADH is made only if the patient is
euvolemic. A hallmark of this disorder is excessive renal retention of free water with resultant
hypervolemia, not blood volume contraction (choice A), and dilutional hyponatremia, not
hypernatremia (choice C). The volume expansion leads to increased, not decreased (choice B),
secretion of atrial natriuretic peptide (ANP) by atrial myocytes. The increased ANP is one of the
factors that causes the kidney to increase sodium excretion and produce an inappropriately
concentrated urine. Urine is typically hypertonic to plasma in this disorder. The volume expansion also
tends to suppress renin secretion, and the resultant decrease in serum aldosterone may also
contribute to the increased renal sodium excretion. The dilutional hypotonicity would inhibit, not
stimulate (choice E), hypothalamic thirst mechanisms.
100. On stimulating the afferent portions of the glossopharyngeal and vagus nerves, which of
the following outcomes would most likely occur?
A. Bradycardia with hypertension
B. Bradycardia with hypotension
C. Sinus arrhythmia with hypotension
D. Tachycardia with hypertension
E. Tachycardia with hypotension
The correct answer is B. The glossopharyngeal nerve (CN IX) and the vagus nerve (CN X) carry
afferent information to the medulla from the carotid sinus and aortic arch baroreceptors, respectively.
The firing rate of these neurons increases with increasing blood pressure. By artificially increasing the
firing rate of these nerves, the medulla therefore receives a false signal that indicates that the blood
pressure is too high. This elicits a baroreceptor reflex, resulting in a decrease in sympathetic outflow
and an increase in parasympathetic outflow, which leads to bradycardia and hypotension.
Physiologically the glossopharyngeal nerve innervates the sensory nuclei of the medulla oblongata.
The somatic motor portion innervates the muscles involved with swallowing. The visceral motor
portion affects the parotid salivary gland by way of the otic ganglion. The vagus nerve innervates the
sensory fibers to sensory nuclei and autonomic centers of the medulla oblongata. The visceral motor
fibers go to muscles of the palate, pharynx, digestive, respiratory, and cardiovascular systems in the
thoracic and abdominal cavities.
101. Which of the following controls the salivary secretion?
A. Parasympathetic nervous system
B. Gastrin
C. Secretin
D. CCK
The correct answer is A. Salivary secretion is almost entirely under control of the parasympathetic
system. Sympathetic stimulation causes transient minor stimulation. Parotid secretion is stimulated by
parasympathetic stimulation from CN IX, whereas submandibular and sublingual secretion is
stimulated by parasympathetic stimulation from CN VII. Choice B - Gastric secretion is stimulated by
gastrin. Choice C - Pancreatic secretion is stimulated by secretin. Choice D - CCK stimulates bile
secretion.
102. Erythropoietin stimulates which of the following intermediates in hematopoiesis?
A. Basophilic erythroblasts
B. Colony forming units-erythroid (CFU-E)
C. Multipotential stem cells
D. Proerythroblasts
E. Reticulocytes

39

The correct answer is B. The colony forming unit-erythroid (CFU-E) is a unipotential stem cell that
develops from a burst forming unit-erythroid (BFU-E), which develops eventually from the
multipotential stem cell. The BFU-E is somewhat responsive to erythropoietin, but the CFU-E is
completely dependent on erythropoietin. Erythropoietin is normally released from the kidney in
response to hypoxic or anemic conditions.
The basophilic erythroblast (choice A) differentiates from the proerythroblast. It is recongnizable by
light microscopy and has a dark basophilic staining due to hemoglobin synthesis. It is not directly
affected by erythropoietin, but is instead indirectly increased by the increase in precursor cells from
the increase in CFU-E earlier in development.
The multipotential stem cell (choice C) appears earlier in development than CFU-E and does not
increase with erythropoietin. The development of all major components of blood (RBC, WBC, and
platelets) begins with the multipotential stem cell (CFU-S). This cell is noncommitted and can selfrenew. It is located in the bone marrow and is not recognizable by light microscopy.
The proerythroblast (choice D), which is the first recognizable cell in the red cell lineage, develops
from the CFU-E cell. It is not affected directly by erythropoietin, but instead increases in number from
the increased CFU-E cells.
The reticulocyte (choice E) is the enucleated cell just before the mature red blood cell. Reticulocytes
enter the peripheral circulation but continue to synthesize hemoglobin. This cell is not directly
stimulated by erythropoietin, but increases in number as a result of the increase in precursors.
103. In a controlled experiment, radiolabeled ATP is injected into an isolated muscle. The
muscle is stimulated and allowed to contract for 10 seconds. An autoradiogram from a biopsy
of the muscle will show radiolabeled ATP bound to
A. actin
B. myosin
C. sarcoplasmic reticulum
D. tropomyosin
E. troponin C
The correct answer is B. During the contraction cycle, ATP binds to myosin, causing the dissociation
of myosin from actin. Actin (choice A) forms cross-bridges with myosin but does not bind to ATP. The
sarcoplasmic reticulum (choice C) is involved in storing and releasing Ca2+ for muscle contraction.
Tropomyosin (choice D) is a thin filament that runs alongside actin. In the absence of calcium,
tropomyosin lies in the groove of the actin filament and blocks actin's myosin-binding sites. Troponin
C (choice E) is the calcium-binding subunit of the troponin complex. When troponin C binds calcium,
a conformational change causes tropomyosin to shift, thereby exposing the myosin-binding sites on
actin.
104. Which of the following would shift the oxygen-hemoglobin dissociation curve to the
right?
A. Carbon monoxide poisoning
B. Decreased PCO2
C. Decreased pH
D. Decreased temperature
E. Decreased 2,3-DPG
The correct answer is C. The loading of O2 is facilitated when the oxygen dissociation curve shifts to
the left, and the unloading of O2 is facilitated when the oxygen dissociation curve shifts to the right. A
good way to remember the conditions that promote dissociation of O2 is to think of exercising muscle,
which has decreased pH (choice C) because of the accumulation of lactic acid, increased PCO2
(compare with choice B) because of the increased rate of aerobic metabolism, increased temperature
(compare with choice D), and increased 2,3-DPG (2,3-diphosphoglycerate; compare with choice E)
because of increased glycolysis.
Carbon monoxide poisoning (choice A) left-shifts the oxygen dissociation curve, which interferes with
the unloading of O2. Carbon monoxide also strongly binds to available sites on hemoglobin.

40

105. After a total gastrectomy, which of the following digestive enzymes will be produced in
inadequate amounts?
A. Amylase
B. Chymotrypsin
C. Lipase
D. Pepsin
E. Trypsin
The correct answer is D. Gastrectomy is removal of the entire stomach. Pepsin is secreted (in an
inactive, or zymogen, form, as pepsinogen) by the chief cells of the stomach. Pepsinogen is activated
by contact with stomach acid. Although protein digestion usually begins with the actions of
hydrochloric acid and pepsin, pancreatic enzymes complete the job as the food passes into the small
intestine. Amylases (choice A) hydrolyze 1, 4 glycosidic linkages of starches to produce
oligosaccharides, maltose, maltotriose, and dextrins. Amylases are produced by the pancreas and
salivary glands. Chymotrypsin (choice B) is a proteolytic enzyme released by the pancreas as the
inactive proenzyme, chymotrypsinogen. Lipases (choice C) are mostly released by the pancreas and
serve to digest various lipids, including triacylglycerols. Trypsin (choice E) is a proteolytic enzyme
released by the pancreas as the inactive proenzyme trypsinogen.
106. Which of the following cardiovascular changes is most likely to occur when changing
from a standing to a supine position?
A. Decreased myocardial contractility
B. Decreased total peripheral resistance
C. Dilation of large veins
D. Increased pulse
E. Increased renal blood flow
The correct answer is D. Baroreceptors detect pressure changes in the walls of blood vessels and in
portions of the digestive, reproductive, and urinary tracts. The baroreceptor mechanism is important
for maintaining arterial pressure when a person sits or stands from a lying position. When a person
suddenly stands, the blood pressure in the brain and upper body tends to fall, which initiates a strong
sympathetic discharge throughout the body aimed at returning blood pressure to normal. Increasing
sympathetic stimulation to the heart causes an increase in heart rate, conduction velocity, and
myocardial contractility (compare with choice A). The sympathetic stimulation also causes
constriction of nearly all the arterioles in the body, which greatly increases the total peripheral
resistance (compare with choice B). Sympathetic stimulation of the renal vasculature leads to a
decrease in renal blood flow (compare with choice E). Constriction of large veins (compare with
choice C) increases venous return to the heart, causing the heart to pump increased amounts of
blood.
107. A man competing in a weightlifting competition lifts 325 lb over his head and holds it
there for 5 seconds. Suddenly, his arms give way and he drops the weight to the floor. Which
of the following receptors is responsible for this sudden muscle relaxation?
A. Free nerve endings
B. Golgi tendon organ
C. Merkel disk
D. Muscle spindle
E. Pacinian corpuscle
The correct answer is B. Normally, stretching of muscle results in a reflex contraction: the harder the
stretch, the stronger the contraction. At a certain point, when the tension becomes too great, the
contracting muscle suddenly relaxes. The reflex that underlies this sudden muscle relaxation is called
the Golgi tendon organ (GTO) reflex, also known as the inverse stretch reflex or autogenic inhibition.
The GTO is an extensive arborization of nerve endings that is connected in series with the extrafusal
skeletal muscle fibers. As a result, GTOs respond to muscle tension rather than muscle length.
Increased tension leads to stimulation of Ib afferents, which inhibit the homonymous muscle by way of
spinal interneurons. Free nerve endings (choice A) are unmyelinated, unencapsulated nerve endings
that penetrate the epidermis. These types of receptors respond to pain and temperature. Merkel disks
(choice C) are composed of specialized tactile epidermal cells and their associated nerve endings.

41

They are located in the basal layer of the epithelium and are slowly adapting receptors that respond to
touch and pressure.
Muscle spindles (choice D) are spindle-shaped bundles of muscle fibers (intrafusal fibers) that are
arranged in parallel with extrafusal skeletal muscle fibers, so they sense the length of the muscle.
They are innervated by Group Ia and II sensory afferent neurons.
Pacinian corpuscles (choice E) are unmyelinated nerve endings surrounded by thin, concentric layers
of epithelioid fibroblasts. In transverse section, this receptor resembles a sliced onion. They are found
primarily in the deep layer of the dermis, loose connective tissue, male and female genitalia,
mesentery, and visceral ligaments. They are rapidly adapting receptors that respond to touch and
pressure.
108. During the early stages of infection, which of the following compounds exert the most
powerful chemotactic effect on neutrophils, causing them to migrate into an inflamed area?
A. C5a and IL-8
B. IL-1 and tumor necrosis factor
C. LTC4 and LTD4
D. PGI2 and PGD2
E. Thromboxane and platelet activating factor
The correct answer is A. The most important chemotactic factors for neutrophils are the complement
factor C5a and the interleukin IL-8. The cytokines IL-1 and tumor necrosis factor (choice B) have
complex, similar actions, including stimulation of production of many acute-phase reactions,
stimulation of fibroblasts, and stimulation of endothelium. Leukotrienes LTC4 and LTD4 (choice C)
cause increased vascular permeability. Prostaglandins PGI2 and PGD2 (choice D) mediate
vasodilation and pain. Thromboxane and platelet activating factor (choice E) induce platelet changes.
109. Which of the following is decreased as a result of hyperventilation?
A. Arterial oxygen content
B. Arterial oxygen tension (PO2)
C. Arterial pH
D. Cerebral blood flow
E. Cerebrovascular resistance
The correct answer is D. The key symptom is hyperventilation, which occurs if the rate and depth of
respiration exceeds the demands for oxygen delivery and carbon dioxide removal. Hyperventilation
results in hypocapnia, alkalosis, increased cerebrovascular resistance, and decreased cerebral blood
flow. Carbon dioxide plays an important role in the control of cerebral blood flow. An increase in
arterial PCO2 dilates blood vessels in the brain and a decrease in PCO2 causes vasoconstriction.
The anxious, hyperventilating woman is "blowing off" carbon dioxide, which lowers her arterial PCO2,
causing the cerebrovascular resistance (choice E) to increase, thereby decreasing cerebral blood
flow. The decrease in cerebral blood flow has caused the woman to feel faint and to have blurred
vision. Other symptoms commonly associated with the hyperventilation of anxiety states are feelings
of tightness in the chest and a sense of suffocation.
Hyperventilation increases the arterial oxygen content (choice A) and PO2(choice B) in a normal
person.
A decrease in arterial PCO2 causes the arterial pH (choice C) to increase (i.e., the patient becomes
alkalotic).
110.
Following electrical stimulation of the raphe nuclei, which of the following
neurotransmitter levels would be expected to increase?
A. Acetylcholine
B. Dopamine
C. Gamma-amino-butyric acid (GABA)
D. Norepinephrine
E. Serotonin

42

The correct answer is E. Serotonin is the primary neurotransmitter of the raphe nuclei. Ascending
serotonin projections from the dorsal and median raphe nuclei distribute diffusely throughout the
brain. Other raphe nuclei provide descending serotonin projections to the spinal cord and brainstem.
Cell bodies that contain acetylcholine (choice A) are found in the basal nucleus of Meynert (which
degenerates in Alzheimer disease), in the medial septum and diagonal band complex, and in the
striatum.
Dopamine- (choice B) containing cell bodies are found in the midbrain (in the substantia nigra pars
compacta and in the ventral tegmental area). The cell bodies in the hypothalamus contribute to the
tuberoinfundibular pathway.
GABA (choice C) is the primary inhibitory neurotransmitter of the brain and is therefore located
throughout the brain.
Norepinephrine- (choice D) containing neurons are found predominantly in the locus ceruleus of the
pons and midbrain.
111. A boy is found to have multiple arteriovenous fistulas involving small cutaneous and
subcutaneous arteries and veins of the right leg. Which of the following would be increased in
a blood sample taken from the boy's right femoral vein?
A. Carbon dioxide content
B. Hematocrit
C. Oxygen content
D. Plasma sodium concentration
E. Total protein concentration
The correct answer is C. Congenital arteriovenous fistulas are often associated with limb swelling
and hypertrophy, visible pulsations when the fistulas are large, cosmetic changes when the fistulas
are in the subcutaneous tissues and skin, and varicose veins. The venous pressure is frequently
increased and the skin is often warmer compared with the opposite extremity. Because blood flowing
through the fistulas has by-passed the tissues of the extremity, the oxygen content of venous blood
from the involved limb is elevated as compared with the opposite limb, which is the pathognomonic
sign of arteriovenous fistula.
Because much of the femoral venous blood has bypassed the tissues, its carbon dioxide content
(choice A) is expected to be lower compared with venous blood of the opposite extremity (i.e., it
should be closer to that of arterial blood).
The venous hematocrit (choice B) is normally slightly greater compared with arterial hematocrit
because of a "chloride shift" into red blood cells as they pass through the microcirculation. This small
increase in hematocrit is attenuated with arteriovenous fistula as venous blood is diluted with arterial
blood that has bypassed the microcirculation.
There is no reason for sodium concentration (choice D) or total protein concentration (choice E) to
be affected by arteriovenous fistula, as these are normally similar in arterial and venous blood.
112. In the event of carbon monoxide poisoning, the blood sample taken from a systemic
artery reveals
A. normal hemoglobin level and normal oxygen content
B. normal hemoglobin level with reduced oxygen content
C. reduced hemoglobin level with reduced oxygen content
D. increased hemoglobin level with increased oxygen content
The correct answer is B. Oxygen content is the total amount of oxygen carried in blood, that is, ml
oxygen/100 ml of blood. CO competes for oxygen binding sites on Hb, thus decreasing the oxygen
binding capacity of hemoglobin and decreasing the oxygen content of blood. Because CO does not
destroy or damage the hemoglobin molecule itself, the Hb level will remain normal despite the
reduction of oxygen carried by it. Choice A - Normal condition when no pathology is present. Choice
C - Hb concentration and oxygen content are decreased in anemia, as less hemoglobin is produced.
Choice D - Both Hb and oxygen content are increased in polycythemia.

43

113. An increase in which of the following is the most likely stimulus for the proliferation a
large number of blood vessels in an abdominal tumor?
A. Angiostatin
B. Growth hormone
C. Thrombospondin
D. Vascular endothelial growth factor
E. Tissue oxygen partial pressure
The correct answer is D. Vascular endothelial growth factor (VEGF) is a heparin-binding
glycoprotein that increases endothelial cell proliferation in vitro and increases capillary growth (i.e.,
angiogenesis) in vivo. Unlike most other growth factors, VEGF has unique target cell specificity for
vascular endothelial cells. VEGF is overexpressed in solid tumors and in ischemic areas of the heart
and retina. Levels are also reversibly increased in a variety of normal and transformed cells exposed
to a hypoxic environment. These characteristics of VEGF make it an ideal candidate as a regulator of
angiogenesis in physiologic and pathophysiologic situations in which vessel growth is preceded by
deficient perfusion of the tissues.
Angiostatin (choice A) is an antiangiogenic factor expressed by tumors that tends to inhibit
angiogenesis.
Growth hormone (choice B) has a general effect that causes growth of almost all tissues of the body,
but does not appear to stimulate angiogenesis to a significant extent in solid tumors.
Thrombospondin (choice C) is a multifunctional glycoprotein that interferes with tumor growth,
angiogenesis, and metastasis.
An increase in tissue oxygen partial pressure (choice E) is unlikely to be the stimulus for new vessel
growth because (1) solid tumors are invariably hypoxic or ischemic (i.e., the partial pressure of oxygen
is low), and (2) an increase in the partial pressure of oxygen in a tumor would tend to decrease the
expression of VEGF, thereby decreasing the amount of angiogenesis in the tissues.
114. Measurement of the blood pressure of a 65-year-old man reveals a systolic pressure of
190 mm Hg and a diastolic pressure of 100 mm Hg. His pulse is 74/min and pulse pressure is
90 mm Hg. A decrease in which of the following is the most likely explanation for the high
pulse pressure?
A. Arterial compliance
B. Cardiac output
C. Myocardial contractility
D. Stroke volume
E. Total peripheral resistance
The correct answer is A. A decrease in arterial compliance indicates that the arterial wall is stiffer
(i.e., less distensible). When the compliance of the arterial system decreases, the rise in arterial
pressure becomes greater for a given stroke volume pumped into the arteries. In the normal young
adult, the systolic blood pressure is approximately 120 mm Hg and the diastolic blood pressure is
approximately 80 mm Hg. Because the pulse pressure is the difference between the systolic and
diastolic blood pressures, the normal pulse pressure is approximately 40 mm Hg in a healthy young
adult. In older adults, however, the pulse pressure sometimes increases as much as two times normal
because the arteries become hardened by arteriosclerosis.
The cardiac output (choice B) itself has no direct effect on the pulse pressure; however, if a decrease
in cardiac output is associated with a decrease in stroke volume, the pulse pressure would be
expected to decrease. Cardiac output = stroke volume x pulse.
A decrease in myocardial contractility (choice C) would be expected to decrease stroke volume, and
therefore cause the pulse pressure to decrease. A decrease in stroke volume (choice D) causes the
pulse pressure to decrease because a smaller amount of blood enters the arterial system with each
heartbeat, and the increase and decrease of pressure during systole and diastole is decreased. This
would cause cardiac output to decrease. A decrease in total peripheral resistance (choice E) (i.e.,
vasodilation) does not have a significant effect on the pulse pressure of the major arteries under
normal conditions.

44

115. Which of the following hormones is most important in initiating gall bladder contraction?
A. Cholecystokinin (CCK)
B. Gastric inhibitory peptide (GIP)
C. Gastrin
D. Secretin
E. Vasoactive intestinal polypeptide (VIP)
The correct answer is A. Cholecystokinin, or CCK, is synthesized in the duodenal and jejunal
mucosa and stimulates gall bladder contraction and pancreatic enzyme secretion. Other functions
include slowing of gastric emptying, an atrophic effect on the pancreas, and secretion of antral
somatostatin that in turn decreases gastric acid secretion. Gastric inhibitory peptide, or GIP (choice
B), stimulates pancreatic insulin secretion at physiologic doses and inhibits gastric acid secretion and
gastric motility at pharmacologic doses. Gastrin (choice C) prepares the stomach and small intestine
for food processing, including stimulating secretion of HCl, histamine, and pepsinogen. It also
increases gastric blood flow, lower esophageal sphincter tone, and gastric contractions. Secretin
(choice D) stimulates secretion of bicarbonate-containing fluid from the pancreas and biliary ducts.
Vasoactive intestinal polypeptide, or VIP (choice E), relaxes intestinal smooth muscle and stimulates
gut secretion of water and electrolytes.
116. What is the most common cause of an atrial septal congenital heart malformation?
A. Failure of formation of the septum primum
B. Failure of formation of the septum secundum
C. Incomplete adhesion between the septum primum and septum secundum
D. Malformation of the membranous interventricular septum
E. Malformation of the muscular interventricular septum
The correct answer is C. The most common form of atrial septal defect is located near the foramen
ovale (not to be confused with a patent foramen ovale, which is of little or no hemodynamic
significance). They result from incomplete adhesion between the septum primum and the septum
secundum during development. Atrial septal defect is usually asymptomatic until middle age. Right
ventricular lift and S2 widely split and fixed is noted. A grade T-III/VI systolic ejection murmur at the
pulmonary area is often noted. Atrial septal defects less commonly result from failures of formation of
the septum primum (choice A) and septum secundum (choice B). Malformations of the
interventricular septum (choices D and E) cause ventricular septal defects rather than atrial septal
defects.
117. Aspirin reduces fever and cause anticoagulation by inhibiting which of the following
pathways?
A. Phospholipase A2
B. Lipoxygenase
C. Prostacyclin
D. Cyclogenase
The correct answer is D. Aspirin and other NSAIDs inhibit cyclooxygenase, thereby inhibiting the
production of prostaglandins. This then decreases the set-point temperature. Inhibition of
phospholipase A2 (choice A) inhibits arachidonic acid. Inhibition of lipoxygenase (choice B) blocks
leukotrienes production. Blocking prostacyclin (choice C) does not decrease platelet aggregation.
118. Which of the following directly inhibits insulin secretion?
A. Alpha2-adrenergic agonist
B. Beta2-adrenergic agonist
C. Cholecystokinin
D. Glucagon
E. Ingestion of a high-sugar meal
The correct answer is A. Alpha2-receptor agonists directly inhibit pancreatic insulin secretion. Beta2adrenergic agonists (choice B) stimulate insulin secretion. Cholecystokinin (choice C) is a hormone
that causes not only gallbladder contraction, but also insulin secretion from the pancreas. Pancreatic
glucagon release (choice D) acts as a paracrine stimulus for insulin secretion. Ingestion of high-sugar
meals (choice E) is a stimulus for the secretion of insulin from the pancreas.

45

119. Which of the following urinary constituents is probably present in abnormally high
concentration and accounts for bubbles in the urine and generalized edema, especially
noticeable in dependent regions and under the eyes?
A. Albumin
B. Chloride
C. Glucose
D. Sodium
E. Urea
The correct answer is A. The combination of edema and frothy urine suggests nephrotic syndrome.
Urine with high protein content is more able to form stable bubbles than is normal urine, and may be a
clue that a patient (or parent) notices. The edema is caused by loss of albumin from the plasma,
lending to fluid loss from plasma to interstitial space. Chloride (choice B) in urine cannot be detected
grossly and would not cause frothy urine. Some people can detect a sweet smell to urine that contains
large amounts of glucose (choice C), and Greek physicians were known to taste urine to diagnose
diabetes, but the presentation suggests nephrotic syndrome. Sodium (choice D) in urine cannot be
detected grossly and would not cause frothy urine. Very concentrated urine with a high urea (choice
E) content looks darker in color and is not necessarily frothy.
120. Carbon dioxide is transported in blood by a variety of mechanisms. Which of the
following is quantitatively the most important method for transporting CO2?
A. As carbaminohemoglobin
B. As CO2 in gas bubbles
C. As CO2 in physical solution
D. As sodium bicarbonate in red cells
E. As sodium bicarbonate in serum
The correct answer is E. Red blood cells (and many other blood cells) contain the enzyme carbonic
anhydrase, which catalyzes the intracellular conversion of CO2 to bicarbonate and H+ ion. Most of the
bicarbonate in the red cell is exchanged across the plasmalemma for chloride ion. This means that
although the bulk of the production of bicarbonate occurs in the red cell (choice D), the bulk of the
actual transport occurs in serum. Carbonic anhydrase is not present in serum. Bicarbonate can be
produced in serum by nonenzymatic means, but the process is slow.
CO2 is also carried as carbaminohemoglobin (choice A), which forms when CO2 binds to an NH2
side group of the hemoglobin protein, rather than to the heme iron (Fe2+) as with carbon monoxide
and oxygen.
CO2 is not transported in the form of bubbles (choice B), which is a good thing, because gas bubbles
are effectively emboli, which can lead to considerable morbidity or death.
Some CO2 is carried directly dissolved in blood (choice C). It is 20 times more soluble in blood than
is O2.
121. Disease of which of the following structures would most likely cause a large difference in
blood pressure between a patient's legs and arms?
A. Aortic valve
B. Descending aorta
C. Left atrium
D. Left ventricle
E. Mitral valve
The correct answer is B. A patient that has a large difference in blood pressure between his legs
and arms suggests disease of the aorta distal to the arch, where the vessels supplying the arms arise.
In younger individuals, coarctation of the aorta is the most probable diagnosis. In older individuals,
severe atherosclerosis of the abdominal aorta, iliac system, or femoral system is the most probable
diagnosis. Disease of the aortic valve (choice A), left atrium (choice C), left ventricle (choice D), or
mitral valve (choice E) would not produce selective effects on the blood pressure of the arms or legs.

46

122. When a given substance is injected into an experimental animal, it increases intracellular
Ca+ through the formation of inositol 1,4,5-triphosphate. On which of the following receptors
does the given substance act?
A. Beta-1 receptor
B. Beta-2 receptor
C. Alpha-1 receptor
D. Alpha-2 receptor
E. M2 receptor
The correct answer is C. Phenylephrine acts on the alpha-1 receptor to increase intracellular Ca+
through formation of inositol 1,4,5-triphosphate. Isoproterenol acts on the beta-1 and beta-2 receptors
(choices A and B, respectively) through the activation of adenylate cyclase and production of cAMP.
Clonidine (choice D) acts on the alpha-2 receptor through the inhibition of adenylate cyclase and
decreases cAMP. ACH acts on the M2 receptor, which is inhibitory in the heart and decreases heart
rate and conduction velocity in AV node.
123. In the event of propagation of an action potential, the conduction velocity is increased by
A. decrease in fiber size
B. increase in fiber size
C. unmyelinated fiber
D. increased internal resistance
The correct answer is B.Propagation of action potentials occurs by the spread of local currents to
adjacent areas of membrane that are then depolarized to threshold and generate action potentials.
Increasing fiber size increases conduction velocity. The greater the diameter, the greater the
conduction velocity. Choices A and D - Decreasing the diameter of nerve fiber results in increased
internal resistance, thus conduction velocity decreases. Choice C - Myelin acts as an insulator around
nerve axons and increases the conduction velocity. The greater the myelination, the greater the
conduction velocity.
124. During an experiment, the loss of a portion of the adrenal gland, which is controlled by
the autonomic nervous system, results in a decreased capacity of the individual to mobilize
glycogen or fat during exercise. Which of the following hormones of respective regions should
be absent to match the outcome of this experiment?
A. Epinephrine - medulla
B. Epinephrine - zona reticularis
C. Cortisol and androgen - zona reticularis and zona fasciculata
D. Aldosterone - zona glomerulosa
E. Aldosterone - Medulla
The correct answer is A.The major hormone of the adrenal medulla is epinephrine, a catecholamine
regulated by the autonomic nervous system. Its absence produces the above-mentioned effect.
Epinephrine is not a hormone of the cortex (choice B), and aldosterone is not a hormone of the
medulla (choice E). Cortisol and androgen (choice C), the hormones of the adrenal cortex, zona
fasciculata, and reticularis, are regulated by ACTH. Absence of cortisol could cause severe problems
under stressful conditions. Catecholamines do not exert vasoconstriction without cortisol, leading to
circulatory failure. Mineralocorticoid aldosterone, the hormone of the zona glomerulosa (choice D), is
under the control of angiotensin-2. Its absence could result in circulatory shock and death. Therefore,
it is necessary for survival.
125. Which of the following substances is most responsible for the constancy in plasma
sodium concentration when large amounts of sodium are ingested?
A. Aldosterone
B. Angiotensin II
C. Antidiuretic hormone (ADH)
D. Atrial natriuretic factor (ANF)
E. Epinephrine
The correct answer is C. A 5-fold increase in sodium intake causes the plasma sodium
concentration to increase by less than 1%, indicating the existence of a powerful mechanism for

47

maintaining extracellular sodium concentration at a constant level. When the ADH-thirst mechanism is
blocked, however, a 5-fold increase in sodium intake causes the plasma sodium concentration to
increase by more than 10%. The major mechanism for controlling extracellular sodium concentration
(and extracellular osmolarity) therefore is the ADH-thirst mechanism. You should recall that ADH
increases the permeability of the late distal tubule and collecting duct to water, which allows water to
be retained by the body and a concentrated urine to be excreted.
Aldosterone (choice A) and angiotensin II (choice B) are powerful salt-retaining hormones. They
regulate the total amount of sodium in the body, but have relatively little effect on plasma sodium
concentration under normal conditions for the following reasons: (1) they increase reabsorption of
sodium and water to an equal extent, and (2) any tendency for sodium concentration to change is
immediately compensated for by changes in ADH levels, which return sodium concentration to a
normal value.
Atrial natriuretic factor (choice D) is released from the atria when blood volume increases. It acts on
the kidneys to increase the excretion of sodium and water. ANF, however, does not play an important
role in regulating plasma sodium concentration because any tendency for sodium concentration (and
osmolarity) to change is immediately compensated for by changes in ADH levels, as discussed
previously.
Epinephrine (choice E) does not play an important role in regulating extracellular sodium
concentration.
126. To minimize chronic rejection injury to transplanted organs, immunosuppressive therapy
is aimed at downregulating which of the following components of the immune response?
A. Autoantibody production
B. Complement protein synthesis
C. HLA antigen expression
D. Mast cell degranulation
E. T-lymphocyte activity
The correct answer is E. Before any immune response can occur, such as a graft rejection, T cells
can be activated by exposure to an antigen. Chronic rejection of any solid organ entails cellular injury
to endothelial cells, resulting in intimal proliferation, fibrosis, and eventually ischemic injury to the
graft. Immunosuppressive therapy is directed at controlling lymphocyte activity and minimizing cellular
rejection.
Autoantibodies (choice A) are not involved in organ transplant rejection. The antibodies produced are
alloantibodies directed only to the graft, not to the host. Complement proteins (choice B) are involved
in the humoral component of acute rejection, and complement binding to alloantibodies increases
graft damage. Complement protein production, however, is not affected by immunosuppressive
therapy. HLA antigen expression (choice C) is central to recognition of foreign cells in grafted tissue.
HLA antigens are expressed constitutively by all normal cells, and immunosuppression does not affect
their production. Mast cell degranulation (choice D) is a component of the anaphylactic response
(Type I hypersensitivity). Graft rejection is a Type IV hypersensitivity response and does not involve
mast cell degranulation.
127. Which of the following laboratory parameters could be used to assess glycemic control
over the past 2-3 months for a patient with type 2 diabetes?
A. Blood glucose
B. Blood insulin levels
C. Blood ketones
D. Glycosylated hemoglobin
E. Urinary glucose
The correct answer is D. The range of normal for the HbA1c or glycosylated hemoglobin is 3.96.9%. This is directly related to the level of glucose in the blood. Because HbA1c is a stable product,
its concentration reflects glucose levels over the past 2-3 months. HbA1c forms as a result of
nonenzymatic glycosylation, a fundamental biochemical abnormality that accounts for most of the
histopathologic alterations in diabetes mellitus. At first, glucose forms reversible glycosylation
products with proteins by formation of Schiff bases. Rearrangement of Schiff bases leads to more

48

stable, but still reversible, Amadori products and subsequently to irreversible advanced glycosylation
end products (AGE), of which HbA1c is an example.
Blood ketones, blood glucose, urinary glucose, and blood insulin do not reflect longstanding metabolic
abnormalities of diabetes mellitus and cannot be used to assess long-term glycemic control.
Blood glucose (choice A) is elevated in type 1 and type 2 diabetes mellitus. Hyperglycemia is the
diagnostic feature of diabetes mellitus and leads to glycosuria (choice E) when blood glucose
exceeds 160-180 mg/dL.
Blood insulin (choice B) is absent in untreated type 1 diabetes and normal or even slightly increased
in type 2 diabetes, depending on the state of the type 2 diabetes.
Blood ketones (choice C) (acetoacetic acid and beta-hydroxybutyric acid) are synthesized from free
fatty acids in response to severe insulin deficiency. Accumulation of ketone bodies in the blood is a
crucial pathogenetic factor in ketoacidosis, which occurs primarily in type 1 diabetics or type 2
diabetics with advanced disease who are improperly treated.
128. Daily serum samples from a woman with a normal menstrual cycle reveal decreasing
progesterone and 17-estradiol levels. Serum LH and FSH levels are low, and begin rising.
Basal body temperature begins falling. Within three days, which of the following events would
be expected to occur?
A. Markedly increased inhibin levels
B. Menstruation
C. Ovulation
D. Rapidly decreased LH levels
E. Significantly increased basal body temperature
The correct answer is B. A typical menstrual cycle lasts around 26-30 days. The luteal phase (postovulation) generally lasts fourteen days; the length of the follicular phase (pre-ovulation) is far more
variable, and accounts for most of the variability observed in the length of the menstrual cycle. Just
before menstruation, sex steroid levels are low, but gonadotropin levels (especially FSH) begin rising
slightly. Basal body temperature remains high during the luteal phase of the menstrual cycle, but falls
precipitously a few days before the onset of menstruation. Markedly increased inhibin levels (choice
A) are seen in the middle of the luteal phase, dropping to low levels just before menstruation. LH
levels peak approximately 36 hours before ovulation (choice C), then decrease rapidly (choice D)
within a few days to a low level during the mid-luteal phase, gradually decreasing until menstruation.
The basal body temperature significantly increases (choice E) shortly after ovulation, due to the
metabolic effects of progesterone produced by the corpus luteum.
129. Antigens processed by the exogenous antigen presentation pathway are presented in
association with which of following?
A. Fc receptors
B. IgG heavy chains
C. MHC class I molecules
D. MHC class II molecules
E. T cell receptor (TCR)
The correct answer is D. When pathogenic organisms are phagocytized and degraded in the
exogenous antigen presentation pathway, the antigenic molecules are presented on the surface of the
antigen-presenting cell by MHC class II molecules to a CD4+ T lymphocyte with a specific TCR for the
specific antigenic epitope. The Fc (choice A) portion of an antibody molecule is the part of the
immunoglobulin that attaches to the Fc receptors on phagocytic cell surfaces. When a Fab portion of
the antibody is attached to the pathogen and the Fc attaches to the phagocytic cell surface, the
phagocyte can destroy the pathogen more efficiently. The IgG molecule (choice B) is an
immunoglobulin that reacts with the antigen after it has been destroyed and presented to the T cell.
The IgG immunoglobulin is never involved in antigen presentation. In the endogenous antigen
presentation pathway (e.g., a virus infecting a cell), the cell would display epitopes from the virus in
association with class I molecules (choice C) to the CD8+ cytotoxic T cell. The TCR (choice E) is the
area of the mature T cell that reacts with the antigen epitope that is presented by the antigenpresenting cell.
130. Most of the testosterone secreted by the testes exists in the plasma in the form of

49

A. dihydrotestosterone bound to gonadal steroid-binding hormone


B. free dihydrotestosterone
C. free testosterone
D. testosterone bound to albumin
E. testosterone bound to sex-steroid-binding globulin
The correct answer is E. Most circulating testosterone is bound to plasma protein (approximately
98%), rather than existing in free form (choice C). Of this, most is bound to a specific sex (or gonadal)
steroid-binding protein (choice E), and a minority is bound to albumin (choice D).
Dihydrotestosterone is produced from testosterone in the tissues by a specific enzyme, 5-alphareductase, rather than circulating in bound (choice A) or free (choice B) form. Testosterone is the
principal androgen produced by the interstitial cells of the testes.
131. During a surgical procedure, cranial nerves IX and X are accidentally cut bilaterally. What
would be the immediate change in the patient's hemodynamic condition?
A. Bradycardia with hypertension
B. Bradycardia with hypotension
C. Sinus arrhythmia with hypotension
D. Tachycardia with hypertension
E. Tachycardia with hypotension
The correct answer is D. The glossopharyngeal nerve (CN IX) and the vagus nerve (CN X) carry
afferent information to the medulla from the carotid sinus and aortic arch baroreceptors, respectively.
The firing rate of these neurons increases with increasing blood pressure. Severing these nerves
therefore sends the medulla a false signal that the patient has suddenly lost all blood pressure. This
elicits a baroreceptor reflex, resulting in an increase in sympathetic outflow and leading to tachycardia
and hypertension.
Remember physiologically the glossopharyngeal nerve routes to sensory nuclei of the medulla
oblongata. The somatic motor portion destination is the pharyngeal muscles involved in swallowing.
The visceral portion innervates the parotid salivary gland by way of the otic ganglion.
The vagus nerve routes to the sensory fibers to sensory nuclei and autonomic centers of the medulla
oblongata. The visceral fibers innervate the motor fibers and muscles of the palate, pharynx, and
digestive, respiratory, and cardiovascular systems in the thoracic and abdominal cavities.
132. A patient with severe anemia may have which of the following symptoms?
A. Bradycardia
B. Cyanosis
C. Low stroke volume
D. Warm hands
E. Wide pulse pressure
The correct answer is E. The normal blood hemoglobin concentration is approximately 15-16 g/dL
for a man and 13-14 g/dL for a woman. A person is considered to be severely anemic when the
hemoglobin concentration decreases to less than 7.5 g/dL. In severely anemic, the resting cardiac
output is significantly increased with an increase in pulse and stroke volume (choice C). The increase
in stroke volume causes a widening of the pulse pressure, because when a greater amount of blood is
ejected during each systole, the blood pressure increases and decreases to a greater extent.
Bradycardia (choice A) is said to occur when the pulse decreases to less than 60/min. Severely
anemic exhibit tachycardia, which is defined as a pulse greater than 100/min. The increase in pulse is
because the body is trying to increase oxygen perfusion to tissues by increasing the heart rate.
Cyanosis (choice B) refers to a bluish color of the skin and mucous membranes that results from the
presence of deoxygenated hemoglobin in the blood vessels, especially the capillaries. Cyanosis does
not occur in severely anemic despite widespread hypoxia in the tissues because 5 grams of
deoxygenated hemoglobin must be present in each 100 mL of blood to produce overt cyanosis. In
other words, the hemoglobin concentration is too low for a severely anemic to become cyanotic. The
hands of anemic are often cold (choice D) because of decreased blood flow and oxygen perfusion to
the skin.

50

133. Radiographic studies of a 2-year-old child reveal a new fracture of the humerus and
evidence of multiple old fractures in ribs and long bones of the extremities. Physical
examination reveals that the toddler has "peculiar teeth," a blue tinge to the sclera, and
unusually mobile joints. The disease that the physician suspects the child has is characterized
by an abnormality of which of the following biochemical functions?
A. Collagen type I synthesis
B. Collagen type II synthesis
C. Collagen type III synthesis
D. Collagen type IV synthesis
E. Collagen type V synthesis
The correct answer is A. The child has the most common variant (type I) of osteogenesis
imperfecta, which is an autosomal dominant genetic defect in the synthesis of type I collagen,
because of decreased synthesis of the procollagen alpha1(1) amino acid chain. This defect (unlike
that of the perinatal, lethal, type II form of osteogenesis imperfecta) is compatible with survival, but
does cause skeletal fragility, dentinogenesis imperfecta (abnormal teeth), blue sclera, joint laxity, and
hearing impairment. Spontaneous fractures occur in utero and in childhood. Unfortunately, many
families of children with this defect have had their children removed because of "abuse," only to find
that the broken bones continue in the new environment. Type I collagen is found in skin, bone,
tendons, and most other organs. This causes severe osteoporosis. Less severe mutations in type I
collagen are common, resulting in collagen disarray and predisposing to hypogonadal (e.g.,
menopausal) or idiopathic osteoporosis. Type II collagen (choice B) is found in cartilage and vitreous
humor. Type III collagen (choice C) is found in blood vessels, uterus, and skin. Type IV collagen
(choice D) makes basement membranes. Type V collagen (choice E) is a minor component of
interstitial tissues and blood vessels. There are also type VI-XI collagens, which are minor
constituents of various tissues.
134. Which of the following neurotransmitters is most important for the induction of REM
sleep?
A. Acetylcholine
B. Dopamine
C. Epinephrine
D. Norepinephrine
E. Serotonin
The correct answer is A. Acetylcholine is the neurotransmitter of primary importance for the
induction of REM sleep. Some of the other neurotransmitters do function in sleep, but REM sleep can
occur in their absence. In the CNS, ACH synapses are throughout the brain and spinal cord. In the
PNS, ACH is found at the neuromuscular junctions, preganglionic synapses of the ANS, neuroeffector
junctions of parasympathetic division, and in the sympathetic division of the ANS. Dopamine (choice
B) is a neurotransmitter with a role in voluntary movement, mood, cognition, and regulation of
prolactin release. Epinephrine (choice C) is important in sympathetic nervous system responses. It is
also a CNS neurotransmitter. Norepinephrine (choice D) is important in sympathetic nervous system
responses. It is also a CNS neurotransmitter involved in attention, arousal, and mood. Serotonin
(choice E) is a CNS neurotransmitter that plays an important role in mood and sensation. In the
periphery, it is involved in vascular regulation and digestive function.
135. The primary metabolic effect of the principal hormone secreted by the alpha cells of the
pancreas is
A. augmentation of calcium deposition in bone
B. increase of amino acid storage in the liver
C. promotion of lipogenesis in liver and adipose tissue
D. inhibition of gluconeogenesis
E. stimulation of glycogenolysis
The correct answer is E. Glucagon is released from the alpha cells of the pancreas in response to
hypoglycemia and stimulates glycogenolysis to increase serum glucose. Augmented calcium
deposition in bone (choice A) is achieved by calcitonin, which is secreted by the C-cells in the thyroid
gland. Glucagon plays no role in calcium metabolism. Glucagon favors amino acid conversion to
glucose (gluconeogenesis) rather than storage in the liver (choice B). Insulin (which generally has
opposite effects of glucagon) promotes lipogenesis in the liver and in adipose tissue (choice C), and
also promotes glycogen synthesis. Glucagon stimulates gluconeogenesis (choice D).
136. Compensation for high altitude will result in which of the following physiologic changes?

51

A. Decreased production of erythropoietin


B. Decreased 2,3-diphosphoglycerate (2,3-DPG)
C. Increased renal excretion of H+ ions
D. Increased renal excretion of HCO3E. Pulmonary vasodilation
The correct answer is D. Compensation for high altitude includes an increase in the renal excretion
of bicarbonate. The diminished barometric pressure found at high altitude causes arterial hypoxia,
which is sensed by peripheral chemoreceptors. The ventilation rate increases, thereby causing a
respiratory alkalosis. The kidney then compensates by increasing the excretion of HCO3-.
Erythropoietin is increased, not decreased in chronic hypoxia and at high altitude (choice A).
Increased erythropoietin leads to an increased hematocrit.
Another adaptation to high altitude is increased 2,3-DPG (compare with choice B), which shifts the
oxygen dissociation curve to the right. This facilitates the release of O2 in the tissue.
High altitude leads to respiratory alkalosis. The renal compensation is a metabolic acidosis
characterized by decreased H+ excretion and increased HCO3- excretion. Respiratory acidosis is
normally compensated with a metabolic alkalosis that would include increases in H+ excretion
(choice C).
Pulmonary vasoconstriction, not vasodilation (choice E), occurs in response to alveolar hypoxia, such
as would occur at high altitudes.
137. A decrease in which of the following parameters would tend to increase the glomerular
capillary hydrostatic pressure?
A. Afferent arteriolar resistance
B. Bowman's capsular hydrostatic pressure
C. Capillary filtration coefficient
D. Efferent arteriolar resistance
E. Plasma colloid osmotic pressure
The correct answer is A. The glomerular filtration rate is the rate of filtrate formation at the
glomerulus. It is commonly used to assess renal function. A decrease in the resistance of the afferent
arteriole (i.e., arteriolar dilation) directly increases glomerular capillary hydrostatic pressure by
lessening the decrease in blood pressure that normally occurs along the vasculature proximal to the
glomerulus. (Recall that the afferent arteriole is upstream from the glomerulus; the efferent arteriole is
downstream from the glomerulus.) The glomerular capillary hydrostatic pressure is the determinant of
glomerular filtration rate most subject to physiologic control.
Bowman's capsular hydrostatic pressure (choice B), capillary filtration coefficient (choice C), and
plasma colloid osmotic pressure (choice E) are important determinants of GFR but they do not have
any direct effect to increase or decrease the glomerular capillary hydrostatic pressure.
A decrease in efferent arteriolar resistance (choice D) would tend to decrease the glomerular
capillary hydrostatic pressure because the efferent arteriole is downstream from the glomerular
capillaries.
138. Exogenous therapy with which of the following hormones would be most likely to slow or
prevent osteoporosis?
A. Cortisol
B. Epinephrine
C. Estrogen
D. Thyroxine
E. Vasopressin
The correct answer is C. Estrogen replacement in postmenopausal women seems to play an
important role in preventing or limiting development of osteoporosis in postmenopausal women.
Osteoporosis can be asymptomatic or may involve severe backache and spontaneous fractures.

52

There is often a loss of height. Serum parathyroid hormone, calcium, phosphorus, and 25(OH)D2 are
often normal. Demineralization of the spine, hip, and pelvis are common.
Cortisol (choice A) excess, as in endogenous or exogenous Cushing syndrome, is a contributing
cause of osteoporosis. Increased cortisol will worsen the condition.
Epinephrine (choice B) concentrations seem to be unrelated to osteoporosis. Increased levels will
increase pulse and blood glucose level.
Thyroxine (choice D) excess (e.g., in thyrotoxicosis) may contribute to bone loss in some cases of
osteoporosis.
Vasopressin (choice E) concentrations seem to be unrelated to osteoporosis. Vasopressin or
antidiuretic hormone will decrease the amount of water loss from the kidneys.
139. A type 1 diabetic is noncompliant with his required insulin therapy and develops
hyperglycemia. The release of which of the following intestinal hormones would be
stimulated?
A. Gastic inhibitory peptide (GIP)
B. Gastrin
C. Motilin
D. Secretin
E. Somatostatin
The correct answer is A. Gastric inhibitory peptide (GIP) is produced in the duodenal and jejunal
mucosa by K cells and is released in response to intraluminal glucose and fatty acids. GIP is
sometimes called glucose-dependent insulinotropic peptide because it stimulates pancreatic insulin
secretion in the presence of hypergylcemia. Note that although GIP release would be stimulated, the
hormone would not have a pronounced effect in this type 1 diabetic, whose pancreatic islet cells do
not produce adequate amounts of insulin.
Gastrin (choice B) is synthesized and stored primarily in the G cells of the stomach and TG cells of
the stomach and small intestine. The stimuli for gastrin secretion include increased vagal discharge,
digestive products, calcium salts, and gastric distention. Gastrin stimulates HCl secretion by parietal
cells, histamine release from enterochromaffin cells, pepsinogen secretion by chief cells, gastric blood
flow, and contraction of gastric circular smooth muscle. It has a trophic effect on gastric and small
intestinal mucosa and the pancreas, increases lower esphageal sphincter (LES) tone, and is a weak
stimulus for the secretion of pancreatic enzymes and bicarbonate.
Motilin (choice C) is produced in the M and enterochromaffin cells of the duodenum and jejunum.
Secretion occurs during fasting. Motilin acts to regulate the migrating myoelectric complex (MMC).
Secretin (choice D) is synthesized and stored in the S cells of the mucosa of the upper intestine.
Acidification of the duodenal mucosa and the presence of fat and protein degradation produced in the
duodenum stimulate its secretion. The main role of secretin is to stimulate bicarbonate secretion from
the pancreas and liver.
Somatostatin (choice E) is synthesized and stored in the D cells of the pancreatic islets, in the gastric
antrum, and throughout the intestine. It is also present in the hypothalamus. It inhibits the release of
gastrin, cholecystokinin (CCK), and most other gastrointestinal hormones. In brief, it shuts off the gut.
Somatostatin inhibits the release of glucagon by pancreatic alpha cells, as well as the release of
insulin by the pancreatic beta cells (of the islets of Langerhans).
140. Which of the following is most likely to decrease in skeletal muscles during exercise?
A. Arteriolar resistance
B. Carbon dioxide concentration
C. Lactic acid concentration
D. Sympathetic nervous activity
E. Vascular conductance

53

The correct answer is A. The increase in muscle blood flow that occurs during exercise is caused by
dilation of the arterioles (i.e., decreased arteriolar resistance). In normal skeletal muscles, blood flow
can increase as much as 20-fold during strenuous exercise. Most of this increase in blood flow can be
attributed to the dilatory actions of metabolic factors (e.g., adenosine, lactic acid, carbon dioxide)
produced by the exercising muscles.
Exercise causes the concentration of carbon dioxide (choice B) and lactic acid (choice C) to increase
in the muscles. Mass discharge of the sympathetic nervous system (choice D) occurs throughout the
body during exercise, causing arterioles to constrict in most tissues. The arterioles in the exercising
muscles, however, are strongly dilated by vasodilator substances released from the muscles.
A decrease in vascular conductance (choice E) occurs when the vasculature is constricted.
Resistance and conductance are inversely related, so that a decrease in arteriolar resistance is
associated with an increase in arteriolar conductance.
141. When the pulse is greater than 200/min and the blood pressure is 75/40 mm Hg, what
adjustments have probably occurred in the cardiac cycle?
A. Diastolic time has decreased and systolic time has increased
B. Diastolic time has decreased but systolic time has decreased more
C. Systolic time has decreased and diastolic time has increased
D. Systolic time has decreased but diastolic time has decreased more
E. Systolic time has decreased but diastolic time has not changed
The correct answer is D. Under normal conditions, one third of the cardiac cycle is spent in systole
and two thirds spent in diastole. As pulse increases dramatically, the time spent in diastole falls
precipitously but the time spent in systole falls only slightly. A large increase in pulse must produce a
decrease in both diastole and systole (compare with choice A). The major change with increased
pulse is in diastole, not systole (compare with choice B). Pulse cannot increase if diastolic time
increases (choice C). An increase in pulse must be accompanied by a decrease in diastolic time
(compare with choice E).
142. Which of the following is likely to be decreased in a patient with severe anemia (Hb less
than 7 g/dL)?
A. Arterial O2 content
B. Arterial O2 saturation
C. Arterial PO2
D. Cardiac output
E. Pulse
The correct answer is A. A decrease in the hemoglobin concentration of the blood causes a
proportional decrease in the oxygen-carrying capacity of the blood. Each gram of hemoglobin can
normally carry a total of 1.34 grams of oxygen. Thus, each 100 mL of arterial blood can normally carry
approximately 20 mL oxygen at a normal hemoglobin concentration of 15 g/dL blood. With a
hemoglobin concentration of 7 g/100 mL, each 100 mL of blood can carry only 9.4 mL oxygen. The
oxygen saturation of hemoglobin in the arterial blood (choice B) and the arterial PO2 (choice C) are
virtually unaffected by the hemoglobin concentration of the blood. The reduced oxygen-carrying
capacity of the severely anemic patient is associated with a compensatory increase in cardiac output
during resting conditions and especially during exercise. The elevation in cardiac output helps to
maintain oxygen delivery to the tissues at an adequate level. The increase in cardiac output (choice
D) is caused by an increase in pulse (choice E).
143. Maximal ventricular Na+ channel conductance occurs during which phase of the ECG?
A. P wave
B. QRS interval
C. ST interval
D. T wave
E. U wave
The correct answer is B. Phase 0 of the cardiac muscle action potential (AP) corresponds to the
opening of voltage-dependent sodium channels, causing a transient but large increase in sodium
conductance during ventricular depolarization. The shape of the QRS complex of the ECG is

54

determined by the spread of the combined phase 0 (depolarization) of all the ventricular muscle of the
heart. The P wave (choice A) corresponds to atrial depolarization. The ST interval (choice C)
represents the time interval during which all ventricular cells are in phase 2 of their AP. Phase 2 is
dominated by a high, prolonged calcium conductance through slow channels. The length of the ST
interval corresponds closely to the AP duration in ventricular muscle. The T wave (choice D)
corresponds to ventricular repolarization. The U wave (choice E) is found only occasionally in ECGs
and is presumed to be caused by the repolarization of papillary muscle.
144. During normal diastole, which of the following is most important in preventing overdistension of the ventricles?
A. Adjacent lungs
B. Aortic valve
C. Diaphragm
D. Fibrous pericardium
E. Mitral valve
The correct answer is D. The fibrous pericardium, which surrounds the heart, does not simply
separate the heart from other chest structures, but has the important physiologic role of limiting the
distension of the heart during diastole. This helps keep the (normal) heart functioning in a useful part
of Starling's curve. In congestive heart failure, the slow enlargement of the heart also enlarges the
fibrous pericardium, and this protective function may be lost.
The lungs (choice A) and diaphragm (choice C) do not usually significantly limit cardiac expansion
during diastole. Shutting and opening of the aortic (choice B) and mitral valves (choice E) are
mechanical events that occur secondary to the changes in pressure in the cardiac chambers.
145. Physical examination of a pregnant woman in her second trimester reveals a slightly
enlarged, nontender thyroid gland and a normal cardiac examination. The serum thyroxine (T4)
level is increased; however, the serum thyroid-stimulating hormone (TSH) is normal. Which of
the following best explains the laboratory findings in this patient?
A. Decreased estrogen
B. Increased free thyroxine (T4)
C. Increased progesterone
D. Increased serum triiodothyronine (T3)
E. Increased thyroid-binding globulin (TBG)
The correct answer is E. The total serum thyroxine (T4) represents the sum of the T4 bound to
thyroid-binding globulin (TBG) and the free T4. An increased total T4 may be caused by an increase
in TBG or an increase in free T4, the latter leading to signs of thyrotoxicosis.
In a euthyroid state, one third of the binding sites on TBG are occupied by T4. An increase in estrogen
(pregnancy, birth control pills) increases the synthesis of TBG. The T4 bound to the additional TBG
increases the total serum T4. The extra TBG, however, does not alter the free T4 level because of the
equilibrium between the serum concentration of T4 and thyroid gland T4 production. Because the free
T4 level is normal, there is no stimulus to release thyroid-stimulating hormone (TSH) from the pituitary
gland. Regarding the patient's enlarged thyroid gland, heat intolerance, and palpitations, these are
normal findings in pregnancy and do not indicate an overactive thyroid gland.
Estrogen is increased (not decreased) in pregnancy (choice A).
The serum free T4 (choice B) is normal in pregnancy. This explains why the serum TSH is normal in
the presence of an elevated serum T4, which reflects the increase in TBG normally occurring in
pregnancy.
The increase in progesterone (choice C) during pregnancy has no effect on TBG levels.
Although the serum T3 (choice D) concentration is increased in pregnancy for the same reason as
serum T4 (more T3 is bound to TBG), it is not responsible for the increase in synthesis of TBG that
leads to the increase in serum T4.

55

146.Administration of an experimental drug that acts on peripheral nervous system (PNS)


myelin is shown to increase the space constant of an axon in a peripheral nerve. Action
potentials traveling down the axon would be predicted to be
A. faster
B. larger
C. slower
D. smaller
E. unchanged
The correct answer is A. The space constant of an axon reflects the amount of passive or
electrotonic spread of current within the axon. The larger the space constant, the further the current
can spread, allowing action potentials to propagate faster. This is why myelin increases the
conduction velocity of action potentials down an axon. Conversely, demyelination decreases the
space constant and slows action potential conduction.
147. Respiratory rate: 15/min; Arterial pressure: 120/80mm Hg; Cardiac output: 5L/min; Heart
rate: 50/min Basal measurements are shown above. What is the stroke volume during resting
conditions (in mL/min)?
A. 50
B. 75
C. 100
D. 125
E. 150
The correct answer is C. The cardiac output (CO) is equal to the volume of blood ejected from the
heart during each beat of the heart (SV, stroke volume) multiplied by the number of times the heart
beats each minute (HR, heart rate). That is, CO = SV x HR. Therefore, SV = CO/HR, and since CO =
5000 mL/min, and HR = 50/min, SV = 5000/50 = 100 mL.
Note that some of the data listed are irrelevant to the solution of the problem
148. The drug used in the induction of anesthesia facilitates which of the following receptors
action, which increases chloride conduction?
A. GABA-B
B. Glycine
C. GABA-A
D. Glutamate
The correct answer is C. Barbiturates facilitate GABA-A action by increasing the duration of chloridechannel opening, thus decreasing neuron firing. GABA-B receptor (choice A) increases K+
conductance. Glycine (choice B) is an inhibitory neurotransmitter that increases Cl- conductance.
Glutamate (choice D) is an excitatory neurotransmitter in the brain. The kainite receptor for glutamate
is an ion channel for Na+ and K+.
149. A 45-year-old man with acromegaly has an MRI revealing the presence of a 1.5-cm tumor
in the anterior pituitary. Which of the following endocrine abnormalities is likely to be present?
A. Decreased plasma growth hormone concentration
B. Decreased plasma IGF-1 concentration
C. Decreased plasma insulin concentration
D. Impaired glucose tolerance
The correct answer is D. The patient probably has acromegaly caused by a growth hormonesecreting adenoma in the anterior pituitary. Hypersecretion of growth hormone in an adult will not
cause an increase in stature, because the epiphyses of long bones have already fused. Overgrowth of
bone in the face and skull, however, produces the characteristic protruding jaw and forehead
observed in this disorder. Soft tissue proliferation leads to a coarsening of facial features. The hands
and feet are particularly affected, producing large and thickened spade-like fingers and toes.
Excessive growth hormone decreases the sensitivity of peripheral tissues to insulin ("anti-insulin"

56

effect). This tends to raise blood glucose and produce a compensatory hyperinsulinemia (not
decreased plasma insulin, choice C) that functions to limit the hyperglycemia. Approximately 50% of
patients with acromegaly show impaired glucose tolerance.
Plasma levels of growth hormone (choice A) and IGF-1 (choice B) are both increased in acromegaly.
150. Which of the following is a characteristic of steroid hormones?
A. Activation of adenylate cyclase
B. Activation of protein kinases
C. Plasma membrane receptors
D. Stimulation of cellular protein synthesis
E. Termination of effects by phosphodiesterase
The correct answer is D. Peptide/protein hormones and steroid hormones act by very different
mechanisms. Steroid hormones circulate in the bloodstream, then leave the bloodstream by
dissolving in the lipid-rich plasma membrane. Steroids cross the plasma membrane and enter the
cytoplasm, where they bind to a mobile protein receptor. The hormone/receptor complex then enters
the nucleus and triggers RNA synthesis, leading to protein synthesis, thereby changing the cellular
response. Hormones such as cortisol and aldosterone are steroid hormones.
In contrast, protein and amine hormones bind to specific receptors on the outer surface of the plasma
membrane (choice C), but do not enter the cytoplasm. Many activate adenylate cyclase (choice A)
and thereby increase cAMP. The increased cAMP stimulates protein kinases (choice B), which
change the target cell responses by phosphorylating intracellular proteins. Hydrolysis of cAMP by
phosphodiesterases (choice E) terminates the effect of the hormone.
151. Stimulating glossopharyngeal afferent fibers that supply the carotid sinus would most
likely cause which of the following changes?
A. Hypertension with bradycardia
B. Hypertension with tachycardia
C. Hypotension with bradycardia
D. Hypotension with tachycardia
E. No changes in blood pressure or heart rate
The correct answer is C. The glossopharyngeal nerve (CN IX) and the vagus nerve (CN X) carry
afferent information to the medulla from the carotid sinus and aortic arch baroreceptors, respectively.
The firing rate of these neurons increases with increasing blood pressure. Stimulation of the
glossopharyngeal nerve therefore sends the medulla a false signal that the animal has suddenly had
an increase in blood pressure. This elicits a baroreceptor reflex resulting in a decrease in sympathetic
outflow and an increase in parasympathetic outflow, leading to hypotension and bradycardia.
152. What is the role of the macrophage during antibody formation?
A. Activation of cytotoxic CD8 T cells
B. Delayed hypersensitivity reaction
C. Lysis of virus-infected cells
D. Processing antigen and presenting it to T helper CD4 cells
E. Synthesis of immunoglobulin
The correct answer is D. The macrophage is a phagocytic cell of the monocyte-macrophage system.
The macrophage phagocytizes exogenous antigens (e.g., a bacterium), degrading the antigen into
small epitopes and presenting them, on MHC class II molecules on its surface, to CD4 T helper cells.
Macrophages do not activate cytotoxic CD8 T lymphocytes (choice A). The major activator of
cytotoxic CD8 T lymphocytes is IL-2 from CD4+ THl cells.
Delayed hypersensitivity reactions (choice B) are the results of CD4+ THl cells. These cells do not
produce antibody. They secrete gamma interferon and interleukin 2 (IL-2), stimulating more cells to
become involved in the delayed hypersensitivity reaction.

57

The cells that participate in lysis of virus infected cells (choice C) are cytotoxic CD8+ T lymphocytes
that react with MHC class I molecules containing epitopes of the virus from the infected cell.
Macrophages do not participate in this activity.
Macrophages never synthesize antibody (choice E). B cells initially produce antibody, then are
converted to plasma cells or memory B cells. The stimulus for the production of this antibody comes
from T helper cells that were stimulated by epitopes presented to them by macrophages.
153. Which of the following is most important for maintaining adequate cardiac output early in
the course of exercise?
A. Decreased cardiac index
B. Decreased diastolic blood pressure
C. Increased heart rate
D. Increased stroke volume
E. Increased systematic vascular resistance
The correct answer is D. At the beginning of aerobic (isotonic) exercise, increased stroke volume is
the most important adjustment for maintaining adequate cardiac output (CO = HR x SV) or cardiac
output = heart rate x stroke volume.
There is an increased, not decreased, cardiac index (choice A) during isotonic exercise.
Diastolic blood pressure usually remains unchanged during isotonic exercise; it is not decreased
(choice B). In contrast, systolic blood pressure usually rises during isotonic exercise.
Increased heart rate (choice C) becomes more important later in isotonic exercise (beyond 50% of
maximal work capacity).
There is decreased, not increased, systemic vascular resistance (choice E) during isotonic exercise.
154. Which of the following regions of the nephron is capable of the greatest level of sodium
reabsorption?
A. Collecting duct
B. Distal convoluted tubule
C. Proximal convoluted tubule
D. Thick ascending limb of the loop of Henle
The correct answer is C. About 70% of the sodium entering the proximal convoluted tubule is
actively reabsorbed. Chloride and water follow passively.
The collecting duct (choice A) reabsorbs about 3% to 5% of the entering sodium, via aldosteroneregulated, electrogenic Na+-channel pumps. Antidiuretic hormone (ADH) increases H2O permeability
and reabsorption in this region.
The distal convoluted tubule (choice B) reabsorbs about 5% of the entering sodium via Na+/K+/2Clcotransport. This region is relatively impermeable to water.
The ascending limb of the loop of Henle (choice D) reabsorbs about 20% of the entering sodium via
Na+/K+/2Cl- cotransport. This region is relatively impermeable to water.
155. An infant has a coarctation of the aorta that reduces renal blood flow to 50% lower than
normal. Which of the following is increased in this infant?
A. Blood flow in the lower body
B. Glomerular filtration rate
C. Plasma levels of angiotensin II
D. Renal excretion of sodium
E. Renal excretion of water
The correct answer is C. The lower than normal pressure at the level of the kidneys causes renin to
be secreted and angiotensin to be formed. Renin is released by the kidney because of (1)

58

sympathetic stimulation and (2) a decline in renal blood flow. When renin acts on angiotensinogen, a
plasma protein produced in the liver, it converts angiotensinogen to angiotensin I. Angiotensin I is
converted to angiotensin II by the enzyme ACE (angiotensin converting enzyme). The angiotensin
causes salt and water retention so that within a few days to weeks the arterial pressure in the lower
body (at the level of the kidneys) increases.
Blood flow in the lower body (choice A) is lower rather than increased, because of the blockage.
Blood flow, however, can be normal, above, and below the constriction if the body is able to
compensate fully.
The decrease in blood pressure at the level of the kidneys causes the glomerular filtration rate
(choice B) to decrease.
Increased plasma levels of angiotensin II cause salt and water retention; thus, salt and water
excretion (choices D and E) are decreased.
156. A patient with small-cell bronchogenic carcinoma of the lung complains of muscle
weakness, fatigue, confusion, and weight gain. Serum sodium is found to be elevated and the
tumor is secreting ectopic ADH. Which of the following abnormal laboratory results would also
be expected?
A. Decreased plasma atrial natriuretic peptide (ANP) concentration
B. Decreased plasma vasopressin concentration
C. Decreased serum osmolarity
D. Decreased urinary sodium concentration
E. Increased plasma aldosterone concentration
The correct answer is C. Bronchogenic carcinomas can secrete ectopic vasopressin (ADH), leading
to the syndrome of inappropriate ADH (SIADH). As long as water intake is not decreased, the
increased plasma vasopressin (not decreased, choice B) causes excessive water reabsorption by the
renal distal tubule and collecting duct. The increased total body water can explain the weight gain.
Edema is usually absent because the extra free water is distributed to intracellular and extracellular
volumes. The extra plasma water produces a dilutional hyponatremia, which can explain the
weakness, fatigue, and confusion. There will also be a dilutional decrease in serum osmolarity. With
SIADH, the urine sodium is usually increased (not decreased, choice D) compared with normal. This
leads to an inappropriately concentrated urine. The volume expansion resulting from the excessive
water retention may be responsible for the increased urinary sodium. Volume expansion would
increase plasma ANP (not decrease, choice A) and increase renal sodium excretion. The volume
expansion would also inhibit renin secretion from the kidney with subsequent decrease in plasma
aldosterone (not increase, choice E). Decreased plasma aldosterone would then allow for increased
renal excretion of sodium.
157. A decrease in which of the following parameters would tend to increase the glomerular
capillary hydrostatic pressure?
A. Afferent arteriolar resistance
B. Bowman's capsular hydrostatic pressure
C. Capillary filtration coefficient
D. Efferent arteriolar resistance
E. Plasma colloid osmotic pressure
The correct answer is A. A decrease in the resistance of the afferent arteriole (i.e., arteriolar dilation)
directly increases glomerular capillary hydrostatic pressure by lessening the drop in blood pressure
that normally occurs along the vasculature proximal to the glomerulus. [Recall that the afferent
arteriole turns into the glomerulus; the efferent arteriole drains the glomerulus.] The glomerular
capillary hydrostatic pressure is the determinant of glomerular filtration rate most subject to
physiologic control.
Bowman's capsular hydrostatic pressure (choice B), capillary filtration coefficient (choice C), and
plasma colloid osmotic pressure (choice E) are important determinants of GFR, but they do not have
any direct effect to increase or decrease the glomerular capillary hydrostatic pressure.
A decrease in efferent arteriolar resistance (choice D) would tend to decrease the glomerular
capillary hydrostatic pressure because it would allow blood to more easily flow out of the glomerulus.

59

158. Which of the following lung volumes or capacities cannot be measured directly using
simple spirometry?
A. Expiratory reserve volume
B. Functional residual capacity
C. Inspiratory reserve volume
D. Tidal volume
E. Vital capacity
The correct answer is B. The functional residual capacity is the amount of air left in the lungs after a
normal expiration. Because this volume cannot be expired in its entirety, it cannot be measured by
spirometry. Essentially, lung volume that contains the residual volume, which is the amount of air
remaining after maximal expiration (e.g., functional residual capacity and total lung capacity), cannot
be measured by spirometry. These volumes can be determined using helium dilution techniques
coupled with spirometry or body plethysmography.
The expiratory reserve volume (choice A) is the volume of air that can be expired after expiration of a
tidal volume.
The inspiratory reserve volume (choice C) is the volume of air that can be inspired after inspiration of
a tidal volume.
Tidal volume (choice D) is the amount of air inspired or expired with each normal breath.
Vital capacity (choice E) is the volume of air expired after a maximal inspiration.
159. Which of the following would be expected to increase in response to hemorrhage
resulting in loss of 600 mL of blood?
A. Arteriolar diameter in skeletal muscle
B. Sodium excretion
C. Sympathetic nerve activity
D. Vagal nerve activity
E. Water excretion
The correct answer is C. The decrease in blood pressure caused by hemorrhage activates the
baroreceptor reflex, which tends to increase sympathetic nerve activity and decrease parasympathetic
(vagal) nerve activity (choice D). The increase in sympathetic nerve activity constricts arterioles in
skeletal muscle (choice A) and elsewhere in the body. The fact the patient has lost 600 mL blood and
yet her blood pressure has decreased only slightly from a normal value of approximately 120/80 mm
Hg may be attributed to the following compensatory responses: baroreceptor epinephrine released
from the adrenal medulla, formation of angiotensin II, formation of vasopressin, and the capillary fluid
shift mechanism.
Activation of the renin angiotensin system during hemorrhage also plays an important role in
maintaining blood pressure. Angiotensin II increases blood pressure acutely by constricting arterioles
throughout the body (choice A), and chronically by decreasing the renal excretion of salt (choice B)
and water (choice E). The decrease in salt and water excretion returns blood volume to a normal
value.
160. Which of the following areas of the adrenal gland would be expected to increase in
activity in a patient subjected to salt restriction?
A. Adrenal medulla
B. Zona fasciculata of the adrenal cortex
C. Zona glomerulosa of the adrenal cortex
D. Zona reticularis of the adrenal cortex
The correct answer is C. This question requires you to equate salt restriction with an increased
synthesis of aldosterone (aldosterone promotes sodium reabsorption) and then to remember that
aldosterone is produced in the zona glomerulosa (the outermost layer) of the adrenal cortex.

60

The adrenal medulla (choice A) secretes catecholamines.


The zona fasciculata (choice B) is the middle layer of the adrenal cortex. It primarily secretes
glucocorticoids.
The zona reticularis (choice D) is the innermost layer of the adrenal cortex. It primarily secretes
androgens such as dehydroepiandrosterone (DHEA).
161. A middle-aged patient develops acromegaly. Which of the following pair of hormones
normally regulates the hormone responsible for the symptoms seen with this disease?
A. Dopamine and norepinephrine
B. LH and hCG
C. Prolactin and FSH
D. Somatostatin and GHRH
E. TSH and ACTH
The correct answer is D. Acromegaly is typically produced by a growth hormone-secreting pituitary
adenoma. Growth hormone synthesis is predominantly regulated by hypothalamic GHRH (growth
hormone releasing hormone), and its pulsatile secretion is predominantly regulated by hypothalamic
somatostatin. Note that the question asks for regulating hormones, not the hormone directly
responsible for the disease.
Dopamine and norepinephrine are catecholamines that regulate smooth muscle tone and cardiac
function (choice A).
Luteinizing hormone (LH) regulates sex steroid hormone production by testes and ovaries. Human
chorionic gonadotropin (hCG) is produced by the placenta and has actions similar to LH (choice B).
Prolactin regulates menstruation and lactation. Follicle stimulating hormone (FSH) regulates ovarian
and testicular function (choice C).
Thyroid stimulating hormone (TSH) regulates secretion of thyroid hormones. Adrenocorticotropin
(ACTH) regulates glucocorticoid secretion (choice E).
162. If the luminal diameter of the right renal artery is decreased by approximately 50%, which
of the following is most likely increased?
A. Afferent arteriolar resistance
B. Glomerular filtration rate
C. Glomerular hydrostatic pressure
D. Interlobar artery pressure
E. Secretion of renin
The correct answer is E. The decrease in renal artery diameter causes a reduction in arterial
pressure within the kidney, which results in an initial decrease in glomerular hydrostatic pressure
(choice C) and glomerular filtration rate (choice B). The decrease in glomerular filtration rate
decreases the amount of sodium chloride that is delivered to the macula densa; in turn, the
juxtaglomerular cells secrete renin, and angiotensin II is formed. The angiotensin then mainly
constricts the efferent arterioles, which increases glomerular hydrostatic pressure and glomerular
filtration rate back toward normal. This macula densa feedback mechanism also attempts to return
glomerular hydrostatic pressure (and therefore glomerular filtration rate) to a normal level by
decreasing afferent arteriolar resistance (choice A).
An obstruction of the renal artery would decrease blood pressure in the interlobar arteries (choice D).

_____________

61

Biochemistry
1. Which of the following is the 3-carbon end-product of fatty acid metabolism?
A. Acetoacetate
B. Acetone
C. -Hydroxybutyrate
D. Diacylglycerol
E. Monoacylglycerol
The correct answer is choice B. The theme of this question is lipid metabolism. Ketone bodies
[acetoacetate (choice A), acetone (choice B), and -hydroxybutyrate (choice C)] are lipid
breakdown products. The levels of these molecules can increase in poorly controlled diabetes mellitus
or starvation. Acetone is the 3-carbon fragment of this group.
Monoacylglycerols (choice E) and diacylglycerols (choice D) are neutral fats made from glycerol with
one or two fatty acids esterified to the glycerol, respectively.
2. Folate plays a role in single-carbon unit transfer in the synthesis of nucleotides. Which of
the following nucleotides require folate for synthesis?
A. Adenosine, cytosine, and uracil
B. Adenosine, guanine, and thymidine
C. Adenosine, guanine, and uracil
D. Cytosine, thymidine, and uracil
E. Guanine, thymidine, and uracil
The correct answer is choice B. Folate is involved in the transfer of carbons 2 and 8 of the purine
nucleus (affecting adenosine and guanosine) and the 5-methyl group of thymidine. This means that
folate is required for synthesis of 3 of the 4 nucleic acid bases of DNA and 2 of the 4 nucleic acid
bases in RNA. It is thus no wonder that folate deficiency affects so many tissues with high mitotic rate.
Megaloblastic changes analogous to those seen in erythrocytes and their precursors can also be seen
in other cells produced by bone marrow (neutrophil, eosinophil, basophil, and macrophage and
megakaryocyte lines) and in epithelia throughout the body, including skin, mucous membranes such
as the mouth and vagina (where the changes can be seen on pap smear), stomach, intestinal linings,
and cells from lung or liver. Similar megaloblastic changes are observed throughout the body when
cobalamin (vitamin B12 ) deficiency is present, since cobalamin plays a role in methionine synthesis,
which is the source of the one-carbon unit "active-formate." Cobalamin is also invovled in the
conversion of methylmalonic acid to succinic acid and is required to maintain the integrity of nerve
cells via an unknown biochemical pathway.
3. A patient is taking hormone supplements. The hormone binds to a receptor in the cell
membrane, which activates tyrosine kinase activity. Which of the following hormones is this
patient taking?
A. Calcitrol
B. Thyroxine
C. Retinoic acid
D. Insulin
E. Protein synthesis

The correct answer is D.Insulin, a water-soluble hormone, binds to receptors in the cell membrane.
Receptor tyrosine kinase is activated, leading to protein phosphorylation. Tyrosine kinase receptors
are also involved in signaling by growth factors like PDGF (platelet derived growth factors) and EGF
(epidermal growth factor). Choice A, B and C - are all lipid soluble harmones, which diffuse through
the cell membrane, and bind to their respective receptors inside the cell. The hormone-receptor
complex then binds to hormone receptor elements in DNA, which then results in the observed
hormone effect.
4. A 5-year-old child has blue-tinged sclera, hearing loss, and small, slightly blue, misshapen
teeth. Radiologic studies confirm the presence of numerous fractures of various ages. No
significant degree of bruising is seen over sites of recent fracture. The disease this child most
likely has is related to abnormal metabolism involving which of the following substances?
A. Collagen
B. Glycogen
C. Mucopolysaccharides
D. Purines
E. Tyrosine
The correct answer is A. The suspected disease is osteogenesis imperfecta, which is a rare genetic
disorder that occurs in both recessive and dominant forms. The clinical presentation, depending on
the specific form, varies from death in utero, to that described in the question stem, to very mild
disease with only a modest increase in bone fragility. Spontaneous fractures occur in utero or during
childhood. The different types all have defects in the synthesis of type I collagen, often with
insufficient or abnormal pro-1(1) or pro-2(1) chains. These deficits produce an unstable collagen triple
helix that is not as strong as normal collagen. Less severe mutations on type I collagen genes are
common, resulting in collagen disarray and predisposing to hypogonadal or idiopathic osteoporosis.
Defective glycogen (choice B) metabolism is associated with the various glycogen storage diseases,
such as von Gierke disease and Pompe disease. These diseases tend to present with profound
hypoglycemia, hepatomegaly, or muscle weakness.
Defective mucopolysaccharide (choice C) metabolism is associated with the mucopolysaccharidoses,
such as Hurler and Hunter syndromes. These diseases tend to present with abnormal facies
("gargoylism"), deformed ("gibbus") back, claw hand, and stiff joints.
Abnormalities of purine metabolism (choice D) are present in gout, which presents with joint
inflammation and often involves the great toe. The net result is due to chronic hyperuricemia.
Abnormalities of tyrosine metabolism (choice E) are associated with phenylketonuria (pale hair and
skin, mental retardation, musty smelling urine), albinism (pale hair, skin, increased skin cancer),
cretinism (decreased T3 and T4), tyrosinosis (liver and kidney disease), and alkaptonuria (chronic
arthritis and urine that turns black upon standing).
5. An IgG2 molecule is composed of which of the following?
A. One alpha, one gamma2, and two kappa chains
B. One gamma1 chain and two kappa chains
C. Two gamma1 chains and one kappa and one lambda chain
D. Two gamma1 chains and two kappa chains
E. Two gamma2 chains and two kappa chains
The correct answer is E. IgG molecules contain two gamma heavy chains of a given subtype and
two light chains (either kappa or lambda). The 2 in IgG2 indicates the subclass to which the molecule
belongs. IgG2 contains two gamma2 chains (since a given B cell can only form one type of heavy

chain). The IgG molecule will contain either two kappa chains or two lambda chains, but never one of
each (choice C). The standard serum concentration is 12 mg/mL.
A given cell produces immunoglobulin molecules with a single type of heavy chain (compare with
choice A). IgG molecules with gamma1 chains (choices B, C, and D) would be of the IgG1 subclass.
6. What is the lipase enzyme that degrades stored triacylglycerols in adipocytes?
A. Gastric lipase
B. Pancreatic lipase
C. Lipoprotein lipase
D. Hormone-sensitive lipase
The correct answer is D.Hormone-sensitive lipase is found in and degrades stored triacylglcerols in
adipocytes.
Gastric lipase (choice A) originates in the stomach, where it degrades dietary triacylglycerols.
Pancreatic lipase (choice B) is found in pancreas and degrades dietary triacylglycerols in the small
intestine.
Lipoprotein lipase (choice C) is found in extrahepatic tissues and acts on surface endothelial-cell
lining. The capillaries degrade triacylglycerols circulating in chylomicrons or VLDL.
7. A 50-year-old man presents with headache, nausea, memory loss, abdominal pain, dark
colored lines in gums, neuropathy, and anemia. Which of the following enzyme/s is/are
inhibited in this patient?
A. ALA dehydrase and ferrochelatase
B. ALA synthase
C. Uroporphyrinogen I synthase
D. UDP-glucuronyl transferase
The correct answer is A. This is a case of lead poisoning. Lead inhibits ALA dehydrase and
ferrochelatase, the enzymes of the heme synthesis pathway. Other signs and symptoms include
coarse basophilic stippling of erythrocytes, lead deposits in abdomen, a gingiva, and epiphysis of
bone.
Choice B - ALA synthase is the rate-limiting enzyme of heme synthesis and is repressed by heme.
Choice C - is the enzyme of heme synthesis; its absence leads to acute intermittent porphyria.
Choice D - UDP-glucuronyl transferase is the enzyme of heme degradation, and is involved in the
formation of the conjugated form of bilirubin.
8. Which of the following metabolic alterations would most likely be present in a chronic
alcoholic compared to a non-drinker?
A. Fatty acid oxidation is stimulated
B. Gluconeogenesis is stimulated
C. Glycerophosphate dehydrogenase is stimulated
D. The ratio of lactate to pyruvate is decreased
E. The ratio of NADH to NAD+ is increased

The correct answer is E. The principal route of metabolism of ethanol is via alcohol dehydrogenase,
which uses hydrogen from ethanol to form NADH from NAD+, markedly increasing the ratio of NADH
to NAD+. The relative excess of NADH has a number of effects, including inhibiting, rather than
stimulating fatty acid oxidation (choice A); inhibiting gluconeogenesis rather than stimulating it
(choice B); inhibiting, rather than stimulating (choice C) glycerophosphate dehydrogenase; and
favoring the formation of lactate rather than pyruvate from glycolysis (thereby increasing, rather than
decreasing the lactate/pyruvate ratio; choice D).
Alcoholism is a syndrome consisting of two phases: problem drinking and alcohol addiction. Alcohol
addiction is defined as a physiologic dependence as manifested by evidence of withdrawal when
intake is interrupted. As a side note, thiamine is commonly deficient in chronic alcoholics. Remember,
thiamine is a necessary ketolase enzyme cofactor.
9. A young adult with albinism is found to be at increased risk for skin cancer. Which of the
following is the precursor of melanin?
A. Tryptophan
B. Tyrosine
C. Arginine
D. Glycine
The correct answer is choice B. Albinism is caused by congenital deficiency of tyrosinase. It results
in an inability to synthesize melanin from tyrosine. Other derivatives of tyrosine are thyroxine,
dopamine, norepinephrine, and epinephrine.
Choice A - derivatives of tryptophan are niacin, serotonin, and melatonin.
Choice C - derivatives of arginine are creatine and urea.
Choice D - porphyrin is derived from glycine and eventually from heme.
10. A chronic alcoholic develops severe memory loss with marked confabulation. Deficiency of
which of the following vitamins would be most likely to contribute to the neurologic damage
underlying these symptoms?
A. Folic acid
B. Niacin
C. Riboflavin
D. Thiamine
E. Vitamin B12
The correct answer is D.Wernicke-Korsakoff syndrome refers to the constellation of neurologic
symptoms caused by thiamine deficiency. Among these, a severe memory deficit, which the patient
may attempt to cover by making up bizarre explanations (confabulation), is prominent. Wernicke's
encephalopathy consists of the of confusion, ataxia and ophthalmoplegia. Anatomical damage to the
mamillary bodies and periventricular structures has been postulated as the cause. In the U.S., severe
thiamine deficiency is seen most commonly in chronic alcoholics. Thiamine deficiency can also
damage peripheral nerves ("dry" beriberi) and the heart ("wet" beriberi).
Folic acid deficiency (choice A) produces megaloblastic anemia without neurologic symptoms.
Niacin deficiency (choice B) produces pellagra, characterized by depigmenting dermatitis, chronic
diarrhea, and anemia.
Riboflavin deficiency (choice C) produces ariboflavinosis, characterized by glossitis, corneal
opacities, dermatitis, and erythroid hyperplasia.

Vitamin B12 deficiency (choice E) produces megaloblastic anemia accompanied by degeneration of


the posterolateral spinal cord. This megaloblastic anemia is commonly known as pernicious anemia.
11. One of the enzymes of the citric acid cycle, which is on the inner mitochondria membrane,
also functions as complex II of the electron transport chain. This enzyme is also responsible
for converting succinate to fumarate, producing FADH2. The enzyme is
A. isocitrate dehydrogenase
B. a ketoglutarate dehydrogenase
C. succinate dehydrogenase
D. succinyl CoA synthase
The correct answer is choice C. All Krebs cycle components are in the matrix of the mitochondria
except succinate dehydrogenase, which is in the inner mitochondrial membrane. Succinate
dehydrogenase reoxidizes FADH2 and passes electrons directly to Coenzyme Q.
Choice A - Isocitrate dehydrogenase, the rate limiting enzyme of citric acid cycle, produces NADH. It
is activated by ADP and inhibited by NADH.
Choice B - a ketoglutarate dehydrogenase, like pyruvate dehydrogenase, requires thiamine, lipoic
acid, CoA, FAD, NAD, and produces NADH.
Choice D - Succinyl CoA synthase catalyses a substrate level phosphorylation of GDP to GTP.
12. The extremely potent vasodilator nitric oxide (NO) is produced naturally by the body from
which of the following amino acids?
A. Arginine
B. Aspartate
C. Glutamate
D. Methionine
E. Proline
The correct answer is A. In the body, the very short acting (5-second half-life) signaling molecule
nitric oxide (NO) is synthesized by nitric oxide synthase (NOS). This enzyme is found in many tissues
and converts arginine to citrulline and NO. The vasodilation produced by NO normally occurs in
response to appropriate local biologic triggers, such as infection. However, this process may also
produce pathology if uncontrolled, since it is thought that the endotoxic shock that can complicate
bacterial septicemia may be related to continued activity of NOS. NO also plays a role in macrophage
activity, where its metabolites are toxic to ingested microorganisms.
Aspartate (choice B) is associated with donation of one of the nitrogens in urea, with the liver enzyme
aspartate aminotransferase (AST), and with nucleotide synthesis.
Glutamate (choice C) is associated with amino acid transport into the cell via the gamma-glutamyl
cycle, and with the liver enzymes aspartate aminotransferase (AST) and alanine aminotransferase
(ALT).
Methionine (choice D) is associated with 1-carbon transfer reactions.
Proline (choice E) is associated with the collagen helix.

13. Which of the following amino acids would most likely be found on the surface of a protein
molecule?
A. Alanine
B. Arginine
C. Isoleucine
D. Leucine
E. Phenylalanine
The correct answer is choice B. This question requires two logical steps. First, you need to
appreciate that hydrophilic amino acids are more likely to appear on the surface of a protein molecule,
whereas hydrophobic amino acids are most likely be found in the interior. Next, you need to figure out
which of the amino acids listed is hydrophilic. If you recall that arginine is a basic amino acid that is
positively charged at physiologic pH, you should be able to answer this question right away.
All the other choices have neutral side chains and are uncharged at physiologic pH. They would most
likely be found in the hydrophobic core of the protein structure. Alanine (choice A), isoleucine (choice
C), and leucine (choice D) all have aliphatic side chains; phenylalanine (choice E) and tryptophan
have aromatic side chains.
14. Dietary intake of which of the following amino acids can substitute for a portion of the
daily requirement of niacin?
A. Alanine
B. Asparagine
C. Methionine
D. Proline
E. Tryptophan
The correct answer is E. Tryptophan is an aromatic amino acid that contains an indole group. By a
complex series of minor enzymatic reactions, a small amount (~2%) of the tryptophan can be
converted to quinolinate, which can then be used in place of niacin (nicotinic acid) in NAD
(nicotinamide adenine dinucleotide) synthesis. Very high tryptophan levels can replace a portion of
the dietary requirements for niacin. The nutritional disease pellegra (characterized by swollen tongue,
dermatitis, neurologic dysfunction, and gastrointestinal dysfunction) usually occurs in the setting of
combined tryptophan and niacin deficiency.
Alanine (choice A), the amino acid with a methyl R group, is a substrate of the liver enzyme alanine
amino transferase (ALT, formerly called SGPT). Asparagine (choice B) is one of the sources of
ammonia for the urea cycle. Methionine (choice C) is a sulfur-containing amino acid that is
associated with methyl group transfer. Proline (choice D) is technically an imino acid, rather than an
amino acid, with a ring structure. You should remember that collagen has a high proline
concentration.
15. A child is noted to be severely retarded. Physical examination reveals a pot-bellied, pale
child with a puffy face. The child's tongue is enlarged. Dietary deficiency of which of the
following substances can produce this pattern?
A. Calcium
B. Iodine
C. Iron
D. Magnesium
E. Selenium

The correct answer is choice B. The disease is cretinism, characterized by a profound lack of
thyroid hormone in a developing child, leading to mental retardation and the physical findings
described in the question stem. Cretinism can be due to dietary deficiency of iodine (now rare in this
country because of iodized salt), to developmental failure of thyroid formation, or to a defect in
thyroxine synthesis. Iodine in the diet is absorbed at the digestive tract as iodide (I-). The follicle cells
in the thyroid gland absorb 120-150 g of iodide ions per day. Iodide ions are actively transported into
the thyroid follicle cells. The active transport mechanism for iodide is stimulated by TSH.
Calcium deficiency (choice A) in children can cause osteoporosis or osteopenia.
Iron deficiency (choice C) can cause a hypochromic, microcytic anemia.
Magnesium deficiency (choice D) is uncommon, but can cause decreased reflexes, and blunts the
parathyroid response to hypocalcemia.
Selenium deficiency (choice E) is rare, but may cause a reversible form of cardiomyopathy.
16. A chronically malnourished patient notices her "hair falling out." She is on a strict fat-free
diet. She probably has a deficiency in which vitamins?
A. Vitamin A
B. Vitamin C
C. Vitamin D
D. Vitamin E
E. Vitamin K
The correct answer is A. While it is hard to develop a deficiency in oil-soluble vitamins (A, D, E, K)
because the liver stores these substances, deficiency states can be seen in chronic malnutrition
(specifically chronic fat deprivation) and chronic malabsorption. Vitamin A deficiency is one of the
most common deficiencies in developing countries. Vitamin A is necessary for formation of retinal
pigments (deficiency can cause night blindness) and for appropriate differentiation of epithelial tissues
(including hair follicles, mucous membranes, skin, bone, and adrenal cortex).
Vitamin C (choice B), which is water soluble rather than oil soluble, is necessary for collagen
synthesis. Deficiency is associated with development of scurvy.
Vitamin D (choice C) is important in calcium absorption and metabolism. Calcium and phosphate
irregularities can occur in deficiencies.
Vitamin E (choice D) is a lipid antioxidant that is important in the stabilization of cell membranes.
Vitamin K (choice E) is necessary for normal blood coagulation. When vitamin K is deficient, clotting
factors 2, 7, 9, and 10 will be decreased leading to an increased pT/INR.
17. Which of the following amino acids is most responsible for the buffering capacity of
hemoglobin and other proteins?
A. Arginine
B. Aspartic acid
C. Glutamic acid
D. Histidine
E. Lysine
The correct answer is D.Histidine, lysine and arginine are amino acids with basic side chains.
Remember that a buffer is most effective when its pKa is within the pH range of the surrounding

medium. Histidine is the only amino acid with good buffering capacity at physiologic pH. The
imidazole side chain of histidine has a pKa around 6.5 and can reversibly donate and accept protons
at physiologic pH.
Arginine (choice A) and lysine (choice E) are basic amino acids with pKa's of 12.5 and 10.5,
respectively; at physiologic pH both will behave as bases and accept protons.
Aspartic acid (choice B) and glutamic acid (choice C) are acidic amino acids with pKa's of
approximately 4; at physiologic pH they will behave as acids and donate protons. Their side chains
are almost always negatively charged.
18. Which of the following cofactors is required for decarboxylation of alpha-ketoacids?
A. Vitamin B1
B. Vitamin B2
C. Vitamin B3
D. Vitamin B5
E. Vitamin B6
The correct answer is A. Vitamin B1, or thiamine, is the coenzyme required (as the pyrophosphate)
for the decarboxylation of alpha-ketoacids. An example of this reaction is pyruvate decarboxylase
reaction in alcoholic fermentation. Other reactions such as that catalyzed by pyruvate dehydrogenase
also rely on thiamine pyrophosphate for decarboxylation, but require other cofactors as well. Thiamine
is also required for the generation of pentose phosphates for nucleotide synthesis in the pentose
phosphate pathway (hexose monophosphate shunt), serving as a cofactor for transketolase. The
primary cause of thiamine deficiency in the US is due to alcoholism. Thiamine deficiency is known as
beri beri. As this condition progresses, Wernicke-Korsakoff syndrome, which is seen in chronic
alcoholics, can develop.
Vitamin B2 (choice B), or riboflavin, is a constituent of FMN (flavin mononucleotide) and FAD (flavin
adenine dinucleotide). It functions in hydrogen and electron transport. Deficiency is associated with
the development of cheilosis, angular stomatitis and glossitis.
Vitamin B3 (choice C), or niacin (nicotinic acid), is a coenzyme that is also involved in hydrogen and
electron transport. Nicotinic acid functions in the form of NAD and NADP. Niacin is used clinically for
the treatment of hypercholesteremia and hypertriglyceredemia.
Vitamin B5 (choice D), or pantothenic acid, is conjugated with coenzyme A to act as a carboxylic acid
carrier.
Vitamin B6 (choice E), or pyridoxine, is required as a cofactor for pyridoxal phosphate and
pyridoxamine phosphate. Both of these cofactors are essential to protein metabolism and energy
production. Deficiencies can lead to the development of mouth soreness, glossitis, cheilosis,
weakness and irritability. Severe deficiency can cause peripheral neuropathy and anemia.
19. A stretch of 25 hydrophilic amino acids in a protein could be found in a
A. signal sequence
B. start transfer sequence
C. stop transfer sequence
D. transmembrane domain
E. triple helix
The correct answer is E. A triple helix, such as the one found in collagen, is composed of three
polypeptide chains wound together to form one structure. The chains are highly enriched with the
hydrophilic amino acids glycine and proline.

A signal sequence (choice A) contains a stretch of 30 hydrophobic amino acids.


The start transfer sequence (choice B) and stop transfer sequence (choice C) are each composed of
25-30 hydrophobic amino acids.
Transmembrane domains (choice D) contain 25-30 hydrophobic amino acids. This is enough to form
an alpha helix that will span a lipid bilayer.
20. The process involved in formation of protein from RNA is
A. Replication
B. Translation
C. Transcription
D. Transformation
The correct answer is choice B.In translation, protein synthesis occurs by peptide bond formation
between successive amino acids, whose order is specified by a gene and thus by an mRNA
Choice A - Replicationis a process in which two daughter DNA molecule are produced, that are each
identical to the parental DNA molecule
Choice C - Transcription is a process in which mRNA is synthesized using DNA as template.
Choice D - Transformation is not involved in protein synthesis either from DNA or RNA
Putting the processes together:
21. Folate plays a role in single-carbon unit transfer in the synthesis of nucleotides. Which of
the following nucleotides require folate for synthesis?
A. Adenosine, cytosine, and uracil
B. Adenosine, guanine, and thymidine
C. Adenosine, guanine, and uracil
D. Cytosine, thymidine, and uracil
E. Guanine, thymidine, and uracil
The correct answer is choice B. In summary, a nucleoside consists of a purine or pyrimidine base
linked to a pentose, and a nucleotide is a phosphate ester of a nucleoside. For example, adenine is
the base of the ribonucleoside adenosine and the ribonucleotide adenylate (AMP). Folate is involved
in the transfer of carbons 2 and 8 of the purine nucleus (affecting adenosine and guanosine) and the
5-methyl group of thymidine. This means that folate is required for synthesis of 3 of the 4 nucleic acid
bases of DNA and 2 of the 4 nucleic acid bases in RNA. It is thus no wonder that folate deficiency has
effects on so many tissues with high mitotic rate. Megaloblastic changes analogous to those seen in
erythrocytes and their precursors can also be seen in other cells produced by bone marrow
(neutrophil, eosinophil, basophil, and macrophage as well as megakaryocyte lines) and in epithelia
throughout the body, including skin, mucous membranes such as the mouth and vagina (where the
changes can be seen on Pap smear), stomach and intestinal linings, and cells from lung or liver.
Similar megaloblastic changes are observed throughout the body when cobalamin (vitamin B12)
deficiency is present, since cobalamin plays a role in methionine synthesis, which is the source of the
one-carbon unit "active-formate." Cobalamin is also involved in the conversion of methylmalonic acid
to succinic acid and is required to maintain the integrity of nerve cells via an unknown biochemical
pathway.

22. A 45-year-old mother has a 5-year-old daughter with a history of repeated respiratory
infections. The child has mental retardation, short stature, hypotonia with depressed nasal
bridge, upslanting palpebral fissure, and epicanthal fold. She is also at an increased risk for
leukemia. What is the probable chromosome abnormality?
A. Trisomy 13
B. Trisomy 18
C. Trisomy 21
D. Turner syndrome (45,X)
The correct answer is C. Trisomy 21 is the most common autosomal trisomy, and it causes Down
syndrome. The increased risk of Down syndrome with increased maternal age is well documented.
Trisomy 13 (choice A) features include oral facial clefts, microophthalmia, polydactyly, and renal
defects. They usually do not survive to 1 year.
Trisomy 18 (choice B) features include small mouth and ears, congenital heart defects, and
overlapping fingers. Ninety percent die during the first year of life.
Turner syndrome (choice D) features include reduced stature, webbed neck, and lymphedema of the
wrists and ankles.
23. The biochemical structure of all the hormones secreted by the anterior pituitary, posterior
pituitary, and pancreas can best be described as which of the following?
A. Amino acid derivatives
B. Catecholamines
C. Glucocorticoids
D. Peptides
E. Steroid hormones
The correct answer is D.In the human body there are three basic types of hormones: peptides
(protein derivatives), amino acid derivatives, and steroid hormones. Some examples of peptide
hormones are those produced in the anterior pituitary (growth hormone, adrenocorticotropin, thyroid
stimulating hormone, luteinizing hormone, and prolactin), the posterior pituitary (vasopressin and
oxytocin), and the pancreas (insulin and glucagon).
Amino acid derivatives (choice A) include tyrosine and triiodothyronine, as well as the
catecholamines (choice B) dopamine, epinephrine, and norepinephrine.
Glucocorticoid hormones (choice C; e.g., cortisol) are a specific type of steroid hormone produced by
the adrenal cortex.
Steroid hormones (choice E) include the hormones produced in the adrenal cortex (cortisol and
aldosterone), the ovaries (estrogen and progesterone), the testes (testosterone), and the placenta
(estrogen and progesterone).
24. Which of the following enzymes is stimulated by glucagon?
A. Acetyl-CoA carboxylase
B. Glycogen phosphorylase
C. Glycogen synthase
D. HMG-CoA reductase
E. Pyruvate kinase

10

The correct answer is choice B. Before you started analyzing all of the answer choices you should
have reminded yourself that glucagon increases serum glucose. So an enzyme stimulated by
glucagon might be involved in either the breakdown of glycogen to glucose (glycogenolysis) or in the
creation of glucose from noncarbohydrate precursors (gluconeogenesis). Glycogen phosphorylase
catalyzes the first step in glycogenolysis; it makes sense that it would be stimulated by glucagon.
Clinical correlate: patients with Type I diabetes lose their response to hypoglycemia (but not to amino
acids in protein containing meals) within a year or so after developing diabetes. Type I diabetics rely
on the sympathetic nervous system to counter regulate hypoglycemia.
Acetyl-CoA carboxylase (choice A) catalyzes the first step in fatty acid synthesis, an anabolic process
that would be stimulated by insulin, not glucagon.
As its name implies, glycogen synthase (choice C) is involved in the synthesis of glycogen. Glucagon
(and epinephrine) stimulates the phosphorylation and inactivation of glycogen synthase.
HMG-CoA reductase (choice D) is the key enzyme involved in the synthesis of cholesterol. Since this
is an anabolic process that occurs in the well-fed state, you would expect it to be stimulated by insulin
and inhibited by glucagon (which it is).
Pyruvate kinase (choice E) catalyzes the last reaction of glycolysis. You would expect it to be
inhibited by glucagon (thus decreasing the amount of glucose consumption). Glucagon promotes the
phosphorylation of pyruvate kinase, which renders it inactive.
25. The majority of ATP generated during a 100 meter race is derived from which of the
following?
A. ATP stores
B. Creatine phosphate
C. Gluconeogenesis
D. Glycolysis
E. Lipolysis
The correct answer is D.The key to this question is understanding how and when the body utilizes
fuel stores. The stores of ATP (choice A) will be used up in less than 1 second once the race has
started. Creatine phosphate (choice B) will be the primary source of energy for the next 3 or 4
seconds. After the creatine phosphate stores are depleted, the majority of ATP needed to complete
the race will be derived from glycolysis (anaerobic respiration). If the race were to last for an extended
period of time, then the processes of gluconeogenesis (choice C) and lipolysis (choice E) might be
utilized. Gluconeogenesis is the process of synthesizing glucose in the liver from non-carbohydrate
sources, such as amino and fatty acids. Lipolysis is the splitting up or decomposition of fat in the
body.
26. As cells in the erythrocytic lineage mature and lose their nuclei, mitochondria, and
ribosomes, which of the following pathways can still be used to produce ATP?
A. Citric acid cycle
B. Electron transport chain
C. Glycolysis
D. Malate shuttle
E. Urea cycle
The correct answer is C. Circulating erythrocytes have a life span of about 60 days and are
dependent on a functioning Na+/K+ ATPase in the plasma membrane. This pump provides the
electrochemical force across the plasma membrane that helps to maintain the volume of the red cell

11

at a constant level by regulating salt, and consequently water, flow into and out of the cell. When the
pump stops, the erythrocytes tend to lyse. It is therefore of extreme importance to the erythrocyte to
supply the ATP necessary to keep the pump running. This is accomplished through the use of
glycolysis, which converts glucose to pyruvate, with a net production of 2 ATP per glucose molecule.
The Krebs, or citric acid, cycle (choice A) is the central metabolic degradative pathway of aerobic
cells. However, it is located in mitochondria and requires the electron transport chain, making it
unavailable for mature red cells to use.
The electron transport chain (choice B) is located on the mitochondrial cristae and helps to convert
the energy of NADH and FADH2 to ATP.
The malate shuttle (choice D) is used to transport cytoplasmic NADH electrons into mitochondria and
is not available to erythrocytes.
The urea cycle (choice E) is used by liver cells for processing nitrogenous wastes, not generating
energy.
27. Megaloblastic anemia with folate deficiency is linked to an inability to perform which type
of enzymatic process?
A. Acyl transfer
B. Carboxylation
C. Decarboxylation
D. Hydroxylation
E. Methylation
The correct answer is E. Folic acid is a pteridine vitamin that exists as tetrahydrofolate (TH4) in its
most reduced form. TH4 can accept methyl, methylene, or formyl carbons and transfer them as
methyl groups. This function is vital in nucleotide and amino acid synthesis.
Pantothenic acid is a key vitamin in acyl transfer reactions (choice A). It forms part of coenzyme A,
which transfers acyl groups in thiol esters as acetyl-CoA, succinyl-CoA, and other acyl-CoA forms.
Important vitamins in carboxylation reactions (choice B) include biotin and vitamin K. Biotin carries
the carboxyl group in the pyruvate carboxylase and acetyl-CoA carboxylase reactions. Vitamin K is
used in post-translational carboxylation of amino acid residues in blood clotting factors.
Oxidative decarboxylation reactions (choice C) require thiamine (vitamin B1). Examples include the
pyruvate dehydrogenase and alpha-ketoglutarate dehydrogenase complexes.
Ascorbic acid (vitamin C) is a coenzyme in the hydroxylation (choice D) of lysyl and prolyl residues
for collagen synthesis.
28. Long term broad spectrum antibiotics with reduced clotting time in localized areas is an
indication of a deficiency in what?
A. Vitamin C
B. Vitamin K
C. Cyanocobalamine (B12)
D. Thiamine (B1)
The correct answer is choice B. Prolonged treatment with broad-spectrum antibiotics eliminates
intestinal bacteria that supply vitamin K. Vitamin K catalyses gamma carboxylation of glutamic acid
residues on various proteins concerned with blood clotting. Deficiency is characterized by prolonged
PT but normal bleeding time.

12

Choice A - Vitamin C deficiency leads to scurvy with increased bleeding time but normal PT. A diet
deficient in vitamin C results in scurvy.
Choice C - Vitamin B12 deficiency is usually caused by malabsorption and results in megaloblastic
anemia and progressive peripheral neuropathy.
Choice D - The most common cause of thiamine deficiency is alcoholism. Because alcohol interferes
with absorption, deficiency results in dry and wet beri-beri.
29. The finding that almost all of the melanoma cells have very large, visible nucleoli suggests
that these cells are making large amounts of which of the following?
A. Cell surface markers
B. Golgi apparatus
C. Immunoglobulins
D. New DNA
E. Ribosomes
The correct answer is E. The nucleolus is the site of manufacture of ribosomal RNA with its
subsequent packaging into ribosomes. Consequently, very large nucleoli indicate an increased rate of
ribsome production, which, in turn, suggests an increased rate of production of proteins. Large
nucleoli are seen in many active cancers but can also be prominent in benign conditions
characterized by high metabolic rate, such as tissue repair after trauma.
Depending on the cells involved, the increased protein production associated with more ribosomes
might produce more cell surface markers (choice A), but it is impossible to predict this simply from
the presence of a large nucleolus.
You should associate a large Golgi apparatus (choice B) with transport of material to the cell surface.
Plasma cells that produce large amounts of immunoglobulins (choice C) may have large nucleoli, but
more specifically have a very large endoplasmic reticulum and Golgi apparatus.
The nucleolus is not involved with new DNA (choice D) synthesis; this is done elsewhere in the
nucleus.
30. Which type of enzyme reaction is effected by a folic acid deficiency?
A. Acyl transfer
B. Carboxylation
C. Decarboxylation
D. Hydroxylation
E. Methylation
The correct answer is E. Folic acid deficiency results in the development of macrocytic anemia that
yields macro-ovalocytes and hypersegmented neutrophils on the peripheral blood smear. Folic acid is
a pteridine vitamin that exists as tetrahydrofolate (TH4) in its most reduced form. TH4 can accept
methyl, methylene, or formyl carbons and transfer them as methyl groups. This function is vital in
nucleotide and amino acid synthesis. By far the most common cause of folate deficiency is
inadequate dietary intake. Alcoholics and those with poor diets are at the highest risk for developing
this type of anemia.
Pantothenic acid is a key vitamin in acyl transfer reactions (choice A). It forms part of coenzyme A,
which transfers acyl groups in thiol esters as acetyl CoA, succinyl CoA, and other acyl CoA forms.

13

Important vitamins in carboxylation reactions (choice B) include biotin and vitamin K. Biotin carries
the carboxyl group in the pyruvate carboxylase and acetyl CoA carboxylase reactions, and vitamin K
is utilized in post-translational carboxylation of amino acid residues in blood clotting factors.
Oxidative decarboxylation reactions (choice C) require thiamine (vitamin B1). Examples include the
pyruvate dehydrogenase and alpha-ketoglutarate dehydrogenase complexes.
Ascorbic acid (vitamin C) is a coenzyme in the hydroxylation (choice D) of lysyl and prolyl residues of
collagen.
31. Which of the following inhibits the activity of acetyl-CoA carboxylase?
A. Citrate
B. Glucagon
C. High-carbohydrate, low-fat diet
D. Insulin
The correct answer is choice B. The key thing to remember here is that acetyl-CoA carboxylase
catalyzes the first and rate-limiting step of fatty acid synthesis. If you got that far, you could have
figured out which of the choices would inhibit the synthesis of fatty acids. Certainly glucagon, a
catabolic hormone released in response to low blood glucose, would be a likely candidate to inhibit
the synthesis of fatty acids. In fact, glucagon inhibits fatty acid synthesis by a cAMP-dependent
phosphorylation of acetyl-CoA carboxylase. Conversely, glucagon stimulates fatty acid oxidation.
Clinical correlate: Glycogen metabolism is profoundly affected by specific hormones. Insulin, a
polypeptide hormone, increases the capacity of the liver to synthesize glycogen. When insulin levels
are high, the production of glycogen is high. The action of insulin is opposed by both glucagon and
epinephrine which will increase blood glucose levels.
Citrate (choice A) is a key player in fatty acid synthesis (citrate shuttle). Therefore, the presence of
citrate would stimulate, not inhibit, acetyl-CoA carboxylase.
A high-carbohydrate, low-fat diet (choice C) would stimulate, not inhibit, the synthesis of fatty acids.
In contrast to glucagon, insulin (choice D) is an anabolic hormone that promotes fatty acid synthesis
and therefore would stimulate acetyl-CoA carboxylase. It does so by dephosphorylating the enzyme.
32. From which intermediate in the glycolytic pathway does the pentose phosphate pathway
(also known as the hexose monophosphate, or pentose, shunt) "shunt"?
A. Fructose-1,6-bisphosphate
B. Fructose-6-phosphate
C. Glucose-6-phosphate
D. Phosphoenolpyruvate
E. Pyruvate
The correct answer is C. The hexose monophosphate shunt is an alternative route for the oxidation
of glucose; it supplies the cell with NADPH and pentose sugars. The NADPH is used in many
biosynthetic processes (e.g., fatty acid and cholesterol synthesis), whereas the pentoses are involved
in the synthesis of nucleotides and some coenzymes. The pathway "shunts" from glucose-6phosphate, which is oxidized in a series of NADPH-generating reactions, to ribulose-5-phosphate.
The nonoxidative phase, which involves the transfer of C2 and C3 units from one sugar to another,
follows. One resulting intermediate is fructose-6-phosphate (choice B), which can serve as a re-entry
point to glycolysis, thereby closing the"shunt" loop.

14

Although all the other choices are glycolytic intermediates, they are not involved in the hexose
monophosphate shunt.
33. Loss of which of the following classes of molecules on the surface of a tumor cell target
would result in loss of susceptibility to killing by host immune cells?
A. CD3
B. CD4
C. CD8
D. MHC class I
E. MHC class II
The correct answer is D.Major histocompatibility complex (MHC) class I proteins are found in the
membranes of all nucleotide cells. If the cell is healthy and the peptides are normal, T cells will ignore
them. If the cytoplasm contains abnormal peptides, they will appear in the cell membrane and the T
cells will be activated which will lead to the destruction of the cell. After the MHC class I molecule has
moved to the surface of the tumor cell, peptide fragments from the tumor are presented in a groove of
the class I molecule. The peptide fragments are presented to cytotoxic CD8 T cells, which recognize
the MHC class I molecules on the cell surface and kill the tumor cell. Loss of this molecule would
therefore prevent the tumor cell from being killed.
The CD3 molecule (choice A) is a marker on all T cells. It is involved in signal transduction, but not
antigen recognition. This molecule would not be on the surface of tumor cells.
The CD4 molecule (choice B) is not on the surface of a tumor cell, but it is on the surface of a CD4+
T helper lymphocyte.
The CD8 molecule (choice C) is not on the surface of a tumor cell, but it is on the surface of a CD8+
cytotoxic T lymphocyte.
MHC class II antigens (choice E) are not involved in killing of tumor cell targets. They present peptide
fragments (derived from intracellular killing of extracellular organisms by macrophages) to CD4 T
lymphocytes.
34. Which of the following citric acid cycle intermediates is required for heme synthesis?
A. a-Ketoglutarate
B. Fumarate
C. Isocitrate
D. Oxaloacetate
E. Succinyl-CoA
The correct answer is E. The porphyrin ring of heme is derived from the citric acid cycle intermediate
succinyl-CoA and the amino acid glycine. The initial synthetic step, which is rate-limiting, is catalyzed
by aminolevulinic acid synthase (ALA synthase).
35. Which of the following metabolic processes occurs exclusively in the mitochondria?
A. Cholesterol synthesis
B. Fatty acid synthesis
C. Gluconeogenesis
D. Ketone body synthesis
E. Urea cycle

15

The correct answer is D. Of the processes listed, only ketone body synthesis occurs exclusively in
the mitochondria. Other mitochondrial processes include the production of acetyl-CoA, the TCA cycle,
the electron transport chain, and fatty acid oxidation. The most common cause of ketone body
formation is ketoacidosis secondary to diabetes. The essential diagnostic characteristics include:
hyperglycemia, acidosis (with blood pH < 7.3), serum bicarbonate < 15 and serum positive for
ketones.
Processes that occur exclusively in the cytoplasm include cholesterol synthesis (choice A; in cytosol
or in ER) and fatty acid synthesis (choice B).
Note that gluconeogenesis (choice C) and the urea cycle (choice E) occur in both the mitochondria
and the cytoplasm.
36. An infant diagnosed with phenylketonuria would be expected to be deficient in which of the
following nonessential amino acids, assuming that it is not obtained from dietary sources?
A. Asparagine
B. Cysteine
C. Glutamine
D. Proline
E. Tyrosine
The correct answer is E. The human body is able to synthesize roughly half the amino acids
necessary to build protein. These amino acids, termed nonessential, include alanine, arginine,
asparagine, aspartate, cysteine, glutamate, glutamine, glycine, proline, serine, and tyrosine. The
amino acids that must be supplied in the diet are termed essential; these include histidine, isoleucine,
leucine, lysine, methionine, phenylalanine, threonine, tryptophan, and valine. Phenylalanine
undergoes hydroxylation to tyrosine, catalyzed by the enzyme phenylalanine hydroxylase. It is
noteworthy that tyrosine becomes an essential amino acid in individuals lacking this enzyme. The
hyperphenylalaninemias, which include phenylketonuria, result from impaired conversion of
phenylalanine to tyrosine, and are also asociated with mental retardation. This condition is associated
with increased phenylalanine in blood and increased phenylalanine and its by-products (e.g.,
phenylpyruvate, phenylacetate, and phenyllactate) in urine.
Asparagine (choice A) is a member of the "oxaloacetate family"; its immediate precursor is aspartate.
The immediate precursor of cysteine (choice B) is serine. Serine is also the precursor of the
nonessential amino acid glycine.
Glutamine (choice C), proline (choice D), and arginine are produced from glutamate. The synthesis
of glutamate occurs by the reductive amination of alpha-ketoglutarate.
37. Which of the following cofactors is required for decarboxylation of alpha-ketoacids?
A. Vitamin B1
B. Vitamin B2
C. Vitamin B3
D. Vitamin B5
E. Vitamin B6
The correct answer is A. Vitamin B1, or thiamine, is the coenzyme required for the decarboxylation
of alpha-ketoacids. An example of this reaction is pyruvate acetyl CoA, catalyzed by the pyruvate
dehydrogenase complex (although this particular reaction requires other cofactors, thiamine is
required in all alpha-ketoacid decarboxylations). Thiamine is also required for the generation of
pentose phosphates for nucleotide synthesis in the pentose phosphate pathway (hexose
monophosphate shunt), serving as a cofactor for transketolase.

16

Vitamin B2 (choice B), or riboflavin, is a constituent of FMN (flavin mononucleotide) and FAD (flavin
adenine dinucleotide). It functions in hydrogen and electron transport.
Vitamin B3 (choice C), or niacin (nicotinic acid), is a coenzyme that is also involved in hydrogen and
electron transport. Nicotinic acid functions in the form of NAD and NADP.
Vitamin B5 (choice D), or pantothenic acid, is conjugated with coenzyme A to act as a carboxylic acid
carrier.
Vitamin B6 (choice E), or pyridoxine, is used as pyridoxal phosphate and pyridoxamine phosphate.
Both of these cofactors are essential to protein metabolism and energy production.
38. Albinism is a disorder involving what substance?
A. Aromatic amino acids
B. Branched chain amino acids
C. Glycolipids
D. Glycoproteins
E. Sulfur-containing amino acids
The correct answer is A. The disease is albinism. The most common form of albinism is caused by a
deficiency of copper-dependent tyrosinase (tyrosine hydroxylase), blocking the production of melanin
from the aromatic amino acid tyrosine. Affected individuals lack melanin pigment in skin, hair, and
eyes, and are prone to develop sun-induced skin cancers, including both squamous cell carcinomas
and melanomas.
Maple syrup urine disease is an example of a disorder of branched chain amino acids (choice B)
causing motor abnormalities and seizures.
Tay-Sachs disease is an example of a disorder of glycolipids (choice C). In this disorder, a deficiency
of hexosaminidase A leads to accumulation of ganglioside GM2.
Hunter's disease is an example of a disorder of glycoproteins
mucopolysaccharidosis is inherited as an X-linked recessive trait.

(choice

D).

This

Homocystinuria disease is an example of a disorder of sulfur-containing amino acids (choice E).


39. A woman vegetarian is found to have a severe riboflavin deficiency. The function of which
of the following enzymes in the citric acid cycle would be most directly affected by the
riboflavin deficiency?
A. Aconitase
B. Citrate synthase
C. Isocitrate dehydrogenase
D. Malate dehydrogenase
E. Succinate dehydrogenase
The correct answer is E. To answer this question you need two separate pieces of information. First,
riboflavin (one of the B vitamins) is used to make the flavin part of FAD (flavin adenine dinucleotide).
Second, of the citric acid cycle enzymes listed, only succinate dehydrogenase (which catalyzes the
conversion of succinate to fumerate) used FAD (which is converted to FADH2) as a cofactor.
Aconitase (choice A) converts citrate to isocitrate and does not require a cofactor.

17

Citrate synthetase (choice B) combines acetyl-CoA and oxaloacetate to make citrate with release of
coenzyme A (which requires pantothenic acid for synthesis).
Isocitrate dehydrogenase (choice C) converts isocitrate to alpha-ketoglutarate and uses NAD+ (which
is converted to NADH + H+). The NAD+ requires the vitamin niacin for synthesis.
Malate dehydrogenase (choice D) converts malate to oxaloacetate and uses NAD+ (which converted
to NADH + H+). The NAD+ requires the vitamin niacin for synthesis.
40. Which of the following pairs of enzymes is required for the process of gluconeogenesis?
A. Fructose-1,6-bisphosphatase and pyruvate carboxylase
B. Glucose-6-phosphatase and phosphofructokinase-1
C. Glucose-6-phosphatase and pyruvate dehydrogenase
D. Phosphoenolpyruvate carboxykinase and glucokinase
E. Pyruvate kinase and pyruvate carboxylase
The correct answer is A. Gluconeogenesis is the process that results in the synthesis of glucose
from non-carbohydrate precursors. This pathway is very important because the brain is highly
dependent on glucose as the primary source of fuel. The three irreversible steps of glycolysis are
catalyzed by hexokinase, phosphofructokinase-1 (choice B), and pyruvate kinase. In
gluconeogenesis, other enzymes are needed to bypass these key steps. Pyruvate cannot be directly
converted to phosphoenolpyruvate in gluconeogenesis. Therefore, pyruvate carboxylase (a
mitochondrial enzyme; choice A) converts pyruvate to oxaloacetate, which can be converted to
phosphoenolpyruvate by phosphoenolpyruvate carboxykinase (choice D), using two ATP equivalents
per molecule of phosphoenolpyruvate. Fructose-1,6-bisphosphatase (choice A) is the enzyme that
splits fructose-1,6-bisphosphate into fructose-6-phosphate and inorganic phosphate. It is also required
for gluconeogenesis.
Glucose-6-phosphatase (choices B and C) is a liver enzyme that hydrolyzes glucose-6-phosphate to
glucose. A deficiency of this enzyme leads to von Gierke disease, also known as glycogen storage
disease type I.
Pyruvate dehydrogenase (choice C) is a mitochondrial enzyme that converts pyruvate to acetyl CoA.
This enzyme requires thiamine pyrophosphate, lipoamide, and FAD as cofactors.
Glucokinase (choice D) is a liver enzyme that converts glucose to glucose-6-phosphate. Unlike
hexokinase, it is specific for glucose and is unresponsive to the level of glucose-6-phosphate. Its
function is to store excess glucose, so it has a very high Km (ie, a low affinity) for glucose, becoming
active only when the concentration of glucose is very high.
Pyruvate kinase (choice E) catalyzes the conversion of phosphoenolpyruvate to pyruvate in the
glycolytic pathway. It is activated by fructose-1,6-bisphosphate, the product of the committed step of
glycolysis, and is allosterically inhibited by ATP, alanine, and acetyl CoA.
41. Cyanide poisoning will directly affect which anabolic process?
A. Breaking of covalent bonds in glucose molecules
B. Formation of carbon dioxide
C. Formation of GTP from GDP
D. Movement of hydrogen ions through channels in the respiratory enzymes
E. Splitting of water molecules into hydrogen and oxygen atoms
The correct answer is D.Cyanide binds to the heme Fe3+ of cytochrome a3 in the electron transport
system, blocking the transfer of electrons to oxygen and consequently the synthesis of ATP. In other

18

words, cyanide blocks the process of oxidative phosphorylation, which produces greater than 90% of
the ATP used by the cells in our body. The major steps involved in this process occur within the
"electron transport system (ETS)," or "respiratory chain," of the mitochondria. The steps at the end of
the elctron transport system, where ATP is generated, are as follows: along the ETS, the repiratory
enzymes continually pump hydrogen ions from the matrix of the mitochondria to the intermembrane
space, which creates a large concentration gradient. At the end of the ETS, hydrogen ions pass
through channels in the respiratory enzymes along the concentration gradient. As the hydrogen ions
pass through these enzymes, the energy created is used to phophorylate ADP to ATP. Inhibition of
mitochondrial respiration can lead to coma and death unless diagnosed and treated early.
The breaking of covalent bonds in glucose molecules (choice A), the formation of carbon dioxide in
the TCA cycle (choice B), and the splitting of water molecules into hydrogen and oxygen atoms
(choice E) all require energy; these processes do not create energy.
The formation of GTP from GDP (choice C) is a process that creates high energy phosphates in the
tricarboxylic acid cycle.
42. An individual lacking the enzyme tyrosinase would be particularly predisposed to develop
which of the following?
A. Glioblastoma multiforme
B. Hemangioblastoma
C. Hepatoma
D. Melanoma
E. Renal cell carcinoma
The correct answer is D.This question is simple if you know that tyrosinase is an enzyme in the
biosynthetic pathway for melanin formation from tyrosine. A lack of tyrosinase causes one form of
albinism; a second form is caused by defective tyrosine uptake. Patients with albinism are vulnerable
to developing cancers of the skin of all types, including basal cell carcinoma, squamous cell
carcinoma, and melanoma. The melanomas are unusual in that they are non-pigmented (amelanotic)
rather than black, since the patients cannot form melanin. Malignant melanoma is the leading cause
of death from skin disease. Tumor thickness (Breslow's Classification) is the single most important
prognostic factor. These lesions are often flat or raised. The borders are typically irregular. Remember
many cases of melanoma occur with lesions on the head and neck.
43. Thiamine is used by which coenzyme in order to prevent a lactic acid acidosis?
A. Lactate dehydrogenase
B. Pyruvate carboxylase
C. Pyruvate dehydrogenase
D. Pyruvate kinase
E. Transketolase
The correct answer is C. Thiamine is a water-soluble vitamin that is converted to the coenzyme
thiamine pyrophosphate. This coenzyme is used by pyruvate dehydrogenase to convert pyruvate to
acetyl coenzyme A. In the absence of thiamine, pyruvate accumulates and can be converted by
lactate dehydrogenase to lactate, which is spilled in the blood causing lactic acidosis.
As a side note, thiamine deficiency is associated with the development of beri-beri - a neurological
and cardiovascular disorder. The most common cause of beri-beri in the US today is seen in
alcoholics. Damage to the nerves is expressed in terms of pain in the limbs and weakness of
musculature. The heart can become enlarged and cardiac output can be decreased.
Lactate dehydrogenase (choice A) produces lactate from pyruvate but does not use thiamine
pyrophosphate.

19

Some lactic acidosis might be produced by decreased pyruvate carboxylase activity (choice B), but
the enzyme requires biotin rather than thiamine pyrophosphate.
Pyruvate kinase (choice D) makes pyruvate from phosphoenolpyruvate, but does not use thiamine
pyrophosphate.
Transketolase (choice E) requires thiamine pyrophosphate, but operates in another pathway
(pentose phosphate pathway). Decreased transketolase activity is not associated with the
development of lactic acidosis.
44. A patient's lab studies demonstrate a deficiency of cytochrome C oxidase activity. A defect
in which of the following subcellular organelles would cause this deficiency?
A. Golgi apparatus
B. Lysosomes
C. Mitochondria
D. Ribosomes
E. Smooth endoplasmic reticulum
The correct answer is C. Cytochrome C oxidase is an electron transport chain component, in the
mitochondria of muscle and brain. Cytochromes and the electron transport chain are involved in
oxidative respiration. Defects can produce mental retardation, weakness, ataxia or seizures.
The Golgi apparatus (choice A) is involved in packaging materials for secretion outside the cell. You
should associate mucolipidosis I-cell disease with Golgi apparatus problems.
Lysosomes (choice B) are the organelles that degrade many cellular products. Defects can produce
a wide variety of lysosomal storage diseases, including Hurler and Hunter syndromes.
Ribosomes (choice D) are the organelles that traslate mRNA into proteins. There are no known
diseases of ribosomes, probably because their function is so crucial that any problems produce death
in utero.
Hypertrophy of the smooth endoplasmic reticulum (choice E) in the liver is associated with conditions
that stimulate the cytochrome p450 detoxification systems (e.g., barbiturate use and alcoholism).
45. A genetic mutation that results in abnormal stimulatory G protein (Gs) structure would
adversely affect which of the following mechanisms?
A. Active transport
B. Facilitated diffusion
C. Pinocytosis
D. Receptor-mediated endocytosis
E. Signal transduction
The correct answer is E. G proteins are involved in signal transduction, which allows
macromolecules to affect a cell's biological functions without crossing the plasma membrane. The
binding of an agonist, such as a hormone, to a receptor on the plasma membrane causes a
conformational change in the receptor, which then interacts with a stimulatory G protein (Gs),
accompanied by the binding of GTP to the alpha subunit of the G protein. The alpha subunit of the G
protein (with attached GTP) dissociates from the beta and gamma subunits, and interacts with
intracellular effector molecules such as adenylate cyclase, which it activates.
Active transport (choice A) is the movement of molecules across a membrane from a region of low
concentration to a region of high concentration; it requires both a carrier protein and the expenditure
of energy (usually supplied by the hydrolysis of ATP by an ATPase).

20

Facilitated diffusion (choice B) is the movement of a molecule down its concentration gradient by
means of a protein carrier. This type of diffusion is suited for molecules with low permeability resulting
from inappropriate size or polarity.
Pinocytosis (choice C) is also known as fluid-phase endocytosis and refers to the uptake of
molecules that are in solution. Receptors are not involved.
Receptor-mediated endocytosis (choice D) refers to the engulfment of a molecule that binds to a
receptor on the surface of the cell, occurring commonly at the clathrin-coated pits on the plasma
membrane.
46. Which of the following is the action of dietary fiber?
A. Increases the blood cholesterol
B. Decreases the blood cholesterol
C. Decreases bowel motility
D. Increases the exposure of gut to carcinogens
The correct ansewer is choice B. Dietary fiber lowers blood cholesterol. Dietary fiber consists of
nondigestable carbohydrate, including cellulose, lignin, and pectin. Other actions include the
increasing of bowel motility, reduced exposure of the gut to carcinogens, softer stools, and
interference with mineral absorption.
47. Which of the following inhibits the activity of acetyl-CoA carboxylase?
A. Citrate
B. Glucagon
C. High-carbohydrate, low-fat diet
D. Insulin
The correct answer is choice B. The key thing to remember here is that acetyl-CoA carboxylase
catalyzes the first and rate-limiting step of fatty acid synthesis. If you got that far, you could have
figured out which of the choices would inhibit the synthesis of fatty acids. Certainly glucagon, a
catabolic hormone released in response to low blood glucose, would be a likely candidate to inhibit
the synthesis of fatty acids. In fact, glucagon inhibits fatty acid synthesis by a cAMP-dependent
phosphorylation of acetyl-CoA carboxylase. Conversely, glucagon stimulates fatty acid oxidation.
Citrate (choice A) is a key player in fatty acid synthesis (citrate shuttle). Therefore, the presence of
citrate would stimulate, not inhibit, acetyl-CoA carboxylase.
A high-carbohydrate, low-fat diet (choice C) would stimulate, not inhibit, the synthesis of fatty acids.
In contrast to glucagon, insulin (choice D) is an anabolic hormone that promotes fatty acid synthesis
and therefore would stimulate acetyl-CoA carboxylase. It does so by dephosphorylating the enzyme.
48. Addition of which of the following exhaustively 14C labeled substrates would lead to
evolution of 14CO2 from a cell-free suspension containing all the enzymes and substrates
required for the synthesis of uridylic acid?
A. Aspartate
B. Carbamoyl phosphate
C. Glutamine
D. Glycine
E. N10-Formyltetrahydrofolate

21

The correct answer is A. In the first step of pyrimidine synthesis, carbamoyl phosphate condenses
with aspartate to form carbamoyl aspartate, in a reaction catalyzed by aspartate transcarbamoylase.
In subsequent steps, ring closure occurs with the loss of water, followed by oxidation to yield orotic
acid. Addition of ribose-5-phosphate produces orotidylic acid, which is decarboxylated by orotidylate
decarboxylase to yield uridylic acid. The carbon dioxide that is evolved is derived from the alpha
carboxyl group of aspartate.
Carbamoyl phosphate (choice B) condenses with aspartate with the loss of inorganic phosphate to
produce carbamoyl aspartate. The carbamoyl moiety of carbamoyl phosphate is retained.
Glutamine (choice C), glycine (choice D) and N10-formyltetrahydrofolate (choice E) are all used in
purine synthesis. Glutamine also donates an amino group to UTP to form CTP, but this step occurs
after the synthesis of uridylic acid is complete.
49. The receptors for the hormone that causes blood glucose level to quickly drop are located
on which cellular component in target cells?
A. Mitochondrial matrix
B. Nuclear matrix
C. Nuclear membrane
D. Plasma membrane
E. Smooth endoplasmic reticulum
The correct answer is E. The hormone in question is insulin, which helps the body utilize glucose in
the bloodstream. Circulating insulin binds to a receptor on the plasma membrane of target cells. The
insulin receptor is a transmembrane protein that stimulates cytosolic tyrosine kinase activity when
activated. One of the important proteins activated by the tyrosine kinase is an insulin receptor
substrate, which in turn activates the wide variety of enzymes and transport molecules that produce
the actual metabolic changes in sugar, protein, and fat metabolism seen in the presence of insulin.
The mitochondrial matrix (choice A) is not a significant receptor site for hormones.
The nuclear matrix (choice B), sometimes together with cytoplasm, contains receptors for steroid
hormones such as cortisol.
The nuclear membrane (choice C) is not a significant receptor site for hormones.
50. Megaloblastic anemia, due to folate deficiency folate deficiency from dietary causes, is
linked to which dietary problem?
A. Lack of leafy green vegetables
B. Lack of milk products
C. Lack of red meat
D. Lack of yellow vegetables
E. Overcooked food
The correct answer is E. Folates (pteroylglutamic acid and related compounds) are widely
distributed in foodstuffs. Folic acid serves as an important mediator of one-carbon transfers Dietary
deficiency is usually due to overcooked (folates are very labile) and old (folates rapidly decay with
time) food. Folic acid deficiency is a macrocytic anemia that reveals macro-ovalocytes and
hypersegmented neutrophils on the peripheral blood smear. Reduced folate levels can be seen in red
blood cells or serum.

22

51. Absence of which of the following enzymes would impair the rate-limiting step of
glycogenolysis?
A. -1,4-Glucan transferase
B. Glycogen phosphorylase
C. Glycogen synthase
D. Phosphoglucomutase
E. UDP-glucose pyrophosphorylase
The correct answer is choice B. A key to excelling in biochemistry on the NBDE part 1 is to master
the most clinically important elements of metabolic pathways, e.g., rate-limiting steps, irreversible
steps, and steps involving enzymes affected by genetic diseases. In this case, glycogen
phosphorylase is the enzyme involved in the rate-limiting step of glycogenolysis:
Glycogen phosphorylase: (Glucose)n + Pi (Glucose)n1 + Glucose-1-P
Note that this enzyme is activated in response to the binding of glucagon to liver cell receptors or the
binding of epinephrine to muscle receptors, via signal transduction.
-1,4-Glucan transferase(choice A) is a debranching enzyme that removes three of four glucose
units from a branch point and transfers them to the end of another chain. In this reaction, one -1,4
bond is cleaved and another is formed. The elongated chain then becomes a substrate for glycogen
phosphorylase.
Glycogen synthase (choice C) is the enzyme involved in the rate-limiting step of glycogen synthesis:
Glycogen synthase: (Glucose)n + UDP-glucose (Glucose) n+1 + UDP
Note that this enzyme is inactivated in response to the binding of glucagon to liver cell receptors or
the binding of epinephrine to muscle receptors.
Phosphoglucomutase (choice D) is a key enzyme in glycogen synthesis that reversibly converts
glucose-6-phosphate to glucose-1-phosphate.
UDP-glucose pyrophophorylase (choice E) is another key enzyme in glycogen synthesis:
UDP-glucose pyrophosphorylase: Glu-1-P + Uridine-P-P-P

UDP-Glu + PPi

Note that the linkage formed between UDP and glucose is a high-energy bond that can provide
energy to many biosynthetic reactions.
52. Which of the following will be unchanged in a Lineweaver-Burk plot of an enzyme with and
without a competitive inhibitor?
A. Km
B. Slope
C. x-Intercept
D. y-Intercept
The correct answer is D.It is worth taking the time to learn how to read a Lineweaver-Burk plot.
Lineweaver-Burk plots are used to determine the Vmax and Km of an enzyme; they are also used to
differentiate between competitive and noncompetitive inhibition.
Note that in a Lineweaver-Burk plot, the slope is Km/Vmax, the x-intercept is -1Km, and the yintercept is 1/Vmax. In the presence of a competitive inhibitor, the Km (choice A), and therefore the

23

slope (choice B), is increased. Similarly, if Km is increased, -1/Km will become less negative and the
x-intercept (choice C) will shift to the right. Intuitively, this makes sense since a competitive inhibitor
will increase the amount of substrate needed to reach half-maximal velocity (definition of Km). In
contrast, the Vmax, and hence the y-intercept, is unchanged.
53. Which of the following structures is common to all sphingolipids?
A. Carnitine
B. Ceramide
C. Diacylglycerol
D. Sphingomyelin
E. Squalene
The correct answer is choice B. Sphingolipids are a class of lipids that are structural components of
membranes. Specifically, sphingolipids are any lipid containing a long chain base like that of
sphingosine (e.g., ceramides, cerebrosides, gangliosides). They are a common constituent of nerve
tissue. Ceramide is a component of sphingolipids. Ceramide is composed of sphingosine, a longchain amino alcohol with a saturated fatty acid linked to the amino group. Sphingolipids can be
differentiated on the basis of the "X" group that is esterified to the terminal hydroxyl group of
ceramide.
Carnitine (choice A) is involved in the oxidation of fatty acids. Carnitine is important in transferring
fatty acids from the cytoplasm into the mitochondria (the carnitine shuttle).
Diacylglycerol (choice C) is the alcohol common to all phospholipids. The second alcohol (e.g.,
choline, ethanolamine, serine) contributes the polar head that distinguishes the different classes of
phospholipids. Like sphingolipids, phospholipids are found in membranes.
Sphingomyelin (choice D) is a sphingolipid with phosphocholine as its "X" group. It is a component of
the myelin sheath.
Squalene (choice E) is a 30-carbon intermediate in the synthesis of cholesterol.
54. Which of the following structures is common to all sphingolipids?
A. Carnitine
B. Ceramide
C. Diacylglycerol
D. Sphingomyelin
E. Squalene
The correct answer is choice B. Sphingolipids are a class of lipids that are structural components of
membranes. Ceramide is a component of sphingolipids. Ceramide is composed of sphingosine, a
long-chain amino alcohol with a saturated fatty acid linked to the amino group. Sphingolipids can be
differentiated on the basis of the "X" group that is esterified to the terminal hydroxyl group of
ceramide.
Carnitine (choice A) is involved in the oxidation of fatty acids. Carnitine is important in transferring
fatty acids from the cytoplasm into the mitochondria (the carnitine shuttle).
Diacylglycerol (choice C) is the alcohol common to all phospholipids. The second alcohol (e.g.,
choline, ethanolamine, serine) contributes the polar head that distinguishes the different classes of
phospholipids. Like sphingolipids, phospholipids are found in membranes.

24

Sphingomyelin (choice D) is a sphingolipid with phosphocholine as its "X" group. It is a component of


the myelin sheath.
Squalene (choice E) is a 30-carbon intermediate in the synthesis of cholesterol.
55. A young child is well for the first 2 years of his life, and receives all his immunizations
without any complications. Starting around his 2nd birthday he contracts frequent upper
respiratory tract infections, and is hospitalized three times for pneumonia. Laboratory testing
would most likely reveal a deficiency of which of the following immunoglobulins in this child?
A. IgA
B. IgD
C. IgG
D. IgM
The correct answer is A. Selective IgA deficiency (<5 mg/dL) is the most common of all the primary
immunodeficiency diseases. The incidence reported in the US has ranged from 1:250 to 1:1000. IgA
has two subclasses, IgA1 and IgA2. IgA1 predominates in the serum, while IgA2 predominates in
mucosal secretions as a dimer bound together by a J chain with a secretory piece attached. Recurrent
bacterial and viral infections of the respiratory tract can be attributed to a lack of secretory IgA (sIgA),
the predominant immunoglobulin of the mucosal immune system. The most common types of
infection include sinusitis, otitis and bronchitis. Atopic disease and autoimmune disorders can be
associated with IgA deficiency.
IgD (choice B) has not been given any particular function other than to act as a receptor on the B cell.
It can be found in very low levels in serum.
IgG (choice C) is the major immunoglobulin found in the humoral immune response. A patient with a
low IgG will experience pyogenic infections.
IgM (choice D) is found in the early response to an antigen. If the patient was deficient in IgM he
would have also been characteristically low in IgG and would have experienced recurrent pyogenic
infection, usually commencing by the age of 5-6 months.
56. In the citric acid cycle, succinate thiokinase (succinyl-CoA synthetase) catalyzes the
cleavage of the succinyl-CoA thioester bond with formation of a high-energy compound. This
compound can then be used by the body in which of the following biochemical pathways?
A. Cysteine degradation
B. Elongation of the polypeptide chain
C. Epinephrine synthesis from tyrosine
D. Isopentyl pyrophosphate synthesis
E. Oxidative phosphorylation
The correct answer is choice B. In this question, you need to know that GTP is synthesized when
CoASH is cleaved from succinyl-CoA to form succinate in the citric acid cycle. You also need to know
that GTP, rather than ATP, is used as the energy source in protein synthesis, specifically in the
formation of the activated elongation factor to which tRNA binds, and in the transfer of the elongating
chain from the P to the A site in the ribosome.
Cysteine degradation (choice A) requires O2 and produces sulfite and then sulfate, which is secreted
in urine.
Epinephrine synthesis from tyrosine (choice C) requires tetrahydrobiopterin, pyridoxal phosphate,
oxygen, copper, and S-adenosylmethionine.

25

Isopentyl pyrophosphate synthesis from HMG CoA (choice D) requires NADPH and ATP, and
releases CO2.
Oxidative phosphorylation (choice E) used NADH, FADH2, coenzyme Q, oxygen, and a variety of
cytochromes to produce ATP.
57. A 25-year-old woman with sickle cell anemia is diagnosed with gallstones. Of which of the
following compounds are these stones most likely composed?
A. Calcium bilirubinate
B. Calcium oxalate
C. Cholesterol
D. Cholesterol and calcium bilirubinate
E. Cystine
The correct answer is A. Bilirubin is a degradative product of hemoglobin metabolism. Bilirubin
(pigment) stones are specifically associated with excessive bilirubin production in hemolytic anemias,
including sickle cell anemia. Bilirubin stones can also be seen in hepatic cirrhosis and liver fluke
infestation.
Calcium oxalate stones (choice B) and cystine stones (choice E) are found in the kidney, rather than
the gallbladder.
Pure cholesterol stones (choice C) are less common than mixed gallstones, but have the same risk
factors, including obesity and multiple pregnancies.
Mixed stones (choice D) are the common "garden variety" gallstones, found especially in obese,
middle aged patients, with a female predominance.
58. A newborn with severe acidosis and vomiting has elevated serum levels of lactate and
alanine. This suggests a deficiency in which enzyme?
A. Alanine aminotransferase
B. Glutamate dehydrogenase
C. Lactate dehydrogenase
D. Pyruvate carboxylase
E. Pyruvate dehydrogenase
The correct answer is E. Pyruvate dehydrogenase (PDH) catalyzes the irreversible conversion of
pyruvate to acetyl-CoA. Thiamine pyrophosphate (TPP), lipomide and FAD serve as catalytic
cofactors in addition to CoA and NAD+, the stoichiometric cofactors in this reaction. If PDH is absent,
pyruvate will be used in other pathways instead. Pyruvate will be converted to alanine via alanine
aminotransferase (choice A) and to lactate via lactate dehydrogenase (choice C).
Glutamate dehydrogenase (choice B) is involved in oxidative deamination, releasing ammonium ion
for urea synthesis. Deficiency of this enzyme would not cause the symptoms described.
Pyruvate carboxylase (choice D) is a gluconeogenic enzyme that catalyzes the conversion of
pyruvate to oxaloacetate. Deficiency of this enzyme would not cause the symptoms described.

26

59. Observation of a hematoxylin and eosin-stained microscope slide reveals that the nuclei
are blue. What is the basis for this observation?
A. Eosin binds to carbohydrates
B. Eosin binds to lipids
C. Eosin binds to nucleic acids
D. Hematoxylin binds to lipids
E. Hematoxylin binds to nucleic acids
The correct answer is E. Blue hematoxylin binds to polyanions such as RNA and DNA. Nuclei
contain large amounts of DNA and RNA, and they are consequently almost always blue. The nuclei of
dysplastic and cancerous cells are often enlarged and hyperchromatic (e.g., darker blue) compared to
normal cells of similar cell lines, because these altered cells often have extra DNA (are aneuploid)
and/or RNA (are metabolically active).
Pink eosin binds relatively nonselectively to cellular components, particularly proteins. Cytoplasm of
different cell lines can be pink, purple, or blue, depending principally on the number of ribosomes in
the cytoplasm. Consequently, blue-tinged cytoplasm tends to suggest high synthetic activity (e.g.,
abundant ribosomes).
60. In achondroplasia, mutation results in overexpression of FGFR3 and excess inhibition of
bone growth, particularly in long bones. Sons and daughters are equally affected. What is the
mode of inheritance in achondroplasia?
A. Autosomal dominant
B. Autosomal recessive
C. X-linked dominant
D. X-linked recessive
The correct answer is A. Achondroplasia is an autosomal-dominant disease, typically seen in
multiple generations of a pedigree. Men and women are affected in roughly equal frequencies.
Autosomal recessive diseases (choice B) are clinically seen only in the homozygous individual.
Skipped generations are typically seen. Both men and women are affected.
An x-linked dominant disease phenotype (choice C) is seen in multiple generations. Both men and
women are affected.
X-linked recessive diseases (choice D) are seen much more commonly in men than women. Skipped
generations are commonly seen. Male-to-male transmission is not seen in X-linked inheritance.
61. Which of the following vitamins has antioxidant properties, reduces the incidence of heart
attacks and deficiency of which can lead to hemolysis and neurological problems ?
A. Vitamin A
B. Vitamin D
C. Vitamin E
D. Vitamin K
The correct answer is choice C. Vitamin E acts as antioxidant, and may prevent oxidation of LDL
(low density lipoproteins). It protects membrane lipids from peroxidation. Oxidized LDL is thought to
promote heart disease.

27

Choice A - Vitamin A is antioxidant, maintains vision and growth regulation. Consumption of betacarotene in the diet decreases the incidence of lung and skin cancer. Vitamin A deficiency results in
night blindness and follicular hyperkeratosis.
Choice B - In response to hypocalcemia, vitamin D helps normalize serum calcium level. Deficiency
results in Rickets and osteomalacia.
Choice D - Vitamin K exposes calcium binding sites on several calcium dependent proteins.
Deficiency leads to prolonged bleeding and easy bruising with normal bleeding time.
62. Aerobic glycolysis is utilized for energy during intense aerobic exercise. In which of the
following forms will the carbons derived from glucose enter the citric acid cycle?
A. Acetyl-CoA
B. Citrate
C. Oxaloacetate
D. Pyruvate
E. Succinate
The correct answer is A. In aerobic glycolysis, glucose is degraded to pyruvate, which is then
converted to acetyl-CoA, the form in which it actually enters the citric acid cycle. Two acetyl-CoAs,
each containing two of the glucose's carbons, are produced from each glucose molecule. In addition,
a total of two carbons from glucose are released as CO2 when each of the two pyruvates is converted
to acetyl-CoA.
Citrate (choice B) is the product formed in the citric acid cycle when acetyl-CoA condenses with
oxaloacetate.
Oxaloacetate (choice C) is the citric acid cycle intermediate that condenses with acetyl-CoA to form
citrate.
Two pyruvates (choice D) are produced from degradation of glucose. These are then converted to
the two acetyl-CoAs that enter the citric acid cycle.
Succinate (choice E) is another citric acid cycle intermediate that forms when coenzyme A is
removed from succinyl-CoA.
63. A patient has an inherited disorder in which the administration of aspartame could be
detrimental to her health. This patient most likely has which of the following genetic
disorders?
A. Hyperornithinemia
B. Hyperuricemia
C. Hypervalinemia
D. Phenylketonuria
E. Wilson disease
The correct answer is D.The administration of any product that contains phenylalanine, such as
aspartame (an artificial sweetener), to an individual with any of the hyperphenylalaninemias could be
detrimental to his or her general health. The hyperphenylalaninemias result from an impaired
conversion of phenylalanine to tyrosine. The most common and clinically important of these is
phenylketonuria, which is characterized by an increased concentration of phenylalanine in blood,
increased concentration of phenylalanine and its by-products (such as phenylpyruvate, phenylacetate,
and phenyllactate) in urine, and mental retardation. Phenylketonuria is caused by a deficiency of
phenylalanine hydrolase.

28

Hyperornithinemia (choice A) is an inherited disorder of amino acid metabolism that results from a
defect of the enzyme ornithine decarboxylase. This condition is associated with mental retardation,
neuropsychiatric dysfunction, and protein intolerance.
Hyperuricemia (choice B) is a condition associated with higher than normal blood levels of uric acid.
Gout may be produced if the hyperuricemia persists.
Hypervalinemia (choice C) is a genetic disorder of amino acid metabolism that results from a defect
of the enzyme valine aminotransferase. This condition is associated with mental retardation,
neuropsychiatric dysfunction, and protein intolerance.
Wilson disease (choice E) is an autosomal recessive disorder associated with an abnormality of the
hepatic excretion of copper, resulting in toxic accumulations of the metal in the brain, liver, and other
organs.
64. What hormone is responsible for protein breakdown and increased gluconeogenesis?
A. Aldosterone
B. Cortisol
C. Estrogen
D. Testosterone
The Correct Answer is choice B. Cortisol increases gluconeogenesis and is responsible for protein
breakdown. It also has anti-inflammatory action.
Aldosterone (choice A) stimulates renal reabsorption of sodium and excretion of potassium.
Estrogen (choice C) controls the menstrual cycle and promotes the development of female secondary
sex characteristics.
Testosterone (choice D) stimulates spermatogenesis and promotes the development of male
secondary sex characteristics.
65. A complete blood count with differential for a pregnant woman reveals a hematocrit of
30%, with hypersegmented neutrophils and large, hypochromic red cells. Deficiency of which
of the following would be most likely to produce these findings?
A. Ascorbic acid
B. Calcium
C. Copper
D. Folate
E. Iron
The correct answer is D.The patient has a megaloblastic anemia, which can be due to deficiency of
folate or B12. Pregnancy increases the need for folate and other nutrients used by both baby and
mother, and may "unmask" a borderline dietary deficiency. For this reason, most obstetricians
recommend vitamin supplements for pregnant women. Folic acid deficiency is a macrocyte anemia.
Macro-ovalocytes and hypersegmented neutrophils are revealed on peripheral blood smear. Folic
acid is the common term for pteroylmonoglutamic acid.
Ascorbic acid (choice A) is vitamin C, and its deficiency predisposes for capillary fragility and oral
lesions.
Calcium deficiency (choice B) predisposes for osteoporosis/osteopenia.
Copper deficiency (choice C) is rare; when it occurs, it may cause a hypochromic anemia,
neutropenia, osteoporosis, or hypotonia.

29

Iron deficiency (choice E) causes a microcytic, hypochromic anemia, with reduced mental and
physical performance.
66. At which of the following sites is the characteristic triple helical structure of the collagen
initially formed?
A. Extracellular space
B. Golgi body
C. Nucleus
D. Rough endoplasmic reticulum
E. Smooth endoplasmic reticulum
The Correct Answer is choice B. Collagen formation begins with transcription of mRNA from
appropriate DNA genes in the nucleus. While still within the nucleus, the mRNA is spliced. It is then
transported through the cytoplasm to the ribosomes on the rough endoplasmic reticulum. Individual
chains are translated on the ribosomes, with the ends feeding into the endoplasmic reticulum lumen.
Within the lumen, glycosylation of the individual chains occurs. The material then moves toward the
Golgi bodies (whose lumens are connected to the endoplasmic reticulum) where the triple helices of
procollagen form. The procollagen is then secreted into the extracellular space, where cleavage of
pro-peptides and cross- linking of different triple helices occurs, maturing the collagen.
The extracellular space (choice A) is the site of procollagen cleavage and cross- linking.
The nucleus (choice C) is the site of mRNA transcription and splicing.
The rough endoplasmic reticulum (choice D) is the site of chain translation and glycosylation.
The smooth endoplasmic reticulum (choice E) does not participate in collagen synthesis.
67. The biochemical basis of a genetic disease is an inability to add the recognition marker
mannose phosphate to enzymes. In which of the following organelles does this step usually
occur?
A. Endoplasmic reticulum
B. Golgi apparatus
C. Lysosome
D. Mitochondria
E. Ribosome
The Correct answer is choice B. The disease is I cell disease, which is a rare genetic disease that
may receive disproportionate attention both because it is a mucolipidosis (a form of generalized
lysosomal disorder) with accumulation of abnormal chemical material in lysosomes, and because the
biochemical basis illustrates an interesting mechanism. Specifically, the Golgi apparatus in cells of
these patients has an abnormal N-acetyl-glucosaminotransferase (N- acetylglucosamine-1phosphotransferase), and is not able to add the necessary recognition marker mannose phosphate to
enzymes usually destined to enter lysosomes. Physiologically, the Golgi apparatus is a cellular
organelle consisting of a series of membranous plates that give rise to lysosomes and secretory
vesicles. A complete deficiency of this enzyme (type I form of I cell disease) causes death early in life.
Endoplasmic reticulum (choice A) receives the growing peptide chain of the enzymes, but is not the
site of addition of recognition markers.
Lysosomes (choice C) are the normal destination for the enzymes with the mannose phosphate
marker, but do not receive them if the marker is not present.

30

Mitochondria (choice D) supply the ATP to drive these chemical reactions.


Ribosomes (choice E) are the site of synthesis of the amino acid chains of proteins, but not the site of
addition of the destination marker.
68. A patient has been on a self-imposed "starvation diet" for four months, and has lost 60
pounds while consuming only water and vitamin pills. If extensive blood studies were
performed, which of the following would be expected to be elevated?
A. Acetoacetic acid
B. Alanine
C. Bicarbonate
D. Chylomicrons
E. Glucose
The correct answer is A. Long-term starvation induces many biochemical changes. Much of the
body's energy requirements are normally supplied by serum glucose, but in starvation are supplied by
both glucose and lipid-derived ketone bodies, including acetoacetic acid and beta-hydroxybutyric acid.
Glucose cannot be synthesized from lipids, and is instead made from amino acids such as alanine in
the process of gluconeogenesis. Ketosis as seen in diabetic patients would also be expected.
Serum alanine (choice B) drops dramatically in starvation, due to its conversion to glucose.
Bicarbonate (choice C) levels drop as the bicarbonate buffers the hydrogen ions produced by the
ketone bodies.
Chylomicrons (choice D) are the lipid form seen after absorption of dietary fat, and would drop
because the person is not feeding.
Glucose (choice E) is maintained in the blood at a much lower than normal level during starvation.
69. A recent immigrant complains of fatigue, weight gain, constipation, and cold intolerance.
Physical examination demonstrates a diffuse mass in the anterior aspect of the neck. Dietary
deficiency of which of the following nutrients is most likely to have contributed to the patient's
problem?
A. Copper
B. Iodine
C. Iron
D. Selenium
E. Zinc
The correct answer is choice B. Endemic goiter, such as in this patient, is due to dietary iodine
deficiency. This disorder is common world-wide in mountainous areas (where fish are not available),
although the use of iodized salt in the United States has limited its prevalence here. Frank symptoms
of hypothyroidism may or may not be present, possibly because of the increased synthesis of the
more potent triiodothyronine (T3) at the expense of thyroxine (T4). Goiters may become multinodular
and grow to great size. Impaired cognition and hearing may be severe in congenital hypothyroidism.
Copper deficiency (choice A) can cause anemia, neutropenia, hypotonia, psychomotor retardation,
osteoporosis, depigmentation of hair, and glucose intolerance.
Iron deficiency (choice C) can cause anemia, cognitive dysfunction, impaired immunity, impaired
thermoregulation, and reduced levels of physical activity. Iron deficiency most commonly occurs
during pathological bleeding, such as an ulcer.

31

Selenium deficiency (choice D) can cause congestive cardiomyopathy and skeletal muscle
degeneration.
Zinc deficiency (choice E) causes rash, growth retardation, and impairments of immunity, wound
healing, mentation, sexual function, and night vision. Zinc deficiency most commonly occurs in the
elderly.
70. Histones can bind DNA tightly because they have a high isoelectric point. This is due to an
enrichment in the amino acid
A. aspartate
B. glycine
C. lysine
D. proline
E. tyrosine
The correct answer is C. Histones are enriched in the amino acids lysine and arginine. They are
positively charged and bind tightly to DNA, which is negatively charged from phosphate groups.
Proline (choice D) and glycine (choice B) are neutral, aliphatic amino acids. Aspartate (choice A) is
negatively charged. Tyrosine (choice E) is a neutral, aromatic amino acid.
71. Which of the following amino acids is post-translationally hydroxylated in the cytoplasm of
fibroblasts?
A. Cysteine
B. Glycine
C. Proline
D. Serine
E. Tyrosine
The correct answer is C. Hydroxylation of proline in fibroblasts generates the modified amino acid
hydroxyproline. This is an example of post-translational modification. Hydroxyproline is involved in
stabilizing the three-dimensional triple helix structure of collagen. Proline also has an alphatic side
chain but differs from other amino acids in that its side chain is bonded to both the nitrogen and
carbon atoms. Note: Proline contains a secondary rather than a primary amino group, which
specifically makes it an imino acid.
Cysteine (choice A) is unique in its ability to form a covalent disulfide bond with another cysteine
residue elsewhere in the protein molecule, thereby forming a cysteine bond. Such strong disulfide
bonds stabilize the three-dimensional structure of the protein.
Glycine (choice B) is abundant in fibroblasts since it constitutes every third amino acid in the primary
sequence of collagen. However, glycine is not hydroxylated.
Serine (choice D), tyrosine (choice E), and threonine can all be phosphorylated post-translationally
to form phosphoserine, phosphotyrosine, and phosphothreonine, respectively. These phosphorylated
amino acids are believed to play a role in signal transduction.

32

72. Which of the following will be unchanged in a Lineweaver-Burk plot of an enzyme with and
without a competitive inhibitor?
A. Km
B. Slope
C. x-intercept
D. y-intercept
The correct answer is D.It is worth taking the time to learn how to read a Lineweaver-Burk plot.
Lineweaver-Burk plots are used to determine the Vmax and Km of an enzyme; they are also used to
differentiate between competitive and noncompetitive inhibition.
Note that in a Lineweaver-Burk plot, the slope is Km/Vmax, the x-intercept is -1/Km, and the y-intercept
is 1/Vmax. In the presence of a competitive inhibitor, the Km (choice A) and therefore the slope (choice
B) are both increased. Similarly, if Km is increased, -1/Km will become less negative and the xintercept will shift to the right. Intuitively, this makes sense since a competitive inhibitor will increase
the amount of substrate needed to reach half-maximal velocity (definition of Km). In contrast, the Vmax,
and hence the y-intercept, is unchanged (choice D). It is important to note that it is convenient to
transform the Michaelis-Menton equation into one that gives a straight line plot, which can be done by
taking the reciprocal of both sides to
1/V = 1/Vmax + Km/Vmax + 1/[s]
A plot of 1/V versus 1/[s] is called the Lineweaver-Burk plot.
73. A patient has Lesch-Nyhan syndrome caused by an absence of hypoxanthine-guanine
phosphoribosyl transferase. The patient's condition is caused by an enzyme deficiency in
which of the following biochemical pathways?
A. Ganglioside metabolism
B. Monosaccharide
C. Purine metabolism
D. Pyrimidine metabolism
E. Tyrosine metabolism
The correct answer is C. The patient has a classic case of Lesch-Nyhan syndrome, an X-linked
recessive disorder caused by severe deficiency of the purine salvage enzyme hypoxanthine-guanine
phosphoribosyl transferase (HPRT). This defect is associated with excessive de novo purine
synthesis, hyperuricemia. The clinical and symptoms include spasticity, writhing movements,
compulsive biting of fingers and lips, and head-banging. At puberty, the child often develops arthritis
and death from renal failure occurs at age 25 years. The biochemical basis of the often striking selfmutilatory behavior (which may require restraints and even tooth extraction) has never been
established. Treatment with allopurinol inhibits xanthine oxidase and reduces gouty arthritis, urate
stone formation, and urate nephropathy. It does not, however, modify the neurologic/psychiatric
presentation. Mental deficiency, spasticity, and self-mutilation are the most common characteristics of
this condition.
74. Patients with gout will have localized concentrations of needle shaped, negatively
birefringent crystals of which substance?
A. Bile pigments
B. Calcium pyrophosphate
C. Cystine
D. Monosodium urate
E. Struvite

33

The correct answer is D.The patient has gout, which is due to precipitation of monosodium urate
crystals in joint spaces (notably the great toe) and soft tissues (causing tophi, which are often found
on the external ears). Gout is often an acute condition that is typically nocturnal and usually
monarticular, often involving the first metatarsophalangeal joint. Hyperuricemia occurs in most cases.
Identification of urate crystals in joint fluid or tophi is diagnostic.
Bile pigments (choice A) are found in some gallstones.
Calcium pyrophosphate (choice B) crystals are deposited in pseudogout, which classically affects the
knee or other large joints.
Cystine (choice C) and struvite (choice E) can form kidney stones.
75. A young woman with anorexia nervosa consumes about 125g carbohydrate, 15g protein
and 10 g fat daily. Her daily caloric intake is roughly equal to which of the following values?
A. 450 kcal/day
B. 650 kcal/day
C. 850 kcal/day
D. 1050 kcal/day
E. 1250 kcal/day
The correct answer is choice B. You may be asked to make this type of calculation on the NBDE
part 1. You should know that 1 g of either protein or carbohyrate produces about 4 kcal = 4 Calories
(kcal) of energy, and 1 g of fat produces 9 kcal = 9 Calories of energy. The calculation is then
straightforward. The Calories from carbohydrates are 125 X 4 = 500; from protein are 15 X 4 = 60;
and from fat are 10 X 9 = 90. The total is 500 + 60 + 90 = 650 kcal/day.
76. The activity of which of the following enzymes is directly affected by citrate?
A. Fructose-2,6-bisphosphatase
B. Isocitrate dehydrogenase
C. Phosphofructokinase I
D. Pyruvate carboxylase
E. 6-phosphogluconate dehydrogenase
The correct answer is C. Citrate is produced by citrate synthase from acetyl CoA and oxaloacetate.
Note that this is the first step in the citric acid cycle. This reaction takes place in the mitochondria, but
citrate can move freely from the mitochondria into the cytosol. When the citric acid cycle slows down,
citrate accumulates. In the cytosol, it acts as a negative allosteric regulator of phosphofructokinase I,
the enzyme that catalyzes the committed step of glycolysis.
Fructose-2,6-bisphosphatase (choice A) breaks down fructose-2,6-bisphosphate, a potent allosteric
activator of phosphofructokinase I. Fructose-2,6-bisphosphatase is activated by cyclic AMPdependent protein kinase.
Isocitrate dehydrogenase (choice B) converts isocitrate to alpha-ketoglutarate in the citric acid cycle.
It is allosterically stimulated by ADP and inhibited by ATP and NADH. This reaction produces NADH
and CO2.
Pyruvate carboxylase (choice D) is a mitochondrial enzyme that converts pyruvate to oxaloacetate. It
is important in gluconeogenesis and replenishes the oxaloacetate in the citric acid cycle.
6-phosphogluconate dehydrogenase (choice E) converts 6-phosphogluconate to ribulose 5phosphate in the pentose phosphate shunt pathway.

34

77. Which of the following enzymes is responsible for maintaining blood glucose levels by
releasing glucose from its storage form in the liver?
A. Acetyl-CoA carboxylase
B. Glucose 6-phosphate dehydrogenase
C. Glycogen phosphorylase
D. Glycogen synthase
E. Thiolase
The correct answer is C. Glycogen is the storage form of glucose and a readily mobilizable fuel
store. When individuals do not consume adequate quantities of carbohydrates, the body responds by
breaking down glycogen stores in the liver to maintain normal blood glucose levels. The two enzymes
involved in the degradation and synthesis of glycogen are glycogen phosphorylase (choice C) and
glycogen synthase (choice D), respectively. The processes of glycogen synthesis and degradation
are coordinated by a hormone-triggered cascade that ensures that when one enzyme is active, the
other enzyme is inactive.
Acetyl-CoA carboxylase (choice A) is the key enzyme involved in fatty acid synthesis.
Glucose 6-phosphate dehydrogenase (choice B) is involved in the pentose phosphate pathway.
Thiolase (choice E) converts acetoacetyl-CoA into acetyl-CoA.
78. A cell biologist wants to examine the microstruture of an integral membrane protein. She
solubilizes the protein by destabilizing its association with the membrane lipid bilayer. Which
of the following techniques did she most likely use?
A. Alterations in pH
B. Detachment of protein phenyl groups
C. Dissociation of phospholipid polar head groups
D. Increase in ionic strength
E. Interruption of hydrophobic interactions
The correct answer is E. This question is asking you what forces are responsible for retaining
integral membrane proteins within the lipid bilayer, i.e., hydrophobic interactions. Amphipathic agents,
such as detergents, are used to solubilize integral membrane proteins; they do so by disrupting
hydrophobic interactions between integral membrane proteins and other membrane constituents,
such as phospholipids.
Choices A, B, C, and D would destabilize the association of peripheral membrane proteins with the
membrane lipid bilayer.
79. Supplementation of calcium with which vitamin would reduce the risk for fracture caused
by bone loss, particularly in elderly patients?
A. Calcitonin
B. Cholecalciferol
C. Carotene
D. Alpha tocopherol
The correct answer is choice B. Most of the calcium in the body is found as calcium phosphate
crystals in the bones and teeth that contribute to the physical strength of these structures. Vitamin D
(cholecalciferol) increases intestinal absorption of calcium and phosphate.

35

Choice A - Calcitonin is secreted by specialized parafollicular cells of the thyroid gland. It decreases
the resorption of bone, reduces serum calcium, and improves bone architecture.
Choice C - Vitamin A (carotene) is converted to several active forms in the body and is associated
with important functions, including maintenance of healthy epithelium and vision.
Choice D - Vitamin E (alpha tocopherol) prevents peroxidation of fatty acids in cell membranes,
helping to maintain their normal fluidity.
80. Children with dry and wet beriberi typically have a deficiency of which vitamin?
A. Ascorbic acid
B. Retinol
C. Riboflavin
D. Thiamine
E. Vitamin K
The correct answer is D.Thiamine deficiency is most frequently encountered in alcoholics and in
developing countries. Deficiency of this vitamin can take several forms: dilated cardiomyopathy (wet
beriberi), polyneuropathy (dry beriberi), and mamillary body degeneration (Wernicke-Korsakoff
syndrome).
Ascorbic acid (choice A, Vitamin C) deficiency causes scurvy, associated with capillary fragility, bony
abnormalities, and poor wound healing.
Retinol (choice B, Vitamin A) deficiency causes blindness and impaired immune responses.
Riboflavin (choice C) deficiency causes cheilosis, glossitis, and dermatitis.
Vitamin K (choice E) deficiency causes impaired blood clotting because of decreased production of
factors II, VII, IX, and X.
81. Transcription of a prokaryotic gene by RNA polymerase yields an mRNA corresponding to
a single polypeptide chain containing 64 amino acids. The mRNA has a 5' untranslated region
of 120 nucleotides and a 3' untranslated region of 240 nucleotides. Approximately how many
nucleotides are in the coding region of the mRNA?
A. 64
B. 128
C. 192
D. 424
E. 552
The correct answer is C. Regardless of how long the untranslated regions are, the number of
nucleotides in the coding region of an mRNA is three times the number of amino acids, since three
nucleotides are required to code for each amino acid, and 3 X 64 = 192. In reality, three nucleotides
code for the first amino acid (formyl-methionine in prokaryotes, methionine in eukaryotes), which may
be removed in posttranslational steps, and three nucleotides at the 3' end (are needed to terminate
the process (i.e., a STOP codon), so the actual number would likely be slightly higher.

36

82. Which of the following enzymes does the neutrophil use to initiate the production of toxic oxygen
compounds that kill bacteria abscesses and in other bacterial infections?
A. Hydrogen peroxide
B. Myeloperoxidase
C. NADPH oxidase
D. Peroxidase
E. Superoxide dismutase
The correct answer is C. The first step in killing bacteria is the production of superoxide ion, O2- , by
the action of NADPH oxidase on NADPH and O2. The superoxide is then converted to hydrogen
peroxide, either spontaneously or through the action of superoxide dismutase. The hydrogen peroxide
can also be converted to the toxic HOCl. radical by the action of myeloperoxidase.
Hydrogen peroxide(choice A) is not an enzyme but is used as an antiseptic for dental procedures and
for the treatment of apthous ulcers in the mouth.
Myeloperoxidase (choice B) is part of the killing pathway, but is active later in the sequence, when
H2O2
Peroxidase (choice D) is also a heme enzyme. It catalyzes the reaction in which hydrogen peroxide is
reduced to water.
Superoxide dismutase (choice E) is part of the killing pathway, but is active later in the sequence,
when superoxide is converted to hydrogen peroxide. This enzyme is present in all aerobic organisms.
83. Which of the following amino acids is post-translationally hydroxylated in the cytoplasm of
fibroblasts?
A. Cysteine
B. Glycine
C. Proline
D. Serine
E. Tyrosine
The correct answer is C. Hydroxylation of proline in fibroblasts generates the modified amino acid
hydroxyproline. This is an example of post-translational modification. Hydroxyproline is involved in
stabilizing the three-dimensional triple helix structure of collagen.
Cysteine (choice A) is unique in its ability to form a covalent disulfide bond with another cysteine
residue elsewhere in the protein molecule, thereby forming a cystine residue. Such strong disulfide
bonds stabilize the three-dimensional structure of the protein.
Glycine (choice B) is abundant in fibroblasts since it constitutes every third amino acid in the primary
sequence of collagen. However, glycine is not hydroxylated.
Serine (choice D), tyrosine (choice E), and threonine can all be phosphorylated post-translationally
to form phosphoserine, phosphotyrosine, and phosphothreonine, respectively. These phosphorylated
amino acids are believed to play a role in signal transduction.

37

84. During DNA replication, which of the following enzymes produces a short strand of RNA
complementary to the template DNA with a free 3'-OH end?
A. DNA ligase
B. Polymerase I
C. Polymerase III
D. Primase
E. Topoisomerase
The correct answer is D.The primer molecule required by DNA polymerase is a short strand of RNA
(4-10 bases) complementary to the template strand of the DNA molecule. The primer is synthesized
by a specific RNA polymerase known as primase. The growing end of the RNA primer is a free 3'-OH
group. The primer RNA is excised at a later stage of replication. The primase does not itself require a
primer for initiation of nucleotide synthesis.
DNA ligase (choice A) catalyzes the formation of a phosphodiester bond between the 3'-OH of one
fragment of DNA and the 5'-monophosphate group of an adjacent DNA fragment.
Polymerase I (choice B) catalyzes the polymerization of nucleotides and also functions in processing
and repair mechanisms.
Polymerase III (choice C) is a part of a multiprotein complex and is the major replicating enzyme in E.
coli.
Topoisomerases (choice E) produce swivel points in the DNA molecule that relieve the strain induced
by the replication fork. These enzymes cut and reseal the DNA.
85. Histones can bind DNA tightly because they have a high isoelectric point. This is due to an
enrichment in the amino acid
A. aspartate
B. glycine
C. lysine
D. proline
E. tyrosine
The correct answer is C. Histones are enriched with the amino acids lysine and arginine. They are
positively charged and bind tightly to DNA, which is negatively charged because of its phosphate
groups.
Proline (choice D) and glycine (choice B) are neutral, aliphatic amino acids. Aspartate (choice A) is
negatively charged. Tyrosine (choice E) is a neutral, aromatic amino acid.
86. Which of the following amino acids would most likely be found on the surface of a protein
molecule?
A. Alanine
B. Arginine
C. Isoleucine
D. Leucine
E. Phenylalanine

38

The correct answer is choice B. This question requires two logical steps: first, you need to
appreciate that the hydrophilic amino acids are more likely to appear on the surface of a protein
molecule, while hydrophobic amino acids are most likely be found in its interior. Next, you need to
figure out which of the amino acids listed is hydrophilic. If you recall that arginine is a basic amino acid
that is positively charged at physiologic pH, you should be able to answer this question right away. In
addition to arginine, lysine and histidine also have basic side chains. Structurally the planar outer part
of arginine's side chain consists of 3 nitrogens bonded to a carbon atom, which is called a guanidium
group.
All of the other choices have neutral side chains and are uncharged at physiologic pH. They would
most likely be found in the hydrophobic core of the protein structure. Alanine (choice A), isoleucine
(choice C), and leucine (choice D) all have aliphatic side chains; phenylalanine (choice E) has
aromatic side chains.
87. A 25-year-old man with known history of drug abuse walks into the clinic. A routine
screening test for HIV exposure is positive. To confirm HIV exposure, which one of these tests
is ideal?
A. ELISA
B. Southern blot
C. Northern blot
D. Western blot
The correct answer is D.Western blot is used as the confirmatory test for HIV exposure and is highly
specific. The combination of ELISA and Western blot has a positive predictive value greater than
99%. The Western blot primarily eliminates a small percentage of false positives found by the ELISA
test alone.
Choice A - ELISA is used as the primary screening assay because it is very sensitive.
Choice B - Southern blotting is a technique that can be used to detect mutations in DNA.
Choice C - Northern blots analyze RNA extracted from a tissue and are typically used to determine
which genes are being expressed.
88. A 30-year-old man has been fasting for several days. His blood glucose level is now about
60% of its normal value, but he does not feel lightheaded because his brain has reduced its
need for serum glucose by using which of the following substances as an alternate energy
source?
A. Apoprotein B
B. Beta-carotene
C. Beta-hydroxybutyrate
D. C-reactive protein
E. Coenzyme A
The correct answer is C. Ketone bodies, which include acetoacetate, beta-hydroxybutyrate, and
acetone, are produced by the liver in the fasting state by beta-oxidation of fatty acids. They are then
released into the blood stream, where they can be used as alternative energy sources for other
organs, such as muscle, kidney, and brain. The brain specifically still requires a small amount of
circulating glucose to function, but the amount required is reduced when ketone bodies are available.
Apoprotein B (choice A) is one of the proteins that hold lipoproteins together.
Beta-carotene (choice B) is a vitamin with antioxidant properties.
C-reactive protein (choice D) is a serum protein produced by the liver that rises during infections and
in inflammatory states.

39

Coenzyme A (choice E) is found in mitochondria and carries acetyl groups into the citric acid, or
tricarboxylic acid, cycle.
89. When alcohol is consumed, it is metabolized by the liver, with almost half the alcohol
being oxidized to acetaldehyde. In which of the following sites does the reaction occur?
A. Golgi bodies
B. Lysosomes
C. Mitochondrial matrix
D. Peroxisomes
E. Ribosomes
The correct answer is D.Peroxisomes are cell organelles that are present in only small numbers in
most mammalian cells. In the liver, however, these single membrane-bound organelles are present in
large numbers and are import in detoxification and long chain fatty acid metabolism. The clinically
important degradation of ethanol to (potentially toxic) acetaldehyde occurs in humans in both
peroxisomes and the smooth endoplasmic reticulum (P450 system). Acetaldehyde is then oxidized to
(non-toxic) acetate. This reaction, catalyzed by aldehyde dehydrogenase, occurs in the mitochondria.
Golgi bodies (choice A) are involved with packaging substances that are transported out of the cell.
Lysosomes (choice B) are sites of degradation of intracellular waste products.
The mitochondria (choice C) are involved with ATP production and contain both the electron transport
chain and the citric acid (Krebs) cycle.
Ribosomes (choice E) are the sites of protein synthesis.
90. In the CNS, the drug levodopa is metabolized to dopamine, which can be subsequently
converted by some neurons to which of the following substances?
A. Choline
B. Dihydroxyphenylalanine (DOPA)
C. Epinephrine
D. Norepinephrine (NE)
E. Tyrosine
The correct answer is D.Levodopa is used in the treatment of Parkinson disease, a condition caused
by dopamine deficiency in the CNS. When levodopa enters the CNS, it is metabolized to dopamine,
some of which is subsequently metabolized to norepinephrine (NE) in noradrenergic neurons. In other
words, dopamine is the immediate precursor to NE. The synthesis of NE begins in the axoplasm of
the terminal nerve endings of adrenergic fibers; however, its synthesis is completed inside the
vesicles of these fibers.
With respect to the endogenous syntheses of NE, the basic steps are as follows: tyrosine (choice E)
is converted to dihydroxyphenylalanine (DOPA) through the process of hydroxylation. Then, DOPA
(choice B) undergoes decarboxylation to the neurotransmitter dopamine. Dopamine is then
transported into the vesicles of the adrenergic fibers, where it undergoes hydroxylation to NE (choice
D). In the adrenal medulla, NE is transofrmed into epinephrine (choice C) through the process of
methylation.
Choline (choice A) is combined with acetyl-CoA to become acetylcholine.

40

91. The glycolytic degradation of ingested glucose commences with the action of which of the
following enzymes?
A. Aldolase
B. Hexokinase
C. Phosphofructokinase
D. Phosphoglucose isomerase
E. Pyruvate kinase
The correct answer is choice B. The process of glycolysis is defined as the sequence of reactions
that converts glucose into pyruvate with the concomitant production of ATP.
Glycolysis begins when glucose is converted by hexokinase to glucose-6-phosphate. When this
compound interacts with the enzyme phosphoglucose isomerase (choice D), fructose-6-phosphate is
formed. Fructose-6-phosphate is then converted by phosphofructokinase (choice C) to form fructose
1,6-biphosphate, which is subsequently converted to glyceraldehyde 3-phosphate by aldolase
(choice A). After a number of enzymatic reactions, phosphoenolpyruvate is formed.
Phosphoenolpyruvate is converted to pyruvate by pyruvate kinase (choice E), and the glycolytic
pathway is then completed.

41

Pathology
1. Which of the following is the MOST likely explanation for when two days after removal of a
cancerous neck lesion, there is thigh and calf cramps, tingling around the lips, low serum
calcium, and high serum phosphate?
A. Hyperparathyroidism
B. Primary hypoparathyroidism
C. Pseudohypoparathyroidism
D. Renal failure
E. Vitamin D deficiency
The correct answer is B. Surgically related hypoparathyroidism is the most common cause of
primary hypoparathyroidism. Exploration of the anterior neck during thyroidectomy,
parathyroidectomy, or removal of neck lesions can all compromise parathyroid gland function. Often
the problem occurs because the blood supply to the parathyroid glands is interrupted during the
surgery. The decrease in plasma parathyroid hormone leads to hypocalcemia and
hyperphosphatemia. The decreased serum calcium is caused by decreased absorption of dietary
calcium (because vitamin D activation is decreased) and decreased movement of calcium from bone
to extracellular fluid. Low serum calcium can lead to tetany and paresthesias because of
destabilization of excitable tissue membranes. Tetany is most often observed when there is a rapid
decrease in serum calcium, such as that occurring with surgical hypoparathyroidism. The increased
serum phosphate is caused by decreased renal excretion.
Hyperparathyroidism (choice A) is frequently asymptomatic; however, renal stones, polyurine,
hypertension, constipation, fatigue, and mental status changes may be seen. Serum and urine
calcium is elevated. Urine phosphate is high and a low to normal serum phosphate is generally seen.
Pseudohypoparathyroidism (choice C) is a rare genetic defect in which the target tissues are
insensitive to parathyroid hormone. Because parathyroid hormone is less effective, serum calcium
decreases and serum phosphate increases. Pseudohypoparathyroidism is also accompanied by
developmental defects, including mental retardation, short stature, and missing metatarsal or
metacarpal bones.
In renal failure (choice D), hyperphosphatemia occurs because of decreased renal excretion. This
can lead to hypocalcemia as the equilibrium between serum phosphate and serum calcium is pushed
toward hydroxyapatite. Furthermore, dietary absorption of calcium is decreased because vitamin D
activation is decreased by hyperphosphatemia, even before there is significant decrease in renal 1alpha-hydroxylase activity. The low serum calcium produces a compensatory increase in parathyroid
hormone with subsequent bone demineralization (renal osteodystrophy).
With vitamin D deficiency (choice E), serum calcium and phosphate are typically decreased. The
calcium is low because of decreased dietary absorption. The phosphate is low (not high) because of
decreased dietary absorption, and because the secondary increase in parathyroid hormone secretion
(caused by the hypocalcemia) increases renal excretion of phosphate.
2. At what level in the skin do bullae MOST likely develop in a localized cutaneous infection
around the mouth with phage group II Staphylococcus aureus?
A. Across the basal cells
B. Below the basement membrane
C. Between the basal cells and the basement membrane
D. High in the epidermis
E. Just above the basal cells

The correct answer is D. Bullous diseases of the skin are subdivided on the basis of the level at
which the cleavage for blister formation occurs. In general, the lower in the epidermis/dermis that the
cleavage plane occurs, the more dangerous and widespread the blistering. This is because blistering
at lower levels, particularly those involving the basal cell layer, permits loss of substantial amounts of
fluid and heals slowly (often with significant scarring). Scalded-skin syndrome (toxic epidermal
necrolysis) may follow staphylococcal (often phage group II) skin infection. This disorder fortunately
involves the very superficial squamous cells just beneath the granular layer. Consequently, the
disease (which typically produces bright red skin sloughing) usually resolves without sequelae after
antibiotic therapy. "Scalded-skin" may also be observed in association with drug-induced erythema
multiforme. Major inciting agents include phenylbutazones, sulfonamides, barbiturates,
aminopenicillins, oxicam, nonsteroidal antiinflammatory agents, and allopurinol. This form usually
affects the mucosa (eyes, mouth) first, and is much more dangerous because the blistering is
subepidermal, and the entire overlying epidermis becomes necrotic.
3. Which of the following cell types show abnormal function when there is a history in a young
adult of multiple arm and leg fractures following minor falls, with a slight weakness of facial
muscles on the left, mild anemia, and with generalized bony widening with partial obliteration
of marrow spaces?
A. Granulocytic stem cells
B. Megakaryocytes
C. Plasma cells
D. Osteoblasts
E. Osteoclasts
The correct answer is E. The disease described is osteopetrosis (Albers-Schonberg disease), which
is a group of hereditary diseases in which impaired osteoclast function leads to reduced bone
resorption. The abnormal osteoclasts frequently are enlarged, with bizarre shapes. The bones
become thick and brittle; other features include anemia secondary to marrow loss and cranial nerve
deficits secondary to narrowing of bony ostea. An autosomal recessive, severe form of the disease
produces death in childhood. A relatively benign, autosomal dominant form presents in adulthood.
Abnormal proliferation of granulocytic stem cells (choice A) can produce myelocytic leukemias.
Megakaryocyte abnormalities (choice B) can produce platelet disorders, but not deficient bone
resorption.
In multiple myeloma, neoplastic plasma cells (choice C) can cause lytic bone lesions characterized
by excessive resorption of bone.
Abnormally low osteoclast, rather than osteoblast (choice D), function is the problem in osteopetrosis.
4. What of the following is characterized by left lower quadrant periumbilical pain in an elderly
person, with the presence of fever, tender abdomen, leukocytosis, nausea, and vomiting?
A. Acute appendicitis
B. Diverticulitis
C. Gallstones
D. Pancreatitis
E. Pyelonephritis
The correct answer is B. Diverticulitis is a disease of the elderly and usually involves the distal
colon. In severe cases, however, the diverticula may extend throughout the colon and up to the
cecum. Inflammation of a cecal diverticulum can closely mimic acute appendicitis. The essentials of
diagnosis for diverticulitis are acute abdominal pain and fever, left lower abdominal tenderness, and
mass. Leukocytosis is commonly present together with nausea and vomiting.

Acute appendicitis (choice A) is usually a disease of young adults (and sometimes children). It is
rarely seen in the elderly.
Pancreatitis (choice D), pyelonephritis (choice E), and gall bladder disease (choice C), refer pain to
the mid back, lateral back, and right upper quadrant, respectively.
5. Which of the following genetic conditions is MOST likely to be the cause of a child born with
with a small head, small eyes, six fingers on each hand, and congenital heart defects?
A. Trisomy 13
B. Trisomy 18
C. Trisomy 21
D. XXY
E. XYY
The correct answer is A. This is a description of Patau's syndrome or trisomy 13. This disorder is
also associated with severe mental retardation, abnormal forebrain structures, and death within 1 year
of birth. Patau's syndrome has an incidence of 1:6,000 births, making it the second most common
form of autosomal trisomy.
Trisomy 18 (choice B) is Edwards syndrome, characterized by severe mental retardation, rocker
bottom feet (also sometimes seen in Patau's syndrome), low-set ears, micrognathia, clenched hands,
prominent occiput, and death within 1 year. Edwards syndrome has an incidence of 1:8,000, making it
the third most common autosomal trisomy.
Trisomy 21 (choice C) is Down syndrome, and is characterized by mental retardation, flat facial
profile, prominent epicanthal folds, simian crease, duodenal atresia, and congenital heart disease.
Down syndrome is the most common autosomal trisomy, with an incidence of 1:700.
XXY (choice D) is Klinefelter's syndrome and is associated with male hypogonadism and infertility,
eunuchoid body habitus, gynecomastia, and lack of male secondary sexual characteristics.
XYY (choice E) is double Y syndrome. Affected individuals often go undetected, but may be taller
than average and may be more likely to exhibit aggressive, antisocial behavior.
6. If a woman 22 weeks pregnant has ankle edema and proteinuria, the presence of which of
the following would determine if she has preeclampsia?
A. Diabetes mellitus
B. Hyperuricemia
C. Hypertension
D. Systemic lupus erythematosus
E. Thrombocytopenia
The correct answer is C. A pregnant patient is considered to be in preeclampsia if she develops
hypertension, proteinuria, and edema. The hypertension is defined as a sustained elevation of blood
pressure of 140 mm Hg systolic or 90 mm Hg diastolic or more in the absence of chronic hypertension
after 20 weeks' gestation. Eclampsia includes the addition of seizures to the triad. Approximately 7%
of pregnant women develop preeclampsia, typically between 20 weeks' gestation to 6 weeks
postpartum. Predisposing conditions include preexisting hypertension, diabetes (choice A), and
autoimmune diseases such as lupus (choice D). Laboratory features can include hyperuricemia
(choice B) and thrombocytopenia (choice E), but these are not used to define the presence of
preeclampsia.

7. Which of the following blood components can be expected to increase with disseminated
intravascular coagulation?
A. Factor V
B. Fibrin degradation products
C. Fibrinogen
D. Plasminogen
E. Platelets
The correct answer is B. Disseminated intravascular coagulation (DIC or consumptive
coagulopathy) represents pathologic activation of the coagulation system by another underlying
disease, with consequent consumption and depletion of the cellular and humoral components of the
coagulation cascade. The fibrinolytic mechanisms are also activated, and an uncontrolled cycle of
bleeding and clotting develops. The essentials of diagnosis include underlying serious illness,
hypofibrinogenemia, thrombocytopenia, fibrin degradation products, and prolonged prothrombin time.
As a consequence, levels of all clotting proteins (choices A and C) become depleted, platelet counts
drop (choice E), and the fibrinolytic proteins are depleted also (choice D). Fibrin degradation
products (choice B), which are normally low in the serum, increase markedly because of increased
fibrinolysis seen in this disease; identification of these proteins can be an important indicator of DIC.
8. Which of the following additional clinical findings would MOST be associated with a woman
who has oral mucosal swelling, xerostomia, and intense salivary gland destructive
inflammation, as well as antibodies against ribonucleoprotein?
A. Conjunctivitis
B. Goiter
C. Hemolytic anemia
D. Proximal muscle weakness
The correct answer is A. This describes Sjogren's syndrome, an autoimmune disease characterized
by dry eyes (keratoconjunctivitis) and a dry mouth (xerostomia) caused by destruction of the lacrimal
and salivary glands. Sjogren's syndrome is also characterized by autoantibody production. The most
diagnostic autoantibodies are those against ribonucleoproteins.
Goiters (choice B) are not typical of Sjogren's syndrome. Although autoimmune thyroiditis is
associated with Sjogren's syndrome, ocular involvement is much more characteristic than thyroid
involvement.
Hemolytic anemia (choice C) is not characteristic of Sjogren's syndrome. Primary autoantibodies,
drugs, and systemic lupus erythematosus may be associated with hemolytic anemia.
Proximal muscle weakness (choice D), in association with autoantibodies, is expected in polymyositis
or dermatomyositis. Although polymyositis may occur in association with Sjogren's syndrome,
keratoconjunctivitis would be much more common than muscle weakness.
9. What phenomenon is responsible when a person sets off the metal detector at the airport,
despite removing watch, belt buckle, and every other obvious source of metal?
A. Argyria
B. Gall stones
C. Hemochromatosis
D. Kidney stones
E. Wilson's disease

The correct answer is C. Hemochromatosis is an iron storage disorder that can cause cirrhosis (with
increased risk for hepatocellular carcinoma), skin pigmentation, pancreatic damage leading to
diabetes mellitus, and congestive heart failure. These complications are attributable to damage
caused by deposition of iron in tissues; the total body iron in some of these individuals may reach 50
g, sufficient to set off some airport metal detectors.
Argyria (choice A) is a blue-gray skin discoloration related to silver poisoning.
Neither gallstones (choice B) nor kidney stones (choice D) contain metal.
In Wilson's disease (choice E), copper is deposited in the liver and brain, but not enough to be
detected by metal detectors.
10. Which of the following is the explanation for the laboratory findings in an alcoholic with
chronic obstructive lung disease, secondary to cigarette smoking, of persistently lower serum
levels than expected of theophylline which is being used as a bronchodilator, even though the
drug is taken according to schedule?
A. Cirrhosis of the liver
B. Decreased absorption
C. Enhanced liver metabolism
D. Increased urinary clearance
E. Noncompliance
The correct answer is C. Alcohol and smoking normally enhance the cytochrome P450 system in the
smooth endoplasmic reticulum (SER) of the liver. This system is responsible for the metabolism of
drugs, hence, the low theophylline levels are most likely caused by enhanced liver metabolism. The
hepatocyte SER undergoes hyperplasia as a response to alcohol ingestion and synthesizes the
enzyme gamma-glutamyl transferase (GGT). An elevation of GGT would help confirm the likelihood of
increased hepatic drug metabolism as the cause of low drug levels.
Cirrhosis of the liver (choice A) would likely increase the serum levels of theophylline because of poor
metabolism of the drug.
Decreased absorption of the drug (choice B) in the gastrointestinal tract is a possible choice. The
history of excess alcohol intake, chronic smoking, and lack of a history of malabsorption, however,
suggest increased hepatic metabolism.
Increased clearance of theophylline in the urine (choice D) implies an increase in the glomerular
filtration rate, which would not be expected in this patient.
In most circumstances, the lack of an expected response to a medication is because of patient
noncompliance until proven otherwise; however, the question stem rules this out.
11. Which of the following could develop with a mucosal neuroma on the lower lip, family
history of medullary thyroid carcinoma, and recent introduction of severe headaches,
perspiration, palpitations, and hypertension?
A. Gastrinoma
B. Insulinoma
C. Parathyroid adenoma
D. Pheochromocytoma
E. Pituitary adenoma
The correct answer is D. This describes multiple endocrine neoplasia, specifically, MEN III (formerly
MEN II b). Features of this autosomal dominant condition include medullary carcinoma of the thyroid,

pheochromocytoma, and oral and intestinal ganglioneuromatosis (including mucosal neuromas).


Pheochromocytomas typically cause attacks of severe headache, perspiration, palpitations,
hypertension, anxiety, and tremor.
Gastrinomas (choice A) and insulinomas (choice B) are found in MEN I.
Parathyroid adenomas (choice C) are found in MEN I and II.
Pituitary adenomas (choice E) are found in MEN I.
11. Which of the following substances is significantly elevated in the serum of a young adult
male with bilateral parotid gland swelling, orchitis, and fever and malaise?
A. Alanine aminotransferase
B. Amylase
C. Aspartate aminotransferase
D. Cortisol
E. Creatine phosphokinase, MB isoenzyme
The correct answer is B. The disease described is mumps, caused by a paramyxovirus. In children,
mumps causes a transient inflammation of the parotid glands, and less commonly the testes,
pancreas, or central nervous system. Mumps tends to be a more severe disease in adults than in
children as it involves the testes (causing orchitis) and pancreas with some frequency. Pancreatic
involvement can cause elevation of serum amylase.
Alanine aminotransferase (ALT) (choice A) and Aspartate aminotransferase (AST) (choice C) are
markers for hepatocellular damage.
Cortisol (choice D) levels are increased in patients with Cushing's syndrome. This condition is
associated with central obesity, muscle wasting, thin skin, easy bruisability, psychologic changes,
hirsutism, and purple stria. Osteoporosis, hypertension, hyperglycemia, and glycosuria are also noted.
Creatine phosphokinase, MB isoenzyme (CPK-MB) (choice E) is the isoenzyme of CPK that is
relatively specific for the myocardium. This enzyme is increased in the early stages of a myocardial
infarction.
12. Which of the following forms of gastritis would MOST likely be found with a history of longterm use of nonsteroidal antiinflammatory drugs and complaints of heartburn, nausea, and
vomiting for several days?
A. Acute gastritis
B. Chronic antral gastritis
C. Chronic fundal gastritis
D. Hypertrophic gastritis
E. Lymphocytic gastritis
The correct answer is A. Acute gastritis, characterized by patches of erythematous mucosa,
sometimes with petechiae and ulceration, can be seen as a complication of a variety of other
conditions (alcohol use, aspirin and other NSAIDs use, smoking, shock, steroid use, and uremia),
which usually have in common disruption of the mucosal barrier of the stomach.
Chronic antral (type B) gastritis (choice B) is associated with Helicobacter pylori.
Chronic fundal (type A) gastritis (choice C) is associated with pernicious anemia.

Hypertrophic gastritis (Menetrier's disease; choice D) is an idiopathic condition characterized by


markedly enlarged mucosal folds.
Lymphocytic gastritis (choice E) is believed to be a gastric manifestation of celiac sprue.
13. What is responsible for mild temporary hyperthyroidism after a person complains of a
severe sore throat?
A. Diffuse nontoxic goiter
B. Grave's disease
C. Hashimoto's thyroiditis
D. Subacute granulomatous thyroiditis
The correct answer is D. It is due to subacute granulomatous (de Quervain) thyroiditis, which
frequently develops after a viral infection. Microscopically, it is characterized by microabscess
formation within the thyroid, eventually progressing to granulomatous inflammation with
multinucleated giant cells. Clinically, patients may experience fever, sudden painful enlargement of
the thyroid, or symptoms of transient hyperthyroidism. The disease usually abates within 6-8 weeks.
Diffuse nontoxic goiter (choice A) by definition does not produce hyperthyroidism. Goiter is usually
associated with low thyroid function.
The hyperthyroidism of Grave's disease (choice B) does not spontaneously remit.
Hashimoto's thyroiditis (choice C) can cause transient hyperthyroidism, but then goes on to cause
hypothyroidism.
14. What reaction has occured if after receiving a tuberculosis vaccine, the area becomes
indurated and erythematous, having a 12 mm diameter?
A. Antibody-dependent cell-mediated cytotoxicity
B. Local anaphylaxis
C. T-cell mediated cytotoxicity
D. Type III hypersensitivity
E. Type IV hypersensitivity
The correct answer is E.The tuberculin reaction is an example of delayed-type hypersensitivity (a
form of Type IV hypersensitivity) in which the bulk of the tissue damage is done by macrophages that
are stimulated by a few previously sensitized CD4+ memory T-cells recognizing antigens presented
by the macrophages. In contrast, in T-cell mediated cytotoxicity (choice C, another form of Type IV
sensitivity) the damage is done by CD8+ cytotoxic T-cells that recognize "foreign" cell surface
antigens and directly lyse targeted cells.
Antibody-dependent cell-mediated cytotoxicity reactions (choice A, a form of Type II hypersensitivity)
involves cells coated with a thin layer of antibody that triggers attack by cells (monocytes, neutrophils,
eosinophils, and natural killer cells) that can bind to Fc receptors.
Local anaphylaxis (choice B, a form of Type I hypersensitivity) is caused by the release of vasoactive
substances by mast cells and basophils stimulated by memory (CD4+) T-cells reacting to antigen.
Type III (choice D) hypersensitivity is caused by deposition of circulating antigen-antibody complexes,
often in small blood vessels.

15. Which of the following is characterized by extreme weakness, fatigue, nausea, stomach
cramps, hypotensive while sitting and even more on standing, hyperkalemia, increased
freckling around the eyes, and darkening of the palmar creases?
A. Addison's disease
B. Conn's syndrome
C. Cushing's syndrome
D. Secondary adrenal insufficiency
E. Tertiary adrenal insufficiency
The correct answer is A. Addison's disease usually occurs because of autoimmune destruction of
the adrenal cortex (all three zones are typically involved), resulting in decreased secretion of cortisol,
aldosterone, and adrenal androgens. Hyperpigmentation is the classic physical finding, resulting from
increased serum ACTH caused by loss of negative feedback inhibition by cortisol at the pituitary or
hypothalamus. The increase in pigmentation may occur because the first 13 amino acids of ACTH are
identical to alpha-melanocyte stimulating hormone. Low serum levels of cortisol produce
gastrointestinal symptoms such as nausea, vomiting, and anorexia. Fatigue and weakness are almost
always reported. Blood pressure is usually low and orthostatic hypotension may be present, because
arterioles are less responsive to the constrictor effects of catecholamines in the absence of cortisol.
The cardiovascular symptoms are worsened by the loss of blood volume caused by aldosterone
deficiency. Hyperkalemia is a manifestation of the low serum aldosterone; hyponatremia may also be
present.
Conn's syndrome (choice B) results from hypersecretion of aldosterone. It is characterized by
hypertension, hypernatremia, and hypokalemia.
Cushing's syndrome (choice C) occurs because of excessive secretion of cortisol. It is characterized
by central obesity, buffalo hump, moon facies, hypertension, and hypokalemia.
Secondary (choice D) and tertiary (choice E) adrenal insufficiency result in low serum levels of
ACTH. The subsequent hypocortisolism can produce the gastrointestinal complaints and fatigability
but not hyperkalemia. With deficiency of CRH or ACTH, serum aldosterone usually remains in the
normal range, and signs of mineralocorticoid deficiency are not present. Furthermore, low serum
levels of ACTH would not produce hyperpigmentation.
16. A 45-year-old man presents to a physician with back pain, facial pain, coarse facial
features, and kyphosis. His laboratory studies show elevated alkaline phosphatase. X-ray
studies demonstrate skull thickening with narrowing of foramina and bowing of the femur and
tibia. Bone biopsy reveals a mosaic pattern of bone spicules with prominent osteoid seams.
Which of the following neoplasms occurs at an increased frequency in patients with this
disorder?
A. Astrocytoma
B. Hodgkin lymphoma
C. Meningioma
D. Non-Hodgkin lymphoma
E. Osteosarcoma
The correct answer is E. The phrase "mosaic pattern" of newly formed woven bone is specific for
Paget disease of bone and is not seen in other bone conditions. The clinical and radiologic
presentation are typical; an increased hat size may also be a clue. In early stages, Paget disease is
characterized by osteolysis, producing patchwork areas of bone resorption with bizarre, large
osteoclasts. In the middle stage of the disease, secondary osteoblastic activity compensates with new
bone formation, producing the mosaic pattern. In late Paget disease, the bones are dense and
osteosclerotic. Paget disease is suspected to be related to prior viral infection, but the cause remains
unknown. Complications include myelophthisic anemia, high output cardiac failure, pain secondary to

nerve compression, deformities secondary to skeletal changes, and in approximately 1% of patients,


osteosarcoma or other sarcoma, typically involving the jaw, pelvis, or femur.
An increased incidence of astrocytomas (choice A) is associated with tuberous sclerosis.
Hodgkin lymphoma (choice B) is usually a disease of young adults, although older patients may have
the lymphocyte-depleted form.
Meningiomas (choice C) are mostly benign tumors that affect adults, especially women. There may
be an association with breast cancer, possibly related to high estrogen states.
Non-Hodgkin lymphoma (choice D) is more common in AIDS and other immunodeficiency states,
although the incidence in the immunocompetent is increasing.
17. Which of the following can be present with swollen and painful toes and knees, morning
low back stiffness, conjunctivitis, as well as mouth and skin ulcerations in a young adult?
A. Gout
B. Lyme disease
C. Reiter's syndrome
D. Rheumatoid arthritis
E. Septic arthritis
The correct answer is C. This is a description of Reiter's syndrome. Patients typically present with
the acute onset of arthritis (usually asymmetric and additive), with involvement of new joints occurring
over a period of a few days to 2 weeks. Joints of the lower extremities are the most commonly
involved, but wrists and fingers can also be affected. Dactylitis (sausage digit), a diffuse swelling of a
solitary finger or toe, is a distinctive feature of Reiter's arthritis and psoriatic arthritis. Tendonitis and
fasciitis are common, as are spinal pain and low back pain. Oligoarthritis, conjunctivitis, urethritis, and
mouth ulcers are the most common features. The mucocutaneous lesions may include balanitis,
stomatitis, and keratoderma blennorrhagicum. Microorganisms that can trigger Reiter's syndrome
include Shigella spp., Salmonella spp., Yersinia spp., Campylobacter jejuni, and Chlamydia
trachomatis. Most patients are younger males.
Gout (choice A) usually presents as an explosive attack of acute, very painful, monarticular
inflammatory arthritis. Hyperuricemia is the cardinal feature and prerequisite for gout. The first
metatarsophalangeal joint is involved in more than 50% of first attacks.
Lyme disease (choice B), caused by Borrelia burgdorferi, presents with a red macule or papule at the
site of the tick bite. This lesion slowly expands to form a large annular lesion with a red border and
central clearing. The lesion is warm, but usually not painful. The patient also has severe headache,
stiff neck, chills, arthralgias, and profound malaise and fatigue. Untreated infection is associated with
development of arthritis in the large joints (e.g., knees) lasting for weeks to months.
Rheumatoid arthritis (choice D) begins insidiously with fatigue, anorexia, generalized weakness, and
vague musculoskeletal symptoms leading up to the appearance of synovitis. Pain in the affected
joints, aggravated by movement, is the most common manifestation of established rheumatoid
arthritis. Generalized stiffness is frequent, especially in the morning, and is usually greatest after
periods of inactivity. Rheumatoid arthritis is more common in females. The metacarpophalangeal and
proximal interphalangeal joints of the hands are characteristically involved.
Septic arthritis (choice E) is caused by a variety of microorganisms, including Neisseria gonorrhoeae
and Staphylococcus aureus. Hematogenous spread is the most common route in all age groups.
Approximately 90% of patients present with involvement of a single joint, usually the knee. The usual
presentation is moderate to severe pain, effusion, muscle spasm, and decreased range of motion.

18. Which of the following antigens would autoantibodies be directed against when there is
symmetrical swelling of the proximal phalangeal joints and large subcutaneous nodules over
the extensor surfaces of both arms of a older woman?
A. Acetylcholine receptor
B. Double stranded DNA
C. Histones
D. IgG
E. Ribonucleoprotein
The correct answer is D. The disease described is rheumatoid arthritis, and the autoantibody is
rheumatoid factor, which is usually an IgM or IgG (or less commonly IgA) directed against the
constant region of autologous IgG.
Autoantibody directed against acetylcholine receptors (choice A) is a feature of myasthenia gravis.
Autoantibody directed against double stranded DNA (choice B) is a feature of systemic lupus
erythematosus.
Autoantibody directed against histones (choice C) is a feature of drug-induced lupus.
Autoantibody directed against ribonucleoprotein (choice E) is a feature of mixed connective tissue
disease.
19. What is the BEST explanation for rapid weight gain, buffalo hump formation, prominent
vertical purple abdominal striae, and the increase of fasting blood glucose, plasma levels of
ACTH and cortisol, and also the presence of osteoporosis, hypertension, poor wound healing,
and hypokalemia in an older male?
A. Addison's disease
B. An ectopic ACTH-secreting tumor
C. Conn's syndrome
D. Cushing's disease
E. Primary hypercortisolism
The correct answer is D. This is a description of "Cushingoid" signs and symptoms caused by
hypercortisolism. Although the acute effect of cortisol is to produce lipolysis, patients with chronically
increased cortisol levels develop a characteristic central obesity and buffalo hump. The extremities
are often thinned. The mechanism for the redistribution of body fat is not known but may involve an
interaction between cortisol and insulin. The weight gain with hypercortisolism usually results from
increased appetite. Cortisol excess causes protein catabolism, which leads to poor wound healing,
decreased connective tissue, and fragile blood vessels. The combination of thin skin and fragile blood
vessels leads to abdominal stretch marks (striae) that are characteristically purple in color. Because of
increased gluconeogenesis and decreased peripheral insulin sensitivity, blood glucose may be
increased. Osteoporosis, hypertension, poor wound healing, and hyperkalemia are also commonly
seen. If the hypercortisolism is caused by a functional tumor in the adrenal cortex (primary
hypercortisolism, choice E), plasma concentration of ACTH should be low because of negative
feedback suppression. Increased cortisol and increased ACTH could result from a functional ACTHsecreting tumor in the pituitary (Cushing's disease) or an ectopic tumor (such as a small cell
carcinoma of the lung, choice B).
Addison's disease (choice A) is primary adrenal insufficiency, and whereas plasma ACTH is
increased (producing hyperpigmentation), plasma cortisol and aldosterone are decreased (not
increased) compared with normal.

10

Conn's syndrome (choice C) results from hypersecretion of aldosterone by the adrenal cortex. Some
of the clinical manifestations overlap with Cushing's disease: for example, both may exhibit
hypertension. In the case of Conn's syndrome, this is caused by excessive renal sodium and water
reabsorption because of increased aldosterone levels. In Cushing's disease, it is due in part to the
mineralocorticoid-like effects of high plasma cortisol.
20. What condition is marked by dilated cardiomyopathy, slow speech and intellectual
function, fatigue, lethargy, cold intolerance, listlessness, thickened facial features, periorbital
edema, dry, coarse skin, and peripheral edema, and also with low serum levels of T4 and high
ones of TSH?
A. Cretinism
B. Grave's disease
C. Hyperthyroidism
D. Myxedema
E. Thyroid cancer
The correct answer is D. Myxedema is caused by long-standing hypothyroidism in adults.
Myxedema can result from the many causes of hypothyroidism: Hashimoto's thyroiditis, idiopathic
primary hypothyroidism, iodine deficiency, drugs, pituitary lesions, hypothalamic lesions, and damage
to the thyroid by surgery or radiation.
Cretinism (choice A) is caused by hypothyroidism in infancy.
Grave's disease (choice B) usually produces hyperthyroidism.
Hyperthyroidism (choice C) describes the signs and symptoms associated with an overproduction of
thyroid hormone. Because the body's metabolism is increased,patients often feel hotter than those
around them and can slowly lose weight even though they may be eating more. The weight issue is
confusing sometimes since some patients actually gain weight because of an increase in their
appetite. There is usually fatigue at the end of the day, but have trouble sleeping. Trembling of the
hands and a hard or irregular heartbeat (called palpitations) may develop. These individuals may
become irritable and easily upset. When severe, there can be shortness of breath, chest pain, and
muscle weakness.
Thyroid cancer (choice E) often causes a painless swelling in the region of the thyroid. Thyroid
function tests are usually normal.
21. Which of the following occurs after an upper respiratory infection and involves severe
lower back pain, generalized muscle weakness, and distal paresthesia and demyelination, but
there is no appreciable sensory loss and there is minimal residual sequelae?
A. Creutzfeldt-Jakob disease
B. Friedreich ataxia
C. Huntington's disease
D. Multiple sclerosis
The correct answer is D. This describes Guillain-Barr syndrome, also known as acute idiopathic
inflammatory polyneuropathy. This condition, which typically follows an upper respiratory or other
infection by several days to a month, is caused by an autoimmune attack on the myelin of peripheral
nerves. In this respect, it is most similar to multiple sclerosis, which is an autoimmune attack on the
myelin in the brain and spinal cord. Most cases of Guillain-Barr syndrome resolve spontaneously. A
few patients have recurrences, and rare patients die during the acute episode of respiratory muscle
failure (artificial ventilation may be required).
Creutzfeldt-Jakob disease (choice A) is an Alzheimer-like condition caused by a prion (protein
infectious agent).

11

Friedreich ataxia (choice B) is an autosomal recessive disorder associated with spinocerebellar


degeneration. Ataxia and paralysis are seen beginning in adolescence.
Huntington's disease (choice C) is an autosomal dominant degeneration of the caudate and frontal
lobes characterized by movement disorder and dementia.
22. Which of the following can involve lassitude, myalgia, mylar rash, joint pain, elevated
urinary protein, with the blood showing leukopenia and a high titer of antinuclear antibodies?
A. Generalized fatigue
B. Goodpasture's syndrome
C. Systemic lupus erythematosus
D. Scleroderma
The correct answer is C. Systemic lupus erythematosus (SLE) is a prototype connective tissue
disease. The diagnosis requires four criteria to be met from a list of 11 possible criteria: malar rash,
discoid rash, photosensitivity, oral ulcers, arthritis, serositis, renal disorder, neurologic disorder,
hematologic disorder, immunologic disorder, and antinuclear antibody. The appearance of a malar or
"butterfly" rash seen in approximately half of all patients is generally the easiest way to diagnose this
condition on the dental boards. Antinuclear antibodies (ANA) are present in 95-100% of cases of SLE;
anti-double-stranded DNA is found in 70% of the cases.
Generalized fatigue (choice A) can occur in anyone, but the presence of the other criteria make SLE
more likely.
Goodpasture's syndrome (choice B) is characterized by linear disposition of immunoglobulin and
often C3 along the glomerular basement membrane (GBM). Glomerulonephritis, pulmonary
hemorrhage, and occasionally idiopathic pulmonary hemosiderosis occur.
Scleroderma (choice D) is characterized by thickening of the skin caused by swelling and thickening
of fibrous tissue, with eventual atrophy of the epidermis. ANA are often associated with the disease,
but the staining pattern is generally nucleolar.
23. Which of the following cancers may develop when there are islands of red tissue noted
above the gastroesophageal junction in situations involving chronic reflux of gastric contents
into the esophagus?
A. Adenocarcinoma of the esophagus
B. Adenocarcinoma of the stomach
C. Sarcoma of the esophagus
D. Sarcoma of the stomach
E. Squamous cell carcinoma of the esophagus
The correct answer is A. The lesion is Barrett's esophagus, which is related to chronic reflux of
gastric contents into the esophagus. This lesion predisposes for the development of adenocarcinoma
of the distal esophagus, which is the most serious complication of Barrett's esophagus. The
development of adenocarcinoma of the esophagus is approximately 40 times higher in patients with
Barrett's esophagus compared with those without the lesion.
Conditions predisposing for adenocarcinoma of the stomach (choice B) include chronic atrophic
gastritis, pernicious anemia, and postsurgical gastric remnants.
Sarcoma of the esophagus (choice C) or stomach (choice D) is rare.
Plummer-Vinson syndrome predisposes for squamous cell carcinoma of the esophagus (choice E).

12

24. Which of the following when removed from the diet will help improve a history of weight
loss, diarrhea, flatulence, greasy stools, and increased fecal fat and marked atrophy of villi?
A. Beef
B. Eggs
C. Potatoes
D. Tomatoes
E. Wheat
The correct answer is E. This describes celiac sprue which is caused by an allergic, immunologic, or
toxic reaction to the gliadin component of gluten from wheat, as well as rye, barley, and oats, but not
rice or corn. The symptoms and pathologic changes usually reverse with complete removal of gliadin
from the diet. Therapeutic failures are frequently caused by hidden wheat in the diet. Patients with
celiac disease have an increased risk for developing gastrointestinal lymphoma. The classic signs and
symptoms of celiac sprue include weight loss, flatuence, greasy stools, and increased fecal fat. A
diagnosis can be confirmed when clinical improvement is made on a gluten-free diet.
25. Which of the following is the MOST likely explanation for a rash in a leukemic patient that
consists of multiple erythematous patches on the arms, legs, palms, and soles, some of which
show central clearing and surrounding ring formation?
A. Urticaria
B. Erythema multiforme
C. Kaposi's sarcoma
D. Psoriasis
The correct answer is B. The presence of erythematous patches with central clearing, known
clinically as target lesions, is associated with erythema multiforme. Both erythema multiforme and its
severe, life-threatening version, known as Stevens-Johnson syndrome, are produced by immune
complex deposition in dermal blood vessels. In approximately 50% of patients, no specific
precipitating cause is identified. In the remainder of patients, however, a variety of causes have been
implicated, including certain infections (herpes simplex, enteroviruses, Mycoplasma pneumoniae,
Chlamydia, histoplasmosis), drugs, neoplasia, sarcoidosis, and foods. Some penicillins and
corticosteroids can also cause this condition.
Urticaria (choice A) causes wheals that are intensely pruritic, but does not produce target lesions.
Kaposi's sarcoma (choice C) causes purple lesions with no target lesions.
Psoriasis (choice D) causes erythematous plaques with silvery scales but does not produce target
lesions.
26. A 72-year-old man with a significant smoking history presents with dyspnea, facial
erythema, and facial, truncal, and arm edema with prominence of thoracic and neck veins.
Chest x-ray reveals a mass in the right mediastinum with adenopathy. Which of the following
is the most likely diagnosis?
A. Adenocarcinoma
B. Hodgkin lymphoma
C. Large cell carcinoma
D. Non-small cell carcinoma
E. Small cell carcinoma
The correct answer is E. Superior vena cava (SVC) syndrome is characterized by obstruction of
venous return from the head, neck, and upper extremities. More than 85% of cases of SVC syndrome
are related to malignancy. Bronchogenic carcinomas (most commonly small cell cancer and

13

squamous cell cancer) account for more than 80% of these cases. Among bronchogenic carcinomas,
the most common causes of SVC syndrome (in order of frequency) are small cell carcinoma,
epidermoid carcinoma, adenocarcinoma (choice A), and large cell carcinoma (choice C).
Lymphomas such as Hodgkin disease (choice B) and non-Hodgkin lymphoma are uncommon causes
of SVC syndrome. Rare tumors associated with SVC syndrome include primary leiomyosarcomas and
plasmacytomas. Infectious etiologies include tuberculosis, syphilis, and histoplasmosis. SVC
syndrome can also occur as a result of an enlarged goiter, and from thrombus formation caused by
indwelling intravenous lines or pacemaker wires.
Non-small cell carcinoma (choice D) is not commonly associated with SVC syndrome.
27. What can be present when there is digital clubbing and tenderness over the distal ends of
the radius, ulna, and fibula?
A. Gastrointestinal cancer
B. Liver cancer
C. Lung cancer
D. Renal cancer
E. Testicular cancer
The correct answer is C. This patient has the finger clubbing and hypertrophic pulmonary
osteoarthropathy that can be associated with bronchogenic carcinoma (other than squamous cell
carcinoma), benign mesothelioma, and diaphragmatic neurilemmoma. X-ray of the bones generally
shows formation of new periosteal bone; arthritis may be present. The etiology of these changes
remains a mystery. An alert clinician may identify a cancer at an earlier, potentially curable stage by
investigating a possible paraneoplastic syndrome. Digital clubbing is caused by chronically low
oxygen levels. This condition affects the fingers and toes in which proliferation of the distal tissues,
especially the nail beds, results in broadening of the extremities of these digits. The affected nails are
abnormally curved and shiny.
28. Which of the following lesions are present in an adult with newly diagnosed tuberculosis?
A. A single lesion in a lung apex
B. A single lesion in the gastrointestinal tract
C. A single lesion subjacent to the pleura
D. A lesion subjacent to the pleura in the lower part of an upper lobe and active disease in the
mediastinal lymph nodes
E. Multiple tiny masses throughout the body
The correct answer is A. Because tuberculosis is transmitted primarily by "droplet" transmission by
way of the nasopharynx, it is essential that the dental student understand the basic principles of
tuberculosis. Primary tuberculosis infection characteristically involves the lung subjacent to the pleura
in either the lower part of the upper lobe or the upper part of a lower lobe of one lung. The mediastinal
nodes are also usually involved (choice D), rather than having a single lesion subjacent to the pleura
(choice C) without lymph node involvement. Roughly 80% of newly diagnosed pulmonary
tuberculosis cases in adults are actually caused by reactivation of an often clinically unsuspected
infection acquired years to decades previously. The reinfection site usually is in the apex of the lung.
The source of reinfection is usually a Gohn complex, or calcified granuloma of giant cells,
mycobacteria leukocytes, and fibrous cells.
The lungs are not the only site where tuberculosis can occur (it can occur throughout the body), and
isolated gastrointestinal involvement (choice B) is (uncommonly) also seen.
Miliary tuberculosis (rare) is a widely disseminated and dangerous form of tuberculosis characterized
by small lesions throughout the body (choice E). It is more common in immunocompromised patients.

14

29. Which of the following is involved when there is excessive bleeding after labor, and the
laboratory studies demonstrate decreased platelets, prolonged prothrombin time and partial
thromboplastin time, and increased fibrin split products?
A. Disseminated intravascular coagulation
B. Hemophilia A
C. Severe liver disease
D. Vitamin K deficiency
E. Von Willebrand's disease
The correct answer is A. Thsi is a description of disseminated intravascular coagulation (DIC), a
feared and often life-threatening complication of many other disorders, including amniotic fluid
embolism, infections (particularly gram-negative sepsis), malignancy, and major trauma. This is
suspected when a decrease in platelets and a prolongation of PT and PTT times are observed. The
observed hematologic abnormalities are caused by consumption of platelets and clotting factors,
caused by extensive microclot formation with accompanying fibrinolysis (reflected by the increased
fibrin split products).
Hemophilia (choice B) alters the PTT without affecting the other indices.
Severe liver disease (choice C) produces alterations comparable to those in vitamin K deficiency;
platelets can also be decreased secondary to a generalized metabolic marrow dysfunction, but fibrin
split products would not be increased.
Vitamin K deficiency (choice D) is associated with alterations in PT and PTT, but platelets will not be
decreased, nor will fibrin split products be increased.
Von Willebrand's disease (choice E) produces impaired platelet adhesion and increases the bleeding
time and the PTT, but will not produce the other features described.
30. Which of the following can account for a child developing fever, conjunctivitis,
photophobia, cough, white spots on a bright red background on the buccal mucosa, and a
rash that begins around the hairline, then spreads to the trunk and extremities?
A. Aphthous ulcers
B. Herpetic stomatitis
C. Laryngeal papillomas
D. Measles
E. Oral thrush
The correct answer is D. This description is of measles, with the appearance of Koplik's spots (white
spots on the buccal mucosa) followed by a rash beginning along the neck and hairline and spreading
to the trunk and extremities. The sequela of this condition may be postinfectious encephalomyelitis
that can follow infection with measles, varicella, rubella, mumps, influenza, or vaccination with
vaccinia vaccine or rabies vaccine derived from nervous tissue.
Aphthous ulcers (choice A) are easy to recognize because they are round ulcerations with yellowgray fibrinoid centers surrounded by red halos.
Herpetic stomatitis (choice B) is common, mild, short-lived, and requires no intervention. There is an
initial burning followed by small vessicles that rupture to form scabs.
Laryngeal papillomas (choice C) are common lesions of the larynx and other sites where ciliated and
squamous epithelia meet. Hoarseness progresses to stridor over the course of weeks to months.
Oral thrush (choice E) is associated with the development of painful, creamy white, curd- like patches
overlying erythematous mucosa.

15

31. What is present when an elderly person can no longer recognize people and common
objects, plan activities, and wanders with an absent look on their face, but who is still happy,
yet frustrated by the memory loss, and speech is limited to simple two- or three-word
sentences?
A. Alzheimer's disease
B. Amnestic disorder
C. Clinical depression
D. Substance-induced persisting dementia
E. Parkinson's disease
The correct answer is A. This is a description of dementia of the Alzheimer's type. A gradual onset
of symptoms, general pervasive memory deficit, difficulties with language, and inability to plan,
leading to severe impairment of daily functioning are all characteristic of dementia of the Alzheimer's
type.
Amnestic disorder (choice B) is limited to memory problems; and not cognitive dysfunction, such as
alterations in language and the loss of the ability to plan.
Clinical depression (choice C) is associated with feelings of helplessness and hopelessness.
Cognitive function is not impaired.
The diagnosis of substance-induced persisting dementia (choice D) requires evidence of a history of
substance abuse. It is the second most likely diagnosis, however, and should be carefully explored.
Parkinson's disease (choice E) is associated with develpment of dyskinesia, postural instability,
moonlike facies, and difficulty performing daily activities.
32. Which of the following substances would be MOST likely be elevated due to the
development of large osteoblastic bone lesions in a patient with prostate cancer?
A. Prostatic acid phosphatase
B. Prostate specific antigen
C. Serum alkaline phosphatase
D. Serum uric acid
E. Leukocytes
The correct answer is C. Osteoblastic cells respond to metastatic prostate carcinoma by forming
bone (osteoid), and secreting alkaline phosphatase, which is believed to initiate or facilitate
mineralization.
Prostatic acid phosphatase (choice A) and prostatic-specific antigen (choice B) are synthesized by
the tumor and would most likely be elevated; however, they are elevated because of the prostatic
cancer independent of the bony metastasis. Serum uric acid (choice D) would be expected to be
seen in patients with gout. White blood cells (choice E) can be elevated by a bacterial infection,
rheumatoid arthritis, and several medications, including glucocorticoids and lithium. In other words,
elevated white blood cells is often a nonspecific indicator.
33. Which of the following pair of hormones regulates the hormone responsible for a middleaged man experiencing gradual coarsening of facial features, progressive protrusion of the
lower jaw, and having to wear larger shoes and gloves?
A. Dopamine and norepinephrine
B. LH and hCG
C. Prolactin and FSH
D. Somatostatin and GHRH
E. TSH and ACTH

16

The correct answer is D. The disease described is acromegaly, which is typically produced by a
growth hormone-secreting pituitary adenoma. Growth hormone synthesis is predominantly regulated
by hypothalamic GHRH (growth hormone-releasing hormone), and its pulsatile secretion is
predominantly regulated by hypothalamic somatostatin. Clinical features of acromegaly include
excessive growth of hands and feet in adults, protusion of the lower jaw, coarsening of facial features,
and a deeper voice.
Dopamine and norepinephrine (choice A) are catecholamines that regulate smooth muscle tone and
cardiac function.
Choice B is incorrect because luteinizing hormone (LH) regulates sex steroid hormone production by
testes and ovaries; human chorionic gonadotropin (hCG) is produced by the placenta and has actions
similar to LH.
Choice C is incorrect because prolactin regulates menstruation and lactation, whereas follicle
stimulating hormone (FSH) regulates ovarian and testicular function.
Choice E is incorrect because thyroid stimulating hormone (TSH) regulates secretion of thyroid
hormones and adrenocorticotropin (ACTH) regulates glucocorticoid secretion.
34. Which of the following tissues or organs may be associated with hyperplasia and
neoplastic proliferation if there is fatigue, feeling of constant coldness, diffusely enlarged and
rubbery thyroid gland, while laboratory tests show low T3 and T4, high TSH, large numbers of
lymphocytes of all degrees of maturation, a few abnormal follicular cells with eosinophilic
granular cytoplasm, and only rare normal follicular cells?
A. Colon
B. Esophagus
C. Peripheral nerve
D. Skin
E. Thymus
The correct answer is E. The thyroid disease is Hashimoto's thyroiditis, an autoimmune disease in
which the thyroid parenchyma is destroyed by a lymphocytic infiltrate. The infiltrate typically contains
mature follicles; the remaining scanty follicular cells often have eosinophilic granular cytoplasm and
are called Hurthle cells or oncocytes. Clinically, patients usually have hypothyroidism, although brief
periods of hyperthyroidism ("Hashitoxicosis") may also be seen. Like myasthenia gravis, Hashimoto's
disease may be associated with thymic disorders, including thymic hyperplasia, benign thymomas,
and malignant thymomas.
Colon (choice A) cancer is associated with ulcerative colitis and adenomatous polyps.
The risk for esophageal cancer (choice B) is increased with Barrett's esophagus and in PlummerVinson syndrome.
You should associate neurofibromas of peripheral nerve (choice C) with cafe au lait spots on the skin.
Skin cancer (choice D) occurs with greater frequency in association with xeroderma pigmentosa and
actinic keratosis.
35. What is occuring when an esophagus is dilated, kinked, tortuous, partly filled with
undigested foods, and is associated with chronic dysphagia?
A. Achalasia
B. Barrett's esophagus
C. Hiatal hernia
D. Plummer-Vinson syndrome
E. Zenker's diverticulum

17

The correct answer is A. Achalasia (from the Greek "unrelaxed") is a disease of ganglion cells in the
esophageal myenteric plexus causing a failure of relaxation in the lower esophageal (cardiac)
sphincter. The cause of achalasia is usually not determined. The peristaltic waves in the esophagus
stop before the sphincter, and the food collects in the esophagus, which becomes dilated and
elongated.
Barrett's esophagus (choice B) is metaplastic replacement of the squamous esophageal epithelium
with columnar epithelium. Barrett's esophagus is an important risk factor for esophageal
adenocarcinoma.
Hiatal hernia (choice C) is a protrusion of the stomach into the thorax by way of the diaphragmatic
hiatus, at the lower esophageal sphincter. Although hiatal hernia can produce gastroesophageal
reflux, the esophagus does not become distended and food passes normally into the stomach.
Plummer-Vinson syndrome (choice D) is a constellation of physical findings associated with severe
iron-deficiency anemia including koilonychia, atrophic glossitis, and dysphagia caused by atrophy of
the pharyngeal mucosa and mucosal webs in the upper esophagus.
Zenker's diverticulum (choice E) is an oropharyngeal diverticulum occurring at the junction of the
pharynx and esophagus which occurs because of wall weakness in the esophagus at this location,
and may produce dysphagia.
36. What is involved in a smoker with scant clear mucoid sputum, quiet sounding chest,
hemoptysis, weight loss, detruction of alveolar septae around the respiratory bronchioles,
with marked enlargement of the airspaces, and heavy deposits of anthracotic pigment?
A. Asthma
B. Chronic bronchitis
C. Emphysema
D. Pulmonary hypertension
E. Silicosis
The correct answer is C. Emphysema is a pulmonary disease characterized by enlargement of the
alveolar airspaces caused by destruction of the septae without consequent fibrosis. The gross
appearance of emphysematous lungs is characteristic: alveoli are sufficiently dilated to allow
visualization with the naked eye and destruction of structural support to lymphatic vessels produces
heavy pigment deposition in the tissue. Microscopic findings that confirm the diagnosis include
enlarged, round airspaces with club-like ends of broken septae sticking into the alveoli. There is scant
clear mucoid sputum. The chest is very quiet without adventitious sounds.
Asthma (choice A) is a disease of airway hyperreactivity and is characterized by hypertrophy of the
bronchial basement membranes and smooth muscle, with glandular hyperplasia and thick mucus
plugs in the bronchi. Wheezing will be heard on expiration.
Chronic bronchitis (choice B) produces marked hypersecretion of mucopurulent mucus in the large
airways and can be identified by hypertrophy of mucous glands in the bronchi and goblet cell
hyperplasia in the smaller airways. The chest is noisy with rhonci invariably present. Wheezing is
common.
Pulmonary hypertension (choice D) affects neither the airways nor the alveoli. It is characterized by
thickening of the arterial smooth muscle with intimal hyperplasia and fibrosis. Atherosclerotic changes
in the normally plaque-free larger pulmonary arteries may be seen.
Silicosis (choice E), one of the forms of pneumoconiosis, is an interstitial fibrosing disease that
produces thick pleural scars and dense nodules of collagen that may calcify. The silica particles may
be visualized within the nodules using polarized light.

18

37. What presents with painful widespread erosions on the mucous membranes with friability,
no well-defined borders, acantholysis, and with direct immunofluorescence demonstrate an
intraepidermal band of IgG and C3?
A. Bullous pemphigoid
B. Dermatitis herpetiformis
C. Herpes simplex I
D. Herpes simplex II
E. Pemphigus vulgaris
The correct answer is E. This is a description of pemphigus vulgaris, in which autoantibody directed
against transmembrane cadherin adhesion molecules induces acantholysis (breakdown of epithelial
cell-cell connections) with resulting intraepidermal blister formation. It has an insidious onset of flaccid
bullae in crops or waves. It may develop spontaneously or following triggers such as drugs (thiols,
penicillamine), physical injury (burns), cancer, pregnancy, other skin diseases, and emotional stress.
Pemphigus vulgaris is a relatively rare blistering disease; it is seen more commonly in patients with
Jewish or Mediterranean heritage. The epidermis at the edge of these erosions is often easily
disrupted by sliding pressure (Nikolsky sign). Pemphigus vulgaris begins in the mouth in 50% of
cases. Acantholysis is seen on biopsy.
Bullous pemphigoid (choice A) is characterized by deeper blisters occurring at the dermal-epidermal
junction.
Dermatitis herpetiformis (choice B) is characterized by severe, intense pruritus and groups of papules
and vesicles.
Herpes simplex I (choice C) or II (choice D) can show multinucleated giant cells on scrapings of the
ulcer base.
38. Which of the following thyroid diseases is most likely related to restlessness, fever,
profuse sweating, marked tachycardia and tremor during labor, later developing delirium,
nausea, vomiting, and abdominal pain between contractions?
A. Follicular carcinoma
B. Grave's disease
C. Hashimoto's thyroiditis
D. Hypertensive urgency
E. Papillary carcinoma
The correct answer is B. This is a description of a thyrotoxic crisis that occurs most commonly in
untreated or inadequately treated Grave's disease. The onset is typically abrupt and may be
precipitated by stressors that can include infection, trauma, radioiodine treatment, and childbirth. The
condition, if unrecognized, may progress to congestive cardiac failure, pulmonary edema, and death.
Both follicular (choice A) and papillary (choice E) carcinomas of the thyroid gland are usually
nonsecretory and consequently do not produce hyperthyroidism.
Hashimoto's thyroiditis (choice C) is an autoimmune thyroiditis that may transiently produce
hyperthyroidism before producing hypothyroidism, but thyrotoxic crisis is not usually a feature.
Hypertensive urgency (choice D) is a situation in which blood pressure must be reduced in a few
hours. Most patients are asymptomatic with severe hypertension (systolic > 220 mmHg or diastolic >
125 mmHg).

19

39. Which of the following is the most probable etiology for a young person with acute
bacterial endocarditis limited to the tricuspid valve?
A. Congenital heart disease
B. Illicit drug use
C. Rheumatic fever
D. Rheumatoid arthritis
E. Rheumatic heart disease
The correct answer is B. The most probable etiology of bacterial endocarditis involving the tricuspid
valve is illicit intravenous drug use, which can introduce skin organisms into the venous system that
then attack the tricuspid valve. Staphylococcus aureus accounts for 60-90% of cases of endocarditis
in intravenous drug users. This is an important concept for dentists because some patients with drug
seeking behaviors, such as the intravenous drug abuser, may present to a dental office in search of
narcotic medications.
The endocarditis associated with congenital heart disease (choice A) typically involves either
damaged valves or atrial or ventricular septal defects. The tricuspid valve is not particularly
vulnerable.
Rheumatic fever (choice C) most commonly damages the mitral and aortic valves, and tricuspid
damage is usually less severe and seen only when the mitral and aortic valves are heavily involved.
Consequently, secondary bacterial endocarditis involving only the tricuspid valve in a patient with a
history of rheumatic fever would be unusual.
Rheumatoid arthritis (choice D) is not associated with bacterial endocarditis, unless there is an
association with SLE.
Rheumatic heart disease (choice E) which occurs after rheumatic fever and again involves mainly the
mitral and aortic valves.
40. What is the specific mechanism by which death is produced from breathing carbon
monoxide in a closed space?
A. Damage to the plasmalemma
B. Decreased oxygen-carrying capacity of blood
C. Increased calcium transport into mitochondria
D. Poisoning of oxidative phosphorylation
E. Rupture of lysosomes
The correct answer is B. Carbon monoxide has a very high affinity for hemoglobin, and binds, nearly
irreversibly, in such a manner that oxygen cannot bind, drastically decreasing the oxygen-carrying
capacity of the blood. Carbon monoxide also causes the oxygen-hemoglobin dissociation curve to
shift to the left, making oxygen more difficult to unload. Traditionally, carbon monoxide poisoning have
been described as having "cherry red" blood and skin, but this change is somewhat unreliable in real
life.
Choices A, C, and E list secondary changes that are commonly observed in injured cells, no matter
what the cause of the injury.
Cyanide acts by poisoning oxidative phosphorylation (choice D).

20

41. Which of the following clinical features would be most likely to be present when thyroid
function tests reveal increased TSH, decreased total T4, decreased free T4, and decreased T3
uptake?
A. Diarrhea
B. Heat intolerance
C. Hyperactivity
D. Palpitations
E. Weight gain
The correct answer is E. The laboratory studies are typical for primary hypothyroidism, in which the
thyroid fails to produce adequate T4 despite appropriate TSH signals from the pituitary gland. Clinical
features of hypothyroidism include weight gain, cold intolerance, hypoactivity, fatigue, lethargy,
decreased appetite, constipation, weakness, decreased reflexes, facial and periorbital myxedema, dry
and cool skin, and brittle hair.
In contrast, diarrhea (choice A), heat intolerance (choice B), hyperactivity (choice C), and
palpitations (choice D) are features of hyperthyroidism.
42. Which of the following findings would be MOST consistent for a young person in a coma
having an acetone-like odor?
A. Alcohol intoxication
B. Diabetic hyperosmolar coma
C. Diabetic ketoacidosis
D. Heroin overdose
E. Profound hypoglycemia
The correct answer is C. The smell of acetone on the breath of a comatose person is an important,
rapid diagnostic clue that strongly suggests ketoacidosis and is usually seen in patients with poorly
controlled type 1 diabetes mellitus. Other features of diabetic ketoacidosis include high blood glucose,
increased serum osmolality, hypovolemia, acidosis, and electrolyte imbalance. It is important to note
that diabetic ketoacidotic coma is usually preceded by a day or more of polyuria and polydipsia
associated with marked fatigue, confusion, and nausea.
In alcohol intoxication (choice A), the breath will smell like alcohol. Some may confuse a patient with
diabetes in ketoacidosis as being "drunk," however, because of the similar smell.
Diabetic hyperosmolar coma (choice B) usually is seen in older patients with type 2 diabetes mellitus
and is not characterized by ketoacidosis. Because there is no acetone production, there is no specific
scent to the breath. In heroin overdose (choice D), no acetone production occurs and there is no
specific scent to the breath. In hypoglycemic coma (choice E), which can occur in diabetics with
insulin overdose, no acetone production occurs and there is no specific scent to the breath.
43. A 24-year-old man presents with complaints of itching on his arms and face. He states that
he took four 500-mg capsules of amoxicillin 1 hour before a dental procedure. Physical
examination reveals well circumscribed wheals with raised, erythematous borders and
blanched centers. Which form of hypersensitivity is this patient probably exhibiting?
A. Acute serum sickness (Type III)
B. Antibody-dependent cell-mediated cytotoxicity (Type II)
C. Antireceptor antibodies (Type II)
D. Delayed type hypersensitivity (Type IV)
E. Immediate type hypersensitivity (Type I)

21

The correct answer is E. Urticaria (hives) is a good example of a local anaphylaxis reaction that is
classified as a Type I hypersensitivity reaction. Type I hypersensitivity reactions involve preformed IgE
antibody bound to mast cells or basophils that release vasoactive and spasmogenic substances when
they react with antigens. Certain allergens, especially drugs, insect venoms, latex, and foods may
induce an IgE antibody response, causing a generalized release of mediators from mast cells leading
to a systemic reaction.
Acute serum sickness (choice A) is now uncommon but was formerly seen when animal sera were
used for passive immunization. Serum-like sickness has also been seen in patients receiving
cephalosporins, such as cefaclor.
The eosinophil-mediated cytotoxicity against parasites is an example of antibody-dependent cellmediated cytotoxicity (choice B).
Myasthenia gravis is an example of a disease caused by antireceptor antibodies (choice C).
The tuberculin (PPD) reaction used to test for tuberculosis exposure is an example of delayed-type
hypersensitivity (choice D).
44. Which of the following notations in a chart indicates the greatest likelihood that a 1-cm,
flat,white patch on the buccal mucosa will progress to a malignancy?
A. Hairy leukoplakia
B. Leukoplakia
C. Linea alba
D. Oral thrush
E. Squamous papilloma
The correct answer is B. Leukoplakia is a white plaque on the oral mucosa for which a more specific
diagnosis cannot be rendered. For example, unlike oral candidiasis the white lesion cannot be
removed by rubbing the oral mucosal surface. Leukoplakia is often associated with hyperkeratosis
and may or may not show dysplastic squamous epithelium. On average, 2-6% represent either
dysplasia or early invasive squamous cell carcinoma.
Hairy leukoplakia (choice A) is an oral infective lesion seen almost exclusively in patients with HIV
infection. It is a fluffy, white, hyperkeratotic lesion in which a destructive piling up of keratotic squames
is seen. Hairy leukoplakia is associated with viral infection, mostly EBV, HPV, or HIV. It does not
progress to cancer.
Linea alba (choice C) is a linear white area on the buccal mucosa formed from hyperkeratosis due to
occlusion, and may be excessive in those that clench or grind.
Oral thrush (choice D) is a superficial candidal infection, typically occurring in immunosuppressed or
very young patients. Thrush is an infectious, non-neoplastic disease.
Squamous papilloma (choice E) is a benign human papillomavirus (HPV) infection of the oral
mucosa. Typically associated with HPV genotypes 6 and 11, squamous papilloma only rarely
progresses to squamous carcinoma.
45. Which of the following is present in a elderly person that has thin arms and legs, a swollen
abdomen, red tongue, dry, thin, and slightly yellow skin, gynecomastia, testicular atrophy,
multiple spider angiomas, tremor, yellow discoloration of sclera, and short-term memory loss?
A. Bronchogenic carcinoma
B. Colon carcinoma
C. Congestive heart failure
D. Type 2 diabetes mellitus
E. Hepatic cirrhosis

22

The correct answer is E. The physical examination is typical for an alcoholic with advanced hepatic
cirrhosis. It is important to recognize these symptoms, as these patients are notorious for
"underestimating" and even denying their alcohol use.
Bronchogenic carcinoma (choice A) typically presents with cough or respiratory changes, but can
present with mass effects in the chest or involvement of mediastinal nerves or vessels.
Colon cancer (choice B) typically presents with changes in the stool or bowel habits.
Congestive heart failure (choice C) is typically heralded by shortness of breath or peripheral edema,
or both.
Type 2 diabetic patients (choice D) are typically obese and present with usual signs and symptoms of
diabetes, such as polyuria, polydipsia, and polyphagia (but not ketoacidosis).
46. Which of the following laboratory tests is performed following a head injury so as to
differentiate between nephrogenic and neurogenic diabetes insipidus?
A. Creatinine in a 24-hour urine collection
B. Urine osmolality after vasopressin administration
C. Serum renin after infusion of hypotonic saline
D. Serum sodium after infusion of hypertonic saline
The correct answer is B. Diabetes insipidus is characterized by the excretion of abnormally large
volumes of dilute urine (polyuria) with a commensurate increase in fluid intake (polydipsia). There are
two types, neurogenic and nephrogenic. The most common type is neurogenic diabetes insipidus,
which is caused by inadequate secretion of antidiuretic hormone (ADH) or vasopressin. In the
absence of ADH, a brisk diuresis results; the osmolality of the urine may be extremely low. Many with
neurogenic (central) diabetes insipidus have a history of head trauma, brain tumors, or brain
infections that damage the hypothalamus or neurohypophysis. Nephrogenic diabetes insipidus, which
presents in much the same way, can be seen in association with certain renal diseases caused by a
defect in the renal V2 vasopressin receptor, Gs protein, or other steps in the formation of cyclic AMP.
Plasma levels of vasopressin are usually increased, because of the hyperosmolarity of the serum.
Water deprivation will fail to increase urine osmolarity in neurogenic and nephrogenic diabetes
insipidus. Because V2 receptors are functional in neurogenic diabetes insipidus, however,
administration of exogenous ADH will still concentrate the urine, whereas this would be ineffective in
nephrogenic diabetes insipidus.
Measurement of creatinine (choice A) in a 24-hour urine collection, coupled with plasma creatinine
values and the urine flow rate can be used to approximate glomerular filtration rate.
Renin secretion would be inhibited by infusion of hypotonic saline (choice C) in neurogenic and
nephrogenic diabetes.
Serum sodium (choice D) would rise with infusion of hypertonic saline in either type of diabetes
insipidus.
47. Which of the following laboratory results would be expected in a child with 2 standard
deviations below the expected mean height, delayed bone maturation, goiter, and a point
mutation in the thyroid hormone receptor?
A. Decreased radioactive iodine uptake test
B. Increased plasma T4 concentration
C. Decreased plasma TSH concentration
D. Increased basal metabolic rate
The correct answer is B. Generalized resistance to thyroid hormone is a rare genetic abnormality
(Refetoff syndrome). It results from mutations of the thyroid hormone receptor gene. Depending on
the severity of the disorder, patients may be only mildly affected or may exhibit striking hypothyroid-

23

like symptoms including decreased basal metabolic rate (BMR) (not increased, choice D). Growth
can be stunted, there may be deaf mutism, and attention span may be short. Because the thyroid
hormone resistance is generalized, the normal negative feedback effects of T4 and T3 at the
hypothalamus and pituitary are also deficient. This would lead to an increased plasma TSH
concentration (not decreased, choice C). Because of the increase in plasma TSH, iodine trapping by
the thyroid follicular cells will be increased, leading to an increase in radioactive iodine uptake test
(RAIU) (not decreased, choice A) and an increase in serum T4.
48. What form of arthritis is present in an elderly woman that complains of ill-fitting dentures,
dry eyes and mouth, and severe arthritic pain?
A. Gonococcal arthritis
B. Gouty arthritis
C. Osteoarthritis
D. Pseudogout
E. Rheumatoid arthritis
The correct answer is E. Dry eyes and mouth in an elderly woman are probably caused by Sjogren's
syndrome, an autoimmune attack on the salivary and tear glands. Sjogren's syndrome is associated,
in some cases, with rheumatoid arthritis.
Gonococcal arthritis (choice A) is a systemic complication of gonorrhea, typically observed in young
women who have had a recent sexual encounter.
Gout (choice B) and pseudogout (choice D) can cause arthritis secondary to crystal deposition in
joints.
Osteoarthritis (choice C) seems to be caused by repetitive low-level trauma to joints.
49. A 7-year-old boy is not having any behavioral problems in school, but he is having a great
deal of trouble paying attention in class. He has normal IQ and cognitive function, but
occasionally asks that questions be repeated after staring blankly into space for a few
seconds. Which of the following disorders most likely accounts for these symptoms?
A. Absence seizures
B. Attention deficit hyperactivity disorder
C. Schizophrenia with childhood onset
D. Parkinson disease
The correct answer is A. The patient described in this question is suffering from absence seizures
that typically appear during childhood between the ages of 5 and 7 years. In absence seizures, the
patient has many episodes of brief disruption of consciousness throughout the day. These seizures
are not accompanied by the convulsions and complete loss of consciousness often associated with
epilepsy, but rather by the absence of motor or sensory symptoms (hence the blank look on the
patient's face).
Children with attention deficit hyperactivity disorder (choice B) also have a limited attention span and
normal intelligence. They also exhibit hyperactivity, impulsiveness, emotional lability, and irritability,
however, which lead to behavioral problems in school.
Schizophrenia with childhood onset (choice C) is rare. Children with this disorder demonstrate normal
intelligence and may show a limited attention span. These children also manifest the same psychiatric
symptoms seen in adult-onset schizophrenics, however, including hallucinations, delusions, abnormal
affect, and limited social skills that lead to behavioral problems in school.
Parkinson disease (choice D) most commonly onsets between ages 45-65 years. It is associated with
any combination of tremor, rigidity, bradykinesia, and progressive postural instability.

24

50. Which of the following hormones is most likely secreted by mass in the head of the
pancreas in a person with multiple intractable gastric and duodenal peptic ulcers and marked
elevation of gastric acid secretion?
A. Calcitonin
B. Epinephrine
C. Gastrin
D. Parathormone
E. Vasoactive intestinal peptide
The correct answer is C.This is a description of Zollinger-Ellison syndrome in which a duodenal or
pancreatic gastrin-secreting endocrine tumor causes hypersecretion of gastric acid. Two thirds of
these tumors are malignant. One third are related to MEN I that also causes parathyroid
hyperplasia/adenomas and pituitary adenomas.
Calcitonin (choice A) is usually secreted by medullary carcinoma of the thyroid.
Epinephrine (choice B) is secreted in pheochromocytoma.
Pancreatic endocrine tumors may also secrete vasoactive intestinal peptide (VIP, choice E),
parathyroid hormone (choice D), parathyroid hormone-related peptide, insulin, glucagon,
somatostatin, ACTH, or growth hormone-releasing hormone (GHRH). Secretion of these hormones is
less common than secretion of gastrin and is not specifically suggested by the peptic ulcer history.
51. Which of the following findings would MOST likely also be present with a history of
scleroderma that has a near absence of smooth muscle peristalsis and lower esophageal
sphincter tone?
A. Anemia
B. Atrophic glossitis
C. Hourglass-shaped stomach on barium swallow
D. Massively dilated colon
E. Muscle replaced with connective tissue
The correct answer is E. Near complete absence of muscle tone and peristalsis is characteristic of
involvement of the esophagus with scleroderma, which causes replacement of muscle by dense
connective tissue. Similar changes in the dermis cause the skin to be thickened. The thickened, shiny
skin of the hands may cause them to resemble claws with telangiectasia and areas of increased
pigmentation and depigmentation. Systemic features include dysphagia and hypomotility of the
gastrointestinal tract.
Anemia (choice A) and atrophic glossitis (choice B) are associated with esophageal webs in
Plummer-Vinson syndrome.
An hourglass-shaped stomach within the thoracic cavity (choice C) is a feature of a sliding hiatal
hernia. A massively dilated esophagus (megaesophagus) can be caused by Chagas' disease, a
trypanosomal disease that can also cause massive dilation of the colon (choice D).
52. Which of the following is the MOST significant risk factor for breast cancer in a 40-year-old
woman?
A. Breast-feeding after pregnancy
B. Caffeine consumption
C. Cigarette smoking
D. Family history
E. Nulliparity

25

The correct answer is D. Positive family history is definitively the most significant risk factor for
developing breast cancer among the ones mentioned. Approximately 5-10% of cases are attributable
to inheritance of autosomal dominant genes. Most hereditary cases of breast cancer are caused by
two genes, BRCA1 and BRCA2, which are tumor suppressor genes probably involved in DNA repair.
Note, however, that 80-90% of women with breast cancer do not have a positive family history.
Breast-feeding after pregnancy (choice A) is not considered a risk factor for breast cancer. Although
research is ongoing, it may be that the breast cancer risk in women who breast-feed is decreased.
There is no convincing evidence linking caffeine consumption or cigarette smoking (choices B and C)
to breast cancer.
Breast cancer is more frequent in nulliparous women than in multiparous women. Nulliparity (choice
E) is therefore a risk factor, but its role is considered less important than family history. Similar to a
long duration of reproductive life and late age at first intercourse, nulliparity (never given birth to a
child) seems to increase breast cancer risk by increasing exposure to endogenous estrogen during
the menstrual cycle.
53. A child with cystic fibrosis may have a history of meconium ileus, chronic cough, frequent
respiratory tract infections, and bulky, foul-smelling stools and _______ ?
A. Cystinuria
B. Hypoglycemia
C. Iron deficiency anemia
D. Laryngeal papillomas
E. Pancreatic insufficiency
The correct answer is E. This is a descriptionof cystic fibrosis. In this disorder, an abnormality of
chloride channels causes all exocrine secretions to be more viscous than normal. Because of
pancreatic insufficiency, the pancreatic secretion of digestive enzymes is often severely impaired, with
consequent steatorrhea and deficiency of fat-soluble vitamins, including vitamin A.
Cystinuria (choice A) is a common disorder in which a defective transporter for dibasic amino acids
(cystine, ornithine, lysine, arginine) leads to saturation of the urine with cystine, which is not very
soluble in urine, and precipitates out to form stones.
Hypoglycemia (choice B) is not a prominent feature of children with cystic fibrosis who are on a
normal diet. Hyperglycemia may occur late in the course of the disease.
Iron deficiency anemia (choice C) is not found with any regularity in children with cystic fibrosis.
Laryngeal papillomas (choice D) are common lesions of the larynx. Unlike in the oral cavity, when
found in the larynx they are likely to be symptomatic, with hoarseness that progresses to stridor over
weeks to months.
54. What are multiple long-standing tan to brownish lesions on the face and back of older
individuals that are slightly raised with a rough surface,typically 0.5-1.5 cm in diameter, and
that show an ability to be peeled off?
A. Eczema
B. Melanoma
C. Psoriasis
D. Seborrheic keratoses
E. Verruca vulgaris
The correct answer is D.Chronic eczema (choice A) produces dry, thick, and sometimes discolored
skin.

26

Melanomas (choice B) characteristically look like dark moles with irregular margins and variations in
the degree of pigmentation.
Psoriasis (choice C) produces erythematous plaques with a silvery scale.
Verruca vulgaris (choice E), the common wart, produces verrucous papules that are most commonly
found on the hands. The face and back would be unusual sites.
55. Which of the following laboratory findings would be MOST likely in a teenage girl with
delayed puberty, short stature, webbed neck, low-set ears, fish-like mouth, and ptosis?
A. Decreased plasma growth hormone
B. Decreased plasma growth hormone
C. Increased plasma follicle stimulating hormone
D. Increased plasma inhibin
E. Increased plasma estrogen
The correct answer is C. This is a description of Turner's syndrome which is a result of a 45, XO
karyotype and which is characterized by ovarian dysgenesis and a variety of somatic abnormalities
including micrognathia, a fish-like mouth, a shield chest, low-set ears, ptosis, and a webbed neck.
Other findings can include coarctation of the aorta, hypertension, and renal abnormalities. Short
stature is invariably present; the cause is not known because plasma levels of growth hormone
(choice A) and thyroid hormone (choice B) are typically not decreased. Clinical studies have shown,
however, that injections of human growth hormone can increase the final height. The ovaries are
usually streak-like and exhibit only fibrous stroma which leads to decreased secretion of estrogen (not
increased, choice E) and inhibin (not increased, choice D) and persistent infantilism. Plasma levels
of FSH are markedly increased because of the lack of feedback inhibition by ovarian secretions.
56. Which of the following presents with a painful, swollen knee, geographic tongue, and
resolving psoriasiform rash on the palms and soles, as well as urethritis and conjunctivitis?
A. Gout
B. Bacillary arthritis
C. Osteoarthritis
D. Osteoporosis
E. Rheumatoid arthritis
The correct answer is E. This is an example of Reiter's syndrome (urethritis, cervicitis in females,
conjunctivitis, arthritis, and mucocutaneous lesions). The term keratoderma blennorrhagicum is used
for the lesions of the palms and soles that resemble pustular psoriasis. Geographic tongue is caused
by erosions histologically characterized by prominent spongiform pustules. Lesions of the glans penis
(balanitis circinata) may also be present. Reiter's syndrome is associated with chlamydial infection in
70% (or more) of cases. The arthritis that can be seen in the disease is an inflammatory arthritis
similar to that of rheumatoid arthritis.
Gout (choice A) is a crystal arthritis; other examples of crystal arthritis include apatite-associated
arthritis and pseudogout caused by calcium pyrophosphate dihydrate crystals.
If you believed the patient has gonococcal arthritis (a septic arthritis), then gram-negative bacillary
arthritis (choice B) would be a good choice. Neisseria gonorrhoeae is not noted for causing
conjunctivitis, however, which should have steered you toward Reiter's syndrome.
Osteoarthritis (choice C) is a degenerative disorder without systemic manifestations. X-ray findings
typically show narrowed joint space, osteophytes, and an increased density of subchondral bone.
Osteoporosis (choice D) can be asymptomatic to severe backache from vertebral fractures.
Demineralization of the spine, hip, and pelvis is common.

27

57. Which of the following can be noted in a rheumatoid arthritic that has acute caries and
chronic parotid gland enlargement?
A. Oral squamous cell carcinoma
B. Polyarteritis nodosa
C. Sjogren's syndrome
D. Systemic lupus erythematosus
E. Thyrotoxicosis
The correct answer is C. Rheumatoid arthritis can coexist with a variety of autoimmune diseases
(including those listed in the answers), but is most frequently associated with Sjogren's syndrome.
Sjogren's syndrome classically presents with keratoconjunctivitis (dry eyes) and xerostomia (dry
mouth, often resulting in dental caries and fissures in the oral mucosa). These symptoms are caused
by autoimmune involvement with subsequent scarring of the salivary and lacrimal glands. Parotid
gland enlargement is common and may be chronic or relapsing and develops in approximately one
third of patients with Sjogren's syndrome. Vasculitis, Raynaud's phenomenon, hyperviscosity
syndrome, and peripheral neuropathy may also be seen.
The development of oral squamous cell carcinoma (choice A) is not related to the presence of dental
caries.
Polyarteritis nodosa (choice B) is a systemic necrotizing vasculitis. There is typically present with lowgrade fever, weakness, and weight loss. Abdominal pain, hematuria, renal failure, hypertension, and
leukocytosis may occur.
Systemic lupus erythematosus (choice D) is an autoimmune disease characterized by vasculitis,
rash, renal disease, hemolytic anemia, and neurologic disturbances.
Thyrotoxicosis (choice E) is not related to the development of dental caries.
58. Which of the following is the cause of joint pain involving the large leg joints in a young
adult accompanied by diarrhea that is often bloody?
A. Amebic colitis
B. Chronic appendicitis
C. Diverticulosis
D. Pseudomembranous colitis
E. Ulcerative colitis
The correct answer is E. Several gastrointestinal diseases are associated with rheumatologic
complaints. The most frequent of these are the chronic inflammatory bowel diseases, ulcerative
colitis, and Crohn's disease, which can be associated with sacroiliitis or lower limb arthritis. Other
gastrointestinal diseases associated with arthropathy include bypass surgery, Whipple's disease,
Behcet's syndrome, and celiac disease. Ulcerative colitis is accompanied by bloody diarrhea, lower
abdominal cramps and fecal urgency, anemia, low serum albumin, and negative stool culture.
Sigmoidoscopy is the key to diagnosis.
Amebic colitis (choice A) is caused by ingestion of infectious cysts (typically from Entamoeba
histolytica). Symptoms include abdominal pain and diarrhea; malaise and weight loss may occur.
Cecal amebiasis can resemble acute appendicitis.
Chronic appendicitis (choice B) may be asymptomatic or cause poorly defined abdominal pain.
Diverticulosis (choice C) is usually a disease of older adults. It is often asymptomatic unless
inflammation supervenes.
Pseudomembranous colitis (choice D) is a severe form of diarrhea usually seen in the setting of prior
antibiotic use. The causative organism is almost always Clostridium difficile.

28

59. Two diabetic patients are seen by a clinician. The first patient Comparing an older person
just diagnosed with diabetes that was identified by the presence of hyperglycemia and
diabetes in a young person who had previously presented with polyuria and polydipsia, the
younger person is more likely to:
A. be obese
B. become euglycemic with oral hypoglycemic agents
C. develop ketoacidosis
D. have relatively high endogenous insulin levels
E. have tissue insulin insensitivity
The correct answer is C. The younger person would have type 1 (used to be called juvenile onset)
diabetes mellitus, whereas the older person probably has type 2 (used to be called maturity onset)
diabetes mellitus. These two types of diabetes differ in many respects. Ketoacidosis is more apt to
develop in type 1, although it can occur in patients with type 2 after several years. As type 2
progresses, the amount of insulin secretion decreases more like type 1; therefore, the incidence of
ketoacidosis increases.
Type 2 tend to be obese (choice A), whereas type 1 are often thin.
Type 1 is usually apparently caused by viral or immune destruction of beta cells, whereas type 2 is
apparently usually caused by increased resistance to insulin; consequently the older diabetic rather
than the younger diabetic is more likely to have relatively high endogenous levels of insulin (choice
B).
Type 2 can often be controlled with oral hypoglycemic agents (choice D), whereas IDDM generally
require insulin. Note that some NIDDM also may require insulin as the disease evolves.
Tissue insulin insensitivity (choice E) or insulin resistance are the primary etiologic factors causing
NIDDM.
60. Which of the following processes is related to a asymptomatic perimenopausal woman
demonstrating bony swellings in the distal interpharyngeal joints; however no inflammation is
apparent and the proximal interphalangeal joints, hands, and wrists are not involved?
A. Autoantibody formation
B. Bacterial infection
C. Crystal deposition
D. Joint trauma
E. Viral infection
The correct answer is D. The disease is osteoarthritis and the bony swellings are called Heberden's
nodes, which may or may not be symptomatic. Osteoarthritis is believed to be related to repetitive
joint trauma. Osteoarthritis is a degenerative disorder with systemic manifestations. Pain is relieved by
rest. Radiographic findings include narrowed joint space, osteophyte, increased density of
subchondral bone, and bony cysts.
Autoantibodies (choice A) are important in rheumatoid arthritis.
Bacterial infections (choice B) cause septic arthritis.
Crystal deposition (choice C) is important in gout and pseudogout.
Viral infections (choice E) can cause transient arthralgias and arthritis.

29

61. Which of the following is present with a complaint of tiredness and muscle weakness, as
well as increased blood pressure, plasma sodium and plasma aldosterone, and decreased
plasma potassium, hematocrit, and plasma renin activity?
A. Addison's disease
B. Conn's syndrome
C. Cushing's syndrome
D. Hypothyroidism
E. Pheochromocytoma
The correct answer is B. Conn's syndrome is hyperaldosteronism caused by a hypersecreting
adrenal adenoma. Conn's syndrome is one of several endocrine causes of hypertension. The
hypertension is caused by volume expansion secondary to increased renal sodium and water
retention. The excessive aldosterone also causes increased renal excretion of potassium leading to
hypokalemia, which can explain the tiredness and muscle weakness. The decreased hematocrit is
also consistent with blood volume expansion. The increase in blood volume, blood pressure, and
plasma sodium all contribute to the suppression of renin secretion.
Addison's disease (choice A) is primary adrenal insufficiency and is characterized by decreased
secretion of cortisol and aldosterone. It is accompanied by hyponatremia and hyperkalemia.
Patients with Cushing's syndrome (choice C) can also be hypertensive. This may be, in part, to an
increased permissive action of cortisol on catecholamine-mediated vascular tone. In addition, cortisol
in high levels can have significant mineralocorticoid activity and can produce sodium retention and
potassium loss. The hypertension, sodium retention, and hypervolemia tend to suppress renin
secretion and may decrease aldosterone secretion. Cushing's syndrome is also characterized by a
redistribution of body fat, producing central obesity and a buffalo hump with the extremities being
thinned.
Hypothyroidism (choice D) is associated with weakness, fatigue, cold intolerance, constipation,
weight changes, and hoarseness. Dry skin, bradycardia, and a delayed return of deep tendon reflexes
are also present. This condition is caused by low T4 levels. TSH levels are generally elevated in
patients with primary hypothyroidism.
Pheochromocytoma (choice E) is another endocrine cause of hypertension. It is caused by a
catecholamine-secreting tumor. The hypertension is caused by excessive vasoconstriction and
increased cardiac output.
62. Which of the following additional findings would MOST likely be present in a child with
freckles all over his body, including the buccal mucosa, lips, palms, soles, and skin not
exposed to sun?
A. Colonic polyps
B. Desmoid tumors
C. Epidermoid cysts
D. Osteomas of the jaw
E. Pigmented ocular fundus
The correct answer is A. The widespread freckles (spots of melanin pigmentation) described are
associated with hamartomatous colonic polyps in the autosomal dominant condition Peutz-Jeghers
syndrome. The polyps in Peutz-Jeghers syndrome do not progress to colon cancer. Peutz-Jeghers
syndrome is associated with an increased potential to develop carcinomas of the pancreas, breast,
ovary, uterus, and lung. The hamartomas can become large enough to cause obstruction, bleeding, or
intussusception.
All of the other features listed are components of Gardner's syndrome, a variant of familial
adenomatous polyposis syndrome that carries a greatly increased risk for colon cancer.

30

63. In a PPD skin test that shows >10 mm of induration, which of the following type of reactive
cells would be found?
A. B lymphocyte
B. Eosinophil
C. Mast cell
D. Neutrophil
E. T lymphocyte
The correct answer is E. The CD4+ population of T lymphocytes, specifically TH1 cells, are
responsible for the delayed hypersensitivity reaction seen with a skin test in a previously sensitized
patient. This response to a PPD test is typically seen in reactivation or adult-type tuberculosis. A 10mm reaction on a skin test is considered positive if the person is in a high-incidence group (foreignborn, medically underserved, low-income population, or resident of a long-term care facility).
B lymphocytes (choice A) are involved in humoral immune reactions. Antibody production is not a
feature of tuberculosis hypersensitivity.
Eosinophils (choice B) are important in type I hypersensitivity reactions and in immune-mediated
responses to parasitic infections (ADCC). They are not associated with tuberculosis hypersensitivity.
Mast cells (choice C) are tissue cells that are involved in type I hypersensitivity reactions. They have
surface receptors for the Fc fragment of the IgE molecule.
Neutrophils (choice D) are associated with acute inflammatory reactions. They are considered a
"nonspecific" cell in that they do not interact with an antigen. The neutrophil is not a classic cell type
seen in tuberculosis hypersensitivity.
64. Which of the following is the MAJOR chemotactic factor responsible for attracting
neutrophils?
A. C3b
B. C5a
C. IgM
D. IL-2
E. Lysozyme
The correct answer is B. In active inflammation, the complement system has been activated and
C5a is being produced, which is a strong chemoattractant to neutrophils and other phagocytic cells.
C3b (choice A) is an excellent opsonin of pathogenic organisms; when an organism is coated with
C3b, it is more easily phagocytized. C3b is formed by way of the classic and alternative complement
pathways.
IgM (choice C) is the first immunoglobulin produced in the primary immune response. IgM cannot
cross the placenta, but it is a powerful activator of complement; elevated levels in the newborn are
associated with an acute infection with a pathogen.
IL-2 (choice D) is a powerful interleukin that stimulates T helper 1 cells. It also stimulates natural killer
cells and T cytotoxic CD8 lymphocytes, but is not chemotactic for neutrophils.
Lysozyme (choice E) is a material present in tears, mucus, vaginal secretions, and other body fluids.
It is active against the peptidoglycan of bacterial cell walls, splitting the backbone structure of the
peptidoglycan (N-acetylglucosamine and N-acetyl muramic acid polymers).

31

65. Which of the following is characterized by a young boy with failure to thrive, recurrent
seizures, hepatomegaly, renomegaly, severe hypoglycemia, hyperlipidemia, lactic acidosis,
and ketosis?
A. Gaucher's disease
B. McArdle's disease
C. Niemann-Pick disease
D. Pompe's disease
E. von Gierke's disease
The correct answer is E. von Gierke's disease is a glycogen storage disease caused by a deficiency
of glucose-6-phosphatase. It typically presents with neonatal hypoglycemia, hyperlipidemia, lactic
acidosis, and ketosis. Failure to thrive is common in early life; convulsions may occur because of
profound hypoglycemia. The glycogen accumulation in von Gierke's disease occurs primarily in the
liver and kidneys, accounting for the enlargement of these organs. Gout may develop later because of
the derangement of glucose metabolism.
Even if you do not remember all of the details of the presentation of these genetic diseases, you
should be able to narrow the choices:
Gaucher's disease (choice A) and Niemann-Pick disease (choice C) are lipid storage diseases and
would not be expected to produce hypoglycemia.
The other diseases are glycogen storage diseases, but McArdle's disease (choice B) and Pompe's
(choice D) disease affect muscle rather than liver and would not be expected to produce profound
hypoglycemia, because the liver is the major source for blood glucose.
66. Which of the following disorders is associated with the same oncogenic virus that is the
likely cause of nonkeratinizing squamous cell carcinoma of the nose, pharynx, and sinuses?
A. Adult T-cell leukemia
B. Burkitt's lymphoma
C. Cervical carcinoma
D. Hepatocellular carcinoma
E. Kaposi's sarcoma
The correct answer is B. The cancer described is nasopharyngeal carcinoma, which is associated
with Epstein Barr virus (EBV). This virus is also associated with the African form of Burkitt's lymphoma
that characteristically involves the jaw.
HTLV-1, or human T-lymphocyte virus, is associated with adult T-cell leukemia (choice A).
HPV, or human papillomavirus, is associated with cervical carcinoma (choice C), penile carcinoma,
and anal carcinoma. Hepatitis B virus (HBV) is associated with hepatocellular carcinoma (choice D).
HHV 8, a member of the herpes family, is associated with Kaposi's sarcoma (choice E).
67. What is at risk to develop when there is substernal burning pain exacerbated by large
meals, cigarettes, and caffeine, and laying on the back, especially when sleeping at night, but
improved with antacids?
A. Cardiac ischemia
B. Columnar metaplasia of the distal esophagus
C. Mallory-Weiss lesion in the esophagus
D. Squamous cell carcinoma
E. Zenker's diverticulum

32

The correct answer is B. This is a description of reflux esophagitis, a condition in which the lower
esophageal sphincter (LES) does not adequately prevent acidic gastric contents from refluxing back
into the distal esophagus. Most commonly, there is a defect in the LES mechanism itself, in addition to
secondary causes such as pregnancy (from increased abdominal pressure) and some medications.
Symptoms can mimic cardiac chest pain and must be carefully evaluated for complications including
esophageal strictures, ulcerations, laryngitis, pulmonary aspiration, and Barrett's esophagus
(columnar metaplasia of the distal esophagus). Barrett's esophagus is considered a premalignant
state, with roughly a 40-fold increase in the incidence of esophageal adenocarcinoma. The normal
squamous epithelium of the esophagus transforms into columnar epithelium similar to gastric
epithelium as a result of recurrent reflux of acidic gastric contents.
Anginal pain, signaling cardiac ischemia (choice A), is generally not burning in nature and is not
relieved by antacids.
Mallory-Weiss lesions (choice C) are actual tears of the epithelia of the proximal stomach or distal
esophagus as a result of retching, nausea, and vomiting (seen in patients with anorexia and
alcoholism).
The incidence of pure squamous cell carcinoma (choice D) is not increased by acid reflux disease.
Zenker's diverticula (choice E), the most common of esophageal diverticuli, are not true diverticuli,
but only mucosal herniations that can cause obstructive symptomatology.
68. Which of the following in a middle-aged person can cause a persistent, long term
productive cough with a past history of smoking and with the recent presence of a green
sputum and fever?
A. Bronchogenic carcinoma
B. Chronic bronchitis with superimposed infection
C. Cystic fibrosis
D. Emphysema
E. Pulmonary tuberculosis
The correct answer is B. Chronic bronchitis requires the presence of chronic productive cough over
at least 3 months of the year for 2 successive years. The green productive sputum and fever suggest
that there is an acute infection superimposed on chronic bronchitis (history of cigarette smoking,
history of excessive mucus production over many years). Histologically, the mucus-producing glands
in the bronchi would show hyperplasia and hypertrophy and extend to a greater depth in the bronchial
wall, resulting in a higher Reid index (ratio of thickness of mucus gland to thickness of bronchial wall).
Although smoking is involved and there is an increased risk for bronchogenic carcinoma (choice A),
this is unlikely. Hemoptysis and weight loss would also be present if cancer was involved. Cystic
fibrosis (choice C) presents earlier in life and may be associated with severe production of mucus,
especially if bronchiectasis supervenes. Age and the relative late onset of disease preclude this.
Emphysema may be involved (choice D), as chronic bronchitis and emphysema are often coexistent.
Although pure emphysema might cause dyspnea, it would not, however, be associated with a fever or
a productive cough. Pulmonary tuberculosis (choice E) would typically present with hemoptysis rather
than abundant green sputum. Weight loss, night anorexia, malaise, and weakness may also be
present.
69. What is noted histologically with extensive pruritic wheals following ingestion of certain
seafoods?
A. Dilated superficial lymphatic channels
B. Granular complement and IgG at the dermal/epidermal junction
C. Microscopic blisters
D. Munro microabscesses
E. Solar elastosis

33

The correct answer is A. Urticaria (hives) are pruritic wheals that form after mast cells degranulate
and trigger localized dermal edema with dilated superficial lymphatic channels with an allergic
response (commonly to certain types of shellfish). The mast cell degranulation is sometimes but not
always triggered by IgE-antigen interactions.
Granular complement and IgG deposition at the dermal/epidermal junction (choice B) is a
characteristic of systemic lupus erythematosus.
Microscopic blisters (choice C) are a characteristic of dermatitis herpetiformis.
Munro abscesses (choice D) are a characteristic of psoriasis.
Solar elastosis (choice E) is found in actinic keratoses.
70. Which will increase in the plasma and can cause an acid-base disorder in an uncontrolled
type 1 diabetic having breath that smells like nail polish remover?
A. Formic acid
B. Glycolic acid
C. Ketoacids
D. Lactic acid
E. Oxalic acid
The correct answer is C. Ketoacidosis can easily occur in patients with type 1 diabetes mellitus
secondary to the body's inability to produce insulin. This is metabolic acidosis (low pH, low HCO3-).
The decrease in arterial CO2 is a compensatory response to the acidosis. The smell of acetone
(which is the primary solvent of nail polish) on the breath of a diabetic suggests ketoacidosis and
uncontrolled type 1 diabetes mellitus. Because acetone is highly volatile, it is excreted mainly by the
lungs. Other common causes of ketoacidosis include starvation and the acute drinking-vomiting binge
of the alcoholic.
Plasma levels of formic acid (choice A) increase when methanol is ingested.
Plasma levels of glycolic acid (choice B) and oxalic acid (choice E) increase when ethylene glycol is
ingested.
Because lactic acid (choice D) is a product of anaerobic metabolism, its rate of production is
increased by decreases in tissue blood flow and oxygen delivery and when oxygen use is impaired.
Lactic acidosis commonly occurs in circulatory failure. Although diabetes mellitus can cause lactic
acidosis, it is far less common than ketoacidosis.
71. A 25-year-old man experiences the gradual onset of intermittent diarrhea that over years
progresses to severe diarrhea, alternating with constipation, rectal bleeding, and passage of
mucus. On examination, the abdomen is tender over the colon, and stool examination fails to
reveal parasites. Colonoscopy demonstrates inflammation limited to the rectum, with no
higher lesions. Which of the following diseases is most consistent with this history?
A. Celiac disease
B. Pseudomembranous colitis
C. Traveler's diarrhea
D. Ulcerative colitis
E. Whipple disease
The correct answer is B. The presentation is classic for Crohn disease and ulcerative colitis. Family
members of those afflicted show increased incidence of ulcerative colitis and Crohn disease, evidence
that these two diseases are actually different ends of the same spectrum. In contrast to Crohn disease
in which the lesions may be patchy and involve the distal ileum and even the esophagus, ulcerative
colitis lesions involve the rectum and may extend continuously proximally for varying distances up to
the cecum and very distal end of the ileum.

34

Celiac disease (choice A) is a small intestinal disease related to gluten intolerance. Flattening of villi,
elongated crypts, and marked inflammation in the lamina propria are noted histologically.
Pseudomembranous colitis (choice B), or antibiotic induced colitis, is commonly seen following
antibiotic therapy, especially with clindamicin. This condition is treated with metronidazole or
vancomycin and is caused by the toxin produced by Clostridium difficile.
Traveler's diarrhea (choice C) is generally seen in patients traveling from one country to another. It
has an acute onset and is generally associated with severe diarrhea.
Whipple disease (choice E) is an intestinal diarrheal disease that has been shown to be caused by a
bacterial infection.
72. What disease state can be present when focal calcification is found above the sella turcica,
and there is visual field abnormalities and diabetes insipidus, while needle biopsy shows
tissue resembling tooth enamel?
A. Craniopharyngioma
B. Glioblastoma multiforme
C. Large pituitary adenoma
D. Medulloblastoma
E. Pituitary microadenoma
The correct answer is A. The tumor described is a craniopharyngioma, alternatively known as an
adamantinoma or ameloblastoma. Craniopharyngiomas may arise in, or more commonly above, the
sella turcica. The histologic pattern recapitulates the enamel organ of the tooth, with nests or cords of
stratified squamous or columnar epithelium embedded in a loose fibrous stroma. Calcification (and
even metaplastic bone formation) is common in these benign tumors that are believed to arise from
vestigial remnants of Rathke's pouch.
Glioblastoma multiforme (choice B) characteristically shows at least some enlarged cells with bizarre
nuclei.
Large pituitary adenomas (choice C) contain nests of uniform glandular cells.
Medulloblastoma (choice D) is made of small basophilic cells with relatively large nuclei for their size.
Pituitary microadenomas (choice E) contain nests of uniform glandular cells.
73. Which of the following locations listed is the more common primary site for oral squamous
cell carcinoma?
A. Ventral tongue surface
B. Buccal mucosa
C. Floor of mouth
D. Palate
E. Tip of tongue
The correct answer is C. Oral cancers are highly associated with alcohol and tobacco use,
excessive sun exposure, and HPV-16 (human papilloma virus type 16, which is also found in cervical
carcinomas) is found in nearly half of all oral cancers. The pattern of tumor development possibly
reflects exposure-related or tissue classification factors. In decreasing order of frequency in regards to
risk of oral cancer, the following can be stated: (1) floor of the mouth (choice C); (2) tip of the tongue
(choice E); (3) hard palate (choice D); (4) ventral tongue surface (choice A); and (5) buccal mucosa
(choice B).
It is important to note that a systematic intraoral examination, including the entire oral cavity and lips,
with particular emphasis on the lateral tongue, floor of the mouth, and soft palate complex, as well as
palpation of the neck for enlarged hard lymph nodes should be part of any general head and neck

35

examination. This is especially true for patients over the age of 45 years who smoke tobacco or drink
alcohol excessively or are exposed to excessive amounts of sun.
74. Which of the following conditions explains the pain in the upper portion of the neck on
swallowing, with regurgitation of undigested food shortly after eating?
A. Mallory-Weiss tears
B. Plummer-Vinson syndrome
C. Schatzki rings
D. Traction diverticula
E. Zenker's diverticulum
The correct answer is E. This is a description of Zenker's diverticulum, which is a false diverticulum
formed by herniation of the mucosa at a point of weakness at the junction of the pharynx and
esophagus in the posterior hypopharyngeal wall. The cause is believed to be loss of elasticity of the
upper esophageal sphincter. Zenker's diverticulum is also associated with halitosis, and if the
diverticulum fills completely with food, it can cause dysphagia or obstruction of the esophagus.
Mallory-Weiss tears (choice A) are mucosal tears at the gastroesophageal junction secondary to
repeated, forceful vomiting. They are often seen in alcoholics and result in the appearance of blood in
the emesis.
Plummer-Vinson syndrome (choice B) is the triad of dysphagia (caused by esophageal webs in the
upper esophagus), atrophic glossitis, and iron-deficiency anemia.
Schatzki rings (choice C) are mucosal rings found in the distal esophagus at the squamocolumnar
junction.
In contrast to a Zenker's diverticulum, the usually asymptomatic traction diverticula (choice D) are
true diverticula involving all of the layers of the esophagus. They are typically caused by adherence of
the esophagus to a scarred mediastinal structure.
75. Which of the following findings would suggest pemphigus vulgaris as opposed to bullous
pemphigoid?
A. Eosinophils within bullae
B. Negative Nikolsky's sign
C. IgG autoantibody activity
D. Oral mucosal lesions
The correct answer is D. Pemphigus vulgaris is associated with chronic, severe bullae formation on
the skin and oral mucosa. Mucosal lesions are extremely rare in bullous pemphigoid. This can be
used clinically to guide therapy, although a skin biopsy should be taken to confirm the diagnosis. Both
diseases are characterized by formation of tender bullae that can rupture, leaving red, raw areas.
Pemphigus vulgaris patients eventually become febrile and lose weight, and if untreated, most will die
within 1 year. Bullous pemphigoid lesions tend to heal and the patients do very well. This prognostic
difference is an important distinction in dermatologic medicine.
Eosinophils within blisters (choice A) provide an important clue supporting bullous pemphigoid as the
diagnosis that must be ascertained with histologic examination. The vesicles in pemphigus vulgaris
mostly contain rounded acantholytic keratinocytes "floating" within.
Nikolsky's sign consists of separation of the epidermis on manual stroking of the skin. Bullous
pemphigoid is characterized by a negative Nikolsky's sign (choice B), but Nikolsky's sign is positive in
pemphigus vulgaris because of the IgG-mediated destruction of intercellular bridges between
keratinocytes.
IgG autoantibody activity (choice C) is common to pemphigus vulgaris and bullous pemphigoid.

36

76. A 30-year smoker walks into clinic, breathing heavily and complaining of dyspnea and
constant fatigue. On physical examination, the physician observes a "barrel chest" (expanded,
with increased anteroposterior diameter) and hypertrophy of the accessory respiratory
muscles. No cyanosis is evident. Occasionally he develops episodes of nonproductive cough,
but he denies asthma attacks. Blood gas analysis shows minimal hypoxemia and normal CO2.
Which of the following underlying pathogenetic mechanisms is most likely responsible for this
patient's condition?
A. Airway obstruction
B. Bronchospasm
C. Chest wall deformity
D. Interstitial infiltration
E. Loss of elastic recoil
The correct answer is E. This patient is the classic pink puffer with chronic obstructive pulmonary
disease (COPD). COPD is an umbrella term that refers to overlapping clinical conditions resulting
from a combination of emphysema, asthma, bronchiectasis, and chronic bronchitis. If emphysema is
predominant, patients with COPD have severe dyspnea (puffers), scanty sputum production, and
nearly normal O2 arterial pressure, and thus no cyanosis (pink). Loss of elastic recoil is characteristic
of emphysema, which is caused by destruction of alveolar walls and enlargement of airspaces distal
to terminal bronchioles. Cigarette smoking is clearly the most important cause of COPD. Destruction
of the pulmonary elastic fibers brings about increased resistance to airflow. The lungs become
overexpanded; although total pulmonary capacity increases, the functioning lung parenchyma
decreases. In emphysema the major complaint is dyspnea, often severe. Cough is rare with scant
clear mucoid sputum. No peripheral edema is noted.
Airway obstruction (choice A) is prevalent in patients who have COPD with predominant chronic
bronchitis (i.e., blue bloaters. Decreased PaO2 manifests with cyanosis (blue), and bronchitis causes
abundant sputum production. Pulmonary hypertension and right ventricular overload produce
peripheral edema (bloaters). The patient in this case does not fit this description.
Bronchospasm (choice B) is associated with asthma, a frequent component of COPD. Attacks of
asthma are caused by spasm of bronchiolar smooth muscles, resulting in increased resistance to
expiration. The clinical history clearly rules out bronchospasm as the fundamental mechanism of this
patient's condition.
Chest wall deformity (choice C), such as severe kyphoscoliosis and obesity, and interstitial infiltration
(choice D), usually caused by interstitial fibrosis, are responsible for restrictive pulmonary disease.
Restrictive pulmonary disease leads to decreased lung compliance and reduction in all respiratory
volumes. The barrel-chest deformity of this patient is a consequence, not a cause, of the underlying
pathologic change (i.e., overexpansion of the lungs).
77. What is the MOST likely cause for profuse upper gastrointestinal bleeding with
hematemesis and unconsciousness in an alcholic?
A. Barrett's esophagus
B. Helicobacter gastritis
C. Mallory-Weiss tear
D. Schatzki ring
E. Zenker's diverticulum
The correct answer is C. This is a description of a Mallory-Weiss tear, which is a mucosal tear at the
gastroesophageal junction secondary to recurrent vomiting (the stomach temporarily evulses through
the esophagus, tearing the esophagus). The result can be massive hematemesis, but the lesions
usually heal without problems if the patient does not die from exsanguination. Mallory-Weiss tears
account for 5-10% of cases of upper gastrointestinal bleeding. Most patients report history of heavy
alcohol abuse.
Barrett's esophagus (choice A) is characterized by the replacement of normal esophageal epithelium
with gastric-type epithelium. Barrett's causes an increased risk for adenocarcinoma, not bleeding.

37

Helicobacter gastritis (choice B) does not usually cause profuse bleeding.


Schatzki rings (choice D) are benign mucosal rings at the squamocolumnar junction below the aortic
arch.
Zenker's diverticula (choice E) are esophageal evaginations at the junction of the pharynx and
esophagus. They are not typically associated with bleeding.
78. Which of the following clinical conditions is most likely to result in hoarseness during
speech?
A. Right lung apical segment bronchogenic carcinoma
B. Congestive heart failure
C. Emphysema
D. Pulmonary thromboembolism
E. Thymoma
The correct answer is A. A tumor of the apical segment of the upper lobe may impinge on the
recurrent laryngeal nerve because it ascends from the superior mediastinum to the root of the neck in
a groove between the trachea and esophagus. The recurrent laryngeal nerve supplies all intrinsic
muscles of the larynx except the cricothyroid.
Congestive heart failure (choice B) is associated with exertional dyspnea, fatigue, orthopnea,
paroxysmal nocturnal dyspnea, and rales. Speech patterns are often normal.
Emphysema (choice C) is a condition associated with dyspnea, often severe, in which the presence
of a cough is rare. Speech patterns are generally normal.
Pulmonary embolus (choice D) causes an obstruction to arterial blood flow of the lung, resulting in
infarction of the affected segment. It may also cause pleurisy, resulting in pain conveyed by intercostal
nerves.
Thymoma (choice E), or tumor of the thymus gland, may cause dyspnea (difficulty breathing) caused
by pressure on the trachea. It may also cause engorgement of deep and superficial veins of the neck
caused by pressure on the superior vena cava.
79. Which of the following features characterizes apoptosis but not necrosis?
A. Disaggregation of polyribosomes
B. Eosinophilia
C. Inflammation
D. Karyolysis
E. Peripheral aggregation of chromatin
The correct answer is E. Apoptosis is a form of cell death that serves to eliminate unwanted cells
during development, maintain cell numbers in intact organs or tissues, and eliminate immune cells
after an immune response has faded. Apoptosis also occurs in response to noxious agents and in the
aging process by way of execution of a genetic program. In the process of apoptosis, cells shrink and
cytoplasmic organelles become more densely packed. Cytoplasmic blebs may form, and apoptotic
bodies (membrane-bound cellular fragments) can be produced. The most characteristic feature of
apoptosis is a distinctive peripheral aggregation of chromatin, sometimes accompanied by breaking
up of the nucleus into several fragments. In necrosis, the chromatin may become more pale
(karyolysis; choice D), or form irregular clumps, and the nucleus itself may shrink into a dense
pyknotic body.
Disaggregation of polyribosomes (choice A) is characteristic of the initial stages of cellular injury and
necrosis.
Eosinophilia (choice B) characterizes both apoptosis and necrosis.

38

Inflammation (choice C) is typically absent in apoptosis, in contrast to necrosis.


80. Which of the following can occur in a child after an upper respiratory infection who starts
to form multiple petechial hemorrhages and blood tests show marked reduction in platelets?
A. Idiopathic thrombocytopenic purpura
B. Iron deficiency anemia
C. Pernicious anemia
D. Von Willebrand's disease
The correct answer is A. The process described is commonly called acute idiopathic
thrombocytopenic purpura (ITP), even though the autoimmune basis has been clearly established
(some investigators use "immune thrombocytopenic purpura," so that the initials still work). The
thrombocytopenia in this disorder seems to be secondary to splenic destruction of opsonized platelets
and usually follows a viral upper respiratory tract infection. The acute form of ITP is usually explosive
but self-limited; a chronic form in adults may respond to steroid therapy or splenectomy.
Iron deficiency anemia (choice B) is almost always caused by bleeding in adults. Absent bone
marrow iron stores are noted or serum ferritin is low.
Pernicious anemia (choice C) is a macrocytic anemia with macro and hyposegmented neutrophils on
peripheral blood smear.
In von Willebrand's disease (choice E), deficient von Willebrand's factor produces platelet
dysfunction, but thrombocytopenia is not prominent.
81. Patients with Plummer-Vinson syndrome are at an increased risk for
A. Adenocarcinoma of the esophagus
B. Barrett's esophagus
C. Candida infection
D. Cytomegalovirus esophagitis
E. Squamous cell carcinoma of esophagus
The correct answer is E. Plummer-Vinson syndrome is characterized by atrophic glossitis,
esophageal webs, and iron-deficiency anemia. Patients with this syndrome are at increased risk for
developing squamous cell carcinoma of the esophagus.
Barrett's esophagus (choice B) and adenocarcinoma of the esophagus (choice A) are associated
with reflux esophagitis.
Candida (choice C) and cytomegalovirus (choice D) esophagitis can be seen in immunosuppressed
patients, including AIDS patients.
82. What is a flesh-colored lesion that presents on the nose that is 1 cm in diameter with
induration and central ulceration?
A. Basal cell carcinoma
B. Eczema
C. Psoriasis
D. Urticaria
E. Verruca vulgaris
The correct answer is A. The description is typical for basal cell carcinoma. These skin cancers
typically occur on sun-exposed areas of the head, neck, and upper trunk. Basal cell carcinoma only
rarely metastasizes, but can become locally mutilating if neglected. When located on the face, it may
be difficult to adequately excise without damaging facial structures.

39

Eczema (choice B) typically involves a larger area of skin and may cause dryness, discoloration, and
thickening of the involved area. Blistering, erythema, or oozing may also be observed.
Psoriasis (choice C) is characterized by erythematous plaques with a silvery surface.
Urticaria (choice D) causes transient, nonpitting, erythematous wheals.
Verruca vulgaris (choice E), the common wart, causes well-demarcated verrucous papules, often on
the hands.
83. Which of the following is may cause sudden extreme hematemesis and eventual loss of
consciousness?
A. Esophageal varices
B. Mallory-Weiss tear
C. Barrett's esophagus
D. Schatzki ring
E. Zenker's diverticulum
The correct answer is A. This is a description of life-threatening bleeding from esophageal varices
(tortuous, dilated, submucosal esophageal vessels). Sclerotherapy of the vessels and pressure on the
bleeding site with an "esophageal balloon" may temporarily control the problem, but unfortunately
bleeding often recurs and exsanguination is a frequent cause of death in these patients.
In contrast, Mallory-Weiss tears (choice B) occur as a complication of repeated vomiting and do not
often result in this amount of blood loss.
The other esophageal conditions listed do not usually cause hematemesis.
Barrett's esophagus (choice C) does not produce specific symptoms. Associated symptoms,
however, are generally a consequence of gastroesophageal reflux disease.
Schatzki rings (choice D) are benign mucosal rings found at the squamocolumnar junction of the
esophagus, below the aortic arch.
Zenker's diverticulum (choice E) is an esophageal evagination at the junction of the pharynx and
esophagus.
84. Which of the following characteristics is typical in the early stages of type 1 diabetes
mellitus, but not the early stages of type 2 diabetes mellitus?
A. Adult onset
B. Nearly complete twin concordance
C. Increased serum insulin levels
D. Ketoacidosis
E. Obesity
The correct answer is D. Type 1 diabetes mellitus, previously known as juvenile onset, is caused by
low insulin production as a consequence of autoimmune destruction of pancreatic beta cells. Severe
insulin deficiency causes marked increases in the use of fats as a source of energy. Ketones,
acetoacetate, and beta-hydroxybutyrate are produced in excess, and diabetic ketoacidosis may
develop with potentially dire consequences. Type 2 diabetes (or adult onset) is a consequence of
insulin resistance by the tissues, despite very high levels of serum insulin, initially (insulin levels
typically decrease as the disease progresses). Ketoacidosis is highly unusual in type 2 in the early
stages, because insulin is present. As the disease progresses, however, and insulin levels begin to
decrease, the incidence of ketoacidosis increases.

40

In type 1 there is usually complete loss of beta cells by puberty; thus, insulin dependence generally
begins in childhood. Type 2 usually has an adult age of onset (choice A).
There is approximately 50% twin concordance in type 1, suggesting that environmental factors must
also play a "triggering role" in type 1. The twin concordance rate is much higher in type 2 (~90%)
(choice B).
Insulin levels are nearly zero in type 1. Conversely, type 2 is a disease of insulin resistance and is
usually associated with increased insulin levels (choice C).
Body weight has no bearing on the pathogenesis of type 1, whereas type 2 occurs predominantly in
the obese (choice E).
85. Which of the following tests should be performed on a young adult woman who presents
with with fatigue, malaise, low-grade fever, arthralgias, elevated BUN, proteinuria, and butterfly
rash on her face?
A. Anticentromeric antibody
B. Antimitochondrial antibody
C. Antinuclear antibody
D. Anti-TSH receptor antibody
E. Rheumatoid factor
The correct answer is C. Systemic lupus erythematosus (SLE) and should be considered in any 1545-year-old woman with a malar (butterfly) rash, anti-double-stranded DNA antibodies, and renal
involvement. Antinuclear antibody is a good screening test (although it is also positive in some other
autoimmune diseases). Renal involvement is unique to this autoimmune disease. SLE mainly occurs
in young women and joint symptoms are seen in 90% of all patients. The most common features of
SLE include: malar rash, discoid rash, photosensitivity, oral ulcers, arthritis, serositis, renal disease,
neurologic disease, and positive ANA.
Anticentromeric antibody (choice A) is a marker for the CREST form of scleroderma.
Antimitochondrial antibody (choice B) is a marker for primary biliary cirrhosis. Anti-TSH receptor
antibody (choice D) is a marker for Grave's disease. Rheumatoid factor (choice E) is a marker for
rheumatoid arthritis.
86. Which of the following can develop in a diabetic with a long-standing intrauterine
contraceptive device that develops chronic pelvic pain and whose latest endometrial biospy
shows a prominent infiltrate composed of lymphocytes, plasma cells, and histiocytes?
A. Acute endometritis
B. Candida infection
C. Chronic endometritis
D. Endometriosis
E. Simple hyperplasia of endometrium
The correct answer is C. This is a description of chronic endometritis, evidenced by the chronic
inflammatory infiltrate of lymphocytes, plasma cells, and histiocytes. This disorder may be idiopathic
but is more often associated with an obvious predisposing factor, such as chronic pelvic inflammatory
disease, tuberculosis, retained gestational tissue, or, as in this case, an intrauterine contraceptive
device. Chronic endometritis can cause abnormal bleeding, pain, and infertility.
Acute endometritis (choice A) is characterized by a prominent neutrophilic infiltrate and usually
occurs after delivery or miscarriage.
Candidiasis (choice B) occurs in an estimated 75% of women during their lifetime. Although diabetes
is an important risk factor, this is not a description of vulvovaginal candidiasis. Common symptoms
include acute vulvar pruritus, burning vaginal discharge and dyspareunia, and the characteristic "curdlike" discharge.

41

Endometriosis (choice D) refers to abnormally located patches of endometrium (except in the


myometrium, where it would be called adenomyosis).
Simple hyperplasia of endometrium (choice E) causes cystically dilated glands in endometrium.
87. A patient has intermittent diarrhea and abdominal pain and begins to pass fecal material in
his urine. Which of the following diseases is most likely to cause this complication?
A. Celiac disease
B. Crohn disease
C. Diverticulitis
D. Pseudomembranous colitis
E. Ulcerative colitis
The correct answer is B. Passing fecal material in urine strongly suggests the possibility of a fistula
between the bowel and bladder. Of the diseases listed, only Crohn disease (a type of inflammatory
bowel disease) commonly produces fistulas. Fistulas are produced in Crohn disease because the
disease affects the entire thickness of the bowel wall rather than being restricted to the mucosa (e.g.,
ulcerative colitis - choice E).
Celiac disease (choice A) is a mucosal disorder of the small intestine caused by intolerance to certain
components of gluten from wheat and other grains.
Diverticulitis (choice C) can cause bowel perforation with peritonitis but does not usually cause fistula
formation.
Pseudomembranous colitis (choice D), or antibiotic induced colitis, is usually caused by the toxin
produced by the bacteria Clostridium difficile. The most common complaint is severe diarrhea.
88. A patient's complete blood count demonstrates a hematocrit of 62%. The peripheral smear
shows normocellular erythrocytes, with increased reticulocytes and nucleated red cells. Bone
marrow biopsy demonstrates increased numbers of erythrocytic precursors. Cancer of which
of the following organs would be most likely to cause these findings?
A. Colon
B. Kidney
C. Ovary
D. Prostate
E. Thyroid
The correct answer is B. This is a pathophysiology question that can be answered easily by simply
understanding that a low hematocrit is probably caused by a low production of erythropoietin that is
synthesized and secreted in the kidney. The hematologic finding is erythrocytosis, which can be
caused by abnormal erythropoietin secretion by renal cell carcinoma (i.e., a paraneoplastic
syndrome). Absolute erythrocytosis also occurs in several other conditions, such as hypoxia, other
types of renal disease, some tumors (e.g., hepatocellular carcinoma, meningioma,
pheochromocytoma, cerebellar hemangioblastoma, adrenal adenoma), androgen therapy, Bartter
syndrome, or in polycythemia vera. Cancers of the colon (choice A), prostate (choice D), and thyroid
(choice E) do not usually produce inappropriate hormones. Cancers of the ovary (choice C) can
produce male or female sex steroids, but do not produce erythropoietin.
89. Arthritis with a history significant for morning stiffness and symmetric deformities of the
affected joints is most likely associated with which condition?
A. Ankylosing spondylitis
B. Gouty arthritis
C. Osteoarthritis
D. Rheumatoid arthritis
E. Septic arthritis

42

The correct answer is D. Involvement of the cervical spine occurs in almost 80% of patients with
rheumatoid arthritis and involves the atlanto-axial joint in up to 25% of those hospitalized for
rheumatoid arthritis. Fortunately, large degrees of subluxation of the joint are rare, but if present can
be associated with quadriplegia and even sudden death if the odontoid peg separates from the arch of
the atlas and compresses the spinal cord. The onset of rheumatoid arthritis is usually insidious and in
the small joints. Progression is centripetal and symmetric. Deformities are common. Extraarticular
manifestations include subcutaneous nodules, pleural effusion, pericarditis, lymphadenopathy, and
splenomegaly.
Serious complications of ankylosing spondylitis (choice A) include atrioventricular block, bladder and
bowel dysfunction, uveitis, pulmonary fibrosis, psoriasis, and inflammatory bowel disease.
Serious complications of gout (choice B) include renal impairment.
Serious complications of osteoarthritis (choice C) are uncommon, but the local manifestations can be
crippling. They do not cause quadriplegia. Pain and stiffness are most extreme at the end of the day,
which differs from rheumatoid arthritis.
Serious complications of septic arthritis (choice E) include joint destruction, osteomyelitis, and
systemic infection.
90. Which of the following can predispose a women to have vaginal itchiness and white curdy
discharge that upon microscopic examination demonstrates fungal hyphae and yeast forms?
A. Crohn's disease
B. Diabetes mellitus
C. Disseminated gonococcal infection
D. Rheumatoid arthritis
E. Systemic lupus erythematosus
The correct answer is B. The patient has vulvovaginitis secondary to Candida infection.
Predisposing factors include a high vaginal pH, diabetes, and use of antibiotics. The increased
vulnerability in patients with diabetes may reflect increased glucose concentrations in vaginal
secretions and relative immunosuppression, both of which provide an excellent environment for
fungus to grow.
Crohn's disease (choice A) can predispose for fistulas involving the vagina, rather than vulvovaginitis.
Dissemination of Neisseria gonorrhoeae (choice C) can cause septic arthritis.
Neither rheumatoid arthritis (choice D) nor systemic lupus erythematosus (choice E) are specifically
associated with Candida vulvovaginitis.
91. Which of the following most likely precedes the development of non-Hodgkin's lymphoma
in the gland?
A. Follicular thyroid carcinoma
B. Grave's disease
C. Hashimoto's thyroiditis
D. Nodular goiter
E. Papillary thyroid carcinoma
The correct answer is C. Hashimoto's disease (autoimmune thyroiditis) confers a 60-80-fold
increased risk for developing thyroid lymphoma. This increased risk is not surprising considering the
histology exhibited in Hashimoto's disease, in which sheets of benign but activated lymphocytes
infiltrate and eventually destroy the thyroid gland, producing eventual thyroid failure.
Neither follicular (choice A) nor papillary (choice E) thyroid carcinomas predispose for lymphoma.
Lymphoma is not related to Grave's disease (choice B) or benign nodular goiter (choice D).

43

92. Which of the following most likely describes a person who is pleasant, emotionally warm
and happy, lives at home, works in a sheltered workshop, did not complete high school, and
talks in three-word sentences?
A. Autistic disorder
B. Mental depression
C. Mental retardation
D. Mixed receptive-expressive language disorder
E. Parkinson's disease
The correct answer is C. This describes simple mental retardation.
Autistic disorder (choice A) is not correct because persons with this are rarely able to interact with
others to the point of holding a job, and they characteristically do not use language for purposes of
communication.
Mental depression (choice B) is associated with feelings of worthlessness and helplessness. It
generally does not affect speech patterns.
Mixed receptive-expressive language disorder (choice D) is a neurologic condition and affects
language skills. This description does not discuss the use of both receptive and expressive language
skills used to the point of being productively employed.
Parkinson's disease (choice E) is associated with postural instability, bradykinesia, and mask-like
facies. It does not affect cognitive function.
93. If polydipsia and polyuria are present, but without glycosuria, which of the following
should be considered?
A. Addison's disease
B. Diabetes insipidus
C. Diabetes mellitus
D. Grave's disease
The correct answer is B. Diabetes insipidus is characterized by the excretion of abnormally large
volumes of dilute urine (polyuria) with a commensurate increase in fluid intake (polydipsia). The most
common type is caused by inadequate secretion of antidiuretic hormone (also called vasopressin) and
is usually referred to as "neurogenic" diabetes insipidus. This condition rarely causes severe problems
as long as the person has plenty of water to drink. Placing a patient on overnight water restriction can
cause severe dehydration and a greatly elevated plasma sodium concentration. The possibility of
diabetes mellitus (choice C), which can also be associated with polyuria and polydipsia, but it does
not have glucosuria.
Addison's disease (choice A) results from failure of the adrenal cortices to produce adrenocortical
hormones. The lack of aldosterone leads to decreases in sodium reabsorption allowing large amounts
of sodium to be lost into the urine. Polyuria and polydipsia are not characteristic of Addison's disease.
Grave's disease (choice D) or thyrotoxicosis is associated with marked tachycardia, profuse
sweating, delirium, nausea, vomiting, and fever.
94. Microscopically, which of following do the proliferating cells in Kaposi's sarcoma
associated with HIV infection MOST closely resemble?
A. Angiosarcoma
B. Carcinosarcoma
C. Lymphoma
D. Malignant fibrous histiocytoma
E. Melanoma

44

The correct answer is A. Kaposi's sarcoma is a spindle cell neoplasm that is highly associated with
AIDS and with the Herpes simplex virus type 8. The tumor has an appearance similar to that of
angiosarcoma-proliferating stromal cells and endothelium, creating vascular channels that contain
blood cells.
Carcinosarcoma (choice B) is a tumor that contains malignant epithelial cells and malignant stromal
cells. There is no epithelial element in Kaposi's sarcoma.
Although lymphoma (choice C) occurs with increased frequency in AIDS, it does not resemble
Kaposi's sarcoma. Lymphoma involves neoplastic lymphocytes, whereas Kaposi's sarcoma involves
neoplastic vascular structures.
Malignant fibrous histiocytoma (MFH); (choice D) is an extremely poorly differentiated (anaplastic)
stromal malignancy. MFH does not produce any recognizable mesenchymal structures; thus, the
production of vascular structures by Kaposi's sarcoma differentiates the two tumors.
Melanoma (choice E) does produce colored skin lesions; however, the histologic appearance of the
malignant melanocytes is unlike Kaposi's sarcoma. Melanoma in situ appears as small nests of cells
with large, red nucleoli in the dermis and epidermis; this lesion can progress to a variety of forms, but
none resemble Kaposi's sarcoma.
95. What is the cause for the anemia that develops in a person with systemic lupus
erythematosus, chronic renal failure, and with a slightly yellow sclerae, enlarged spleen and
positive Coombs test?
A. Bone marrow aplasia
B. Antibody directed against red blood cells
C. Spleen sequestration
D. Renal failure
The correct answer is B. The anemia must be a hemolytic form, because it is associated with
unconjugated hyperbilirubinemia (hence the yellow sclerae), resulting from increased destruction of
red blood cells. Increased erythrocyte destruction is the cause (not the effect) of splenomegaly. A
positive Coombs test implies that hemolysis is mediated by antibodies attached to red blood cells.
Antibody-coated red cells are then sequestered by the spleen, where hemolysis occurs, thus
explaining splenomegaly.
Bone marrow aplasia (choice A) is caused by failure or suppression of myeloid stem cells, with
decreased production of red blood cells, platelets, and leukocytes (pancytopenia).
Spleen sequestration (choice C) may cause anemia in case of massive splenomegaly because of an
exaggeration of the normal role of the spleen as repository of blood cells.
Renal failure (choice D) causes normochromic, normocytic anemia because of decreased synthesis
of erythropoietin. Erythropoietin administration is currently the standard treatment for this form of
anemia.
96. Autoantibodies to which of the following components would most likely be found in a
person who is not taking any medications but has multiple oral ulcers and flaccid skin bullae
that were biopsied and showed a separation of epithelial cells above the basal layer?
A. Epidermal basement membrane proteins
B. Glycoprotein IIb/IIIa
C. Intercellular junctions of epidermal cells
D. Intrinsic factor
E. Type IV collagen
The correct answer is C. Bullae with the cleavage plane above the basal layer of the epidermis
suggests pemphigus vulgaris, which is caused by autoantibodies to intercellular junctions of epidermal
cells. The autoantibodies decrease the ability of the keratinocytes to adhere to one another, permitting
formation of vesicles and bullae. Oral involvement is common, and often precedes the characteristic

45

skin lesions. There will also be separation of the epidermis on manual stroking of the skin which is
known as Nikolsky's sign. This sign is present in other disorders such as Stevens-Johnson syndrome,
but we are told this person is healthy, and thus is not taking any medications, a typical cause of
Stevens-Johnson syndrome in the adult population.
Antibodies to epidermal basement membrane proteins (choice A) are seen in bullous pemphigoid,
which is a bullous disease characterized by blisters with a cleavage line between the epidermis and
dermis.
Antibodies to glycoprotein IIb/IIIa (choice B) are seen in autoimmune thrombocytopenic purpura.
Antibodies to intrinsic factor (choice D) are seen in pernicious anemia.
Antibodies to Type IV collagen (choice E) are seen in Goodpasture syndrome.
97. Which of the following is the reason for a CBC with reticulocyte count that shows very low
white blood cells and platelet counts, as well as having a bone marrow biopsy that
demonstrates cellularity less than 25%?
A. Anemia of chronic disease
B. Aplastic anemia
C. Myelodysplasia with myelofibrosis
D. Pure red cell aplasia
The correct answer is B. Aplastic anemia is characterized by pancytopenia (low levels of all blood
cells and platelets). Bone marrow biopsy typically reveals a normal architecture with a decrease in
cellularity to levels less than 25% of normal. Absolute neutrophil counts are also often extremely low.
Aplastic anemia may be either hereditary or acquired. Chloramphenicol, some anticonvulsant drugs,
phenylbutazone, and a variety of other agents may produce aplastic anemia in an idiosyncratic
manner. Aplastic anemias have also been linked to exposure to environmental toxins, such as
benzene and insecticides, and are often found to be preceded by viral infections from
cytomegalovirus, parvovirus, and hepatitis.
Anemia of chronic disease (choice A) produces an isolated deficit of red cell production and may
resemble iron deficiency.
The myelodysplastic syndromes (e.g., myelodysplasia with myelofibrosis, choice C) are characterized
by replacement of the bone marrow with abnormal (dysplastic) stem cells and ineffective
hematopoiesis.
Pure red cell aplasia (choice D) would produce a selective deficit of the erythrocytic lineage, not all
three lineages.
98. Which of the following should a person be evaluated for if they have rheumatoid arthritis
and suddenly develop moderate caries?
A. Diabetes insipidus
B. Polyarteritis nodosa
C. Sjogren's syndrome
D. Systemic lupus erythematosus
The correct answer is C. Rheumatoid arthritis can coexist with a variety of autoimmune diseases
(including those listed in the answers), but is most frequently associated with Sjogren's syndrome.
Sjogren's syndrome is caused by autoimmune involvement with subsequent scarring of the salivary
and lacrimal glands, leading to dry eyes and dry mouth. Secondary effects include parotid gland
enlargement, dental caries (from dry mouth or xerostomia), and recurrent tracheobronchitis.
Diabetes insipidus (choice A) is associated with polyuria and polydipsia but not dental caries.
Polyarteritis nodosa (choice B) is a systemic necrotizing vasculitis. Patients present with low-grade
fever, weakness, and weight loss. They may also have abdominal pain, hematuria, renal failure,
hypertension, and leukocytosis.

46

Systemic lupus erythematosus (choice D) is an autoimmune disease characterized by vasculitis


(which may produce a variety of symptoms depending on the site of the lesion), rash, renal disease,
hemolytic anemia, and neurologic disturbances.
99. Which of the following physical findings would be the MOST reliable indicator that an older
woman with shortness of breath is experiencing heart failure?
A. A third heart sound (S3)
B. A fourth heart sound (S4)
C. Ascites
D. Orthopnea
E. Pulmonary rales
The correct answer is A. A third heart sound (S3) is a low-pitched sound occurring at the termination
of rapid filling. In patients over 40 years of age, the appearance of a third heart sound strongly
suggests congestive heart failure. It also occurs in patients with atrioventricular valve incompetence
and can be a normal finding in some young athletes. Other common signs and symptoms of heart
failure include the following:
Left ventricular failure: exertional dyspnea, fatigue, orthopnea, cough, cardiac enlargement, rales,
gallop rhythm, and pulmonary venous congestion.
Right ventricular failure: elevated venous pressure, hepatomegaly, and dependent edema.
A fourth heart sound (S4; choice B) can be a normal finding in some older patients who do not have
congestive heart failure.
Ascites (choice C) can also occur in patients with renal, hepatic, or local conditions not associated
with cardiac factors.
Both orthopnea (choice D) and pulmonary rales (choice E) often occur secondary to heart failure;
however, they both are associated with noncardiac disorders also.
100. Which of the following is an example of a type II hypersensitivity?
A. Cutaneous reactions to drugs
B. Heat intolerance, sinus tachycardia, and proptosis caused by endocrine disease
C. Eczematous reaction on the dorsum of the foot after new detergent use
D. Glomerulonephritis with systemic lupus erythematosus
E. Wheal and flare reactions and vesicles due to fire ant bites
The correct answer is B. Type II hypersensitivity is mediated by antibodies directed toward antigens
that are present on the surface of cells or other tissue components. The antigen may be intrinsic to
the cell membrane or may take the form of an exogenous antigen that is adsorbed to the cell surface.
The situation described is Grave's disease, which is an autoimmune form of hyperthyroidism
produced by autoantibodies directed against the TSH (thyroid stimulating hormone) receptor. These
antibodies are called LATS (long-acting thyroid stimulator) and stimulate thyroid function, resulting in
the release of thyroid hormones.
A drug reaction (choice A) and wheal and flare reactions (choice E) are cutaneous manifestations of
type I hypersensitivity. Certain allergens, especially drugs, insect venoms, latex, and foods may
induce a type I IgE antibody response.
An eczematous reaction (choice C) associated with washing clothes in a new detergent may either
represent type IV hypersensitivity or a nonimmune reaction associated with direct toxicity from some
component of the soap penetrating the skin.
Glomerulonephritis in systemic lupus erythematosus (choice D) is caused by the deposition of
antigens in the glomerular basement membrane with the resultant formation of antigen-antibody

47

complexes. These complexes activate the complement cascade, which causes neutrophils to enter
the area and produce tissue damage.
101. Examination of a peripheral blood smear demonstrates a leukemia composed of small
mature lymphocytes without blast forms. If a diagnosis of chronic lymphocytic leukemia is
made, which of the following is the most likely age of this patient?
A. 1 year
B. 5 years
C. 20 years
D. 45 years
E. 65 years
The correct answer is E. Different leukemias tend to affect populations of different ages. The
disease described is chronic lymphocytic leukemia (CLL), a disease of older adults.
Both the 1-year-old (choice A) and the 5-year-old (choice B) would be most likely to have acute
lymphocytic leukemia (ALL).
The 20-year-old (choice C) would be most likely to have acute myelocytic leukemia (AML).
The 45-year-old (choice D) would be likely to have either AML or chronic myelogenous leukemia
(CML).
102. Which of the following serum chemistry studies will MOST likely be abnormal in a
preteen with polydipsia that develops weight loss despite eating large amounts of food over a
three month period?
A. Blood urea nitrogen
B. Serum bicarbonate
C. Serum calcium
D. Serum glucose
E. Serum sodium
The correct answer is D. Although it is commonly known that diabetes mellitus is associated with
polyuria and polydipsia, many do not realize it is also associated with weight loss despite increased
eating, also known as polyphagia. In our extremely weight- and food-conscious society, this
phenomenon may be much more striking to the patient and his family than a change in the amount of
fluid intake. The lesson learned is that a child who develops weight loss despite increased food intake
should have a blood glucose test.
All of the other answer choices would be expected to be normal in this patient.
103. Which of the following is involved in hypoxia in a smoker with noticeable increase in
anteroposterior chest width if oxygen therapy is effective?
A. Anemia caused by blood loss
B. Edematous tissues
C. Emphysema
D. Localized circulatory deficiencies
E. Right-to-left cardiac shunts
The correct answer is C. Chronic pulmonary emphysema is characterized by distention of small air
spaces distal to the respiratory bronchioles and destruction of alveolar septa. Long-term cigarette
smoking is the usual cause. The marked loss of lung parenchyma associated with emphysema leads
to a decrease in the diffusion capacity of the lungs that reduces their ability to oxygenate blood and
remove carbon dioxide, contributing to the "barrel chest" appearance of these patients. Cigarettes
also inhibit Alpha-1 antitrypsin allowing trypsin to break down proteins in the alveoli. When arterial
hypoxemia is persistent and severe, oxygen therapy should therefore be considered.

48

Oxygen therapy is less effective for treating the hypoxia associated with anemia (choice A),
edematous tissues (choice B), localized circulatory deficiencies (choice D), and right-to-left cardiac
shunts (choice E), because in each case, there is already adequate oxygen available in the alveoli.
The problem in each of these situations is inadequate transport of oxygen to the tissues, blunting the
effects of increasing the oxygen tension of the inspired air. Oxygen therapy nonetheless increases the
amount of dissolved oxygen carried in the blood, which may be life-saving in some instances.
104. Which of the following characterizes an eczematous reaction that occurs 72 hours after
wearing latex gloves?
A. Irritant dermatitis
B. Type I reaction
C. Type II reaction
D. Type III reaction
E. Type IV reaction
The correct answer is E. Sensitization to latex has become a major healthcare problem. Local skin
irritations are common but more severe allergic reactions occur, up to and including rare anaphylactic
reactions that are occasionally fatal. The immune responses to latex are immediate-type
hypersensitivity (type I) reactions, expressing themselves in minutes, or delayed-type hypersensitivity
(type IV) reactions that express themselves in 48-72 hours. The type I reactions are caused by the
IgE-mediated sensitivity to latex proteins, whereas the type IV reactions are caused by a cellmediated response to the chemicals that are added in the processing of latex. The type IV response
in this circumstance would be referred to as contact dermatitis.
Irritant dermatitis (choice A) can be observed in the early stages of sensitization and can be caused
by sweating, rubbing, and residual soap. The timing of the reaction after 72 hours points to the type IV
reaction rather than an irritant dermatitis.
A type I reaction (choice B) would have been apparent within minutes with characteristic rhinitis,
conjunctivitis, urticaria, asthma, angioedema, or anaphylaxis immediately after wearing latex gloves.
A type II reaction (choice C) is a cytolytic response mediated by an antibody. The type II reaction is
observed in the context of hemolytic anemias, thrombocytopenia, and neutropenia.
A type III reaction (choice D) is an immune-complex response that sets into motion an inflammatory
response.
105. What is the explanation for an older person having a fainting spell with a resting blood
pressure of 130/60 mm Hg and a pulse reading of 40 beats per minute?
A. Aortic valve obstruction
B. Cardiac tamponade
C. Complete heart block
D. Heart failure
E. Hypertension
The correct answer is C. In complete (third degree) heart block, the ventricles beat independently of
SA node activity and P waves become completely dissociated from QRS-T complexes. The rate of the
ventricular beat is usually 30-45 per minute. Because resting cardiac output (CO) is normal and
because CO = stroke volume x heart rate, the stroke volume is increased in complete heart block.
When the stroke volume increases, a greater amount of blood must be accommodated in the arterial
tree with each heartbeat, which causes a greater increase and decrease in pressure during systole
and diastole. Note that the pulse pressure is 70 mm Hg (normal pulse pressure is 30-50 mm Hg).
The pulse pressure is decreased in aortic valve obstruction (choice A), cardiac tamponade (choice
B), heart failure (choice D). This is clearly not hypertension (choice E).

49

106. A well-behaved toddler, accompanied by his parents, presents with multiple fractures,
humpback and blue sclera. What is the probable diagnosis?
A. Child abuse
B. Ehlers- Danlos syndrome
C. Osteogenesis imperfecta
D. Scurvy
The correct answer is choice C. Osteogenesis imperfecta, also known as Brittle bone disease, is
characterized by retarded wound healing. . The most common form is autosomal dominant with
abnormal collagen type 1 synthesis due to a mutation in the collagen gene.
Choice A- Child abuse is unlikely in this well-behaved toddler with no other signs and symptoms of
abuse. Blue sclera are also typical of osteogenesis imperfecta, and not related to child abuse.
Choice B- Mutation in the collagen gene and lysine hydroxylase gene results in hyperextensible,
fragile skin, hypermobile joints, dislocations, varicose veins, ecchymoses, and arterial intestinal
ruptures. Inheritance varies. There are 10 types with any one of several enzyme deficiencies.
Choice D- Scurvy is characterized by anemia, petechiae, ecchymoses, bleeding gums, loose teeth,
poor wound healing and poor bone development. Deficient hydroxylation of collagen due to vitamin C
deficiency results in defective connective tissue.
107. When a histologic section is taken of an intraoral abscess, many of the observed
neutrophils show a degenerative change in which the nucleus has undergone fragmentation.
This process is known as
A. caseous necrosis
B. coagulative necrosis
C. karyolysis
D. karyorrhexis
E. pyknosis
The correct answer is D. Karyorrhexis refers to a pattern of nuclear degradation in which a pyknotic
or partially pyknotic nucleus undergoes fragmentation followed by complete lysis. This pattern is
common in the neutrophils present in acute inflammation.
The type of necrosis seen in an abscess is liquefactive necrosis. Caseous necrosis (choice A) is
seen in tuberculosis and some other granulomatous diseases; coagulative necrosis (choice B) is
seen following infarctions of many organs (other than the brain).
Karyolysis (choice C) is also a degenerative change affecting nuclei. In this case, however, it is seen
as a decrease in nuclear basophilia, which is presumably the result of DNAse activity.
Pyknosis (choice E) is characterized by nuclear shrinking and basophilia, apparently as a result of
DNA condensation.
108. If hypocalcemia is noted, excessive secretion of which of the following hormones MOST
likely contributed to hand and forearm bone erosion and resorption during chronic renal
failure?
A. Aldosterone
B. Calcitonin
C. Parathyroid hormone
D. Renin
E. Vitamin D
The correct answer is C. In renal failure, the ability of the kidney to secrete phosphate is impaired.
The resultant hyperphosphatemia causes hypocalcemia and triggers excretion of large amounts of

50

parathyroid hormone. The released parathyroid hormone is a major contributor to bony changes (e.g.,
osteitis fibrosa, a form of localized bone resorption) seen with chronic renal failure. Note that excess
parathyroid hormone normally causes hypercalcemia, but calcium resorption still cannot bring calcium
levels back to normal.
Aldosterone (choice A) is a regulator of serum sodium.
Calcitonin (choice B) levels are usually low in chronic renal failure unless the parathyroids have so
hypertrophied as to cause "tertiary hyperparathyroidism" with hypercalcemia.
Renin (choice D) is normally secreted by the kidney and may be decreased or increased in varying
stages and forms of kidney disease. Renin regulates blood pressure and aldosterone secretion, rather
than bone metabolism.
The active form of vitamin D (choice E), cholecalciferol, is formed in the kidney from vitamin D
absorbed from the gut and then processed by the liver. Uremia interrupts this pathway and
consequently causes a functional vitamin D deficiency.
109. Which of the following would be MOST likely present on a biopsy of a thyroid gland and
associated tissues that has a woody hardness when palpated but the needle biopsy shows no
evidence of malignancy?
A. Marked fibrous reaction with gland destruction
B. Masses of hyperplastic follicles
C. Multinucleated giant cells
D. Small foci of lymphocytic infiltration
E. Prominent lymphocytic infiltrate with gland destruction
The correct answer is A. With Riedel's thyroiditis, an uncommon form of chronic (possibly
autoimmune) thyroiditis that is characterized by dense fibrosis that destroys the thyroid gland and also
extends into the adjacent muscle and connective tissue of the neck. The condition is clinically
important because it may mimic malignancy. A key to the diagnosis is the woody hardness of fibrosis.
All other choices are likely to be swollen and soft by palpation.
Masses of hyperplastic follicles (choice B) are a feature of multinodular goiter.
Multinucleated giant cells (choice C) are a feature of de Quervain thyroiditis.
Lymphocytic infiltration (choice D) is a part of many thyroid diseases, but is seen in isolation in
subacute lymphocytic thyroiditis.
Sheets of lymphocytes with gland destruction (choice E) are seen in Hashimoto's thyroiditis.
110. Which of the following gastrointestinal diseases is most likely to be associated with
megaloblastic anemic who has been found to be deficient in vitamin B12, while the levels of all
other essential vitamins are within normal limits?
A. Atrophic gastritis
B. Celiac sprue
C. Duodenal ulcer
D. Ulcerative colitis
E. Zollinger-Ellison syndrome
The correct answer is A. An important cause of vitamin B12 deficiency is pernicious anemia
(megaloblastic), an autoimmune disease associated with atrophic gastritis. In atrophic gastritis, the
gastric epithelium undergoes intestinal metaplasia, replacing gastric chief and parietal cells with
goblet cells. The metaplastic epithelium produces insufficient intrinsic factor to bind the dietary vitamin
B12deficiency.

51

Celiac sprue (choice B) is a condition associated with weight loss, flatulence, greasy stools, and
increased fecal fat. Clinical improvement is seen with a gluten-free diet (a diet free of wheat, rye,
barley, and oats).
Duodenal ulcers (choice C) may also produce iron deficiency anemia because of chronic blood loss.
They are caused by damage to the duodenal epithelium by gastric acids and are associated with
cirrhosis, COPD, chronic renal failure, and hyperparathyroidism.
Ulcerative colitis (choice D) is an inflammatory bowel disease almost entirely restricted to the large
intestine. It usually causes intestinal distress and diarrhea, and it may produce a malabsorption
syndrome that includes vitamin B12deficiency.
Zollinger-Ellison syndrome (choice E) is caused by gastric acid hypersecretion leading to peptic ulcer
disease. Diarrhea is common.
111. If left untreated, severe squamous dysplasia on a vocal cord of a hoarse smoker may
progress to which of the following?
A. Adenocarcinoma
B. Lymphoepithelioma
C. Mucoepidermoid carcinoma
D. Squamous cell carcinoma
E. Squamous papilloma
The correct answer is D. Squamous cell carcinoma is the most frequent type of cancer of the larynx.
As with squamous cell carcinoma of the uterine cervix, the development of laryngeal carcinoma is
related to an orderly sequence of morphologic changes. These begin with epithelial hyperplasia,
proceed through increasingly severe degrees of dysplasia up to in situ carcinoma, and culminate with
invasive carcinoma. Cigarette smoking is the most important risk factor for the development of
laryngeal carcinoma. Any patient over the age of 50 years with hoarseness that has persisted beyond
2-3 weeks should be evaluated by indirect laryngoscopy. Odynophagia, hemoptysis, weight loss,
referred otalgia, vocal cord immobility and cervical adenopathy suggest more advanced disease.
Adenocarcinoma (choice A) and mucoepidermoid carcinoma (choice C) are rare forms of laryngeal
cancer. Squamous dysplasia is not a precursor of either type of tumor.
Lymphoepithelioma (choice B) is a form of squamous cell carcinoma that most frequently occurs in
the nasopharynx, although it has been reported in the larynx also. Its name is derived from the fact
the tumor is rich in lymphocytes. This tumor occurs frequently in southern China and certain regions in
Africa. Epstein-Barr virus is implicated in its pathogenesis.
Squamous papilloma (choice E) is a benign laryngeal neoplasm caused by human papillomavirus
types 6 and 11. It is not associated with squamous dysplasia.
112. What is the BEST explanation for the significant elevation of liver enzymes, ammonia, and
hypoglycemia experienced five days after the onset of chicken pox in a child that was given
aspirin for fever-control?
A. Crigler-Najjar syndrome
B. Dubin-Johnson syndrome
C. Gilbert's syndrome
D. Reye's syndrome
E. Rotor's syndrome
The correct answer is D. The use of aspirin in a child with chicken pox can cause Reye's syndrome.
Reye's syndrome (fatty liver with encephalopathy) is an acute (and potentially fatal) postviral injury
that is characterized by severe mitochondrial damage affecting the liver, brain, skeletal muscle, heart,
and kidneys. The rapidly progressive hepatic failure and encephalopathy is associated with a 30%
fatality rate. Most are children, although adult cases have been described. Varicella and influenza A
and B are the most common precipitating illnesses. Aspirin use has been linked to the development of

52

this disorder, but cases occur in the absence of salicylate ingestion. Hypoglycemia, elevated serum
aminotransferases and blood ammonia, prolonged prothrombin time, and change in mental status all
occur within 2-3 weeks after onset.
Crigler-Najjar (choice A) syndrome is a rare, mild to severe form of inherited unconjugated
hyperbilirubinemia.
Dubin-Johnson syndrome (choice B) is an inherited conjugated hyperbilirubinemia associated with a
darkly pigmented liver.
Gilbert's syndrome
hyperbilirubinemia.

(choice

C)

is

common,

benign

form

of

inherited

unconjugated

Rotor's syndrome (choice E) resembles Dubin-Johnson syndrome, but is associated with a normalcolored liver.
Although most of these other symptoms are rare, it is important to recognize the signs, symptoms,
and causes of Reye's syndrome.
113. Which of the following is the MOST likely the reason for persistent and marked anasarca
noted with jaundice?
A. Lymphatic obstruction
B. Reduced central venous pressure
C. Reduced plasma oncotic pressure
D. Sodium retention
E. Venous thrombosis
The correct answer is C. Hepatic failure occurring in cirrhosis reduces the capacity of the liver to
synthesize sufficient quantities of plasma proteins (mostly albumin) necessary to maintain plasma
oncotic pressure. Low plasma oncotic pressure allows fluid from the intravascular fluid component to
move into the interstitial space, producing plasma volume contraction and edema.
Lymphatic obstruction (choice A) occurs as a result of mechanical blockage of lymphatics by tumor,
inflammatory processes, or certain parasitic infections. Cirrhosis does not lead to lymphatic
obstruction.
Reduced central venous pressure (choice B) does not cause edema. Conversely, increased central
venous pressure, which may arise with congestive heart failure, thrombosis, or cirrhosis can lead to
increased hydrostatic pressure and edema.
Sodium retention (choice D) is an important cause of edema in patients with poor renal perfusion.
The kidneys respond by retaining sodium and increasing plasma volume in an effort to increase renal
blood flow. Any sodium retention in cirrhosis is secondary to the decrease in plasma oncotic pressure
and consequent decrease in plasma volume.
Venous thrombosis (choice E) can lead to edema; however, the diminished synthesis of coagulation
proteins in cirrhosis predisposes to bleeding, not thrombosis.
114. What of the following is characterized by left lower quadrant periumbilical pain in an
elderly person, with the presence of fever, tender abdomen, leukocytosis, nausea, and
vomiting?
A. Acute appendicitis
B. Diverticulitis
C. Gallstones
D. Pancreatitis
E. Pyelonephritis

53

The correct answer is B. Diverticulitis is a disease of the elderly and usually involves the distal
colon. In severe cases, however, the diverticula may extend throughout the colon and up to the
cecum. Inflammation of a cecal diverticulum can closely mimic acute appendicitis. The essentials of
diagnosis for diverticulitis are acute abdominal pain and fever, left lower abdominal tenderness, and
mass. Leukocytosis is commonly present together with nausea and vomiting.
Acute appendicitis (choice A) is usually a disease of young adults (and sometimes children). It is
rarely seen in the elderly.
Pancreatitis (choice D), pyelonephritis (choice E), and gall bladder disease (choice C), refer pain to
the mid back, lateral back, and right upper quadrant, respectively.
115. Which of the following would most likely be associated with Type A chronic gastritis
resulting from autoimmune destruction of parietal cells?
A. Decreased growth of luminal bacteria
B. Decreased likelihood of developing gastric carcinoma
C. Decreased plasma concentration of gastrin
D. Increased production of macrocytic red blood cells
E. Increased secretion of pancreatic bicarbonate
The correct answer is D. Autoimmune destruction of parietal cells would lead to decreased secretion
of gastric acid and intrinsic factor. The diminished availability of intrinsic factor would result in poor
absorption of dietary vitamin B12. Over time, the vitamin B12 deficiency could lead to pernicious
anemia, which is characterized by increased production of macrocytes (megaloblasts) by the bone
marrow.
Because of the decrease in gastric acid secretion, luminal bacteria (choice A) would most likely
exhibit increased (not decreased) growth. One of the functions of HCl secreted by the parietal cells is
to sterilize the gastric lumen.
Patients with Type A gastritis have an increased likelihood of developing gastric carcinoma (not
decreased, choice B).
A decrease in acid secretion leads to increased secretion of gastrin (not decreased, choice C) by
antral G cells. This is because low gastric pH (less than 3.0) inhibits gastrin secretion by way of
paracrine release of somatostatin from cells in the gastric mucosa that can sense the acidity. With
decreased parietal cells, the pH of the gastric lumen would rise and remove this inhibitory component.
Because less acid would be delivered to the duodenum with parietal cell destruction, less secretin
would be released into the blood. This would result in decreased pancreatic bicarbonate secretion
(not increased, choice E).
116. What is present in an older woman when there is increased serum levels of calcium and
urinary cAMP levels, below normal levels of serum phosphate, and no other complaints but
weakness?
A. A calcitonin-secreting tumor
B. Primary hyperparathyroidism
C. Primary hypoparathyroidism
D. Thyrotoxicosis
E. Vitamin D deficiency
The correct answer is B. Primary hyperparathyroidism is often asymptomatic and only incidentally
discovered during routine blood work, however, there may be vague complaints of fatigue or
weakness and constipation. These neuromuscular manifestations are caused by the hypercalcemia
that can "hyperstabilize" excitable tissue membranes and reduce normal responsiveness. Primary
hyperparathyroidism incidence increases greatly after age 50 years, and is more common in women
than men. The hypercalcemia is caused by the excess plasma concentration of parathyroid hormone
(PTH). Approximately 80% of cases are caused by a single adenoma in a parathyroid gland. In the

54

other 20% of cases, the hypersecretion of PTH is caused by hyperplasia in multiple parathyroid
glands. The increased PTH also causes renal excretion of phosphate, producing hypophosphatemia.
PTH acts by increasing cAMP formation in target tissues. The cAMP formed in renal tubules can
diffuse into the lumen and be measured in the urine.
Tumors that secrete calcitonin (choice A) include medullary carcinoma of the thyroid and occasionally
small and large cell carcinomas of the lung. Despite calcitonin's high blood concentration, serum
phosphate is rarely abnormal and calcium levels would decrease rather than increase.
Primary hypoparathyroidism (choice C), which is caused by decreased secretion of PTH, is
associated with hypocalcemia and hyperphosphatemia. Furthermore, urinary cAMP concentration
would be decreased. The low calcium in extracellular fluid "destabilizes" excitable tissue membranes
and can lead to spontaneous action potentials that produce tetany.
Thyrotoxicosis (choice D) or hyperthyroidism is associated with sweating, anxiety, heat intolerance,
irritability, fatigue, muscle weakness, tachycardia, and warm moist skin. With vitamin D deficiency
(choice E), serum calcium is decreased because of diminished absorption from the diet. PTH
secretion is increased to compensate, resulting in bone demineralization (osteomalacia).
117. If there is a large retinal detachment in the right eye and nothing noted in the left, how do
the eyes react to a pupillary light reflex examination?
A. Constriction of the right pupil and constriction of the left
B. Constriction of the right pupil and dilatation of the left
C. Dilatation of the right pupil and constriction of the left
D. Dilatation of the right pupil and dilatation of the left
E. No reaction of the right pupil and constriction of the left
The correct answer is D. The eyes are exhibiting the Marcus-Gunn phenomenon. When light strikes
the retina, the pupillary light reflex is automatically triggered, leading to simultaneous constriction of
both pupils. In the absence of adequate light entering the eye, for example, following retinal
detachment or optic neuritis, paradoxic dilatation of the pupils occurs. The retina receives far less light
than it normally would, and the pupils dilate to absorb as much light as possible.
118. Which of the following findings is particularly indicated with a marked elevation in serum
C-reactive protein?
A. Developing autoimmune reaction
B. Ineffective immune response
C. Respiratory compromise
D. Nonspecific inflammation
The correct answer is D. C-reactive protein is one of the most commonly measured acute-phase
reactants, which are a group of serum proteins showing a rapid increase in concentration in response
to any inflammatory process. This finding is entirely nonspecific. It only indicates a recent
inflammatory process. C-reactive protein would be expected to be elevated following a dental
procedure and in cases of pharyngitis (viral and bacterial).
An autoimmune reaction (choice A), which is certainly a concern with streptococcal pharyngitis, is
suggested by the development of a rising ASO (antistreptolysin O) titer weeks after the illness. Acute
phase reactants are not specific to autoimmune processes.
Increases in C-reactive protein indicate a healthy immune response to an infective pathogen. An
ineffective immune response (choice B) would not elicit acute-phase reactions.
Respiratory compromise (choice C) produces changes in arterial blood gases and blood pH. Acutephase reactants do not reflect respiratory status.

55

119. Which of the following is characteristic when there is a blood pressure of 165/95 mm Hg,
tiredness, muscle weakness, polydipsia, while plasma sodium and serum aldosterone is
slightly increased and plasma potassium and plasma renin activity is significantly decreased?
A. Addison's disease
B. Conn's syndrome
C. Cushing's syndrome
D. Type 1 diabetes mellitus
E. Pheochromocytoma
The correct answer is B. Conn's syndrome, or primary hyperaldosteronism, results from an adrenal
tumor that secretes excessive aldosterone. The increased mineralocorticoid effects of aldosterone
lead to renal sodium and water retention (which explains the hypertension) and increased renal
potassium excretion (hypokalemia). The volume expansion also explains the decrease in hematocrit.
The increased blood volume, increased blood pressure, and hypernatremia all tend to suppress renin
secretion in an attempt to compensate for the increased aldosterone.
Addison's disease (choice A), or primary adrenal insufficiency, is characterized by low plasma
concentration of aldosterone, hyponatremia, hypotension, and hyperkalemia.
In Cushing's syndrome (choice C), blood pressure may be increased because of crossover
mineralocorticoid activity of the increased plasma cortisol. Furthermore, cortisol makes blood vessels
more responsive to catecholamines, which could increase peripheral resistance. The combination of
increased blood pressure and hypokalemia would, if anything, tend to suppress secretion of
aldosterone.
Type 1 diabetes mellitus (choice D) is associated with polyuria, polydipsia, muscle weakness, and
chronic tiredness. Renin and aldosterone levels are normal.
Pheochromocytoma (choice E) is another endocrine cause of hypertension. The increased plasma
concentration of catecholamines can cause increased cardiac output and increased peripheral
resistance. Plasma renin activity may be increased because of increased beta receptor activation on
juxtaglomerular cells. This could produce increased aldosterone secretion and subsequent salt
retention.
120. Which of the following BEST describes the inflammatory response that occurs upon
contact with certain plants and presents as a weeping, vesicular, erythematous, and itchy rash
on exposed arms, legs, neck, and face?
A. Erythema nodosum
B. Pemphigus
C. Psoriasis
D. Spongiotic dermatitis
E. Urticaria
The correct answer is D. Spongiotic dermatitis (intracellular edema of the epidermis) is seen in with
contact dermatitis, such as poison ivy exposure. The accumulation of inflammatory cells in the
superficial dermis causes marked edema, which splays epidermal keratinocytes apart and gives a
spongy appearance to intercellular bridges. Grossly, the skin has a weepy appearance with frequent
blistering. Erythema nodosum (choice A) is a form of panniculitis, which is chronic inflammation in the
subcutaneous fat lobules. Erythema nodosum presents as painful erythematous nodules, often with
fever and malaise. It is associated with infections and drug reactions and is not a contact dermatitis.
Pemphigus (choice B) is a genetic blistering disorder caused by the production of antibodies to the
intercellular cement substances in skin and mucous membranes. Psoriasis (choice C) is a common
chronic inflammatory disease causing plaques and scales, typically on elbows, knees, and scalp. The
pathogenesis of psoriasis is still unclear; it may be a complement-mediated autoimmune process.
Urticaria (choice E) is an IgE-driven hypersensitivity process. Urticaria is characterized by wheals
(edematous pruritic plaques) and typically affects the trunk and distal extremities.

56

Microbiology
1. An unvaccinated dental student has a needle-stick accident involving an empty syringe that
had been previously used on a patient with a known hepatitis B infection. Which of the
following is the most probable outcome for the medical worker?
A. Acute hepatitis followed by recovery
B. "Healthy" carrier
C. Persistent infection followed by recovery
D. Persistent infection progressing to chronic hepatitis
E. Subclinical disease followed by recovery
The correct answer is E. Hepatitis B infection can produce a wide variety of clinical outcomes. The
most common outcome (60% to 65%), however, turns out to be subclinical disease followed by
complete recovery. The other choices listed show other possible outcomes, and their statistical
impact. Approximately 20% to 25% of infected persons develop acute hepatitis (choice A), which is
followed in 99% of these cases by recovery and in about 1% of cases by fulminant hepatitis.
Approximately 5% to 10% of cases become "healthy" carriers (choice B). Approximately 4% of cases
develop persistent infection, 67% to 90% of which then recover (choice C) and 10% to 33% of which
have chronic hepatitis (choice D).
2. Biopsy of an ulcerated gastric lesion on a 60-year-old smoker demonstrates the presence of
a gastric carcinoma. If the patient noted that he had severe "heart burn" for the past several
years, with which of the following infectious agents has this type of lesion been most strongly
associated?
A. Epstein-Barr virus
B. Helicobacter pylori
C. Human papilloma virus
D. Molluscum contagiosum virus
E. Schistosoma haematobium
The correct answer is B. The patient has gastric carcinoma, which has been strongly linked, in at
least some studies, to prior gastric infection with Helicobacter pylori. H. pylori has also been
implicated in the etiologies of gastric peptic ulcer, chronic gastritis, and (questionably) gastric
lymphoma. It is believed that treatment of the H. pylori infection with a combination antibiotic
treatment regimen, such as amoxicillin + metronidazole + proton pump inhibitor + bismuth
subsalicylate will decrease the incidence of this type of carcinoma. Epstein-Barr virus (choice A) has
been linked to African Burkitt lymphoma and nasopharyngeal carcinoma. Human papilloma virus
(choice C) has been linked to a variety of warts, condyloma, and genital cancers. Molluscum
contagiosum virus (choice D) is a poxvirus that causes small tumor-like papules of the skin.
Schistosoma haematobium(choice E) has been linked to bladder cancer.
3. The biological attribute of influenza A virus, which allows the sudden appearance of
dramatically new genetic variants, is also present in a limited number of other viral families.
Which of the following viruses also possesses this biological attribute?
A. Coronavirus
B. HIV
C. Measles virus
D. Rotavirus
E. Rubella virus
The correct answer is D. Pandemics of influenza A can be caused by the ability of the virus to
undergo dramatic genetic changes of type by reassortment of its segmented RNA genome, a trait
called genetic shift. The only virus on the list that possesses a segmented genome is the rotavirus, in
the reovirus family, which possesses 10-11 segments in its genome. Coronavirus (choice A) is not
segmented and is a cause of the common cold. HIV (choice B) is not segmented and is known for its
genetic drift (minor mutational changes over time caused by an error-prone polymerase), not genetic
shift. Measles virus (choice C) is not segmented and is controlled largely by vaccination. The virus is
known as a paramyxovirus. Rubella virus (choice E) is not segmented. The togavirus causes
arthralgia (primarily in young women), fever, malaise, coryza, lymphadenopathy, and a fine
maculopapular rash. Three months after a needle-stick exposure to blood from a patient with hepatitis

B, a nurse is evaluated for infection with the virus. Laboratory results reveal: HBsAg absent anti-HBs
antibody absent IgM anti-HBc present IgG anti-HBc absent HBeAg absent
4. On the basis of these results, which of the following most accurately describes the nurse's
hepatitis B status?
A. She had been effectively vaccinated against hepatitis B bfore the needle-stick exposure occurred
B. She has mounted an inappropriate antibody response to hepatitis B as a result of an
immunocompromised state
C. She is a carrier of hepatitis B
D. She is actively infected with hepatitis B
E. She was not infected with hepatitis B
The correct answer is choice D. The nurse's elevated IgM anti-HBc indicates that she was infected
with hepatitis B. Formerly, HBsAg (surface antigen) and anti-HBsAg (antibody to surface antigen)
were used exclusively to determine this. Typically, HBsAg is positive for up to 6 months, and anti
HBsAg is positive for years after that. Unfortunately, this simple scheme has the disadvantage that
many patients have a 2-week to 4-month "window" period, when the surface antigen (HBsAg) and the
antibody (anti-HBs) are not detectable. Presumably, for a relatively brief period, HBsAg production
exactly matches antibody production, and the two coprecipitate such that neither free species is
present in adequate concentration to be detectable. This problem can be circumvented by concurrent
measurements of other antigens and antibodies, including HBeAg, anti HBe, and anti-HBc (HBcAg is
not reliable). During the window period, IgM anti-HBc may be the only marker of recent HBV infection,
as it is in this nurse.
If the nurse had been effectively vaccinated for hepatitis B (choice A), she would have had an
elevated anti-HBs antibody level and no HBsAg present in the serum. Anti-HBc antibody would have
been absent as well. The antibody response to hepatitis B infection was appropriate in this person,
which argues against immunocompromise (choice B). Carriers (choice C) have elevated anti-HBs
and may have persistently elevated HBsAg (in approximately 10% of cases). IgG anti-HBc (not IgM)
predominates in these chronic patients. Had she not been infected with hepatitis B at all (choice E),
IgM anti-HBc would be absent.
5. An important factor in determining the likelihood of converting to HIV+ status after a
needlestick injury is which of the following?
A. Depth of the needlestick
B. Volume of blood in the needle
C. Type of needle
D. Health status of source patient
E. All of the above
The correct answer is E. If a healthcare worker is accidently stuck with an HIV-infected bloody
sharp, the worker may or may not contract HIV disease. Conversion following needlestick accidents is
low overall, sometimes estimated at 0.3% (3 per thousand). Several factors, however, have been
identified that make it MORE likely that the accident will transfer the virus to the worker. They include
a large volume of blood, a hollow-bore needle, visible blood on the needle, a deep (not superficial)
stick into deep skin or muscle, and a source patient in late stages of AIDS. Followup of needlestick
injuries is a rapidly evolving field, so be sure to use your most recent notes and test reviews to stay on
top of this subject.
6. A 16-year-old boy with sickle cell disease is hospitalized for a severe infection. His
symptoms include fever, chills, cough, and chest pain. Bacteria from the patient's sputum
yield optochin-sensitive organisms with a positive Quellung reaction. Which of the following is
the most likely pathogen?
A. Escherichia coli
B. Haemophilus influenzae
C. Klebsiella pneumoniae
D. Neisseria gonorrhoeae
E. Streptococcus pneumoniae

The correct answer is E. The combination of optochin sensitivity and positive Quellung reaction is
characteristic of a single organism, Streptococcus pneumoniae (diplococcus). The Quellung reaction
is a capsular swelling caused by contact with specific capsular antisera. The other encapsulated
organisms that have Quellung-positive reactions are Haemophilus influenzae (choice B), Neisseria
meningitidis, and Klebsiella pneumoniae (choice C). None of these organisms, however, are optochin
sensitive. The other choices, Escherichia coli (choice A) and Neisseria gonorrhoeae (choice D), are
not encapsulated.
7. A 33-year-old woman has recently returned from overseas and presents with severe, acute,
right upper quadrant abdominal pain, bloody diarrhea, and tenesmus. CT scan of the liver
demonstrates lesions identified as abscesses. Which of the following organisms is the most
likely cause of her illness?
A. Ascaris lumbricoides
B. Entamoeba histolytica
C. Enterobius vermicularis
D. Salmonella typhi
E. Shigella species
The correct answer is B. The patient probably has hepatic amebiasis, a life-threatening complication
of intestinal infection with Entamoeba histolytica. E. histolytica is transmitted by way of the fecal-oral
route. Intestinal colonization (which may be asymptomatic) always precedes infection of the liver. The
abscesses generally contain necrotic debris, with amoebae located along the edges of the abscess.
Patients often present initially with frequent bloody, small-volume stools, often associated with fever,
abdominal cramps, tenesmus, and fecal urgency. When untreated, this condition can result in the
development of hepatic amebiasis. Ascaris lumbricoides (choice A) can cause intestinal obstruction,
but does not usually cause hepatic abscesses. Enterobius vermicularis (choice C) is the pinworm,
which inhabits the rectum. Bloody diarrhea can also be seen with some strains of Salmonella (choice
D) and Shigella (choice E), but these organisms do not usually cause hepatic abscesses. Bacterial
causes of hepatic abscesses include E. coli, Klebsiella, Streptococcus, Staphylococcus, Bacteroides,
and Pseudomonas.
8. A 65-year-old man presents with fever, severe headache, and nuchal rigidity. Lumbar
puncture reveals cloudy cerebrospinal fluid (CSF) with elevated neutophils, elevated protein,
and decreased glucose. Which of the following is the most probable etiologic agent of this
condition in a patient of this age?
A. Arbovirus
B. Herpesvirus
C. Mycobacterium tuberculosis
D. Streptococcus pneumoniae
E. Neisseria meningitidis
The correct answer is D. The clinical manifestations (fever, headache, and nuchal rigidity), along
with the CSF findings (increased neutrophils, elevated protein, and reduced glucose), strongly
indicate acute pyogenic (bacterial) meningitis as the underlying condition. Of the microorganisms
listed, either Neisseria meningitidis or Streptococcus pneumoniae can cause this form of meningitis;
however, Streptococcus pneumoniae is by far the most frequent organism causing acute meningitis in
elderly patients. Arboviruses and herpesviruses (choices A and B) can cause an encephalitis
characterized by lymphocytic infiltration of the brain parenchyma and leptomeninges. In this case,
CSF findings would include an increased number of lymphocytes and a normal glucose concentration,
although the protein level in CSF would be increased. Mycobacterium tuberculosis(choice C) may
cause a chronic meningoencephalitis, with a prolonged clinical course. It is characterized
pathologically by a dense granulomatous infiltrate of the base of the brain. Associated CSF findings
include increased lymphocytes and normal or slightly decreased glucose.
9. A child has a history of recurrent infections with organisms having polysaccharide antigens
(i.e., Streptococcus pneumoniae and Haemophilus influenzae). This susceptibility can be
explained by a deficiency of
A. C3 nephritic factor
B. C5
C. IgG subclass 2
D. secretory IgA

The correct answer is C. IgG is the predominant antibody in the secondary immune response. IgG
subclass 2 is directed against polysaccharide antigens and is involved in the host defense against
encapsulated bacteria. C3 nephritic factor (choice A) is an IgG autoantibody that binds to C3
convertase, making it resistant to inactivation. This leads to persistently low serum complement levels
and is associated with Type II membranoproliferative glomerulonephritis. C5 (choice B) is a
component of the complement system. C5a is an anaphylatoxin that effects vasodilatation in acute
inflammation. It is also chemotactic for neutrophils and monocytes and increases the expression of
adhesion molecules. A deficiency of C5a would affect the acute inflammatory response against any
microorganism or foreign substance. Secretory IgA (choice D) is the immunoglobulin associated with
mucous membranes. Selective IgA deficiency is the most common hereditary immunodeficiency. In
this disorder, there is failure of the B cell to switch the heavy chain class from IgM to IgA. Patients
have an increased incidence of sinopulmonary infections, diarrhea, allergies, and autoimmune
diseases.
10. A sexually active 25-year-old man with epididymitis and orchitis demonstrates a prominent
leukocytic infiltrate with numerous neutrophils on biopsy. Which of the following organisms is
the most likely cause of this man's infection?
A. Escherichia coli
B. Mycobacterium tuberculosis
C. Neisseria gonorrhoeae
D. Pseudomonas sp.
E. Treponema pallidum
The correct answer is C. Acute epididymitis and orchitis with prominent neutrophils in a sexually
active male are most likely caused by infection with Neisseria gonorrhoeae or Chlamydia trachomatis.
N. gonorrhoeae can produce a nonspecific pattern of acute inflammation (nonspecific epididymitis and
orchitis) or can be sufficiently severe as to cause frank abscesses within the epididymis.
Escherichia coli(choice A) is an important cause of nonspecific epididymitis and orchitis in children
with congenital genitourinary abnormalities and in older men. Mycobacterium tuberculosis(choice B)
can cause tuberculosis of the epididymis and testes, characterized by granuloma formation.
Pseudomonas sp.(choice D) has been implicated as an important cause of nonspecific epididymitis
and orchitis in older men. Treponema pallidum(choice E), the causative agent of syphilis, can cause
testicular involvement with (in later stages) gumma formation, endarteritis, or a prominent plasma cell
infiltrate.
11. A 5-year-old child develops a febrile disease with cough, a blotchy rash, and cervical and
axillary lymphadenopathy. Also noted is an erythematous, maculopapular rash behind the ears
and along the hairline, involving the neck and, to a lesser extent, the trunk. Examination of this
patient's oropharynx would likely reveal which of the following lesions?
A. Adherent thin, whitish patch on gingiva
B. Cold sores on the lips
C. Curdy white material overlying an erythematous base on the oral mucosa
D. Large shallow ulcers on the oral mucosa
E. Multiple small white spots on the buccal mucosa
The correct answer is E. The disease described is measles (rubeola), which has the typical
presentation described in the question stem. Measles is caused by a Morbillivirus, an RNA virus
belonging to the Paramyxovirus family. Koplik's spots, which are pathognomonic for measles, are
small, bluish-white spots on the buccal mucosa in the early stages of measles. These lesions appear
just before the onset of the characteristic rash which is a brick red irregular maculopapular rash that
onsets 3-4 days after the onset of the prodrome (which can also involve the extremities) and fade as
the rash develops. This is usually a prodrome of fever, coryza, cough, conjunctivitis, malaise, irritibility
and photophobia as well as koplik's spots. Leukoplakia is a premalignant condition characterized by
adherent whitish patches on the gingiva (choice A) and other sites in the oral cavity. Histologically,
they are similar to hyperkeratoses. Cold sores of the lips (choice B) are due to infection with herpes
viruses. Candida infection (thrush) produces curdy white material loosely attached to an erythematous
base (choice C). When removed there is typically a painful lesion that may bleed. Aphthous ulcers
are large shallow ulcers of the oral mucosa (choice D), commonly known as canker sores. No
systemic involvement is seen.

12. A 38-year-old woman vacationing in Connecticut is bitten by a tick and develops chronic
arthritis of the knee and hip joints and paralysis of the left facial muscles. A physical
examination during the early stages of the disorder would most likely have revealed
A. aphthous ulcers in the mouth
B. erythema chronicum migrans
C. flaccid paralysis of limb flexors
D. purpuric lesions in a bathing trunk distribution
E. spastic paralysis of limb extensors
The correct answer is B. Lyme disease should be suspected in a patient who is bitten by a tick in the
northeastern United States. Lyme disease was named after a township in eastern Connecticut where
the disease was endemic. The disease is spread by way of a tick vector that transmits a spirochete
that causes a systemic illness. Erythema chronicum migrans is usually the first sign of the illness. This
is a large red patch on the buttocks or chest that slowly expands as the center blanches. Generally,
patients also have constitutional symptoms, such as fever and chills, during this phase. Stiff neck may
develop, along with other signs of meningeal irritation, because of an aseptic meningitis. Other
neurologic complications of Lyme disease include Bell palsy caused by involvement of branches of
the facial nerve. Arthritis is a prominent feature in approximately half the patients with Lyme disease.
It tends to appear several months after the infection but may persist for several years. The course of
the chronic arthritis shows exacerbations and remissions; the most commonly affected joints are the
knees and hips. Cardiac abnormalities in Lyme disease include pericarditis and heart block.
Skin manifestations do not include aphthous ulcers (choice A). Flaccid or spastic paralysis of limbs
(choices C and E) does not accompany Lyme disease; neurologic involvement is generally limited to
cranial nerves and meningitis. Purpura (choice D) is associated with vasculitis and does not occur in
Lyme disease.
13. A 31-year-old HIV-positive man develops a severe pneumonia. Lower respiratory tract
secretions stained with methenamine silver stain demonstrate cup-shaped cysts with sharply
outlined walls. Which of the following organisms is the most likely pathogen in this case?
A. Candida albicans
B. Giardia lamblia
C. Haemophilus influenzae
D. Pneumocystis carinii
E. Streptococcus pneumoniae
The correct answer is D. The organism described is Pneumocystis carinii, which is an opportunistic
parasite that seems to be more closely related to fungi than to protozoa. Its cyst form, when stained
with silver stains, has the distinctive appearance described in the question stem, and is typically found
in frothy material that occupies the lumen of alveoli. Pneumocystis pneumonia is a common infection
among AIDS patients, and is very uncommon in other clinical settings. Formerly, many AIDS patients
died with Pneumocystis pneumonia, but the combination of early drug treatment (with
trimethoprim/sulfamethoxazole
or
pentamidine)
and
prophylaxis
(usually
with
trimethoprim/sulfamethoxazole) has decreased the number of fatal infections. In severe cases,
Pneumocystis infection can sometimes be demonstrated in extrapulmonary sites. Candida
albicans(choice A) can infect the lung and stain with methenamine silver, but the respiratory tract
secretions would probably include fungal hyphae and yeast forms. Giardia lamblia (choice B) causes
diarrhea, rather than pneumonia. Haemophilus influenzae(choice C) and Streptococcus
pneumoniae(choice E) are bacteria that would not stain with silver stains and are not especially
common in HIV-positive patients.
14. A neonate is born in very poor condition, with a severe, generalized encephalitis. If fever is
noted as well as temporal lobe involvement, which of the following viruses is the most likely
pathogen in this setting?
A. Eastern equine encephalitis virus
B. Herpes simplex type II
C. Herpes zoster-varicella virus
D. Poliomyelitis virus
E. St. Louis encephalitis virus

The correct answer is B. Viral causes of neonatal encephalitis include three members of the herpes
family of viruses: herpes simplex I, herpes simplex II, and cytomegalovirus. All three types can have
devastating effects on the neonate, with extensive CNS damage leading to mental retardation,
seizures, and focal neurologic problems. Acyclovir may be of some help in modifying these infections,
but both treatment and prognosis remain very problematic. Herpes simplex encephalitis presents with
nonspecific symptoms: a flu-like prodome followed by headache, fever, behavioral and speech
disturbances. A distinguishable feature is a propensity to involve the temporal lobe with mass effect
on imaging studies and temporal lobe seizure foci on EEG's. Eastern equine encephalitis virus
(choice A) and St. Louis encephalitis virus (choice E) are causes of epidemic encephalitis but are not
the most likely cause of neonatal encephalitis. Herpes zoster-varicella virus (choice C), unlike herpes
simplex, is not usually a cause of neonatal encephalitis. Poliomyelitis virus (choice D) is a
gastrointestinally transmitted virus that is not usually encountered in neonates (or anyone else in the
U.S. currently).
15. A 23-year-old woman with a history of sickle cell disease presents with fever and severe
bone pain localized to her left tibia. An x-ray film reveals a lytic lesion, and blood cultures
reveal infection. A bone culture grows gram-negative rods. Which of the following best
describes the infecting organism?
A. It is comma-shaped and sensitive to acidic pH
B. It is a facultative intracellular parasite
C. It is motile and does not ferment lactose
D. It is motile and oxidase positive
E. It is a nonmotile facultative anaerobe
The correct answer is C. The presence of sickle cell disease in a question stem is usually a
significant clue. This question tests the fact that patients with sickle cell anemia are more susceptible
to osteomyelitis caused by Salmonella. The patient's fever, bone pain, and x-ray results indicate
osteomyelitis. Note, however, that Staphylococcus aureus (gram-positive coccus) is the most
common cause of osteomyelitis in sicklers and nonsicklers. If it has not been ruled out on bone
culture, you should have looked for it in the answer choices. Notice that you were required to know
more than just the organism's name; you needed to know its distinguishing features. Choice C
describes Salmonella (a gram-negative rod) accurately. Salmonella exists in more than 1,800
serotypes and is known to contaminate poultry.
A comma-shaped organism that is sensitive to acidic pH (choice A) is Vibrio cholerae, a gramnegative rod that causes severe enterotoxin-induced diarrhea, with "rice water" stools and
dehydration. The toxin acts by stimulating adenylyl cyclase to overproduce cAMP in the brush border
of the small intestine. A facultative intracellular parasite (choice B) is Legionella, a catalase-positive
gram-negative rod. It contaminates air-conditioning cooling towers and causes Legionnaire disease (a
type of pneumonia). A motile and oxidase-positive organism (choice D) is Pseudomonas, a gramnegative rod with pili that sometimes produces a polysaccharide slime layer. P. aeruginosa is the
prototype and commonly colonizes the lungs of patients with cystic fibrosis. It is associated with bluegreen pus. A nonmotile, facultative anaerobe (choice E) is Shigella, a gram-negative rod that does
not produce H2S. All Shigella contain an endotoxic lipopolysaccharide. The organism causes bacillary
dysentery, with abdominal cramps, fever, and mucoid, bloody diarrhea.
16. A patient suffers an infarct in the territory of the middle cerebral artery. Pathologic
examination of the patient's brain would likely show
A. caseous necrosis
B. coagulative necrosis
C. enzymatic fat necrosis
D. gangrenous necrosis
E. liquefactive necrosis
The correct answer is E. Liquefactive necrosis occurs in brain or other neural tissues and in
pancreatic tissue. In this type of necrosis, the tissue appears liquefied under the microscope, without
preservation of cell outlines. Liquefactive necrosis can also be seen in some bacterial infections,
especially those caused by pyogenic Staphylococci, Streptococci, or certain coliform bacteria.
Caseous necrosis (choice A) is generally an indication of infection by Mycobacterium tuberculosis.

The term caseous refers to the appearance of the tissue, i.e., soft, white necrotic areas that have a
cheese-like appearance. Microscopically, the necrotic areas are lightly eosinophilic (stain light pink),
with little or no discernible cellular detail. The eosinophilia reflects staining of residual cellular proteins.
Coagulative necrosis (choice B) is a more common type of necrosis, characteristic of anoxic injury
and most infarcts. Cellular outlines are preserved, but proteins are denatured, and the cells stain in an
eosinophilic manner. Enzymatic fat necrosis (choice C) is seen primarily with pancreatic injury when
pancreatic lipases are released and digest fat to form free fatty acids. These fatty acids complex with
calcium, resulting in the production of calcium soaps (saponification) in the pancreatic tissue or in
extrapancreatic fatty tissues (eg, omentum). In gangrene (choice D), bacterial infection is
superimposed on a background of massive necrosis and putrefaction.
17. A 4-year-old girl presents with a maculopapular rash on her hands and feet and painful
ulcers distributed anteriorly on her lips, palate, tongue, and buccal mucosa. Systemic features
and lymphadenopathy are absent. Which of the following viruses is most likely to have caused
this disorder?
A. Coronavirus
B. Coxsackievirus type A16
C. Herpes simplex virus type 1
D. Parainfluenza type 3
E. Rhinovirus
The correct answer is B. Hand-foot-and-mouth disease is characterized by the appearance of ulcers
in the mouth and a maculopapular or vesicular rash on the hands and feet. It is most frequently
caused by coxsackievirus type A16, although other coxsackieviruses have occasionally been
implicated. The disease usually affects young children. Systemic features and lymphadenopathy are
absent, and recovery is uneventful.
Coronavirus (choice A) is a cause of the common cold. Herpes simplex virus type 1 (choice C)
causes a variety of diseases, including gingivostomatitis, pharyngotonsillitis, herpes labialis, genital
herpes, keratoconjunctivitis, and encephalitis. Parainfluenza virus (choice D) is responsible for croup.
Croup, or acute laryngotracheobronchitis, is an acute febrile illness with stridor, hoarseness, and
cough. Rhinovirus (choice E) is a member of Picornaviridae. It is the most common cause of the
common cold.
18. Evaluation of an adult third world immigrant to this country demonstrates chronic
headaches, chronic mild nuchal rigidity, and chronic inflammatory infiltrate of the CSF with
lymphocytes, plasma cells, macrophages, and fibroblasts. If an acid-fast organism is
identified, which of the following is the most likely etiologic agent?
A. Herpes virus
B. Mumps virus
C. Mycobacterium tuberculosis
D. Neisseria meningitidis
E. Streptococcus pneumoniae
The correct answer is C. Although this may seem to be a difficult clinical question, by simply
knowing that mycobacteria are acid-fast organisms, one can answer the question. Nuchal rigidity
suggests meningitis. It is convenient to classify meningitis based on the cerebrospinal fluid (CSF)
findings: (1) acute pyogenic meningitis if neutrophils are markedly increased, (2) acute lymphocytic
meningitis if lymphocytes (alone) are markedly increased, and (3) chronic meningitis if lymphocytes,
plasma cells, macrophages, and fibroblasts are increased. This patient has chronic meningitis. The
classic cause of chronic meningitis is tuberculosis, whose etiologic agent is Mycobacterium
tuberculosis. Other causes include other indolent meningeal infections such as syphilis, brucellosis,
and chronic fungal infections. The granulomas that are characteristic findings in other tissues may or
may not be present in the meningeal tissue, and are usually not recognizable in CSF. Tubercular
meningitis is now uncommon in this country. In immigrants from third world countries, a history of
pulmonary tuberculosis may be distant, undiagnosed, or deliberately concealed. Herpes virus (choice
A) and mumps virus (choice B) are causes of acute lymphocytic meningitis. These organisms are not
acid-fast. Neisseria meningitidis(choice D) is a gram-negative bacteria. Streptococcus
pneumoniae(choice E) is a gram-positive bacteria. Both are causes of acute pyogenic meningitis.
Neither is acid-fast.

19. A 45-year-old white man with a history of alcohol abuse and periodontal disease has a
spiking fever, chills, and lung consolidation. A chest x-ray shows a cavity in the right lower
lobe that has an air/fluid level. Based on the clinical presentation, which of the following would
be the most likely cause of the symptoms?
A. Anaerobic bacteria
B. Aspergillus fumigatus
C. Entamoeba histolytica
D. Staphylococcus aureus
E. Streptococcus pyogenes
The correct answer is A. Anaerobic bacteria derived from the oral flora in the clinical setting of
periodontal disease are the most common isolates from lung abscesses. Single lung abscesses are
the most common pattern, with the superior segment of a lower lobe or the posterior segment of an
upper lobe being affected most often. Prominent members of the oral anaerobic flora include
fusobacteria, prevotella, and aerobic spirochetes. Aspergillus fumigatus(choice B) presents in the
lung as hemorrhagic infarctions, aspergillomas (fungus balls) in cavitary tuberculosis cavities, or as
allergic bronchopulmonary aspergillosis. Entamoeba histolytica(choice C) is associated with
pulmonary abscesses as an extension of a liver abscess across the diaphragm. Staphylococcus
aureus(choice D) usually presents as multiple lung lesions in noncontiguous sites because the
spread is embolic. The source of the infection is usually tricuspid endocarditis in intravenous drug
abusers. Streptococcus pyogenes(choice E) typically produces a bronchopneumonia pattern
following an upper respiratory infection.
20. A young mother takes her infant to the pediatrician, who notices the infant's teeth are
yellow and brownish striped. The antibiotic this mother most likely took during pregnancy
A. inhibits aminoacyl-tRNA binding
B. inhibits peptidyl transferase
C. interferes with cell wall synthesis
D. is a large, cyclic lactone-ring structure
The correct answer is A. This question relates to an NDBE favorite side effect: the teeth mottling
that occurs when a child is exposed to tetracycline in utero. You should remember that tetracycline is
contraindicated in pregnancy and early childhood up to approximately age 8 years. Tetracycline is a
bacteriostatic drug that binds to the 30s subunit of ribosomes, preventing aminoacyl-tRNA from
binding with complementary mRNA. This inhibits peptide bond synthesis. Resistance is plasmidmediated. Inhibition of peptidyl transferase (choice B) occurs with chloramphenicol, a broad-spectrum
bacteriostatic agent that binds to the 50s subunit of ribosomes. Resistance is plasmid-mediated. It has
high toxicity (gastrointestinal disturbances, aplastic anemia, and gray baby syndrome), so it is used
mainly in severe infections or as a topical agent. Interference with cell wall synthesis (choice C)
occurs with penicillins and cephalosporins, the beta-lactam antibiotics. Resistance to these drugs
appears in organisms that have developed beta-lactamases (penicillinases), enzymes that destroy the
beta-lactam ring of these medications. The wider spectrum ampicillin, amoxicillin, ticarcillin, and
carbenicillin are particularly penicillinase susceptible. Large, cyclic, lactone-ring structures (choice D)
describe the macrolides: erythromycin, azithromycin, and clarithromycin. They inhibit bacterial protein
synthesis by reacting with the 50s ribosomal subunit and preventing the release of the uncharged
tRNA. Resistance is plasmid-mediated. Common side effects include gastrointestinal irritation, skin
rashes, and eosinophilia. Erythromycin is a popular choice for patients with penicillin hypersensitivity.
It is a cytochrome p450 inhibitor and therefore must be used with caution in patients taking other
drugs.
21. A 38-year-old woman complains of cold and painful fingertips, as well as difficulty
swallowing and indigestion. Physical examination shows a thickened, shiny epidermis over
the entire body, with restricted movement of the extremities, particularly the fingers, which
appear claw-like. Which of the following autoantibodies will likely be found in this patient's
serum?
A. Anti-DNA topoisomerase I (anti-Scl-70)
B. Anti-double-stranded DNA (ds DNA)
C. Anti-IgG
D. Anti-Sm
E. Anti-SS-A

The correct answer is A. This patient has systemic sclerosis, also called scleroderma. Antibodies to
topoisomerase I (anti-Scl-70) occur in up to 70% of patients with diffuse systemic sclerosis, but only
rarely in other disorders. Systemic sclerosis is characterized intitially by excessive fibrosis and edema
of the skin, especially the hands and fingers, producing sclerodactyly (characteristic changes in the
fingers, which resemble claws). Raynaud phenomenon is common. The diffuse type of systemic
sclerosis generally spreads to include visceral organs, such as the esophagus (producing dysphagia),
the lungs (producing pulmonary fibrosis), the heart (leading to heart failure or arrhythmia), and the
kidneys (renal failure causes 50% of scleroderma deaths). A more restricted variant of systemic
sclerosis with a somewhat more benign course is CREST syndrome (Calcinosis, Raynaud syndrome,
Esophageal dysmotility, Sclerodactyly, and Telangiectasia), characterized by the presence of anticentromere antibodies (although 10% of CREST patients have antitopoisomerase antibody also).
Anti-ds DNA (choice B) and anti-Sm (Smith antigen; choice D) are characteristic of systemic lupus
erythematosus (SLE) but are not common in patients with systemic sclerosis. Rheumatoid factor is an
autoantibody directed against IgG (choice C). It is found in patients with rheumatoid arthritis. Anti-SSA (choice E) is typically seen in Sjogren syndrome (although it may also be seen in SLE).
22. A young patient presents with bilateral swelling of the parotid gland, accompanied by
swelling of one testicle. Which of the following is the most likely diagnosis?
A. Gonorrhea
B. Mumps
C. Nonspecific epididymitis and orchitis
D. Syphilis
E. Tuberculosis
The correct answer is B. Mumps, once a common childhood disease, is now much less common
because of immunization. It still should be considered as a potential cause of parotitis in medically
underserved populations, however. It is caused by the paramyxovirus and is spread by droplet
infection. Although testicular involvement by mumps in school-aged children is extremely uncommon,
postpubertal males who develop this viral infection have a 20-30% chance of subsequently
developing acute interstitial orchitis, usually in one testis. Microscopically, the inflammatory reaction
consists of lymphocytes, plasma cells, and macrophages. Although the process can be locally
destructive, sterility does not usually develop because typically only one testicle is involved. The
incubation period is 14-21 days before the onset of symptoms. Patients initially present with painful,
swollen salivary glands, usually the parotid. In unvaccinated individuals, other tissues frequently
become involved. These tissues include the testes, pancreas, and meninges. Neglected gonorrhea
infection (choice A) of the epididymis and testis occurs in the setting of previous sexual activity and
would not usually cause parotitis. Nonspecific epididymitis and orchitis (choice C) occur in children
with genital urinary congenital abnormalities (gram-negative rods), in sexually active young adults
(Chlamydia and gonorrhea), and in older men (gram-negative rods and Pseudomonas). Syphilis
(choice D) can cause orchitis with either gumma formation or a leukocytic infiltrate with prominent
plasma cells. Tuberculosis (choice E) can cause granulomatous involvement with acid-fast
organisms of the epididymis and testis.
23. Which of the following cell surface markers is required for lysis of IgG-coated target cells
(antibody-dependent, cell-mediated cytotoxicity, or ADCC) by natural killer cells?
A. CD3
B. CD16
C. CD19
D. CD21
E. CD56
The correct answer is B. CD16 is a cell surface marker used to identify natural killer (NK) cells
(lymphocytes lacking most T- and B-cell markers). CD16 is an Fc receptor for IgG, allowing the NK
cells to bind to the coated target cell during ADCC, facilitating lysis. CD3 (choice A) is a fivepolypeptide cluster that represents the nonvariable part of the T-cell receptor complex. The variable
part is able to rearrange itself to adapt to specific antigens. NK cells are CD3-negative. CD19 (choice
C) is a B-cell marker. It is a signal-transducing molecule that is expressed in early B-cell
differentiation. NK cells are negative for CD19. C21 (choice D) is also a B-cell marker. It is a
complement receptor, and is also the same receptor the Epstein-Barr virus uses to bind to cells during
infection. CD56 (choice E) is an NK cell marker, but is not involved with ADCC.

24. Which of the following organisms would most likely cause infection after a partial
sterilization procedure that killed vegetative cells but did not kill spores?
A. Chlamydia
B. Clostridium
C. Escherichia
D. Pseudomonas
E. Streptococcus
The correct answer is B. Although bacterial spores are extensively discussed in microbiology
courses, you should be aware that only Bacillus (aerobic) and Clostridium (anaerobic) species
produce spores. This means that the list of diseases related to bacterial spore formation is also small:
anthrax (Bacillus anthracis), some forms of food poisoning (Bacillus cereus), botulism (Clostridium
botulinum), tetanus (Clostridium tetani), gas gangrene (Clostridium perfringens and others), and
pseudomembranous colitis (Clostridium difficile). This is one of the reasons why some clostridium
infections are difficult to treat. With respect to dentistry, clostridium difficile induced colitis can be seen
following treatment with antibiotics, especially clindamycin. Remember, clindamycin is commonly
used to treat a variety of dental-related infections because it has excellent anaerobic coverage. Note
also that spores (bacillus stearothermophilus) are used to test autoclave function.
25. An elderly man develops a vesicular rash localized to a narrow circumferential band on one
side of his chest. The rash is very painful and the vesicles are confluent with some ulceration.
No other significant findings are demonstrated on physical examination. Which of the
following diagnoses is most likely to be correct?
A. Chicken pox
B. Herpes simplex I infection
C. Herpes simplex II infection
D. Measles
E. Shingles
The correct answer is E. This is shingles, the recurrent form of herpes zoster infection, which is
usually (except in the case of immunosuppressed patients) confined to a single, unilateral dermatome.
Isolated vesicles may be seen outside the dermatome. The primary herpes zoster infection (chicken
pox) precedes the development of shingles by years or decades; the prevalence of shingles rises
steadily with age, to the point that 1% of people older than 80 years of age have the condition.
Shingles lesions are infections and should be considered an infectious hazard in the hospital setting.
Acyclovir can ameliorate the condition. Varicella, or chicken pox (choice A), is the primary form of
herpes zoster infection and affects face and trunk diffusely. It is generally seen in young patients. The
macules evolve to papules and vesicles then crust over; all stages are simultaneously present.
Herpes simplex I (choice B) affects oral and perioral sites. Herpes simplex II (choice C) primarily
affects genital sites, producing a painful set of beefy red lesions. Measles (choice D) causes a
blotchy rash, rather than a dermatomal one. Patients present with a maculopapular rash that is brick
red. It begins on the head and neck and spreads downward and outward. Koplik spots often appear
on the buccal mucosa.
26. A 35-year-old man who recently traveled to a third world country develops chronic, severe
dysentery with ulceration of the cecum. Biopsy reveals 15-40 micron amoebae with ingested
erythrocytes and small nuclei with distinctive tiny central karyosomes. If hepatomegaly is a
late complication of this patient's condition, which of the following organisms is the most
likely culprit?
A. Isospora belli
B. Cryptosporidium parvum
C. Entamoeba histolytica
D. Giardia lamblia
E. Microsporidia spp.
The correct answer is C.Entamoebahistolytica is the usual cause of intestinal amebiasis, and has
the microscopic features described in the question stem. A particularly helpful (but not always
present) feature of this organism is the presence of ingested red blood cells within the amoebae.

10

These amoebae cause flask-shaped ulceration of the intestinal mucosa and submucosa, with a
particular propensity for involving the cecum and ascending colon. The disease manifestations range
from none (asymptomatic carriers) to mild chronic diarrhea, to severe, purging dysentery. In
symptomatic cases, the liver may develop destructive amoebic liver abscesses that tend to become
secondarily (and potentially life-threateningly) infected by bacteria. Patients initially present with
recurrent diarrhea and abdominal cramps. As the condition worsens severe colitis may be present.
Then hepatic involvement may occur in fulminant cases. Isospora belli (choice A), Cryptosporidium
parvum (choice B), and Microsporidia spp. (choice E) are commonly seen in AIDS patients. These
organisms can cause severe diarrhea. None of these organisms are amebae. Giardia lamblia(choice
D) is a small intestinal protozoa with a distinctive pear-shaped morphology that appears to have a
"face."
27. Which of the following characteristics would help to differentiate Streptococcus agalactiae
from Streptococcus pneumoniae?
A. Alpha-hemolysis
B. Carbohydrate capsule
C. Cytochrome enzyme system
D. Growth in bile
E. Oxacillin sensitivity
The correct answer is A. Streptococci are usually initially speciated by their hemolytic capacity on
sheep blood agar. Beta-hemolytic streptococci include groups A, B, and D. S. agalactiae is the classic
group B streptococcus. The non-beta-hemolytic streptococci consist principally of the pneumococci
and the viridans group. These groups are both alpha-hemolytic. Streptococcus is a genus of grampositive facultative cocci occurring in pairs or chains. The genus is separable into the pyrogenic
group, viridans group, enterococcus group, and lactic group. Because streptococci are so common, it
is essential that the dental student understand how to differentiate all the different species. Note that
alpha hemolysis produces a green halo around the colony in RBC culture, whereas beta hemolysis
produces a clear halo. Both S. agalactiae and pneumococcus have a carbohydrate capsule (choice
B), an important virulence factor and means of subtyping streptococcal species. None of the
streptococci use cytochrome enzymes (choice C). They derive all of their energy from the
fermentation of sugars to lactic acid. Neither pneumococcus nor S. agalactiae can grow in bile
(choice D). This ability is specific for the enterococcus group (group D) of streptococci. Both
pneumococci and S. agalactiae are usually treated with penicillin-type antibiotics, although group B
streptococci require a penicillinase-resistant type such as oxacillin (choice E).
28. A 73-year-old woman with a history of diabetes presents with left ear pain and drainage of
pus from the ear canal. She has swelling and tenderness over the left mastoid bone. Which of
the following microorganisms is the most likely causative agent?
A. Haemophilus influenzae
B. Klebsiella pneumoniae
C. Mucor sp.
D. Pseudomonas aeruginosa
E. Streptococcus pyogenes
The correct answer is D.Pseudomonas aeruginosa causes malignant otitis externa, which is a
severe necrotizing infection of the external ear canal. Infection tends to spread to the mastoid bone,
temporal bone, sigmoid sinus, base of the skull, meninges, and brain. Patients at increased risk
include the elderly, those with diabetes, and the immunocompromised. Haemophilus
influenzae(choice A) produces a variety of clinical syndromes. H. influenzae is the third most
common cause of meningitis in children aged 1 month to 18 years. It is the most common cause of
acute epiglottitis, the most common cause of purulent bacterial conjunctivitis, and the second most
common cause of otitis media. Patients at risk include those with COPD and cystic fibrosis, alcoholics,
splenectomized patients, and young patients. Klebsiella pneumoniae(choice B) is a gram-negative
organism that produces a necrotizing pneumonia in diabetic patients and alcoholics. Patients typically
present with an abrupt onset of fever, shaking chills, and purulent, foul-smelling sputum. Mucor
(choice C) is a fungal infection that is particularly severe in the diabetic or immunocompromised
patient. In the acidotic diabetic, the fungus produces a life-threatening, invasive rhinocerebral
infection. The infection begins in the nasal passages, extends into the paranasal sinuses, and

11

spreads through the cribiform plate to the frontal lobes of the brain. Patients typically complain of
headache, facial pain, and orbital swelling. Streptococcus pyogenes(choice E) causes bacterial
pharyngitis, otitis media, and sinusitis. It is also associated with toxin-related diseases and skin
infections.
29. A poor African community is experiencing an epidemic of severe hepatitis. The mortality
among pregnant women is particularly high. Which of the following viruses is the most likely
cause of the epidemic?
A. Cytomegalovirus (CMV)
B. Hepatitis A virus (HAV)
C. Hepatitis C virus (HCV)
D. Herpes simplex I
E. Hepatitis E virus (HEV)
The correct answer is E. Although hepatitis E is not generally seen in this country, it is essential that
the student have an understanding of all forms of hepatitis. Hepatitis E is an important, and until
recently, unrecognized cause of epidemics of enterically transmitted (fecal-oral) acute hepatitis.
Hepatitis E is caused by a virus that occurs primarily in India, Asia, Africa, and Central America.
Infection is associated with a 10-20% mortality among pregnant women. This type of hepatitis needs
to be considered in patients traveling to and from endemic areas. CMV (choice A) can cause acute
hepatitis, but the disease is usually mild and often goes unrecognized, except in profoundly
immunosuppressed patients. Hepatitis A virus (choice B) is the major cause of epidemics of
enterically transmitted viral hepatitis, especially in the United States, but is not a significant cause of
mortality in pregnant women. Hepatitis C virus (choice C) is usually transmitted parenterally rather
than enterically and is not a significant cause of mortality in pregnant women. Herpes simplex (choice
D) usually causes significant hepatitis only in profoundly immunosuppressed patients.
30. Several students ate lunch at a restaurant at which they all were served pork with
vegetables and fried rice. All of the students developed nausea, vomiting, abdominal pain, and
diarrhea within 4 hours of eating lunch. Which of the following is the most likely cause of
these symptoms?
A. Bacillus cereus
B. Clostridium botulinum
C. Clostridium perfringens
D. EHEC (enterohemorrhagic escherichia coli)
E. Staphylococcus aureus
The correct answer is A. Bacillus cereus produces a self-limited diarrhea caused by ingestion of the
preformed enterotoxin in contaminated fried rice (especially when reheated) and seafood. The
incubation period is typically approximately 4 hours. The degree of vomiting is usually greater than
that of diarrhea. B. cereus is also associated with keratitis, producing a corneal ring abscess.
Clostridium botulinum (choice B) produces a neurotoxin that blocks the release of acetylcholine,
resulting in a symmetric descending paralysis that may lead to respiratory complications causing
death. Symptoms include blurred vision, photophobia, dysphagia, nausea, vomiting, and dysphonia.
Most cases are associated with the ingestion of contaminated home-canned food.
Clostridium perfringens (choice C) produces a severe diarrhea with abdominal pain and cramping
(sometimes called "church picnic" diarrhea). The incubation period is 8-24 hours after ingesting
contaminated meat, meat products, or poultry. The meats have usually been cooked, allowed to cool,
and then warmed, which causes germination of the clostridial spores.
EHEC, enterohemorrhagic escherichia coli (choice D), produces a bloody, noninvasive diarrhea
caused by the ingestion of verotoxin found in undercooked hamburger at fast food restaurants. Some
patients develop a life-threatening complication called hemolytic-uremic syndrome. Staphylococcus
aureus (choice E) produces a self-limited food poisoning syndrome with nausea, vomiting, and
abdominal pain followed by diarrhea beginning 1-6 hours after ingestion of the enterotoxin. The
organism is found in foods such as potato salad, custard, milk shakes, and mayonnaise.

12

31. Which of the following organisms is most likely to be implicated as a cause of urethritis
that persists after antibiotic therapy for gonorrhea?
A. Actinomyces
B. Chlamydia
C. Mycobacteria
D. Nocardia
E. Rickettsia
The correct answer is B.Chlamydia, unlike the other choices, is a type of sexually transmitted
disease, as is gonorrhea. Gonococcal infections typically produce profuse urethal discharge,
especially in men, yielding a positive smear. Fever, rash and arthritis can occur with disseminated
disease. Chlamydia, Mycoplasma, and Ureaplasma are not effectively treated by penicillins and
cephalosporins, and are important causes of post-gonococcal urethritis. Chlamydial urethritis can be
diagnosed by using fluorescent antibodies to identify inclusions in epithelial cells. Actinomyces(choice
A) is a mouth commensal that rarely causes a deeper oral infection. Mycobacteria(choice C) cause
chronic granulomatous diseases such as tuberculosis and leprosy. Nocardia(choice D) can cause
necrotizing pneumonia and disseminated disease. Rickettsia(choice E) cause typhus and Rocky
Mountain spotted fever.
32. Which of the following organisms would be most likely to cause an outbreak of enteritis in
a day care center in the United States?
A. Helicobacter jejuni
B. Salmonella typhi
C. Shigella species
D. Vibrio cholerae
E. Yersinia enterocolitica
The correct answer is E. Enteritis is an inflammation of the intestine, especially the small intestine.
Yersinia enterocolitica is an important cause of mini-epidemics of pediatric diarrhea. Adults can also
be affected, but less commonly than children. Some diarrheal cases are severe (and occasionally
fatal) and may be complicated by severe dysentery, appendicitis, or chronic relapsing ileocolitis that
may require antibiotics to shorten the course. Yersinia is a non-motile ovoid or rod shaped,
nonencapsulated gram-negative bacteria. Helicobacter jejuni (choice A) is an important cause of
hospital-acquired diarrhea, especially in immunocompromised individuals. Salmonella typhi (choice
B) causes typhoid fever. It is a gram-negative bacteria. Shigella species (choice C) causes epidemics
of dysentery in military camps and other close quarter areas. It is a gram-negative bacteria. Vibrio
cholerae (choice D) causes cholera. It is a gram-negative bacteria. Gastroenteritis can occur in the
United States upon eating improperly cooked seafood from other countries.
33. In a closed system, spores are formed during which of the following phases of bacterial
growth?
A. Decline phase
B. Exponential phase
C. Lag phase
D. Log phase
E. Stationary phase
The correct answer is E. Spore formation usually occurs during the stationary phase, when cell
growth ceases because of a developing lack of nutrients or accumulation of toxins. During the phase
of decline (choice A), the lack of nutrients and the accumulation of toxin become so severe that any
viable organisms usually die before they can form spores.
The exponential phase (choices B) and log phase (choice D) are descriptors for active growth
occurring after the lag phase and before the stationary phase. Many antibiotics are most effective in
this period.
The lag phase (choice C) is the initial period of adaptation, prior to growth, which occurs when
organisms are introduced to a new environment.

13

34. On physical examination, a man has several disfiguring lesions on his face and loss of
cutaneous sensation to fine touch, pain, and temperature. An acid-fast organism is observed
in scrapings from a skin lesion. Which of the following organisms is the most likely cause of
this patient's disease?
A. Bartonella henselae
B. Listeria monocytogenes
C. Mycobacterium avium-intracellulare
D. Mycobacterium leprae
E. Nocardia asteroides
The correct answer is D. The disease in question is leprosy, or Hansen disease. The key feature in
the description is the fact the organism is acid-fast. These acid-fast bacilli are commonly found in skin
lesions or nasal scrapings. Both of the mycobacteria, M. avium-intracellulare and M. leprae are
strongly acid-fast, that is, they retain the carbol fuchsin dye in the face of acid-alcohol decolorization.
M. leprae has a predilection for the skin and cutaneous nerves, thereby producing the symptoms of
depigmentation and anesthetic cutaneous lesions. This loss of peripheral nerve function leads to
many of the disfiguring features of the disease; because patients do not have normal pain sensation,
they sustain repeated injuries. In addition, the organism attacks cartilage and causes granuloma
formation in the skin, leading to some of the facial disfigurement. Patients often have a history of
residence in an endemic area in childhood.
Bartonella henselae(choice A) is a very small, gram-negative bacterium that is closely related to the
rickettsia, although it is able to be cultured on lifeless media. It is the cause of cat-scratch disease, a
local, chronic lymphadenitis most commonly seen in children. Listeria monocytogenes(choice B) is a
ubiquitous microbe that causes disease in more than 100 animal species. Although it is best known as
an agent of meningitis in the newborn, it is a cause of multiple other diseases. A characteristic feature
of these infections is the development of granulomas at the site of the infection. The organism is not
acid-fast. M. avium-intracellulare(choice C) causes tuberculosis-like pulmonary disease in the
immunosuppressed, primarily in HIV infected individuals. Nocardia asteroides(choice E) primarily
produces pulmonary infections in humans. The organism is considered to be "weakly" acid-fast.
35. A 28-year-old HIV-positive man complains of pain on swallowing. Physical examination
shows white plaque-like material on his tongue and buccal mucosa, which is biopsied. The
man is diagnosed with acquired immunodeficiency syndrome (AIDS). With which of the
following agents is the man most likely infected?
A. Candida albicans
B. Cytomegalovirus
C. Herpes simplex I
D. Human herpesvirus 8
E. Human papilloma virus
The correct answer is A. The most common early form of oral lesions in HIV infected individuals
includes thrush, hairy leukoplakia, and aphthous ulcers. Candida albicans produces oral thrush, an
AIDS-defining lesion, which is common in acute HIV disease, and becomes increasingly common as
the CD4 + cell count falls, especially as the CD4 decreases to less than 300. As the CD4 count
decreases to less than 100, the esophagus, trachea, bronchi, or lungs may be infected. The lesions
are usually painless. Diagnosis is by demonstration of pseudohyphae using a wet smear with
confirmation by culture. Although cytomegalovirus (choice B) is associated with numerous clinical
scenarios in the AIDS population, including odynophagia (painful swallowing), it would not produce
white plaques on the oral mucosa. Herpes simplex I (choice C) produces vesicular lesions occurring
in clusters in the oral cavity. There is an increased risk for herpes infections in the AIDS group, but the
lesions do not resemble those described in the question. These lesions are commonly red and
extremely painful. Human herpesvirus 8 (choice D) is the causative agent of Kaposi sarcoma, a
malignancy arising from endothelial cells that appears as hemorrhagic nodules in different organ
systems. It is the most common cancer in the HIV infected population. Human papilloma virus (choice
E) is associated with a variety of lesions, including warts and intraepithelial neoplasias of the vulva
and cervix. It is associated with anal condyloma, which can occur in the AIDS population.

14

36. A viral organism was isolated from a painful blister on the lip of a teenage girl. The agent
was found to double-stranded, linear enveloped DNA. The patient had a similar sore
approximately 2 months ago following a week long trip to the beach. Which of the following is
the most likely causative organism?
A. Adenovirus
B. Coxsackie virus
C. Herpes simplex type 1 virus
D. Herpes zoster virus
E. Papilloma virus
The correct answer is C. Herpes simplex often causes recurrent grouped vesicles on an
erythematous base. The Tzanck smear is positive for multi-nucleated epithelial giant cells. Herpes
simplex is an enveloped, linear DNA virus that is a very common infectious agent; most adults will
have anti-Herpes simplex antibodies in their serum, although many may never have had any clinical
signs of disease. The hallmark of this disease is these painful skin vesicles, often called "cold" sores,
or "fever" blisters to denote the precipitating event that preceded the appearance of the lesions. The
virus has a propensity to become latent in the host's nervous tissue. Activation of the infection occurs
following mild trauma (e.g., a visit to the dentist), hormonal changes (e.g., menses),
immunosuppression (e.g., following organ transplantation), or may follow minor infections, stress or
sun exposure. Other, more serious, manifestations of disease include encephalitis, pneumonia, and
hepatitis; these are particularly likely to be seen in immunodeficient patients such as those with AIDS.
Adenoviruses (choice A) are naked, linear, double-stranded DNA viruses that cause acute, usually
self-limiting, influenza-like illnesses occurring in the fall and winter. The symptoms include pharyngitis,
fever, cough, and general malaise. Epidemic pharyngoconjunctivitis and pneumonia can occur in
closed populations such as military installations.
Coxsackie viruses (choice B) are naked, single-stranded, polycistronic viruses with an RNA genome.
They are divided into groups A and B based on their virulence. Coxsackie A causes herpangina and
hand-foot-and-mouth disease, while Coxsackie B is seen in patients with pleurodynia, myocarditis,
and pericarditis. Both groups cause upper respiratory infections, febrile rashes, and meningitis.
Herpes zoster (choice D), the varicella virus, is an enveloped, double-stranded DNA virus that is a
very common infectious agent in children. Chickenpox is a mild, self-limiting illness in children that is
evidenced as a fever followed by a macular rash that progresses to papules, then vesicles of the skin
and mucous membranes. Shingles is a recurrence of a latent varicella infection in which the virus has
taken refuge in sensory ganglia of spinal or cranial nerves. Various factors that decrease the immune
status of the patient contribute to the exacerbation of the infection. Severe dermatomal pain occurs
with a vesicular eruption, fever, and malaise.
Papilloma viruses (choice E) are members of the Papovavirus family. They are non-enveloped and
possess a double-stranded, circular DNA genome. They cause skin, plantar, and genital warts.
37. Electron micrographs of glomeruli reveal prominent deposits between the podocytes and
the basement membrane of the glomerular capillaries. If the patient was noted to have a skin
infection before this condition, these findings are most likely related to prior infection with
which of the following genera?
A. Escherichia
B. Klebsiella
C. Neisseria
D. Pseudomonas
E. Streptococcus
The correct answer is E. The disease is postinfectious glomerulonephritis, which is usually related to
prior sore throat or skin infection by Streptococcal species; Staphylococcus also causes some cases.
The prominent deposits are also known as subepithelial humps, and consist of immunoglobulin and
complement. Postinfectious glomerulonephritis commonly occurs after impetigo. Onset occurs within
1-3 weeks after infection (average 7-10 days). Other causes of postinfectious glomerulonephritis
include bacteremic states, such as systemic staphylococcus infections.

15

Gram-negative rods such as Escherichia(choice A), Klebsiella (choice B), and Pseudomonas
(choice D) have not been implicated as significant causes of postinfectious glomerulonephritis.
The gram-negative cocci Neisseria(choice C) have not been implicated in postinfectious
glomerulonephritis.
38. Which of the following interleukins is produced by macrophages and stimulates fever
production by its action on hypothalamic cells?
A. IL-1
B. IL-2
C. IL-3
D. IL-4
E. IL-5
The correct answer is A. IL-1 is produced by macrophages and other antigen-presenting cells. It has
several actions, including stimulation of T cells to secrete IL-2, chemotactic activity for neutrophils and
monocytes, increased expression of intercellular adhesion molecules (ICAMs) on vascular endothelial
cells, and activation of macrophages and natural killer (NK) cells. It is also pyrogenic (fever inducing)
because of a direct effect on the hypothalamus.
IL-2 (choice B) is produced by activated T helper cells. It stimulates the proliferation of other T cells
as well as activated B cells. It also activates NK cells and stimulates lymphokine secretion.
IL-3 (choice C) stimulates all stem cells to produce hematopoietic cells; it is also known as
multilineage colony stimulating factor (CSF).
IL-4 (choice D) is produced by T helper 2 (Th 2) cells and mast cells. It has several functions,
including inducing cells to express class II major histocompatibility complex (MHC) antigens and
stimulation of B-cell proliferation. It is involved in the induction of atopic allergies by its mitogenic
activity for mast cells and its enhancement of immunoglobulin class switching to IgG and IeE.
IL-5 (choice E) is secreted by activated T helper cells. It promotes B-cell proliferation, production of
eosinophils, and stimulates B-cell class switching to IgA.
39. A patient received second- and third-degree burns over his body. He later developed a
wound infection with a bluish-green exudate. Treatment with chloramphenicol and tetracycline
was unsuccessful. A gram-negative, motile organism was isolated. Which of the following
organisms was most likely isolated?
A. Candida albicans
B. Clostridium perfringens
C. Escherichia coli
D. Pseudomonas aeruginosa
The correct answer is D. Although this is a clinically-based question, it is essentially asking about
the microbiology of each bacteria. Pseudomonas aeruginosa is a very common opportunist in burn
patients and patients on ventilators, in whom it classically causes secondary wound infections and
septicemia. It may also cause cystitis in patients with urinary catheters and pneumonia in patients with
cystic fibrosis. The organism is found in water and usually gains access to the body as a contaminant
in the water used in respirators or in water baths, etc., used to cleanse wounds. This organism is a
nonfermenter, that is, it does not metabolize sugars by classic pathways. It produces a blue-green,
water-soluble pigment (pyocyanin), and has a fruity odor when grown on laboratory media. It has a
propensity for developing antibiotic resistance; current therapy uses the synergistic combination of an
aminoglycoside, such as amikacin, with a cell wall synthesis inhibitor (carbenicillin, ticarcillin, or
piperacillin).
Candida albicans (choice A) is a normal flora yeast that appears as large, weakly gram-positive,
spherical to ovoid organisms with budding daughter cells in Gram-stained preparations. Candidiasis is
an opportunistic infection in individuals with a compromised immune system. The fungus usually
causes mucocutaneous lesions, but in severely compromised individuals like patients with AIDS,
systemic disease may occur. Oral candidiasis appears as creamy white patches of exudate that can
be scraped off an inflamed tongue or buccal mucosa to reveal a painful lesion that may bleed.

16

Clostridium perfringens(choice B) is a gram-positive, spore-forming, anaerobic rod. It is a common


cause of gas gangrene when introduced into a wound. The organism produces a variety of toxins and
enzymes that enable it to destroy muscle tissue and spread through the soft tissues of the body.
Escherichia coli(choice C) is a lactose-fermenting, gram-negative rod seen as normal flora of the
intestine. It is the most common cause of urinary bladder infections, pyelonephritis, and sepsis in
patients with indwelling urinary catheters. It is also the major cause of traveler's diarrhea and is a very
important pathogen in neonates, who become infected during passage through the birth canal.
40. An 8-year-old girl is bitten in the leg by a neighbor's cat. She presents the next day with
fever and bone pain localized to her right calf. An x-ray film reveals a lytic lesion of the right
tibia. Which of the following is the most likely pathogen?
A. Brucella melitensis
B. Eikenella corrodens
C. Francisella tularensis
D. Pasteurella multocida
E. Yersinia pestis
The correct answer is D. This patient has osteomyelitis due to a cat bite that penetrated the
periosteum. Whenever you see dog or cat bites in a question stem, consider Pasteurella multocida as
a primary cause of wound infection. This organism is a short, encapsulated, gram-negative rod
demonstrating bipolar staining. Rapidly arising cellulitis is particularly indicative of this organism.
Brucella melitensis (choice A) is a small, acapsular, gram-negative rod that causes brucellosis
(undulant fever) and is associated with contact with goats or sheep. Brucella abortus and Brucella
suis are variants associated with contact with cows and pigs, respectively. The organisms enter the
body through the skin or through contaminated dairy products, such as unpasteurized imported goat's
milk or cheeses.
Eikenella corrodens(choice B) is commonly found in human bites. It is a gram-negative rod that is
part of the normal flora of the human mouth. Francisella tularensis(choice C) is a small, pleomorphic,
gram-negative rod that causes tularemia (rabbit fever). It occurs most commonly in rural areas. In the
U.S., rabbits are the main reservior for this organism, which is transmitted to humans by the
Dermacentor tick or by contact with infectious animal tissues. Yersinia pestis(choice E) is responsible
for bubonic plague, which has been known to occur in the western U.S. Its main reservoir is the prairie
dog, and its vector is the rat flea.
41. A 27-year-old IV drug user presents with difficulty swallowing. Examination of the
oropharynx reveals white plaques along the tongue and the oral mucosa. Which of the
following best describes the microscopic appearance of the microorganism responsible for
this patient's illness?
A. Budding yeast and pseudohyphae
B. Encapsulated yeast
C. Mold with nonseptate hyphae
D. Mold with septate hyphae
The correct answer is choice A. The patient has Candida esophagitis. Any time a patient presents
with dysphagia or odontophagia, along with whilte plaques in the oropharynx (thrush), you can
assume that the Candida is affecting the esophagus as well. The fact that the patient is an IV drug
user make an opportunistic infection such as Candida more likely. Candida appears as budding yeast
with pseudohyphae in vivo. The other answer choices represent the morphology of other important
opportunistic fungi: Cryptococcus are encapsulated yeast (choice B). You should think about
Cryptococcus neoformans when you're presented with an immunocompromised patient with
neurologic symptoms. The classic clue is the presence of encapsulated organisms observable in an
India ink preparation. Mucor and Rhizopus are molds with nonseptate hyphae (choice C). You should
think about Mucor when you are presented with a diabetic (especially ketoacidotic) or a leukemic
patient with a severe sinus infection. Aspergillus is a mold with septate hyphae (choice D). In
immunocompromised patients, aspergillosis can present with acute pneumonia, often with a cavitation
(aspergillomas = fungus balls in the lungs).

17

42. A 32-year-old woman with increased frequency of urination, suprapubic pain, and dysuria
for the past 3 days comes to the emergency department. She has no fever, nausea, or
vomiting. A Gram stain reveals gram-negative rods. Which of the following is the most likely
pathogen?
A. Escherichia coli
B. Neisseria gonorrhoeae
C. Shigella dysenteriae
D. Streptococcus pneumoniae
E. Treponema pallidum
The correct answer is A. This patient has the symptoms of a urinary tract infection (UTI).
Escherichia coli is the leading cause of community-acquired UTIs. The proximity of the urinary tract to
the anus facilitates colonization of the tract by fecal flora. Other gram-negative rods causing UTIs
include Enterobacter cloacae, Klebsiella pneumoniae, Serratia marcescens, Proteus mirabilis, and
Pseudomonas aeruginosa. None of the other choices listed cause UTIs.
43. A 24-year-old AIDS patient develops chronic abdominal pain, low-grade fever, diarrhea, and
malabsorption. Oocysts are demonstrated in the stool. Which of the following organisms is
most likely to be the cause of the patient's diarrhea?
A. Chlamydia psittaci
B. Entamoeba histolytica
C. Giardia lamblia
D. Isospora belli
E. Microsporidia
The correct answer is D. All of the organisms listed are protozoa. There are two intestinal protozoa
specifically associated with AIDS that can cause transient diarrhea in immunocompetent individuals
but can cause debilitating and potentially life-threatening chronic diarrhea in AIDS patients. These
organisms are Isospora belli, treated with trimethoprim-sulfamethoxazole (or other folate antagonists)
and Cryptosporidium parvum (no treatment presently available).
Chlamydia psittaci (choice A) is associated with the development of fever, chills, cough, and atypical
pneumonia with slightly delayed appearance of signs of pneumonitis. Entamoeba histolytica (choice
B) and Giardia lamblia(choice C) are both causes of diarrhea, but they are not specifically associated
with AIDS. Giardiasis, for example, can cause mild to severe bulky, greasy, frothy, malodorous stools,
free of blood and pus. Microsporidia (choice E) are a protozoan cause of diarrhea, but produce
spores rather than oocysts.
44. A middle-aged man presents with fever, anorexia, and jaundice. He is diagnosed with
hepatitis caused by an enveloped DNA virus. What is the most likely causative organism?
A. Hepatitis A
B. Hepatitis B
C. Hepatitis C
D. Hepatitis E
The correct answer is B. Hepatitis B is a hepadnavirus, which is enveloped DNA. Hepatitis A
(choice A) is a picornavirus and is a naked-capsid RNA. Hepatitis C (choice C) is a flavivirus, which
is RNA enveloped. Hepatitis E (choice D) is a Calicivirus, which is a naked-capsid RNA.
45. A 6 month-old child who has been breast-fed since birth develops voluminous, non-bloody,
watery diarrhea and vomiting. Which of the following viruses is the most likely cause of the
child's diarrhea?
A. Coronavirus
B. Lymphocytic choriomeningitis virus
C. Norwalk agent
D. Orbivirus
E. Rotavirus

18

The correct answer is E. Rotavirus is the major cause of diarrhea in infants and children under the
age of 2. It is a major cause of diarrheal morbidity worldwide due to dehydration. The virus replicates
in the intestinal mucosa, producing a profuse, watery, non-bloody diarrhea, often coupled with nausea
and vomiting. Transmission is by the fecal-oral route. Treatment is symptomatic with fluid and
electrolyte replacement. Coronaviruses (choice A) usually cause cold-like illnesses. Lymphocytic
choriomeningitis virus (choice B) can cause headache, malaise, myalgia, conjunctivitis, and,
occasionally, meningitis. Norwalk agent (choice C) can also cause diarrhea, but usually affects
patients older than 2 years. This causes profuse vomiting and diarrhea and is often transmitted by
food, especially shellfish. Orbivirus (choice D) is the cause of Colorado tick fever, which is the only
tick-borne viral disease in the United States.
46. With respect to the bacterial spectrum of coverage, which of the following antibiotics has
the broadest activity against gram-negative organisms?
A. Cefuroxime (second generation cephalosporin)
B. Cephalexin (first generation cephalosporin)
C. Ceftriaxone (third generation cephalosporin)
D. Metronidazole
E. Vancomycin
The correct answer is C. For NBDE Part 1, it is important to know general properties of antibiotic
families. Ceftriaxone is a third generation cephalosporin commonly used in the treatment of a variety
of life-threatening infections caused by gram-negative organisms. The agent has moderate grampositive activity. A general rule to remember about the cephalosporins is that when progressing from
first generation to third generation, the gram-negative activity is increased and gram-positive activity is
decreased. In other words, first generation cephalosporins have the most activity against grampositive organisms and third generation cephalosporins have the most activity against gram-negative
organisms. Metronidazole (choice D) is an antibiotic with excellent anaerobic bacterial coverage.
Vancomycin (choice E) is an antibiotic with excellent gram-positive coverage; it is generally used in
the treatment of life-threatening infections.
47. If a newborn child develops an infection with Chlamydia trachomatis, how does infection
with this organism cause blindness?
A. Cataract formation
B. Hemorrhage into the anterior chamber
C. Hemorrhage into the posterior chamber
D. Retinal detachment
E. Scarring of the cornea
The correct answer is E.Chlamydiae are obligate intracellular parasites. Studies in the United States
demonstrate that 5-25% of pregnant women have C. trachomatis infections of the cervix. In these
women, approximately one half to two thirds of infants are exposed and if untreated can develop
inclusion conjunctivitis. The lesions begin with formation of lymphoid follicles in the conjunctiva. With
disease progression, there is tissue necrosis, granulation tissue deposition, and scar formation,
leading to lacrimal duct obstruction and distortion of the eyelids. With the loss of an adequate tear
system, the cornea becomes vulnerable to dehydration and opacification. Also, the vigorous
inflammatory response can directly involve the cornea, with resulting opacity.
48. A 25-year-old man with a history of IV drug abuse presents with a high fever and
generalized malaise. Physical examination reveals a systolic murmur, and echocardiography
shows bulky vegetations attached to the tricuspid valve leaflets. Which of the following
microorganisms will be most likely be isolated?
A. Candida albicans
B. Hemophilus influenzae
C. Staphylococcus aureus
D. Staphylococcus epidermidis
E. Viridans (a-hemolytic) streptococci
The correct answer is C. The patient has a fever and is extremely ill. The most important clue to the
diagnosis is the presence of bulky vegetations on the tricuspid valve, indicating that he has infective

19

endocarditis. On the basis of the rapid clinical course, this is likely a case of acute infective
endocarditis. The diagnosis of this condition must be confirmed by blood cultures, which are also
necessary to determine bacterial antibiotic sensitivity. S. aureus (commonly present on the skin) is the
most frequent etiologic agent of infective endocarditis in intravenous drug abusers. It commonly
affects the tricuspid valve. Because of its high virulence, S. aureus-related endocarditis follows an
acute course and may lead to death within a few days. The causative agents of infective endocarditis
differ depending on host factors. Fungal organisms, such as Candida albicans(choice A), may cause
infective endocarditis in severely immunosuppressed patients, such as those with AIDS. A minority of
cases of infective endocarditis are caused by a number of normal commensals in the oral cavity, i.e.,
the "HACEK" group: Hemophilus(choice B), Actinobacillus, Cardiobacterium, Eikenella, and Kingella.
S. epidermidis(choice D) and other coagulase-negative staphylococci tend to produce endocarditis in
recipients of prosthetic valves. Viridans streptococci (choice E are the most frequent agents causing
endocarditis in previously abnormal valves, such as those damaged by rheumatic disease, or
congenitally abnormal valves. Coagulase-negative staphylococci and viridans (a-hemolytic)
streptococci are less virulent than S. aureus and are thus associated with a subacute (more
prolonged) clinical course and a better prognosis.
49. Which of the following is a feature of gram-positive bacteria rather than gram-negative
bacteria?
A. Lipid A-containing lipopolysaccharide
B. Lipoprotein in periplasmic space
C. Outer membrane
D. Peptidoglycan in periplasmic space
E. Thick peptidoglycan cell wall
The correct answer is E. Most of the features listed are those of gram-negative bacteria, which have
a complex cell envelope consisting of a cytoplasmic (inner) membrane, a periplasmic space
containing peptidoglycan (choice D) and lipoprotein (choice B), an outer membrane (choice C), and
sometimes a capsule. The outer membrane contains lipopolysaccharide (choice A) which is a major
component of endotoxin. The peptidoglycan cell wall of the gram-negative bacteria is thin, while that
of the gram-positive bacteria is thick. Other features of gram- positive bacteria include a fairly simple
surface of cytoplasmic membrane, peptidoglycan, cell wall, and sometimes, an outer capsule. The cell
wall contains lipoteichoic acids. Gram-positive bacteria retain the stain or resist decolorizationby
alcohol in Gram's method of staining. This is primarily characteristic of bacteria whose cell wall is
composed of peptidoglycan or teichoic acid.
50. A diabetic patient has chronic sinusitis, which is eventually treated by evacuation of the
contents of the maxillary and ethmoid sinuses. Mucor species are found when the material is
examined histologically. The pathologist should notify the clinician immediately because
Mucor can cause a virulent
A. meningitis
B. pneumonia
C. septicemia
D. skin infection
E. urinary tract infection
The correct answer is choice A. Mucormycosis, to which diabetics are particularly vulnerable
(perhaps because of the high glucose content of the diabetic nasal secretions), typically involves the
nasal sinuses. It is especially dangerous because the organism can erode into the bones of the
cranium, causing life-threatening meningitis and/or encephalitis that is very difficult to treat. Mucor can
cause pneumonia (choice B), but this is not the most immediate threat. Mucor can cause septicemia
(choice C), but this usually occurs in more seriously ill patients and is not an immediate risk in this
patient. Mucor can cause a skin infection (choice D), but this generally occurs in conjuncition with
severe burns. Mucor does not usually cause urinary tract infections (choice E).
51. In trying to sterilize some dental materials, the dentist notices that one of the materials
melts at 115 C. Which of the following sterilization methods is most appropriate for this
material?
A. Steam Autoclave
B. Dry heat autoclave
C. Glutaraldehyde (Cidex) soak (6 hours)
D. Ethylene oxide
E. Ethyl alcohol

20

The correct answer is choice D. Note firstly that ethyl alcohol and glutaraldehyde are chemicals that
can only high level disinfect, and cannot be used for sterilization. In particular, they are not sporicidal.
Choices A, B and D are the three most common methods of instrument sterilization. However, the
steam autoclave operates at 121 C, and the dry heat sterilizer at 160 C, both too hot for the material
which melts at 115 C. So we can only use a non-heat sterilization method. The only non-heat method
is ethylene oxide, at 8-12 hours of contact time.
52. Global eradication of Lyme disease is unlikely because
A. Borrelia burgdorferi can be maintained in nature indefinitely by a tick vector
B. Borrelia burgdorferi is resistant to antibiotics and disinfectants
C. Borrelia burgdorferi is resistant to environmental stresses
D. human disease may reactivate after the primary infection (Brill-Zinsser disease)
E. humans are the primary reservoir for Borrelia burgdorferi
The correct answer is A. Borrelia burgdorferi can be maintained in nature indefinitely by a tick
vector. This organism is the tick-transmitted spirochete that causes Lyme disease. The tick, Ixodes
dammini, can infect the white-footed mouse and large mammals such as deer during its life cycle,
making them reservoirs. The tick itself is a reservoir, however, because it acquires the disease
through transovarial passage of the organism. Together, these factors make Lyme disease an
endemic infection with little hope for eradication. Lyme disease is characterized by erythema migrus,
a flat or slightly raised red lesion that expands with central clearing. Headache or stiff neck are
common. Arthritis is common, chronic, and recurrent. Borrelia burgdorferi is not resistant to antibiotics
and disinfectants (compare with choice B). The spirochete can be successfully treated with
penicillins, tetracycline, and ceftriaxone. Borrelia burgdorferi is a delicate spirochete and is not
resistant to environmental stresses (compare with choice C). Brill-Zinsser disease (choice D) is the
reactivation of epidemic typhus infection caused by Rickettsia prowazekii. It can occur many years
after an infection that was not treated with antibiotics. Humans are incidental hosts, rather than the
primary reservoir, for Borrelia burgdorferi(choice E). The primary reservoirs are ticks, mice, and large
mammals.
53. A chef cut his finger. After a week, the site of the injury is warm, red, and swollen, and
begins draining pus. He contaminates some pastries with drainage from the lesion which later
were eaten by patrons of the restaurant. Within 4 hours they developed diarrhea and vomiting
with no fever. Which of the following organisms would be most likely to cause these
symptoms?
A. Bacillus cereus
B. Clostridium perfringens
C. Escherichia coli
D. Shigella sonnei
E. Staphylococcus aureus
The correct answer is E. Cellulitis is an acute infection involving the epidermis and dermis. Cellulitis
can lead to the development of an abscess, as seen in this patient. The most common types of skin
infections (cellulitis or abscess) usually involve gram-positive organisms such as staphylococcus and
streptococci. The chef had a staphylococcal abscess on his finger. S. aureus, produces enterotoxin A,
which was likely transferred to the pastries. When ingested, the toxin causes severe nausea and
vomiting within a few hours (the average incubation time is 3-6 hours). There is little diarrhea
associated with this type of food poisoning outbreak.
Bacillus cereus(choice A) is a gram-positive spore-forming rod that is associated with food poisoning
outbreaks following the ingestion of fried rice. The time of onset and symptoms would mimic
staphylococcal disease; the major differentiating feature is the food involved. The organism survives
the boiling of the rice because it is a spore-former. It germinates as the rice cools, grows, and
elaborates an enterotoxin that is responsible for the nausea and vomiting characteristic of the
disease. Clostridium perfringens(choice B) is a gram-positive spore-forming anaerobe that can cause
a longer incubation (18-24 hour) food poisoning, typically with marked diarrhea. Once again, the
spores allow the organism to survive the heating process used in the preparation of the food. Both
Clostridium perfringens and Clostridium botulinum are associated with home-canned vegetable and
sausages.

21

The symptoms of Escherichia coli(choice C) food poisoning are usually watery diarrhea (traveler's
diarrhea) with minimal nausea and vomiting, or a bloody diarrhea caused by enteroinvasive strains of
the agent. Also, E. coli would be an unlikely cause of the primary infection in the chef. Shigella sonnei
(choice D) causes enterocolitis characterized by fever, cramps, and diarrhea after an incubation
period of one to four days. Transmission is fecal-oral, associated with poor hygiene. A wide range of
foods has been implicated.
54. A Native American man is brought to a rural hospital in New Mexico with severe
bronchopneumonia, chills, fever, and headache. One day later, the man complains of chest
pain and difficulty breathing, and coughs up blood-tinged sputum. Chest x-ray reveals patchy
infiltrates and segmental consolidation. Which of the following organisms is the most likely
cause of this man's pneumonia?
A. Poliomyelitis virus
B. Clostridium perfringens
C. Paramyxovirus
D. Listeria monocytogenes
E. Yersinia pestis
The correct answer is E. Any previously healthy person in the southwestern United States who
develops septic shock or severe pulmonary disease should be evaluated for plague. Plague is not an
extinct disease, but is still encountered in sporadic cases in various places, including Asia, Africa,
parts of Europe, and the American Southwest. The causative organism is Yersinia pestis, which is
endemic in many wild animal populations, and can be transmitted to humans either by direct contact
or by arthropod bite. Human plague may take many forms, including pestis minor (mild
lymphadenopathy); bubonic plague (prominent lymphadenopathy); pneumonic plague (as described
above); and septicemic plague. Antibiotics are most effective if given within the first 24 hours, which
can be problematic if medical staff do not suspect the disease. Because plague is rare in the United
States, a high degree of clinical suspicion is required to make a rapid diagnosis and to institute timely
treatment. If the diagnosis is missed, the mortality rate is high.
Poliomyelitis virus (choice A) is an enterovirus that causes muscle weakness, headache, stiff neck,
sore throat, fever, nausea, and vomiting. Lower motor neuron lesions (flaccid paralysis) can be seen
as well as deep tendon reflexes and muscle wasting. Clostridium perfringens(choice B) causes gas
gangrene and gastroenteritis. This is the most common cause of gangrene. Paramyxovirus (choice
C) is the causative organism of the mumps. In this condition, exposure to paramyxovirus occurs 14-21
days before onset of symptoms. Patients present with painful swollen salivary glands, usually the
parotids. Frequent involvement of the other tissues is common, such as the testes, pancreas, and
meninges. Listeria monocytogenes(choice D) causes listeriosis. Infection during pregnancy may
result in sepsis, abortion, or premature delivery. Infection in the neonate may produce meningitis. In
immunocompromised adults, either meningitis or sepsis may occur.
55. A patient with large, penetrating vegetations on his mitral and aortic valves develops
severe headaches. Acute bacterial endocarditis is diagnosed. Which of the following
organisms is the most likely cause of the patient's disorder?
A. Herpesvirus
B. Mycobacterium tuberculosis
C. Staphylococcus aureus
D. Streptococcus pneumoniae
E. Treponema pallidum
The correct answer is C. The cardiac lesions described are characteristic of acute bacterial
endocarditis. Intravenous drug users are among those susceptible to acute endocarditis. The fact the
valvular vegetations are large and penetrating strongly suggests that they are caused by a virulent,
pyogenic pathogen. Staphylococcus aureus is the most common cause of acute bacterial
endocarditis. Brain abscess, which produces ring-enhancing radiologic lesions, is a known
complication of bacterial endocarditis, occurring when the vegetations fragment and release septic
emboli. The essential diagnostic characteristic for infective endocarditis includes a preexisting organic
heart lesion, fever, new or changing heart murmur, evidence of systemic emboli, positive blood
culture, and evidence of vegetation on endocardiography.

22

Herpesvirus (choice A) can cause encephalitis, but is not a cause of vegetative endocarditis.
Mycobacterium tuberculosis(choice B) can cause brain masses, but does not usually cause
endocarditis. Streptococcus pneumoniae(choice D) can cause acute pyogenic meningitis, but does
not usually cause endocarditis. Treponema pallidum(choice E), the causative agent of syphilis, can
cause aortic aneurysms and a variety of neurologic problems, but does not usually cause endocarditis
or brain abscess.
56. A can of disinfectant spray states that it kills HIV virus. This statement:
A. is important because of the difficulty in killing this virus
B. shows that the disinfectant will kill most other pathogens
C. illustrates the use of a "benchmark" organism
D. is not a good indication of disinfectant strength
The correct answer is D. HIV virus is notoriously easy to kill on most environmental surfaces. Many
disinfectants and other common physical and chemical exposures will kill the virus fairly easily.
Therefore, ability to kill HIV is NOT an important criterion in choosing disinfectants. Mycobacterium
tuberculosis is chosen as the "benchmark organism" (the standard against which the disinfectant is
compared). It is very resistant to surface disinfectants, partially because of its waxy cell wall. Another
highly resistant pathogen on environmental surfaces is Hepatitis A virus.
57. A 35-year-old, sexually active man presents with a painless penile vesicle and inguinal
lymphadenopathy. The infecting organism is definitively diagnosed and is known to exist in
distinct extracellular and intracellular forms. Which of the following is the most likely
pathogen?
A. Calymmatobacterium granulomatis
B. Chlamydia trachomatis
C. Haemophilus ducreyi
D. Neisseria gonorrhoeae
E. Treponema pallidum
The correct answer is choice B. This patient has lymphogranuloma venereum caused by Chlamydia
trachomatis (type L1, 2, or 3). Chlamydia exhibit distinct infectious and reproductive forms. The
extracellular infectious form is known as the elementarly body (EB), which cannot reproduce. It
attaches to the host cell and enters through endocytosis. Once inside the cell, the EB is transformed
into the reticulate body (RB) within the endosome. The RB is capable of binary fission and divides
within the endosome; fusion with other endosomes occurs to form a single large inclusion. Eventually,
the RBs undergo DNA condensation and disulfide bond bridgings of the major outer membrane
protein, forming EBs. The EBs are then released. Note that C. trachomatis is responsible for several
sexually or perinatally transmitted diseases, including ocular trachoma (types A, B and C), neonatal
conjunctivitis, nongonococcal urethritis, cervicitis, and pelvic inflammatory disease (types D-K).
Calymmatobacterium granulomatis (choice A) is a gram-negative rod that causes superficially
ulcerated genital or inguinal papules that coalesce to form substantial lesions. The appearance of
Donovan bodies in histiocytes is diagnostic of this infection. Haemophilus ducreyi (choice C) is a
gram-negative rod that causes a soft, painful penile chancre, unlike that of a chlamydial or syphilitic
lesion. This infection is common in the tropics. Neisseria gonorrhoeae (choice D) is a gram-negative
diplococcus responsible for gonorrhea. Patients typically present with purulent penile discharge, not
genital lesion. Treponema pallidum (choice E) is the spirochete responsible for syphilis. It may cause
a firm, painless ulcer as a manifestation of primary syphilis, but the organism does not exist in distinct
extracellular and intracellular forms as does Chlamydia. Secondary syphilis is associated with the
appearance of condyloma lata -- flat, gray wart-like lesions.
58. A 23-year-old man develops explosive watery diarrhea with blood, fecal leukocytes, and
mucus approximately 3 days after eating chicken that was improperly cooked. Comma-shaped
organisms were also found in the fecal smear. Which of the following pathogens is the most
likely cause of these symptoms?
A. Campylobacter jejuni
B. Enterotoxigenic E. coli
C. Shigella sonnei
D. Staphylococcus aureus
E. Vibrio cholera

23

The correct answer is A. This is another question in which knowing the microbiological
characteristics is essential. Campylobacter jejuni is a pathogen causing an invasive enteric infection
associated with ingestion of raw or undercooked food products, or through direct contact with infected
animals. In the United States, ingestion of contaminated poultry that has not been sufficiently cooked
is the most common means of acquiring the infection. The patients typically have bloody diarrhea,
abdominal pain, and fever. The presence of fecal leukocytes indicates an invasive infection.
Campylobacter are microaerophilic, motile, gram-negative "comma-shaped" rods.E. coliare not
comma-shaped.
Enterotoxigenic E. coli(choice B) causes the classic traveler's diarrhea. The infection is noninvasive
and is acquired by way of the fecal-oral route through consumption of unbottled water or uncooked
vegetables. The major manifestation is a copious outpouring of fluid from the gastrointestinal tract
presenting as explosive diarrhea. This is caused by the action of one of two types of enterotoxins on
the gastrointestinal tract mucosa. Shigella sonnei(choice C) produces a syndrome very similar to C.
jejuni. The organism, however, appears as a gram-negative rod on Gram's stain. It does not have a
comma shape. Transmission is from person to person by way of the fecal-oral route. Infection
requires a low infective dose because the organism is fairly resistant to gastric acidity.
Staphylococcus aureus(choice D) produces food poisoning through the ingestion of a preformed
enterotoxin. The organism is present in food that is high in salt content, such as potato salad, custard,
milk shakes, and mayonnaise. The patient presents with nausea, vomiting, and abdominal pain,
followed by diarrhea beginning 1-6 hours after ingestion of the enterotoxin.S. aureus are seen as
clusters of coli and are not comma-shaped. Vibrio cholerae(choice E) produces a secretory diarrhea
caused by increases in cAMP in the intestinal cells. The organism is comma-shaped, but is not
invasive. The patient presents with the sudden onset of painless, watery diarrhea that becomes
voluminous, followed by vomiting. The stool appears nonbilious, gray, and slightly cloudy with flecks
of mucus, no blood, and a sweet odor.
59. A 3-year-old boy presents with a 1-day history of loose stools, fever, abdominal cramping,
headache, and myalgia. He has no blood in the stool. A careful history reveals that he has had
several pet turtles. Which of the following is the most likely pathogen?
A. Chlamydia psittaci
B. Entamoeba histolytica
C. Salmonella spp.
D. Staphylococcus aureus
E. Yersinia enterocolitica
The correct answer is C. Salmonella spp., including S. enteriditis and S. typhimurium, produce a
gastroenteritis or enterocolitis. Patients with decreased gastric acidity, sickle cell disease, or defects in
immunity, and children younger than 4 years of age have a more severe course of disease.
Salmonella spp. are carried in nature by animal reservoirs such as poultry, turtles, cattle, pigs, and
sheep. The incubation period is 8-48 hours after ingestion of contaminated food or water. Chlamydia
psittaci (choice A) produces an interstitial pneumonitis accompanied by headache, backache, and a
dry, hacking cough. A pale, macular rash is also found on the trunk (Horder spots). Patients at risk
include pet shop workers, pigeon handlers, and poultry workers. Entamoeba histolytica (choice B)
produces a diarrhea (frequently bloody or heme-positive), right lower quadrant crampy abdominal
pain, and fever. Patients frequently have weight loss and anorexia. There is usually a history of travel
outside the United States. Most cases are chronic. Complications include liver abscesses.
Staphylococcus aureus (choice D) produces a self-limited gastroenteritis caused by the production of
preformed, heat-stable enterotoxins. The incubation period is 16 hours. The toxins enhance intestinal
peristalsis and induce vomiting by a direct effect on the CNS. Yersinia enterocolitica (choice E)
usually produces a chronic enteritis in children. These patients have diarrhea, failure to thrive,
hypoalbuminemia, and hypokalemia. Other findings include acute right lower quadrant abdominal
pain, tenderness, nausea, and vomiting. The infection mimics appendicitis or Crohn disease.
60. A 48-year-old presents with malaise, loss of appetite, nausea, moderate fever, and
jaundice. Laboratory tests indicate a marked increase in serum transaminases and presence
of HBsAg, HBc IgM antibody, and HCV antibody. Antibody tests for HBsAb and HAV are
negative. The results indicate:
A. A dual infection of HBV and HAV
B. Chronic hepatitis A infection
C. Chronic hepatitis B infection
D. Hepatitis C infection
E. The presence of an acute HBV infection.

24

The correct answer is E.The presence of hepatitis B surface antigen (HBsAg) along with hepatitis B
core IgM antibody (HBc IgM Ab), and the absence of hepatitis B surface antibody (HBsAb) indicates
the presence of the early stages of an acute infection with Hepatitis B. The presence of antibody to
Hepatitis C (HCV) only indicates exposure, but not a specific time of exposure; however, 85% of
patients who are infected with HCV develop chronic infections, indicating that this patient has an 85%
chance of having a dual infection with HBV and HCV. A dual infection of HBV and HAV (choice A) is
not plausible since the IgM anti-HAV serology is negative. Hepatitis A does not cause chronic disease
(choice B). Chronic HBV infection (choice C) is unlikely because the patient has HBc IgM Ab, which
is characteristic of an acute infection, rather than a chronic infection. Hepatitis C infection (choice D)
is not confirmed by these data because the presence of HCV Ab only indicates exposure to the virus,
and not the state of infection. This could be caused by exposure at some earlier time (the elevated
serum transaminases might be due to HBV infection). An active or chronic HCV infection can only be
confirmed by PCR.
61. A 47-year-old man presents with a nonproductive cough and rales. A chest x-ray film
suggests atypical pneumonia. The pneumonia resolves after treatment with azithromycin. A
diagnosis of psittacosis is established by the presence of complement fixing antibodies
against Chlamydia psittaci in the convalescent serum. Which of the following is the most likely
occupation of this patient?
A. Cat breeder
B. Florist
C. Homeless shelter worker
D. Poultry farmer
E. Slaughterhouse worker
The correct answer is D. When you see Chlamydia psittaci, one word should come to mind: birds!
Infection with this organism is an occupational hazard for anyone who works with birds (e.g.,
veterinarians, pet store employees), including poultry farmers. Individuals who work with cats (choice
A) would be at an increased risk for infection with Pasteurella multocida (which is acquired primarily
through cat bites), Bartonella henselae (cat-scratch fever), and the protozoa Toxoplasma gondii
(which can be acquired from ingestion of food contaminated with cat feces). Florists (choice B) are at
increased risk for infection with the fungus Sporthrix schenckii, a primary pathogenic fungus acquired
by inoculation (e.g., a rose thorn puncturing the skin). A person who works in a homeless shelter
(choice C) would be at increased risk for infection with Mycobacterium tuberculosis, because this
organism is spread through the air and is prevalent in the homeless community. Slaughterhouse
workers (choice E) are at an increased risk for infection with Brucella, a bacterium that is acquired by
handling infected animals.
62. A 3-year-old child with cystic fibrosis presents with weight loss, irritability, and a chronic
productive cough. On physical examination, he is febrile and wheezing with rhonchi and rales.
Chest x-ray demonstrates patchy infiltrates and atelectasis, and Gram stain of the sputum
reveals slightly curved, motile, gram-negative rods that grow aerobically. The microorganism
responsible for this child's pneumonia is also the most common cause of which of the
following diseases?
A. Croup
B. Epiglottitis
C. Meningitis
D. Otitis externa
E. Otitis media
The correct answer is D. This is an example of a two-step question. Step 1: what is the organism
that causes pneumonia in cystic fibrosis? Then, this organism is commonly seen in cases of...?
Anytime you see pneumonia in a cystic fibrosis patient you should suspect Pseudomonas aeruginosa.
The Gram stain revealing aerobic, gram-negative rods confirms your suspicion in this case. Now the
question is: which of the diseases listed is also caused by Pseudomonas? The answer is otitis
externa. P. aeruginosa is often found in the external ear, especially if the conditions are moist
("swimmer's ear") and there is any sort of inflammation. External otitis is usually a benign process with
the only symptoms being an itchy, painful ear. If, however, the organism penetrates the epithelium
and invades the soft tissue, cartilage, and cortical bone, the process becomes malignant otitis

25

externa, which can progress to osteomyelitis leading to cranial nerve palsies. This condition is most
common in diabetics. Croup (choice A), also called laryngotracheobronchitis, is a respiratory disease
of children that presents with a characteristic "barking" cough. Croup is caused by parainfluenza virus.
Epiglottitis (choice B) is a potentially fatal infection in children, caused by H. influenzae, which
presents with drooling, difficulty breathing, and stridor. The incidence of this disease has dropped
dramatically with the introduction of the H. influenzae type b (Hib) vaccine. Meningitis (choice C) is
caused by numerous different bacteria, depending on the age of the patient. The most common
causes include S. pneumoniae (elderly patients), H. influenzae (unvaccinated children), Group B strep
and E. coli (neonates) and N. meningitidis (1 month to adult). Although P. aeruginosa can cause
meningitis, it is not a common cause. The most common causes of otitis media (choice E) include S.
pneumoniae and H. influenzae. Even in cases of external ear infections with P. aeruginosa, the
middle ear is typically spared.
63. A combined chemical/heat autoclave (Chemiclave) uses what set of chemicals/conditions?
A. Alcohol at 121 C
B. Ethylene oxide at 160 C
C. Alcohol/formaldehyde at 132 C
D. Glutaraldehyde at 121 C
E. Glutaraldehyde/formaldehyde at 160 C
The correct answer is C. Besides the steam autoclave, dry heat autoclave, and ethylene oxide
sterilizer, there is a heat/chemical combination known as a Chemiclave or heat/chemical sterilizer. Its
conditions are formaldehyde and alcohol at a temperature of 132 C for periods of 20-30 minutes at
20-40 psi (pounds per square inch). The chemical mixture is provided by the manufacturer and is recollected for disposal.
64. A 10-year-old boy develops an itchy, vesicular rash, which is maximal on his face and
trunk. Physical examination demonstrates a mixture of lesions, with macules, papules,
vesicles, and crusted lesions. The mother reports that the lesions seem to be occurring in
crops. Which of the following is the most likely diagnosis?
A. Herpes simplex I
B. Herpes simplex II
C. Measles
D. Shingles
E. Varicella
The correct answer is E. This is varicella (chicken pox), which is the primary form of infection by the
herpes zoster (varicella-zoster) virus. Recurrence due to virus harbored in neurons tends to be
dermatomal in distribution and is called shingles. Fever, malaise, headache, and myalgia may also be
present, particularly in the prodromal phase. The rash is pruritic, papular, changing to vesicular ("dew
drops on a rose petal"), pustular and finally crusting. Tzanck smear of the base of a vesicle may
demonstrate multinucleated giant cells. Immunocompromised patients can be treated with acyclovir to
prevent dissemination. Chicken pox may be complicated by secondary bacterial infection, pneumonia
and systemic spread (immunosuppressed patients). Herpes simplex I (choice A) causes oral vesicles
and ulcers. The lesions are often very painful. A rash is not seen with herpes simplex I. Herpes
simplex II (choice B) causes genital vesicles and ulcers. The lesions are described as being beefy
red and very painful. Measles (choice C) causes a blotchy, nonvesicular rash. Prodrome consists of
fever, coryza, malaise, photophobia and Koplik's spots. The rash is brick red, irregular and
maculopapular. Shingles (choice D) is the recurrent form of herpes zoster infection and usually is
localized to a single dermatome. It is typically seen in the elderly.
65. Which of the following organisms is the most common cause of community-acquired
pneumonia?
A. Chlamydia pneumoniae
B. Haemophilus influenzae
C. Mycoplasma pneumoniae
D. Staphylococcus aureus
E. Streptococcus pneumoniae

26

The correct answer is E. The most common bacterium implicated in community-acquired pneumonia
is the pneumococcus Streptococcus pneumoniae. When community-acquired pneumonia occurs in
elderly patients or patients with comorbidity, aerobic gram-negative bacilli and Staphylococcus aureus
are added to the list. The organisms listed in choices A, B, and C are important causes of communityacquired pneumonia without comorbidity in patients younger than 60 years of age, but are not the
most frequent causes. Staphylococcus aureus (choice D) is an important cause of communityacquired pneumonia (particularly in the elderly and in patients with comorbidity) but is not the most
frequent cause.
66. A 21-year-old man presents with cough, fever, and hemoptysis. Blood tests show
significantly elevated BUN and creatinine. Immunofluorescent microscopy reveals a diffuse
linear pattern of fluorescence along the basement membranes of alveolar septa and
glomerular capillaries. Which type of hypersensitivity is associated with this disease?
A. I
B. II
C. III
D. IV
The correct answer is B. This patient has Goodpasture syndrome, which affects both the renal and
pulmonary systems. In the kidney, it causes a rapidly progressive glomerulonephritis associated with
antibodies directed against a collagen component of the glomerular basement membrane (anti-GBM
antibody a classic clue to this diagnosis). They are also active against the basement membrane of
respiratory alveoli, accounting for the pulmonary component of the disease. These antibodies create a
linear pattern on immunofluorescence.
Autoimmune reactions such as those found in Goodpasture syndrome, certain drug allergies, blood
transfusion reactions, and hemolytic disease of the newborn, are classified as Type II
hypersensitivities (antibody-mediated cytotoxicity). IgG or IgM antibody reacts with membraneassociated antigen on the surface of cells, causing activation of the complement cascade and,
ultimately, cell destruction.
Type I (choice A) reactions (immediate, atopic, or anaphylactic) require an initial (sensitizing)
exposure to an antigen. On re-exposure to the antigen, cross-linking of IgE receptors occurs on the
surface of basophils and mast cells. The mast cells then release a variety of mediators, including
histamine. Clinical syndromes include asthma, atopic dermatitis, eczema, hives, and allergic rhinitis.
Type III (choice C) hypersensitivity (immune complex-mediated hypersensitivity) is caused by
antibodies to foreign antigens. Immune complexes of IgG or IgM with the antigen activate
complement. This results in the generation of C3b, which promotes neutrophil adherence to blood
vessel walls. The complexes also generate C3a and C5a (anaphylatoxins), which lead to inflammation
and tissue destruction. The hallmark signs of Type III sickness, which occur 7-14 days after exposure
to the offending antigen, include urticaria, angioedema, fever, chills, malaise, and glomerulonephritis.
Clinical syndromes include serum sickness (e.g., penicillin, streptomycin, sulfonamide,
phenylbutazone hypersensitivity) and the Arthus response. Immune complexes are also observed in
systemic lupus erythematosus (SLE). Type III glomerulonephritis (e.g., poststreptococcal
glomerulonephritis) is characterized by a lumpy bumpy appearance on immunofluorescence using
labeled antibody specific for immunoglobulin or complement.
Type IV (choice D) is also known as delayed-type hypersensitivity (DTH). Unlike the other types,
which are mediated by antibody, DTH depends on TDTH cells that have been sensitized to a
particular antigen. T cells react with antigen in association with MHC class I gene products and
release lymphokines. Examples include tuberculin skin sensitivity and contact dermatitis (e.g., poison
ivy rash).
67. Which of the following is found only in gram-negative microorganisms?
A. Cell envelope
B. Exotoxin
C. Peptidoglycan
D. Periplasmic space
E. Teichoic acids

27

The correct answer is D. This question requires you to appreciate the key structural difference
between gram-positive and gram-negative microorganisms. Most gram-negatives (other than
exceptional microorganisms, such as Mycoplasma, that lack a cell wall) have a more complex cell
envelope than gram-positive microorganisms. It includes both a cytoplasmic membrane and an outer
membrane. Between these two membranes is the periplasmic space, which contains enzymes such
as phosphatase and penicillinase, binding proteins for the transport of various nutrients, and
peptidoglycan, as well as a portion of the lipoprotein that firmly anchors the outer membrane to the
peptidoglycan. Gram-positives do not have outer membranes.
Choice A, the cell envelope, is incorrect because both gram-positive and gram-negative
microorganisms have this structure, which is defined as all the layers that enclose the cytosol of the
bacterium. It is the composition of the envelope that differs between gram-positive and gram-negative
microorganisms. Choice B, exotoxin, is not exclusive to gram-negative microorganisms, but is also
found in some gram-positives. By contrast, endotoxin (lipopolysaccharide; LPS) is found exclusively in
gram-negatives. Choice C, peptidoglycan, is found in the cell walls of both gram-positive and gramnegative microorganisms. Note that there is a larger amount of peptidoglycan in gram-positive
microorganisms. Choice E, teichoic acids, are found exclusively in gram-positive organisms.
68. A 6-week-old infant has a 10-day history of coughing and choking spells. The child is
gasping for breath and experiencing paroxysms of coughing. Encapsulated, gram-negative
rods are cultured. The organism is most likely
A. Bordetella pertussis
B. Haemophilus influenzae type b
C. Klebsiella pneumoniae
D. Legionella pneumophila
E. Mycoplasma pneumoniae
The correct answer is A. The child has whooping cough, or more accurately if one goes by the
chronology of the symptoms, "coughing whoop," as the patient is subject to a series of coughing
episodes that are followed by a rapid inspiration of air through a narrow airway (the "whoop"). The
infection of the respiratory tract by Bordetella pertussis causes a hyperreactivity of the respiratory
apparatus; even the slightest stimulus will trigger a coughing episode. The offending pathogen
excretes adenylate cyclase and also produces an exotoxin that inactivates the inhibitory subunit of the
G-protein complex, thus activating adenylate cyclase within the cells of the respiratory system. The
symptoms of pertussis last about 6 weeks and are divided into 3 stages: 1) Catarhal stage: Insidious
onset with lacrimation, sneezing, anorexia, coryza and a hacking cough at night 2) Paroxysmal stage:
Characterized by rapid bursts of consecutive coughs followed by a deep, high-pitched inspiration
(whoop) 3) Convalesent stage: Begins 4 weeks after onset and is associated with a reduction in
frequency and severity of paroxysms of cough
Haemophilus influenzae type b (choice B) was the major cause of infant meningitis before the Hib
conjugate vaccine nearly eradicated this pathogen from the United States. Nonencapsulated strains
cause otitis media in children and pneumonia in adults. Klebsiella pneumoniae (choice C) is a gramnegative, encapsulated rod that is a significant pulmonary pathogen in individuals with a compromised
respiratory apparatus. It is a common cause of aspiration pneumonia and pulmonary abscesses in
alcoholics and patients with chronic obstructive pulmonary disease. Legionella pneumophila (choice
D) is another fastidious, gram-negative, respiratory pathogen that may cause either a fulminating
disease or a mild "walking pneumonia-like" condition (i.e., an atypical pneumonia). Mycoplasma
pneumoniae (choice E) is the most common cause of primary atypical pneumonia. The disease is
considered "atypical" because the patients have a very mild disease with low-grade fever, little in the
way of constitutional signs, and a non-productive cough. These organisms are fastidious and are
difficult to grow in the laboratory.
69. An AIDS patient develops symptoms of pneumonia, and Pneumocystis carinii is suspected
as the causative organism. Which of the following stains would be most helpful in
demonstrating the organism's cysts?
A. ELISA
B. Hematoxylin and eosin
C. Methenamine silver
D. Prussian blue
E. Western blot

28

The correct answer is C. The appropriate stain is methenamine silver, since routine hematoxylin and
eosin does not adequately demonstrate the organisms. The cysts, when stained with methenamine
silver, have a characteristic cup or boat shape; the trophozoites are difficult to demonstrate without
electron microscopy. It is also worth knowing that sputum samples are not nearly as effective as
bronchial washes in demonstrating the organisms. ELISA (choice A) is enzyme-linked
immunosorbent assay. It is the primary test for HIV antibody. It is not a stain. Hematoxylin and eosin
(choice B) is the routine tissue stain used in pathology laboratories. Prussian blue (choice D) is good
for demonstrating iron. The western blot (choice E) is the confirmatory test of choice for diagnosing
HIV infection. The western blot test is generally performed once the ELISA test has been shown to be
positive.
70. A sexually active man presents to a dermatologist because of a severe mucocutaneous
rash that involves most of his body, including his palms and soles. These mucous patches are
described as being painless, silvery ulcerations of mucous membrane with surrounding
erythema. If iritis and arthritis are also seen, which of the following is the most likely causative
agent of this rash?
A. Herpes simplex I
B. Herpes simplex II
C. HIV
D. Neisseria gonorrhoeae
E. Treponema pallidum
The correct answer is E. The rash described is that of secondary syphilis, caused by Treponema
pallidum. Involvement of palms and soles by a rash is unusual, and secondary syphilis should come
to mind. Not all patients with secondary syphilis have a severe form of the rash, and consequentially
some cases are missed. Primary syphilis takes the form of a painless, button-like mass called a
chancre. Tertiary syphilis, which is now rare, has a propensity for involving the aorta and central
nervous system and can also cause "gummas" (granulomatous-like lesions) in many sites, notably
including liver and bone. The mucous patches are described as being painless silvery ulcerations of
mucous membrane with surrounding erythema. Other symptoms include cranial neuropathies (II VIII), iritis, glomerulonephritis, arthritis and periositis. Herpes simplex I (choice A) usually causes
perioral vesicular lesions. Herpes simplex II (choice B) usually causes genital vesicular lesions. HIV
(choice C) does not itself cause a rash, although co-infection with other organisms can result in a
rash. Neisseria gonorrhoeae(choice D) does not typically cause a rash but does not cause urethral
discharge and dysuria.
71. An African child develops massive unilateral enlargement of his lower face in the vicinity of
the mandible. Biopsy demonstrates sheets of medium sized blast cells with admixed larger
macrophages. This type of tumor has been associated with which of the following?
A. Epstein-Barr virus
B. Hepatitis B
C. Herpesvirus
D. HIV
E. Human papillomavirus
The correct answer is A. The patient has Burkitt lymphoma. This high-grade B-cell lymphoma occurs
endemically in Africa (it is the most common neoplasm in children in an equatorial belt that includes
Africa and New Guinea) and sporadically in the U.S. and Europe. The sporadic form is often in an
abdominal site and occurs in young adults. The African form of Burkitt lymphoma has been strongly
associated with antibodies directed against Epstein-Barr virus; the association is weaker in sporadic
cases. Hepatitis B (choice B) is associated with hepatocellular carcinoma. Herpesvirus (choice C)
type 8 is associated with Kaposi sarcoma. HIV (choice D) is linked to AIDS. Some patients also
develop primary lymphomas (not usually Burkitt). Human papillomavirus (choice E) is linked with
common warts, genital condylomata, and genital cancers.
72. A 4-month-old infant presents with failure to thrive, progressive muscular weakness, poor
head control, dysphagia, and dry mouth. The infant often eats soy-based formula sweetened
with honey. Which of the following organisms is most likely to be responsible for the child's
presentation?
A. Clostridium botulinum
B. Clostridium difficile
C. Clostridium perfringens
D. Clostridium tetani
E. Corynebacterium diphtheriae

29

The correct answer is A. The child has infant botulism (floppy baby syndrome)that is caused by
germination of Clostridium botulinum spores (found in honey) in the baby's gastrointestinal tract.
Patients improve when honey is removed from the diet. This disorder is most common in children
under the age of 6 months; older children and adults do not seem to be vulnerable to this form of
botulism, but are susceptible to botulism caused by ingestion of preformed toxin. Patients usually
present with a sudden onset of dry mouth, diplopia, dysphagia, dysphonia, and muscle weakness
progressing to respiratory paralysis. Pupils are often fixed and dilated. Clostridium difficile(choice B)
causes pseudomembranous colitis, especially after antibiotic therapy. It causes severe diarrhea
through the production of toxin from these bacteria. Clostridium perfringens(choice C) causes gas
gangrene and gastroenteritis, and it is not associated with ingestion of honey. Clostridium
tetani(choice D) causes tetanus and does not cause a food-borne illness in infants. Corynebacterium
diphtheriae(choice E) causes diphtheria in susceptible individuals.
73. A 66-year-old man with an untreated urinary tract infection develops a spiking fever,
tachypnea, and severe hypoxemia. If adult respiratory distress syndrome is caused as a result
of the urinary tract infection, blood cultures would most likely reveal
A. Gram-negative diplococci
B. Gram-negative rods
C. Gram-positive cocci
D. Gram-positive diplococci
E. Gram-positive rods
The correct answer is B. Escherichia coli is a very common pathogen associated with urinary tract
infections and is a common cause of cystitis. In fact, more than 90% of all "first" infections are caused
by Escherichia coli. It is part of the normal flora of the gastrointestinal tract. Patients with cystitis can
develop bacteremia and subsequent septic shock and adult respiratory distress syndrome (ARDS).E.
coli is a gram-negative rod. The key to this question is knowing that most urinary tract infections are
caused by E. coli.
Gram-negative diplococci (choice A) might be Neisseria spp. or Moraxella catarrhalis. Neisseria
gonorrhea is the cause of gonorrhea, a sexually transmitted disease that presents with urethritis or
may be asymptomatic. Patients are usually young, sexually active males. Moraxella spp. are gramnegative cocci that can cause a wide variety of infections, usually implicated as a cause of otitis media
and sinusitis in children, or as a cause of purulent tracheobronchitis and pneumonia in a population of
patients who are over 50 years of age and have underlying obstructive lung disease. Gram-positive
cocci (choice C), such as Staphylococcus aureus, S. epidermidis and Streptococci, rarely cause
cystitis. Staphylococcus saprophyticus causes urinary tract infections, but the patients are typically
young, sexually active women. Gram-positive diplococci (choice D) would be a description of
Streptococcus pneumoniae, which is the most common cause of community-acquired pneumonia.
This is rarely a cause of UTIs. Gram-positive rods (choice E) would include members of the following
genera: Clostridium, Bacillus, Listeria, and the coryneform bacteria. The only significant member of
this group to produce urinary tract infections is Corynebacterium urealyticum.The organism creates an
alkaline urine environment with the potential for stone formation. Patients are usually
immunocompromised or have had recurrent urinary tract infections.
74. Which organism is LEAST likely to be involved in lesions of chronic adult periodontitis?
A. Spirochetes
B. Bacteroides melaninogenicus
C. Capnocytophaga
D. Streptococcus mitis
E. Fusobacteria
The correct answer is choice D. Periodontal disease pathogens are usually anaerobic or
capnophilic. The term capnophilic means "carbon-dioxide loving". In the anaerobic environment deep
in the sulcus, the following genera are often said to contribute to the tissue destruction of periodontal
disease: Bacteroides, Porphyromonas gingivalis, Borelia, Spirochetes,Capnocytophaga, Fusobacteria
and Eichenella. Streptococci (mutans, mitis, sanguis, salivarius) are all gram positive facultative
anaerobes that do not require anaerobic conditions to survive. They are Viridans streptococci,
generally non-pathogenic, and not linked to periodontal disease, although S. mutans is clearly linked
to caries.

30

75. If a dentist were to administer a bacteriostatic antibiotic, such as a macrolide, and a


bactericidal antibiotic, such as a cephalosporin, concomitantly to a patient with a severe
intraoral infection, the most likely pharmacological result for the patient would be which of the
following?
A. A small extension of bacterial coverage between the two agents
B. Antagonism of the antibacterial effects of both agents
C. Profound toxicity for the patients taking both agents
D. Synergism of the antibacterial effects of both agents
E. The appearance of mild antifungal activity
The correct answer is B. The concomitant administration of bacteriostatic and bactericidal antibiotics
would result in antagonism of the antibacterial effects of both agents. The reason for the antagonism
between the two is that each agent interferes with the mechanism of action of the other agent. By
definition, a bacteriostatic agent is one that prohibits the growth and development of bacteria and a
bactericidal agent is one that directly causes the death of a microbe during the growth and
development stages. An extension of bacterial coverage between two agents (choice A) is typically
seen when two bacteriostatic or two bactericidal agents are given together. The administration of a
macrolide and a cephalosporin may cause some discomfort for the patient, such as nausea or
stomach cramps; however, profound toxicity for the patients taking both agents (choice C) is very
unlikely. Synergism (choice D) is the correlated action of two agents that results in a final effect that is
greater than that of each agent acting separately. A classical example of synergism is seen when an
aminoglycoside, such as gentamicin, is administered concomitantly with extended spectrum penicillin,
such as piperacillin, for the treatment of a pseudomonas infection. The appearance of mild antifungal
activity (choice E) would not be seen when bacteriostatic and bactericidal antibiotics are given
together, if neither have any antifungal activity.
76. Which of the following viruses is capable of replication in enucleated cells?
A. Adenovirus
B. Cytomegalovirus
C. Influenza virus
D. JC virus
E. Poliovirus
The correct answer is E. Most RNA viruses (eg, poliovirus) replicate in the cytoplasm and therefore
can replicate in enucleated cells. Poliovirus belongs to the family Picornaviridae. These viruses are
nonenveloped and have an icosahedral nucleocapsid that contains positive-sense RNA.
The exception to the rule regarding RNA viruses is the family Orthomyxoviridae, the influenza viruses
(choice C). Orthomyxoviruses undergo transcription and RNA replication in the nucleus of the host
cell because they need to cannibalize the capped 5' termini of cellular RNAs for use as primers for
viral mRNA transcription. Adenoviruses (choice A) are nonenveloped and have an icosahedral
nucleocapsid that contains a double-stranded linear DNA genome. Cytomegalovirus (choice B) is a
member of family Herpesviridae. It is an enveloped virus with an icosahedral nucleocapsid that
contains a double-stranded linear DNA genome. JC virus (choice D) belongs to family Papovaviridae.
It is nonenveloped and has an icosahedral nucleocapsid that contains a double-stranded circular DNA
genome. For most DNA viruses, transcription and DNA replication occur in the nucleus of the host
cell. The exception to this observation is the family Poxviridae, which carries out its replication in the
cytoplasm. Poxviridae includes variola virus, vaccinia virus, molluscum contagiosum, and orf virus.
77. Blood cultures for a patient are positive for alpha-hemolytic gram-positive diplococci.
Immunity to the causative organism is based on
A. alternative complement pathway activation
B. antibody to an alpha-helical coiled fimbria
C. IgA antibodies to C carbohydrate
D. IgG antibodies to C carbohydrate
E. IgG antibodies to a surface acidic polysaccharide
The correct answer is E. The patient in this question has pneumococcal pneumonia. It is caused by
Streptococcus pneumoniae, an alpha-hemolytic, gram-positive coccus that grows in chains. It can be

31

easily distinguished from other alpha-hemolytic streptococci because it is exquisitely sensitive to bile
and bile-like compounds, such as optochin. It is the most common cause of community-acquired
pneumonia and the most common cause of community-acquired meningitis in adults older than 30
years of age. The only recognized virulence factor of S. pneumoniae is its carbohydrate capsule
(which contains acidic polysaccharides). Antibody to a specific capsule type is necessary to overcome
infection. More than 80 capsule types have been recognized. The 23 types that most commonly
cause disease are contained in a vaccine (pneumovax) that is recommended for high-risk groups,
including the elderly and those undergoing splenectomy. Increased susceptibility is also found in
patients with Hodgkin disease, chronic lymphocytic leukemia, and myeloma. The vaccine should still
be given to patients with these conditions, but it is less successful. The alternative complement
pathway (choice A) is important in clearing Neisseria infections. Individuals with deficiencies in C5
through C8 are at increased risk for disease from Neisseria. The fimbria of Streptococcus pyogenes
(Group A beta-hemolytic streptococcus) is composed of an alpha-helically coiled M protein. Antibody
against a specific M type (choice B) will prevent infection. Raising antibodies to M proteins, however,
can lead to rheumatic fever, so strep throat infections are routinely treated with penicillin to prevent an
antibody response. The C carbohydrate is an antigen of beta-hemolytic streptococci used to divide
them into different groups. Antibody against C carbohydrate (choices C and D) is not protective.
78. A microbiology laboratory reports growth of Staphylococcus aureus from pus drained from
a breast abscess. What is the most likely condition predisposing the patient to the
development of a breast abscess?
A. Breast feeding
B. Endocarditis
C. Inflammatory breast carcinoma
D. Menopause
E. Paget disease of the breast
The correct answer is A. Acute mastitis, frequently complicated by breast abscess formation,
typically develops in the postpartum period when the nipples are first subjected to the physical
stresses of breast feeding. Bacteria enter the breast by way of cracks in the nipple and flourish in the
microenvironment of the lactating breast. Acute mastitis causes redness, pain, and swelling in the
affected breast; S. aureus is the most common pathogen. There is a very apparent cellulitis and fever
may be present. Penicillin antibiotics are generally administered because they are effective and safe
to use during breast-feeding. Although endocarditis (choice B) could send septic emboli to the breast,
S. aureus endocarditis is more commonly associated with bacteremia and fevers, proliferative
glomerulonephritis, valvular dysfunction, and emboli to the brain, kidneys, heart, and gut.
Inflammatory breast carcinoma (choice C) does not invoke an inflammatory response, and it is not
associated with an infection. Inflammatory breast carcinoma is characterized by tumor spread into the
dermal lymphatics, producing diffuse induration and skin tenderness with the typical peau d'orange
(orange peel) appearance. Mastitis arising in the perimenopausal period (choice D) is usually a
chronic mastitis caused by obstruction of ducts from inspissated secretions. Chronic mastitis is sterile.
The breast tissue shows lymphocytes and plasma cells surrounding dilated ducts filled by cellular
debris. Paget disease (choice E) presents as skin changes on the breast or nipple in association with
ductal carcinoma in the underlying breast that is percolating out into the epidermis. Paget disease
produces an eczematous skin lesion that may be crusted or weeping, but it is not related to bacterial
infection.
79. A 3-year-old child has multiple isolated lesions on his face and neck. The lesions are up to
4 cm in diameter with golden crusts, whereas in other sites small blisters and weeping areas
are seen. Which of the following is the most likely diagnosis?
A. Aphthous ulcers
B. Erysipelas
C. Herpes simplex I
D. Impetigo
E. Measles
The correct answer is D. This is impetigo, which is typically seen in preschool children with poor
hygiene, particularly in the summer in warm climates. The characteristic lesion has a large golden
crust. Most cases are caused by Staphylococcus aureus; Streptococcus pyogenes is occasionally

32

implicated. Impetigo is highly infectious, and mini-epidemics can occur in daycare settings. The initial
treatment is typically with penicillins and topical preparations, primarily muciprocin ointment.
Methicillin-resistant strains are presently rare in this setting, but can occur. Itching is the only
symptom. The lesions consist of macules and vesicles. Aphthous ulcers (choice A), commonly known
as "canker sores," are painful, shallow ulcers of the oral cavity. Erysipelas (choice B) is a different
type of skin infection, often caused by Streptococcus pyogenes (also sometimes others including
Staphylococcus), and is characterized by large erythematous patches. Herpes simplex I (choice C)
causes tiny oral and perioral vesicles, but not large golden crusts. Measles (choice E) causes a
blotchy erythematous rash on the face and all over the body. These lesions typically do not "weep" as
seen with impetigo.
80. A woman delivers a full term baby with possible toxoplasmosis, but no obvious signs of
infection with the protozoan parasite. The best test to diagnose acute infection in the newborn
would be to look for:
A. IgA
B. IgE
C. IgG 1
D. IgG 4
E. IgM
The correct answer is E. IgM immunoglobulin directed against Toxoplasma would provide evidence
of infection in the newborn baby. IgM is the only antibody that a baby can form with an acute infection.
It is the first antibody that is formed in an infection and it is also the antibody that is present on the
surface of immature and mature B cells. IgA (choice A) is the antibody that is produced in response
to mucosal infections. It cannot cross the placenta. The baby's immune system is still not well
developed at birth and cannot produce IgA immunoglobulin. In adults or older children, IgA exists in
serum in a monomeric form but is present in seromucous secretions as a dimer. IgE (choice B) is the
antibody that is produced in response to an allergen. It cannot cross the placenta, and is not present
in the baby at birth or for some time after birth. The baby's immune system is still not well developed
at birth and can initially only form IgM immunoglobulin. IgG 1 (choice C) and IgG4 (choice D)
immunoglobulins might be present in the baby, but would not indicate infection in the infant. The
presence of these isotypes would indicate that the mother was infected with the organism and
produced antibody that was transported across the placenta in utero. IgG is the only immunoglobulin
that can cross the placenta, providing protection to the baby during the first few months of life.
81. Two weeks after birth, a neonate develops sepsis, skin vesicles, and conjunctivitis. Over
the next several days, the infant's condition deteriorates with development of seizures, cranial
nerve palsies, and lethargy. The infant dies approximately 1 week after onset of symptoms.
Which of the following infectious agents would most likely cause this clinical presentation?
A. Cytomegalovirus
B. Herpes simplex
C. Rubella
D. Syphilis
E. Toxoplasmosis
The correct answer is B. All of the agents listed can cause devastating congenital infections with
high mortality and often major organ malformation (the TORCH agents: Toxoplasma, other, rubella,
cytomegalovirus, herpes simplex). It is herpes simplex type II, however, typically acquired during
delivery, that causes the devastating neonatal encephalitis described in the question stem. The
mortality rate for neonatal herpes is approximately 65%. Only 10% of these infants escape without
neurologic sequelae. A distinguishing feature is the propensity to involve the temporal lobe, with mass
effect on imaging studies and temporal lobe seizure foci on EEGs. A point worth remembering is that
adult herpes encephalitis (in nonimmunosuppressed individuals) is usually caused by herpes simplex
I, whereas neonatal and congenital herpes are usually caused by herpes simplex II.
82. A young woman presents with a confluent maculopapular rash that began on her face then
spread downward over her trunk. She also has fever and headache, with bilateral neck pain
and joint pain. Which of the following diseases does she most likely have?
A. Infectious mononucleosis
B. Lyme disease
C. Roseola
D. Rubella
E. Rubeola

33

The correct answer is D. Rubella, also called German measles or 3-day measles, is a disease
caused by a Togavirus, which is a small, enveloped, single-stranded (+) linear RNA virus.
Approximately 40% of patients are asymptomatic or have mild symptoms. In symptomatic patients,
the clinical presentation typically consists of an erythematous rash that begins on the head and
spreads downward to involve the trunk, and lasts for approximately 3 days. In addition, symptoms
include fever, posterior cervical lymphadenopathy, and arthralgias. The greatest danger from rubella
is to the fetus. If clinical rubella develops or seroconversion is demonstrated, there is a high risk for
congenital abnormalities or spontaneous abortion. Women of childbearing age should be warned not
to become pregnant within 2-3 months from the time of immunization. Mild arthralgias and other
symptoms may develop in 25% of immunized women. Enteroviral rashes may mimic rubella and
rubeola.
Infectious mononucleosis (choice A) is caused by the Epstein-Barr virus, a herpesvirus. Classic
findings include fever, exudative pharyngitis, generalized lymphadenopathy, severe malaise (most
common complaint), and hepatosplenomegaly. A rash is not a characteristic feature unless the patient
has been treated with ampicillin. Lyme disease (choice B) is caused by the spirochete Borrelia
burgdorferi. The disease is transmitted by the bite of the tick Ixodes dammini. The initial lesion is an
annular rash with central clearing and a raised red border (erythema chronicum migrans) at the bite
site. Patients also have fever, malaise, myalgias, arthralgias, headache, generalized
lymphadenopathy, and, occasionally, neurologic findings. Roseola (choice C) is caused by human
herpesvirus 6. Other names include exanthem subitum or sixth disease. Children have a febrile period
of 3-5 days with rapid defervescence followed by an erythematous maculopapular rash lasting 1-3
days. Rubeola (choice E), or regular measles, is a disease caused by a paramyxovirus. Patients
present with an upper respiratory prodrome and characteristic oral lesions (Koplik spots) that precede
the rash. The nonpruritic maculopapluar rash begins on the face and spreads to the trunk and
extremities, including palms and soles. The incubation period is 10-14 days. Patients also have a
posterior cervical lymphadenopathy. The virus is not associated with risk for a fetus.
83. A 29-year-old woman AIDS has had a progressive blurring of vision in her right eye. Which
of the following is the most appropriate therapy for this patient?
A. Acyclovir
B. Amantadine
C. Flucytosine
D. Ganciclovir
E. Zidovudine
The correct answer is choice D. The best drug treatment for CMV this infection of the retina is
ganciclovir. Acyclovir (choice A) is not effective in CMV infections. It is used more for HSV type 1 and
2 infections. Amantadine (choice B) is used either therapeutically or prophylactically for the influenza
A virus. Flucytosine (choice C) is an antifungal agent. Zidovudine (choice E) is a reverse
transcriptase inhibitor and is used as a first-line drug for the treatment of AIDS. The drug by itself is
ineffective against CMV retinitis.
84. The causative organism of gonorrhea has which of the following characteristics?
A. Both hyphae and spores
B. Can live within neutrophils
C. Cannot make ATP
D. No true cell wall
E. Stains with silver stains
The correct answer is B. The causative organism is Neisseria gonorrhoeae, which is a sexually
transmitted gram-negative coccus that can live in phagocytic vacuoles in neutrophils and
macrophages. The organism is the most common cause of septic arthritis in otherwise healthy,
sexually active adults. Cervicitis in women with purulent discharge or women who are asymptomatic,
yielding a positive culture, is a common charactristic. Fever, rash, tenosynovitis and arthritis occur
with disseminated disease. Hyphae and spores (choice A) would be features of Candida, which
usually causes arthritis as a rare complication of systemic candidiasis in immunocompromised
patients. Furthermore, there would be the appearance of other classic characteristics, such as the
appearance of creamy white lesions. Chlamydia is an intracellular organism that cannot make ATP

34

(choice C) and does not usually cause septic arthritis. Mycoplasma do not have true cell walls
(choice D) and do not usually cause septic arthritis, but are commonly associated with respiratory
conditions, such as mycoplasma tuberculosis. Syphilis is caused by a spirochete that stains with silver
stains (choice E). Secondary syphilis can (uncommonly) cause an acute arthritis, but the interval
between acquiring the infection and developing the arthritis is usually months to years.
85. A large lung abscess is biopsied from which pure Fusobacterium nucleatum is cultured.
What is the most likely source of the Fusobacterium?
A. Blood
B. Infected aerosols
C. Oral cavity
D. Stomach
E. Upper respiratory tract
The correct answer is C. Anaerobic bacteria make up most of the normal human oral flora. For
example, prominent members of the normal microbial flora of the mouth include anaerobic
spirochetes, fusobacteria, and prevotella. Anaerobic lung abscesses commonly arise from aspirated
oral secretions in patients with compromised neurologic status (e.g., anesthesia, coma, intoxication),
or in individuals with a depressed cough reflex. The most common anaerobic organisms isolated from
lung abscesses include Bacteroides, Fusobacterium, and Peptococcus species, all of which are
typical oral flora.
Blood-borne pathogens (choice A) producing lung abscesses probably originate as emboli from
vegetations on the right heart. The most likely pathogens in endocarditis are Staphylococcus and
Streptococcus species; Fusobacterium would be very unusual. Fusobacterium usually is not
pathogenic and lives in a commensal relationship with the host. Infective aerosols (choice B)
generally contain organisms that are intrinsically pathogenic (e.g., mycobacteria, influenza virus). It is
the acidity of the gastric aspirate that most seriously injures the lung, not the gastric bacteria.
Fusobacterium do inhabit the gastrointestinal tract, but viable bacteria are much more likely to be
transmitted from the oral secretions than from the stomach (choice D). The upper respiratory tract
(choice E) is generally sterile, although COPD is frequently complicated by chronic bronchitis.
Numerous bacterial and viral pathogens can be involved in chronic bronchitis; however,
Fusobacterium is not an upper respiratory tract pathogen.
86. A 67-year-old woman in a skilled nursing facility complains of flu-like symptoms of high
fever, dyspnea, and a productive cough. A chest x-ray shows a cavitary lesion in her left lung
which is confirmed to be an abscess. If a gram-positive bacteria is noted, which of the
following organisms would most likely be identified from examination of her sputum?
A. Candida albicans
B. Klebsiella pneumoniae
C. Pneumocystis carinii
D. Staphylococcus aureus
E. Streptococcus pneumoniae
The correct answer is D. The woman has developed a pulmonary abscess, as evidenced by the
chest radiograph. Of the organisms listed, Staph. aureus is the most likely cause of bacterial
pneumonia complicated by abscess formation. Bacteria commonly producing pneumonia developing
in the context of influenza include Staphylococcus aureus, Haemophilus influenzae, and
Streptococcus pneumoniae, but Streptococcus pneumoniae is not a frequent cause of lung
abscesses. Staph. aureus pneumonia/abscess are more common in patients in chronic care facilities,
nosocomial infections, influenza epidemics and cystic fibrosis. Candida albicans(choice A) would not
be a likely cause of this woman's abscess, or of the pneumonia that preceded it.Candida pneumonia
in a nonimmunosupressed individual is rare. Note also that Candida is a fungus. Klebsiella
pneumoniae(choice B) is more likely in men who are alcoholic or diabetic, or suffering from chronic
obstructive lung disease. It is also a plump gram-negative encapsulated rod. Pneumocystis
carinii(choice C) is associated with pneumonia in immunocompromised hosts, especially AIDS
patients, but there is no indication of immune compromise in this patient. Furthermore, it would not
show as a gram-positive organism, as it is a fungus. Streptococcus pneumoniae(choice E) is only
rarely associated with lung abscess. It commonly causes pneumonia in chronic lung diseases. It is a
gram-positive diplococcius.

35

87. Following a barbecue hosted by a hunter who served "bear hamburgers," several guests
develop abdominal pain, diarrhea, fever, myalgia, periorbital edema, and petechial
hemorrhages in the conjunctiva. A peripheral blood smear shows an increased eosinophil
count. Which of the following is the most likely cause of these symptoms?
A. Anthrax
B. Bacillus cereus food poisoning
C. Botulism
D. Escherichia coli gastroenteritis
E. Trichinosis
The correct answer is E. Although inadequately cooked pork and trichinosis are commonly
associated, the quality of purchased pork and the degree of cooking in this country are sufficiently
high that pork-related trichinosis is uncommon. The more common scenario is actually the one
described in the question stem, for two reasons. First, bear meat is not inspected, and it may often be
served "rare," or relatively uncooked. The infected meat contains cysts that are released in the
gastrointestinal tract after ingestion. The Trichinella spiralis larvae penetrate the intestinal mucosa and
develop into adult worms in 30-40 hours. After reaching adulthood, they mate, and the females
produce larvae that grow to maturity and seek out muscle in which to encyst (often in the orbital
muscles) within approximately 5-8 days. If you didn't know the answer to this question, you still might
have noticed the presence of eosinophilia, which should have clued you in to the presence of a
parasitic infestation, making trichinosis the only plausible answer.
Anthrax (choice A) after eating bear meat is very unlikely. In addition, anthrax would not produce the
described symptoms; instead, cutaneous lesions (95% of cases) or respiratory disease culminating in
death (5% of cases; wool sorter disease) would occur. Bacillus cereus food poisioning (choice B) and
Escherichia coli gastroenteritis (choice D) might lead to similiar gastrointestinal symptoms but would
not cause the eye findings. Botulism (choice C) causes flaccid paralysis, not the symptoms
described.
88. An alcoholic patient is brought to the emergency department in respiratory distress. A
chest x-ray film demonstrates lobar consolidation of the right lower lung. Which of the
following organisms should be highest on the differential diagnosis?
A. Klebsiella pneumoniae
B. Legionella spp.
C. Mycoplasma pneumoniae
D. Pneumocystic carinii
E. Staphylococcus aureus
The correct answer is A. Lobar pneumonia, in which an entire lobe of the lung becomes rapidly
affected with pneumonia, is acutally a relatively uncommon pattern for pneumonia. Common
causative organisms include Streptococcus pneumoniae (pneumococcus), Haemophilus influenzae,
and Klebsiella pneumoniae. K. pneumoniae is specifically associated with alcohol abuse, diabetes
mellitus, and nosocomial infections. Legionella spp. (choice B) can cause either a patchy or lobar
consolidation but are not specifically associated with alcoholism. Instead, Legionella infection is
associated with inspiration of aerosolized contaminated water. Mycoplasma pneumoniae (choice C)
causes extensive patchy infiltrates rather than lobar pneumonia. It is the most common cause of
pneumonia in young adults. Pneumocystis carinii (choice D) causes diffuse interstitial and alveolar
infiltrates, typically in patients immunosupressed by AIDS, cytotoxic drug therapy, or cancer.
Staphylococcus aureus (choice E) produces patchy infiltrates. Staphylococcal pneumonia may
complicate influenza during epidemics, or it may be a nosocomial infection. It is associated with
empyema formation.
89. An otherwise healthy patient who has just received a prosthetic aortic valve develops
postoperative fever. Blood cultures are done, and she is given broad-spectrum antibiotics.
Two days later she is still febrile and clinically deteriorating. Which of the following is the most
likely pathogen?
A. Actinomyces israelii
B. Candida albicans
C. Histoplasma capsulatum
D. Nocardia asteroides
E. Trichophyton rubrum

36

The correct answer is B. The patient likely has candidal infection of the prosthetic aortic valve. That
is why she did not respond favorably to antibacterial therapy, which is known to promote fungal
infection. Note that Candida tends to colonize foreign bodies, such as intravenous and Foley
catheters, prosthetic valves, and ventricular shunts. Actinomyces israelii (choice A) is known to cause
cervicofacial infection in patients undergoing dental work. Histoplasma capsulatum (choice C) causes
histoplasmosis, a pulmonary infection common in the midwestern river valleys. Multiorgan
involvement is usually seen only in the immunocompromised. Transmission of the organism occurs
through the inhalation of airborne microconidia (infectious) spores. The organism is found in bird and
bat droppings and in the soil. Nocardia asteroides (choice D) is an actinomycete that causes a
chronic lobar pneumonia that may metastasize to the brain. It is more common in
immunocompromised patients. It is found in the soil and in aquatic environments. Trichophyton
rubrum (choice E) is one of the organisms that commonly produces a variety of cutaneous mycoses,
including tinea corporis (ringworm), tinea cruris (jock itch), and tinea pedis (athlete's foot).
90. A formula-fed, 1-month-old boy is exposed to his sister, who has chickenpox. He does not
develop signs of varicella. His mother had the infection 5 years ago. Which class of
immunoglobulins did he acquire from his mother in utero that protected him from this virus?
A. IgA
B. IgD
C. IgE
D. IgG
E. IgM
The correct answer is D. This infant is exhibiting passive immunity acquired from his mother in utero.
IgG is the only class of immunoglobulin that can cross the placenta. As such, IgG molecules diffuse
into the fetal circulation, providing immunity. This circulating maternal IgG protects the newborn during
the first 4-6 months of life. Note that IgG is also capable of opsonization and complement activation (a
feature shared with IgM). IgA (choice A) functions in the secretory immune response. The secretory
form of this immunoglobulin (sIgA) is found in tears, colostrum, saliva, breast milk, and other
secretions. It is produced by the plasma cells in the lamina propria of the gastrointestinal and
respiratory tracts. IgD (choice B) functions as a cell surface antigen receptor on undifferentiated B
cells. IgE (choice C) is involved in the allergic response and immediate hypersensitivity reactions.
The Fc region of IgE binds to the surface of basophils and mast cells. Antigen binding to two IgE
molecules leads to mast cell degranulation and the release of leukotrienes, histamine, eosinophil
chemotactic factors, and heparin. IgM (choice E) is the first antibody detected in serum after
exposure to antigen. IgM circulates as a pentamer and thus has five Fc regions. This structure makes
it especially effective in fixing complement.
91. Which of the following organisms is a natural transformer?
A. Escherichia coli
B. Neisseria gonorrhoeae
C. Plasmodium vivax
D. Pseudomonas aeruginosa
E. Staphylococcus aureus
The correct answer is B. Transformation is the uptake and integration of naked DNA by a bacteria
from the environment. Transformation can be induced in the laboratory (a technique used to introduce
gene-carrying plasmids into bacteria) or, relatively uncommonly, occurs naturally. Only a few
medically important species undergo natural transformation: Haemophilus species, Streptococcus
species, Neisseria gonorrhoeae, and Helicobacter pylori.
92. A patient presents with chronic, symmetric polyarthritis involving the knees. He gives a
history of having developed an extensive rash after a deer hunting trip in Connecticut after
which he felt "sick," and developed knee pain that prevented him from climbing stairs for
several months, but then it partially resolved. Which of the following organisms is most likely
etiologically related to the patient's arthritis?
A. Fungus
B. Gram-negative cocci
C. Gram-negative rod
D. Gram-positive cocci
E. Spirochete

37

The correct answer is E. The history is characteristic for Lyme disease, caused by the spirochete
Borrelia burgdorferi. The clues are deer, Connecticut, rash, knees, and acute arthritis followed by
chronic arthritis. Patients with Lyme disease initially have erythema migrus, a flat or slightly raised
lesion that expands with central clearing. Headache and stiff neck are also common. Arthralgias,
arthritis, and myalgias are seen. The arthritis is often chronic and recurrent. Fungal arthritis (choice
A) can be caused by coccidioidomycosis, blastomycosis, sporotrichosis, and candidiasis. N.
gonorrhoeaeare gram-negative cocci (choice B) that can cause septic arthritis. Septic arthritis is
associated with acute arthritis, usually monarticular, most often in large weightbearing joints. Previous
joint damage or intravenous drug abuse are common risk factors. Haemophilus (in young children)
and Salmonella (in sickle cell patients) are gram-negative rods (choice C) that can cause septic
arthritis. Staphylococcus are gram-positive cocci (choice E) that can cause septic arthritis.
93. What is the mechanism of action of the antibiotic ciprofloxacin?
A. It inhibits dihydrofolate reductase
B. It inhibits DNA-dependent RNA polymerase
C. It inhibits protein synthesis by binding to the 30s ribosomal subunit
D. It inhibits protein synthesis by binding to the 50s ribosomal subunit
E. It inhibits topoisomerase II (DNA gyrase)
The correct answer is E. Antibiotic mechanism questions are NBDE Part 1 favorites. Ciprofloxacin
and norfloxacin belong to a category of antibiotics called the fluroquinolones. They are bactericidal
and work by inhibiting topoisomerase II (DNA gyrase). They are effective against gram-negative rods
and are the only oral agents effective against Pseudomonas. Inhibition of dihydrofolate reductase
(choice A) is the mechanism of action of trimethoprim, which is typically used in combination with
sulfonamides which inhibit an earlier step in folate synthesis (dihydropteroate synthase).
Trimethoprim-sulfa is used in the treatment of Shigella, Salmonella, recurrent urinary tract infections,
and Pneumocystis carinii pneumonia. Inhibition of DNA-dependent RNA polymerase (choice B) is the
mechanism of action of rifampin. Rifampin is used (along with other drugs) in the treatment of
tuberculosis. Inhibition of the 30s ribosomal subunit (choice C) is the mechanism of action of two
important classes of antibiotics - the tetracyclines (tetracycline, doxycycline, democlocycline) and the
aminoglycosides (gentamycin, tobramycin, streptomycin, etc.). The tetracyclines inhibit the
attachment of the aminoacyl-tRNA to the ribosome, whereas the aminoglycosides inhibit the formation
of the intitiation complex. Inhibition of the 50s ribosomal subunit (choice D) is the mechanism of
action of the macrolides (e.g., erythromycin), the lincosamides (e.g., lincomycin, clindamycin), and
chloramphenicol. Chloramphenicol inhibits the 50s peptidyl transferase, whereas erythromycin blocks
translocation.
94. An injection of lipopolysaccharide (LPS) into the vascular system will rapidly produce
myocardial dysfunction, hypotension, disseminated intravascular coagulation, and coma. This
sequence of events most closely mimics what type of shock?
A. Anaphylactic
B. Cardiogenic
C. Hypovolemic
D. Neurogenic
E. Septic
The correct answer is E. Since lipopolysaccharide can be seen in gram-negative bacteria, it is
important to understand how it will affect the body. Septic shock is a complex, multisystem organ
failure that can be produced either by LPS (which is present in the cell wall of all gram-negative
bacteria) or certain toxins released by gram-positive bacteria and fungi. LPS binds to a serum protein
and stimulates CD14 receptors on endothelial cells and circulating inflammatory cells, eliciting a broad
range of end-organ responses. Anaphylactic shock (choice A) is brought about by an exaggerated
Type I hypersensitivity reaction mediated by IgE antibodies bound to mast cells and basophils. The
resulting degranulation produces massive histamine and adenosine release, which produces
constriction of the bronchi and pulmonary circulation. Cardiogenic shock (choice B) reflects the
inability of the heart to maintain arterial pressure sufficiently to perfuse the systemic vasculature.
Cardiogenic shock is intrinsic to the heart and usually a consequence of ischemia, arrhythmia, or
obstruction. Hypovolemic shock (choice C) occurs when blood volume decreases to a point at which
it is inadequate to maintain arterial pressure in the vital organs. Hypovolemic shock is due to
hemorrhage, fluid loss from burns, or severe diarrhea and vomiting. Neurogenic shock (choice D) is
an unusual form of shock that occurs in catastrophic nervous system injuries that cause diffuse
vasodilation and hypotension.

38

95. A 28-year-old male intravenous drug user presents with fever, chills, weakness, dyspnea,
cough, arthralgia, diarrhea, and abdominal pain. On examination, a heart murmur is present,
and small, tender nodules are found on the finger and toe pads, along with small hemorrhages
on the palms and soles. Infective endocarditis, caused by which organism, should be
suspected?
A. Enterococci
B. Pseudomonas aeruginosa
C. Staphylococcus aureus
D. Streptococcus pneumoniae
E. Streptococcus pyrogenes
The correct answer is C. The question is essentially asking "Which of the following is the most
common cause of infective endocarditis (IE) in intravenous drug users?" Being an intravenous drug
user places him at a very high risk for the development of IE. The factors that determine the clinical
presentation of IE include the nature of the organism, the valve infected, and the route of the infection.
Although the clinical characteristics of IE can vary, most patients present with a febrile illness lasting
several days to 2 weeks. This illness is often accompanied by a variety of nonspecific signs and
symptoms, such as chills, weakness, dyspnea, cough, arthralgia, diarrhea, and abdominal pain. Heart
murmurs occur in approximately 90% of all patients, but may be absent in patients with right-sided
infections. Other clinical signs include Osler nodules (purplish or erythematous subcutaneous papules
or nodules on the pads of the fingers and toes), Janeway lesions (hemorrhagic painless plaques on
the palms and soles), petechiae (small erythematous painless hemorrhagic lesions that may appear
anywhere), and splinter hemorrhages (thin linear hemorrhages found under the nail beds of fingers
and toes). The diagnosis of IE is dependent on positive blood cultures and echocardiographic
evidence of "valvular vegetation" or valvular injury. Staphylococci, Streptococci, and Enterococci are
possible pathogens; however, Staphylococcus aureus accounts for most cases of acute IE.
Enterococci(choice A) cause a minority of cases of valvular endocarditis, but are not usually involved
in endocarditis associated with intravenous drug abuse. Gram-negative organisms such as
Pseudomonas aeruginosa(choice B) are rarely the cause of IE. They primarily cause pneumonia in
cystic fibrosis patients and those on ventilators. Streptococcus pneumoniae(choice D) and
Streptococcus pyogenes(choice E) are not common causes of IE. They are, however, common
causes of pneumonia and pharyngitis, respectively.
96. A construction worker with sustained trauma later develops tetanus. What is the causative
organism?
A. Gram-negative anaerobic rod
B. Gram-positive cocci
C. Gram-positive anaerobic rod
D. Gram-positive aerobic rod
The correct answer is C. The causative organism of tetanus is Clostridium tetani , which is a grampositive, spore-forming anaerobic rod. Bacteroides is gram-negative anaerobic rod (choice A).
Examples of gram-positive cocci (choice B) are staphylococci and streptococci. The causative
organism of anthrax, Bacillus anthracis , is a gram-positive, spore-forming aerobic rod (choice D).
97. Last year's influenza A vaccine is unlikely to be effective today because influenza A
A. has a heavy polysaccharide coat
B. immunosuppresses the patient
C. kills lymphocytes
D. resists inactivation by complement
E. undergoes genetic reassortment
The correct answer is E. The difficulty with developing a vaccine against influenza A arises because
the influenza virus genome is composed of eight strands of single-stranded RNA. Minor shifts
(antigenic drift) in surface antigens that occur as point mutations in the genes accumulate. Influenza A
can also undergo larger, abrupt changes in antigen expression (antigenic shift), however, as a
consequence of reassortment of some of the RNA fragments between human and nonhuman hosts.

39

Thus, last year's vaccine does not necessarily work against this year's virus. Polysaccharide coats
(choice A) are a virulence factor of some bacteria, not of viruses. Influenza A can compromise the
lungs sufficiently to predispose to secondary infections, producing a functional immunosupression
(choice B), but this attribute does not make it difficult to produce vaccines against the virus. Unlike
AIDS, influenza virus does not selectively target lymphocytes (choice C). Influenza A, bound to
antibody and complement, can be effectively phagocytized (compare with choice D).
98. Infections in hyaline cartilage typically destroy the cartilage because
A. cartilage contains chondroitin sulfate
B. cartilage contains collagen
C. cartilage is not innervated
D. cartilage is relatively avascular
E. cartilage tends to become calcified
The correct answer is D. Hyaline cartilage contains very few, sparsely distributed, small blood
vessels. The tissue consequently is very vulnerable to infection because the immune system (white
cells and serum with clotting factors and antibodies) cannot access the cartilage in an efficient
fashion. Chondroitin sulfate (choice A) is an important constituent of cartilage that can be attacked by
bacteria, but this is not the cause of uncontrollable infections in the cartilage, because if the infection
is controlled, chondrocytes will continue to synthesize chondroitin. Collagen (choice B) is present in
cartilage, but does not predispose the cartilage for uncontrollable infection. Collagen is not innervated
(choice C), but this does not contribute to the development of uncontrollable infection. The tendency
of cartilage to undergo calcification (choice E), even when not acting as the site of ossification, does
not predispose for uncontrollable infection.
99. The highest level of chronic disease state is associated with which of the following
diseases?
A. Hepatitis A
B. Hepatitis B
C. Hepatitis C
D. Hepatitis D
E. None of these diseases have chronic carrier stages
The correct answer is choice C. Hepatitis C is a disease about which much has been learned
recently, but much more is unknown. It seems to be transferred through both bloodborne and sexual
routes (not fecal/oral). It is harder to contract than HBV, but easier than HIV. It is distinguished by
having the highest percentage of cases going into a chronic destructive stage, of all hepatitis types. It
is a single stranded RNA virus. It was formerly known as NANB (Non-A Non-B) or "transfusion
hepatitis" because tests for hepatitis A or B did not detect it. Hepatitis B also has a destructive carrier
state, but it is less common. Hepatitis A has NO carrier state, and exposure to Hepatitis A gives
lifelong immunity.
100. Which of the following organisms would most likely be isolated from the vagina of a
normal 5-year-old girl?
A. Candida
B. Lactobacillus
C. Neisseria
D. Staphylococcus
E. Streptococcus
The correct answer is D. The vagina of prepubertal girls and postmenopausal women is colonized
by colonic and skin bacteria, including Staphylococcus epidermidis, which is normally found on the
skin. The vagina of women of childbearing age tends to be colonized by Lactobacillus (choice B)
species, yeasts such as Candida (choice A), and Streptococcus species (choice E). The presence of
Neisseria (choice C), such as N. gonorrhoeae (the cause of gonorrhea), in the vagina of a 5-year-old
girl strongly suggests sexual abuse.
101. In cases of strepococcus pneumonia, why is the patient's sputum filled with pus?
A. Teichoic acids and peptidoglycan are chemotactic for neutrophils
B. The capsule of the causative agent is chemotactic for neutrophils
C. The causative agent is an intracellular organism
D. The causative agent is beta hemolytic
E. The organism produces an IgA protease

40

The correct answer is A. The answer to this question requires that the student realize that pus
consists of bacteria and dead and dying neutrophils. This fact reflects that the patient has typical
(rather than atypical) pneumonia. In the United States, the most common agent of this would be
Streptococcus pneumoniae, a gram-positive extracellular pathogen rich in teichoic acids and
peptidoglycan that elicit the neutrophilic exudate. The capsule of this organism is a polysaccharide
and primarily elicits an antibody response rather than attracting neutrophils (choice B). Streptococcus
pneumoniae is an extracellular, not an intracellular (choice C) organism. Streptococcus pneumoniae
is alpha hemolytic, not beta hemolytic (choice D). This organism can be distinguished from other
bacteria by the inhibition of growth in the presence of optochin or bile. Another common alpha
hemolytic bacteria is the Viridans streptococci group. Streptococcus pneumoniae does produce an
IgA protease (choice E) that enhances the ability of the organism to infect the respiratory mucosa, but
this does not contribute to pus formation.
102. Ethylene oxide sterilization procedures usually require exposure times of
A. 20-30 minutes
B. 1-2 hours
C. 2-3 hours
D. 8-10 hours
E. 24 hours
The correct answer is D. Ethylene oxide is used for sterilization of heat sensitive instruments and
materials. Its advantage is that it does not melt, damage, or corrode instruments, as heat may. Its
disadvantages include long cycle time (8-10 hours) and additional time to air out the materials so that
they do not contain ethylene oxide residue. A period of 20-30 minutes is a common steam autoclave
time. A period of 1-2 hours is a common dry heat autoclave time. A period of 24 hours was a time
recommended for "cold sterilization" by chemicals. Cold sterilization is a misnomer, as it is actually
high level disinfecton.
103. Approximately 7-8 days after starting an antibiotic therapy for an intraoral abscess, the
patient begins to experience severe diarrhea. If a diagnosis of antibiotic-induced colitis caused
by clostridium difficile is made, the colitis is most likely to be treated with which of the
following?
A. Attapulgite
B. Clindamycin
C. Loperamide
D. Metronidazole
The correct answer is D. Pseudomembranous colitis is caused by a necrolytic toxin produced by
Clostridium difficile, which can proliferate in the bowel secondary to the use of antibiotics that disrupt
the normal bowel flora. Almost all antibiotics (but most commonly clindamycin and ampicillin) can
cause this disorder, which results in profuse watery or bloody diarrhea. The most effective treatment
for pseudomembranous colitis is vancomycin or metronidazole. Metronidazole is a bactericidal agent
that is indicated for the treatment of serious infections caused by anaerobic bacteria and is active
against most protozoa. It is primarily used for the treatment of amebiasis and trichomoniasis in
outpatient settings and for prophylactic use in colorectal, gynecologic, and abdominal surgery. The
use of antidiarrheal agents, such as attapulgite (choice A) and loperamide (choice C), is
contraindicated. Clindamycin (choice B) is a bacteriostatic antibiotic used primarily in the treatment of
many different anaerobic infections. This is one of the primary antibiotics implicated in the
development of pseudomembranous colitis, and so will not be used to treat it.
104. What is the treatment of choice for an infection with Cryptococcus neoformans?
A. Amphotericin B
B. Isoniazid
C. Clotrimazole
D. Metronidazole
E. Nystatin
The correct answer is A. Amphotericin B is the most appropriate drug listed for the treatment of
cryptococcal meningitis. It is a polyene antibiotic that binds to ergosterol in the fungal cell membrane,

41

creating an artificial pore. Flucytosine is often prescribed as an adjunct medication. Fluconazole is


used long term to prevent recurrence in AIDS patients. These are all systemic antifungal. Isoniazid
(choice B) inhibits the biosynthesis of mycolic acids in the mycobacterial cell wall. It is the primary
drug used against tuberculosis. It is used alone for TB prophylaxis and in combination with other
antituberculars to treat patients with active disease. Clotrimazole (choice C) is a typical antifungal
that, like nystatin, is used primarily in the treatment of oral candidiasis. It is from the imidazole class of
antifungals. Metronidazole (choice D) is an antiprotozoal drug useful in treating a variety of parasitic
infections. It is the drug of choice for trichomoniasis and giardiasis and provides general anaerobic
coverage. Nystatin (choice E) is an antifungal polyene that is usually used topically but can be taken
orally for oral and esophageal candidiasis. Candidal infections of the skin, mucous membranes, and
vagina usually respond well to this drug. It may also be used to prevent intestinal fungal overgrowth in
patients on chemotherapy.
105. Which of the following terms describes a different condition from all of the others?
A. ARC
B. AIDS prodrome
C. Pre-AIDS
D. AIDS
E. AIDS-related complex
The correct answer is D. The disease caused by HIV virus progresses in stages that vary greatly in
length from individual to individual. HIV+ individuals early in the disease are often asymptomatic and
otherwise healthy. The HIV test shows only exposure to the virus and formation of antibody (the test
tests for antibody, not virus). This asymptomatic stage may continue for several years. Much later,
AIDS defining conditions (e.g., Kaposi sarcoma, cryptococcal meningitis, PCP pneumonia) combined
with HIV+ blood test, combine to give the diagnosis of AIDS. In between however, an early
symptomatic stage is known as ARC (AIDS-related complex), AIDS prodrome or pre-AIDS. Oral
candidiasis, night sweats, and weight loss are common at this stage.
106. Which of the following is the first antibody that an infant makes and is characteristically
produced during the primary immune response?
A. Immunoglobulin (Ig)G
B. IgM
C. IgA
D. IgE
E. IgD
The correct answer is B. IgM, produced during the immune response and is the first antibody that an
infant makes, is the most efficient immunoglobulin at activating complement. IgG (choice A) is
produced during the secondary immune response and is the only maternal immunoglobulin to cross
the placenta. IgA (choice C) is the main immunoglobulin in external secretions, such as saliva and
tears. It also protects mucosal surfaces. IgE (choice D) mediates immune hypersensitivity reactions
and is important in providing immunity against some parasites. IgD (choice E) is present in the
membrane of mature B cells and functions in antigen recognition by B cells.
107. An antibiotic, such as penicillin, which modifies cell wall synthesis, tends to be most
effective during which phase of bacterial growth in a closed system?
A. Lag phase
B. Log phase
C. Phase of decline
D. Stationary phase
The correct answer is B. Penicillin is a bactericidal agent with broad gram-negative and grampositive coverage. This agent inhibits the biosynthesis of cell wall mucopeptides. Bacterial growth in a
closed system is characterized by four phases: (1) In the initial lag phase, no growth occurs as the
organisms adapt to the new environment. (2) In the exponential, or log phase, the organisms grow at
the fastest rate and antibiotics that interfere with cell growth or division are most likely to be effective.
(3) In the stationary phase, when nutrients have been largely exhausted, organisms tend to stop
growing but may remain viable for long periods of time. (4) In the phase of decline, cell deaths
increase due to cell starvation or exposure to toxins.

42

108. A child with sickle cell anemia states that she has been feeling very tired lately, and then
states that she may have "come down with a virus." The girl is very pale except for her red
cheeks, and a complete blood count shows severe anemia. A bone marrow aspirate contains
no erythroid precursor cells. The girl was probably infected with which of the following
viruses?
A. Coxsackie virus
B. Echovirus
C. Hepadnavirus
D. Herpes virus
E. Parvovirus
The correct answer is E. Patients with sickle cell anemia typically present with a fiery red "slapped
cheek" appearance. Parvoviruses are small single-stranded DNA viruses, of which only serotype B19
is pathogenic for humans. This virus causes three distinct syndromes: a childhood febrile rash known
as erythema infectiosum ("Fifth disease"); aplastic crisis in individuals with chronic hemolytic diseases
(e.g., sickle cell anemia, thalassemia); and congenital infections that can present as stillbirth, hydrops
fetalis (analogous to severe Rh incompatibility), or severe anemia. In immunosuppressed patients and
those with sickle cell disease, anemia caused by red cell hypoplasia occurs as a consequence of
binding to erythrocyte P antigen. Coxsackie viruses (choice A) usually cause cold-like illness, but can
cause herpangina, myocarditis, and meningitis. Echoviruses (choice B) can infect a variety of organ
systems (gastrointestinal, CNS, eyes, heart, respiratory, skin), but are not a cause of aplastic crises.
Hepadnavirus (choice C) is the causative agent of hepatitis B. Herpes viruses (choice D) cause a
variety of acute to chronic infections, including herpes simplex types I and II, chicken pox, chronic
herpes zoster, cytomegalovirus infection, and Epstein Barr virus infections.
109. A baby born at 32 weeks gestation developed a fever and leukocytosis; lumbar puncture
revealed pleocytosis with increased protein, decreased glucose, and gram-positive rods.
Which one of the following organisms was most likely isolated from the CSF?
A. Escherichia coli
B. Listeria monocytogenes
C. Neisseria meningitidis
D. Streptococcus agalactiae
E. Streptococcus pneumoniae
The correct answer is B. The three major causes of neonatal meningitis are group B streptococci
(Streptococcus agalactiae; choice D), Escherichia coli (choice A), and Listeria monocytogenes. All
can be found in the vaginal tract of normal women and may contaminate the infant during passage
through the birth canal. They colonize the upper respiratory tract and can cause pneumonia,
septicemia, and/or meningitis in the neonate. They are readily distinguished on morphologic grounds;
the streptococci are gram-positive cocci in chains, E. coli is a gram-negative rod, and L.
monocytogenes is a gram-positive pleomorphic rod. There are other gram-positive rods that resemble
Listeria (e.g., the diphtheroid bacilli found in the upper respiratory tract and on the skin), hence a
motility test is done to confirm the identification. L. monocytogenes is also associated with drinking
unpasteurized milk. Neisseria meningitidis(choice C) is the most common cause of meningitis in
school-age children and young adults. It is a fastidious, non-motile, gram-negative diplococcus that
would be a very rare cause of meningeal disease in very young patients, such as this one.
Streptococcus pneumoniae(choice E) is a gram-positive coccus that grows in pairs and short chains.
It is the number one cause of pneumonia, septicemia, and meningitis in the elderly. There is a vaccine
composed of the capsular carbohydrate of 23 serotypes of this organism that is routinely given to
individuals over the age of 60, as well as to individuals with splenic abnormalities (e.g., sickle cell
disease) who are at increased risk for the development of pneumococcal sepsis.
110. A hospitalized burn patient has developed a severe pneumonia caused by a gramnegative, nonfermenting rod. The organism produces a blue-green pigment on growth media
and has a grape-like fruity odor. The organism isolated is most likely
A. Escherichia coli
B. Klebsiella pneumoniae
C. Legionella pneumophila
D. Pseudomonas aeruginosa
E. Moratella catarrhalis

43

The correct answer is D.Pseudomonas aeruginosa is a common opportunist in burn patients, in


whom it classically causes secondary wound infections and septicemia. Pseudomonal infections are
also commonly seen in patients on ventilators. It may also cause cystitis in patients with urinary
catheters and pneumonia in patients with cystic fibrosis. The organism is found in water and usually
gains access to the body as a contaminant in the water used in respirators or in water baths used to
cleanse wounds. This organism is a nonfermenter, that is, it does not metabolize sugars by classic
pathways. It produces a blue-green, water-soluble pigment (pyocyanin), and has a fruity odor when
grown on laboratory media. Escherichia coli(choice A) is a lactose-fermenting, gram-negative rod
commonly seen as normal flora of the intestine. It is the most common cause of urinary bladder
infections, pyelonephritis, and sepsis in patients with indwelling urinary catheters. It is also the major
cause of traveler's diarrhea and is a very important pathogen in neonates who become infected during
passage through the birth canal. Klebsiella pneumoniae(choice B) is a gram-negative, highly
encapsulated rod that is a significant pulmonary pathogen in individuals with a respiratory
compromise. It is a common cause of aspiration pneumonia and pulmonary abscesses in alcoholics
and patients with chronic obstructive pulmonary disease. Legionella pneumophila(choice C) is a
fastidious, gram-negative respiratory pathogen that may cause either a fulminating disease or a mild
"walking pneumonia-like" condition (i.e., an atypical pneumonia). It usually affects older or debilitated
patients. Moratella catarrhalis (choice E) is generally seen in patients with preexisting lung disease,
elderly patients, and immunosupressed patients. It is a gram-negative diplococcus that produces a
patchy infiltrate with occasional lobar consolidation.
111. debilitated 72-year-old woman develops dry cough, fever, headache, and muscular pains.
She treats herself with ampicillin without any improvement. Chest x-ray films reveal scattered
opacities, suggestive of interstitial infiltration. Laboratory investigations demonstrate the
presence of cold agglutinins. She is treated with erythromycin, and her symptoms rapidly
improve. Which of the following is the most likely etiologic agent of this condition?
A. Influenza virus
B. Mycoplasma pneumoniae
C. Pneumocystis carinii
D. Respiratory syncytial virus
E. Streptococcus pneumoniae
The correct answer is B. The patient's clinical presentation is typical of primary atypical pneumonia.
Gram stains will often show PMN's and monocytes. No bacteria is seen. In contrast to bacterial
pneumonia, primary atypical pneumonia presents with the following features: - Caused by M.
pneumoniae; less frequently by viruses (influenza, respiratory syncytial virus, adenovirus,
rhinoviruses, rubeola and varicella virus), Chlamydia, or Coxiella burnetii - Characterized
pathologically by interstitial, rather than intra-alveolar, inflammation - Characterized clinically by
nonspecific symptomatology and few "localizing" symptoms. Why is M. pneumoniae, and not
influenza virus (choice A) or respiratory syncytial virus (choice D), the cause of this patient's
pneumonia? First, M. pneumoniae infections are often associated with the appearance of cold
agglutinins in the serum, detection of which is diagnostically important. Second, the patient responded
quickly to treatment with erythromycin, an antibiotic effective against M. pneumoniae, but obviously
not effective in treating viral infections. As a side note: the macrolides, such as erythromycin are the
drugs of choice for atypical pneumonia. Pneumocystis carinii(choice C) is a fungal organism causing
pneumonia in severely immunocompromised hosts, especially AIDS patients. P. carinii pneumonia
(PCP) is characterized by accumulation of a frothy exudate containing numerous organisms within
alveolar spaces. Also, P. carinii is not sensitive to erythromycin. The drug of choice for treatment of
PCP is trimethoprim-sulfamethoxazole. Streptococcus pneumoniae(choice E) is the usual causative
agent of lobar pneumonia, characterized by consolidation of a single lobe due to intra-alveolar acute
inflammatory exudation. Lobar pneumonia is more prevalent in young, healthy individuals, whereas
primary atypical pneumonia favors old, debilitated patients. S. pneumoniae is highly sensitive to
penicillin.
112. Microscopic examination of a urine sample demonstrates many gram-negative rods,
many neutrophils, and a few white blood cell (WBC) casts. Which of the following is the
specific significance of the presence of WBC casts?
A. One or both kidneys are involved in the infection
B. One or both ureters are involved in the infection
C. The bladder is involved in the infection
D. The urethra is involved in the infection
E. The uterus is involved in the infection

44

The correct answer is A. The patient described has a urinary tract infection. The presence of white
blood cell casts specifically implies renal involvement, because these form when the leukocytes are
compressed together in the renal tubules. Similarly, the presence of red blood cell casts in a bloody
urine would indicate that at least some of the bleeding was occurring in the kidney. White cell casts
are seen in pyelonephritis and interstitial nephritis and are indicative of infection or inflammation. Note
also that the presence of bacteria merely indicates infection somewhere in the excretory system.
Infection of other sites in the urinary tract, including ureters (choice B), bladder (choice C), and
urethra (choice D), does not cause cast formation. Although spread of a urinary tract infection to the
uterus (choice E) is always of concern in a pregnant woman, the presence of casts does not
specifically suggest that this has occurred.
113. A 3-month-old infant presents with a 3-day history of fever, cough, and poor feeding. The
infant seems ill and has a temperature of 102 F and a respiratory rate of 32. A chest x-ray film
shows bilateral patchy infiltrates in the lungs. Which of the following is the most likely
etiologic agent?
A. Coronavirus
B. Influenza type A
C. Parainfluenza type 1
D. Respiratory syncytial virus
E. Rhinovirus
The correct answer is D. Respiratory syncytial virus is the most common cause of bronchiolitis,
tracheobronchitis, and pneumonia in children younger than 1 year of age. The average incubation
period is 5 days. Outbreaks occur seasonally in winter and early spring. Infection does not result in
lasting immunity, and reinfection can occur. In bronchiolitis, proliferation and necrosis of bronchiolar
epithelium develop, producing obstruction from sloughed epithelium and increased mucus secretion.
Signs often include fever, tachypnea, and wheezes. Hyperinflated lungs and otitis media are frequent
conditions. Coronavirus (choice A) causes approximately 20% of common colds (nasal obstruction
and discharge, sneezing, no fever or mild fever, occasional sore throat, and cough) and acute
pharyngitis (sore throat, with or without cervical adenopathy, ulceration, and conjunctivitis). Influenza
type A (choice B) is the leading cause of influenza. Influenza is a systemic illness characterized by
the sudden onset of fever, headache, myalgias, malaise, and prostration, followed by cough, nasal
obstruction, and sore throat. The lower respiratory tract may also be involved. Parainfluenza viruses
(choice C) are the leading cause of croup, or acute laryngotracheobronchitis, in children. This
infection involves the upper and lower respiratory tracts. Inflammation in the subglottal area leads to
hoarseness, dyspnea, a barking cough, and inspiratory stridor. Rhinovirus (choice E) is the most
common cause of the common cold.
114. Which of the following substances takes part in the nonoxidative killing pathway by
which pathogenic bacteria can be killed?
A. Hydrogen peroxide
B. Hypochlorous acid
C. Lysozyme
D. Myeloperoxidase
E. Superoxide ions
The correct answer is C. This is a microbiology question that requires knowledge about the
physiology of bacteria. Lysozyme is present in tears, saliva, mucus, vaginal secretions, and several
other body fluids. This material lyses the peptidoglycan layer of the cell wall of bacteria without
participation of any of the elements of the oxidative killing pathway. Hydrogen peroxide (choice A) is
formed in the oxidative killing pathway from the superoxide ions. This material is also very toxic to
bacterial pathogens. Hypochlorous acid (choice B) is formed from hydrogen peroxide and halide ions
in the presence of myeloperoxidase. Myeloperoxidase (choice D) is an enzyme present in the
oxidative killing pathway that breaks down hydrogen peroxide. Superoxide ions (choice E) are formed
by NADPH oxidase in the first reaction of the oxidative killing pathway. These superoxide ions are
extremely toxic to bacterial organisms.
115. A 65-year-old man undergoes coronary artery bypass surgery that entails grafting the left
internal mammary artery to the left main coronary artery to form an alternative conduit for
blood flow. This patient received a(n)
A. allograft
B. autograft
C. homograft
D. isograft
E. xenograft

45

The correct answer is B. Tissue grafts in which the same individual acts as both the donor and the
recipient are termed autografts. Allografts (choice A) refer to tissue transplants from one person to
another. Homograft (choice C) is a synonym for allograft. Isografts (choice D) refer to tissue
transplants between genetically identical individuals (e.g., an identical twin donates her kidney to her
twin sister). Xenografts (choice E) refer to tissue transplants from another species (e.g., baboon heart
transplanted into a human).
116. A 10-year-old boy is attending summer camp, complains of a sore throat, headache,
cough, malaise, a low-grade fever, patchy exudate, and keratoconjunctivitis. Within hours,
several other campers show similar symptoms. The most likely pathogen involved is:
A. Adenovirus
B. Chlamydia spp
C. Gram-negative diplococcus
D. Gram-positive enterococcus
E. Herpesvirus
The correct answer is A. There are more than 40 antigenic types of adenoviruses that produce a
variety of symptoms. The incubation period is 4-9 days. Adenoviruses are nonenveloped (naked),
icosahedral DNA viruses causing a variety of clinical syndromes. Adenoviruses cause a
pharyngoconjunctivitis that affects children and sometimes adults who are living in the same
household. Contaminated swimming pools have been implicated as sources for the virus. The virus is
latent in the lymphoepithelial tissue of the nasopharynx and other sites. Adenoviruses also cause
watery, nonbloody diarrhea. Fever often lasts 2-12 days and is accompanied by malaise and myalgia.
Sore throat is often manifested by a patchy exudate and cervical lymphadenopathy. Conjunctivitis is
often present. Chlamydia spp. (choice B) produce a variety of clinical syndromes, including a sexually
transmitted urethritis, pelvic inflammatory disease, neonatal pneumonia and inclusion conjunctivitis,
lymphogranuloma venereum, adult interstitial pneumonia, and a zoonotic pneumonitis. A gramnegative diplococcus (choice C) would be a Neisseria spp. or Moraxella. Neisseria meningitidis would
be the logical choice, because it is associated with outbreaks among children. The clinical syndrome
caused by infection with the meningococcus, however, is one of a fulminating, progressive septicemia
or meningitis with fever, vascular collapse, and disseminated intravascular coagulation. N.
gonorrhoeae does not cause a clinical syndrome as described in the question. Moraxella catarrhalis
causes pneumonia in patients with chronic obstructive pulmonary disease. Gram-positive
enterococcus (choice D) would refer to Group D streptococcus. These organisms are associated with
endocarditis and genitourinary infections. Herpesviruses (choice E) are large, enveloped DNA viruses
with an icosahedral shape. Possible diagnoses for this patient include Epstein-Barr virus, producing
infectious mononucleosis, or cytomegalovirus, producing a mononucleosis-like syndrome. The
patients in the question did not have hepatosplenomegaly (characteristic of mononucleosis), but did
have keratoconjunctivitis. The organism causing the outbreak in the question also has a higher level
of infectivity and a short incubation time.
117. A 70-year-old nursing home resident develops pneumonia. Examination of sputum
demonstrates many neutrophils and many lancet-shaped gram-positive cocci in pairs. Which
of the following organisms is the most likely cause of the patient's pneumonia?
A. Haemophilus influenzae
B. Mycobacterium tuberculosis
C. Mycoplasma pneumoniae
D. Staphylococcus aureus
E. Streptococcus pneumoniae
The correct answer is E. Pneumonia can be caused by a large variety of agents including bacteria,
Mycoplasma, fungi, and viruses. Streptococcus pneumoniae, or Pneumococci are classically
described as lancet-shaped gram-positive cocci in pairs, or diplococci. Historically, this organism was
specifically associated with lobar pneumonia, but with modern rapid antibiotic therapy, a
bronchopneumonia pattern may also be seen. Pneumococcal pneumonia remains common in
community and hospital-acquired settings. Complications include bacteremia, meningitis,
endocarditis, pericarditis, and empyema. Haemophilus influenzae(choice A) is a gram-negative rod,
more specifically a pleomorphic gram-negative coccobacilli. Mycobacterium tuberculosis(choice B) is
an acid-fast rod. Mycoplasma pneumoniae(choice C) is an intracellular organism and an atypical
bacteria. Gram stains typically show PMNs and monocytes but no bacteria. Staphylococcus
aureus(choice D) is a gram-positive coccus that grows in grape-like clusters. Complications include
empyema and endocarditis.

46

118. Bilateral tonsillectomy is performed on an 11-year-old female with recurrent upper


respiratory tract infections and trismos. On sectioning the tonsils, numerous small, yellow
granules are noted which have dense, gram-positive centers and numerous branching
filaments at the periphery. The granules are most likely composed of which of the following
organisms?
A. Actinomyces israelii
B. Aspergillus fumigatus
C. Blastomyces dermatitidis
D. Candida albicans
E. Corynebacteria diphtheriae
The correct answer is A. The patient is presenting with cervicofacial actinomycosis. This type of
infection commonly follows a tooth extraction or other trauma. Actinomyces are normal inhabitants of
the gastrointestinal tract that grow under anaerobic and microaerophilic conditions. Although they are
gram-positive rods, they grow as branching filaments and have been confused with fungi. The yellow
colonies ("sulfur granules") are found in low-oxygen niches like the tonsils and in actinomycotic
abscesses. Cervicofacial actinomycosis often develops slowly. The affected area becomes markedly
indurated and the overlying skin becomes reddish or cyanotic. Abscesses draining to the surface may
also present. There is little pain unless a secondary infection sets in. Trismus indicates that the
muscles of mastication are involved. Aspergillus fumigatus (choice B) may be present in the
respiratory tract as an opportunistic pathogen; however, fungus balls are generally seen only in preexisting cavities (e.g., bronchiectasis, TB), not in the tonsils. Blastomyces dermatitidis (choice C) is a
respiratory pathogen that is seen as thick-walled yeasts within granulomas. Candida albicans (choice
D), also a normal inhabitant of the oral cavity, would present as whitish plaques and would appear
microscopically as budding yeasts. When these white plaques are removed, they reveal a painful
bleeding lesion. Diphtheria, caused by Corynebacteria diphtheriae(choice E), is a gram-positive rod.
The disease presents with a gray-white membrane in the oropharynx, and large colonies would not be
noted.
119. A 4-year-old child has repeated infections with staphylococci and streptococci, but has
normal phagocytic function and delayed hypersensitivity responses. Lymph node biopsy
would most likely reveal
A. absence of postcapillary venules
B. absence of germinal centers
C. defective chemotactic response of neutrophils
D. depletion of paracortical areas
E. depletion of thymus-dependent areas
The correct answer is B. The combination of repeated staphylococcal infections, normal phagocytic
function, and normal delayed hypersensitivity responses is suggestive of Bruton
hypogammaglobulinemia. The disease is characterized by an IgG level less than 100 mg/dL, other
immunoglobulins deficient or absent, B cells deficient or absent, intact cellular (T-cell) immunity, and
onset of infections after the sixth month of life, when maternal antibodies are no longer present. Bcell-dependent areas, therefore, such as germinal centers, would be absent or greatly diminished in
size and number. Neither postcapillary venules (choice A) nor thymus-dependent areas (choice E)
are depleted in this disease. The chemotactic response of neutrophils (choice C) is part of the
phagocytic function assay, which was normal in the case presented. Because T-cell functions are
intact, the architecture of the portions of lymphoid tissues where T cells reside (e.g., paracortical
areas, choice D) should be normal.
120. A patient with tertiary syphillis has aortic regurgitation. Which of the following organisms
is the most likely culprit?
A. Haemophilus influenzae
B. Herpes simplex I
C. Neisseria gonorrhoeae
D. Neisseria meningitidis
E. Treponema pallidum
The correct answer is E. The findings described are those of tabes dorsalis, a form of tertiary
syphilis caused by Treponema pallidum. Tabes dorsalis and other forms of tertiary syphilis are now

47

uncommon in this country, possibly because the common use of antibiotics may "treat" many
unsuspected cases of syphilis. Late (tertiary) syphilis is associated with infiltrative tumors of skin,
bones, and liver (gummas). Aortic regurgitation and CNS disorders (including meningovascular and
degenerative changes), shooting pains, and abnormal reflexes are seen. Haemophilus influenzae
(choice A) and Neisseria meningitidis (choice D) can cause meningitis. Neisseria gonorrhoeae
(choice C) causes gonorrhea, which usually does not involve the CNS. Herpes simplex I (choice B)
can cause an encephalitis that typically involves the frontal and temporal lobes. Cardiac and liver
involvement, however, are not seen.
121. A 19-year-old man presents to the emergency department with pneumonia. Since the age
of 6 months, he has had recurrent pneumonia and sinusitis caused by Streptococcus
pneumoniae and Haemophilus influenzae. Careful assessment of his immune function would
likely reveal abnormal function of the
A. B-lymphocytes
B. macrophages
C. natural killer cells
D. platelets
E. T-lymphocytes
The correct answer is A. The symptoms in the question suggest a deficiency known as common
variable hypogammaglobulinemia, characterized by very low serum levels of IgG. In this congenital
disease, the number of B cells is normal, but their ability to synthesize IgG and the other
immunoglobulins is severely compromised, leading to recurrent streptococcal and Haemophilus
infections. The onset of the recurrent infections usually begins between 6 and 12 months of age,
concurrent with the decreasing levels of maternal IgG in the newborn. Treatment often involves giving
intravenous gamma-globulin to reduce the number of infections. There are no known diseases or
infections associated with deficiencies in macrophages (choice B) or natural killer (NK) cells (choice
C). Furthermore, one would not expect a deficiency in macrophage or NK cell function to present as
recurrent bacterial infections, because these cells are primarily involved in cellular, rather than
humoral, immunity. Platelets (choice D) are not directly involved in the immune response.
Deficiencies in platelets lead to problems in clotting and produce bleeding disorders. T-cell
deficiencies (choice E) result in severe viral, fungal, and protozoal infections, rather than recurrent
bacterial infections.
122. Which of the following viruses produces disease or sequelae that is/are more severe if
the infection occurs at a very young age?
A. Epstein-Barr virus
B. Hepatitis B virus
C. Measles virus
D. Poliovirus
E. Varicella zoster virus
The correct answer is B. Infection with hepatitis B virus (HBV) at birth or a very young age is
associated with chronic HBV infection and the development of hepatocellular carcinoma later in life. In
fact, infants born to hepatitis B surface antigen (HBsAg)-positive mothers are commonly infected, and
approximately 90% become chronic carriers of the virus. In chronic carriers, hepatocellular carcinoma
develops at an incidence more than 200 times higher than in noncarriers. The current
recommendation for infants born of HBsAg-positive mothers is administration of hepatitis B
immunoglobulin (HBIg) in the delivery room, with the first dose of the hepatitis B vaccine given at the
same time or within 1 week. The second and third doses of the vaccine are then given at 1 and 6
months. With this protocol, 94% protection is achieved. The Epstein-Barr virus (EBV; choice A) is the
agent of heterophile-positive infectious mononucleosis. In children, primary EBV infection is often
asymptomatic. The measles virus (choice C) often causes a more severe disease in adults. The
incidence of complications, including pneumonia, bacterial superinfection of the respiratory tract,
bronchopspasm, and hepatitis, is much higher in adults older than 20 than in children. Poliovirus
(choice D) causes asymptomatic or inapparent infections 95% of the time. Frank paralysis occurs in
approximately 0.1% of all poliovirus infections. However, the probability of paralysis increases with
increasing age. Varicella zoster virus (choice E) is the agent of chickenpox and shingles. In
immunocompetent children, it is a benign illness with a mortality rate of less than 2 per 100,000
cases. This risk is increased more than 15-fold in adults. Much of the increase is due to varicella
pneumonitis, a complication that occurs more frequently in adults.

48

123. A 22-year-old woman comes to the sexually transmitted disease (STD) clinic for her first
visit. She tells the nurse practitioner that she has had four different sexual partners in the last
6 months and only one of them used a condom. She also admits that she used IV drugs on
several occasions 2 years ago. She notes fever, weight loss, lack of appetite, and periodic
difficulty breathing over the past few months. She has an HIV test performed, which is
positive. The physician decides to do a
A. ELISA (enzyme-linked immunosorbent assay)
B. FACS (fluorescence activated cell sorting)
C. RAST (radioallergosorbent test)
D. RID (radial immunodiffusion)
E. Western blot
The correct answer is E.Because of the prevalence of AIDS, it is essential that the dental student be
familiar with these concepts. The Western blot is the most appropriate test for confirmation of HIV
infection. It identifies several different antibodies against HIV (anti-gp120, -gp41, -p24, and -p17). The
Western blot is generally performed when an ELISA test is positive. The initial HIV test this patient
had was most likely an ELISA. The ELISA (choice A) can be used to detect p24 antibody in the
patient, but is not as specific as the Western blot. The ELISA test is the standard test used in the
screening of HIV infections. Fluorescence activated cell sorting (choice B) is a technique used to
separate and count specific numbers and types of cells in a sample. An example of this would be to
count the number of B cells and T cells in a specific blood sample. RAST testing (choice C) is used to
determine the level of specific IgE present in a patient that reacts with a specific allergen that has
been applied to a disk or glass bead. Radial immunodiffusion (choice D) is an excellent test used for
quantitation of immunoglobulin levels in patients. This is used to determine the IgG, IgM, and IgA
levels in patient's serum.
124. A one-week-old baby develops nuchal rigidity and fever. A lumbar puncture is performed
and the cerebrospinal fluid demonstrates large numbers of neutrophils. The glucose level of
the CSF is lower than normal. If a diagnosis of meningitis is made, which of the following is
the most likely causative agent, taking into account the patient's age?
A. Coxsackievirus
B. Escherichia coli
C. Herpes virus
D. Mycobacterium tuberculosis
E. Neisseria meningitidis
The correct answer is B. The nuchal rigidity (stiff neck) suggests meningitis. Numerous neutrophils
in the cerebrospinal fluid suggest a bacterial pathogen. Furthermore, low CSF glucose is common in
bacterial meningitis. The best answer of those listed is Escherichia coli, which is normally a gut
organism, but can infect neonates who acquire the organism during passage through the birth canal.
The common characteristics of bacterial meningitis include: elevated polymorphonuclear neutrophils,
low CSF glucose, elevated CSF protein and markedly elevated opening pressure. Coxsackievirus
(choice A) is a cause of acute lymphocytic meningitis. Herpes virus (choice C) is a cause of
lymphocytic meningitis. Mycobacterium tuberculosis(choice D) is a cause of chronic meningitis and is
the causitive pathogen in tuberculosis. Neisseria meningitidis(choice E) causes bacterial meningitis,
usually in the second or third decade of life. It is rare to see this bacteria in infants.
125. Ciprofloxacin is believed to exert its antibacterial effect by which of the following
mechanisms?
A. Competitive inhibition of para-amino-benzoic acid
B. Inhibition of bacterial cell wall synthesis
C. Inhibition of DNA-gyrase
D. Irreversible binding to the 30S subunit of bacterial ribosomes
E. Irreversible binding to the 50S subunit of bacterial ribosomes
The correct answer is C. NBDE Part 1 questions often involve antibiotic mechanisms.
Fluoroquinolone antibiotics, such as ciprofloxacin and lomefloxacin, inhibit DNA-gyrase, which is an
enzyme necessary for bacterial DNA replication and repair. The fluoroquinolones are generally not
used for dental procedures because they have very poor anaerobic coverage. They have very good

49

gram-negative coverage, however. These agents are commonly used in the treatment of upper and
lower respiratory infections and in genitourinary tract infections. Sulfonamide antibiotics exert their
antibacterial effect through the competitive inhibition of para-amino-benzoic acid (PABA) (choice A),
thereby inhibiting folic acid biosynthesis required for bacterial growth. Cephalosporins (cephalexin,
cefuroxime, and cefixime) are believed to exert their antibacterial effect through the inhibition of
bacterial cell wall synthesis (choice B). These agents bind to one or more of the penicillin-binding
proteins located on the cell walls of susceptible organisms which results in the inhibition of the third
and final stage of bacterial cell wall synthesis. These effects account for the bactericidal effect of
cephalosporins and their cousins the penicillins. Aminoglycosides, such as gentamicin and
streptomycin, irreversibly bind the 30S subunit of bacterial ribosomes (choice D), which inhibits
bacterial protein synthesis. Lincosamides, such as clindamycin, irreversibly bind to the 50S subunit of
bacterial ribosomes (choice E), which suppresses bacterial protein synthesis. Note: macrolides, such
as erythromycin, also bind to the 50S subunit of bacterial ribosomes, which suppresses bacterial
protein synthesis.
126. If a dentist were to administer a bacteriostatic antibiotic, such as a macrolide, and a
bactericidal antibiotic, such as a cephalosporin, concomitantly to a patient with a severe
intraoral infection, the most likely pharmacological result for the patient would be which of the
following?
A. A small extension of bacterial coverage between the two agents
B. Antagonism of the antibacterial effects of both agents
C. Profound toxicity for the patients taking both agents
D. Synergism of the antibacterial effects of both agents
E. The appearance of mild antifungal activity
The correct answer is B. The concomitant administration of bacteriostatic and bactericidal antibiotics
would result in antagonism of the antibacterial effects of both agents. The reason for the antagonism
between the two is that each agent interferes with the mechanism of action of the other agent. By
definition, a bacteriostatic agent is one that prohibits the growth and development of bacteria and a
bactericidal agent is one that directly causes the death of a microbe during the growth and
development stages. An extension of bacterial coverage between two agents (choice A) is typically
seen when two bacteriostatic or two bactericidal agents are given together. The administration of a
macrolide and a cephalosporin may cause some discomfort for the patient, such as nausea or
stomach cramps; however, profound toxicity for the patients taking both agents (choice C) is very
unlikely. Synergism (choice D) is the correlated action of two agents that results in a final effect that is
greater than that of each agent acting separately. A classical example of synergism is seen when an
aminoglycoside, such as gentamicin, is administered concomitantly with extended spectrum penicillin,
such as piperacillin, for the treatment of a pseudomonas infection. The appearance of mild antifungal
activity (choice E) would not be seen when bacteriostatic and bactericidal antibiotics are given
together, if neither have any antifungal activity.
127. The term "biofilm" as it applies in dentistry is most often used to describe:
A. A safer type of x-ray film
B. a detector that protects tissue from x-ray damage
C. the surface of dentin inside a cavity preparation
D. the surface of dental waterlines
E. none of the above
The correct answer is choice D. The problem of bacteria in dental waterlines is probably going to be
the next intensive infection control issue faced by dentistry. The narrow bore of most dental waterlines
encourages a thin film colony of bacterial slime (biofilm) to form on the inner circumference of the
waterline. Water entering the dental unit, although chlorinated at the source, is clean, but not sterile.
The microorganisms it contains can form extensive colonies inside the tubes. While no specific
disease cases have been linked to these microorganisms, it is clear that they should be minimized.
Several systems of filtration or chemical disinfection are now available. Other alternatives include
bottled irrigation water and flushing the lines for several minutes several times each day.
128. After eating a dinner of leftovers that included rewarmed vegetable fried rice, a 17-yearold boy develops diarrhea and stomach pain. Which of the following is the most likely
pathogen?
A. Bacillus cereus
B. Campylobacter jejuni
C. Clostridium botulinum
D. Clostridium difficile
E. Escherichia coli

50

The correct answer is A. Bacillus cereus contaminates grains, such as rice, and produces spores
resistant to quick frying and steaming. If you see the words fried rice on the NBDE, the odds are that
the correct answer is B. cereus. Campylobacter jejuni (choice B) causes enterocolitis with bloody
diarrhea, crampy abdominal pain, malaise, and fever. Clostridium botulinum (choice C) produces a
constellation of signs and symptoms, including bulbar palsy, descending weakness or paralysis,
progressive respiratory weakness, absence of fever, dry mucous membranes, and autonomic
dysfunction. Clostridium difficile (choice D) causes pseudomembranous colitis, classically resulting
from clindamycin use. Escherichia coli (choice E) comes in a variety of forms. The enterotoxigenic
type is the most common cause of traveler's diarrhea.
129. A 7-year-old boy is brought to a physician because of a nearly confluent, fine,
erythematous, macular rash that is most pronounced on his trunk. He has had a mild fever for
36 hours, but does not seem very ill. Physical examination demonstrates a reddened throat
with tonsillar exudates, enlarged cervical nodes including the occipital node, and questionable
splenomegaly. The most likely cause of his condition is
A. Bullous pemphigoid
B. Dermatitis herpetiformis
C. Herpes simplex
D. Measles
E. Rubella
The correct answer is E. This presentation (fine, nonblotchy, truncal rash in a not-very-ill child) is
characteristic of rubella, or German measles, that is caused by a togavirus. The rash typically spreads
from trunk to extremities. IgM specific for rubella can often be detected in serum within 1-2 days of
developing the rash. The principal significance of this disease is that it can cause a devastating
congenital infection characterized by ocular problems (cataracts, retinopathy, microphthalmos,
glaucoma), cardiovascular problems (patent ductus arteriosus, ventricular septal defect, pulmonary
stenosis), deafness, thrombocytopenic purpura, leukopenia, hepatosplenomegaly, CNS problems,
and bony lesions. Arthralgia is seen, particularly in women. Exposure occurs 14-21 days before onset.
Bullous pemphigoid (choice A) produces large, tense blisters in flexural areas that appear with
exacerbations and remissions. Dermatitis herpetiformis (choice B) causes recurrent crops of small
vesicles or papules that appear mainly on the elbows, knees, buttocks, posterior neck, and scalp.
Herpes simplex (choice C) is characterized by crops of vesicles on oral or genital sites, where herpes
simplex I causes oral and perioral manifestations and herpes simplex II causes genital lesions.
Measles (choice D) causes a blotchy, maculopapular erythematous rash that begins on the face and
spreads downward. Patients with measles are usually much sicker than those with German measles,
and Koplik spots may be seen on the buccal mucosa.
130. A patient with diabetes has chronic sinusitis that has not responded to a 6-week course of
antibiotics. The physician should suspect infection with which of the following organisms?
A. Actinomyces
B. Aspergillus
C. Cryptococcus
D. Mucor
E. Pneumocystis
The correct answer is D. There is a specific association between diabetes (particularly in brittle
diabetics who may have episodes of ketoacidosis) and chronic sinusitis caused by saprophytic
Zygomycetes, including Mucor and Rhizopus. These fungi can spread rapidly from the sinuses to the
nearby skull bones and brain, potentially causing massive tissue destruction and death. The term
"rhinocerebral mucormycosis" is used in these cases. Less commonly, other sites may be involved
(lung, gastrointestinal tract), depending on the port of entry. The physician should suspect
mucormycosis in any patient with chronic sinusitis who appears unusually ill and does not respond to
antibiotic therapy. Unfortunately, most cases are diagnosed at autopsy. Actinomyces (choice A) are
part of the normal flora of the mouth. Actinomycosis also occurs in humans and may affect the
cervicofacial region (typically following dental procedures or maxillofacial injuries), lungs, abdomen
(typically following surgery, trauma, or intestinal penetration), or pelvis (related to IUD use). There is
no specific association with diabetes mellitus. Aspergillus (choice B) can be present in the sinuses,

51

and does have somewhat increased incidence in patients with diabetes, but it is not the organism
about which the physician should be most concerned. Cryptococcus (choice C) is found in pigeon
feces and is usually introduced into the body by way of the respiratory tract. It can disseminiate to the
meninges and other sites in immunocompromised patients (often AIDS patients). It would not be of
particular concern in this patient with sinusitis. Pneumocystis carinii (choice E) causes pneumonia in
severely immunosupressed patient ( e.g., patients with AIDS).
131. A 39-year-old HIV-positive man has a seizure accompanied by loss of consciousness and
leg and arm jerking. The patient is lethargic, unable to answer simple questions, and has an
obvious left-sided hemiparesis. The causative organism is most likely:
A. Cryptococcus neoformans
B. Herpes simplex
C. Mycobacteria tuberculosis
D. Isospora belli
E. Toxoplasma gondii
The correct answer is E. Although all five pathogens can cause central nervous system (CNS)
manifestations, toxoplasmosis (caused by Toxoplasma gondii) presents with seizures in 15-25% of
cases. It is the most common cause of secondary CNS infections in AIDS patients. The disease is
spread by ingestion of cysts from undercooked meat or from cat feces. Tachyzoites develop from
cysts phagocytized by macrophages, then spread to the brain, muscle, and other tissues, where they
encyst and multiply. Cryptococcal meningitis (choice A) usually presents as a subacute meningitis
with headache, nausea, vomiting, and confusion. Cranial nerve abnormalities are common with
cryptococcal infections. Herpes simplex encephalitis (choice B) typically has a subacute onset with
headache, meningismus, and personality changes. Mycobacterium tuberculosis(choice C) is a cause
of basilar meningitis, which can present insidiously with headache and mental changes over a week
or two, or can present acutely as confusion, lethargy, altered sensorium, and a stiff neck. Cranial
nerve palsies, focal cerebral ischemia, and hydrocephalus are characteristic. Isospora belli(choice D)
is a leading cause of diarrhea in AIDS patients. It typically occurs when the CD4 count is less than
300/uL.
132. An otherwise healthy patient who wears contact lenses develops a small ulceration of the
eye. Which of the following organisms is most likely involved?
A. Acanthamoeba
B. Cytomegalovirus
C. Herpes simplex
D. Toxocara
E. Toxoplasma
The correct answer is A. All the agents listed can infect the eyeball. The agent specifically
associated with contact lens use is Acanthamoeba, which can infect lens solution. This amoeba is
dangerous because it causes an intractable ulcerative keratitis that may progress to uveitis. If the
lesion is suspected, the clinical laboratory should be notified and specific directions for collecting
samples for culture obtained. The parasites may be difficult to see in histologic sections or corneal
scrapings. Cytomegalovirus (choice B) and herpes (choice C) infections are most often seen in
immunocompromised patients, particularly AIDS patients. Circulating larvae of the helminth
Toxocara(choice D) can lodge in the eye (particularly in the vitreous or retina); Toxocara infections
are seen more commonly in children. Toxoplasmosis (choice E) of the eye is most often congenital,
but it can be acquired.
133. A 4-year-old boy is brought to the emergency department in extreme respiratory distress,
with a temperature of 103.8 F. He is drooling, has difficulty swallowing, and exhibits
inspiratory stridor and swelling of the epiglottis. He has had no previous vaccinations. Which
of the following agents is the most likely cause of these symptoms?
A. Haemophilus influenzae
B. Klebsiella pneumoniae
C. Legionella pneumophila
D. Mycoplasma pneumoniae
E. Streptococcus pyogenes

52

The correct answer is A. Epiglottitis is the most common disease of the upper respiratory tract
produced by Haemophilus influenzae type b, a gram-negative encapsulated rod. Epiglottitis is
characterized by an abrupt onset of high fever, drooling, and inability to handle secretions. Stridor and
respiratory distress result from laryngeal obstruction. The epiglottis is described as being cherry red
and swollen. H. influenzae is also a common cause of otitis media in children and may cause
bronchitis, bronchiolitis, and pneumonia in adults. The incidence of serious disease caused by
Haemophilus influenzae type b decreased greatly with the introduction of an effective vaccine, which
is composed of the H. influenzae type b capsular polysaccharides coupled to a carrier molecule and
given to children from 2-15 months of age. The patient had not received the Hib conjugate vaccine
and therefore was susceptible to this organism. Klebsiella pneumoniae(choice B) causes pneumonia
and pulmonary abscesses, but is not considered to be a pathogen in the upper respiratory tract.
Klebsiella is a gram-negative encapsulated rod. Legionella pneumophila(choice C) causes
pneumonia in humans. The disease may be mild (an atypical pneumonia) or a fulminating disease
with high mortality (30%). On a gram stain, one would see no bacteria because it is an "atypical"
organism. Mycoplasma pneumoniae(choice D) causes community-acquired atypical pneumonia. It is
the most common cause of pneumonia in young adults. On a gram stain, one would see no bacteria
because it is an "atypical" organism. Streptococcus pyogenes(choice E) is the most common cause
of pharyngitis; however, this patient's presentation strongly suggests epiglottitis. It is a gram-positive
organism.
134. Which of the following is an example of type II hypersensitivity?
A. Allergic rhinitis
B. Erythroblastosis fetalis
C. Food allergy
D. Serum sickness
E. Tuberculosis
The correct answer is B. Type II hypersensitivity occurs when antibodies react with antigens present
on the surface of cells or other tissue components. It is conveniently subclassified into diseases
produced by three distinct mechanisms. One mechanism involves complement fixation by the
antibody-antigen complex, which facilitates phagocytosis; this mechanism occurs in transfusion
reactions, erythroblastosis fetalis, autoimmune hemolytic anemia, agranulocytosis, or
thrombocytopenia. In a second form of type II hypersensitivity, target cells coated with low levels of
IgG are lysed (without phagocytosis) by monocytes, neutrophils, or natural killer cells; this mechanism
is thought to operate in the destruction of large parasites, possibly some tumor cells, and in graft
rejection. The third mechanism involves antibody-mediated cellular dysfunction, such as occurs in
Graves disease or myasthenia gravis. Allergic rhinitis (choice A) is an example of type I (allergic or
anaphylactic) hypersensitivity. Food allergy (choice C) is an example of type I (anaphylactic)
hypersensitivity. Serum sickness (choice D) is an example of type III (immune complex diseases)
hypersensitivity. Tuberculosis (choice E) is an example of type IV (cell-mediated or delayed)
hypersensitivity.
135. A gram stain of the sputum from a patient with lobar pneumonia involving the left lower
lobe demonstrates gram-positive, encapsulated, lancet-shaped diplococci. Which of the
following is the most probable causative organism?
A. Haemophilus influenzae
B. Neisseria gonorrhoeae
C. Pneumocystic carinii
D. Staphylococcus aureus
E. Streptococcus pneumoniae
The correct answer is E. This is the classic microscopic description of the pneumococcus
Streptococcus pneumoniae, which is a common cause of lobar pneumonia. Most strains are still
sensitive to penicillins, although some drug-resistant strains have been isolated. Haemophilus
influenzae (choice A) is a gram-negative bacillus. Atypical strains may cause pneumonia in elderly
patients with chronic respiratory disease. Neisseria gonorrhoeae (choice B) is a gram-negative
diplococcus that is not typically associated with pneumonia. Pneumocystis carinii (choice C) is a
small, hat-shaped fungus that is a common cause of pneumonia in HIV-positive
patients.Staphylococcus aureus (choice D) occurs as grapelike clusters of large, gram-positive cocci.
It may cause pneumonia after surgery or after a viral respiratory infection, such as influenza, and is
associated with empyema formation.

53

136. Global eradication of Lyme disease is unlikely in the foreseeable future because Borrelia
burgdorferi
A. can be maintained in nature indefinitely by a tick vector
B. has a tough outer coat that is resistant to environmental stresses
C. has humans as its primary reservoir
D. is resistant to antibiotics and disinfectants
E. may reactivate and cause Brill-Zinsser disease
The correct answer is A.Borrelia burgdorferi, the tick-transmitted spirochete that causes Lyme
disease, can be maintained in nature indefinitely by a tick vector. The tick, Ixodes dammini, can infect
white-footed mice and large mammals such as deer during its life cycle, making these animals into
reservoirs. The tick itself, however, is a reservoir because it acquires the disease through transovarial
passage of the organism. Together these factors make Lyme disease an endemic infection with little
hope for eradication. B. burgdorferi is a delicate spirochete that is vulnerable to several chemical and
physical agents. It does not have a tough outer coat (unlike choice B). Humans are incidental hosts,
not primary reservoirs (choice C), for B. burgdorferi. The primary reservoirs are ticks, mice, and large
mammals. B. burgdorferi can be successfully treated with penicillins, tetracycline, and ceftriaxone
(unlike choice D). Brill-Zinsser disease (choice E) is actually the reactivation of epidemic typhus
infection caused by Rickettsia prowazekii. It can occur many years after an infection that was not
treated with antibiotics.
137. A patient experiences a severe bite wound from a pit bull. He receives a booster injection
of tetanus toxoid and an injection of penicillin G. Several days later, the wound is inflamed and
purulent. The exudate is cultured on blood agar and yields gram-negative rods. Antibiotic
sensitivity tests are pending. Which of the following is the most likely pathogen?
A. Bartonella henselae
B. Brucella canis
C. Clostridium tetani
D. Pasteurella multocida
E. Toxocara canis
The correct answer is D. Pasteurella multocida is a gram-negative rod that is normal flora of the oral
cavity of dogs and cats. It often causes a local abscess following introduction under the skin by an
animal bite. Most cases occur in children who are injured while playing with a pet. Note that penicillin
G is not highly effective against gram negative rods. Bartonella henselae (choice A)is a very small
gram-negative bacterium that is closely related to the rickettsia. It is the cause of cat-scratch disease
(a local, chronic lymphadenitis most commonly seen in children) and bacillary angiomatosis (seen
particularly in AIDS patients). In this latter patient population, the organism causes proliferation of
blood and lymphatic vessels, causing a characteristic "mulberry" lesion in the skin and subcutaneous
tissues of the afflicted individual. Brucella canis(choice B) is a gram-negative rod that is a zoonotic
agent. Its normal host is the dog, but when it gains access to humans, it causes an undulating febrile
disease with malaise, lymphadenopathy, and hepatosplenomegaly. The normal route of exposure is
by way of ingestion of the organism. Clostridium tetani (choice C) is a gram-positive spore-forming
anaerobic rod. It causes tetanus (a spastic paralysis caused by tetanospasmin, which blocks the
release of the inhibitory neurotransmitters glycine and gamma-aminobutyric acid [GABA]). There may
be no lesion at the site of innoculation and exudation would be extremely rare. Toxocara canis
(choice E), a common intestinal parasite of dogs, is a metazoan parasite that causes visceral larva
migrans. Young children are most likely to be affected, as they are most likely to ingest soil
contaminated with eggs of the parasite.
138. A patient with nuchal rigidity and headache undergoes lumbar puncture. The CSF
contains markedly increased numbers of lymphocytes, leading to a presumptive diagnosis of
viral meningitis. Which of the following groups of viruses is most likely to be involved?
A. Adenoviruses
B. Enteroviruses
C. Human papillomaviruses
D. Poxviruses
E. Reoviruses

54

The correct answer is B. Viral meningitis is relatively common, accounting for 10,000 cases of
meningitis per year in the United States. The vast majority of cases occur in individuals younger than
30 years of age. Usually the symptoms are relatively mild and death is uncommon. Enteroviruses,
arboviruses, and type 2 herpes simplex virus are the most common causes of viral meningitis. Also,
up to 10% of HIV patients develop an acute meningitis, typically at the time of seroconversion.
Adenovirus (choice A) infection is associated with upper respiratory tract infections (URIs), sinusitis,
ocular disease, enteric infections, and bladder infections. It does not typically cause aseptic
meningitis. Human papillomaviruses (choice C) are associated with warts on the skin and genital
areas. Poxviruses (choice D) include the causative agents of smallpox, cowpox, and molluscum
contagiosum. Reoviruses include the most common family member, rotavirus, responsible for
gastroenteritis in young children. These agents do not typically cause meningitis.
139. A patient who is unable to tolerate clindamycin therapy for the treatment of an intraoral
anaerobic infection would be most likely prescribed which of the following agents?
A. Ciprofloxacin
B. Metronidazole
C. Tetracycline
D. Tobramycin
E. Vancomycin
The correct answer is B. Anaerobic bacteria exist as part of the normal flora on the mucosal
surfaces (the mouth, gastrointestinal tract, skin, and female genital tract) of humans and animals.
Anaerobic infections occur when the normal balance between aerobic and anaerobic bacteria is
disrupted.The two medications primarily used for the treatment of anaerobic infections are
clindamycin and metronidazole. Clindamycin is typically used in the treatment of serious anaerobic
infections when other less toxic medications cannot be used. A potential serious side effect is the
onset of pseudomembranous colitis. Metronidazole is a bacterial agent that is indicated for the
treatment of serious infections caused by anaerobic bacteria and is active against most protozoa. It is
primarily used for the treatment of gynecologic and abdominal infections and can be used for the
treatment of antibiotic-induced pseudomembranous colitis. The fluoroquinolones, such as
ciprofloxacin (choice A), have very poor anaerobic coverage. Ciprofloxacin therefore would be
relatively ineffective for the treatment of an oral infection caused by an anaerobic infection. These
agents are commonly used in the treatment of upper and lower respiratory infections and in
genitourinary tract infections. Tetracycline (choice C) is primarily used in the treatment of chlamydial
and mycoplasmal infections, gonorrhea, and acute exacerbations of chronic bronchitis. Tobramycin
(choice D) is an intravenous antibiotic typically reserved for the treatment of serious and lifethreatening gram-negative bacterial infections and some gram-positive infections. Vancomycin
(choice E) is also typically reserved for the treatment of serious and life-threatening bacterial
infections; however, it is used primarily in the treatment of gram-positive infections.
140. Serum analysis of a symptomatic patient yields elevated ALT, HBsAg, Anti-HBc, HBeAg,
and bilirubin. All other values are normal. What is the hepatitus B status of this recruit?
A. Asymptomatic carrier
B. Chronic active carrier
C. Fulminant hepatitis B
D. Recovered from acute self-limited HBV
E. Vaccinated against HBV
The correct choice is B. The presence of elevated ALT, HBsAg, anti-HBc, HBeAg, and bilirubin all
point to active hepatitis B. An asymptomatic carrier (choice A) would not have elevated ALT and
bilirubin. The absence of other symptoms rules out fulminant hepatitis B (choice C). Recovery from
acute self-limited HBV (choice D) is associated with the presence of anti-HBs and a decrease in
HBsAg and HBeAg. Someone who is vaccinated with HBV (choice E) has only anti-HBs in serum.
141. A 23-year-old woman has Toxic Shock Syndrome. A diffuse erythematous rash with areas
of desquamation over the hands and feet is noted. Infection with which of the following agents
is the most likely cause of these signs and symptoms?
A. Clostridium perfringens
B. HIV-1
C. Shigella dysenteriae
D. Staphylococcus aureus
E. Staphylococcus epidermidis

55

The correct answer is D. Toxic Shock Syndrome (TSS), a multisystem syndrome caused by a toxin
(TSST-1) formed by certain strains of S. aureus. TSS usually affects several organ systems
(gastrointestinal, renal, hepatic, hematopoietic, musculoskeletal, pulmonary) and can result in death.
TSS historically has been associated with the use of tampons (as well as cervical cups and
diaphragms) in young women, but also can occur in other patient populations. Symptoms often have a
very abrupt onset, including fever, hypotension, diarrhea, and diffuse rash with desquamation of the
hands and feet. Blood cultures are negative, because sypmtoms are caused by the toxin not the
invasive organism. Management of shock, renal failure, and adult respiratory distress syndrome
(ARDS) are a priority if these conditions are present, in addition to appropriate antibacterial treatment.
C. perfringens (choice A) causes gas gangrene, with necrosis of soft tissues, usually after a traumatic
wound. It is also a cause of food poisoning. Although HIV-1 (choice B) can cause many diverse
findings and should never be immediately ruled out, the findings in this patient are most specific for
TSS. Shigella dysenteriae(choice C) is a cause of dysentery characterized by fever, abdominal
cramps, and bloody diarrhea. S. epidermidis (choice E) is part of the normal skin flora, but is
notorious for causing infections of intravenous lines and prosthetic heart valves.
142. A patient was recently prescribed fludrocortisone inhalation therapy to improve his
current asthma treatment regimen. Ten days after starting therapy he presents with white
patches on the inside of the cheeks that can be easily wiped off leaving a red, bleeding, sore
surface. This patient would be best treated with which of the following?
A. Acyclovir Capsules
B. Ampicillin Capsules
C. Cephalexin Suspension
D. Clarithromycin Tablets
E. Clotrimazole Troches
The correct answer is E. Candidiasis can appear in any area of the oral mucosa. One of the most
common causes of candidiasis in the adult population is the use of oral inhalation corticosteroids,
such as fludrocortisone. The pseudomembranous form typically appears with white patches that can
be easily wiped off leaving red, bleeding, sore surfaces. There is also an erythematous form that
presents with flat red and white lesions that cannot be rubbed off. Based on the description of the
signs and symptoms in this question, the patient has the pseudomembranous form of oral candidiasis
or oral thrush. Oral candidiasis responds very well to antifungal therapy. Clotrimazole and nystatin are
antifungal agents used locally for treatment of infections caused by many different Candida species.
Clotrimazole Troches are the generic name for the brand name product Mycelex Troches. A troche is
essentially an orally disintegrating tablet that dissolves in the mouth and is generally used in adults
with candidiasis. Acyclovir (choice A) is an antiviral agent used in the treatment of infections caused
by herpes simplex virus types 1 and 2 and varicella-zoster virus. This agent would be indicated in
individuals with herpes zoster infections that typically appear as vesicular eruptions and/or ulcers on
the cheek, tongue, gingival or palate. Both ampicillin (choice B) and cephalexin (choice C) are betalactam antibiotics used to treat a number of bacterial infections. Both agents would be ineffective in
fungi. Clarithromycin (choice D) is a macrolide antibiotic that may be safely and effectively used for
the treatment of bacterial pharyngitis, as well as various other infections caused by susceptible
organisms. It is also used in SBE prophylaxis for dental procedures.
143. A patient is suffering from pneumonia, and culture shows numerous gram-positive cocci
that are identified as Streptococcus pneumoniae. Which of the following immune effector
mechanisms is most important in completely clearing this infection?
A. ADCC (antibody-dependent cell cytotoxicity)
B. Complement-mediated opsonization
C. Cytotoxic T cell lymphocytes
D. LAK cells
E. Natural killer cells
The correct answer is B. One of the most efficient mechanisms for eliminating extracellular
pathogenic bacteria is by opsonization and phagocytosis by macrophages. The IgG and IgM antibody
produced in response to the organism reacts with the capsular structure, stimulating the activation of
the classic pathway of the complement system. This pathway produces large amounts of C3b that

56

coat the organism, preparing it for phagocytosis. ADCC cells (choice A) are actually natural killer
(NK) cells that find virally infected cells and tumor cells that have been coated with antibody and react
with and destroy them. These cells do not destroy antibody-coated bacteria, only body cells that are
coated with antibody. Cytotoxic T lymphocytes (choice C) react only with cells that have antigen
epitopes presented in association with class I MHC molecules. An example would be a virus epitope
from a virally infected cell presented by class I molecules on the surface of the cell. LAK cells (choice
D) are NK cells that have been activated by IL-2. They are considered to be superactivated NK cells.
NK cells (choice E) are cells of the innate immune system that destroy virally infected cells or tumor
cells. This does not involve antibody and it does not involve extracellular pathogens.
144. A 33-year-old man with AIDS and a history of shingles develops a severe, multifocal
encephalitis. Which of the following viruses is the most likely cause of his encephalitis?
A. Cytomegalovirus
B. Herpes simplex type I
C. Herpes simplex type II
D. Herpes zoster-varicella
E. Measles virus
The correct answer is D. The specific clue to the cause of the severe encephalitis in this patient is
the history of shingles, caused by reactivation of the herpes zoster-varicella virus. In otherwise healthy
adults, the virus (which is usually introduced to the body as a childhood case of chickenpox) remains
dormant in a dorsal root ganglion, only to reactivate in later life as a painful dermatomal vesicular
eruption. In patients with AIDS, the virus can cause a severe, multifocal encephalitis that may be
resistant to acyclovir therapy. Cytomegalovirus (choice A), Herpes simplex type I (choice B) and
herpes simplex type II (choice C), can cause disseminated disease (including brain infection) in AIDS
patients, but is less likely in this patient, given the past history of shingles and the lack of response to
acyclovir. Measles virus (choice E) appears to be related to subacute sclerosing panencephalitis, but
this condition is not specifically increased in AIDS patients.
145. A patient with a lung lesion coughs up sputum that contains thin, acid-fast positive rods.
Which of the following features would most likely be associated with these bacteria?
A. Nutritional requirement for factors V and X
B. Streptokinase
C. Toxic shock syndrome toxin
D. Visible under dark field illumination
E. Waxy envelope
The correct answer is E. M. tuberculosis is a non-spore-forming thin aerobic bacteria. Once stained
the bacilli are not discolorized by acid alcohol; hence the term acid-fast. Mycobacteria are also
considered "acid fast" because they have an envelope that contains large amounts of lipids and even
true waxes (unlike envelopes of other types of bacteria) that prevents the acid-fast stain
(carbolfuchsin) from leaking out. The other characteristics listed in the answers are commonly tested
features of specific bacteria: Nutritional requirement for factors V and X (choice A) is a feature of
Haemophilus influenzae. Streptokinase (choice B) is produced by Streptococci. Toxic shock
syndrome toxin (choice C) is a feature of Staphylococcus aureus. Visibility under dark field
illumination (choice D) is a feature of the syphilis organism Treponema pallidum. Furthermore, T.
pallidum does not grow on artificial media and therefore cannot be cultured in a laboratory.
146. A 58-year-old alcoholic with multiple dental caries develops a pulmonary abscess and is
treated with antibiotics. Several days later he develops nausea, vomiting, abdominal pain, and
voluminous green diarrhea. Which of the following antibiotics is most likely responsible for
this patient's symptoms?
A. Chloramphenicol
B. Clindamycin
C. Gentamicin
D. Metronidazole
E. Vancomycin
The correct answer is B. Any time you see the development of diarrhea in the same question stem
with "treated with antibiotics," you should immediately think of pseudomembranous colitis. This

57

condition is caused by Clostridium difficile and typically occurs as a result of treatment with
clindamycin or ampicillin. You would confirm your suspicion by sending a stool culture to be tested for
the presence of the C. difficile toxin. The most test-worthy side effect of chloramphenicol (choice A) is
aplastic anemia, not diarrhea. In addition, you might have been able to eliminate this choice simply
because of the extremely low probability that this patient would receive this antibiotic in the United
States. The key side effects of gentamicin (choice C) include ototoxicity and nephrotoxicity.
Metronidazole (choice D) and vancomycin (choice E) do not cause pseudomembranous colitis; they
are used to treat it.
147. A 37-year-old man is admitted to the hospital with shortness of breath, cyanosis, and
fever. Chest x-rays reveal consolidation of the right lower lobe with relative sparing of the
remaining lobes. A clinical diagnosis of lobar pneumonia is made and supported by the results
of sputum cultures. Which of the following is the genus of the bacterium most likely to be
isolated from this patient's sputum?
A. Haemophilus
B. Klebsiella
C. Streptococcus, alpha-hemolytic
D. Streptococcus, beta-hemolytic
E. Streptococcus, gamma-hemolytic
The correct answer is C. The clinical and radiologic characteristics of this patient's condition are
consistent with lobar pneumonia, a respiratory infection that, in its classic presentation, involves a
single pulmonary lobe. In 90-95% of cases, the etiologic agent is Streptococcus pneumoniae (AKA
pneumococcus), an alpha-hemolytic streptococcus present in the throat of 40-70% of healthy
individuals. This organism is best described as a gram-positive diplococcus. Penicillin is the drug of
choice for pneumococcal pneumonia, but sputum cultures are necessary to identify the infectious
agent and determine its antibiotic sensitivity.S. pneumoniae is a major cause of purulent meningitis in
the elderly. Other alpha-hemolytic streptococci (viridans streptococci) cause subacute endocarditis in
patients with previously altered cardiac valves. Bacteria belonging to the genera Haemophilus(choice
A) and Klebsiella (choice B) cause respiratory infections; however, in immunocompetent individuals,
these bacterial infections usually result in bronchopneumonia, which leads to multilobar, and often
bilateral, pulmonary involvement. Beta-hemolytic streptococci (choice D) cause countless infections
in humans. Recall that classification of beta-hemolytic streptococci is based on their surface antigens
known as Lancefield antigens. Human diseases caused by this group of bacteria include: Streptococcus pyogenes (group A): pharyngitis, scarlet fever, erysipelas, impetigo, rheumatic fever,
and glomerulonephritis. - Streptococcus agalactiae (group B): neonatal sepsis and urinary infections.
- Enterococcus faecalis (group D): endocarditis and urinary infections. Respiratory involvement is
rarely seen. Gamma-hemolytic streptococci (choice E) are streptococci that do not produce
hemolysins (nonhemolytic streptococci) and are not a significant cause of human disease.
148. Which of the following organisms would most likely be isolated from shunt tubing or
indwelling catheters in hospitalized children who develop meningitis?
A. Bacteroides fragilis
B. Corynebacterium diphtheriae
C. Escherichia coli
D. Staphylococcus epidermidis
E. Streptococcus pneumoniae
The correct answer is D. Staphylococcal meningitis is fairly rare, occurring mostly in patients with
indwelling ventricular-peritoneal shunts. Staphylococcus epidermidis, a coagulase-negative organism
that normally colonizes the skin, is the most common organism causing this disorder; Staphylococcus
aureus meningitis occasionally occurs. Bacteroides fragilis(choice A) is a common cause of
anaerobic infections, including sepsis and peritonitis, but it does not commonly cause meningitis.
Diphtheroids (including Corynebacterium diphtheriae, choice B) are sometimes isolated from
indwelling shunts, but in many cases they are simply contaminants, because some diphtheroids are
normal skin flora. Escherichia coli(choice C) is isolated from 30-50% of neonates with bacterial
meningitis, but is not typically associated with shunt infections. Streptococcus pneumoniae(choice E)
is the most common cause of bacterial meningitis in people older than 30 years of age; it is much less
common in children.

58

149. A 25-year-old woman presents with an erythematous rash that began on her face, then
spread downward over her trunk. She has a fever and headache, bilateral pain of the front and
back of her neck, and arthralgia. Which of the following diseases does she most likely have?
A. Infectious mononucleosis
B. Lyme disease
C. Roseola
D. Rubella
E. Rubeola
The correct answer is D. Rubella, or German measles, is a disease caused by a Togavirus, which is
a small, enveloped, single-stranded, (+) linear RNA virus. Approximately 40% of patients are
asymptomatic or have mild symptoms. In symptomatic patients, the clinical presentation typically
consists of an erythematous rash beginning on the head that spreads downward to involve the trunk,
lasting for approximately 3 days. In addition to the rash, symptoms include fever, malaise, and
arthralgias coinciding with the eruption. Leukopenia also may be seen. Enteroviral rashes may mimic
rubella and rubeola. Infectious mononucleosis (choice A) is caused by the Epstein-Barr virus, a
herpesvirus. Classic findings include fever, exudative pharyngitis, generalized lymphadenopathy,
severe malaise (most common complaint), and hepatosplenomegaly. A rash is not a characteristic
feature unless the patient has been treated with ampicillin. This is a self-limited disease that is rarely
fatal unless splenic rupture occurs. Lyme disease (choice B) is caused by the spirochete Borrelia
burgdorferi. The disease is transmitted by the bite of the tick, Ixodes dammini. Reservoirs in nature
include deer and mice. The initial lesion is an annular rash with central clearing and a raised red
border (erythema chronicum migrans) at the bite site. The rash is warm, but not painful or itchy.
Patients also have fever, malaise, myalgias, arthralgias, headache, generalized lymphadenopathy,
and, occasionally, neurologic findings. Roseola (choice C) is caused by human herpesvirus 6. Other
names include exanthem subitum or sixth disease. Children have a febrile period of 3-5 days with
rapid defervescence followed by an erythematous maculopapular rash lasting 1-3 days. Rubeola
(choice E), or measles, is caused by a paramyxovirus. Patients present with an upper respiratory
prodrome and characteristic oral lesions (Koplik spots) that precede the rash. The nonpruritic
maculopapular rash begins on the face and spreads to the trunk and extremities, including palms and
soles. The incubation period is 10-14 days. Patients also have a posterior cervical lymphadenopathy.
The virus is not associated with risk to a fetus.
150. A 60-year-old alcoholic smoker abruptly develops high fever, shakes, a severe headache,
muscle pain, and a dry, insignificant cough. Later he develops marked shortness of breath
requiring assisted ventilation. Chest x-ray reveals left lower lobe shadowing that spreads until
both lungs are extensively involved. Culture of bronchoalveolar lavage fluid demonstrates a
coccobacillary pathogen. Which is the most likely causative organism?
A. Legionella pneumophila
B. Listeria monocytogenes
C. Pseudomonas aeruginosa
D. Staphylococcus aureus
E. Streptococcus pneumoniae
The correct answer is A. The patient has a severe, potentially fatal, pneumonia with prominent
systemic symptoms. Culture revealing a coccobacillus is the specific clue that the organism is
Legionella pneumophila. The disease is respiratory Legionellosis, also known as Legionnaire disease.
Patients tend to be older (40-70 years of age) and may have risk factors including cigarette use,
alcoholism, diabetes, chronic illness, or immunosuppressive therapy.
The bacteria can also be seen in the sputum with direct fluorescent antibody staining. Listeria
monocytogenes(choice B) causes listeriosis and is not a notable cause of pneumonia. Pseudomonas
aeruginosa (choice C) is a gram-negative rod that is a cause of pneumonia in patients on ventilators
and those with cystic fibrosis.
Staphylococcus aureus(choice D) can cause pneumonia, but is a plump gram-positive cocci in
clumps. Streptococcus pneumoniae(choice E) can cause pneumonia, but is a gram-positive
diplococci.

59

151. The electron transport system of Neisseria is located on which of the following
structures?
A. Cytoplasmic membrane
B. Mesosome
C. Mitochondria
D. Nuclear membrane
E. Polyribosome DNA aggregates
The correct answer is A. This is a good microbiology question that uses the principles of bacterial
physiology. Unlike eukaryotic cells in which the electron transport system is located on mitochondria,
the electron transport system of bacteria is located on the cytoplasmic (plasma) membrane.
Mesosomes (choice B) and polyribosome DNA aggregates (choice E) are also found in bacterial
cells and function in cell division and protein synthesis, respectively. Bacteria do not have
mitochondria (choice C) or membrane-bounded nuclei (choice D).
152. A 32-year-old man presents to the emergency room with a severe headache. Nuchal
rigidity is found on physical examination. Lumbar puncture demonstrates cerebrospinal fluid
with markedly increased lymphocytes. Other cell populations are not increased. If the fluid is
clear in appearance, which of the following agents is the most likely cause of his symptoms?
A. Escherichia coli
B. Haemophilus influenzae
C. Herpes virus
D. Mycobacterium tuberculosis
E. Treponema pallidum
The correct answer is C. The clinically suspected diagnosis is meningitis, which is confirmed by the
abnormal cerebrospinal fluid. The markedly increased lymphocytes suggests acute lymphocytic
meningitis, which is distinguished from acute pyogenic meningitis (increased neutrophils as well as
lymphocytes). Acute lymphocytic meningitis is usually viral in origin. Among the many viruses that
have been implicated, mumps, herpes, Epstein-Barr, echovirus, and Coxsackie virus are the most
common. One of the easiest ways to differentiate between a viral and bacterial meningitis is the
appearance of the cerebral spinal fluid. A viral meningitis will not generally change the color of the
CSF. However, a bacterial meningitis often produces a cloudy CFS. Escherichia coli(choice A) and
Haemophilus influenzae(choice B) cause acute pyogenic meningitis and would be expected to
produce a cloudy cerebral spinal fluid. Mycobacterium tuberculosis(choice D) and Treponema
pallidum(choice E) cause chronic meningitis and would be expected to produce a cloudy fluid.
153. Which of the following diseases is associated with the "Dane Particle"?
A. Hepatitis A
B. Hepatitis B
C. Hepatitis C
D. Hepatitis D
E. Hepatitis E
The correct answer is choice B. A Dane particle is the name for the entire Hepatitis B virion (viral
particle). It is one of three entities that will test positive as Hepatitis B surface antigens. The other two
are pieces of viral coats known as spheres or filaments, which, although they are not complete viral
particles, they do contain the surface antigen (HBsAg). Other hepatitis B antigens include the core
antigen (HBcAg) and an "e-antigen" (HBeAg). The "e-antigen" is associated with increased viral
replication and infectivity of the patient.

_____________

60

Dental Anatomy & Histology


1. Which type of dentin is most regular in structure?
A. Primary dentin
B. Secondary dentin
C. Reparative dentin
D. All have the same regular structure
The correct answer is A. All dentin is formed by odontoblasts located at the border of the pulp and
dentin. It consists of tubules surrounding odontoblastic processes, as well as intertubular dentin. In
primary dentin, the tubules are regular in pattern. In secondary dentin, formed later in life, tubules are
less regular, less numerous and more wavy. Reparative dentin (sclerotic dentin) forms in response to
caries, heat, deep fillings, etc. It is least regular and most wavy, and tubules are least numerous.
2. Which cusp becomes smaller and less conspicuous as you go from maxillary first to second
to third molar?
A. Mesiobuccal
B. Mesiolingual
C. Distobuccal
D. Distolingual
The correct answer is D. The MB, ML, and DB cusps of the maxillary molars are the largest, and
form the primary cusp triangle, or trigon, of the tooth. The distolingual cusp is the smallest, and is not
part of the trigon. It is sometimes referred to as the talon or talon cusp. It is most noticeable in the first
molar, less noticeable in the second, and often absent in the third molar. The maxillary third molar
therefore often does not have the rhomboid appearance of a maxillary molar, and the occlusal aspect
will appear triangular or heart-shaped instead.
3. The masseteric sling is composed of the masseter and the:
A. medial pterygoid
B. lateral pterygoid
C. anterior digastric
D. temporalis
The correct answer is A. The masseteric sling is a powerful pair of muscles which wrap underneath
the angle and ramus of the mandible and act to both support and close (elevate) the mandible. On the
lateral surface of the mandible we find the masseter, while on the medial surface we find the medial
pterygoid. Although the temporalis (choice D) also strongly elevates, it is not part of this structure. The
lateral pterygoid (choice B) attaches to the condyle and TMJ disc, and pulls the condyles forward and
laterally. The lateral pterygoid is not part of the masseteric sling.
4. Which cusp on the primary mandibular first molar is highest and sharpest?
A. Mesiobuccal
B. Distobuccal
C. Mesiolingual
D. Distolingual
The correct answer is C. The primary first mandibular molar is a highly unusual tooth. It is molar-like
in form, with two roots (mesial and distal), but has a very pronounced mesial development of the
crown. The mesial section comprises two-thirds of the crown, and the distal portion is much reduced.
The mesiolingual cusp is the highest and sharpest, although the mesiobuccal is overall largest as
viewed from the occlusal. It is followed in size by the ML, DB, and DL cusps.
5. Which of the following is NOT considered an important reason for restoring proximal
contact in dental restorations?
A. Contact protects the periodontal tissue
B. Contact prevents food impaction
C. Contact increases retention of the restorations
D. Contact prevents tooth drifting
The correct answer is C. Contact is re-established in proximal areas when Class II restorations are
completed. This restoration of contact at the correct contact level prevents food from wedging in

between teeth and causing buildup of plaque and debris. This, in turn, protects the periodontal
ligament, because the chief etiology of periodontal bone loss is calculus and plaque accumulation.
Stability of the arch is maintained and tooth drifting is also minimized. Without contact, teeth may drift
mesially or distally into the space left by the faulty restoration. Proximal contact is NOT considered to
be related to filling material retention. This is because each restoration's retention is supposed to
depend solely on the retentive properties of that individual tooth, tooth preparation, and material. In
other words, retention stands alone within the individual tooth. Adjacent teeth are never considered as
part of restoration retention.
6. The anterior tooth with the most pronounced lingual ridge is the:
A. Maxillary central incisor
B. Mandibular lateral incisor
C. Maxillary canine
D. Mandibular canine
The correct answer is C. A lingual ridge runs from the incisal edge down the center of the lingual
surface of the crown, toward the cervical end of the crown. It splits the lingual surface in half, and
usually has a depression on either side (lingual fossae) which separate the lingual ridge from the
mesial and distal marginal ridges. Lingual ridges are found on canines, but not on incisors (choices A
and B). The lingual ridge is more pronounced on the maxillary canine than on the mandibular (choice
D). This is in keeping with a general rule that the lingual anatomy of the maxillary canine is much
more pronounced than that of the mandibular.
7. In centric occlusion, the mesiolingual cusp of the maxillary third molar will contact:
A. the central fossa of the mandibular second molar
B. the central fossa of the mandibular third molar
C. the distal marginal ridge of mandibular first molar and mesial marginal ridge of the second molar
D. the distal marginal ridge of mandibular second molar and mesial marginal ridge of the third molar
The correct answer is B. The mesiolingual cusp of a maxillary third molar is a holding cusp. The
general rule for maxillary holding cusps is as follows: a maxillary holding cusp contacts the distal
marginal ridge of its mandibular counterpart and the mesial marginal ridge of the mandibular tooth
distal to its counterpart, EXCEPT FOR THE MESIOLINGUAL CUSPS OF THE MOLARS, WHICH
CONTACT THE CENTRAL FOSSAE OF THEIR COUNTERPARTS. This should be the central fossa
of the counterpart, the mandibular third molar.
8. In comparison to those of permanent molars, the common root trunks of primary molars are
proportionately:
A. larger
B. smaller
C. relatively equal
D. variable, depending on the tooth
The correct answer is B. The common root trunk refers to that area of root apical to the crown but
coronal to the bifurcation or trifurcation of the roots. It can be viewed as the combined root area from
which the individual roots emerge. It is characteristic of primary molars that they have little or no
common root trunk. That is, the roots appear to furcate almost immediately below the crown. This is
due to the fact that the primary molar roots must flare widely in order to leave room for the developing
crowns of the succedaneous premolars.
9. The attrition at the contact point between teeth is known to cause:
A. Reduced embrasures interproximally
B. Diastemas
C. Longer clinical crowns
D. Shorter clinical crowns
The correct answer is A. Over time, posterior teeth lose small amounts of enamel as the contact
points wear against each other during chewing. The contacts become slightly broader, the teeth
themselves become slightly closer, through drifting, and the embrasures between the teeth become

slightly smaller. Diastemas (choice B) are not created, because the teeth drift to close any space
created by the attrition. The process is at a microscopic level; the wear is very gradual, as is the
accompanying drifting, so that contact is not lost. This process does not affect crown length (choices
C and D), which is occluso-cervical in direction. It only affects the mesio-distal dimension.
10. The epithelium of the mucous membrane of the mouth is:
A. simple squamous
B. stratified squamous
C. simple columnar
D. stratified columnar
E. cuboidal
The correct answer is B. Squamous cells are flattened, pancake-like cells, and when in layers are
known as stratified. The stratified squamous cells lining the mucosa of the oral cavity are similar to
those lining the outer skin, although less keratinized. They do, however, flake off continuously into the
mouth. Simple squamous tissue would be too thin and not allow for the constant flaking and
replacement. Columnar epithelium is found in both the respiratory system and other organs of the
digestive system.
11. The glenoid (articular) fossa in which the mandibular condyle articulates is a depression
within which cranial bone?
A. Sphenoid
B. Zygomatic
C. Temporal
D. Parietal
E. A combination of more than one bone
The correct answer is C. The names for the fossa include mandibular, articular, glenoid, and
temporal. This is due to the location of the fossa wholly within the temporal bone. The anterior border
of the fossa is the articular eminence of the temporal bone, and the posterior border is the tympanic
section of the temporal bone. Slightly more posterior is the mastoid process and associated styloid
process. So the entire eminence is a temporal bone feature.
12. The mesiobuccal cusp of the maxillary first molar occludes in:
A. the mesial marginal ridge of the mandibular second molar
B. the distal marginal ridge of the mandibular first molar
C. the central groove of the mandibular first molar
D. the facial embrasure between the mandibular first and second molars
E. the buccal groove of the first mandibular molar
The correct answer is E. Buccal cusps of maxillary teeth are guiding cusps, not holding cusps.
Therefore, they do not occlude on marginal ridges or central fossae. The general rule for maxillary
buccal cusps is that they occlude in the facial embrasure between their mandibular counterpart and
the tooth distal to their counterpart, EXCEPT for the mesiobuccal cusps of the molars, which occlude
in the buccal grooves of their mandibular counterparts, and the distobuccal cusp of the first molar,
which opposes the distobuccal groove of the mandibular first molar. In this question we are dealing
with the exception of the mesiobuccal cusps of maxillary molars, which occlude with the buccal
groove of their mandibular counterpart. In this case, the maxillary first molar mesiobuccal cusp
contacts the mandibular first molar buccal groove.
13. In extraction of maxillary first premolars, the root anatomy guides the dentist to use a
motion that is primarily:
A. buccal-lingual luxation
B. mesiodistal luxation
C. rotation
D. all of the above
The correct answer is A. One of the best reasons to know standard and unusual root shapes for the
various maxillary and mandibular teeth is to use the correct motion on the forceps when performing

extractions. In general, rounded roots are primarily rotated, including the maxillary central incisors and
maxillary canines. All double- and triple-rooted teeth cannot be rotated and must be luxated in the
buccal and lingual directions. Mesial-distal motion is normally ruled out because of the existence of
adjacent teeth. It is used in rare cases where there is no mesial and distal adjacent tooth. The
maxillary first premolar is invariably double rooted with a buccal and lingual root, and can never be
rotated without breaking the crown off of the root.
14. How many teeth (out of 32) in the normal dentition oppose only ONE other tooth?
A. None
B. 1
C. 2
D. 4
E. 6
The correct answer is D. Because of the shift of the mandibular teeth in a mesial direction due to the
smaller size of the anterior teeth, most teeth have two opposing teeth. That is, they oppose their
counterpart in the other arch, and a tooth either mesial or distal to that counterpart. There are two
classes of exceptions. One of the exceptions is the mandibular central incisors. These two teeth (#24
and #25) oppose only the maxilary centrals (#8 and #9). The other exception is the maxillary third
molars (#1 and #16), which oppose only the mandibular third molars. Note that the mandibular third
molars oppose both the maxillary second and third molars.
15. Which premolar is considered to be most molar-like in form?
A. Maxillary first
B. Maxillary second
C. Mandibular first
D. Mandibular second
The correct answer is D. The maxillary premolars are considered to be much more alike than are the
mandibular premolars. They are most premolar-like in form, with fairly similar sized buccal and lingual
cusps and greater overall symmetry. The mandibular premolars, by contrast, are significantly different
from each other. The first premolar is very canine-like with a small lingual cusp and relatively large
buccal cusp. The mandibular second premolar, while more premolar-like than the first, has attributes
of molars as well. The chief molar-like attribute of the mandibular second is the tendency toward three
cusps rather than two. The most common form of this tooth has a large buccal cusp with two smaller
lingual cusps making up the rest of the crown. The two lingual cusps are the mesiolingual and
distolingual and are separated by a lingual groove. There is a less common two-cusped variety which
is less molar-like in form.
16. The distal inclination in roots of the permanent mandibular first molar is:
A. more pronounced in either root, depending on the tooth
B. usually more pronounced in the mesial root
C. usually more pronounced in the distal root
D. usually equal in both roots
The correct answer is B. In the mandibular first molar, one of the major distinctions between roots is
the greater distal curvature of the mesial root. The distal root, by contrast, is usually more straight,
without significant curvature. Another distinction is the deep root concavity seen running the length of
the mesial surface of the mesial root. The distal root, by comparison, will have no similar concavity or
a very slight depression. This concavity will sometimes express itself as a partial split (bifurcation) of
some small section of the apical end of the mesial root into two roots. This feature is rare to unknown
on the distal root.
17. The greatest curvature of the cervical line interproximally is found on the:
A. Mesial of the maxillary central
B. Mesial of the maxillary lateral
C. Distal of the maxillary central
D. Distal of the maxillary lateral

The correct answer is A. When viewed from the proximal, all incisors show a distinct curvature of the
cervical line as the line curves markedly upward (toward the incisal). It then returns to a much lower
(apical) level on both the facial and lingual. It is most noticeable on the mesial of the central incisor.
This is one of those most, biggest, longest, shortest type questions common in this section.
18. Which incisor commonly exhibits the most variety in anatomy in both crown form and root
form?
A. Maxillary central
B. Maxillary lateral
C. Mandibular central
D. Mandibular lateral
The correct answer is B. It is commonly said that the third molars and the maxillary lateral incisor
are the most variable teeth in the mouth. The common forms of maxillary lateral variation include the
peg lateral, with an ice cream cone-shaped crown; the deep lingual pit, with a deep (often carious)
invagination; and the dens-in-dente, a deep lingual invagination giving a "tooth-within-a-tooth"
appearance on x-ray. By contrast, variations in the other incisors are minor and less pronounced.
19. The mesiobuccal cusp of the maxillary third molar occludes in:
A. the mesial marginal ridge of the mandibular second molar
B. the distal marginal ridge of the mandibular second molar
C. the mesial marginal ridge of the mandibular third molar
D. the facial embrasure between the mandibular second and third molars
E. the buccal groove of the third mandibular molar
The correct answer is E. Buccal cusps of maxillary teeth are guiding cusps, not holding cusps.
Therefore, they do not occlude on marginal ridges or central fossae. The general rule for maxillary
buccal cusps is that they occlude in the facial embrasure between their mandibular counterpart and
the tooth distal to their counterpart, EXCEPT for the mesiobuccal cusps of the molars, which occlude
in the buccal grooves of their mandibular counterparts, and the distobuccal cusp of the first molar,
which opposes the distobuccal groove of the mandibular first molar. In this question we are dealing
with the exception of the mesiobuccal cusps of maxillary molars, which occlude with the buccal
groove of their mandibular counterpart. In this case, the maxillary third molar mesiobuccal cusp
contacts the mandibular third molar buccal groove.
20. The mesiobuccal cusp of the mandibular second molar occludes with which maxillary
tooth surfaces?
A. The mesial marginal ridge of the second molar and distal marginal ridge of the first molar
B. The distal marginal ridge of the second molar and mesial marginal ridge of the third molar
C. The embrasure between the first and second molars
D. The central fossa of the maxillary second molar
The correct answer is A. The mesiobuccal cusp of a mandibular molar is a holding (supporting)
cusp. The general rule of occlusion of mandibular holding cusps is as follows: the holding cusps of the
mandibular teeth occlude on the mesial marginal ridge of their maxillary counterpart, and the distal
marginal ridge of the maxillary tooth mesial to their counterpart, EXCEPT distobuccal cusps of
mandibular molars occlude with central fossae of their counterparts, the distal cusp of the mandibular
first molar occludes with the distal triangular fossa of its counterpart, and the first premolar occludes
only with the mesial marginal ridge of its counterpart (but not the canine). In this case, the maxillary
counterpart is the maxillary second molar, and the tooth immediately mesial to it is the maxillary first
molar.
21. The most prominent cingulum is found on which incisor?
A. Maxillary central
B. Maxillary lateral
C. Mandibular central
D. Mandibular lateral

The correct answer is B. The maxillary lateral is often the most distinct in almost any anatomical
form. Its cingulum is usually more notable relative to its size than that of the central. It should be noted
that maxillary cingula are almost always more prominent than their mandibular counterparts. In
addition to the prominent cingulum, the maxillary lateral often features a deep lingual fossa, deep
lingual pits, and tubercles on the cingulum.
22.Which premolar has a lingually inclined occlusal table?
A. Maxillary first
B. Maxillary second
C. Mandibular first
D. Mandibular second
The correct answer is C. The mandibular first premolar is very canine-like in form, with a much
reduced lingual cusp, almost resembling a cingulum. The occlusal table thus inclines apically from
buccal to lingual, as in a downward slope. This is important in operative dentistry, as the occlusal
preparation for this tooth also is slanted to the lingual. All other premolars are essentially flat in their
occlusal table, or possibly very slightly lingually leaning if the buccal cusp is slightly higher than the
lingual. However, only the mandibular first premolar has this trait so pronounced that it is very different
from the other three.
23. When viewed from the buccal, the tooth closest to vertical in the mesiodistal angulation of
its main axis is the:
A. maxillary central incisor
B. maxillary lateral incisor
C. maxillary canine
D. maxillary first premolar
The correct answer is A. Another way of asking the question is: which tooth does not lean toward
the mesial or distal? The maxillary central incisor is almost straight vertically, estimated to be about 2
degrees from vertical. The lateral tends to lean slightly mesially, the canine even more in a distal
direction, and the first premolar slightly in a distal direction. Do not confuse this leaning with the
buccolingual leaning (when viewed from the mesial or distal). These measurements differ significantly
for the teeth listed, with the premolar being most vertical.
24. The distobuccal cusp of the mandibular first molar occludes with which maxillary tooth
surfaces?
A. The mesial marginal ridge of the second molar and distal marginal ridge of the first molar
B. The central fossa of the first molar
C. The mesial marginal ridge of the first molar and the distal marginal ridge of the second premolar
D. The embrasure between the first and second molars
The correct answer is B. The distobuccal cusp of a mandibular molar is a holding (supporting) cusp.
The general rule of occlusion of mandibular holding cusps is as follows: the holding cusps of the
mandibular teeth occlude on the mesial marginal ridges of their maxillary counterparts, and the distal
marginal ridges of the maxillary tooth mesial to their counterparts, EXCEPT distobuccal cusps of
mandibular molars occlude with central fossae of their counterparts, the distal cusp of the mandibular
first molar occludes with the distal triangular fossa of its counterpart, and the first premolar occludes
only with the mesial marginal ridge of its counterpart (but not the canine). In this case, the maxillary
counterpart is the maxillary first molar, and the cusp should occlude in the central fossa of this tooth.
25. Canaliculi and Haversian systems:
A. are found in acellular cementum
B. are found in cellular cementum
C. are found in both types of cementum
D. are not found in cementum
The correct answer is D. Cementum is often described as the dental tissue which most resembles
bone. It has a similar mineral content, and has an organic matrix primarily composed of collagen

fibers. However, the Haversian systems, concentric tissue layers, lacunae, canaliculi, etc., found in
bone are not found in cementum. Remember that these features are not always found in bone. Only
compact bone (rather than trabecular bone) has these features. Remember also that acellular
cementum could not have such features as Haversian systems, because Haversian systems contain
osteocytes (cells) within bony lacunae. However, neither acellular nor cellular cementum actually has
these features.
26. At the cervical line, a cross section of the maxillary canine would show:
A. a round shape
B. an oval shape wider on the lingual than the labial
C. a triangular shape with a distinctly wider labial surface
D. an oval shape with a slightly wider labial surface
The correct answer is D. The maxillary canine, at the cervix, will have a cross section which is oval
and flattened mesiodistally. It is generally slightly wider at the labial end than at the lingual. The pulp
cavity at this point is lens-shaped, double convex. It is similar in cross-sectional shape to the
mandibula canine.
27. An example of a guiding (nonsupporting) cusp is:
A. distolingual of #30
B. mesiobuccal of #18
C. distolingual of #14
D. lingual of #5
E. palatal of #13
The correct answer is A. Note that the holding (supporting, occluding) cusps of the posterior
dentition are the lingual (palatal) cusps of the maxillary teeth and the buccal (facial) cusps of the
mandibular teeth. Buccals of maxillary teeth and linguals of mandibular teeth are guiding cusps.
Choice A is the lingual of a mandibular first molar, so it is not a holding cusp, and is a guiding cusp.
Choice B is a buccal of a mandibular molar and is a holding cusp. Choice C is a lingual cusp of a
maxillary molar and is a holding cusp. Choice D is the lingual of a maxillary premolar and is a holding
cusp. Choice E is the palatal (lingual) of a maxillary premolar and is a holding cusp.
28. Which incisor is most commonly congenitally missing?
A. Maxillary central
B. Maxillary lateral
C. Mandibular central
D. Mandibular lateral
The correct answer is B. The maxillary lateral and the third molars show more variability than any
other teeth. Besides a wide variety of unusual morphologies (peg lateral, etc.), the maxillary lateral
incisor is fairly commonly congenitally missing. In these patients, the canines drift mesially toward the
centrals. Also commonly congenitally missing are third molars and premolars.
29. A patient is congenitally missing the maxillary lateral incisors and mandibular second
premolars. This individual is said to have the condition known as:
A. anodontia
B. oligodontia
C. hypodontia
D. hyperdontia
The correct answer is C. Anodontia (choice A) refers to the complete absence of tooth formation. In
oligodontia (choice B), most but not all teeth are missing. Both of these conditions are often
associated with ectodermal dysplasia. Hypodontia refers to a small number of congenitally missing
teeth. This is fairly common, and is not usually associated with a systemic disease or disturbance.
30. Cervical bulges are found on which surfaces of primary molar teeth?
A. Mesial and distal
B. Buccal and lingual
C. Lingual only
D. Buccal only

The correct answer is D. Cervical bulges are overhangs of the crown sticking out above the cervical
line of the primary teeth. In primary anteriors, the cervical bulge is found on both the buccal and
lingual surfaces. In the molars, it is found only on the buccal. The most conspicuous cervical bulge in
the primary molars is that of the primary mandibular first molar. It is often described as being potbelly
when seen from the proximal view. The other molars are not as exaggerated in this area, but still
show a pronounced buccal cervical bulge.
31. In rare cases, a permanent mandibular first molar may have a complete supernumerary
third root. This root is usually:
A. a second mesial root
B. a second distal root
C. a buccal root
D. a lingual root
The correct answer is B. Despite the fact that the mandibular first molar has a distinct mesial root
concavity on the mesial side of the mesial root, and that this root sometimes furcates partially
upwards from the mesial, complete additional whole roots are usually second distal roots. Buccal
and/or lingual additional roots are not found.
32. In centric occlusion, the distolingual cusp of the maxillary first molar will contact:
A. mesial marginal ridge of mandibular first premolar and distal marginal ridge of second premolar
B. the distal marginal ridge of mandibular second premolar and mesial marginal ridge of first molar
C. the distal marginal ridge of mandibular first molar and mesial marginal ridge of the second molar
D. the distal marginal ridge of the mandibular second molar and mesial marginal ridge of first molar
The correct answer is C. The distolingual cusp of a maxillary molar is a holding cusp. The general
rule for maxillary holding cusps is as follows: a maxillary holding cusp contacts the distal marginal
ridge of its mandibular counterpart and the mesial marginal ridge of the mandibular tooth distal to its
counterpart, EXCEPT FOR THE MESIOLINGUAL CUSPS OF THE MOLARS, WHICH CONTACT
THE CENTRAL FOSSAE OF THEIR COUNTERPARTS. This should be the distal marginal ridge of
the mandibular first molar and mesial marginal ridge of the second molar.
33. The mesiolingual cusp of the mandibular second molar contacts:
A. the lingual embrasure between the maxillary first molar and second molar
B. the distal marginal ridge of the maxillary second molar
C. the central fossa of the maxillary second molar
D. the lingual groove of the maxillary second molar
E. the lingual embrasure between the maxillary second molar and third molar
The correct answer is A. Lingual cusps of mandibular teeth are guiding cusps, not holding cusps.
Therefore, they do not occlude on marginal ridges or central fossae. The general rule for mandibular
lingual cusps is that they occlude in the lingual embrasures between their maxillary counterparts and
the teeth mesial to their counterparts, EXCEPT for the distolingual cusps of the mandibular molars,
which occlude in the lingual grooves of their maxillary counterparts. In this question we are not dealing
with the exception of the distolingual cusps of mandibular molars, so in this case, the mandibular
second molar mesiolingual cusp contacts the lingual embrasure between its counterpart (the maxillary
second molar) and the tooth mesial to it (the maxillary first molar).
34. The epithelial root sheath of Hertwig is composed of:
A. inner enamel epithelium and stellate reticulum
B. stellate reticulum only
C. inner and outer enamel epithelium
D. all of the tissues listed above
E. none of the above
The correct answer is C. Only the original inner and outer enamel epithelium take part in root
formation. There is no stellate reticulum in between them, as would be found in the original enamel

organ. The root form is originally outlined by the epithelial root sheath of Hertwig, which then induces
formation of odontoblasts, which then produce root dentin. Pieces of the root sheath, which do not
disappear, remain as the epithelial rests of Malassez. These small groups of tissue are ectodermal in
origin, as the original enamel epithelium is ectodermal.
35. At the cervical line, a cross section of the maxillary lateral incisor would show:
A. a round shape
B. an oval shape wider on the lingual than the labial
C. a triangular shape with a distinctly wider labial surface
D. an oval shape with a slightly wider labial surface
E. mesiodistal flattening
The correct answer is A. The maxillary lateral incisor exhibits an almost round root cross section
shape at the cervical line. It is not flattened mesiodistally or labiolingually. The pulp chamber mimics
the root shape by being round at this point as well. It is very similar in both root shape and pulp shape
to those of the maxillary central. Note that as the pulp enters the coronal section, it will begin to widen
mesiodistally (flatten buccolingually) in keeping with the crown shape.
36. Because of the staining effect, tetracycline is not given to children younger than
approximately what age?
A. 2 years
B. 5 years
C. 8 years
D. 11 years
E. 14 years
The correct answer is C. This may not seem like dental anatomy, but it is actually a calcification
question. In order to avoid tetracycline staining or any other intrinsic stain of enamel, we must avoid
all ages where tooth crown calcification is occurring. The last tooth to calcify (and erupt) is the third
molar. Most third molars erupt at 17 to 21 years, but calcification of enamel occurs mostly at about
age 8. So pregnant mothers and children age 8 and younger are not given tetracycline.
37. Which structure is involved in pulling the disc of the TMJ forward?
A. Stylomandibular ligament
B. Medial pterygoid muscle
C. Mandibular condyle
D. TMJ meniscus
E. Lateral pterygoid muscle
The correct answer is E. The lateral pterygoid connects from the greater wing of the sphenoid and
the lateral surface of the lateral pterygoid plate to the mandibular condyle, articular disc, and capsule.
It is the only muscle that attaches to these structures. The stylomandibular ligament, sometimes given
a vague protective function, extends from the styloid process to the lingula of the mandible. The
medial pterygoid muscle is an elevator with no direct attachment to the TMJ. The mandibular condyle
is separated from the glenoid (temporal) fossa by the articular (TMJ) disc. The condyle does not pull
the disc. The disc is also called the meniscus; they are the same thing.
38. The largest cusp of a maxillary first molar is the:
A. mesiobuccal
B. mesiolingual
C. distobuccal
D. distolingual
E. Carabelli
The correct answer is B. The three largest cusps (trigon, or primitive cusp triangle) of the maxillary
first molar are the MB, ML, and DB cusps. The DL cusp is smallest, and is known as the talon cusp
(or talon). This cusp is also the one which is even less conspicuous as you go from first to second to

third molar. Note also that the DB cusp decreases in relative size as you go from first to second and
third molar. The order of size of the cusps in this tooth, then, is from largest to smallest: ML, MB, DB,
DL, Carabelli.
39. Tooth #B is replaced by tooth #:
A. 2
B. 3
C. 4
D. 5
E. 6
The correct answer is D. Remember that primary molars are succeeded by permanent premolars.
The permanent molars erupt distal to all primary teeth and do not replace them. So the maxillary
replacements are 4 for A, 5 for B, 6 for C, 7 for D, 8 for E, 9 for F, 10 for G, 11 for H, 12 for I, and 13
for J. 1, 2, 3, 14, 15, and 16 do not replace primary teeth.
40. Which two molars can be considered to be five-cusped teeth?
A. Maxillary first and second
B. Mandibular first and second
C. Maxillary and mandibular first
D. Maxillary and mandibular second
The correct answer is C. Most maxillary molars are considered four-cusped teeth. However, in some
cases, the Carabelli cusp (trait) can be large enough to be considered a fifth cusp on a maxillary first
molar. The maxillary second molar is usually four-cusped. The mandibular first molar is normally fivecusped with an ML, MB, DL, and DB cusp and a fifth distal cusp. The mandibular second lacks the
distal cusp and is normally a four-cusp tooth.
41.The shape of the pulp chamber in cross section cut at the cervical line in the maxillary
canine is:
A. oval (flattened mesiodistally)
B. oval (flattened bucco-lingually)
C. round
D. figure eight
The correct answer is A. Only maxillary incisors have roundish pulp chamber cross sections when
cut at the cervical line. This is true for both maxillary centrals and laterals. In general, mandibular
centrals and both arch canines will have ovalish pulp chambers, flattened mesiodistally. (Imagine
squashing a round pulp from both the mesial and distal sides at once, resulting in a squashed oval,
pointing to the buccal and lingual). Remember that the shape of a pulp chamber is usually related to
the shape of the surrounding root.
42. In the maxillary arch, the narrowest incisal or occlusal embrasure is located:
A. between first premolar and canine
B. between canine and lateral incisor
C. between lateral incisor and central
D. between central incisors
The correct answer is D. The best way to answer the question is to imagine contact points and
proximal-incisal angles for the anterior teeth. A small incisal embrasure will be located wherever the
contact points are high (incisal) and where proximal-incisal line angles are almost perpendicular. The
best example of this arrangement is the central incisors. Their mesial incisal line angles are very
square, and the contact is in the incisal third of the tooth. As you go further distally, the line angles
become more rounded, especially for canines. These rounded line angles at the incisal, as well as the
more apical contacts, dropping to the middle third as you reach the distal of the canine, ensure much
larger incisal embrasures.
43. The inner enamel cuticle, found on the surface of enamel of an erupting tooth crown, is:
A. also known as Nasmyth's membrane
B. also known as the stratum intermedium
C. cellular
D. formed by odontoblasts
The correct answer is A. As the tooth crown nears eruption, the ameloblasts produce their final
product, known as the inner enamel cuticle. This material is acellular, and is firmly adherent to the

10

enamel surface as the tooth erupts. Its older name is Nasmyth's membrane. This cuticle has on its
outside a second acellular layer formed from the keratinized remnants of the dental sac. This
adherent double layer may persist for some time after the tooth erupts but is eventually worn away,
leaving the enamel exposed in the oral cavity.
44.. Which ligament of the temporomandibular joint originates from a thin, pointed extension
of the temporal bone?
A. Temporomandibular
B. Stylomandibular
C. Sphenomandibular
D. Lateral
The correct answer is B. The bony origins of the three TMJ ligaments are primarily from three
different bones. The stylomandibular ligament originates from a thin, bony extension of the temporal
bone, known as the styloid process. The spine of the sphenoid bone is the major origin of the
sphenomandibular ligament. The lateral ligament (temporomandibular ligament) has its origin from the
zygomatic process. This process is a fusion of the temporal and zygomatic bones.
45. Which of the following is the most coronally located periodontal fiber group?
A. Alveolar crestal
B. Apical
C. Horizontal
D. Oblique
The correct answer is A. We can eliminate choice B, as the apex is at the opposite end from the
crown. The apical fibers are, of course, most apical, and run from the apex of the alveolar bone socket
to the root tip. The oblique fibers are slightly more coronal. They run from the cementum near the
apical end, obliquely coronally to alveolar bone. The horizontal fibers are more coronal and run
horizontally from cementum to alveolar bone around mid-root. The alveolar crestal fibers are most
coronal, and run from the most coronal part of the root to the alveolar crest of bone.
46. In a distal view of a permanent maxillary first molar, how many roots should be visible?
A. 1
B. 2
C. 3
D. 4
The correct answer is C. This may not be as obvious as it seems. Rule out 4, as the molar only has
three roots: the palatal, mesiobuccal, and distobuccal. However, how should they look in a distal
view? The palatal root will clearly be visible as the only root on the palatal side. It is massive, and
often hooked. On the buccal end, a distal view should easily show the full length of the distobuccal
root, so we have two roots so far. What about the mesiobuccal? In this case we can see this root
mostly because it is larger than the distobuccal, so some, but not all, of the mesiobuccal root is
blocked by the distobuccal. We should, however, see some small sections of it outside of the profile of
the distobuccal root.
47. The heights of contour of the mandibular first premolar, when viewed from the mesial, are
located within which third?
A. buccal in occlusal third, lingual in middle third
B. both in middle third
C. both in occlusal third
D. buccal in cervical third, lingual in occlusal third
E. both in cervical third
The correct answer is D. As a general rule, the buccal heights of contour of the premolars are
located within the cervical thirds. The lingual heights of contour will normally be more occlusal. On the
mandibular first premolar, it is especially occlusal, within the occlusal third.

11

48. Which of the following oral muscles is NOT innervated by CN V?


A. Masseter
B. Buccinator
C. Medial pterygoid
D. Lateral pterygoid
E. Mylohyoid
The correct answer is B. The general rule for innervation by CN V (the trigeminal) is that it
innervates the muscles of mastication (temporalis, lateral pterygoid, medial pterygoid, masseter), the
mylohyoid, the anterior digastric, and the two tensors (tensor tympani and tensor veli palatini). Note
that the buccinator is not included. The buccinator is in the group of muscles of facial expression,
which are all innervated by CN VII (the facial nerve).
49. The contact of the permanent mandibular incisor with the canine occurs at which level of
the lateral?
A. Incisal third
B. Junction of incisal and middle thirds
C. Middle third
D. Gingival third
The correct answer is A. Mandibular incisors are widest near the incisal tip, and their contacts (both
central and lateral) are always near the incisal edge. The mandibular canine contact with the lateral
incisor is thus in the incisal third. In the canine, the contact is also in the incisal third, but not as incisal
as in the lateral. The distal contact of the canine is also in the incisal third, but near the junction of the
incisal and middle thirds.
50. Dens-in-dente is most commonly noted in which tooth?
A. Maxillary central
B. Maxillary lateral
C. Maxillary canine
D. Mandibular central
E. Mandibular lateral
The correct answer is B. The maxillary lateral is the most highly variable tooth in the mouth. It can
be normal incisor-shaped, peg-shaped, or congenitally missing, or have a wide variety of
invaginations, from small pits to deep pits to dens-in-dente, or tooth-within-a-tooth. This is the case
when the invagination is so deep that the radiographic appearance appears to show a small tooth
inside the maxillary lateral.
51. Dentinal tubules which enter into enamel are known as:
A. enamel spindles
B. enamel tufts
C. enamel lamellae
D. none of the above
The correct answer is A. There are a number of somewhat confusing anomalies at the
dentinoenamel junction. Enamel spindles are odontoblastic processes and dentinal tubules which
extend a short way into enamel. Enamel lamellae, on the other hand, extend from enamel into dentin.
They are pieces of uncalcified organic enamel material. Enamel tufts are hypocalcified enamel rods.
They are individual entities; however, they appear as groups (tufts) when a group of them are viewed
laterally.
52. Guiding cusps normally contact:
A. mesial marginal ridges
B. distal marginal ridges
C. central fossae
D. embrasures

12

The correct answer is D. In occlusion, cusps are defined as either holding (supporting) cusps or
guiding cusps. Holding cusps, in central occlusion, make contact with the opposing arch, establish
vertical dimension of occlusion, and support the forces of occlusion. They are the lingual cusps of the
maxillary arch and buccal cusps of the mandibular. When the teeth are in centric occlusion, these
cusps are in contact. The other cusps (guiding) are not. The other cusps are either lingual to the
contact (lingual cusps of the mandibular arch) or buccal to the contact (buccal cusps of the maxillary
arch). Holding cusps generally occlude in the marginal ridge and central fossa areas of the opposing
arch. Guiding cusps generally lie in embrasures between teeth, or between cusps of the lingual or
buccal surfaces of the opposing arch.
53. Which premolar, when viewed from the facial, has a longer mesial cusp ridge than distal
cusp ridge?
A. Maxillary first
B. Maxillary second
C. Mandibular first
D. Mandibular second
The correct answer is A. The question depends on the fact that the maxillary first premolar has an
asymmetric positioning of its two cusps. Rather than having the buccal and lingual cusps in a straight
line, the buccal cusp is displaced slightly distally, and the lingual cusp is displaced slightly mesially.
This gives the occlusal view of the tooth a slightly twisted appearance. Cusp ridges run from the cusp
tip to the proximal edge of the tooth at the marginal ridge. Therefore, if you look facially at the
maxillary first premolar, you will see the buccal cusp shifted toward the distal. This makes the cusp tip
farther from the mesial, and thus there will be a longer mesial cusp ridge on this tooth.
54. In centric occlusion, the distolingual cusp of the maxillary second molar will contact:
A. the central fossa of the mandibular second molar
B. the distal marginal ridge of the mandibular first molar and mesial marginal ridge of second molar
C. the distal marginal ridge of the mandibular second molar and mesial marginal ridge of first molar
D. the distal marginal ridge of the mandibular second molar and mesial marginal ridge of third molar
The correct answer is D. The distolingual cusp of a maxillary second molar is a holding cusp. The
general rule for maxillary holding cusps is as follows: a maxillary holding cusp contacts the distal
marginal ridge of its mandibular counterpart and the mesial marginal ridge of the mandibular tooth
distal to its counterpart, EXCEPT FOR THE MESIOLINGUAL CUSPS OF THE MOLARS, WHICH
CONTACT THE CENTRAL FOSSAE OF THEIR COUNTERPARTS. This should be the distal
marginal ridge of the mandibular second molar and mesial marginal ridge of the third molar.
55. Of the following premolars, the one which can never be rotated during extraction is the:
A. maxillary first
B. maxillary second
C. mandibular first
D. mandibular second
The correct answer is A. For the most part, rotation of a tooth during extraction works best on teeth
with rounded roots, such as the maxillary central incisor and maxillary canine. It can be used in
conjunction with buccal-lingual luxation on teeth with single ovalish roots. It can never be used on
double-rooted teeth. Of the premolars above, the maxillary first is always double rooted. Of the others,
the maxillary second is sometimes double rooted and sometimes single. For the mandibular
premolars, the mandibular first is the most likely to have a single ovalish, but close to round, root. It is
theMOST likely premolar on which rotation may be part of the extraction.
56. In centric occlusion, the lingual cusp of the maxillary second premolar contacts:
A. mesial marginal ridge of mandibular first premolar and distal marginal ridge of second premolar
B. distal marginal ridge of mandibular first premolar and mesial marginal ridge of second premolar
C. mesial marginal ridge of the mandibular first molar and distal marginal ridge of second premolar
D. distal marginal ridge of the mandibular second premolar and mesial marginal ridge of first molar

13

The correct answer is D. The mesiolingual cusp of a maxillary second premolar is a holding cusp.
The general rule for maxillary holding cusps is as follows: a maxillary holding cusp contacts the distal
marginal ridge of its mandibular counterpart and the mesial marginal ridge of the mandibular tooth
distal to its counterpart, EXCEPT FOR THE MESIOLINGUAL CUSPS OF THE MOLARS, WHICH
CONTACT THE CENTRAL FOSSAE OF THEIR COUNTERPARTS. This should be the distal
marginal ridge of the mandibular second premolar and mesial marginal ridge of the first molar.
57. The maxillary molar most likely to have fused roots is the:
A. first
B. second
C. third
D. fused roots are rare for all three molars
The correct answer is C. While all three maxillary molars commonly have three roots, the roots and
their arrangement differ distinctively from one to the other. Most notable in the first molar is the plierslike appearance of the two buccal roots. The mesiobuccal hooks distally and the distobuccal hooks
mesially. The second molar does not have this curvature, and the two buccal roots are more parallel,
with both roots inclined distally. The third molar often has three fused roots forming a cone-like
structure, or sometimes all distally inclined. These factors are important in exodontia of maxillary
molars. Maxillary third molars, in particular, can sometimes be elevated out distally in a rotating
manner because of the fused cone-like distally facing roots.
58.The distobuccal cusp of the mandibular second molar occludes with which maxillary tooth
surfaces?
A. The mesial marginal ridge of the second molar and distal marginal ridge of the first molar
B. The distal marginal ridge of the second molar and mesial marginal ridge of the third molar
C. The embrasure between the first and second molars
D. The central fossa of the maxillary second molar
The correct answer is D. The distobuccal cusp of a mandibular molar is a holding (supporting) cusp.
The general rule of occlusion of mandibular holding cusps is as follows: the holding cusps of the
mandibular teeth occlude on the mesial marginal ridge of their maxillary counterpart, and the distal
marginal ridge of the maxillary tooth mesial to their counterpart, EXCEPT distobuccal cusps of
mandibular molars occlude with central fossae of their counterparts, the distal cusp of the mandibular
first molar occludes with the distal triangular fossa of its counterpart, and the first premolar occludes
only with the mesial marginal ridge of its counterpart (but not the canine). In this case, the maxillary
counterpart is the maxillary second molar, and the mandibular cusp will occlude in its central fossa.
59. The major blood supply of the PDL is from:
A. pulpal vessels
B. gingival vessels
C. periosteal vessels
D. periapical vessels
The correct answer is C. All of the vessels listed supply blood to the PDL, as all of them are in the
vicinity of the PDL and all anastomose or send branches there. Out of this group, however, the
MAJOR source is vessels branching from the periosteum surrounding the alveolar bone. Note that
both lymphatics and nerves follow the path of the blood vessels in the PDL.
60. Which tooth below is MOST likely to be caries-resistant?
A. Maxillary first premolar
B. Mandibular second premolar
C. Mandibular lateral incisor
D. Mandibular second molar
E. Maxillary central incisor
The correct answer is C. Look for a mandibular anterior tooth to be your answer here. You will often
notice clinically that partially edentulous patients often retain mandibular anteriors despite loss of most

14

other teeth. This is due to the following three factors: 1) they have few significant pits or fissures; 2)
they are bathed in saliva most of the time, which resists plaque buildup; and 3) they are cleaned by
the normal movements of the tongue. Other teeth cannot fit all three of these categories. Note that
mandibular canines are often the last teeth left. This is because, although the mandibular incisors are
rarely lost to caries, they are often lost to periodontal disease, partially due to their short roots and
easy calculus buildup.
61.A patient presents for examination with both mandibular first molars distal to the maxillary
first molars. This occlusion is known as:
A. Class I, distocclusion
B. Class II, mesiocclusion
C. Class II, distocclusion
D. Class III, mesiocclusion
E. Class III, distocclusion
The correct answer is C. The normal (Class I) jaw relationship will result in the mandibular first molar
being located one cusp (one-half tooth) mesial to the maxillary first molar. If the mandibular molar is
located distal to the maxillary, it indicates a small mandible (micrognathy) or another problem causing
the maxillary teeth to be too far mesial and the mandibular too far distal. This is distocclusion is
Angles Class II. Mesiocclusion is Angles Class III, where the mandibular first molar is more than onehalf tooth mesial to the maxillary first molar.
62. Which of the following is NOT a periodontal fiber?
A. Oblique
B. Transseptal
C. Apical
D. Horizontal
The correct answer is B. Periodontal fibers must run from the cementum of the tooth into alveolar
bone. The main periodontal fiber types are alveolar crestal, horizontal, oblique, and apical, named for
either their position or alignment. The transseptal fiber is classified with the gingival group, as it does
not enter alveolar bone. Instead, as the name suggests, it travels from one tooth to another tooth,
crossing over the alveolar crest. It attaches directly from the cementum of one tooth into the
cementum of another, but does not attach to bone.
63. The anterior tooth with the most pronounced lingual ridge is the:
A. Maxillary central incisor
B. Mandibular lateral incisor
C. Maxillary canine
D. Mandibular canine
The correct answer is C. A lingual ridge runs from the incisal edge down the center of the lingual
surface of the crown, toward the cervical end of the crown. It splits the lingual surface in half, and
usually has a depression on either side (lingual fossae) which separate the lingual ridge from the
mesial and distal marginal ridges. Lingual ridges are found on canines, but not on incisors (choices A
and B). The lingual ridge is more pronounced on the maxillary canine than on the mandibular (choice
D). This is in keeping with a general rule that the lingual anatomy of the maxillary canine is much
more pronounced than that of the mandibular.
64. Oblique ridges are found on which molar teeth?
A. Maxillary first and second
B. Mandibular first and second
C. Maxillary and mandibular first
D. Maxilary and mandibular second
E. First and second molars in both arches
The correct answer is A. Oblique ridges are characteristic of maxillary molar teeth. They connect the
mesiolingual and distobuccal cusps. The oblique ridge is also considered to be the distal end of the
trigon, the major cusp area of the maxillary molar, composed of the ML, MB, and DB cusps. The DL

15

cusp is often reduced and is referred to as the talon (as opposed to the trigon). The trigon is
evolutionarily the primitive cusp triangle of the molar crown. Oblique ridges are not found on
mandibular molars or on any other teeth.
65. The most highly mineralized oral or dental tissue is:
A. enamel
B. dentin
C. bone
D. cementum
The correct answer is A. Enamel is the most mineralized and hardest tissue in the human body.
Most estimates place the mineral content of enamel at about 95 to 98%. Dentin, though highly
mineralized, still has more organic content than does enamel. Most estimates place the mineral
content of dentin at about 80%. Cementum is least mineralized, and is most similar to bone. There are
varying estimates of its mineral content, but most place it at about 60 to 65%.
66. The embrasures which surround the contact points of anterior teeth are:
A. mesial, distal, lingual, buccal
B. mesial and distal only
C. buccal and lingual only
D. buccal, lingual, cervical, gingival
E. incisal, cervical, buccal, lingual
The correct answer is E. The best way to answer the question is to imagine that the embrasures are
spaces surrounding a small point of contact between two incisors. This small contact point will have
space above it, below it, in front of it, and in back of it. These spaces in dental anatomy will be,
respectively, incisal, cervical (gingival), buccal (facial), and lingual (palatal). Note that there is no
space mesial or distal to the contact point. Immediately mesial or distal to the contact would be tooth
structure.
67. The distolingual cusp of the mandibular second molar contacts:
A. the lingual embrasure between the maxillary first molar and second molar
B. the lingual groove of the maxillary second molar
C. the central fossa of the maxillary second molar
D. the lingual groove of the maxillary first molar
E. the lingual embrasure between the maxillary second molar and third molar
The correct answer is B. Lingual cusps of mandibular teeth are guiding cusps, not holding cusps.
Therefore, they do not occlude on marginal ridges or central fossae. The general rule for mandibular
lingual cusps is that they occlude in the lingual embrasures between their maxillary counterparts and
the teeth mesial to their counterparts, EXCEPT for the distolingual cusps of the mandibular molars,
which occlude in the lingual grooves of their maxillary counterparts. In this question we are dealing
with the exception of the distolingual cusps of mandibular molars, so in this case, the mandibular
second molar distolingual cusp contacts the lingual groove of its counterpart (the maxillary second
molar).
68. The distobuccal cusp of the maxillary second molar occludes in:
A. the mesial marginal ridge of the mandibular second molar
B. the buccal groove of the mandibular second molar
C. the central groove of the mandibular third molar
D. the facial embrasure between the mandibular first and second molars
E. the facial embrasure between the mandibular second and third molars
The correct answer is E. Buccal cusps of maxillary teeth are guiding cusps, not holding cusps.
Therefore, they do not occlude on marginal ridges or central fossae. The general rule for maxillary
buccal cusps is that they occlude in the facial embrasures between their mandibular counterparts and
the teeth distal to their counterparts, EXCEPT for the mesiobuccal cusps of the molars, which occlude
in the buccal grooves of their mandibular counterparts, and the distobuccal cusp of the first molar,

16

which opposes the distobuccal groove of the mandibular first molar. In this question, we are NOT
dealing with the exception of the mesiobuccal cusps of the maxillary molars or the distobuccal cusp of
the maxillary first molar, so this cusp should follow the general rule and occlude with the facial
embrasure between its mandibular counterpart (the mandibular second molar) and the mandibular
molar distal to it (the mandibular third molar).
69. The mandibular incisors of a patient are worn down so that dentin is visible on the incisal
edge. This is due to grinding of these edges against the natural maxillary incisors. This loss of
tooth structure can be termed:
A. attrition
B. abrasion
C. erosion
D. internal resorption
The correct answer is A. Attrition is the mechanical wearing of teeth due to physiologic processes,
including chewing and bruxism. Abrasion (choice B) is the mechanical wearing away of tooth structure
due to some outside object, such as toothbrush abrasion at the cervical lines of teeth due to hard
sideways brushing, or from habitually holding bobby pins or nails with the teeth. Erosion (choice C) is
the chemical dissolving of tooth structure. It can be caused by a number of factors, including sucking
on lemons and other acidic fruit, excessive intake of acidic beverages (cola), or excessive vomiting,
as in bulimia. Bulimics often exhibit normal facial surfaces with severely eroded lingual surfaces.
70. In centric occlusion, the lingual cusp of the maxillary first premolar contacts:
A. mesial marginal ridge of mandibular first premolar and distal marginal ridge of second premolar
B. distal marginal ridge of mandibular first premolar and mesial marginal ridge of second premolar
C. mesial marginal ridge of the mandibular first molar and distal marginal ridge of second premolar
D. distal marginal ridge of the mandibular second premolar and mesial marginal ridge of first molar
The correct answer is B. The lingual cusp of a maxillary premolar is a holding cusp. The general rule
for maxillary holding cusps is as follows: a maxillary holding cusp contacts the distal marginal ridge of
its mandibular counterpart and the mesial marginal ridge of the mandibular tooth distal to its
counterpart, EXCEPT FOR THE MESIOLINGUAL CUSPS OF THE MOLARS, WHICH CONTACT
THE CENTRAL FOSSAE OF THEIR COUNTERPARTS. This should be the distal marginal ridge of
the mandibular first premolar and mesial marginal ridge of the second premolar.
71. When a fourth canal is found in the permanent maxillary first molar, it is found in the:
A. mesiobuccal root
B. distobuccal root
C. palatal root
D. mesiolingual root
The correct answer is A. Note that the three-rooted maxillary first molar does not have a
mesiolingual root. The maxillary first molar most often has three roots: mesiobuccal, distobuccal, and
palatal. The palatal root is largest, and its canal is largest and widest. In the case of a fourth canal, it
is invariably found in the mesiobuccal root. The distobuccal and palatal roots are invariably onecanalled.
72. Odontoblasts laying down dentin matrix would be expected to contain large numbers of
well-developed:
A. nuclei
B. lysosomes
C. rough ER
D. smooth ER
E. mitochondria
The correct answer is C. This is a case of a best answer with a reasonable second-best answer.
The best answer is rough endoplasmic reticulum. The rough ER contains numerous ribosomes, and is
known for producing proteins for export. Collagen fibers of the dentin matrix would be good examples

17

of protein made for export (outside of the cell). The second-best answer is probably mitochondria, in
that the synthesis of collagen would be energy-consuming. Odontoblasts are single nuclei cells (unlike
osteoclasts, which are multinucleated). Note that smooth ER lacks ribosomes, and that the digestive
enzyme activity of lysosomes would not be needed for this process.
73. A mandibular permanent first molar is likely to have a deep root concavity running apically
down:
A. both roots
B. the mesial root
C. the distal root
D. neither root
The correct answer is B. One of the distinctions between the mandibular first-molar mesial and distal
roots is the deep root concavity seen running the length of the mesial surface of the mesial root. The
distal root, by comparison, will have no similar concavity or a very slight depression. Other differences
include the greater likelihood of the mesial root having a distal curvature, and the greater likelihood of
the mesial root being partly split by a partial furcation running partway up from the apical end.
74. The distobuccal cusp of the mandibular third molar occludes with which maxillary tooth
surfaces?
A. The mesial marginal ridge of the third molar and distal marginal ridge of the second molar
B. The central fossa of the third molar
C. The mesial marginal ridge of the third molar only
D. The distal marginal ridge of the third molar only
The correct answer is B. The distobuccal cusp of a mandibular molar is a holding (supporting) cusp.
The general rule of occlusion of mandibular holding cusps is as follows: the holding cusps of the
mandibular teeth occlude on the mesial marginal ridge of their maxillary counterpart, and the distal
marginal ridge of the maxillary tooth mesial to their counterpart, EXCEPT distobuccal cusps of
mandibular molars occlude with central fossae of their counterparts, the distal cusp of the mandibular
first molar occludes with the distal triangular fossa of its counterpart, and the first premolar occludes
only with the mesial marginal ridge of its counterpart (but not the canine). In this case, the maxillary
counterpart is the maxillary third molar, and the cusp should occlude in the central fossa of this tooth.
Remember that the mandibular third molar is located one-half tooth mesial to the maxillary.
75. Damage to the right lateral pterygoid results in:
A. the mandible turning right on protrusion
B. inability to elevate the mandible
C. the mandible turning left on protrusion
D. none of the above
The correct answer is A. As a rule, the mandible will move toward the side of injury when the lateral
pterygoid is damaged. In this case, the right condyle will not move because of the muscle damage.
The left condyle moves forward, but because there is no movement on the right side, the left condyle
moves out and rotates to the right, with the nonmoving right condyle acting as a pivot point. Damage
to the lateral pterygoid will not affect elevation (closing), because the lateral pterygoid is not an
elevator. The elevators are the temporalis, medial pterygoid, and masseter.
76. The distal cusp of the mandibular first molar occludes with which maxillary tooth
surfaces?
A. The mesial marginal ridge of the first molar and distal marginal ridge of the second molar
B. The distal marginal ridge of the second molar and mesial marginal ridge of the first molar
C. The embrasure between the first and second molars
D. The distal triangular fossa of the first molar
The correct answer is D. The distal cusp of the mandibular first molar is a holding cusp. The general
rule of occlusion of mandibular holding cusps is as follows: the holding cusps of the mandibular teeth

18

occlude on the mesial marginal ridge of their maxillary counterpart, and the distal marginal ridge of the
maxillary tooth mesial to their counterpart, EXCEPT DISTOBUCCAL CUSPS OF MANDIBULAR
MOLARS OCCLUDE WITH CENTRAL FOSSAE OF THEIR COUNTERPARTS, THE DISTAL CUSP
OF THE MANDIBULAR FIRST MOLAR OCCLUDES WITH THE DISTAL TRIANGULAR FOSSA OF
ITS COUNTERPART, AND THE FIRST PREMOLAR OCCLUDES ONLY WITH THE MESIAL
MARGINAL RIDGE OF ITS COUNTERPART (BUT NOT THE CANINE). Note that mesiobuccal cusps
of mandibular molars and buccal cusps of premolars contact marginal ridges.
77. Hypocalcified enamel rods which enter into enamel from the DEJ are known as:
A. enamel spindles
B. enamel tufts
C. enamel lamellae
D. none of the above
The correct answer is B. There are a number of somewhat confusing anomalies at the
dentinoenamel junction. Enamel spindles are odontoblastic processes and dentinal tubules which
extend a short way into enamel. Enamel lamellae, on the other hand, extend from enamel into dentin.
They are pieces of uncalcified organic enamel material. Enamel tufts are hypocalcified enamel rods.
They are individual entities; however, they appear as groups (tufts) when a group of them are viewed
laterally.
78. The mesiolingual cusp of the mandibular third molar contacts:
A. the lingual embrasure between the maxillary second molar and third molar
B. the distal marginal ridge of the maxillary second molar
C. the central fossa of the maxillary second molar
D. the lingual groove of the maxillary third molar
E. the buccal embrasure between the maxillary second molar and third molar
The correct answer is A. Lingual cusps of mandibular teeth are guiding cusps, not holding cusps.
Therefore, they do not occlude on marginal ridges or central fossae. The general rule for mandibular
lingual cusps is that they occlude in the lingual embrasures between their maxillary counterparts and
the teeth mesial to their counterparts, EXCEPT for the distolingual cusps of the mandibular molars,
which occlude in the lingual grooves of their maxillary counterparts. In this question we are not dealing
with the exception of the distolingual cusps of mandibular molars, so in this case, the mandibular third
molar mesiolingual cusp contacts the lingual embrasure between its counterpart (the maxillary third
molar) and the tooth mesial to it (the maxillary second molar).
79. Based on root morphology, the tooth most likely to be successfully rotated during exodontia is the:
A. Maxillary central
B. Mandibular central
C. Mandibular lateral
D. Mandibular canine
The correct answer is A. During extraction, rotation can be used on teeth with rounded conical roots.
Two primary examples are maxillary centrals (especially) and maxillary laterals. Rotation may also be
used somewhat with maxillary canines. If roots are flattened mesiodistally, then buccal-lingual
movement is used instead of rotation, as rotation may cause crown fracture. Mandibular centrals,
laterals, and canines (choices B, C, and D) are all flattened mesiodistally, and therefore are usually
not rotated during extraction. Mandibular canines also may occasionally have two roots (buccal and
lingual), which will prohibit rotation.
80. As compared to the cross section of the maxillary central incisor at the cervical line, the
cross section of the mandibular central is:
A. more flattened mesiodistally
B. more flattened faciolingually
C. rounder
D. almost identical

19

The correct answer is A. As a general rule, maxillary incisors have round root sections and round
pulp chamber cross sections. Mandibular incisors have more oval ones, and are flattened
mesiodistally. This is in keeping with the general root shape of each. Maxillary incisor roots are round,
and mandibular ones are flattened to a greater degree. Note that in both arches, as the pulp moves
coronally it will flatten buccolingually, in keeping with the coronal shape of all incisors.
81.The marrow space inside alveolar bone surrounding teeth:
A. does not exist
B. is usually red marrow
C. is usually hematopoietic marrow
D. is usually yellow marrow
The correct answer is D. Two types of marrow space are found within bone, red (hematopoietic),
which is the source of blood cells; and yellow (fatty), which does not produce blood cells. The alveloar
bone is similar to most bone in the body in having a compact outer layer of lamellar bone and an inner
layer of spongy bone. This spongy bone contains marrow space, usually of the yellow (fatty) type,
although some red marrow exists, especially in the ramus and condyle of the mandible.
82. In the Posselt envelope of motion, protruded contact position (PCP) is:
A. achieved with the posterior teeth in occlusion
B. an extreme border position
C. the same as edge-to-edge
D. the same as an Angle Class II relationship
The correct answer is B. The Posselt envelope of motion is a tracing of the extreme border
movements of the mandible. It shows how far the mandible can protrude, retrude, open, and close,
and all motions connecting these points. Protruded contact position is caused by protruding the
mandible as far as possible. This will normally push the mandible past edge-to-edge, until the
mandible appears to be in a Class III relationship. As the mandible protrudes, anterior tooth contact
causes separation of the teeth out of occlusion.
83. Rests of Malassez are composed of:
A. ectodermal derivatives
B. former enamel epithelium cells
C. former root sheath of Hertwig cells
D. all of the above
The correct answer is D. The epithelial rests of Malassez are found in the periodontal ligament and
are leftovers from the root formation process. The root is originally outlined by the epithelial root
sheath of Hertwig, which is composed of the inner and outer enamel epithelium tissues, without a
stellate reticulum in between. The root sheath induces formation of odontoblasts, which then produce
root dentin. Pieces of the root sheath, which do not disappear, remain as the epithelial rests of
Malassez. These small groups of tissue are ectodermal in origin, as the original enamel epithelium is
ectodermal.
84. When compared to the buccal roots of the maxillary first molar, the buccal roots of the
maxillary second molar are:
A. more parallel, with a pliers-like appearance
B. less parallel, lacking a pliers-like appearance
C. more parallel, with both having a mesial tip orientation
D. more parallel, with both having a distal tip orientation
The correct answer is D. A characteristic curve of the two buccal root tips of the maxillary first molar
toward each other is sometimes referred to as a pliers-handled appearance or pincer appearance. In
effect, the roots grow apically and then turn toward each other, forming a U shape. This shape helps
explain the stability of the tooth, especially when combined with the tripod-like arrangement of the
palatal root. Note that the same roots of the second maxillary molar often both point distally,
distinguishing one molar from the other. These roots in the second molar are also more parallel than
those in the first molar. In buccal view they seem to sway distally, with the palatal root in the midline
and slightly mesially tilted.

20

85. Which of the following permanent teeth is MOST likely to be missing a distolingual cusp?
A. Mandibular first molar
B. Maxillary first molar
C. Mandibular second premolar
D. Maxillary second molar
The correct answer is C. The mandibular first molar is invariably a five-cusped tooth, with MB, ML,
DB, DL, and distal cusps. Maxillary first and second molars are normally four-cusped teeth. The
distolingual cusp is usually less pronounced on the second molar. It is often absent on the maxillary
third molar, but that answer is not listed. The mandibular second premolar is either a two- or threecusped tooth. The two-cusped variety has a single buccal cusp and a single lingual cusp, while the
three-cusped version has a buccal cusp and paired mesiolingual and distolingual cusps. However,
both varieties are common, so it is common to find this tooth lacking the distolingual cusp.
86. The buccal cusp of the maxillary second premolar occludes in:
A. the mesial marginal ridge of the mandibular first premolar
B. the distal marginal ridge of the mandibular second premolar
C. the mesial marginal ridge of the mandibular second premolar
D. the facial embrasure between the mandibular first and second premolars
E. the facial embrasure between the mandibular second premolar and first molar
The correct answer is E. Buccal cusps of maxillary teeth are guiding cusps, not holding cusps.
Therefore, they do not occlude on marginal ridges or central fossae. The general rule for maxillary
buccal cusps is that they occlude in the facial embrasure between their mandibular counterpart and
the tooth distal to their counterpart, EXCEPT for the mesiobuccal cusps of the molars, which occlude
in the buccal grooves of their mandibular counterparts, and the distobuccal cusp of the first molar,
which opposes the distobuccal groove of the mandibular first molar. In this question, the contact is the
embrasure between second mandibular premolar (the counterpart) and the first molar (the tooth distal
to the counterpart).
87. In occlusion, guiding cusps are defined as:
A. lingual cusps of maxillary and mandibular arches
B. bucccal cusps of maxillary and mandibular arches
C. buccal cusps of the maxillary arch and lingual cusps of the mandibular arch
D. lingual cusps of the maxillary arch and buccal cusps of the mandibular arch
The correct answer is C. In occlusion, cusps are defined as either holding (supporting) cusps or
guiding cusps. Holding cusps, in central occlusion, make contact with the opposing arch, establish
vertical dimension of occlusion, and support the forces of occlusion. They are the lingual cusps of the
maxillary arch and buccal cusps of the mandibular. When the teeth are in centric occlusion, these
cusps are in contact. The other cusps (guiding) are not. The other cusps are either lingual to the
contact (lingual cusps of the mandibular arch) or buccal to the contact (buccal cusps of the maxillary
arch).
88. Protrusive movement causes the condyle of the mandible to move:
A. backward and downward
B. backward and upward
C. forward and downward
D. forward and upward
The correct answer is C. Protrusive movement is defined as moving the mandible outward, away
from the head. If you are in centric occlusion, you will protrude to move toward an edge-to-edge
position. The lateral pterygoid moves both condyles forward when it contracts, so the initial protrusive
movement is forward. When the condyle contacts the articular eminence in the glenoid fossa, it
cannot move directly forward anymore and begins to glide downward along the surface of the fossa.

21

89. As compared to those of the permanent mandibular first molar, the roots of the second
molar are:
A. more mesially inclined
B. straighter
C. more distally inclined
D. very similar in inclination to the first molar
The correct answer is C. In the permanent mandibular first molar, the mesial root is usually slightly
curved distally, and the distal root is straighter but pointing toward the distal. In the second molar, both
roots are usually significantly inclined toward the distal and curved distally at the end. They tend to
resemble each other more than the two roots of the first molar do. They are also closer together. The
roots in the first molar are more widely spread.
90. Which of the following fiber sets does NOT attach to cementum?
A. Alveolar crestal
B. Dento-gingival
C. Circular
D. Oblique
The correct answer is C. The attachment fibers are usually divided into two groups, the gingival
group and the periodontal group. Gingival fibers attach tissues, not including alveolar bone. For
example, circular fibers run only within gingiva, encircling the tooth. Dento-gingival fibers run from
cementum to gingiva. Dento-periosteal fibers run from cementum to periosteum. Periodontal fibers
run from alveolar bone to cementum, and include alveolar crestal, oblique, horizontal, and apical.
91. The mesiobuccal cusp of the maxillary second molar occludes in:
A. the buccal groove of the second mandibular molar
B. the facial embrasure between the mandibular first and second molars
C. the central groove of the mandibular second molar
D. the distal marginal ridge of the mandibular first molar
E. the buccal groove of the mandibular third molar
The correct answer is A. Buccal cusps of maxillary teeth are guiding cusps, not holding cusps.
Therefore, they do not occlude on marginal ridges or central fossae. The general rule for maxillary
buccal cusps is that they occlude in the facial embrasures between their mandibular counterparts and
the teeth distal to their counterparts, EXCEPT for the mesiobuccal cusps of the molars, which occlude
in the buccal grooves of their mandibular counterparts, and the distobuccal cusp of the first molar,
which opposes the distobuccal groove of the mandibular first molar. In this question we are dealing
with the exception of the mesiobuccal cusps of maxillary molars, which occlude with the buccal
grooves of their mandibular counterparts. In this case, the maxillary second molar mesiobuccal cusp
contacts the mandibular second molar buccal groove (its counterpart).
92. The shape of the pulp chamber in cross section cut at the cervical line in the maxillary
central incisor is:
A. oval (flattened mesiodistally)
B. oval (flattened bucco-lingually)
C. round
D. figure eight
The correct answer is C. Only maxillary incisors have roundish pulp chamber cross sections when
cut at the cervical line. Laterals are egg-shaped with the widest portion labial, not round. This is true
for both maxillary centrals and laterals, which are egg-shaped with the widest portion of the labial not
round. In general, mandibular incisors and both arch canines will have oval-shaped pulp chambers,
flattened mesiodistally. (Imagine squashing a round pulp from both the mesial and distal sides at
once, resulting in a squashed oval, pointing to the buccal and lingual). Remember that the shape of a
pulp chamber is usually related to the shape of the surrounding root.

22

93. The appearance of a second root in a canine is:


A. never found
B. found more often in the maxillary than the mandibular
C. found more often in the mandibular than the maxillary
D. found about equally in both arches
The correct answer is C. Two roots are a fairly rare occurrence in canines, but are occasionally
found in the mandibular canine. They are almost unknown in the maxillary. If they are found in the
mandibular canine, they will be buccal and lingual, similar to those found in premolars.
94. The submucosa of gingival masticatory mucosa is:
A. similar in thickness to that of lining mucosa
B. much thicker than that of lining mucosa
C. found between the epithelium and lamina propria
D. thin or absent
The correct answer is D. The gingival masticatory mucosa is characterized by keratinization, thin or
absent submucosa, and a firm, direct binding of the lamina propria to the periosteum. In general, the
submucosa is thicker in the lining mucosa. Note that submucosa, when present, is always found
beneath lamina propria, and is never between epithelium and lamina propria. If all four tissues are
present in a digestive mucosa, the order from the outside will be: epithelium, lamina propria,
muscularis mucosae, submucosa.
95. In centric occlusion, the distolingual cusp of the maxillary third molar will contact:
A. the central fossa of the mandibular second molar
B. the central fossa of the mandibular third molar
C. the distal marginal ridge of the mandibular first molar and mesial marginal ridge of second molar
D. the distal marginal ridge of the mandibular second molar and mesial marginal ridge of first molar
E. None of the above
The correct answer is E. The distolingual cusp of a maxillary third molar is theoretically a holding
cusp, however it is often missing on this tooth. The general rule for maxillary holding cusps is as
follows: a maxillary holding cusp contacts the distal marginal ridge of its mandibular counterpart and
the mesial marginal ridge of the mandibular tooth distal to its counterpart, EXCEPT FOR THE
MESIOLINGUAL CUSPS OF THE MOLARS, WHICH CONTACT THE CENTRAL FOSSAE OF
THEIR COUNTERPARTS. This should be the distal marginal ridge of the mandibular third molar and
no mesial marginal ridge, because there is no tooth distal to the third molar. So the answer is either
the cusp does not exist, or it contacts only the distal marginal ridge of the mandibular third molar.
96. Collagen fibers inserting into compact bone of the alveolar socket are called:
A. Tomes processes
B. Sharpey's fibers
C. Von Ebner lines
D. Contour lines of Owen
E. None of the above
The correct answer is B. The fibers of the periodontal ligament are collagenous and insert into the
compact, lamellar outer layer of the tooth socket. This is similar to fibers of a tendon inserting into
bone. The appearance of the bone with the collagen fibers causes it to be named bundle bone. The
fibers themselves are called Sharpey's fibers. Tomes processes (choice A) are projections of
ameloblasts into developing enamel. Von Ebner lines (choice C) are found in dentin and show the
incremental laying down of dentin tissue. Lines of Owen (choice D) are found in dentin as well, and
are exaggerated Von Ebner lines found in areas of trauma during dentin formation.
97. The most numerous cell type of the dental pulp is the:
A. neuron
B. fibroblast
C. leukocyte
D. macrophage
E. ameloblast

23

The correct answer is B. The dental pulp is a connective tissue characterized by multiple collagen
fibers running in all directions, and large numbers of fibroblasts, which produce the fibers. There are
also capillaries, neurons, lymphatic channels, and different types of leukocytes, including
macrophages, neutrophils, and eosinophils. However, the predominant cell type is the fibroblast. Note
that the pulp is lined by the dentin-forming odontoblasts, but there are no ameloblasts, which form
enamel.
98. Which two tissues are formed by the same part of the tooth bud?
A. Enamel and dentin
B. Dentin and cementum
C. Dentin and pulp
D. Pulp and cementum
E. None of the above
The correct answer is C. The three sections of the tooth bud are the enamel organ, the dental sac,
and the dental papilla. The enamel organ, not surprisingly, produces enamel and contains the inner
enamel epithelium, stratum intermedium, stellate reticulum, and outer enamel epithelium. The dental
papilla, a mesodermal derivative, produces both dental pulp and dentin. The dental sac, also
mesodermal, produces both cementum and the periodontal ligament.
99. Which ligament of the temporomandibular joint inserts into the lingula of the mandible?
A. Temporomandibular
B. Stylomandibular
C. Sphenomandibular
D. Lateral
The correct answer is C. The origins and insertions of the TMJ ligaments are as follows: The
sphenomandibular ligament arises from a spine on the sphenoid bone and runs forward and
downward to insert on the lingula and deep ramus of the mandible. The stylomandibular ligament
arises from the spine of the temporal bone known as the styloid process and inserts on the lower
ramus and angle of the mandible. The lateral ligament is also known as the temporomandibular
ligament. It descends from the lower border and tubercle of the zygoma to the posterior lateral
condyle. Its fibers merge with those of the articular capsule.
100. The most common arrangement of canals in the roots of a permanent maxi. first molar is:
A. ML, MB, DL, DB
B. ML, MB, and DB only
C. ML, MB, and palatal
D. MB, DB, and palatal
E. none of the above
The correct answer is D. Most often, the maxillary first molar has three canals: a mesiobuccal, a
distobuccal, and a palatal. The palatal is usually largest and widest. Occasionally (30%) the first molar
may have a fourth canal, located within the mesiobuccal root, slightly lingual to the mesiobuccal canal.
When found, it is the smallest and thinnest of all canals in the tooth.
101. Alveolar bone is composed of:
A. cortical bone only
B. cancellous bone only
C. cortical bone surrounding cancellous bone
D. cancellous bone surrounding cortical bone
The correct answer is C. Alveolar bone is a combination of cortical (compact) bone and trabecular
(spongy) bone. The cortical bone is on the outside surface, both next to the tooth root and also
continuous with the cortical bone of the maxilla and mandible. The cortical bone adjacent to the tooth
root is sometimes referred to as lamina dura. Nerves and blood vessels travel primarily through the
more porous cancellous bone between the two cortical plates. Note that alveolar bone can be lost
rapidly either in periodontal disease or following tooth extraction.

24

102. When you move the mandible from centric occlusion (maximum intercuspation) to
protruded contact (edge-to-edge) position:
A. vertical dimension decreases
B. horizontal overlap increases
C. vertical overlap increases
D. None of the above
The correct answer is D. As you move from centric occlusion to edge-to-edge position, the
mandibular teeth separate from the maxillary and a space is created between the two arches. Try this
yourself, as you protrude and slide the mandible forward. The anterior teeth act as a guide as you
slowly separate from occlusal contact until only incisal edges of incisors contact. At this point, the
increased space between the arches adds to vertical dimension. There is no vertical overlap (overbite;
choice C) nor is there any horizontal overlap (overjet; choice B) at this point. So protrusion to edge-toedge increases vertical dimension while decreasing both vertical and horizontal overlap.
103. From a labial view, which incisors are bilaterally symmetrical?
A. Both mandibular central and lateral
B. Neither mandibular central nor lateral
C. Mandibular central only
D. Mandibular lateral only
The correct answer is C. The mandibular central incisor is often described as the most symmetrical
tooth when viewed from the labial. The incisal edge is horizontal, the mesial and distal contacts are at
the same height, and the mesioincisal and distoincisal line angles are equal. In addition, the tooth is
not rotated. The mandibular lateral incisor crown is rotated, and its distal contact point is more apical
than the mesial.
104. The Golgi apparatus in odontoblasts producing dentin can be expected to be:
A. absent
B. less developed than in an average cell
C. developed to the same degree as that of an average cell
D. more highly developed than that of an average cell
The correct answer is D. Remember that the function of Golgi apparatus is to modify and package
proteins produced for export by the cell. In this way, an odontoblast would be similar to any other
protein-producing secretory cell. Proteins produced by ribosomes on the rough ER will enter the ER
and travel to the Golgi apparatus. Here, within the flattened sacs of the Golgi, the collagen and other
fibers will be modified and packaged to be sent to the edge of the cell for secretion as dentin matrix.
So the large production demands of producing collagen and other proteins for predentin should result
in odontoblasts having large, numerous, and active Golgi bodies.
105. The longest tooth in the mouth (apico-incisally) is the:
A. Maxillary central
B. Maxillary canine
C. Mandibular canine
D. Maxillary second premolar
The correct answer is B. This is one of the biggest, shortest, roundest, flattest type questions that
NBDE Part 1 loves so much. The maxillary canine is the longest tooth in the mouth, measured cusp
tip to root tip. It is about 3 mm longer than the maxillary central (choice A), 1 mm longer than the
mandibular canine (choice C), and 4 mm longer than the maxillary second premolar (choice D). Note
that the mandibular central is the narrowest tooth in the mouth.
106. The least likely area to find caries on teeth is the:
A. lingual surface of maxillary molars
B. occlusal surface of mandibular molars
C. lingual surface of mandibular molars
D. buccal surface of mandibular molars
E. occlusal surface of mandibular premolars

25

The correct answer is C. Caries is most commonly found in grooves, pits, and fissures of teeth.
Sucrose and other substrates collect and S. mutans secretes lactic acid to dissolve enamel. All
posterior occlusal surfaces are susceptible to caries, as they contain pits, fissures, and grooves.
Maxillary molars have distinct lingual pits or grooves which may become carious. Likewise, buccal pits
are common on mandibular molars. Note that lingual surfaces of mandibular molars are generally
smooth, without pits and fissures.
107. When extracting maxillary teeth, the root tip most likely to be forced into the maxillary
sinus is that of a:
A. canine
B. first premolar
C. second premolar
D. first molar
E. second molar
The correct answer is D. Anatomically, roots of the maxillary first molar are closest to the sinus,
which dips downward in this area. Periapical x-rays of the area sometimes reveal that the roots of the
first molar border right on the sinus margin. Although the first molar is the MOST likely tooth to have a
root enter the sinus, roots of the second premolar and second molar can occasionally end up there as
well.
108. The class of teeth considered to have a long axis which is most vertical in a buccolingual
direction (as viewed from the mesial or distal) is the:
A. incisors
B. canines
C. premolars
D. molars
The correct answer is C. Premolars do not vary far from a vertical line when viewed from the mesial
or distal. Both maxillary and mandibular incisors tip their roots greatly toward the lingual, and canines
less so. Mandibular molar roots lean buccally while maxillary molar roots lean palatally.
109. The lingual surfaces of most maxillary teeth of a patient are worn down so that dentin is
visible through the remaining enamel on the lingual side. This is due to the dissolving of the
mineral of the enamel by strong acid. This loss of tooth structure can be termed:
A. attrition
B. abrasion
C. erosion
D. internal resorption
The correct answer is C. Erosion is the chemical dissolving of tooth structure. It can be caused by a
number of factors, including sucking on lemons and other acidic fruit, excessive intake of acidic
beverages (cola), or excessive vomiting, as in bulimia. Bulimics often exhibit normal facial surfaces
with severely eroded lingual surfaces. This is due to the effect of strong mineral acid from the stomach
(hydrochloric acid at pH 2). Attrition (choice A) is the mechanical wearing of teeth due to physiologic
processes, including chewing and bruxism. Abrasion (choice B) is the mechanical wearing away of
tooth structure due to some outside object, such as toothbrush abrasion at the cervical lines of teeth
due to hard sideways brushing, or from habitually holding bobby pins or nails with the teeth. Internal
resorption (choice D) is the loss of dentin from pathologic dissolving by processes inside the pulp
chamber. It can follow tooth trauma.
110. The formation of dual teeth with combined enamel and dentin crowns but with only one
root is known as:
A. gemination
B. dilaceration
C. concrescence
D. dens-in-dente
E. taurodont

26

The correct answer is A. Gemination is a form of fusion, where a single root attempts to form two
crowns, and these crowns share dentin and enamel. A dilaceration is a sharply bent root, especially
near the apex. Concrescence is joining of two teeth by cementum union. Taurodonts are teeth,
usually molars, with short roots relative to crown size, and large pulpal chambers, giving rise to a bulllike appearance. Dens-in-dente refers to a tooth-within-a-tooth appearance and is due to invagination,
usually in a maxillary lateral incisor.
111. The buccal cusp of the mandibular second premolar occludes with which maxillary tooth
surfaces?
A. The mesial marginal ridge of the first molar and distal marginal ridge of the second premolar
B. The mesial marginal ridge of the second premolar and distal marginal ridge of the first premolar
C. The embrasure between the first and second molars
D. The embrasure between the second premolar and first molar
The correct answer is B. The buccal cusp of a mandibular premolar is a holding (supporting) cusp.
The general rule of occlusion of mandibular holding cusps is as follows: the holding cusps of the
mandibular teeth occlude on the mesial marginal ridge of their maxillary counterpart, and the distal
marginal ridge of the maxillary tooth mesial to their counterpart, EXCEPTdistobuccal cusps of
mandibular molars occlude with central fossae of their counterparts, the distal cusp of the mandibular
first molar occludes with the distal triangular fossa of its counterpart, and the first premolar occludes
only with the mesial marginal ridge of its counterpart (but not the canine). In this case, the maxillary
counterpart is the maxillary second premolar, and the tooth mesial to it is the first premolar.
112. In cases of TMJ dislodgement, where the condyle becomes locked due to movement
beyond the articular (temporal) fossa, the dentist should move the condyle in which direction
initially to return the joint to its normal position?
A. Anteriorly
B. Posteriorly
C. Inferiorly
D. Superiorly
The correct answer is C. This may seem counterintuitive, as excess motion of the condyle results in
the mandible being locked in a forward and open position. However, anatomically, what is happening
is that the condyle has moved forward, down the surface of the articular eminence, past the tip of the
eminence, and past the bulk of the articular tubercle. It is now past a large, inferiorly projecting mound
of bone (the tubercle). To return the condyle to normal position, a downward (inferior) force is needed
to positon the condyle once again below the articular tubercle and eminence. The inferior directed
force is also needed to counteract the elevating forces of the medial pterygoid, masseter, and
temporalis muscles. On repositioning, the mandible often snaps strongly into normal position, due to
these strong elevations.
113. Which pair of angles of a maxillary first molar are acute when the tooth is viewed from
the occlusal?
A. MB and ML
B. DB and DL
C. MB and DB
D. ML and DL
E. None of the above
The correct answer is E. If you view the maxillary first molar from the occlusal, its shape is that of a
rhombus, or equal-sided parallelogram. In any parallelogram, one pair of opposite corners will be
obtuse, while the other pair of opposite corners will be acute. In the maxillary first molar, the opposite
pair of MB and DL are acute, while the pair ML and DB are obtuse.
114. Which characteristic most distinguishes permanent mandibular central incisors from
lateral incisors?
A. Mesiodistal length
B. Buccolingual length
C. Root length
D. Rotation of the crown around the root axis
E. None of the above

27

The correct answer is D. The mandibular central and lateral incisors can be viewed as almost twins.
They are similar in dimension both mesiodistally and buccolingually. (The lateral may be slightly, but
not significantly, larger). Their roots are similar in length, although the lateral may have a more
pronounced distal inclination. The crowns are of similar anatomy. However, the lateral has a twisted
or rotated crown, as if you took a central and then slightly rotated the crown around the long axis of
the root. The root may appear to face forward, while the incisal edge is curved in keeping with the
normal curve of cusp tips of the mandibular arch.
115. The lingual cusp of the mandibular first premolar contacts:
A. the mesial marginal ridge of the maxillary first premolar
B. the distal marginal ridge of the maxillary second premolar
C. the central fossa of the maxillary second premolar
D. the lingual embrasure between the maxillary first and second premolars
E. the lingual embrasure between the maxillary first premolar and canine
The correct answer is E. Lingual cusps of mandibular teeth are guiding cusps, not holding cusps.
Therefore, they do not occlude on marginal ridges or central fossae. The general rule for mandibular
lingual cusps is that they occlude in the lingual embrasures between their maxillary counterparts and
the teeth mesial to their counterparts, EXCEPT for the distolingual cusps of the mandibular molars,
which occlude in the lingual grooves of their maxillary counterparts. In this question we are not dealing
with the exception of the distolingual cusps of mandibular molars, so in this case, the mandibular first
premolar lingual cusp contacts the lingual embrasure between its counterpart (the maxillary first
premolar) and the tooth mesial to it (the maxillary canine).
116. Alveolar bone is composed of:
A. compact bone
B. spongy bone
C. bundle bone
D. all of the above
E. none of the above
The correct answer is D. The alveloar bone is similar to most bone in the body in having a compact
outer layer of lamellar bone and an inner layer of spongy bone. This spongy bone contains marrow
space, usually of the yellow (fatty) type, although some red marrow exists. Collagen fibers of the
periodontal ligament insert into the outer cortical compact layer, and are known as Sharpey's fibers.
Compact bone with Sharpey's fibers is also known as bundle bone.
117. The TMJ structure consists of:
A. One synovial compartment
B. An upper synovial and lower nonsynovial compartment
C. Two synovial compartments
D. Two nonsynovial compartments
The correct answer is C. The TMJ has an upper and lower compartment, separated by the articular
disc, which is composed of fibrous connective tissue. Both the upper and lower compartments are
synovial; that is, they produce synovial fluid. Synovial fluid aids in lubricating the articular surfaces of
the joint, and is found in many movable joints. Note that the synovial membrane lines the inside of the
joint but not the actual articulating surfaces. These surfaces are fibrous connective tissue over hyaline
cartilage.
118. Odontoblasts are noted for highly developed:
A. smooth ER
B. rough ER
C. lysosomes
D. mitochondria
E. flagella
The correct answer is B. The odontoblast function is to lay down the protein matrix for the formation
of dentin. Protein matrix is an exported protein, and therefore the cell must have highly developed

28

machinery for production and export of protein. This usually involves ribosomes attached to ER (rough
ER) and Golgi bodies. The ribosomes translate RNA messages into assembled proteins and send
them in the ER to the Golgi apparatus, where they are modified and packaged for export.
Odontoblasts do not have flagella, and there is no special reason to have highly developed lysosomes
for internal digestion. As metabolically active cells they would have elevated mitochondria levels, but
this would not be as notable as the extensive protein producing apparatus.
119.When primary mandibular incisors are retained too long and the permanent incisors erupt
with the primaries still in place, the permanents usually erupt in what position relative to the
primaries?
A. Mesially
B. Distally
C. Lingually
D. Buccally
The correct answer is C. This is a fairly common occurrence. A child of about 6 years will often
present to the dental clinic with a parent, and the parent will be concerned about a double row of
teeth. In these cases, the permanent mandibular incisors are erupting before the primaries have
completely exfoliated. An x-ray usually confirms partial resorption of the primary roots, and the
primaries are allowed to exfoliate. No further treatment is usually indicated. In rare cases where the
permanents erupt buccally, extraction of the primaries and tooth movement of the permanents is in
order, however this is highly unusual.
120. Which of the following is NOT typical of masticatory mucosa?
A. Simple squamous epithelium
B. Highly keratinized
C. Poorly developed submucosa
D. Thick epithelium
E. Covers the hard palate
The correct answer is A. Both types of oral epithelium (masticatory and lining) are stratified, not
simple, squamous epithelium. In masticatory epithelium it is highly keratinized and thickened. This
type of mucosa covers the hard palate and gingival areas. The submucosa of these tissues is usually
poorly developed or absent. Lining mucosa is generally thin and nonkeratinized, and has a glandular
lamina propria and a well developed submucosa.
121. The buccal cusp of the mandibular first premolar contacts which surfaces on maxillary
teeth?
A. The mesial marginal ridge of the first premolar and distal marginal ridge of the canine
B. The mesial marginal ridge of the second premolar and distal marginal ridge of the first premolar
C. The distal marginal ridge of the first premolar and the mesial marginal ridge of the canine
D. None of the above
The correct answer is D. While theoretically it may appear that the correct answer is choice A, most
authors state that there is no contact with the maxillary canine distal marginal ridge, and the canine
remains slightly out of contact but near the area of the buccal cusp of the mandibular first premolar.
The general rule of occlusion of mandibular holding cusps is as follows: the holding cusps of the
mandibular teeth occlude on the mesial marginal ridge of their maxillary counterpart, and the distal
marginal ridge of the maxillary tooth mesial to their counterpart, EXCEPT DISTOBUCCAL CUSPS OF
MANDIBULAR MOLARS OCCLUDE WITH CENTRAL FOSSAE OF THEIR COUNTERPARTS, THE
DISTAL CUSP OF THE MANDIBULAR FIRST MOLAR OCCLUDES WITH THE DISTAL
TRIANGULAR FOSSA OF ITS COUNTERPART, AND THE FIRST PREMOLAR OCCLUDES ONLY
WITH THE MESIAL MARGINAL RIDGE OF ITS COUNTERPART (BUT NOT THE CANINE).
122. The buccal cusp of the mandibular canine occludes with which maxillary tooth surfaces?
A. The mesial marginal ridge of the first premolar and distal marginal ridge of the canine
B. The mesial marginal ridge of the canine and distal marginal ridge of the lateral incisor
C. The embrasure between the canine and first premolar
D. None of the above

29

The correct answer is D. The buccal cusp of a mandibular canine is not considered to be a holding
(supporting) cusp. Therefore, the general rule of occlusion of mandibular holding cusps does not
apply. The canine, as an anterior tooth, will normally be slightly out of contact between its maxillary
counterpart and the tooth immediately mesial to it. In this case, the maxillary counterpart is the
maxillary canine, and the tooth mesial to it is the lateral incisor.
123. The lamina propria is:
A. epithelial tissue
B. connective tissue
C. adipose tissue
D. muscle tissue
The correct answer is B. The lamina propria is a loose connective tissue located within the mucosal
layer, just underneath the epithelium. In the oral cavity, it will be found below the outer layer of
stratified squamous epithelium. The lamina propria in the oral cavity often forms wedge-like
extensions into concavities in the epithelium, known as connective tissue papillae. The corresponding
epithelial extensions into the connective tissue are known as rete pegs.
124. Which of the following is not normally found in cases of occlusal trauma?
A. Resorption of alveolar bone
B. Increased mobility
C. Thicker cementum (hypercementosis)
D. Widened periodontal ligament space
The correct answer is C. Increased cementum production (hypercementosis) is a common x-ray
finding. Its cause is not completely known, but it seems to occur more often in teeth that have lost
function and/or are supererupted, rather than in teeth with excessive function (occlusal trauma).
Occlusal trauma is recognized clinically by heavy contact (using articulating paper), and high mobility.
It is recognized radiographically by a widened PDL space and possibly with accompanying bone
resorption. Note that in loss of function, teeth often have a narrowed PDL.
125. The earliest succedaneous tooth in the mouth to erupt is the:
A. Primary mandibular central incisor
B. Primary mandibular lateral incisor
C. Permanent mandibular first molar
D. Permanent maxillary first molar
E. Permanent mandibular central incisor
The correct answer is E. The question depends entirely on the definition of succedaneous. A
succedaneous tooth succeeds (replaces) another tooth. So only permanent teeth can be
succedaneous. Note, however, that permanent molars are never succedaneous, as they erupt distal
to all primary teeth (choices C and D). Only permanent incisors, canines, and premolars can be
succedaneous (refer to choices A and B). Of all succedaneous teeth, the permanent mandibular
central incisor is usually first, appearing at about age 6, close to the eruption of the nonsuccedaneous
permanent mandibular first molar.
126. The time between eruption of a tooth crown and root completion is often closest to:
A. 6 months
B. 1 year
C. 2-3 years
D. 4-5 years
E. 10 years or more
The correct answer is C. Although the time varies, 2 to 3 years is a common average time.
Remember that as a crown erupts, the root is still forming, and x-rays of newly erupted teeth show
varying levels of root formation. This has important implications in endodontics, when we may try to
cause a tooth to complete its root formation in order to better manage it for restoration. A few
examples of typical time lags are: maxillary central incisor, eruption 7-8 years, root completion 10
years; maxillary canine, eruption 11-12 years, root completion 14 years; mandibular second molar,
eruption 11-13 years, root completion 14-15 years.

30

127. A mandibular permanent first molar is more likely to have a partially bifurcated:
A. mesial root
B. distal root
C. both roots are equally likely to be bifurcated
D. neither root is bifurcated
The correct answer is A. One of the distinctions between the mandibular first-molar mesial and distal
roots is the deep root concavity seen running the length of the mesial surface of the mesial root. The
distal root, by comparison, will have no similar concavity or a very slight depression. This concavity
will sometimes express itself as a partial split (bifurcation) of some small section of the apical end of
the mesial root into two roots. This feature is rare to unknown on the distal root. Other differences
include the greater likelihood of the mesial root having a pronounced distal curvature.
128. The mesial and distal heights of contour of the maxillary canine are, respectively:
A. incisal third, incisal third
B. junction of incisal/ middle third, junction of incisal/middle third
C. junction of incisal/middle third, middle third
D. middle third, middle third
E. middle third, cervical third
The correct answer is C. One way to eliminate answers is to note that the labial view of the maxillary
canine is not symmetrical. The mesial cusp ridge is shorter than the distal cusp ridge. This means that
the distance from the cusp tip to the mesial contact is shorter than that from the cusp tip to the distal
contact. Another way to look at it is that the distal cusp ridge dips lower (more cervical). Therefore, the
distal height of contour will be more cervical than the mesial. The mesial height of contour is at about
the junction of incisal and middle thirds, while the distal is at the middle of the middle third. Do not
consider choice E, even though it follows the pattern discussed above. These contacts (middle,
cervical) are too cervical for an anterior tooth.
129. Sharpey's fibers in the periodontal ligament:
A. enter only alveolar bone
B. enter only cementum
C. are collagenous in composition
D. enter cancellous bone to form bundle bone
The correct answer is C. Sharpey's fibers is an older name for the collagenous bundles of the
periodontal ligament which connect the tooth to the tooth socket. The two tissues entered by these
fibers are cementum on the tooth, and the cortical bone plate of the alveolar bone. When they enter
the cortical plate, the resulting bone, when viewed under the microscope, is termed bundle bone. The
fibers do not penetrate the bone enough to reach the cancellous inner layers of the alveolar bone.
130. The most common arrangement of canals in the roots of a permanent mandibular first
molar is:
A. two mesial, two distal
B. two mesial, one distal
C. one mesial, two distal
D. one mesial, one distal
E. none of the above
The correct answer is B. Although some variation exists, the most common arrangement of canals
in the mandibular first molar is two canals in the mesial root (MB and ML), and one in the distal. The
largest is normally the distal, followed by the MB, then the ML. Approximately 25% of mandibular first
molars will have four canals: MB, ML, DB, and DL.
131. The lingual fossa is normally deepest on which incisor?
A. Maxillary central
B. Maxillary lateral
C. Mandibular central
D. Mandibular lateral
The correct answer is B. The maxillary lateral is noted for its variability, and one area of great variety
is the lingual fossa. It is normally more pronounced than that of any other incisor and often contains a
pit, from narrow and shallow to deep and extensive. Extremely deep pits may give a dens-in-dente

31

appearance. In operative dentistry the maxillary lateral is the only incisor in which you will occasionally
see lingual pit amalgams. The lingual surface is often marked by a lingual marginal groove and
tubercles on the cingulum. In general, expect any type of unusual variety to be most commonly found
on maxillary laterals.
132. The buccal cusp of the maxillary first premolar occludes in:
A. the mesial marginal ridge of the mandibular first premolar
B. the distal marginal ridge of the mandibular second premolar
C. the mesial marginal ridge of the mandibular second premolar
D. the facial embrasure between the mandibular first and second premolars
E. the facial embrasure between the mandibular first premolar and canine
The correct answer is D. Buccal cusps of maxillary teeth are guiding cusps, not holding cusps.
Therefore, they do not occlude on marginal ridges or central fossae. The general rule for maxillary
buccal cusps is that they occlude in the facial embrasure between their mandibular counterpart and
the tooth distal to their counterpart, EXCEPT for the mesiobuccal cusps of the molars, which occlude
in the buccal groove of their mandibular counterpart, and the distobuccal cusp of the first molar, which
opposes the distobuccal groove of the mandibular first molar. In this question, the contact is the
embrasure between first and second mandibular premolars.
133. In restoring a mandibular first molar, lingual cusps are important for which movement?
A. Centric occlusion
B. Protrusive
C. Retrusive
D. Working
E. Nonworking
The correct answer is D. To answer this question, place your own teeth in centric occlusion
(maximum intercuspation). Notice that the occlusion is held in place by the lingual cusps of maxillary
teeth and the buccal cusps of mandibular teeth. Mandibular lingual cusps are not in contact. In
protrusion, the anterior teeth act to separate the maxilla and mandible as the mandible moves
forward. The anterior teeth act as guides in retrusion as well (try it). So far the mandibular lingual
cusps are not involved. If you make a left working movement (slide your mandible left), notice that the
mandibular lingual cusps on the left, drag against the lingual side of the maxillary lingual cusps. In the
nonworking side (in this case, the right side), the mandibular lingual cusps move AWAY from contact.
134. The mucosa found on the surface of the hard palate is known as:
A. lining mucosa
B. masticatory mucosa
C. specialized mucosa
D. none of the above
The correct answer is B. Masticatory mucosa consists of the gingiva and lining of the hard palate.
Most of the other areas of the oral cavity are lined by lining mucosa. Specialized mucosa is found on
the dorsum of the tongue and contains taste buds and various papillae. The characteristics of
masticatory mucosa are keratinization, thin or absent submucosa, and tight binding of the lamina
propria to underlying periosteum. Lining mucosa is generally nonkeratinized, with a thin lamina
propria. Near the teeth, the gingiva and lining mucosa (alveolar mucosa) meet at the mucogingival
junction.
135. Which primary teeth MOST differ from their permanent counterparts?
A. Incisors
B. Canines
C. First molars
D. Second molars
The correct answer is C. For the most part, despite an overall size difference and some differences
in ratio (crown:root, crown height:mesiodistal length), the primary teeth generally resemble the

32

permanents for all teeth EXCEPT the first molars. Primary central and lateral incisors and canines are
fairly close to small versions of the permanents. Primary second molars very closely resemble
permanent first molars. However, primary first molars are unique. The primary maxillary first is
premolar-like in crown form but possesses three roots. The primary mandibular molar is molar-like in
form, but with an unusually pronounced mesial section and a much less developed distal section. It
does not resemble any permanent tooth.
136. The premolar with the buccal cusp most distally placed relative to the lingual is the:
A. maxillary first
B. maxillary second
C. mandibular first
D. mandibular second
The correct answer is A. An unusual aspect of the maxillary first premolar is the mesial-distal
placement of the cusps. The buccal cusp is off center toward the distal, while the lingual cusp is off
center toward the mesial. If you look at the tooth from the lingual, therefore, you will always see two
distinct cusp tips, one slightly to the mesial or distal of the other. This placement, when viewed
occlusally, is said to give the tooth a twisted appearance. This situation is not found in the second
premolar, where the cusps are almost equal in size and straight in alignment.
137. Thin organic structures which extend from enamel at the DEJ into the dentin r known as:
A. enamel spindles
B. enamel tufts
C. enamel lamellae
D. none of the above
The correct answer is C. There are a number of somewhat confusing anomalies at the
dentinoenamel junction. Enamel spindles are odontoblastic processes and dentinal tubules which
extend a short way into enamel. Enamel lamellae, on the other hand, extend from enamel into dentin.
They are pieces of uncalcified organic enamel material. Enamel tufts are hypocalcified enamel rods.
They are individual entities; however, they appear as groups (tufts) when a group of them are viewed
laterally.
138. In the oral cavity, red marrow is found:
A. in all spongy bone of tooth sockets
B. in the condyle
C. in the lamellar layers
D. in most of the hard palate
The correct answer is B. Two types of marrow space are found within bone: red (hematopoietic),
which is the source of blood cells; and yellow (fatty), which does not produce blood cells. The alveloar
bone is similar to most bone in the body in having a compact outer layer of lamellar bone and an inner
layer of spongy bone. This spongy bone contains marrow space, usually of the yellow (fatty) type,
although some red marrow exists. The chief places where red marrow is found in the oral cavity are
the condyle, the angle of the mandible, and the maxillary tuberosity.
139. Which tooth is LEAST likely to have a divided pulp canal?
A. Mandibular canine
B. Maxillary first premolar
C. Maxillary central incisor
D. Maxillary lateral incisor
The correct answer is C. Some teeth almost invariably have a divided pulp canal, such as the
maxillary first premolar. This tooth normally has two roots, although the level at which the division
occurs can vary. The other three teeth listed are normally single rooted with single undivided pulp
canals. However the maxillary central incisor almost never varies from a single rounded root. In
contrast, you can sometimes, although rarely, find divided roots, especially near the apex, for the
mandibular lateral and canine.

33

140. The distal contact point and height of contour of the maxillary lateral incisor is located:
A. at the incisal edge
B. in the incisal third
C. in the middle third
D. in the cervical third
The correct answer is C. As is generally the case, the distal contact of this tooth is more cervical
than is the mesial contact. On the mesial side, where it contacts the central incisor, the contact will be
near the junction of incisal and middle thirds. However, on the distal side, it is slightly more cervical,
usually around the middle of the middle third. This is the contact with the maxillary canine. On the
canine, this height of contour is at the junction of the incisal and middle third.
141. The distobuccal cusp of the maxillary first molar occludes in:
A. the mesial marginal ridge of the mandibular second molar
B. the distal marginal ridge of the mandibular first molar
C. the central groove of the mandibular first molar
D. the facial embrasure between the mandibular first and second molars
E. the distobuccal groove of the first mandibular molar
The correct answer is E. Buccal cusps of maxillary teeth are guiding cusps, not holding cusps.
Therefore, they do not occlude on marginal ridges or central fossae. The general rule for maxillary
buccal cusps is that they occlude in the facial embrasures between their mandibular counterparts and
the teeth distal to their counterparts, EXCEPT for the mesiobuccal cusps of the molars, which occlude
in the buccal grooves of their mandibular counterparts, and the distobuccal cusp of the first molar,
which opposes the distobuccal groove of the mandibular first molar. In this question we are dealing
with the exception of the distobuccal cusps of the maxillary first molar, which occludes with the
distobuccal groove of its mandibular counterpart, the mandibular first molar.
142. Which teeth exhibit isomorphy with each other?
A. Primary maxillary first and second molars
B. Primary maxillary second and primary mandibular second molars
C. Primary mandibular first molars and primary maxillary first molars
D. Primary maxillary second molars and permanent maxillary first molars
E. None of the above
The correct answer is D. Isomorphy refers to a close similarity in morphology and crown shape
between two teeth. The most notable examples of isomorphy are the primary second molars with the
permanent first molars IN BOTH ARCHES. Thus, by looking at a primary second molar in either arch,
you can very accurately predict the morphology of the permanent first molar in that same arch.
Isomorphy does not exist between primary first and second molars, as both first molars are very
unusual in shape and both second molars closely resemble permanent first molars.
143. Synovial fluid-producing tissue in the TMJ:
A. is found throughout all surfaces
B. is not found within the joint
C. is found only on articular surfaces
D. is found only on non-articular surfaces
The correct answer is D. The TMJ is a synovial joint, similar to most other movable joints. It is
different in having a fibrous articular disc. The surface of the disc is fibrous connective tissue, and the
surface of the articulating surfaces of the glenoid fossa are fibrous connective tissue overlying hyaline
cartilage. These surfaces are not covered with synovial tissue; however, their smooth functioning is
dependent on the production of synovial fluid elsewhere in the joint. This production occurs in all nonarticulating surfaces of the joint, which are lined with synovial tissue.
144. The primary maxillary first molar is often said to have a premolar shaped crown. This
tooth will invariably have how many roots?
A. One
B. Two
C. Three
D. Four

34

The correct answer is C. Despite a somewhat premolar-shaped crown, the primary maxillary first
molar is consistent in root form with all maxillary molars, primary or permanent, in having three roots.
They are the mesiobuccal, distobuccal, and palatal. Premolars will routinely have either one or two
roots. Primary molars lack a common root trunk, and have widespread roots which allow space for the
developing premolar crown underneath.
145. Mesial-occlusal-distal (MOD) cavity preparations are most difficult in which of the
following teeth?
A. Permanent mandibular first premolar
B. Permanent maxillary second molar
C. Primary mandibular second molar
D. Primary mandibular first molar
The correct answer is D. MOD preparations can be difficult because tooth structure must be
removed from three tooth surfaces. The remaining tooth structure can be weak, and pulp horns may
be exposed. They are especially hard in small teeth with large pulp horns. The primary mandibular
first molar is a good example. It has a very narrow distal end, is small overall, and has very high
mesial pulp horns. Additionally, little tooth structure remains after the MOD is cut. Permanent
mandibular first premolars are small, but not like the primaries. They should have a slanted occlusal
floor, but an MOD is possible. Permanent maxillary second molars are generally large and would be
the best of this group for making an MOD preparation. The primary mandibular second molar, while
small, is larger than the first molar, and has the same shape as a permanent first molar, making it not
as difficult to perform an MOD preparation.
146. Which cell type of the enamel organ is widely separated by liquid between the cells?
A. Outer enamel epithelium
B. Stellate reticulum
C. Stratum intermedium
D. Inner enamel epithelium
The correct answer is B. Both the inner and outer enamel epithelium are solid lines of cells with no
space between the cells in each line. Between these two layers of cells we find the stratum
intermedium, close to the inner enamel epithelium; and the stellate reticulum, filling the space
between the stratum intermedium and outer enamel epithelium. The stratum intermedium is generally
a few layers of squamous epithelium. The stellate reticulum is a reticulum (net-like network) of widely
separated cells (think of stars in the sky). These cells are surrounded by a clear liquid matrix and are
connected by cell processes.
147. The primary second maxillary molar is usually exfoliated between which ages?
A. 4-5 years
B. 6-7 years
C. 8-9 years
D. 10-11 years
E. 12-13 years
The correct answer is choice D. Eruption and exfoliation questions are common NBDE Part 1
topics. Answers are approximate, of course, and subject to variation. However, you should know the
most common average ages for all teeth. For maxillary primaries, the centrals exfoliate at year 7-8,
laterals at 8-9, canines at 11-12, first molars at 10-11, and second molars at 11-12. Remember that
these are averages. Always eliminate wildly wrong answers first.
148. In a mandibular canine, which surface is usually most parallel to the long axis of the
tooth?
A. Buccal
B. Lingual
C. Mesial
D. Distal

35

The correct answer is C. Reviewing the surfaces, the lingual surface of the canine contains the
bulbous cingulum. The surface of the lingual is thus slanted at roughly 45 degrees from the long axis
(as represented by the root). Likewise, the facial surface slants inward as you go from cervical to
incisal. The distal surface slants lingually as the tooth narrows coronally. Our answer is mesial. The
mesial surface is vertical overall, with no large bulge. Notice that the distal is always more rounded
and bulbous than the mesial, which is relatively straight and, if extended, would roughly parallel the
root.
149. The mesiolingual cusp of the mandibular first molar contacts:
A. the lingual embrasure between the maxillary first molar and second premolar
B. the distal marginal ridge of the maxillary second molar
C. the central fossa of the maxillary first molar
D. the lingual groove of the maxillary first molar
E. the lingual embrasure between the maxillary first molar and second molar
The correct answer is A. Lingual cusps of mandibular teeth are guiding cusps, not holding cusps.
Therefore, they do not occlude on marginal ridges or central fossae. The general rule for mandibular
lingual cusps is that they occlude in the lingual embrasures between their maxillary counterparts and
the teeth mesial to their counterparts, EXCEPT for the distolingual cusps of the mandibular molars,
which occlude in the lingual grooves of their maxillary counterparts. In this question we are not dealing
with the exception of the distolingual cusps of mandibular molars, so in this case, the mandibular first
molar mesiolingual cusp contacts the lingual embrasure between its counterpart (the maxillary first
molar) and the tooth mesial to it (the maxillary second premolar).
150. Which muscle of mastication controls movement of the articular disc of the TMJ?
A. Medial pterygoid
B. Lateral pterygoid
C. Temporalis
D. Masseter
The correct answer is B. The lateral pterygoid muscle has two heads. The superior head arises from
the greater wing of the sphenoid bone, and the inferior head from the lateral plate of the lateral
pterygoid bone. They extend posteriorly to the mandibular condyle, TMJ capsule, and TMJ disc.
Fibers from the upper head enter the capsule and merge with the fibrous connective tissue of both the
capsule and disc. This is related to the function of the muscle in protruding the mandible, and pulling
the disc with the condyle during protrusion, depressing (opening), and lateral movements. No other
muscle enters the TMJ.
151. The inclinations of the root tips of the mesiobuccal and distobuccal roots of the maxillary
first molar are, respectively:
A. mesial, mesial
B. mesial, distal
C. distal, mesial
D. distal, distal
E. straight, straight
The correct answer is C. A characteristic curve of the two buccal root tips of the maxillary first molar
toward each other is sometimes referred to as a pliers-handled appearance or pincer appearance. In
effect, the roots grow apically and then turn toward each other, forming a U shape. This shape helps
explain the stability of the tooth, especially when combined with the tripod-like arrangement of the
palatal root. Note that the same roots of the second maxillary molar often both point distally,
distinguishing one molar from the other. These roots in the second molar are also more parallel than
those in the first molar.
152. The predentin:
A. is more mineralized than dentin
B. contains the odontoblasts
C. contains the ameloblasts
D. is primarily collagen
E. None of the above

36

The correct answer is D. Predentin would be found in between odontoblasts and true dentin. The
odontoblasts lay down predentin, which is primarily protein matrix composed almost entirely of
collagen. As it mineralizes farther away from the odontoblastic layer, it becomes true dentin, with its
high mineral content. Ameloblasts form enamel and are not involved in dentin or predentin production.
153. Which premolar exhibits H, Y, and U occlusal patterns?
A. Maxillary first
B. Maxillary second
C. Mandibular first
D. Mandibular second
The correct answer is D. The mandibular second premolar has a variety of occlusal appearances
due mostly to its two-cusped or three-cusped nature. There is a slightly larger buccal cusp area, and
then either an approximately equal-sized lingual cusp, or a pair of lingual cusps, known as the
mesiolingual and distolingual cusps. Together, these two form an area slightly smaller than the buccal
cusp. In the event of the three cusps, an occlusal Y groove pattern is common. On the two-cusped
variety, either an H (more common) or U (less common) pattern may be found.
154. Which fiber group is bound on both sides by cementum?
A. Circular
B. Transseptal
C. Oblique
D. Horizontal
E. Apical
The correct answer is B. Both circular and transseptal fibers are classified in the gingival fiber group.
As such, they do not enter alveolar bone. Circular fibers run circumferentially around the tooth,
contained completely within the gingiva. Transseptal fibers insert in the cementum of one tooth, travel
horizontally above the alveolar crest, and insert into the cementum of an adjacent tooth. Oblique,
horizontal, and apical fibers are all periodontal fibers, and insert in cementum on one end and alveolar
bone on the other.
155. Which of the following permanent teeth is NOT succedaneous?
A. Maxillary lateral incisor
B. Mandibular first molar
C. Maxillary second premolar
D. Mandibular canine
E. Maxillary canine
The correct answer is B. Succedaneous refers to a tooth that succeeds another tooth; that is, a
tooth that replaces a previous tooth. Remember that permanent incisors and canines replace primary
incisors and canines (choices A, D, and E). Permanent premolars replace primary molars (choice C).
Thus, all the permanent teeth mentioned so far are succedaneous. Permanent molars are not
succedaneous as they do not replace primary teeth. The permanent first molar develops distal to all
primary teeth, and later the second and third molars do likewise. Molars do not replace any primary
teeth and are therefore not succedaneous.
156. Which of the following is not a fiber of the gingival unit?
A. Dento-gingival
B. Dento-periosteal
C. Alveolar crestal
D. Circular
The correct answer is C. The alveolar crestal fibers are examples of fibers of the periodontal
ligament, rather than gingival group fibers. The distinction is that periodontal fibers attach on one end
into cementum, and on the other into alveolar bone. Gingival fibers do not attach to bone. The
alveolar crest fibers attach from cementum to alveolar bone at the alveolar crest. Dento-gingival fibers
attach from cementum to gingiva, dento-periosteal fibers from cementum to periosteum, and circular
fibers run circularly around the tooth within the gingiva.

37

157. Of the following anterior teeth, the one most likely to be double rooted is the:
A. Maxillary lateral
B. Mandibular lateral
C. Maxillary canine
D. Mandibular canine
The correct answer is D. Although none of the teeth listed is commonly double rooted, the
mandibular canine is the most likely of this group. Maxillary laterals (choice A) are highly variable and
are often the most variable teeth in the mouth. However, their variability does not usually include
double-rooted appearance. Mandibular centrals and laterals (choice B) often have proximal root
grooves. If these grooves are deep, there may be an appearance of becoming double rooted, but they
rarely are. Maxillary canines (choice C) are invariably single rooted. Mandibular canines are
occasionally double rooted. The point of separation into two roots may be toward the cervical of the
root, or almost at the apex. This double-rooted possibility is significant for both exodontia and
endodontics.
158. Which periodontal fibers are most in line with the long axis of the root?
A. Alveolar crestal
B. Apical
C. Horizontal
D. Oblique
The correct answer is B. If we arbitrarily call the long axis of the root vertical, only the apical
periodontal fibers are nearly parallel (vertical). The apical fibers run vertically from the apex of the
alveolar bone socket to the root apex cementum. More coronally, the next fibers, the oblique, run
obliquely (slanted) from near apical cementum to more coronal alveolar bone. Horizontal fibers run
horizontally from cementum to bone, which is thus perpendicular to the root axis. The most coronal
alveolar fibers run obliquely from coronal cementum, apically slanted toward the alveolar crest of
bone.
159. In protrusive movements, the lingual surface of the maxillary lateral incisor will contact
which mandibular teeth?
A. Central incisor only
B. Central and lateral
C. Lateral incisor only
D. Lateral and canine
E. Canine only
The correct answer is D. Remember that the mandibular teeth, being smaller, are set one-half tooth
mesial to the corresponding maxillary tooth. That is, the maxillary central contacts the mandibular
central and half of the mandibular lateral. The maxillary lateral contacts both the mandibular lateral
and canine. Note that the maxillary canine cusp tip sits in the embrasure between the mandibular
canine and first premolar, although the incisors are the main contacts in protrusive movement.
160. The Carabelli cusp (trait) is found attached to which cusp?
A. Mesiobuccal
B. Mesiolingual
C. Distobucal
D. Distolingual
The correct answer is B. The Carabelli cusp (trait) is a variable cusp or extension of the mesiolingual
cusp of the maxillary first molar. Its expression varies from that of a full cusp, at its largest, to a small
protuberance at its smallest. Its expression varies among various races and ethnic groups. Some
anatomists describe it as a cusp, making the maxillary first molar a five-cusped tooth. Others consider
it a variety or trait of a four-cusped tooth. It is sometimes missing, and a groove or pit is found in the
normal Carabelli cusp region of the mesiolingual cusp.

38

161. If vertical dimension of occlusion is VDO, and vertical dimension of rest is VDR, and
freeway space is FS, then:
A. VDO + VDR = FS
B. VDO + FS = VDR
C. VDR + FS = VDO
D. None of the above
The correct answer is B. The vertical dimension of occlusion (VDO) is a vertical measurement on
the front of the face when the teeth are in full occlusion (centric occlusion). When the face is at rest,
the teeth are slightly apart, and the vertical dimension of the front of the face is slightly longer. This is
the vertical dimension of rest (VDR). The distance between the teeth at this point is the freeway space
(FS). So when we take the smaller VDO and add the few millimeters of the FS, we get the slightly
longer VDR. VDR is generally 2 to 5 mm more than VDO.
162. The shape of the pulp chamber in cross section cut at the cervical line in the mandibular
central incisor is:
A. oval (flattened mesiodistally)
B. oval (flattened bucco-lingually)
C. round
D. figure eight
The correct answer is A. Only maxillary incisors have roundish pulp chamber cross sections when
cut at the cervical line. This is true for both maxillary centrals and laterals. In general, mandibular
incisors and both arch canines will have ovalish pulp chambers, flattened mesiodistally. (Imagine
squashing a round pulp from both the mesial and distal sides at once, resulting in a squashed oval,
pointing to the buccal and lingual). Remember that the shape of a pulp chamber is usually related to
the shape of the surrounding root.
163. Enamel is formed from which embryonic germ layer?
A. Endoderm
B. Mesoderm
C. Ectoderm
D. None of the above
The correct answer is C. Of the tissues comprising the tooth structure, only enamel is formed from
ectoderm. It is made by ameloblasts, which are ectodermal derivatives. Cementum, produced by
cementoblasts, and dentin, produced by odontoblasts, are both mesodermal derivatives (choice B).
Note also that pulpal tissue is mesodermal. No tooth structural tissue is endodermal (choice A).
164. In a noncarious, unrestored, and fully erupted tooth, dentin continues to form at the
border of the pulp and dentin. This dentin is known as:
A. primary dentin
B. secondary dentin
C. tertiary dentin
D. reparative dentin
E. none of the above
The correct answer is B. Dentin forms the bulk of both the crown and root of the tooth. In the crown,
it underlies the outer enamel. In the root, it underlies a thin layer of cementum. The original dentin of
the crown and root is primary dentin and is the most regular in structure. The pulp continues to form
dentin later in life at the dentin-pulp border. This results in smaller pulp chambers with age. This
dentin is secondary dentin, and is less regular in structure than is primary dentin. Reparative dentin is
formed as a defense against caries, irritation, deep restorations, etc. It is the least regular in structure,
and is alo known as sclerotic dentin. There is no tertiary dentin.
165. The primary crown most UNLIKE any permanent tooth is the:
A. central incisor
B. lateral incisor
C. canine
D. first molar
E. second molar

39

The correct answer is D. The primary first molars, both maxillary and mandibular, are unique in
morphology and bear little resemblance to any permanent teeth. Primary incisors and canines
resemble their permanent counterparts closely, except in certain proportional ways. Likewise, primary
second molars are notable for closely resembling their permanent FIRST molar counterparts. The
primary maxillary first molar is a little like a premolar in shape, but generally has one minor and two
major cusps. It is both small and squat in appearance. The mandibular primary first molar is fourcusped but has a distinctly larger development of the mesial end. The MB and ML cusps generally
comprise two-thirds of the area of the crown.
166. Osteons, Haversian systems, and concentric bone layers:
A. are not found in alveolar socket bone
B. are found in the outer layer of alveolar bone
C. are found in the inner layer of alveolar bone
D. are found throughout alveolar bone
The correct answer is B. The alveloar bone is similar to most bone in the body in having a compact
outer layer of lamellar bone and an inner layer of spongy bone. This spongy bone contains marrow
space, usually of the yellow (fatty) type, although some red marrow exists. Haversian systems, with
concentric rings of bone with osteocytes and canaliculi, which surround a canal with artery, vein, and
nerve, are found only in compact (lamellar) bone. These systems do not exist within the inner spongy
layers of bone, whether we are looking at alveolar bone or any other bone in the body.
167. Which muscle of mastication both strongly elevates and strongly retrudes the mandible?
A. Masseter
B. Medial pterygoid
C. Lateral pterygoid
D. Temporalis
E. Anterior digastric
The correct answer is D. The temporalis is a strong, broad, flat muscle which can both elevate
(close) and retrude (pull back) the mandible. This is due to the fact that it contains both vertical and
nearly horizontal muscle fibers. The vertical fibers pull the mandible straight upward (elevation), while
the horizontal fibers pull straight backward (retrusion). The most anterior fibers of the temporalis are
the vertical. As you move posteriorly, they become first diagonal, and then horizontal. NBDE
questions in the past have referred to this muscle as being an elevator in the anterior and an
elevator/retruder in the posterior.
168. Which incisor is most commonly found doubled (supernumerary)?
A. Maxillary central
B. Maxillary lateral
C. Mandibular central
D. Mandibular lateral
The correct answer is B. As is often the case, the maxillary lateral incisor is the answer for weirdest
or most variable tooth. In this case, it is the most common supernumerary (doubled incisor), as it is
also the most common missing, oversized, or undersized incisor. Note that the most distal tooth
(lateral incisor, second premolar, third molar) in a set is always more likely to be missing or have a
supernumerary. Note also that the most common supernumerary is the mesiodens. This tooth is
found in the maxillary midline. It is not, however, considered an additional central. It has a morphology
unique to itself and different from centrals.
169. The primary mandibular central incisor usually exfoliates between which ages?
A. 4-5 years
B. 6-7 years
C. 8-9 years
D. 10-11 years
E. 12-13 years

40

The correct answer is B. Eruption and exfoliation questions are common in this section. The
mandibular primary centrals are often the first teeth lost, most commonly at age 6. Often they exfoliate
as the permanent first molars are erupting. For mandibular primaries, the centrals exfoliate at year 67, the laterals at year 7-8, the canine at year 9-10, the first molar at year 10-11, and the second molar
at age11-12. Remember that all exfoliation questions are approximations of average ages. The wrong
answers must be fairly far off the mark.
170. In centric occlusion, the mesiolingual cusp of the maxillary second molar will contact:
A. central fossa of the mandibular second molar
B. central fossa of the mandibular first molar
C. distal marginal ridge of the mandibular first molar and mesial marginal ridge of the second molar
D. distal marginal ridge of the mandibular second premolar and mesial marginal ridge of first molar
The correct answer is A. The mesiolingual cusp of a maxillary second molar is a holding cusp. The
general rule for maxillary holding cusps is as follows: a maxillary holding cusp contacts the distal
marginal ridge of its mandibular counterpart and the mesial marginal ridge of the mandibular tooth
distal to its counterpart, EXCEPT for the mesiolingual cusps of the molars, which contact the central
fossae of their counterparts. This should be the central fossa of the counterpart, the mandibular
second molar.
171. When assigning types of mandibular motion to the compartments of the TMJ, it is
generally agreed that which compartment is associated with which motion type?
A. Anterior: rotation, posterior: translation
B. Anterior: translation, posterior: rotation
C. Superior: rotation, inferior: translation
D. Superior: translation, inferior: rotation
The correct answer is D. First, eliminate choices A and B, as the disc divides the joint compartment
into upper (superior) and lower (inferior) compartments only. In the upper compartment, the upper
surface of the disc acts as a cushion as the mandibular condyle slides forward. This cushion slides
against the articular eminence in the movement of the condyle in a forward and downward direction.
When rotating about an axis during opening, the condyle slides against the lower surface of the disc
which conforms to the shape of the condylar head. So we say that translation occurs in the upper
compartment and rotation in the lower.
172. The mesial height of contour of the mandibular central incisor is:
A. near the incisal edge
B. at the junction of the incisal and middle thirds
C. in the middle third
D. at the junction of the middle and cervical thirds
The correct answer is A. Mandibular central incisors contact at the most incisal point found on any
tooth. The contact is located just below the incisal edge. On the distal surface, it is still incisal and
near the edge, but is slightly more cervical than the contact found on the mesial side of the tooth.
173. When compared to that of permanent teeth, the ratio of mesio-distal diameter to crown
height of primary teeth is:
A. greater
B. less
C. the same
D. variable, depending on the individual tooth
The correct answer is A. Primary teeth are often said to have a short or squat appearance of their
crowns compared to those of permanent teeth. This is because these teeth are often relatively wide
mesiodistally and short incisocervically. This results in a LARGE ratio of mesiodistal diameter to
incisocervical crown height. This ratio is smaller in the permanent teeth, which lack the short, squat
appearance of primaries. One example: for maxillary central incisors, the primary tooth ratio is 1.083
(the tooth is just slightly bigger mesioincisally than in crown height), while for the corresponding
permanent incisor it is .809 (the mesiodistal diameter is only 80% of the crown height).

41

174. When compared to that of permanent teeth, primary teeth color tends to be:
A. whiter
B. darker
C. similar in shade
D. variable, depending on the individual tooth
The correct answer is A. As a general rule, primary teeth are whiter than their permanent
counterparts. This shade difference is often noticed by parents. It is common for parents to notice, for
example, that newly erupted permanent mandibular incisors are a few shades darker or more
yellowish than the primary maxillary incisors that may remain as the mandibular permanent incisors
erupt. In the mixed dentition (normal, unstained, not heavily carious, and no tetracycline effects), you
can often pick out the primary versus permanent teeth by color alone.
175. Which is the narrowest anterior tooth in a mesiodistal direction?
A. Maxillary lateral
B. Mandibular central
C. Mandibular lateral
D. Mandibular canine
The correct answer is B. In general, canines are wider mesiodistally than lateral incisors(both
arches), so we should eliminate the canine (choice D). In general, maxillary incisors are wider
mesiodistally than their mandibular counterparts, so we should eliminate the maxillary lateral (choice
A). So it should only be a question of mandibular lateral versus central. On the average, the central is
slightly smaller, about 0.5 mm smaller, and this gives it the distinction of narrowest anterior tooth
mesiodistally. The Dental Board exam really loves longest, shortest, roundest, fattest, etc., type
questions. Note that the maxillary canine is the longest tooth in the mouth.
176. Which of the following angles in a maxillary incisor is sharpest?
A. Mesioincisal of central
B. Mesioincisal of lateral
C. Distoincisal of central
D. Distoincisal of lateral
The correct answer is A. In looking at a maxillary central incisor from the facial, you will see that the
mesioincisal angle is a sharp, nearly right angle. This is especially noticeable where the two centrals
meet and a very small embrasure is present. The distoincisal angle, by contrast, is more rounded
(choice C). On the lateral, the mesioincisal (choice B) is again sharper than the distoincisal (choice D),
which is very rounded. However the angle on the lateral does not approach the sharpness of that on
the central.
177. As you go from mandibular first to second to third molars, mesiodistal crown length:
A. increases, then decreases
B. increases
C. decreases
D. decreases, then increases
The correct answer is C. While the mandibular third molar is often so variable that it is hard to make
useful predictions about it, these generalities apply to mandibular first, second, and third molars as
you move distal in the arch: 1) they decrease in mesiodistal length, and 2) their roots decrease in
length. Third-molar crowns may resemble normal molars or may be missing whole cusps. In rare
cases they are roundish or ovoid, but they are always smaller than their second-molar counterparts,
which are always smaller than the first molars. Note that buccolingual length is always shortest in the
third molar but about equal in the first and second.
178. Which of the following permanent teeth is MOST likely to be missing a distolingual cusp?
A. Mandibular first molar
B. Maxillary first molar
C. Maxillary third molar
D. Maxillary second molar

42

The correct answer is C. The mandibular first molar is invariably a five-cusped tooth, with MB, ML,
DB, DL, and distal cusps. Maxillary first and second molars are normally four-cusped teeth. The
distolingual cusp is usually less pronounced on the second molar. It is often absent on the maxillary
third molar. This is an arch trait: the gradual diminishment in size of the distolingual cusp as you go
posteriorly in the maxilla. In the lower arch, the mandibular second premolar can also be viewed as
possibly lacking a distolingual cusp. This tooth is either two- or three-cusped. The two-cusped variety
has a single buccal cusp and a single lingual cusp, while the three-cusped version has a buccal cusp
and paired mesiolingual and distolingual cusps. However, both varieties are common, so it is common
to find this tooth lacking the distolingual cusp as well, although that answer is not listed here.
179. The protein matrix of enamel is:
A. composed of collagen
B. composed of keratin
C. composed of protein which is not collagen or keratin
D. absent, there is no protein matrix
The correct answer is C. Chemical analysis of the enamel matrix has not adequately identified the
protein involved, but it is clear that it is neither keratin, as was once thought, nor collagen, which is the
matrix of both dentin and bone. The protein is extremely high in proline, but makes up less than 1% of
the enamel mass.
180. Cervical bulges are found on which surfaces of primary anterior teeth?
A. Mesial and distal
B. Buccal and lingual
C. Lingual only
D. Buccal only
The correct answer is B. Primary teeth are notable for cervical bulges; that is, extension of the
crown at the cervical line. In the primary anteriors, these bulges extend over both the buccal and
lingual surfaces, as though the cervical line was a tightened belt and was tightened further to make
the crown overhang in both front and back. In the primary molars, by contrast, the cervical bulge is
buccal only.
181. Which of the following is a function of pulpal tissue?
A. Sensory
B. Formation of dentin
C. Nutritive
D. Defensive
E. All of the above
The correct answer is E. It should be noted that the MAJOR function of pulp is the production of
dentin, as dentin is the major structural tissue of both crown and root. Any question on the dental
boards that asks for the "most important" function will have dentin formation as the answer. However,
the pulp is also sensory (clearly). All sensory fibers in the pulp carry pain sensation, regardless of
whether the original stimulus is hot, cold, or touch. The nutritive function of pulp consists of providing
materials to the dentin through the odontoblastic processes. The defensive function of the pulp
consists of a limited ability to produce a defensive inflammatory response, and the formation of
reparative dentin when the tooth is damaged.
182. Functions attributed to the ligaments of the temporomandibular joint include all of the
following EXCEPT:
A. increasing stability of the joint
B. aiding in forward movement of the condyle
C. increasing the strength of the joint
D. limiting extreme motion of the joint
E. All of the above are ligament functions
The correct answer is B. The ligaments, being fibrous connective tissue in composition, play no role
in movement of the condyle. Muscular tissue is necessary for this movement, specifically fibers from

43

the upper head of the lateral pterygoid muscle. There is disagreement among anatomy sources as to
the function or role of the ligaments. None give the ligaments an extremely important function, but
most claim that they help limit motion of the mandible beyond a certain point, or act as barriers to
excessive forward or downward motion. Others give the ligaments a general strengthening function
which adds stability to the joint. Most importantly, the Dental Boards usually agree with the general
notion of adding strength, stability, and limits of movement to the joint.
183. Which permanent mandibular molar is most likely to be marked by supplemental occlusal
grooves, crenulations, and unpredictable placement of pits?
A. First molar
B. Second molar
C. Third molar
D. All molars about equally
The correct answer is C. The question may be answered on general principle, in that the third
molars are often the most unpredictable teeth in either arch. The specific patterns referred to here are
the pit-groove patterns. In third molars, you will often find supplemental grooves at right angles to the
main grooves, and additional pits and fissures not normally seen in first and second molars. The
crenulated pattern refers to a highly grooved overall occlusal surface with grooves running in all
directions, leaving a lacy, nook-and-cranny occlusal surface.
184. Which cell type is NOT found within the periodontal membrane?
A. Cementoblasts
B. Osteoblasts
C. Odontoblasts
D. Osteoclasts
The correct answer is C. Odontoblasts form dentin and surround the dental pulp. The dentin forms
the bulk of both the tooth crown and root. However, dentin is not one of the opposing surfaces within
the periodontal ligament. Within the ligament, the alveolar bone of the tooth socket lies next to the
cementum surface of the tooth root. On the cemental side of the ligament, cementum-forming
cementoblasts are found. On the bone side, bone-forming osteoblasts and bone-remodeling
osteoclasts are both found. In addition, numerous collagen fiber bundles insert in both the cementum
and alveolar bone, forming the connection that keeps the tooth in place in the socket.
185. The Carabelli cusp (trait) is found on which molar tooth?
A. Maxillary first
B. Mandibular first
C. Maxillary second
D. Mandibular second
The correct answer is A. The Carabelli cusp (trait) is a variable cusp or extension of the mesiolingual
cusp of the maxillary first molar. Its expression varies from that of a full cusp, at its largest, to a small
protuberance at its smallest. Its expression varies among various races and ethnic groups, including
an increased incidence in people of Asian descent. Some anatomists describe it as a cusp, making
the maxillary first molar a five-cusped tooth. Others consider it a variety or trait of a four-cusped tooth.
It is sometimes missing, and a groove or pit is found in the normal Carabelli cusp region of the
mesiolingual cusp.
186. In a left lateral (working movement):
A. the left mandibular teeth are the working side and move to the left
B. the left mandibular teeth are the working side and move to the right
C. the left mandibular teeth are the balancing side and move to the left
D. the left mandibular teeth are the balancing side and move to the right
The correct answer is A. By definition, a working movement occurs when the mandibular teeth on
one side move laterally across the surfaces of the maxillary teeth toward their own side. In other

44

words, in a left working movement, left mandibular cusps move laterally left across the cusps of the
maxillary left teeth. The other side is known as the nonworking (balancing) side. In this case, the
balancing side is the right side. In a left working movement, the balancing side also moves left, as the
mandible cannot move both left and right at the same time.
187. Of the following incisors, the one which can most often be rotated during extraction is
the:
A. maxillary central
B. maxillary lateral
C. mandibular central
D. mandibular lateral
The correct answer is A. For the most part, rotation of a tooth during extraction works best on teeth
with rounded roots, such as the maxillary central incisor and maxillary canine. It can be used in
conjunction with buccal-lingual luxation on teeth with single ovalish roots. It can never be used on
double-rooted teeth. Of the incisors above, the maxillary central is most likely to have a roundish root.
The others tend to have roots which are more ovalish rather than round and are luxated primarily, with
some rotation. In general, maxillary incisor roots are more rounded than are mandibular roots.
188. In the permanent mandibular third molar, when viewed from the occlusal:
A. the mesial end is larger than the distal
B. the distal end is larger than the mesial
C. the mesial and distal ends are the same size
D. it is impossible to generalize about mandibular third molars
The correct answer is A. Beware of choice D. Although mandibular third molars are notoriously
variable in both crown and root form, there are some good generalizations. Almost invariably, the
mesial (trigonid) end of the crown is wider, larger, and more distinct than the distal (talonid). The most
common pattern is a four-cusp tooth, with MB and ML being the largest cusps and the whole occlusal
surface tapering toward the distal.
189. When viewed from the meisal, the tooth closest to vertical in the buccolingual angulation
of its main axis is the:
A. maxillary central incisor
B. maxillary lateral incisor
C. maxillary canine
D. maxillary first premolar
The correct answer is D. Another way of asking the question is: which tooth does not lean toward
the buccal or lingual? The maxillary first premolar is almost straight vertically, estimated to be about 5
degrees from vertical. Premolars, as a class, have the least bucco-lingual angulation. The central
tends to lean greatly toward the lingual (28 degrees), the lateral about the same (26 degrees), and the
canine slightly less. Do not bother to remember degree measures, which are mentioned only for
comparison. Do not confuse this leaning with the mesiodistal leaning (when viewed from the buccal or
lingual). These measurements differ significantly for the teeth listed, with the central incisor being
most vertical.
190. Tooth #23 replaces which primary tooth?
A. K
B. L
C. M
D. N
E. O
The correct answer is D. Both 23 and N are mandibular left incisors. Remember that A through J are
maxillary, while K through T are mandibular. The mandibular replacements are 20 for K, 21 for L, 22
for M, 23 for N, 24 for O, 25 for P, 26 for Q, 27 for R, 28 for S, and 29 for T. Remember that
permanent molars 17, 18, 19, 30, 31, and 32 do not replace primary teeth.

45

191. The mesiobuccal cusp of the mandibular third molar occludes with which maxillary tooth
surfaces?
A. The mesial marginal ridge of the third molar and distal marginal ridge of the second molar
B. The distal marginal ridge of the third molar and mesial marginal ridge of the second molar
C. The mesial marginal ridge of the third molar only
D. The central fossa of the maxillary third molar
The correct answer is A. The mesiobuccal cusp of a mandibular molar is a holding (supporting)
cusp. The general rule of occlusion of mandibular holding cusps is as follows: the holding cusps of the
mandibular teeth occlude on the mesial marginal ridge of their maxillary counterpart, and the distal
marginal ridge of the maxillary tooth mesial to their counterpart, EXCEPT distobuccal cusps of
mandibular molars occlude with central fossae of their counterparts, the distal cusp of the mandibular
first molar occludes with the distal triangular fossa of its counterpart, and the first premolar occludes
only with the mesial marginal ridge of its counterpart (but not the canine). In this case, the maxillary
counterpart is the maxillary third molar, and the tooth immediately mesial to it is the maxillary second
molar.
192. In the primary mandibular first molar, the most noticeable cusps are:
A. MB and DB
B. MB and ML
C. DB and DL
D. MB and DL
The correct answer is B. The primary mandibular first molar is a highly unusual tooth. Although it
technically contains four cusps, in typical molar fashion it has a great variety in the prominence of
those cusps. The MB and ML cusp are most prominent, and the mesial section of the crown
comprises two-thirds of the crown area. Both the DB and DL cusps are much reduced. The range of
cusp size, in decreasing order is: MB, ML, DB, DL. The ML cusp is notable for a pointy, cone-like
shape.
193. The central fossa of a maxillary first molar is NOT bordered by which cusp?
A. Mesiobuccal
B. Mesiolingual
C. Distobuccal
D. Distolingual
The correct answer is D. The central fossa, containing a central pit, is located within the main cusp
triangle (trigon) of the maxillary first molar. The three major cusps: MB, ML, and DB, surround the
fossa and form the trigon, or primitive cusp triangle. The DL cusp (talon) is not part of the trigon, does
not surround the central fossa, and is the smallest cusp of the tooth (excluding the Carabelli cusp or
trait).
194. Which position is achieved entirely by soft tissue, without guidance from teeth?
A. Postural (physiologic rest)
B. Centric occlusion
C. Centric relation
D. Protruded contact
The correct answer is A. In postural rest position, the mandible stays a comfortable distance from
the maxilla, with the teeth slightly separated. The freeway space between the teeth is often 2-5 mm.
There is no tooth contact, and the position is determined and maintained by the muscles of occlusion
and related tendons and ligaments. Centric occlusion involves full tooth contact. Centric relation is
normally a few millimeters distal to centric occlusion and is achieved by moving distal from centric
occlusion. This procedure involves tooth contact. Protruded contact involves contacting guidance of
the maxillary and mandibular teeth during protrusion.

46

195. The faciolingual dimension of a mandibular first molar, when compared to the mesiodistal
dimension, is:
A. smaller
B. larger
C. the same
D. larger or smaller, depending on the tooth
The correct answer is A. Mandibular first molars are the largest teeth in the mouth, in mesiodistal
dimension. While they are also large buccolingually, they are invariably larger mesiodistally by about
0.5 to 1.0 mm. This is in contrast to the maxillary first molar, which is slightly larger buccolingually
than mesiodistally, usually by about 1.0 mm. The mandibular second molar is about equal in both
directions and is thus more symmetrical than the first molar.
196. In a left working movement:
A. the left side moves laterally, and the right side medially
B. the left side moves medially, and the right side laterally
C. both sides move medially
D. both sides move laterally
The correct answer is A. In a left working movement, the mandible shifts to the left. From a central
position, the left side of the mandible is moving away from center (laterally). The right side of the
mandible, although also moving left, is moving toward the center (medially). The left TMJ purely
rotates when the right is rotating and translating mechanically. Note that it is impossible for both sides
of the mandible to move either laterally or medially at the same time.
197. The premolar which is most often double rooted is the:
A. maxillary first
B. maxillary second
C. mandibular first
D. mandibular second
The correct answer is A. Maxillary first premolars are almost always double rooted. When single
rooted, they always have two separate root canals. The level of division of the two roots varies, from
complete division up to the crown, to only a small separation apically. The mandibular first premolar is
most often singlerooted, and the mandibular second premolar is almost always single rooted (double
roots are rarer than in the mandibular first). The maxillary second premolar is most often single
rooted. The extreme likelihood that the maxillary first premolar will have two roots has implications for
both endodontics and exodontia of this tooth. It is also reported that some small percentage (5% in
one study) are actually triple rooted.
198. In centric occlusion, the mesiolingual cusp of the maxillary first molar will contact:
A. mesial marginal ridge of mandibular first premolar and distal marginal ridge of second premolar
B. distal marginal ridge of the mandibular first molar and distal marginal ridge of the second molar
C. distal marginal ridge of the mandibular second premolar and mesial marginal ridge of first molar
D. central fossa of the mandibular first molar
The correct answer is D. The mesiolingual cusp of a maxillary molar is a holding cusp. The general
rule for maxillary holding cusps is as follows: a maxillary holding cusp contacts the distal marginal
ridge of its mandibular counterpart and the mesial marginal ridge of the mandibular tooth distal to its
counterpart, EXCEPT FOR THE MESIOLINGUAL CUSPS OF THE MOLARS, WHICH CONTACT
THE CENTRAL FOSSAE OF THEIR COUNTERPARTS. This should be the central fossa of the
counterpart, the mandibular first molar.
199. A rule regarding congenitally missing teeth is:
A. A mandibular tooth is more likely to be missing than its maxillary counterpart
B. A distal tooth of a type is more likely to be missing than the mesial tooth of the same type
C. A mesial tooth of a type is more likely to be missing than a distal tooth of the same type
D. None of the above is true

47

The correct answer is B. In any set of teeth (incisor, premolar, molar), the distal tooth is more likely
to be congenitally missing than the mesial. So lateral incisors are missing more than centrals, second
premolars are missing more than first premolars, and third molars are more commonly missing than
first or second. Of all of these, the MOST commonly missing is the maxillary lateral incisor. Note that
choice A is therefore incorrect.
200. The crowns of the incisors when viewed from the incisal
A. are wider mesiodistally than buccolingually in both arches
B. are wider buccolingually than mesiodistally in both arches
C. are wider mesiodistally in the maxilla, and wider buccolingually in the mandible
D. are wider mesiodistally in the mandible and wider mesiodistally in the maxilla
The correct answer is C. This is known as an arch trait, as it is true for both incisors of each arch.
Both incisors of the maxilla are wider mesiodistally. Both incisors of the mandible are wider
buccolingually. The maxillary central, in particular, is known for the greatest assymetry in this regard.
It is much wider mesiodistally than buccolingually, and that difference, expressed as a ratio, is
greatest for that incisor. It is also the largest incisor, in both dimensions, in absolute size.
201. Which of the following will cause reduction of the size of the pulp chamber in mid-life?
A. Formation of primary dentin
B. Formation of secondary dentin
C. Internal resorption
D. External resorption
E. Pulp fibrosis
The correct answer is B. Primary dentin (choice A) surrounds the pulp chamber when the tooth is
first formed and erupts. It is the initial dentin that makes up the base of both the crown and root. If the
young tooth has a given size pulp chamber, further dentin formation, secondary dentin, later in life, will
begin to fill the chamber with dentin, making it smaller. Internal resorption (choice C) is loss of dentin,
resulting in a larger pulp chamber. External resorption (choice D) involves the exterior of the root and
does not change the pulp chamber size. Pulp fibrosis (choice E) would change the histology of the
cells in the chamber, but not the chamber's size.
202. The largest root of the maxillary first molar is the:
A. mesiobuccal
B. distobuccal
C. palatal
D. varies depending on the individual tooth
The correct answer is C. The palatal or lingual root of the maxillary first molar is the most massive
by far. It is normally straight but palatally inclined, giving the three roots a tripod-like appearance. The
mesiobuccal root (choice A) is second in size, and the distobuccal (choice B) is smallest. The
mesiobuccal is often curved distally and the distobuccal curved mesially. This pliers-like appearance
is not found in the maxillary second and third molars.
203. Which is the first primary tooth to erupt?
A. Mandibular central
B. Mandibular lateral
C. Maxillary central
D. Maxillary lateral
The correct answer is A. If you see enough small children in your practice, you'll often notice that a
6-month-old child will have only two teeth: the two mandibular centrals. These are commonly the first
teeth to erupt, and most commonly do so at 6 months. Mandibular laterals are often seen at about 9
months. Maxillary centrals and maxillary laterals average about 8 months. First molars then precede
canines in both arches, and second molars are last in both arches. The last primary is usually the
maxillary second molar at about 24 months. These times may vary considerably. Know the most
common order: central, lateral, first molar, canine, second molar.

48

204. In performing a left working movement:


A. Only the left lateral pterygoid contracts
B. Only the right lateral pteyrgoid contracts
C. Both lateral pterygoids contract
D. Neither lateral pterygoid contracts
The correct answer is B. In a left working movement, the mandible moves toward the left. The left
side is then known as the working side and the right side as the nonworking (balancing) side. The
lateral pterygoids pull the mandibular condyles forward, so the pull of both together results in
protrusion. For only a left-sided movement, the left lateral pterygoid does not contract and the left
condyle stays in a relatively unchanging position (it does rotate slightly). The right lateral pterygoid
contracts and pulls the right side of the mandible outward, and then it turns left, as the left side of the
mandible is stationary. Remember that the right lateral pterygoid moves the mandible left, and the left
lateral pterygoid moves the mandible right.
205. On which premolar can you see more of the occlusal table from the mesial than from the
distal?
A. Maxillary first
B. Maxillary second
C. Mandibular first
D. Mandibular second
The correct answer is C. The mandibular first premolar is the exception to premolar form in many
ways. It is canine-like with a reduced lingual cusp, resembling a cingulum. Its occlusal table tilts
downward from buccal to lingual in a slanted fashion. It is also slightly higher on the distal side than
the mesial, and its mesial marginal ridge is less prominent than its distal marginal ridge. This means
that in a mesial view you can see some of the occlusal table because it slants toward you. From the
distal view, this is not true. The higher distal end blocks any occlusal view from that direction.
206. At the cervical line, a cross section of the mandibular canine would show:
A. a round shape
B. an oval shape wider on the lingual than the labial
C. a triangular shape with a distinctly wider labial surface
D. an oval shape with a slightly wider labial surface
The correct answer is D. The mandibular and maxillary canines exhibit similar cross sectional
shapes at the cervical line. The mandibular canine cross section is oval and flattened mesiodistally. It
is generally slightly wider at the labial end than at the lingual. The pulp cavity at this point is lensshaped, double convex.
207. In a mandibular first molar, which pulp horn is likely to be smallest?
A. Mesiobuccal
B. Mesiolingual
C. Distobuccal
D. Distolingual
E. Distal
The correct answer is E. There are two facts or concepts involved in correctly answering this
question. The first is that the mandibular first molar is a five-cusped tooth, named corresponding to
the cusps listed in the answer choices. The second is that the pulpal anatomy reflects the cuspal
anatomy (or vice versa). Tall, large cusps are likely to have tall, large pulp horns. Small cusps
generally have small associated pulp horns. In this case, the distal cusp is generally the smallest of
the cusps of the mandibular first molar, so its pulp horn is likely to be smallest as well.
208. The premolar most likely to have a three-cusped appearance when viewed from the
occlusal is the:
A. maxillary first
B. maxillary second
C. mandibular first
D. mandibular second

49

The correct answer is D. The premolars are usually considered to be two-cusped teeth (bicuspids).
The exception to the rule, to a small extent, is the mandibular second premolar. Often, the lingual
cusp area is divided into a mesiolingual cusp and a distolingual cusp. This gives the tooth a threecusped appearance. A two-cusped variety also exists, with a large lingual cusp instead of the two
smaller ones. In the three-cusped variety, a lingual groove separates the two lingual cusps. Note that
the two-cusped and three-cusped varieties are not grossly different in size, as the single lingual cusp
is about the same size as a mesioloingual and distolingual combined. The other premolars exist
primarily in double-cusped varieties only.
209. The distolingual cusp of the mandibular first molar contacts:
A. the lingual embrasure between the maxillary first molar and second premolar
B. the lingual groove of the maxillary second molar
C. the central fossa of the maxillary first molar
D. the lingual groove of the maxillary first molar
E. the lingual embrasure between the maxillary first molar and second molar
The correct answer is D. Lingual cusps of mandibular teeth are guiding cusps, not holding cusps.
Therefore, they do not occlude on marginal ridges or central fossae. The general rule for mandibular
lingual cusps is that they occlude in the lingual embrasures between their maxillary counterparts and
the teeth mesial to their counterparts, EXCEPT for the distolingual cusps of the mandibular molars,
which occlude in the lingual grooves of their maxillary counterparts. In this question we are dealing
with the exception of the distolingual cusps of mandibular molars, so in this case, the mandibular first
molar distolingual cusp contacts the lingual groove of its counterpart (the maxillary first molar).
210. Which of the premolars is most likely to have a pronounced mesial marginal groove
extending into a pronounced mesial root concavity?
A. Maxillary first
B. Maxillary second
C. Mandibular first
D. Mandibular second
The correct answer is A. The mesial marginal groove, extending from the marginal ridge, along the
mesial side of the crown into the proximal area (mesial concavity) and down into the mesial root
concavity, is a characteristic of many maxillary first premolars. This has implications for both calculus
removal (scaling and root planing) and restoration (placement of well adapted matrix bands). It is not
found in the other premolars, although they may sometimes have some less pronounced root
concavities. Root concavities are less common and less pronounced in the mandibular premolars.
211. At age 6 1/2, an average child is MOST likely to have how many erupted permanent teeth?
(Assume that you average the teeth present in a sample of 100 children).
A. 4-8
B. 9-12
C. 13-16
D. 17-20
E. 21-24
The correct answer is A. This can vary considerably, but using average eruptions, all permanent first
molars usually erupt at around age 6 to 7, so four first molars are likely. Rule out all premolars, which
generally erupt between age 9 and 12. Likewise, canines usually erupt between age 9 and 12. For the
incisors, usually the mandibular centrals are in at about age 6, so that makes eight so far. Mandibular
laterals and maxillary centrals and laterals usually erupt between 7 and 9 years. So the average 6year-old has all first molars and mandibular centrals as erupted permanent teeth.
212. The major function of contact points in the dentition is to:
A. protect the incisal surface
B. protect the periodontium
C. protect the alveolar mucosa
D. protect restorations from poor retention
E. none of the above

50

The correct answer is B. Contacts ensure that food does not get lodged between teeth and into the
sulcus. Food, debris, and plaque in that area will lead to inflammation, tissue destruction, and bone
loss. Contacts do not protect the incisal surface, as they are below (cervical to) that area. They do
protect the gingiva (see above), but not especially the alveolar mucosa, which is more cervical and
basically below the tooth region. The alveolar mucosa begins at the mucogingival junction. Contact is
not involved in restoration retention. All restorations must be retained by retention within the individual
tooth itself.
213. The maxillary canine has:
A. a mesial marginal ridge only
B. a distal marginal ridge only
C. neither a mesial nor distal marginal ridge
D. both a mesial and distal marginal ridge
The correct answer is D. We often do not think of anterior teeth as having marginal ridges, as they
are less distinct than on the posterior teeth; and in the posterior teeth, they are often points of occlusal
contact. However, the maxillary canine, as well as other anterior teeth, does have both a mesial and
distal marginal ridge. The lingual anatomy of the maxillary canine has a cusp tip, with mesial and
distal marginal ridges sloping down from it. Centrally, a lingual ridge splits the lingual surface in two. In
between the marginal ridge and the lingual ridge, we find two depressions, the mesiolingual fossa and
the distolingual fossa.
214. As the tooth bud develops into a tooth, the ameloblasts and odontoblasts:
A. become farther apart
B. become closer together
C. stay approximately the same distance apart
D. are always immediately next to each other
The correct answer is A. Early in tooth bud formation, the tissues of the enamel organ-outer enamel
epithelium, stellate reticulum, stratum intermedium, and inner enamel epithelium-form adjacent to the
dentin-forming tissues of the dental papilla. In fact, at this stage, the ameloblasts and future
odontoblasts are in contact along the length of the tooth bud. The dentin production begins first, and
predentin is laid down by odontoblasts. This predentin production induces the ameloblasts to begin
enamel production by cells of the inner enamel epithelium. Thus, dentin is produced above the
odontoblasts while enamel is formed below the inner enamel epithelium. Both dentin and enamel are
deposited between these two layers, and the former site of the junction of the two layers becomes the
DEJ, or dento-enamel junction. By tooth completion, the enamel organ is at the surface of the gingiva
while the odontoblasts are producing dentin for the tooth root. Thus, these cell layers are very far
apart at this point.
215. In extraction of maxillary central incisors, the root anatomy guides the dentist to use a
motion that is primarily:
A. buccal-lingual luxation
B. mesio-distal luxation
C. rotation
D. all of the above
The correct answer is C. One of the best reasons to know standard and unusual root shapes for the
various maxillary and mandibular teeth is to use the correct motion on the forceps when performing
extractions. In general, rounded roots are primarily rotated, including the maxillary central incisors and
maxillary canines. All double and triple rooted teeth cannot be rotated and must be luxated in the
buccal and lingual directions. Mesial-distal motion is normally ruled out because of the existence of
adjacent teeth. It is used in rare cases where there is no mesial and distal adjacent tooth. The
maxillary central is invariably single rooted with a rounded root, and can almost always be rotated
without breaking the crown off of the root.
216. The most common number of mammelons on an incisor is:
A. 1
B. 2
C. 3
D. 4

51

The correct answer is C. Mammelons are bumps on the incisal edge of incisors. They are most
commonly found on central incisors of both arches, and if found on the laterals, they are less
prominent than those found on the centrals. There are normally three, but this is variable. The three
are termed mesial, central, and distal. They are often worn off on adult teeth through years of
occlusion.
217. As you go from mandibular first to second to third molars, overall root length:
A. increases, then decreases
B. increases
C. decreases
D. decreases, then increases
The correct answer is C. While the mandibular third molar is often so variable that it is hard to make
useful predictions about it, these generalities apply to mandibular first, second, and third molars as
you move distally in the arch: 1) they decrease in mesiodistal length, and 2) their roots decrease in
length. Third-molar roots may resemble normal molars or may be missing roots or have additional
roots. Roots are often partially or totally fused. In some cases they form a cone-like, fused mass,
making exodontia much simpler in these cases. In all cases, however, their roots are shorter than
those of their second-molar counterparts, which are always smaller than those of the first molars.
218. Incisal contact between the maxillary and mandibular incisors is seen normally:
A. in centric occlusion
B. in working movements
C. in protrusive movements
D. in retrusive movements
E. never
The correct answer is C. Posterior contact is normal during centric occlusion and maintains the
vertical dimension of occlusion. In a normal dentition, however, the incisors are slightly out of contact
during centric occlusion. When the mandible moves forward (protrusion), the incisal edges of the
mandibular anteriors meet the lingual surfaces of their maxillary counterparts as the teeth separate
during opening. (Try it in your mouth!). Note that during retrusion (moving the mandible from centric
occlusion to a more distal position), no such contact occurs. During working (lateral) movements, only
posterior teeth and canines contact (group function), or canines contact alone (canine/cuspid
protected guidance).
219. Severing the left lateral pterygoid will cause the mandible to move in what direction if the
patient attempts to protrude?
A. Posteriorly
B. Closure (elevation)
C. Right
D. Left
E. Downward (depression)
The correct answer is D. If you like rules, the rule is that if a lateral pterygoid is injured, the mandible
moves toward the injured side. If you wish to understand this rule better, imagine two intact lateral
pterygoids protruding the mandible normally. Now cut the left lateral pterygoid, and imagine that the
left side stays in place but the right side continues to protrude. In effect, the mandible will rotate
around the axis of the left side. The left side stays in place while the right side rotates left. Lateral
pterygoids have little effect on closing.
220. Which characteristic is NOT typical of lining mucosa?
A. Nonkeratinized
B. Lines the surface of the hard palate
C. Freely moveable
D. Contains a well developed submucosa
E. Epithelium thin

52

The correct answer is B. Lining mucosa covers most of the oral cavity except for the hard palate and
gingival areas. It is usually loose, freely movable, and has a well-developed submucosa. The lamina
propria is often highly glandular. The epithelium is thin and nonkeratinized. Epithelial ridges and rete
pegs are uncommon and short.
221. Which ligament of the temporomandibular joint merges with the fibers of the articular
capsule?
A. Lateral
B. Stylomandibular
C. Sphenomandibular
D. None of the above
The correct answer is A. The lateral ligament, also known as the temporomandibular ligament, is
found extending downward from the lateral surface of the zygoma. The zygoma is formed from the
contact of the zygomatic section of the temporal bone and the temporal portion of the zygomatic bone.
The ligament is roughly triangular in shape, and on its medial side is found the articular capsule and
joint. Fibrous connective tissue composes both the ligament and capsule, and there is a fibrous
connection between them.
222. The central portion of the articular disc of the TMJ is best described as:
A. convex and avascular
B. convex and vascular
C. concave and avascular
D. concave and vascular
The correct answer is C. The articular disc of the TMJ is composed of fibrous connective tissue.
Some sources state that chondrocytes are found within it, so it is sometimes referred to as fibrous
connective tissue with an underlying base of fibrocartilage. Its shape is biconcave, somewhat like that
of an erythrocyte, with a raised outer portion and depressed middle. The central portion is avascular,
and only the periphery, especially the posterior periphery is vascularized.
223. The tissue located between the inner and outer enamel epithelium is known as:
A. stellate reticulum
B. dental sac
C. dental papilla
D. enamel organ
The correct answer is A. The stellate reticulum is an ectodermal tissue located between the inner
and outer enamel epithelium. The stellate, or star-like, appearance of the tissue is caused by the
arrangement of few cells within a network, separated by intracellular fluid. The inner enamel
epithelium will differentiate into ameloblasts and form enamel. The outer enamel epithelium eventually
merges with the gingiva as the tooth erupts. A small cell layer between the inner enamel epithelium
and stellate reticulum is referred to as the stratum intermedium. Its presence is necessary for proper
ameloblast function. The dental sac is mesodermal and forms cementum and the periodontal
ligament. The dental papilla is mesodermal and forms pulp and dentin. The enamel organ consists of
inner and outer enamel epithelium, stellate reticulum and stratum intermedium and forms enamel. It is
ectodermal in origin.
224. Which ligament of the temporomandibular joint inserts into the ramus and angle of the
mandible?
A. Temporomandibular
B. Stylomandibular
C. Sphenomandibular
D. Lateral
The correct answer is B. The origins and insertions of the TMJ ligaments are as follows: The
stylomandibular ligament arises from the spine of the temporal bone known as the styloid process and
inserts on the lower ramus and angle of the mandible. The sphenomandibular ligament arises from a

53

spine on the sphenoid bone and runs forward and downward to insert on the lingula and deep ramus
of the mandible. The lateral ligament is also known as the temporomandibular ligament. It descends
from the lower border and tubercle of the zygoma to the posterior lateral condyle. Its fibers merge with
those of the articular capsule.
225. The premolar with a root cross section and pulp chamber shape at the cervical line which
is most similar to that of a canine is the:
A. mandibular first
B. mandibular second
C. maxillary first
D. maxillary second
The correct answer is A. You could answer this on general principal, that the mandibular first
premolar is the most canine-like premolar tooth in almost every way. However, in detail, the
mandibular first premolar has a round root cross section and a roundish pulp chamber at the cervix.
The second premolar has a generally more flattened shape in the mesiodistal direction and a
correspondingly oval pulp chamber. It may be constricted at the center with a corresponding
hourglass shape. The maxillary first premolar root is flatttened mesiodistally, as is the pulp chamber. It
is usually constricted at the center, indicating the two roots and two canals found in the tooth. The
second maxilllary premolar, with one root, is less likely to be constricted into an hourglass shape.
226. The mucosa found on the surface of the dorsum of the tongue is known as:
A. lining mucosa
B. masticatory mucosa
C. specialized mucosa
D. none of the above
The correct answer is C. Specialized mucosa is found on the dorsum of the tongue and contains
taste buds and various papillae, including circumvallate, foliate, fungiform, and filiform. Masticatory
mucosa consists of the gingiva and covering of the hard palate. Most of the other areas of the oral
cavity are lined by lining mucosa. The characteristics of masticatory mucosa are keratinization, thin or
absent submucosa, and tight binding of the lamina propria to underlying periosteum. Lining mucosa is
generally nonkeratinized, with a thin lamina propria. Near the teeth, the gingiva and lining mucosa
(alveolar mucosa) meet at the mucogingival junction.
227. Extreme bruxism over time may cause masseteric hypertrophy. It can be noted on the
face as:
A. Swelling in the area of the temporal fossa
B. Swelling in the lateral mandibular ramus area
C. Swelling on the medial surface of the jaw
D. Loss of muscle substance near the ramus
E. Loss of muscle mass above and below the lips
The correct answer is B. Masseteric hypertrophy means gain in muscle substance of the masseter
muscle. The masseter is located on the lateral part of the ramus and angle, and its growth in
thickness can be seen as a squareness of the lower face on frontal view. People who brux (express
tension by clenching their teeth) often develop this larger-than-normal size masseter. It can
sometimes be confused with parotid swelling, although the parotids are slightly higher in location.
Remember that the medial pterygoid is medial, not lateral, to the ramus. Loss of muscle substance
would be atrophy rather than hypertrophy.
228. The mesiobuccal cusp of the mandibular first molar occludes with which maxillary tooth
surfaces?
A. The mesial marginal ridge of the first molar and distal marginal ridge of the second premolar
B. The distal marginal ridge of the first molar and mesial marginal ridge of the second molar
C. The embrasure between the first and second molars
D. The distal triangular fossa of the first molar

54

The correct answer is A. The mesiobuccal cusp of a mandibular molar is a holding (supporting)
cusp. The general rule of occlusion of mandibular holding cusps is as follows: the holding cusps of the
mandibular teeth occlude on the mesial marginal ridge of their maxillary counterpart, and the distal
marginal ridge of the maxillary tooth mesial to their counterpart, EXCEPT DISTOBUCCAL CUSPS OF
MANDIBULAR MOLARS OCCLUDE WITH CENTRAL FOSSAE OF THEIR COUNTERPARTS, THE
DISTAL CUSP OF THE MANDIBULAR FIRST MOLAR OCCLUDES WITH THE DISTAL
TRIANGULAR FOSSA OF ITS COUNTERPART, AND THE FIRST PREMOLAR OCCLUDES ONLY
WITH THE MESIAL MARGINAL RIDGE OF ITS COUNTERPART (BUT NOT THE CANINE). In this
case, the maxillary counterpart is the maxillary first molar, and the tooth mesial to it is the second
premolar.
229. Which is the most common supernumerary tooth?
A. Mesiodens
B. Second lateral incisor
C. Third mandibular
D. Fourth maxillary molar
The correct answer is A. A mesiodens is a supernumerary located midline in the maxilla. It has a
wide variety in morphology and is usually extracted. It is the most common and frequently causes a
diastema between the central incisors. The other relatively common supernumeraries are the second
maxillary lateral incisor, third mandibular premolar, and fourth maxillary molar.
230. Which cusp of the maxillary molar is NOT part of the trigon (primitive cusp triangle)?
A. Mesiolingual
B. Mesiobuccal
C. Distolingual
D. Distobuccal
The correct answer is C. In evolution, three cusps of the maxillary molar are considered to be the
primary cusp triangle, and the original three cusps of the evolving molar. They form the trigon, and are
the MB, ML, and DB cusps. The fourth cusp, the DL, is the smallest, is considered to be a minor cusp,
and is known evolutionarily as the talon, or talon cusp. This cusp is less conspicuous as you go from
first to second to third maxillary molar.
231. Which factor does NOT affect pulp cavity size in a tooth?
A. Caries
B. Age of tooth
C. Action of dentin-forming ameloblasts
D. Abrasion
The correct answer is C. The answer depends only on the fact that ameloblasts form enamel, not
dentin. Clearly, odontoblasts, as dentin formers, influence pulp size because the pulp is surrounded
by dentin. Caries may cause deposition of reparative or secondary dentin. The same is true for other
factors, such as abrasion, heavy occlusal wear, erosion, etc. Pulp tends to become smaller and more
calcified with age, so age is also a factor. Be careful when reading questions on your test!
Occasionally you will find a careful-reading question like this one.
232. A rule regarding congenitally missing teeth is:
A. A mandibular tooth is more likely to be missing than its maxillary counterpart
B. A distal tooth of a type is more likely to be missing than the mesial tooth of the same type
C. A mesial tooth of a type is more likely to be missing than a distal tooth of the same type
D. None of the above is true
The correct answer is B. In any set of teeth (incisor, premolar, molar), the distal tooth is more likely
to be congenitally missing than the mesial. So lateral incisors are missing more than centrals, second
premolars are missing more than first premolars, and third molars are more commonly missing than
first or second. Of all of these, the MOST commonly missing is the maxillary lateral incisor. Note that
choice A is therefore incorrect.
_______________

55

Вам также может понравиться